You are on page 1of 474

P h ầ n I ___________________________________________

ĐỀ THI OLYMPIC TRUYỀN THỐNG 30/4 NĂM 2018


LỚP 10 – ĐỀ CHÍNH THỨC

Choose the best options (A, B, c, or D) that best complete the. following
sentences. (40 PTS)
1. The local authority expressed regret as US drone strike ha s_____ killed innocent hostages.
A. incongruously B. vehemently c. inadvertently D. graciously
2. These days the castle is swamped w ith _____of tourists.
A. mobs B. throngs c. shoals D. cliques
3. He left the meeting early on the unlikely ___that he had a sick friend to visit.
A. excuse B. pretext c. motive D. claim
4. He decided to withdraw from the powerboat race as he could see a (n )_____of danger.
A. prediction B. foreboding c. omen D. dearth
5. 1 slipped briefly back into sleep and emerged when breakfast was being served outside in
a( n) _____ garden-courtyard.
A. whimsical B. extravagant c. extortionate D. enchanting
6. He had a momen tary_____ of concentration and before he knew it the car had spun out of
control.
A. lapse B. loss c. slip D. mistake
7. In the hands o f a careless driver, a car becomes a _____weapon.
. A. fatal B. mortal c. lethal D. venal
8. He glanced at Juliet accusingly and she lo oked_____ abashed.
A. completely B. absolutely c. utterly D. suitably
9. We were sorted out into groups according to the types o f honors and quite a long
wait_____ .
A. ensued B. eventuated c. supervened D. transpired
10. The Prime Minister will decide whether to release the prisoner or not; that's his _____ .
A. derogatory B. abdication c. prerogative D. humanity
11. His new play is not only interesting but also unique. It is really off the beaten
A. road B. path c. route D. track
12. That Peter was born and brought up in a rich family is as clear as the _____ .
A. nose on his face B. tip o f his tongue
c. back o f his hand D. hair on his head
13. I will ttyto finish the job to the best o f my_____ .
A. knowledge B. ability c. means D. command
14. Tim said the meal w as _____ , so we didn’t have to worry about the price.
A. on the house B. on his expense c. for him D. for his money
15. Let H ercules himself do what he may, for a cat will mew and a dog will have his _____ .
A. day B. time c. month D. year
16. Breaking his le g _____ a blow to his chances of becoming a professional footballer.
A. brought B. caused c. dealt D. struck
17. In the acting career, the moment one first cut his _ will be the most memorable with
embarrassment and pride bubbling up inside.
A. nails B. teeth c. fingers ' D. hair
18. His French is ro ughly _____ with my Japanese, so communication was rather difficult.
A. in harmony B. on a par c. on equal term D. on good terms
19. Max has been_____ my ears all night about his new job.
A. bending B. deafening c. rolling D. biting
20. The fighting has stopped, so to _____ , the war is over.
A. all pins and needles B. all chop and change
c. all prim and proper D. all intents and purposes
21. We would sooner Mr. Tram us the urgent information the other night'
A. would have sent B. had sent c. sent D. had been sent
22. Would you b e_____ my letter while I am away?
A. too good as to forward B. so good as to forward
c. as good as to forward D. so good as forwarding
23. He ____ _ us on the last day of the congress, so his presence at the opening ceremony was
something o f a surprise.
A. could have joined B. had to join c. was about to join D. was to join
24. You could have done _______inviting Sam to the party.
B. a lot w orse than
D. nothing as worse as
A. better or worse than
c. much better as Ư. nuuimg vvuuv o
25. He works Until nine o’clock every evening, and that’s quite
he does o ver the weekend.
A. except for B. apart from c. without D. but for
2 6 .1 would rather go skiin g_____ picnicking this weekend.
A. than going B. than to going c. than to go D. than go
the work
6
Tì. It is mandatory that smoking in public_____.
A. is prohibited B. must be prohibited
c. prohibiting D. be prohibited
28. In geometry, an ellipse may be defined as the locus of all points_____
distances from two fixed points is constant.
A. the sum o f whose B. of which the sum
c. whose sum of D. which the sum of
29. Tony gripped his brother’s arm lest h e_____by the mob.
A. would be trampled B. were trampled
c. be trampled D. could have been trampled
30. The mini dress was _____, but now it is making a comeback.
A. a tad once thought to be finished B. once thought a fad to be finishing
c. thought a fad to be finished once D. once thought to be a finishing fad
31. If you never put oil into your car engine, one day it will_____.
A. flake out B. shut down c. seize up D. run off
32. If you pay the restaurant bill with your credit card, it wi ll___ with you later.
A. settle down B. settle up c. pay back D. pay off
33. In those days, doctors ladled_____antibiotics to patients.
A. with B. on c. in D. out
34. Let’s find a place where we c an _____the storm.
A. wait out B. wear off c. wind down D. shrug off
35. The schoolboy w inc ed_____ the sight of the cane in the headmaster’s hand.
A. at B. for c. by D. o f
36.1 was completely bowled_____by their warm reception.
A. with B..up c. off D. over
37. I am not liberty to tell you anything about his private life.
A. in B. at c. by D. on
38. The figure is mo re_____ 20 0, 1 think.
A. o f B. at c. like D. with
39. Their performances are real ly_____ compare. How amazing!
A. out o f B. over c. within D. beyond
40. The party was___full swing when I an’ived. Everyone was singing and dancing.
A. in B. on c. about D. with
READING COMPREHENSION 1:
Read the following passage carefully and answer the questions below.
STEP BACK IN TIME
Historical biographer Antonia Fraser reveals the pleasures of studying a
bygone era.
Gibbon was inspired to write The Decline and Fall of the Roman Empire
sitting on the steps of the Capitol at Rome one evening, listening to the sound of
monks chanting vespers. My own inspiration to become a historical biographer
came in rather less elevated circumstances, as a teenager one rainy Oxford
7
afternoon: I began to read Lytton Strachey's Eminent Victorians and was in
particular fascinated by his essay on the worldly Cardinal Manning. This was
going to be the life for me! Once back at school 1 plunged into further research in
the convent library. A very different picture emerged. Gradually as 1 pursued the
topic, 1 became aware of Strachey's daring sallies into "artistic truth" (as opposed
to historical truth). Nevertheless, I never forgot my original sense of being
transported into a world more vivid than my own.
An ability to convey this sensation is, 1 believe, at the heart of the matter. If
you, the biographer, don't thrill to your subject, you can hardly in all fairness
expect the reader to do so. In a sense (not of course the commercial sense) the
choice of subject is irrelevant so long as it meets that requ irem ent. You could
say that I was extremely lucky to choose Mary Queen of Scots for my first foray
since there proved to be a world-wide public for the troubles of the ill-fated
Queen. But you could argue equally that I made my own luck, since I had always
been obsessed by Mary's story from childhood. Nor was success fore-ordained. It
was, after all, the leading publisher Mark Bonham -Carter of (then) Collins who
said to me when I confessed my project, "They say that all books on Mary
Queen of Scots sell and no books on South America do", before adding with a
laugh, "Perhaps yours will be the exception."
Nevertheless I did have luck. In the 60s, so-called narrative biography was
said to be pas sé. Mary Queen of Scots was an early beneficiary from the fact that
the public continued to have an appetite for it, so long as the research was felt to
be solid.
The actual research for a biography - now that's a whole other matter. The
paramount need for it - historical truth not Stracheyesque truth must be
established - means that biographers discover for themselves the reality of Dr.
Johnson's wise dictum: "A man will turn over half a library to make a book."
And what about those fabled things boasted of on blurbs: hitherto
unpublished documents? Obviously it is every researcher's dream to discover
such papers, and their discovery once again may make a project commercial
which would not otherwise be so. At the same time, I would issue a caveat about
hitherto unpublished documents. HUDs are not in themselves more valuable than
the printed sources - it's a historical coincidence that one set has become known
early on, the other not. One needs to evaluate them even more closely. Here I
speak from personal experience. A series of chances led me to the discovery of
some hitherto unpublished letters of Oliver Cromwell ju st as 1 was finishing my
manuscript. I blazoned my finds across the text: only to realize at the proof stage,
that they tnight be unpublished but they were not very important in the grand
scheme o f things.., an expensive mistake.
Where the perils and pleasures of writing historical biography are concerned,
there are two perils which seem to me to raise points of principle. The first is the
8
peril of anachronistic judgements. For example, in the 16th century more or less
everybody took astrology seriously and more or less everybody enjoyed a jolly
afternoon out to see the bears baited. It's no good dismissing the former as
meaningless and cringing from the latter as disgusting.
I would further cite the peril of hindsight. We may know that Henry VIII will
ma ny six times, but he didn't, and he would have been amazed if it had been
predicted at the time of his first marriage to Catherine of Aragon.
And the pleasures? Manifold! Principal among them however is the
opportunity to lead a life less ordinary. As a biographer, I can rule over
kingdoms, lead the cavalry into battle, patronise the great artists of the past and
all w ithout leaving my chair.
41. What did the writer learn while researching a historical figure as a teenager?
A. There was a surprising amount o f information available.
B. It was not possible to take eveiything she read as fact.
c. It was difficult to interpret the true meaning of what she read.
D. It was necessary to consult a wide range o f sources.
42. What does that requirement refer to?
A. the read er's response to a writer's subject
B. the correct choice o f subject
c. the commercial appeal of the book
D. the wri ter’s ability to communicate their enthusiasm
43. What did Mark Bonham-Carter believe about the writer’s choice o f subject?
A. H er long-standing interest in it may ensure her book’s success.
B. It did not guarantee her book’s success.
c. There are already too many books written on it.
D. It was a wise choice for her first biography.
44. The main point that the writer is making in the fourth paragraph is that
A. a b iography is more likely to be successful if it contains new information.
B. researchers must be careful to check all facts thoroughly,
c. research material can include inaccurate information.
D. extensive reading is crucially important.
45. What warning does the writer give to biographers about unpublished documents?
A. They are difficult to obtain as their discovery is down to chance.
B. Their overall significance to the book must be carefully considered,
c. Their use could result in diminished commercial success for a book.
D. It should not be assumed that they are authentic.
46. An e xample of an anachronistic judgement that the writer gives is
A. not being able to imagine oneself living in the sixteenth century.
B. being uninformed about sixteenth century customs and practices.
c. view ing the sixteenth century from a twenty-first centu iy perspective.
D. focusing only on the negative side o f life in the sixteenth century.
9
47. In the article as a whole, the writer implies that her main motivation for
becoming a historical biographer was the chance to
A. carry out extensive research. B. become immersed in history,
c. discover unpublished documents. D. establish historical truth.
48. The word elevated is closest in meaning to
A. lofty B. normal c. raised D. high
49. The word passe is closest in meaning to
A. obsolete B. out o f fashion c. antique D. archaic
50. The word their in the fifth paragraph refers to
A. blurbs B. researchers
c. unpublished documents
D. historical truths
READING COMPREHENSION 2
Read the following passage carefully and answer the questions below.
The Amazonian wilderness harbors the greatest number of species on this
planet and is an irreplaceable resource for present and future generations.
Amazonia is crucial for maintaining global climate and gene tic resources, and
its forest and rivers provide vita! sources of food, building materials,
pharmaceuticals, and water needed by wildlife and humanity.
The Los Amigos watershed in the state o f Madre de Dios, southeastern Peru,
is representative o f the pristine lowland moist forest once found throughout most
of upper Amazonian South America. Threats to tropical forests occur in the form
of fishing, hunting, gold mining, timber extraction, impending road construction,
and slash and burn agriculture. The Los Amigos watershed, consisting of
1.6 million hectares (3.95 million acres), still offers the increasingly scarce
opportunity to study rainforest as it was before the disruptive encroach ment o f
modern human civilization. Because of its relatively pristine condition and the
immediate need to ju stify it as a conservation zone and as a corridor between
Manu National Park and the Tambopata-Candamo Reserved Zone, this area
deserves intensive, long-term projects aimed at botanical training, ecotourism,
biological inventory, and information synthesis.
On July 24, 2001, the government of Peru and the Amazon Conservation
Association, represented by Enrique Ortiz, signed a contractual agreement creating
the first long-term permanently renewable conservation concession. To our
knowledge this is the first such agreement to be implemented in the world. The
conservation concession protects 340,000 acres of old growth Amazonian forest in
the Los Amigos watershed which is located in southeastern Peru. This watershed
protects the eastern flank o f Manu National Park and is part of the lowland forest
corridor that links it to Bahuaja-Sonene National Park. The Los Amigos
conservation concession will serve as a mechanism for the development of a
regional center o f excellence in natural forest management and biodiversity science.
Several major projects are being implemented at the Los Amigos Conservation
Area. Louise Emmons is initiating studies of mammal diversity and ecology in
10
the Los Amigos area. Other projects involve studies of the diversity of
arthropods, amphibians, reptiles, and birds. Robin Foster has conducted
botanical studies át Los Amigos, resulting in the labeling of hundreds of plant
species along .two kilometers of trail in upland and lowland forest. Los Amigos
has also been a major field site for Robin's rapid identification laminated
photographic field guides to tropical plants. Michael Goulding is leading a
fisheries and aquatic ecology program, which aims to document the diversity of
fish, their ecologies, and their habitats in the Los Amigos area and the Madre de
Dios watershed in general.
With support from the Amazon Conservation Association, and in
collaboration with US and Peruvian colleagues, the Botany of the Los Amigos
project has been initiated. At Los Amigos, we are attempting to develop a system
of preservation, sustainability, and scientific research; a marriage between
various disciplines, from human ecology to economic botany, product marketing
to forest management. The complexity of the ecosystem will best be understood
through a multidisciplinary approach, and improved understanding of the
complexity will lead to better management. In essence, we must be informed to
make wise management decisions about Amazonian forests. These forests hold
the greatest number of species on our planet and are an irreplaceable resource for
present and future generations. The future of these forests will depend on
sustainable management and development of alternative practices and products
that do not require irreversible destruction.
The botanical project will provide a foundation of information that is essential
to other programs at Los Amigos. By combining botanical studies with fisheries
and mammology, we will better understand plant/animal interactions. By
providing names, the botanical program will facilitate accurate communication
about plants and the animals that use them. Included in this scenario are humans,
as we will dedicate time to people-plant interactions in order to learn what plants
are used by people in the Los Amigos area, and what plants could potentially be
used by people.
To be informed, we must develop knowledge. To develop knowledge, we
must collect, organize, and disseminate information. In this sense, botanical
inform ation has conservation value. Before we can use plant-based products
from' the forest, we must know what species are useful. We must know what
their names are in order to be able to communicate accurately about them. We
must be able to identify them, to know where they occur in the forest, how many
of them exist, how they are pollinated and when they produce fruit (or other useful
products). Aside from understanding the species as they occur locally at Los
Amigos, we must have information about their overall distribution in tropical
America in order to better understand and manage the distribution, variation, and
viability of their genetic diversity and germplasm. This involves a more complete
understanding of the species through studies in the field and herbarium.
11
5 1. The phrase generic resources refers t o ________ .
A. plant seeds
B. different races of people
c. diverse species of plants and animals
D. cells that can be used in genetic cures for diseases
52. In paragraph 2, the author emphasizes that the current environmental
condition of Amazonian South America i s ______ .
A. mostly unscathed
B. restorable through his project
c. irredeemable everyw here but in the Los Amigos watershed
D. varying from destroyed to virtually pristine
53. The word encroachment in paragraph 2 is closest in meaning to ________ .
A. intrusion B. augmentation c. infringement D. seepage
54. The author implies in paragraph three that the agreement between Peru and
the Amazon Conservation Association is history primarily because it______ .
A. was the first long-term agreement regarding land in the Amazon Rainforest
B. represented the first time a South American government had agreed to
renew a conservation agreement
c. is essentially a perm anent conservation agreement
D. represents the first time such an agreement has been in the form of a
renewable contract
55. The author's main purpose in the passage is to ________ .
A. demonstrate that conservation efforts have been historically successful and
so should be continued
B. garner support for opposition to destructive activities in the Los Amigos
watershed
c. position the Los Amigos watershed .agreement as a success towards the
achievement o f the vital goal o f conservation the Amazonian rainforests
D. argue that the study pristine rainforests is essential for documenting and
studying the myriad new species that the forests contain
56. The author's tone in the passage can be best described a s________ .
A. advocacy for his project over the other competing projects
B. general praise for conservation projects in Amazonian South America,
c. passionate support for his and related projects
D. zealous advocacy for his point o f view
57. The work of Louise Emmons, Robin Foster, and Michael Goulding (in
paragraph 4) are employed in the passage a s________ .
A. colleagues o f the au thor's in his botanical project
B. examples of the kinds of activities the author and his colleagues are trying
to halt
c. scientists who are representative of new trends of study in Amazonian botany
D. scientists involved in projects related and amenable to the author's
12
58. The author's botanical project involved all of the following E X C E PT _____ .
A. studying plants in laboratory
B. studying how plants are used by humans and animals
c. facilitating pharmaceutical use of plants
D. labeling plants in the Los Amigos area
59. When the author says that the botanical project will provide names he means
that the project will ~ __.
A. help recognize new species
B. aid in the standardization of names for new species
c. participate in naming the regio n’s different zones
D. clarify the conclusion surrounding the names of different organizations
working in Amazonia
60. When the author says that botanical information has conservation value he
means that ________ .
A. a robust understanding of conservation ism is aided by botanical information
B. conservationists should strive to preserve botanical information
c. specification is of importance for conservation
D. political discussions about conservation should use botanical nomenclature
GUIDED CLOZE 1: Choose the most appropriate words to fill in the blanks.
The national park movement began in the United States in 1870 when a team
of explorers suggested that part of the Yellowstone River region be (6 1)_____
in order to protect its geothermal (6 2)_____ , wildlife, forests, and (6 3)_____
scenery for the benefit of future generations. Congress (6 4 )_____ by creating
Yellowstone National Park, the world's first, in 1872.
The idea proved (6 5)_____ , and the number o f national parks in this country
grew rapidly, new parks being set up by presidential (6 6)_____ and sometimes
as a result o f gifts by states of the union or by individuals. Administration of this
increasingly comp lex system was in the hands of the u.s. Army for thirty years
from 1886, but thẹn Congress created the National Park Service as part of the
Department of the Interior to ( 67 )_____ it.
Today, in addition to what might be thought of as typical national parks, the
Service also manages places of historic interest, hiking trails, seashores, rivers,
(6 8)_______ of scientific interest and memorials. In all, more than 300 entities
are involved, covering over 32 million hectares. Each unit is directed by a
supe rinte ndent who is responsible for all aspects of the operation. Staff ( 69)___
administrative personnel and, according to the nature of the unit, park rangers,
naturalists, historians, and (7 0) _____ workers.
61. A. set up B. set aside C. set by D. set in
62. A. tokens B. characters C. features D. traits
63. A. breathless B. optimal c. exceptional D. prom inen
64. A. countered B. retrieved C. designated D. responded
65. A. prevailing B. banal C. widespread D. popular
13
66. A. analogy B. proclamation c. constitution D. dissemina
67. A. oversee B. overlook c. overcharge D. overact
68. A. observatories B. reserves c. reservoirs D. estuaries
69. A. cover B. control c. undergo D. undertake
70. A. maintenance B. sustainable c. protectorate D. conservat
GU IDED CL OZ E 2: Choose the most appropriate words to fill in the blanks.
Ocean water plays a(n) (7 1)_____ role in supporting life. The great ocean
basins hold about 300 million cubic miles of water. From this vast amount, about
80,000 cubic miles of water are sucked into the atmosphere each year by
evaporation and returned by precipitation and drainage to the ocean. More than
24,000 cubic miles of rain descend annually upon the continents. This vast
amount is required to ( 72 )_____ the lakes and streams, springs and water tables
on which all flora and fauna are dependent. Thus, the hydrosphere permits
organic existence.
The hydrosphere has strange characteristics because water has (7 3 )_____
unlike those of any other liquid. One (7 4) ________ is that w ater upon freezing
(75) __________ by about 9 percent, whereas most liquids contract on cooling.
(76) ________ this reason, ice floats on water bodies instead of sinking to the
bottom. If the ice sank, the hydrosphere would soon be frozen solidly, except for
a thin layer of surface melt water during the summer season. Thus, all aquatic
life would be destroyed and the interchange of warm and cold currents, which
moderates climate, would be (77 )________ absent.
Another outstanding characteristic of water is that water has a heat capacity
which is the highest of all liquids and solids except ammonia. This characteristic
enables the oceans to absorb and store vast quantities of heat, (7 8)_____ often
preventing climatic extremes. In addition, water dissolves more substances than
any other liquid. It is this characteristic which helps make oceans a great
storehouse for minerals which have been washed (7 9) _____ from the continents.
In several areas of the world these minerals are being commercially exploited.
Solar evaporation of salt is widely (8 0) _____ , potash is extracted from the Dead
Sea, and magnesium is produced from sea water along the American Gulf Coast.
7Ỉ. A. principle B. principal c. dispensable D. expendab
72. A. replenish B. reinforce c. replete D. restore
73. A. property B. characteristics c. assets D. nature
74. A. alienation B. distinction c. contrast D. inconsiste
75. A. extends B. inflates c. increases D.e xpa nds
76. A. For B. By c. In D. With
77. A. remarkably B. vaguely c. unnoticeably D. indefinite
78.A so B. hence c. and then D. consequen
79 A. o ff B. out c. up D. down
80. A. exercised B. exerted c. practised D. proceeded
14
B. WRITTEN TEST (70 PTS)
I. CLOZE TEST (20 PTS)
OP EN CL OZ E 1: Fill in each numbered blank with ONE suitable word.
A new threat to our health seems to have arisen in our midst, confusion and
stress brought (1 )________ by technology. All you need to do to prove this to
(2 )_______ is to telephone a large company; a recorded voice will (3 )________
you with a bewildering list of choices, and when you have finished answering its
questions, you will probably be subjected to several minutes of piped music
before you eventually make contact with a human being. But the stress you
undergo as a result is negligible compared to the (4 )_______ the telegraph made
on people 150 years ago. Until (5 )_______ , messages could only travel as fast
as a messenger could carry them. But now they could be sent great (6 )______ in
seconds. Before long, ( 7 ) ________ cables were laid across the oceans, and
thirty years later, the network reached 20,000 towns around the world.
Information arrived so quickly, often contradicting what had previously
been transmitted, (8 )____________ businessmen had to work much harder to
(9 )________ abreast of developments. If we find difficulty with the Internet,
which is technological evolution, (1 0)________ revolution, our ancestors had
afar harder task in getting used to the invention in the first place.
OPEN CL OZ E 2: Fill ill each numbered blank with ONE suitable word.
Negative impacts from tourism occur when the level o f visitor use is greater
than the environment’s ( 11 )_______ to cope with this use within the acceptable
limits o f change. Uncontrolled conventional tourism poses potential (1 2) ______
to many natural areas around the world. It can put enormous pressure on an area
and lead to impacts such as soil erosion, increased pollution, (1 3)_______ into
the sea, natural habitat loss, increased pressure on endangered species and
heightened (1 4)______ to forest fires. It often puts a strain on water resources
and it can force local populations to compete for the use o f critical resources.
Water, and especially fresh water, is one of the most critical natural resources.
The tourism industry generally (1 5)____________ water resources for hotels,
swimming pools, golf courses and personal use of water by tourists. This can
result (1 6 )________ water shortages and (1 7)_________of water supplies, as
well as generating a greater volume of waste water. Tourism can create great
pressure on local resources like energy, food and other raw materials that may
already be in short supply. Greater extraction and transport of these resources
exacerbates the physical impacts associated with their exploitation. Because of
the 0 8 ) character of the industry, many destinations have ten times
(19) inhabitants in the high season than in the low season. A high
demand is placed upon these resources to (2 0)________ the high expectations
tourists often have (proper heating, hot water, etc.)
15
II. WORD FORMATION (20 PTS)
WORD FORMATION 1: Supply the correct forms o f the words given.
1. To th e __________ , most computer systems seem complex and difficult to
understand. (INITIATIVE)
2. The pieces of evidence fell into place with th e __________ precision of a
well-made jigsaw puzzle. (VOICE)
3. Parents have de ep ___________ about allowing business values to be used in
schools. (GIVE)
4. For many people, social networking offers them a feeling o f ___________
from the real world. (ESCAPE)
5. Not sick, Mai guessed, but pro bab ly___________ now that she drank a lot at
the party last night. (HANG)
6. The boats surrounded the whales, drove them into nets, where they became
____________ and were rendered helpless by harpoon thrusts. (MESH)
7. “P” is a ___________ consonant. (LIP)
8. P er haps___________ , recent computer modeling studies predict fewer
tropical cyclones if the ocean heats up further as a result of global warming.
(INTUITION)
9. New immigrants have been suc ces sfully_____ ______. into the community.
(SIMILAR)
10. We have to learn good examples, to look at our behavior and to stop being
. (RIGHT)
WORD FORMATION .2: Fill in the blank with an appropriate form o f one of
the words given to make a meaningful passage.
bridge live allegation sequential
healthy corporate accompany license
William Sydney Porter (1862-1910), who wrote under the (1 1) ________
of o. Henry, was born in North Carolina. His only formal education was to
attend his Aunt Lina 's school until the age of fifteen, where he developed his
(1 2) __________ love of books. By 1881 he was a ( 13 )___________pharmacist.
However, within a year, on the recommendation of a medical colleague of his
Fath er's, Porter moved to La Salle County in Texas for two years herding sheep.
During the time, Webster's (14) ____________ Dictionary was his constant
(15) ____________ , and Porter gained a knowledge of ranch life that he later
(16) ____________ into many of his short stories. He then moved to Austin for
three years, and during this tim e the first recorded use of his pen name appeared,
(17) _________ __ derived from his habit of calling “Oh, Henry” to a family cat.
In 1887, Porter married Athol Estes. He worked as a draftsman, then as a bank
teller for the First National Bank.
In 1894 Porter founded his own humor weekly, the “ Rolling Stone”, a venture
that failed within a year, and later wrote a column for the Houston Daily Post. In
the meantime, the First National Bank was examined, and the (1 8) ___________
indictment of 1886 stated that Porter had embezzled funds. Porter then fled to
16
New Orleans, and later to Honduras, leaving his wife and child in Austin. He
returned in 1897 because of his wife's continued (1 9)____________ , however,
she died six months later. Then, in 1898 Porter was found guilty and sentenced
to five years' imprisonment in Ohio. At the age of 35, he entered prison as a
defeated man; he had lost his jo b, his home, his wife, and finally his invented
name he now used to hide his identity. He wrote at least twelve stories in jail,
and after (2 0 )____________ his freedom, went to New York City, where he
published more than 300 stories and gained fame as America’s favorite short
story writer. Porter married again in 1907, but after months of poor health, he
died in New York City at the age of 48 in 1910. o. Henry's stories have been
translated all over the world.
III. ERROR IDEN TIFICATION (10 PTS)
Identify 10 mistakes in this passage and suggest corrections.
15 Preserving organisms in
10 museums is one way of
15 retaining them for
posterity, but almost people
agree that it would be nice
to keep a few of
them live in the wild, too. At
the moment, which species
survive, which
decline to threatened or even
status and which succumb
for extinction is
something of a lottery.
WORLDMAP is an easy-to-
use software that
identified geographical
patterns in diversity, rarity
and conservation
priorities. It can perform a
range of specialist biological
analysis for
infinitely countless numbers
of species, with a view to
provide
biodiversity data for
research purposes. The
program divides the surface
area of the world into cells,
usually arranging in a
rectangular grid.
WOR LDMAP can also
predict the likelihood of a
hitherto unobserved
species found in an area on
the basis of theirs known
distribution. Given
the patchiness of most
records, which is a useful
trick. Furthermore, it
cán select complementary
areas for preservation.
Those are not necessarily
cells with the highest
individual biodiversity, but
for those which,
together, maxim ize what is
preserved by picking places
with the least
overlapping species.
1.________________
4 .________________
7 __ _____________
2 ._______________
5 ._______________
8 ._______________
3 ._______________
6 ._______________
9. _________
10.
IV. SEN TEN CE TRANSFORM ATION:
Rewrite the following sentences using the words given. (20 PTS)
1. Linda was very nervous, which made her look like a bashful girl. (CAME)
-> Su ch _________ ___ ___________________________________________ •
2. As soon as the funds ran out, they had to abandon the scheme. (PETERED )
-> The instan t____________ __ ______________________________________
17
3. Why did you reveal my plan to Kathy? (BREATHED)
-> I'd _ ____________ ________ _________________________________.
4. David was responsible for the family business as soon as his father retired.
(CHARGE)
-» Scarc ely _____________________________________________________ .
5. Your encouragement helped to make things less grievous after such a heavy
loss. (CUSHION)
—> It w as_______________________________________________________ .
6. Tina was crazy about stamps, so she spends lots of money on them every
month. (SPLASHED)
—» Had it________________________________________________________ .
7. He tried hard but couldn't compensate for what he had done. (AMENDS)
—> T ry __________________________________ ________________________ .
8. Experts think that all dogs evolved from wolves. (DESCENDED)
-» All do gs ______________________________- _____________ experts.
9. We d idn't learn he still managed to live with very little money as a waiter
until later. (EKED)
—> Not u nt il_____________ _______________________________ ________.
10. Nobody is certain if the project will be permitted to continue. (GO-AHEAD)
—> It's still to uc h___________________________________________ or not.
TRUÔNG THPT CHUYÊN HUỲNH MẪN ĐẠT - KIÊN GIANG
A. MULTIPLE CHOICE (40 RTS)
Circle A, B, C or D that best completes each of the following sentences.
I. WORD CHOICE (5 pts)
1. She was in the office all o f Wednesday and so has a alibi.
A. stale B. considerable c. broad D. cast-iron
2. The camel has adapted to survive in a n ________ environment like the desert
for many days w ithout water.
A. acidic B. alkaline c. arid D. avid
3. A special feature of the room is the huge picture window whi ch ________ a
splendid view of the Quantock hills.
A. allows . B. affords c. enables D. presents
4. He still suffers from a rare tropical disease which h e _______ while in Africa.
A. infected B. complained c. gained D. contracted
5. Some people prefer to watch a film first, and then read the J ____ in the paper.
A. critic B. revue c. review D. criticism
18
6. We want everyone to begin the tes t________ .
A. simultaneously B. unexpectedly c. indefinitely D. continuously
7. You are bound to find information on the stock market crash of 1987 in the
newspaper ________ .
A. files B. archives c. records D. collections
8. What stands out from The Voice Kids is that many young children are
with natural talent for music.
A. bestowed B. conferred c. endowed D. vouchsafed
9. Most critics praise that actor's work but I think he's rat her________ .
A. over-played B. over-rated c. over-blown D. over-priced
10. Marlene is qu ite ________ - 1 don't know she manages to fit everything in.
A. inexhaustible B. tiresome c. inexorable D. indefatigable
11. STRUCTURES AND GRAMMAR (5 pts)
1. Sheila will inherit e verything________ her uncle’s death.
A. on account o f B. in spite of c. in the event of D. in place of
2. The newspaper report co ntained________ important information
A. many B. another c. an D. a lot o f
3. ________ comes a time when you have to make a decision and stick to it.
A. It B. Here c. There D. That
4. There is no point in phoning him. He’s ce rtain________ by now.
A. to leave B. to have left c. left D. having left
5. In such a pl ight________ that we had no choice but to radio for help.
A. we found ourselves B. we ourselves found
c. did we find ourselves D. did we ourselves find
6. Mrs. Lan went t o ________ school to meet her son’s teacher.
A. zero article B. A c. an D. the
7. You may borrow as many books as you like, provided you show them to
•_____is at the desk.
. A. whoever B. who c. whom D. which
to hold the door
8. Would you be
open?
A. too kind B. kind
enough
c. as kind D. so kind
9. I’ll enter the
competition if you
A. would B. should c. will D. shall
10. Why are you so mad? Y ou ________ me you weren’t coming to dinner. I
waited for you for two hours.
A. should tell B. should have told
c. ought to tell D. should be told
III. P REPOSIT IONS AND PHRASAL VERBS (5 pts)
1 He'll have to b uc kle________ _ to his work soon if he wants to pass his finals.
A up B. in c. down D. for
2. His poor handling of the business ________ on negligence.
A. neared B. edged c. approached D. bordered
19
3. After its engine failed, the small bo at________ with the current.
A. waved B. tossed c. hastened D. drifted
4. There is a rumor that the National Bank is going to ________ the company I
work for.
A. take on B. overtake c. take over D. take o ff
5. Why don 't you have a night out? It would take yo ur _______ off your worries.
A. thoughts B. heart c. mind D. head
6. The government decided t o ________ down on income tax evasion.
A. press B. crack c. push D. snap
7. If w e______ over the details, we ’ll never finish filming this episode by today.
A. niggle B. discuss c. huddle D. mob
8. The Corporation is la yin g________ a lot of money on building a new creation
center.
A. down B. of f c. into D. out
9. S he's decided to ________ her German by attending an evening course.
A. brush up B. patch up c. polish off D. dust o ff
10. I d idn't really want to go to the party, but I thought I’d better p u t________
an appearance.
A. away B. ỉn c. off D. on
IV. COLLOCATIO NS AND IDIOMS (5 pts)
1. That Mary is a n ________ liar; you must take w hat she says with a small grain
of salt.
A. incorrigible B. incurable c. irredeemable D. irremediable
2. I knew my m othe r w ou ld_______ a face the minute she saw my new haircut.
A. drag B. lift c. pull D. raise
3. If you don’t stop smoking, you ____ the risk o f developing chronic bronchitis.
A. bear B. suffer c. make D. run
4. He was a pickpocket and had to spend many years be hind ________ .
A. windows B. prisons c. cells D. bars
5. If you say you'd like ________of cream on your strawberries then you don't
want very much cream.
A. a dash B. oodles c. lashings D, a dollop
6. In her speech the Prime Mini ste r_______ tribute to the valuable contributions
to society made by voluntary organizations.
A. paid B. sent c. brought D. gave
7. Tamara has set h er ________ on becoming a ballet-dancer.
A. feet B. brain c. heart D. head
8. The football team Manchester United pai d________ to sign up the Brazilian
twins Rafael and Fabio Da Silva when they were only eighteen.
A. their heart in it B. an arm and a leg
c. a good heart D. their eyes to eyes
20
9. He spoke well though it was his ________ speech.
A. first-hand B. maiden c. slurred D. prime
10. In h is ________ days, he was quite dandy.
A. salad B. green c. fruit D. vegetable
V. READING COMPREHENSIO N (10 pts)
Read the following passages carefully and circle A, B, c or D that best
answers the questions.
READING PASSAGE 1 (5 pts)
THE UNDERGROUND RAILROAD
Slavery was legal for over 200 years in some parts of North America,
particularly the southern states of the United States, where the plantation system
of agriculture depended on the labor o f slaves, most of whom came from Africa.
Slaves had no rights or freedoms because they were thought of as property. From
the time of its origin, slavery had opponents. The abolitionist movement began in
the 1600s when the Quakers in Pennsylvania objected to slavery on moral
grounds and wanted to abolish the institution.
In 1793, Canada passed a law abolishing slavery and declared that any
escaped slaves who came to Canada would be free citizens. Slavery was already
illegal in most northern states; however, slaves captured there by slave hunters
could be returned to slavery in the South. Canada refused to return runaway
slaves or to allow American slave hunters into the country. It is estimated that
more than 30,000 runaway slaves immigrated to Canada and settled in the Great
Lakes region between 1830 and 1865.
The American antislavery movement was at the height of its activity during
the 1800s, when abolitionists developed the Underground Railroad, a loosely
organized system whereby runaway slaves were passed from safe house to safe
house as they fled northwards to free states or Canada. The term was first used
in the 1830s and came from an Ohio clergyman who said, “They who took
passage on it disappeared from public view as if they had really gone to ground”.
Because the Underground Railroad was so secret, few records exist that would
reveal the true number of people who travelled it to freedom. The most active
routes on the railroad were in Ohia, Indiana, and western Pennsylvania.
Runaway slaves usually traveled alone or in small groups. Most were young
men between the ages of 16 and 35. (A) The fugitives hid in wagons under loads
of hay or potatoes, or in furniture and boxes in steamers and on rafts. (B) They
traveled on foot through swamps and woods, moving only a few miles each
night, using the North Star as a compass. Sometimes they moved in broad
daylight. (C) Boys disguised themselves as girls, and girls dressed as boys. In
one well-know n incident, twenty-eight slaves escaped by walking in a funeral
procession from Kentucky to Ohio. (D)
21
The railroad developed its own language. The trains were the large farm
wagons that could conceal and carry a number of people. The tracks were the
backcountry roads that were used to elude the slave hunters. The stations were
the homes and hiding places where the slaves were fed and cared for as they
moved north. The agents were the people who planned the escaped routes. The
“conducto rs” were the fearless men and women who led the slaves toward
freedom. The “passengers" were the slaves who dared to run away and break for
liberty. Passengers paid no fare and conductors received no pay.
The most daring conductor was Harriet Tubman, a former slave who
dedicated her life to helping other runaways. Tubman made 19 trips into the
South to guide 300 relatives, friends and strangers to freedom. She was wanted
dead or alive in the South, but she was never captured and never lost a
passenger. A determined worker, she carried a gun for protection and a supply of
drugs to quiet the crying babies in her rescue parties.
A number of white people joined the effort, including Indiana banker Levi
Coffin and his wife Catherine, who hid runaways in their home, a “station”
conveniently located on three main escape routes to Canada. People, could be
hidden there for several weeks, recovering their strength and waiting until it was
safe to continue on their journey. Levi Coffin was called the “pre sident of the
Underground Railroad” because he helped as many as 3,000 slaves to escape.
The people who worked on the railroad were breaking the law. Although the
escape network was never as successful or as well organized as Southerners
thought, the few thousand slaves who made their way to freedom in this way
each year had a symbolic significance out of proportion to their actual numbers.
The Underground Railroad continued operating until slavery in the United States
was finally abolished in 1865.
1. Why did thousands of runaways slaves immigrate to Canada?
A. They preferred the climate of the Great Lakes region.
B. Working conditions for slaves were better in Canada,
c. Canada had no laws restricting immigration.
D. Former slaves could live as free citizens in Canada.
2. The phrase “The term” in paragraph 3 refers t o ________ .
A. Antislavery movement c. Underground Railroad
B. Abolitionist D. free state
3. The word “fugitives” in paragraph 4 is closest in meaning t o ________ .
A. Leaders B. old men c. runaways D. brave ones
4. All of the following are mentioned as methods of escape on the Underground
Railroad EXCEPT________ ,
A. Hiding in a hay wagon
B. Wearing a disguise
c. riding in a railcar
D. walking in a procession
22
5. The autho r discusses the language of the Underground Railroad in paragraph 5
in orde r t o ________ .
A. Trace .the history o f American English words
B. Illustrate the secret nature of the escape network
c. Point out that some words have more than one meaning
D. Com pare the Underground Railroad to other railways
6. The word “elude” in paragraph 5 is closest in meaning to ________ .
A. avoid B. follow c. find D. assist
7. Which of the following statements is true about passengers on the
Underground Railroad?
A. Th eir destination was in the northern states or Canada.
B. They were not allowed to make stops during the journey,
c. Their babies were disguised to look like baggage.
D. They paid the conductors at the end of the journey.
8. Why was Harriet Tubman wanted dead or alive in the South?
A. She was a criminal who carried a gun and sold drugs.
B. She refused to return the runaway slaves that she captured,
c. She was an escaped slave who led others to freedom.
D. She became the president o f the Underground Railroad.
9. It can be inferred from paragraph 8 that the author most likely believes which
of the following about the Underground Railroad?
A. The people who worked on the railroad should have been arrested.
B. The railroad was unsuccessful because it could not help every slave,
c. Southerners did not know about the railroad until after it closed.
D. The railroad represented a psychological victory for abolitionists.
10. Where would the following sentence best fitted into paragraph 4?
Women and children also escaped, but they were more easily captured.
A. (A) B. (B) C.(C ) D. (D)
READING PASSAGE 2 (5 pts)
The Amazonian wilderness harbors the greatest number of species on this
planet and is an irreplaceable resource for present and future generations.
Amazonia is crucial for maintaining global climate and genetic resources, and its
forest and rivers provide vital sources of food, building materials, pharmaceuticals,
and water needed by wildlife and humanity.
The Los Amigos watershed in the state of Madre de Dios, southeastern Peru,
is re presentative of the pristine lowland moist forest once found throughout most
of upper Amazonia n South America. Threats to tropical forests occur in the form
of fishing, hunting, gold mining, timber extraction, impending road construction,
and slash and bum agriculture. The Los Amigos watershed, consisting o f 1.6 million
hectares (3.95 million acres), still offers the increasingly scarce opportunity to
study rain forest as it was before the disruptive encroachment of modern human
23
civilization. Because of its relatively pristine condition and the immediate need
to justify it as a conservation zone and as a corridor between Manu National Park
and the Tambopata-Candam o Reserved Zone, this area deserves intensive, longterm projects
aimed at botanical training, ecotourism, biological inventory, and
information synthesis.
On July 24, 2001, the government of Peru and the Amazon Conservation
Association, represented by Enrique Ortiz, signed a contractual agreem ent
creating the first long-term permanently renewable conservation concession. To
our knowledge this is the first such agreement to be implemented in the world. The
conservation concession protects 340,000 acres of old growth Amazonian forest in
the Los Amigos watershed which is located in southeastern Peru. This watershed
protects the eastern flank of Manu National Park and is part of the lowland forest
corridor that links it to Bahuaja-Sonene National Park. The Los Amigos
conservation concession will serve as a mechanism for the development of a
regional center o f excellence in natural forest management and biodiversity science.
Several major projects are being implemented at the Los Amigos Conservation
Area. Louise Emmons is initiating studies of mammal diversity and ecology in
the Los Amigos area. Other projects involve studies of the diversity of arthropods,
amphibians, reptiles, and birds. Robin Foster has conducted botanical studies at
Los Amigos, resulting in the labeling of hundreds of plant species along two
kilometers of trail in upland and lowland forest. Los Amigos has also been a
major field site for Robin's rapid identification laminated photographic field
guides to tropical plants. Michael Goulding is leading a fisheries and aquatic
ecology program , which aims to document the diversity of fish, their ecologies,
and their habitats in the Los Amigos area and the Madre de Dios watershed
in general.
With support from the Amazon Conservation Association, and in
collaboration with US and Peruvian colleagues, the Botany of the Los Amigos
project has been initiated. At Los Amigos, we are attempting to develop a system
of preservation, sustainability, and scientific research; a marriage between
variousdisciplines, from human ecology to economic botany, product marketing
to forest management. Thecomplexity of the ecosystem will best be understood
through a multidisciplinary approach, and improved understanding of the
complexity will lead to better management. In essence, we must be informed to
make wise managem ent decisions about Amazonian forests. These forests hold
the greatest number o f species on our planet and are an irreplaceable resource for
present and future generations. The future of these forests will depend on
sustainable manageme nt and development of alternative practices and products
that do not require irreversible destruction.
The botanical project will provide a foundation of information that is essential
to other programs at Los Amigos. By combining botanical studies with fisheries
and mamm ology, we will better understand plant/animal interactions. By
24
prov iding na mes, the botanical program will facilitate accurate communication
about plants and the animals that use them. Included in this scenario are humans,
as we will dedicate time to people-plant interactions in order to learn what plants
are used by p eople in the Los Amigos area, and what plants could potentially be
used by people.
To be informed; we must develop knowledge. To develop knowledge, we
must collect, organize, and disseminate information. In this sense, botan ica l
inform ation has conservatio n va lu e. Before we can use plant-based products
from the forest, we must know what species are useful. We must know what their
names are in order to be able to communicate accurately about them. We must be
able to identify them, to know where they occur in'the forest, how many of them
exist, how they are pollinated and when they produce fruit (or other useful products).
Aside fi'om understanding the species as they occur locally at Los Amigos, we must
have information about their overall distribution in tropical America in order to
better understand and manage the distribution, variation, and viability of their
genetic diversity and germplasm. This involves a more complete understanding of
the species through studies in the field and herbarium.
1. The phrase “gene tic r esou rces” refers to
A. plant seeds
B. different races of people
c. diverse species o f plants and animals
D. cells that can be used in genetic cures for diseases
2. In paragraph 2, the author emphasizes that the current environmental
condition o f Amazonian South America is________ .
A. mostly unscathed
B. restorable through his project
c. irredeemable everywhere but in the Los Amigos watershed
D. varying from destroyed to virtually pristine
3. The word “co nc essio n” in paragraph 3 is closest in meaning to ________ .
A. grant B. acknowledgement c. apology D. compromise
4. The author implies in paragraph three that the agreem ent between Peru and the
Amazone C onservation Association is history primarily because it________ .
A. was the first long-term agreement regarding land in the Amazone Rainforest
B. represented the first time a South American government had agreed to
renew a conservation agreem ent
c. is essentially a permanent conservation agreement
□. represents the first time such an agreement has been in the form of a
renewable contract
5. The a uth or’s main purpose in the passage is to________ .
A. dem onstrate that conservation efforts have been historically successful and
so should be continued
B. garn er supp ort for opposition to destructive activities in the Los Amigos
watershed
25
c. position the Los Amigos watershed agreement as a success towards the
achievement o f the vital goal o f conservation the Amazonian rainforests
D. argue that the study pristine rainforests is essential for documenting and
studying the myriad new species that the forests contain
6. The auth or’s tone in the passage can be best described as________ .
A. advocacy for his project over the other competing projects
B. general praise for conservation projects in Amazonian South America,
c. passionate support for his and related projects
D. zealous advocacy for his point of view
7. The work of Louise Emmons, Robin Foster, and Michael Goulding (in the
fourth paragraph) are employed in the passage as________ .
A. colleagues o f the author’s in his botanical project
B. examples of the kinds of activities the author and his colleagues are trying
to halt
c. scientists who are represent new trends o f study in Amazonian botany
D. scientist involved in projects related and amenable to the author’s
8. The author's botanical project involved all o f the following EXCEPT________ .
A. studying plants in laboratory
B. studying how plants are used by humans and animals
c. facilitating pharmaceutical use of plants
D. labeling plants in the Los Amigos area
9. When the author says that the botanical project will “provid e na mes” he
means that the project will
A. help recognize new species
B. aid in the standardization of names for new species
c. participate in naming the region ’s different zones
D. clarify the conclusion surrounding the names of different organizations
working in Amazonia
10. When the author says that “bo tan ical inform ation has conservatio n va lu e”
he means that________ .
A. a robust understanding of conservation ism is aided by botanical information
B. conservationists should strive to preserve botanical information
c. specification is important for conservation
D. political discussions about conservation should use botanical nomenclature
VI. GU IDED CL OZ E TE ST (10 pts)
Read the follow ing passa ges and cir cle A, B, c or D that bes t fits each
numb ere d blank.
GUIDED CLOZE TEST 1 (5 pts)
GERARD MERCATOR: THE MAN WHO MAPPED THE PLANET
When Gerard Mercator was born in 1512, the geography of the globe still
remained a mystery. It was unclear whether America was part of Asia if there
26
was a vast (1 )___________ of sea at the top of the world or if Australia was
connected to Antarctica.
Mercator’s childhood was spent chiefly in Rupelmonde, a Flemish trading
town on the river, and it was here that his geographical imagination was
(2) ______ • by the ships which passed to and from the rest of the world.
Alongside imagination, he developed two very different skills. The first was the
ability to gather, (3) ________ and co-ordinate the geographical information
provided by explorers and sailors who frequented the margins of the known. He
also had to be able to imagine himself (4) _ ______ from the heavens, to
achieve the visionary (5 )________ of gods in the skies, (6 )_________down on
the world. The main reason why Mercator’s name is familiar to US is because of
the Mercator Projection: the solution he (7) ________ to represent the
spheroidal surface o f the globe on a two-dimensional plane. It is less well known
that Mercator was the first man to conceive o f mapping the (8 )________ surface
of the planet or that he (9 )___ _____the idea o f multiple maps being presented
in bound books, to which he gave the name ‘Atlas'.
It is difficult for us now to be surprised by maps, so many are there, and of
such detail and coverage, but we should bear in mind that Mercator lived at a
time when such knowledge was far from (1 0)________ . He was the man who
altered our worldview forever.
1. A. territory B. distance c. range D. expanse
2. A. raised B. reared c. supplied D. nourished
3. A. congregate B. amass c. assimilate D. construct
4. A.s usp end ed B. located c. situated D. attached
5. A. inspection B. observation c. perspective D. assessmen
6. A. glim psing B. scutinizing c. watching D. gazing
7. A. invented B. contrived c. devised - D. schemed
8. A.sheer B full c. entire D. utter
9. A. pioneered B. initiated c. lead D. prepared
10. A. typical B. common c. routine D. norma!
GUIDED CLOZE TEST 2 (5 pis)
ME AND MY HEALTH
I never seem to stop. I'm not so much a workaholic as the (1) ‘girl
who ca n't say no'. Not only have I never learned to organize my time, I need the
deadline to pass before I get going. Then I wonder why I feel (2)________ . I’m
very good at whipping up false energy. Without (3)________ lest, though, I start
to look grey and then, apart from a good night's sleep, the only thing that brings
me back to life is meditation. I find that 20 minutes’ meditation is (4)________
to a nig ht's sleep and that keeps me going. Somehow though, 1 never get around
to meditating on a daily (5)________ . I'm no good in the morning. By the time
I’ve managed to get up and repair the (6)________ of the night, half of the day's
27
gone. When it comes to food, I'm a hostess’s nightm are. As I suffer from
migraine, I avoid cheese and (7)________ like coffee, red wine and spirits,
which are generally (8)________ to trigger and attack. I only eat rye bread, as
the gluten in wh eat makes me feel bloated and ropy. I’ve (9)________ countless
migraine ‘cures’, from the herbal remedy fev erf ew to acupuncture. Acupuncture
(10)________ balan ce the system, but nothing stops the attacks.
A. innovative B. first c. foremost D. original
A. apathetic B. exhausted c. grueling D.sp ent
A. sufficient B. severe c. acceptable D. minimal
A. com patible B. alike c. equivalent D. proportion
A. way B. basis c. routine D. manner
A. damages B. wrecks c.rava ge s D. ruins
A. stimulations B. stimulants c. stimulus D. stimulatin
A. foreseen B. maintained c. regarded D. thought
A. tried B. experimented c .searched D. proved
1. A. assists B. improves c. restores D. helps
B. WRITTEN TE ST (70 pts)
I. OPEN CLOZE TEST (20 pts)
Fill in each numbered blank with ONE suitable word.
OPEN CLOZE TEST 1 (10 pts)
KARAOKE FEVER
Karaoke is fast becoming the nation’s Number One party pastime. Public
humiliation has (1 )________ been so fashionable. It’s 1 a.m. at an exclusive
location in the heart of London. A major pop singer has taken the stage but rather ,
than sing her latest hit, she treats the crowd (2 )________ a Michael Jackson
song. What was (3 )________ the party habit of teenagers is now favoured by
London's coolest crowd and everyone is having a (4 )________ . So why are so
many of our young celebrities queueing up to make fools of ( 5 )________ in
clubs and bars across the country? Maybe it’s because (6 )________ out a naf f
pop song to a public audience shows that even though you may be a celebrity,
you don’t (7) • yourself too seriously. And if you are a big movie star,
tha t's a good message to get across. Nobody gets away without being laughed
(8 )________ on a karaoke evening, no matter how famous they are. Turning all,
tha t's the whole point of the exercise. (9) ________ for the musical experts
among you, a word of warning: this isn’t about proving to the world that you
know all the lyrics to a serious song. It’s about expressing your inner performer.
Don’t bother (1 0)________ up at a karaoke night if you aren’t prepared to sing;
you’ve got to put in the effort and prove that you are one of the ‘in-crowd’.
Break a leg!
28
OPEN CLOZE TEST 2 (10 pts)
When he was made (1 )________ four years ago, John Spencer set up his own
business dealing in (2 )________ and second-hand books. “I didn ’t expect to
lose my job," he said. “It happened very suddenly and I knew it would be
difficult to find another one. I’d always been interested in books, so that seemed
a good ( 3 ) _____ __ to choose. I run the business from home and send and
receive books by post so I don’t need my own (4 )________ . Sometimes I travel
to book fairs and sometimes I have a stall in the market. It was a bit frightening
at first, being (5 )________ , but I’ve got used to it now and I really appreciate
the feeling o f independence I get from “be my own boss". John got some advice
from his bank manager about the financial (6 )________ of his business and also
took out a small (7 )________ to buy stock. After only two years the business
was making a profit. The secret o f success, according to John, is to (8) ________
in certain area (detective fiction and cookery in his case) so that you always have
the book the serious collector is looking for. John posts books to his consumers
and then waits for them to send (9 )________ . At first, he wasn’t sure whether
people would pay up promptly. “In fact, this hasn’t been the problem I thought it
might be. Most customers are very (10)
' and it’s only the occasional
one that causes problems.
II. WORD FORMS (20 pts)
1. Supply the correct word form of the words in brackets.(10 pts)
1. She stood there completely____so I had no idea at all what she was thinking
(EXPRESS)
2. Any actor who becomes known for one role is in danger of becoming_______;
(TYPE)
3. This school was once____as a military h ospital during the war. (REQUIRE)
4. So far, the United States has said it cannot agree t o ________ all types of
antipersonnel mines. (LAW)
5. In Scotland, there is greater emphasis on________by individual schools.
(VALUE)
6. According to a recent survey in Britain, more women than men emphasized
and trust. (CONFIDENCE)
7. People also read these papers for their reviews o f new books, films and plays
and for their________ . (EDIT)
8. Since most important problems are_____ there are several alternatives to
choose from, each with unique advantages and disadvantages. (FACET)
9 The rocks appear to be stationary but in the high winds that whip across this
desert landscape, they are in reality moving________ . (PERCEIVE)
10. After the cup final, the________ team bussed back to the hotel
parading the
(TRIUMPH)
cup as they went.
29

2. Supply each gap With the correct form o f the words given in the box. (10 pts)
alter slouch course min d
recede round reside real
A GREAT STAYCATION
Holidays at home are usually a last (1) __ ______when all other options have
been ruled out for one reason or another, but, in these tough times when money
is perhaps tighter than ever before, the grim (2 )________ that the stay-at-home
vacation may be the only realistic (3 )________ is one that more and more of us
are faced with.
However, this does not have to mean a (4 )_________ time in the same old
(5 )________ you are in for the other 355-odd days o f the year. For those willing
to think outside the box a little, there are, in fact, a (6) ________ of possibilities
that should be explored.
Ever thought about a house swap, for example? The house swap is ultimate
holiday ( 7 )_______ buster. And there are now websites on which (8 )_________
individuals, couples and families looking to get a flavour of the life lived in
some else’s home can hook up and start house swapping.
Okay, so it's not the two weeks in Gran Canaria you might have hoped for,
but staying in someone else’s (9 )________ for a few days at least, w hether it be
ten, fifty or one hundred miles away, sure beats (1 0)________ around at home
on your sofa.
III. ERROR IDENTIFICATION (10 pts)
There are 10 mistakes in the following passage. Identify the mistakes and
then correct them. (10 pts)
Have you ever tried a strawberry pizza? If you went to Oxnard, the
"Strawberry Capital of California," in May, you could!
Oxnard is in Southern California and the part o f the state takes its strawberries
very serious. At the two-day California Strawberry Festival you can sample
strawberries prepared in all kinds o f ways. In addition to traditional treatment such
as strawberry shortcake, strawberry jam, strawben-y tarts and strawberries dipped
into chocolate, there is strawberry pizza! This dessert pizza is topped with
strawberries, sour cream, cream cheese and whipped cream on a sweetened bread
baked like a pizza. Strawberry kabobs dipped in powdered sugar are another
delicacy. And drinks like a strawberry smoothie can wash it all away.
Strawberries are a big businesses in Oxnard. Twenty-four companies harvest
and cool nearly 16 million trays of berries, which are shipped throughout North
America as long as to Germany and Japan. The festival, which attracts more than
SO
85,000 visitors, features three stages with musical entertainment, 335 arts and
crafting exhibits, strolling musicians, clowns, artists, face-painting, contests, and
a "Strawberry land" for children with puppets, magicians, musicians, and a
petting zoo.
1.
4.
7.
2.
5.
8.
3,_
6."
9."
10~
IV. SE NT EN CE T RA NS FO RM AT IO N (20 pts)
Rewrite the sentences with the given words or beginning in such a wa y that
their meanings remain unchanged.
1. “Because of this new evidence I have no alternative but to release you,” the
judge told the accused.(LIGH T)
—> “ ..................................... ...................... I have no alternative but to release
you,” the judge told the accused.
2. It is important to know the difference between a joke and a lie. (DRA W)
—>It is important to know ......................... ’.............................be tween a joke
and a lie.
3. The government recommends a balance of reward and punishment when
dealing with young offenders. (ST ICK )
—» The government favours a ............................................ to young offenders.
4. Let’s all work together, and w e’ll finish the job very quickly. (NEX T)
-> We'll have this job done.....................................................if we cooperate.
5. He was really jealo us when he saw his brother’s new car. (GRE EN )
—> He w as............................................................see his brother 's new car,
6. It’s sad, but unemploym ent is unlikely to go down this year.
—> S ad.......................... ........................ ....................................................
7. People believe that the Chinese invented paper in 105 A.D.
-» P ap er ....................................................... ..............................................
8. It’s a pity that you wrote that letter.
I'd .............. ........................ .................. ................. ..............................
9. He decided to repair the thing himself and not to take it back to the shop.
-» R athe r.............. .................................. .........................'........................
10. I'm sure it wa sn't Mrs. Elton you saw because she’s in Bristol.
-» It can’t ............... ......................................................................................
31
TRƯỜNG T HPĨ CHUYÊ N LÊ HỔNG PHONG - TP. Hổ CHÍ MINH
A. MULTIPLE CHOICE:
I. WORD CHOICE:
Choose the best options (A, B, c, or D) that best complete the follow ing sentences.
1. We are not kn ow n________ at'a ll, and as we grow, we feel a progressive
lack o f individual personality.
A. gruelingly B. severally c. expensively D. brusquely
2. He has fled to the mountains o f Galicia,______ he cannot possibly escape on
horseback over the border.
A. meanwhile B. heretofore c. whence D. indefinitely
3. Given that Haiti is vulnerable to hurricanes and earthquakes, it would be
to establish building codes and other disaster response initiatives.
A. prudent B. tiny c. profound D. stern
4. While her mother sat at the window, striving to read, the child, who was in
one o f her moods of obstreperous gaiety, began playing a grand game.
A. boisterous B. tentative c. creative D. precarious
5. The setup is intimate: audience members surround the stage on three sides.
A. formal B. unusual c. mutual D. cozy
6. This very morning, she announces, she has managed to procure what might
be the last two crates of peaches in France.
A. obscure B. conceal c. consume D. obtain
7. in their eyes, I saw not only excitement for the equity we offered, but the
______ belief that they would be entrusted to do their jobs with my counsel,
if they sought it, but without unwanted meddling.
A. modest B. positive c. earnest D. mutual
8. Her ivory brow ______ in delicate lines.
A. furrows B. duplicates c. ambles D. mutters
9. The fashion of the last Louis but one, of the line that was never to break - the
14th Louis - w a s______ in their rich furniture; but, it was diversified by
many objects that were illustrations of old pages in the history o f France.
A. varied B. conspicuous c. invisible D. negative
10. A man of stainless reputation, his deeds and words have almost invariably
been on the side o f_____ .
A. balefulness B. succession c. righteousness D. resemblance
11. STRUCTURES AND GRAMMAR:
Choose the best options (A, B, c, or D) that best complete the following sentences.
1. Last weekend,_______ nothing to watch on television, we sang karaoke
together.
A. there being B. there having c. having had D. being
2. The upper branches o f the tallest trees produce more lea ves_______ other
branches.
A. than do B. than have c. than they do D. than it does
32
3. He drove at full speed lest h e_______late for the appointment.
A. was B. would be c. be D. shouldn’t be
4. You _______ the questions in the order they asked. You mixed them up in the
wrong way.
A. had to B. must have answered
c. didn’t need to answer D. should have answered
5. We would sooner Mr. Manh______ US the urgent information the other night.
A. sent B. would have sent c . had sent D. send
6. Ms. Phi is ranked the best student ___she has made a point o f studying hard.
A. hence B. in that c. unless D. let alone
7. Not only the Smiths but also their next-door neighbor_______ more trees in
the neighborhood thus far.
A. was planting B. plant c. have planted D. has planted
8. It was right in the middle o f the school y ard_______ I saw a strange alien.
A. where B. which c. that D. which
9. That is (a n) _______
A. yellow useful Dutch gold alarm clock
B. useful yellow Dutch gold clock alarm
c. useful yellow gold Dutch alarm clock
D. useful yellow Dutch gold alarm clock
10 .1 w on’t _______ those children making a noise in my house!
A. allow ■ B have Ci let D. tell
III. PREPOSITIONS AND PHRASAL VERBS:
1. Luckily, the rai n___ so we were able to play the match.
A. gave out B. got away c. went away D. held o ff -
2. I don't like the way that Jack is always trying to _____trouble between US.
A. dish out B. rub up c. stir up D. spark out
3 .1 am ___with work at the moment.
A. snowed under B. piled up c. messed up D. knocked off
4. This scandal ha s______ criticism raining down on Mr Deby from all sides.
A. taken B. got c. brought D. made
5. I see a lot of people with this new hairstyle. It seems to b e___
A. piling up B. bringing off c. coming off D. catching on
6. ‘Do you think Dennis took the money?’ - ‘1 wouldn’t _____ him.’
A. put it past B. think it throughc. pass it over D. rub it up
7. Rather than take his time to think about the questions, the interv iew ee_____
out the first answer that came into his head.
A. blundered B. blurted c. bungled D. botched
8. Whenever the re’s some fresh scandal about the royal family, the public are
always eager t o _____ it up.
A. flap B. swish c. lap D. gulp
33
9. She's one of those people who are alway s_____ and asking questions about
other pe ople’s private lives.
A. poking around B. digging up c. prying out D. spying on
10. The blue sundress se t________ her long blonde hair.
A. up B. off c. forth D. in
IV. CO LLOC ATION S AND IDIO MS:
1. The luxurious office accentuated the manager’s po sitio n____ It enhanced
his power and his sense of his own worth. And it made other people feel small.
A. on the pecking pole B. in the nibbling line
c. at the nipping post D. in the packing order
2. ‘Have you tried to read the guarantee?’ - ‘There’s so much le gal _____ that
it’s impossible to understand.’
A. mumbo jumbo B. bongo bongo c. abracadabra D. okey-dokey
3. When several companies showed interest in buying the film rights to his
novel, he knew he h ad ___ _.
A. upped the ante B. scooped the bag
c. caught the fat one D. hit the ja ck po t.
4 .1 can't go out wearing something like that. I’d be the ____o f the neighborhood.
A. make-me-laugh B. laughing stock c. laughter maker D. laughing gas
5 .1 grew up in this old house, so I know e ve ry _____o f it.
A. hook and sinker B. lock, stock and barrel
c. step and stop D. nook and cranny
6. A large group of teenagers wer e___ around the entrance to the zoo.
A. mulling B. mudding c mauling D. milling
7. The pally was alre ad y___ by the time we arrived. Everyone was singing and
dancing.
A. in full swing B. up in the air c. over the moon D. under the cloud
8. They have serious problems .That’s why their relationship is on t h e___
A. cliffs B. rocks c. stones . D. grass
9. What she told me was a ___of lies.
A. pack B. load c. herd D. flock
10. Janet will see you if you use the com puter without permission. She has eyes
like a _______ .
A. bird B. goose c. hawk D. fox
V. READING COM PREHENSION
READING COMPREHENSION 1:
Read the following passages carefully and answer the questions below them.
THOU GHT CRIME
As he revisits the fr actur ed antihero o f two p revious novels, James Sallis
talks to Chris Wiegand about writing crime fiction foc using on character
rather than plot.
34
"I felt like I was stumbling, groping around in the dark. Didn't know where I
was going in the next paragraph; didn't know what I was doing in the next
chapter. But 1 somehow found my way." Speaking softly and slowly, with the
slightest Deep South twang, James Sallis is on the line from Phoenix to discuss
his new novel, Salt River. It's the shortest of his increasingly slim tales to date,
but took the longest to write. Sallis says he's relying more on improvisation and
abandoning "the certainties with which I began writing". As Sallis recalls this
struggle for direction, he sounds like his lost-as-hell antihero, Turner. He
introduced the existential detective in Cypress Grove, where he fulfils a selfimposed exile in a
one-horse town outside Memphis. A sequel, Cripple Creek,
followed. Now Salt River completes the Turner trilogy.
Sallis presents Turner as a man defined and haunted by what he no longer is:
a soldier, a cop and a convict. Episodes from his former lives are scattered
through the books as flashbacks. "Turner is a man whose life has gone through
abrupt changes," Sallis explains. "I felt that the novels' structures should reflect
the seeming discontinuities of his life." It's a technique that is at odds with the
thumbnail sketches favoured by crime writers intent on establishing a novel's
cast quickly before cracking on with the plot.
If he takes his time when it comes to characterisation - we don't learn
Turner's first name until midway through the second book - Sallis also has a
laidback approach to story. The barely-there storylines in Salt River almost
evaporate on the page. You don't get lost in his plots, they tend to lose
themselves. "Plots are a contrivance - our lives are plotless - yet they're
necessary, I think, to literary form," explains Sallis. "My way of dealing with
this has been to move the plot offstage a bit, to write around it." Is this why,
when I think of Sallis's books, I'm hit by smells of home-brewed coffee and wild
magnolia rather than anything that actually happened?
"Those are the parts of the world that we own, what comes back to us about
times in our own lives when we think of the past," he insists. "All too often I'm
reading this great book with a solid setting, characters that walk right into my
own life, then somewhere around the fifth or sixth chapter the plot kicks in - and
all that falls into the background. I want all that stuff, that surround, to remain in
the foreground."
He does that "surround" very well, evoking the grit and wit of rural southern
life with ease. (Sallis grew up in Helena, a small town on the banks of the
Mississippi.) He has an ear for sleepy, porch-front wisdom, with his characters
often swapping homilies to the sound of cicadae on long summer nights. " Were
there books in the house when he was young? "We're from lower-class, southern
stock." he says. "My dad was fairly typical, hard-working, blue-collar - hunting
on weekends, fixing lawnmowers ... My brother and I developed this love for
books. The first things I read were science fiction."
35
So were the first things Sallis wrote. He began to sell stories to magazines
then made a life-changing move to London in his early 20s to edit groundbreaking
sci-fi magazine Ne,w Worlds with Michael Moorcock at the fag-end of the 60s.
Working alongside Moorcock opened Sallis's eyes to hard-boiled crime fiction:
"Mike introduced me to books by Raymond Chandler and Dashiell Hammett,
which I had never found here in the States oddly enough." When Sallis began to
write his own crime novels, critics distinguished him as a supposed rare breed -
the "literary crime writer". Sallis finds the tag "useless and foolish" but his novels
have an undeniable intellectual ism that remains rare in the genre.
Sallis gives me the skinny on the next novel: "The major characters are a
contract killer who is dying of cancer, a 12-year-old whose parents have
disappeared and who goes on living by himself in the family house, and a pair of
detectives, with the point o f view shifting among them chapter to chapter."
Long-Legged Fly and Cypress Grove started out as standalones, so who
knows if this will launch another series. As Sallis says: "I never know I'm
jumping in the river, I always think I'm just sticking my feet in."
1. What does James Sallis suggest about his latest novel in the first paragraph?
A. He adapted it as he went along.
B. The main character is based on himself,
c. It caused him to doubt his writing ability
D. He struggled to portray the main character.
2. In the second paragraph, Sallis implies that his novels differ from those of
many other crime writers in that
A. the events in the storylines are not in chronological order.
B. he prefers his characters to feature prominently throughout,
c. his storylines are based mainly on flashbacks.
D. the plots are rather slow to develop.
3. In paragraph three, the writer wonders if his reaction to Sallis' novels are due
to the fact that
A. the plot is not the central focus, c. the writing style is very descriptive.
B. the storylines are hard to follow. D. the storylines milTor human experience.
4. -.What does “all that” refer to?
A. the intricacies o f a plotline
B. aspects of a novel that a reader than identify with
c. detailed descriptions of characters
D. the elements o f a novel that set the scene
5. According to the fourth paragraph, one way in which Sallis' upbringing is
reflected in his books is through
A. his poetic writing style.
B. the themes he explores.
c. a focus on characters from the lower class.
D. some o f the dialogue between characters.
36
6. Wliat do we learn about Sallis in the sixth paragraph?
A. He was heavily influenced by collaboration with other writers.
B. He doesn 't see himself as an exceptionally good crime writer,
c. He is dismissive of the way he is defined as a writer.
D. He wanted to take an established writing style a step further.
7. In the final paragraph, what does Sallis conclude about his writing projects?
A. He always starts with the basis o f a novel and then sees how it develops.
B. He doesn't envisage them being ambitious projects at the start.
c. He enjoys not knowing what direction they will eventually take.
D. He doesn 't base his expectations on previous results.
8. The word “at odds w ith” is closest in meaning to
A. at variance with B. consistent with c. averse to D. in tandem with
9. The word “contrivance” is closest in meaning to
c. loss D. an artificial
A.need B. deception
product
10. The word “skin ny” is closest in meaning to
A. lesser-known information c. little information
B. the bottom line D. the thread J
READING
COMPREHENSION 2
GRAPHIC NOVELS: A FRESH ANGLE ON LITERATURE
Has the graphic novel - a fictiona l story presented in comic-strip form at -
fin ally become intellectually respectable?
Graphic novels have just landed with an almighty kerspla t/Yc n days ago, two
such works were shortlisted for the Shakespeare Book Awards for the first time
in the history of the prize, in two different categories. This was no publicity
stunt: neither panel knew what the other had done. This is, surely, the moment
when the graphic book finally made its entrance into the respectable club room
of high literature. Hang on, though: can you compare a graphic novel with the
literary kind? Wouldn’t that be like comparing a painting with a music video? Or
is it time we started seeing them as comparable mediums for storytelling? If so,
what next?
Robert Macfarlane, the chairman of another major literary award, says he has
no objection in principle to a graphic novel being submitted for the prize. In fact,
he has taught one, Art Spiegelman's Maus, alongside the works of Russian writer
Tolstoy and Don Quixote (by the Spanish writer Miguel de Cervantes) at the
University of Cambridge, where he works in the English Faculty. ‘The idea of
outlawing the graphic novel doesn’t make any sense to me,’ he says. ‘I don’t
seg reg ate it from the novel. The novel is always eating up other languages, media
and forms.' Graphic fiction, he says, is ‘another version of the novel's long
flirtation with the visual’. This is, he declares, ‘a golden age for the graphic novel.’
And he’s right. We are seeing a boom in graphic novels. Since Maus was
awarded a Pulitzer Prize in 1992, they have gone on to devour every literary
37
genre going. But so far, graphic novels have politely stood aside and let
conventional books win the big prizes. Now they want the vote. Fighting for the
graphic novelists’ cause, astonishingly, are some hefty prize-winning writers.
The English novelist and poet A. s. Byatt is passionately in favour of graphic
novels competing with regular ones. Byatt, who is a huge fan óf Spiegelman ’s
Maus, thinks that French-Iranian artist Marjane Satrapi’s graphic novel
Persepolis stands ‘head and shoulders above most novels being produced. It’s
more interesting and more moving. It’s able to be serious because it can carry
itself along on this unserious form. It allowed her to be witty about things that
are terrible. And that’s why it’s a major work of art’.
The genius of the graphic novel, as the English writer Philip Pullman
explains, is that it can bring'into play so many levels of narrative by layering
them on top of each other. Take American Alison Bechdel’s brilliant Are You My
Mother? - in a single page, she can depict a memory o f being with her m other in
her childhood, dialogue between herself and her mother as they chat on the
phone in the present, plus an image o f herself toiling at her desk, trying to write
her memoir. And what Bechdel and her mum are saying on the phone links to the
diaries of the early 20th-century writer Virginia Woolf, which Bechdel also
brings to visual life. Try doing that with words - it would take a chapter.
Bechdel does it in a few panels. That, in the end, is precisely what keeps graphic
literature so distinct from prose narrative.
Graphic novels and traditional novels demand, to be sure, the same amounts
of time, intellect and artistry from the authors. But that doesn’t mean they’re the
same thing. A few years on, will you be clicking the buy button on a graphic
novel as happily as you’d pick up a work by a traditional novelist? Even Bechdel
confesses that her reading habits are still struggling out of the past. ‘Honestly, I
would be slightly more inclined to pick up a non-graphic work ,’ she says. ‘At
this point, there’s not a huge number of graphic novels that are about topics that
interest me. But that, too, is changing. We’re becoming more visually literate.
There’s some reason for these graphic novels creeping into the canon. We’re
reading differently from how we used to 200 years ago.’
1. What does the writer say about the nomination of two graphic novels for the
Shakespeare Book Awards?
A. It revealed the closed-mindedness of the literary establishment.
B. It was the result of confusion among members o f the panel,
c. It generated debate about the true purpose o f the prize.
D. It was not deliberately calculated to attract people’ attention.
2. The w ord ‘panel’ in the text refers to
A. The novelists B. The specialists c. The voters D. The graphic designers
3. What does Robert Macfarlane suggest about graphic novels?
A. Th eir long-term success has now been assured.
B. Their banning from literature courses has backfired.
38
c. They are a logical step in the development of fiction.
D. They tend to be less innovative than traditional novels.
4. The word ‘segregate’ is closest in meaning to
A. exclude B. disengage c. disaffiliate D. victimize
5. In the third paragraph, the writer suggests that, in the past, writers of graphic T
novels j
A. lacked the support of influential figures. Ị
B. were systematically discriminated against.
c. tended to accept their inferior social standing.
D. underappreciated the importance of literary awards.
6. The word ‘hefty’ is closest in meaning to
A. cumbersom e B. prominent c. immense D. vigorous
7. What does Byatt suggest when the novelist and poet says Persepolis stands
‘head and shoulders above most novels being produced’?
A. The work is far superior to most novels being produced.
B. The quality of the graphic novel challenges all the 1 iterary norms.
c. The author of the graphic novel has gone great lengths to finish the work.
D. The work is being published in inordinate numbers.
8. The writer discusses Alison Bechdel’s book to make the point that graphic novels
A. can have just as much narrative depth as traditional novels.
B. are able to. incorporate a surprising range of different novels,
c. can represent the workings of memory in sophisticated ways.
D. enable writers to deal with different aspects o f a story at once.
9. Bechdel is quoted in the last paragraph to make the point that
A. interest in graphic novels reflects a more general trend.
B. many readers lack the skills to fully appreciate graphic novels,
c. it is difficult to persuade people to take graphic novels seriously.
D. graphic novels are far outnumbered by quality traditional novels.
10. In this article, the writer is
A. analyz ing the preoccupations of graphic novelists.
B. outlining the origins o f graphic novels.
c. describing the working practices o f graphic novelists.
D. ev aluating the merits of graphic novels.
VI, GUIDED CLOZE
GUIDED CLOZE 1: Choose the most appropriate words to fill in the blanks.
SUGAR: THE FUEL OF THE FUTURE?
Brazil has a good track (1 )______ in research in many areas of science and
technology. It is in the field of bio-energy, however, that the country (2 )______
to make Its biggest contribution. Brazil is the world’s largest producer of sugar
and since 1975 has been fermenting sugar-cane juice (3 )_______ a substance
called ethanol, which can be used as motor fuel. For many years, the programme
39
(4) ■ in virtual isolation from the rest of the world, using fairly low-tech
methods. Recently, however, the government has been investing (5 )______ in
research aimed at improving all stages in the process from sugar-cane biology to
engine efficiency.
Whilst the motivation for the investm ent is largely (6 )_____ by energy needs
rather than environmental concerns, the fue l's green ( 7 )______ are now also
being emphasized. Net emissions of carbon dioxide from a car ( 8 ) _____ on
sugar ethanol are jus t 20 per cent o f those from a petrol-fuelled vehicle. (9 )____ ,
the bio-energy programm e aims to achieve a significant increase in supply
without a corresp onding rise in the amount o f farmland (1 0) _____ to sugar cane.
1. A. record B. story c. reputation D.success
2. A. stands B. sets c. rests D. ranks
3. A. into B. in c. out of D. on
4. A. took place B. held forth c. carried on D. kept up
5. A. highly B. strongly c. vastly D. heavily
6. A. driven B. stemming c. drawn D. arising
7. A. en dorsements B. credentials c. testam ents' D. referrals
8. A. performing B. working c. running D. burning
9. A. Nonetheless B. Furthermore c. Otherwise D. Instead
10. A. occupied B. applied c. consigned D. devoted
GUIDED CLOZE 2:
HAND-MA DE HISTORY: THE BAYEUX TAPESTRY
If a picture is worth a thousand words, the seventy-three scenes of the Bayeux
Tapestry speak volum es. The tapestry narrates, in pictorial (1) , William
Duke of No rma ndy ’s invasion and conquest of England in AD 1066, when he
defeated the Saxon forces o f King Harold at Hastings. Historians believe that the
work was (2 )______ in England, probably around AD 1092, and that tit was
comm issioned by Odo, Bishop of Bayern, William’s half brother, who ensured
his fame by figuring (3) _____ _ in the tapestry’s later (4) ______. Legends
connecting it with William ’s wife Mathilda have been (5 )______ .
The Bayeux tap estry is not, (6 )______ speaking, a tapestry, in which designs
are woven into the fabric, but rather a crewel form of embroidery, the pictures
being made by stitching woolen threads into a background of plain linen. The
threads, in (7 )______ of red, yellow, blue and green, must turned have been
jew el bright, but have (8 )____ ._ light brown with age. Moreover, one end of
the now 20 inch (50 cm ) broad and 231 feet ( 70 m) long cloth is missing.
You can view the Bayeux Tapestry in the Willian the Conquero r Centre,
Bayeux, Norm andy, France. An enduring (9) ______ of the times, it is as
valuable a (1 0)______ of evidence for the Norman Conquest as photograp hs or
films are today.
40
1. -A. fashion B. type c. design D. form
2. A. originated B. invented c. created D. manufactu
3. A. prom inently B. strongly c. powerfully D. sufficientl
4. A. views B.scenes c. frames D. pictures
5. A. disowned B. dispersed c. disgraced D. discounte
6. A. normally B. strictly c. truly D. sincerely
7. A. colours B. shadows c.sh ad es D. varieties
8-. A. once B. then c. before Di earlier
9. A. witness B. confirmation c. testimony D. proof
10. A. segment B. part c. piece D. portion
B. WRITTEN TEST
I. OPEN CLOZE . , ■
OP EN C LO ZE 1: Fill in each numbered blank with ONE suitable word.
Iris scanning can seem very futuristic, but at the heart of the system is a.
simple CCD camera. It uses both visible and near-infrared light to (1) ____ a
clear, high-contrast picture of a person's iris. With near-infrared light, a person's
pupil is very black, making (2 )____ easy for the computer to isolate the pupil
and iris.
When you (3) __ into an iris scanner, either the camera focuses automatically
or you use a mirror or audible feedback from the system to make sure that you
are positioned correctly. Usually, your eye is 3 to 10 inches (4) _ __ the camera.
Iris scanners are becoming more common in high-security applications
because people's eyes are so unique (the chance of (5 )____one iris code for
another is 1 in 10 to the 78th power). They can allow more than 200 points of
reference for ( 6)____ , as opposed to 60 or 70 points in fingerprints.
The iris is a visible (7) ____ protected structure, and it does not usually .
change over time,- becoming ideal for biometric identification. Most of the time,
people's eyes also remain unchanged (8 )____ eye surgery, and blind people can
use iris scanners so (9 )____as their eyes have irises. Eyeglasses and contact
(1 0) ____ typically do not interfere or cause inaccurate readings.
OP EN C LO ZE 2
In less than three months' time - on 14 June, to be precise - the 21st FIFA
World Cup™ kicks (1)_____ at Moscow's Luzhniki Stadium, as the hosts take
on Saudi Arabia.
That leaves US plenty of time to enjoy a countdown. (2 )_____now and the
start of the World Cup, we will take a closer look at a different statistic from the
history of the tournament each day.
When Brazil took possession of the Jules Rimet Trophy (3 )_____ winning
their third title at the 1970 FIFA World Cup™ in Mexico, FIFA commissioned a
new Trophy for the tenth global finals in 1974. Artists from seven countries
41
submitted a total of 53 (4 )_____ for the new cup, with Italian sculptor Silvia
Gazzaniga’s work ultimately ( 5)_____ the vote.
“The lines spring out (6 )_____ the base, rising in spirals, stretching out to
receive the world," said Gazzaniga, ( 7 )_____ his creation. "From the remarkable
dynamic tensions of the compact body of the sculpture rise the figures of two
athletes at the stirring (8)____ of victory.”
The FIFA World Cup Trophy remains in FIFA's possession at all times and
cannot be won outright by a country.
The current Trophy is 36.8cm high, crafted from solid 18-carat gold and
weighs exactly 6.175kg. The base contains two (9)______ of semi-precious
malachite, while the underside of the Trophy is engraved (1 0) _____ the name of
every FIFA World Cup™ winner since 1974.
II. WORD FORMATION
WORD FORMATION 1: Supply the correct forms of the words given.
1. The cottage has n o ______ _ access but can be reached by a short walk across
the moor. (VEHICLE)
2. He climbed out, leaving the ca r_________ on its roof. (END)
3. Physicians must exercise caution when prescribing_______ . (DEPRESS)
4. He pointed out that e-books were not only cheaper, because of the lack of
wear and tear and thefts, but they also offered great opportunities for older
readers. (HOUSE)
5. One of the United Nations’ earliest success es,w as to promote a largely
peaceful process of _______. (COLONY)
6. After several threatening calls, we decided to g o___________ . (DIRECT)
7. At least in theory, this should make the region increasingly attractive to
foreign investment from fur the r_____ __________. (FIELD)
8. ‘Villa’ was something of a _ __________the place was no more than an old
farmhouse. (NAME)
9. Mickey Mouse made his screen debut on November 18, 1928 and has been a
____________ ever since. (CROWD)
10. A baby leopard has been pictured suckling a lioness in the wild, in the first
ever documented example o f________ parenting of its kind. (SPECIES)
WORD FORMATION 2: Fill in the blank with an appropriate form o f one o f
the words given to make a meaningful passage.
regulate value back fe ed
except provide direct decid e
Children’s educational environment contributes to the IQ score and the way
intelligence is used. For example, a very close positive relationship was found when
42
children’s IQ scores were compared with their home educational (1 )_____ . The
higher the children’s IQ scores, especially over IQ 130, the better the quality of
their educational (2 )______ , measured in terms of reported verbal interactions
with parents, number o f books and activities in their home etc. Because IQ tests
a r e (3 )______ influenced by what the child has learned, they are to some extent
measures of current achievement based on age-norms; that is, how well the
children have learned to manipulate their knowledge and (4) _ _ _ _ _ within the
terms of the test. But IQ tests can neither identify the processes of learning and
thinking nor predict creativity.
Excellence does not emerge without appropriate help. To reach a(n) (5 )____
high standard in any area very able children need the means to learn, which includes
material to work with and focused challenging tuition -and the encouragement to
follow their dream. There appears to be a qualitative difference in the way the
intellectually highly able think, compared with more average-ability or older pupils,
for whom external regulation by the teacher often compensates for lack of
internal regulation. To be at their most effective in their (6) _______ strategies,
all children can be helped to identify their own ways o f learning - (7 )_______ -
which will include strategies o f planning, monitoring, (8)_______, and choosing
what to learn. Yet, in order to learn by themselves, the gifted do need support
from their teachers. Conversely, teachers who have the tendency to (9 )_______
can diminish their gifted pupils’ learning autonomy. Although (1 0) _______ can
produce extrem ely high examination results, these are not always followed by
equally impressive life successes.
III. E RROR IDENTIFICATION
Identify 8 mistakes in this passage and suggest corrections.
Over the last century the world has become increasingly smaller. Not

geographically, of course, but in the sense that media, technology and the
opening of borders has enabled the world’s citizens to view, share and gain
access to a much wider range of cultures, societies and world viewpoints.
5 It stands to reasons that in order to absorb, configure and finally form
opinions about this information-laden planet, children must be supplied with
certain tools. Including in this list of “tools” are: education, social skills,
cultur al awareness and the acquisition of languages, the most important of
these being the later. Until recently, a child who had the ability to speak
10 more than one language would have been considered a very rare entity.
This one-langu age phenomenon could be contributed to a combination of
factors, in which the monolingual environm ent in which a child was raised
played a strong role, so did the limited, biased education of the past.
43
Nowadays, the situation has undergone an almost opposite reversal. In
15 the majority of North Americ an and European countries, most children are
given the opportunity to learn a second or even third language. In some
cases, learning a foreign language is a compulsory subject in the state
school syllabus while in others, children are born into bilingual parents,
who may teach the children two languages. Bringing up one’s child bilingually
is not a decision to be taken slightly. Both parents must consid er long and
hard the impli cations involved in raising a child in two-language home.
1.
4.
7.
2.
5.
8.
3._
6. _
9 .’
io’
IV. SENTENCE TRANSFORMATION:
Rewrite the following sentences using the words given.
1. It’s believed that the school prestige has improved immensely in the last few
decades. (MEASURE)
—> The school pre stige___________________________________________ .
2. If I were him, I would return home no later than 11 o’clock.
-> It is advised that______________________________.___________ latest.
3. It was rash of Jimmy to react so aggressively that his wife felt heartbroken.
(IMPULSE)
—> Had Jim my_________________ _ ______________________________.
4. Tony regretted criticising his friend’s business plan so strongly. (POURED)
-> If ______________________ • ________________________ .
5. Harry did n’t realize who the lady was until she
moved forward into the light.
(D A W )
— — — — — ——— — — •
—> N ot__________ 2___________ i
6. The whole affair has been a set of misfortunes from the beginning. (CHAPTER)
—> From start_________________________:___________________________.
7. You were so pessimistic about what happened. (VIEWED)
-» You shou ld______________________________________________light.
8. Come what may, you should look on the bright side. (CHIN)
-> No matter w ha t_________________________ . ________________
9. Nob ody is sure if the scheme will be allowed to go ahead. (GREEN)
—> It is still in ____________ _____________ _____________ ______ or not.
10. This patient’s condition is rather worrying if you look at his medical history.
-> Given
44
_ _ _ _ _ _ _ _ TRƯỜNG THPT CHUYÊN LÊ KHIẾT - QUÀNG NGÃI_ _ _ _ _ _ _ _ _
A. MULTIPLE CHOICE
I. WOR D CHOICE (5pts)
Mark the letter A, B, c or D on your answer sheet to indicate the correct
answ er to each o f the following questions
1. Brain cancer r eq uires.... ........... treatment such as surgery.
A. aggressive B. confrontational c. malignant D. rigorous
2. He praised his wife for her dignity under t h e ................of the tabloid press.
A. onslaught B. assault c. onset D. offensive
3. Buying a car was an important...... ........ for them.
A. transform ation B. translation c. transaction D. transportation
4. S h e's ................. She tends to think a lot and not to say a lot.
A. an introvert B. inverted c. an extrovert D. subdued
5. It’s ...............that he never mentioned our argument; I wonder why he didn’t.
A. special B. rare c. curious D. eccentric
6. In a ll.............., he’s already left.
A. odds B. .probability c. certainty D. possibilities
7. Many people do n't use their computers to their f ull ..............
A. future B. expectation c. potential D. hope
8. When our friends have bad fortune, we try to sh ow ....... ,.......
A. love B. sympathy c. affection D. pity
9. Are you having a ..,.................... exam before you sit the actual Exam in June?
A. fake B. mock c. false D. dress
10. The job you’ve been offered is a(n)....... opportunity to travel and meet people.
A. sole B. only c. unique D. single
11. STRUCTURES AND GRAMMAR
Choose the word or phrase which best completes each sentence.
11. He works until nine o’clock every evening, and that’s quite .............. the
work he does over the weekend.
A. ex cept for B. apart from c. without D: not for
12. The report makes the recommendation that no more prisons ..................
A. m ust be built B. be built c. had to be built D. should be building
13. I ............. a lot as an accountant but the work was just too boring.
A. was supposed to be earned B. ought to be earning
c. could have been earning D. should have earned
14 ................... I'd like to talk about m yself as the happiest person in the world.
A. Be that it may B. Strange as it might sound
c. How much strange it may be D. Strange though might it sound
15 .... her job, her sons and the housework, she doesn 't have a minute
for herself.
A. W hat w ith B. If if weren ’t for c. Barring D. Given
45
J
16................... as a masterpiece, a work of art must transcend the ideas of the
period in which it was created.
A. Ranking B. To be ranked
c. Being ranked D. In order to be ranking
17. We hav e.............. what action to take.
A. yet decided B. not yet decide c. not yet to decide D. yet to decide
18. So incredibl e...... .
of the Yellowstone area that people didn’t believe in its
existence.
A. explorer John Colter’s descriptions were
B. was explo rer John C olter’s descriptions
c. explorer John Colter’s descriptions
D. were explo rer John Colter’s descriptions
19. On the island ...............the only representation of the island’s handicraft.
A. remains B. does it remain c. did it remain D. remain it
20................. had worsened so quickly surprised the doctor.
A. Which the patient’s condition B. The patient’s.condition
c. That the patien t’s condition D. As the patient’s condition
III. PHRASAL VERBS AND PREPOSITIONS (5pts)
Choose the best option
21. The weather seems to b e ..... i....up after heavy rains.
A. tuning B. improving 'C. picking D. bringing
22. Because of the controversy, he p ac ks .............his job.
A. out B. in c. over D. up
23.1 think she m ad e.............the whole story.
A. up B. out c. up for D. to
24. We have a lot of financial difficulties, but at last, we carry i t .............
A. out B. on c. off D. over
25. Eth an ...............such a great deal of eff ort............. training for the marathon
that it is a pity he broke his leg two days before it took place.
A. wor e/ down B. carried/ on c. set/ off D. put/ into
26. Rebecca .............. on our conversation to tell US that James had jus t been
rushed to hospital.
A. cut in B. faced up c. got ahead D. broke up
27. I waited in Time Square for Alfonzo for more than two hours, but he never
A. show ed up B. hung around c. pulled off D. turned in
28. Issac wishes that he could ............what he had said to Christina because he
really upset her.
A.tal kout B. take back . c. throw up D. tell off
29. If you pay the restaurant bill with your credit card, 1 w ill..........with you later.
A. settle down B. settle up c. pay back D. pay up
30. For a whole month, Muslim s..... . eating and drinking during daylight hours.
A. abstain from B. keep from c. stay from D. stand from
46
IV. C OLLOCATIONS AND IDIOMS (5pts)
Choose the best option
31. Unfortunately, our local cinema is on th e________ of closing down.
A. verge B. hint c. edge D. threat
32. When it is very hot, you m ay _______ the top button of your shirt.
A. undress B. unwrap c, untie D. undo
33. She clearly joined the firm with a (n )______ to improving herself
professionally .
A. view B. aim c. plan D. ambition
3 4 .1 shouldn't go outside without a raincoat because it's raining cats and dogs.
A. it's just started to rain B. it's raining very heavily
c. it's going to rain D. it's raining a little
35. On second thoughts, I believe 1 will go with you to the theater.
A. Upon reflection B. After discussing with my wife
C. For this time only D. For the second time
36.1 reali zed______ that he was a thief.
A. sooner of later B. all a long c. at the beginning D. eventually
37. Mr. Dawson was given the award in ______ of his services to the hospital.
A. spite B. charge c. recognition D. sight
38. The sixth time he called me at night was th e_________ .
A. lost cause B. last straw c. touch and go D. hotair.
39. His flat looks so___ that it is difficult to believe he just had a party last night.
A. spick and span B. by and large c. safe and sound D. sick and tired
4 0 .1 am not able to go anywhere this weekend because 1 am up to my____in work.
A. neck B. nose c. waist D. ankle
V. READING COMPREHENSION
READING 1: You are going to read a passage and choose the answer (A, B, c
or D) which you think fits best according to the text.
41. ______
Telephone, television, radio, and the telegraph all help people communicate
with each other. Because of these devices, ideas and news of events spread
quickly all over the world. For example, within seconds, people can know the
results of an election in Japan or Argentina. An international soccer match
comes into the home of everyone with a television set. News of a disaster
such as an earthqu ake or a flood can bring help from distant countries. Within
hours, help is on the way.
42. ______
How has speed of communication changed the world? To many people, the
world has become smaller. Of course, this does not mean that the world is
physically smaller. Two hundred years ago, communication between the
continents took a long time. All news was carried on ships that took weeks or
even months to cross the oceans. In the seventeenth and eighteenth centuries,
it took six weeks for news from Europe to reach the Americas. This time
47
difference influenced people's actions. For example, one battle in the war of
1812 between the English and the United States armies could have been
avoided if the warring sides had known that a peace agreement had already
been signed. Peace was made in England, but the news of peace took six
weeks to reach America. During those six weeks, the large and serious Battle
of New Orleans was fought and many lives were lost.
43. ______
An important part of the history of the world is the history o f communication. 7
In prehistoric times, people had limited knowledge of the world. They had
little information about geography, the study of the Earth. People knew very
little beyond their small groups except what was happening near their homes.
Later, people were organized into villages, and verbal communication
between small towns was possible. Still, the peop le’s knowledge was limited
because they had no outside information. Kingdoms and small countries then
developed , with a king directing the people. Cities developed, too, but still
comm unication was limited to the small geographical area of the country.
Much later in history, after the invention of the printing press, many more
people learned to read, and communication was improved.
44.
In this modern age, communication is so fast that it is almost instant. People's
lives have been changed because of the immediate spread of news.
Sometimes the speed is so great that it does not allow people time to think.
For example, leaders of countries have only minutes, or, at most, hours to
consider all the parts of a problem. They are expected to answer immediately.
Once they had days and w eeks to think before making decisions.
45. ______
The speed of communication demands a new responsibility from all people of
the world. People in different countries must try harder to understand each
other. An example is that people with different religions must try to
understand each other's beliefs and values, even if they do not accept them.
Sometimes their cultures are quite different.What one group considers a
normal part of life is strange to another culture. In some cases, a normal part
of one culture might be bad or impolite to people of another culture. That
kind of difference is a possible basis for misunderstanding. People must learn
not to judge others, but to accept them as they are. As the world grows
smaller, people must learn to talk to each other more effectively as well as
communicate more rapidly.
Match the headings given in the box below with their appropriate numbers
(41 - 45) that lea d the five paragra phs and write the letters A- H in the
corr espo ndin g n umb ered boxes. (The headings outnumber the paragraphs,
so yo u will not use all o f them).
A. A disadvantage of fast comm unication
B. High speed of communication and its benefits
48
c. Our shrinking world
D. Communication devices
E. A b rief history o f communication development
F. Modern communication and a change in thinking pattern
G. The changing world resulting from fast communication
H. Modern communication and expected responsibility
Then choose the correct answer to each of the following questions by circling
A ,B ,C ,o rD .
46. Modern communications ha ve ______ .
A. affected the results o f elections and news of disasters
B. only allowed people to see world sports events at home
c. kept people better informed of their world and beyond
D. made people happier, busier, but less informed
47. Before the invention of communication devic es, ______.
A. people gave better care to their local affairs
B. there was no transportation between countries
c. people were much interested in world affairs
D. people were mostly kept in the dark about the world
48. A negative aspect of fast communication is that i t______ .
A. makes people think too fast
B. will push governments into dead ends
c. deprives decision makers o f correct information
D. may rush governments into decisions
49. There were instances in which lives could have been saved i f______ .
A. intercommunication had been established
B. there had not been a delay in communication
c. offic ers’ demands of information had been met
D. carrier pigeons had arrived in time
50. The speed of communication has helped create opportunity fo r______ .
A. mutual understanding and cultural tolerance
B. better understanding and freer trade
c. the expansion of cultural differences
D. the growth of the physical world
DEADING 2. Read the following passage and answer questions 51-60.
BRINGING UP CHILDREN
Where one stage of child development has been left out, or not sufficiently
experienced, the child may have to go back and capture the experience of it. A good
home makes this possible - for example, by providing the opportunity for the child
to play with a clockwork car or toy railway train up to any age if he still needs to do
so. This principle, in fact, underlies all psychological treatment of children in
difficulties with their development, and is the basic of work in child clinics.
49
The beginnings of discipline are in the nursery. Even the youngest baby is
taught by gradual stages to wait for food, to sleep and wake at regular intervals
and so on. If the child feels the world around him is a warm and friendly one, he
slowly accepts its rhythm and accustoms himself to confo rming to its dem ands.
Learning to wait for things, particularly for food, is a very important element in
upbringing, and is achieved successfully only if too great demands are not made
before the child can understand them. Every parent watches eagerly the child's
acquisition of each new skill: the first spoken words, the first independent steps,
•or the beginning of reading and writing. It is often tempting to hurry the child
beyond his natural learning rate, but this can set up dangerous feelings of failure
and states of anxiety in the child. This might happen at any stage. A baby might
be forced to use a toilet too early, a young child might be encouraged to learn to
read before he knows the meaning of the words he reads. On the other hand,
though, if a child is left alone too much, or without any learning opportunities,
he loses his natural zest for life and his desire to find out new things for himself.
Learning together is a fruitful source of relationship between children and
parents. By playing together, parents learn more about their children and
children learn more from their parents. Toys and games which both parents and
children can share are an important means of achieving this co-operation.
Building-block toys, jigsaw puzzles and crosswords are good examples.
Parents vary greatly in their degree of strictness or indulgence towards their
children. Some may be especially strict in money matters; others are severe over
times of coming home at night, punctuality for meals or personal cleanliness. In
general, the controls imposed represent the needs of the parents and the values
of the comm unity as much as the child's own happiness and well-being.
With regard to the development of moral standards in the growing child,
consistency is very important in parental teaching. To forbid a thing one day and
excuse it the next is no foundation for morality. Also, parents should realize that
“example is better than precept”. If they are hypocritical and do not practise what
they preach, their children may grow confused and emotionally insecure when
they grow old enough to think for themselves, and realize they have been, to
some extent, deceived. A sudden awareness o f a marked difference between their
parents' ethics and their morals can be a dangerous disillusion.
51. The principle underlying all treatment of developmental difficulties in
child ren ______ .
A. is in the provision o f clockwork toys and trains
B. is to send them to clinics
c. is to capture them before they are sufficiently experienced
Đ. offers recapture o f earlier experiences
52. Learning to wait for things is successfully ta ug ht_____ _.
A. in spite of excessive demands being made
B. only if excessive demands are avoided
50
c. because excessive demands are not advisable
D. is achieved successfully by all children
53. The encouragement o f children to achieve new skills
A. should be focused on only at school B. can never be taken too far
c . will always assist their development D. should be balanced and moderate
54. Parental controls and d iscipline ______.
A. serve a dual purpose
B. are designed to promote the child's happiness
c .reflect only the values of the community
D. should be avoided as far as possible
55. 4 he practice o f the rule "Example is better than precept” ______ .
A. only works when the children grow old enough to think for themselves
B. would help avoid the necessity for ethics and morals
c. will tree a child from disillusion when he grows up
D. is too difficult for all parents to exercise
56. In the 1st paragraph, the author lays some emphasis on the role of th e______
in helping the child in trouble.
A. psychiatrists B. community c. family D. nursery
57. The phrase "conforming to” in the 2nd paragraph means .
A. adapting to B. accepting c. agreeing with D. following
58. The W'ord "zest” in the 2nd paragraph can be best replaced by ______.
A. appetite B. excitement c. enthusiasm D. enjoyment
59. The word, "imposed” in the 4th paragraph is closest in meaning to .... ..............
A. excepted B. introduced c. made D. constrained
60. Hypocrisy on the part of the parents may ____ _ .
A. result in their children ’s wrong behaviour
B. make their children lose faith in them
c. disqualify their teachings altogether
D. impair their childre n's mind
VI. CLOSE TEST
CLOSE TEST1: Fill in each n umbered blank with one suitable word or phrase.
Mobile phones (61) ....... .....microwave radio emissions'. Researchers are.
questioning whether exposures to these radio waves might (62)............ . to
brain cancer.
So far. the data are not conclusive. The scientific evidence does not (63)........
us to stay with certainty that mobile phones are categorically (64)................On
the other hand, current research has not yet (65)................clear adverse effects
associated with the prolonged use of mobile phones.
Num erous studies are now going (66)......... ....... in various countries. Some
of the results are contradictory but others have shown an association between
mobile phone use and cancer. (67)...............these studies are preliminary and the
issue needs further, long-term investigation.
51
(68).... ............. the scientific data are more definite, it is prudent for people to
try not to use mobile phones for long periods of time. Don’t think that hands-free
phones are any safer either. At the moment, research is in fact showing the
(69)................. and they may be just as dangerous.
It is also thought that young people (70)................... bodies are still growing
may be at particular risk.
61. A. charge B.s end c. give D. emit
62. A. bring B. lead c. cause D. produce
63. A. get B. allow c. force D. enable
64. A. risky B.s ecure c. safe D. unhealthy
65. A. demonstrated B. produced c. proved D. caused
66. A. through B. on c. about D. by
67. A. However B.While c. Additionally D. Though
68. A. When B. Provide c. Until D. As
69. A. truth B. way c. fact D. opposite
70. A. whose B. as c. with D. that
CLOZE TEST 2: Read the following passage on commuting, and mark the
letter A, B, c , or D on your answer sheet to indicate the correct answer
to each of the questions.
RENEWABLE ENERGY COMES OF AGE
The British Wind Energy Association was founded 30 years ago by a group •
of scientists. At that time, the term “ alternative energy” was used to describe the
generation of wind, water and solar power. These days, we tend to (71)________
to them as “renewable energy” and the use of this name (72)_________ a real
change in their status. These sources of energy, apa rt from being alternative,
have now become mainstream and are (73)_____________ to make a significant
contribution to the country’s energy needs in the future.
Two closely linked developments (74)___________ behind this (75)________
in status. Firstly, over the past decade or so, the price of oil and gas has been
rising (76)__________ reflecting the extent to which reserves of these fossil fuels
are becoming (77)_________ . However, price is only part of the explanation.
(78)___________ as important is the growing consensus that carbon emissions
must be curbed. The scientific evidence for climate change is now irrefutable,
and both policy makers and the (79)_________ public are finally in agreement
that doing nothing about the prospect of global warming is no longer a viable
option. Renewable energy represents one real way of (80)_________ both issues.
71. A. consider B. refer c. mention D.regard
72. A. regards B.reproduces c. reminds D. reflects
73. A. s.et B. held c. put D. stood
74. A. sit B. reside c. lie D. recline
75. A. move B. shift c. switch D. jump
52
76. A. equably B. serenely c. habitually D. steadily
77. A. depleted B. decreased c. depressed D. debased
78. A. Just B. Still c. Much D Yet
79. A. deeper B. greater c. larger D. wider
80. A. coping B. engaging c. addressing D. dealing
B. WRITTEN TEST
I. OPEN CLOZE TEST
OP EN CL OZ E TE ST 1: Read the texts below and think of the word which
best fits each space. Use only ONE WORD for each space. (20 pts)
When rainforests are cleared and (81)............ , millions of carbon dioxide are
released into the atmosphere affecting climatic conditions and threatening US all
(82)..........severe flooding, drought and drop failure. The rainforests (83)..............
at least half of the e arth ’s species. At the current rate o f devastation an (84)........
50 species worldw ide become extinct every day.
One in four purchases from our chemists is derived from the rainforests.
Scientists are (85)........ caught in a race against time to find rainforest treatments
for cancer, AIDS and heart disease before they are (86)........... ...... forever.
Tribal people in the rainforests have been shot, poisoned and infected with
diseases to which they have no resistance - to make room for logging, mining
and dams. If this destruction continues, only nine (87)................the 33 countries
currently exporting.rainforest timber will have any (88)................by the end of
the decade.
Almost everyone will have part of the rainforests in their home, as do-ityou rself stores still
supply and the construction industry still uses tropical
hardwood for doors, window (89)................and even toilet seats. Please help US
(90 ).................the tropical rainforests now, before it is too late.
OP EN CL OZ E TE ST 2
If you put a group of people who don’t know (91)......... .....other in a room
together and asked them to pair up, they will naturally gravitate towards others
of similar family (92)............... social class and upbringing. We are all looking
for somethin g familiar (93) ...,............ we may not be aware of exactly what it is.
Facial attractiveness has a big (94).............. on our choice of partners, too.
People tend to seek out and form long-lasting relationships with others of similar
level of attractiveness. Several studies have confirmed this. Researchers (95)......
a selection of wedding photos and cut them up to separate the bride and the
groom. They then asked people to rate how attractive each person’s face was.
When the researchers put the photos back into their (96)............pairs, they found
that most o f the couples had been rated at similar levels. Not only (97).................
we rate others, but each of US carries a rough estimate in our heads of how
facially attractive we might be. We realized subconciously that if we approach
53
someone w ho is significantly higher up the scale than we are, we run the (98)
of being rejected.
But (99)................. the explanation for how and why we fall in love, one thing
is clear. Nature has made the whole process as blissful and addictive as possible
(100)................... the purpose of bringing and keeping couples together,
II. WORD FORM (20 pts)
SECTION 1: Complete the following sentences with an appr opriate form of
the word in block capitals
101. For many people, Ludwig Van Beethoven is the m os t____________ figure
in the history o f western classical music. (INFLUENCE)
102. H is _______________ talent was already clearly evident. (ORDINARY)
103. His da y-to-day _______________ with people invariably turned out to be
rather turbulent. (RELATE)
104. H e___________ fell in love with a num ber o f society women. (APPEAR)
105. However, the identity of the girl who lay closest to his heart remains
to this day. (ELUDE)
106. The cargo w as ____________ for safe and efficient shipping. (CONTENT)
107. She __________ questions about whether she plans to run, saying she's
focused on her voter registration and campaign.finance initiatives. (STEP).
108. Mark, a __________com mentator, was often featu red on state television
exp lain ing gov ernment policy. (TE LEVIS ION)
109. Politicians and academics pointed to the bu ild ing's ____________ contours
as a cautionary tale of architectural overreach. (BECO ME)
110. The project was subject to the us ua l______ _____ of exploratory research.
(VICIOUS)
SECTION 2: Use the word given in capitals to form a word that fits in the space.
A live broad cast of any public event, such as a space flight or sporting
occasion, is almost (111)________ (VARIABLE) accompanied by the thoughts
of a (112)_____ (COM MENT). This may be on television, along with the
relevant pictures, alternatively on radio. The technique involved (113)_________
(DIFFEREN T) between the two media, with radio broadcasters needing to be
more explicit and (114)_________ (DESCRIBE) because of the absence of
visual information. TV commentators do not need to paint a picture of their
audience; instead, their various (115)________ (OBSERVE) should add to the
images that are already there, There will sometimes be silences and pauses in a
TV commentary, although these are becoming (116)_________ (INCREASE)
rare. Both types of commentators should try to be informative, but should avoid
sounding (117)______ (OPINION). In sports comm entaries, fairness and (118)
________ ( IMPART) to both sides is vital, but spontaneity and enthusiasm are
valued by those watching or listening. Sports comm entators usually broadcast
54
live in an essentially unscripted way, although they may refer to previously
prepared materials such as sports statistics. Because of the (119)_________
(PREDICT) nature of live events, thorough preparation in advance is vital. The
Internet has helped enormously with this aspect of the job. Anyone interested in
becoming a commentator should have excellent (120)__________ (ORGANISE)
skills, the w illingness to work irregular hours, and a strong voice.
HL ERROR IDENTIFICATION
Read the passage below which contains 10 mistakes. Identify the mistakes
and w rite the corrections in the corresponding numbered boxes.
Example: Line 1: feminine —> feminist
A feminine is a person,
LINE
usually a woman, who
1
believes that women
should be regarded as
2 equally to men. She, or he,
deplores discrimination
against women in the home,
3 place of work or anywhere,
and her
principle enemy is the male
4 chauvinist, who believes
that men are
naturally super. Tired of
5 being referred to as “ the
weaker sex”, women
are becoming more and
6 more militancy and are
winning the age-old
battle of the sexes. They are
7 sick to death of sexy jokes
which poke fun
at women. They are no
8 longer content to be
regarded as second-class
citizens in terms of
9 economic, political and
social status. They criticize
beauty contests and the use
10 of glamour female models in
advertisements
which they describe as the
11 expoit of female beauty,
since women in these
situations were represented
12 as mere sex objects. We no
longer in the
13 male- dominate societies of
the past. Let us hope,
moreover, that the
revolution stops before we
14 have a boring world in
which sex doesn't
make much difference. We
15 already have unisex
hairdressers and
16 fashions. What next?
1. 2. 3.
4. 5. 6.
8. 9.
7.
10.
IV. SENTENC E TRANSFORMATION
Rewrite the sentences with the given words or beginning in such way that
their meanings remain unchanged.(20pts)
131. Thom as was not given details of the company’s new projects. DARK
—> ................................ ..........................................■••••................ .. .. .. .. .. .. .. .. ..
132. In the area, Thailand is much better than all other countries in football.
SHOULDERS
- > .............. ...............................................•................. ........................................
55
133. I'm afraid our problems are just beginning. ICEBERG
- > ......................... ..................................................................................................
134. The final version of the plan was quite different from the initial draft.
RESEMBLANCE
—> ...................................................................... ...........................................................
'..............
13 5.1 expected the film to be good, but it wasn’t at all. LIVE
- > .................................................................................................. ..........
136. You must do something to make sure this doesn’t happen again. STEPS
= > ....................................................................... ..................
137. Even though I admire his courage, I think he is á foolish.
=> M uc h..... .......................................... ............. .................................
138. He sounds as i f he has spent all his life abroad.
=> He giv es ........ .................................................... ...... .
.......................... .
139. His recent behaviors are outrageous.
=> The w ay ........................ ..................... .......................... ..................ordinary.
140. My salary is half what I would be in the job I was offered in January.
= > lf ....................................................... ..................... ........... .......................
TRƯỜNG THPT CHUYÊN LÊ QUÝ ĐÔN - ĐÀ NÂNG
A. M U LTIP LE CHO IC E (40 points)
I. WORD CHOICE (5 points)
Choose the word that best completes each sentence. Write your answer
(A, B, c , or D) in the box provided (20 points)
1. To succeed in this job, you have to be utterly________ .
A. hot-blooded B. single-minded c. kind-hearted D. near-sighted
2. The city zoo is building a new section t o ________ their larger mammals.
A. house B. store c. shade D. capture
3. The review com mittee___ three practicing lawyers and a retired businessman.
A. consists B. comprises c. is made up
4. D on’t worry: this is nothing that ________ you.
A. matters B. entails c. concerns
D. encloses
D. complicates
5. In the event, we found your advice absolutely ______
A. unworthy B. unvaluable c. invaluable D. impecunious
6. With her excellent qualifications it’s not surprising that she
treated as an inferior at work.
A. molests B. exasperates c. resents
7. What a coincidence this is! It’s so strange that y ou ____
same hotel as US.
A. should B. must c. ought to D. can
being
D. embitters
_ be staying in the
56
8. Experts are ________ into the cause of the explosion.
A. seeking B. researching c. inquiring D. investigating
9. I f you wash that pullover in the boiling water, it will________
A. crumple B. shrink c. contract D. dwindle
10. After she had m ade several disastrous decisions, people began to ________
her judg ement.
A. disbelieve B. suspect c. wonder D. question
11. GRAMM AR AND STRUCTURES (5 points)
1. All th in gs ________ , she is the best student to represent our school.
A. considered B. involved c. taken D. dealt with
2. ________ , modelling is actually hard work.
A. Even it may seem glamorous B. Yet it may seem glamorous /
c. How ever glamorous it may seem D. Glamorous as though it is
3. As always, l a m ________ with everything you say.
. A. agree B. agreeing c. agreeable D. in agreement
4. In the end, I jus t lost my ________ and started gabbling incoherently.
A. head B. mind c. brain D. intelligence
5. The team won the championship four years ________.
A. ru nning B. passing c. following D. rotating
6. After so many years, it is great to see h im ________ his ambitions.
A. get B. realise c. possess D. deserve
7. Resign if you w ant to! I assure you, no one w ill_____ any tears when you go.
A. pour B.shed c. leak D. pour
8. I t________ to be seen whether I have made the right decision or not.
A. continues B. remains c. stands D. keeps
9. At first I thought it was a good idea. But then I began to ha ve ________ .
A. second doubts B. second thoughts
c. doubtful seconds D. thoughtful doubts
10. T he re ________ that he was guilty, yet he was convicted by a jury and sent
to prison.
A. were no proofs B. were no evidences
c. was no pro of D. wasn’t any piece of proof
III. PRE POS ITIONS AND PHRASAL VERBS (5 po ints)
1. Lack of sleep over the last few months is fin all y________ Jane.
A. catch ing up with B. getting on with c. coming over D. putting on
2. Despite all the interruptions, h e________ with his work.
A. stuck at B. held on c. hung out D. pressed on
3. When the funds fin ally________ , they had to abandon the scheme.
A. faded away B. clamped down c. petered out D. fobbed off
4 We h ad n't _ ______ there being so much traffic, and we missed the plane.
A. bargained on B. factored on c. counted with D. accounted with
57
5. We had arranged to meet at 8 o ’clock but he didn ’t ________ until half an
hour later.
A. show o ff B. stick up c. turn up D. put in
6. The dentist told me that when the effect of the anaesthetic ________ , I might
feel a little pain. But it was more than a little.
A. dropped off B. went out c. wore off D. turned out
7. The sky is already dark. It must be________ for 8 o ’clock.
A. closing on B. getting up c. getting on D. closing up
8. The plan to build a new motorway _ _ _ _ _ _ due to a lack o f money.
A. dropped off B. fell through c. dropped away D. fell away
9. Their predictions were n o t________ by subsequent events. In fact, the very
opposite occurred.
A. carried out B. borne off c. borne out D. carried o ff
10. It’s been more than ten years since we had the living room - I'm
fed up looking at tit. Let's redecorate it completely,
A. made up B. counted up c. brought of f D. done up
IV. COLL OCATIONS AND IDIOMS (S points)
1. I've had this car for 12 years, but now I’m having more and more problems
with it. Clearly it’s ________ .
A. on its hind legs B. got its back up c. got its heart set D. on its last leg
2. When the morning came, the scene o f where the bomb had fallen was one of
________ devastation.
A. great B. utter . c. entire D. extreme
3. It may be raining, but I 'm ________ enjoying myself.
A. thoroughly - B. highly . c. extremely D. desperately
4. The film is ________ based on a true story, but most o f it is fiction.
A. loosely B. casually c. faintly D. lightly
5. Calls from people trying to sell US things we don’t want are a real _________.
A. ache in the neck B. ache in the back
c. pain in the neck D. pain in the back
6. Sally has an _ _ _ _ _ command o f the Chinese language.
A. excellent B. outstanding c. utter D. intensive
7. There is still a ________ of hope that the rescuers will find survivors.
A. spray B. ray c. light D. spot
8. The bridge was collapsing and the train was plunging into the ravine. But
Superman arrived , as usual, caught the train in his arms o f steel and
carried it to safety.
A. in the nick o f time B. in the fullness of time
c. on the spur o f the moment D. in the heat o f the mom ent
9. I gave th em ________ time to make a decision.
A. spacious B. lavish c. extensive D. ample
10. When Rita was promoted to sales manager, she was ■
A. over the hill B. up in the cloud
c. over the moon . D. in the seventh moon
58
V. READIN G CO M PREH ENSI O N
PASSAGE 1 (5 points)
Exquisite patterns and surface ornamentation were an int egral part of the
aesthetics of the late Victorian era. In America, these developments were
incorporated into the themes of national expositions and artistic movements, as
cottage industries grew and productivity in the decorative arts flourished. The
last three decades of the 19th century saw a change in sensibility that resulted in
new stylistic approaches in American decorative arts, a departure from the
previous era of Rococo and Renaissance Revival excess. Shapes became more
angular, smoother and less flamboyant. The popular carvings and deep modeling
of earlier years disappeared as ornamentation became more linear and lighter in
appearance. Decoration focused on the surface with rich and ele gant patterns
adorning furniture, objects of every sort, and architectural and interior
decorations. This artistic reawakening was prompted by the effects of the
Industrial Revolution on contemporary design.
This new attitu de, with its focus on ornament and decorative, was later
referred to as the Aesthetic Movement, but it also encompassed the early Arts
and Crafts Movem ent as well. The purpose was to bring a refined sensibility and
components of “good taste” to the domestic interior. Art and good taste not only
denoted good character, but also could be used to induce proper moral conduct
and actions, thereby contributing to the betterment of society. This placed a
heavy burden on designers/decorators as well as on women as keepers of the
home. Americans drew inspiration from the writing and work of English artists.
This was a period of great eclecticism. Tastes ranged from the Modern Gothic
through the Persian, Greek and Islamic, to the Japanese, and with more than a
nod to Mother Nature. Yet, regardless of the influence, surface pattern reigned
supreme. English reformers dictated that ornament should be derived from
nature, and pattern should be flat and stylized. Forms were accentuated by
colored outlines, or often with touches of gold. The emphasis was on art and on
development of a refined sensibility. It was all a matter o f taste.
1. What is the main topic o f the passage?
A. Defining the “Aesthetic Movement”
B. Decorative arts in late 19th century America
c. English influences on American decorative arts in the late 19th century
D. The change in tastes from “Rocco and Renaissance Revival” to the
“Aesthetic Movement” in the late 19th century America
2. The word “in tegr al” in line 1 is closest in meaning to
A. essential B. additional c. important D. beautifying
3. According to the passage, during the Aesthetic Movement popular carvings
and deep modeling of earlier years _ ______ .
A. were popular B. again became popular
c. disappeared D. defined good taste
59
4. The word ‘‘ele ga nt” is c los es in me ani ng t o _________.
A. bea utiful B. orn am ental c. colorful D. ref ine d
5. According to th e pas sag e, the p urp ose of the Aesthetic Mov em ent w as to ___ .
A. ind uce p roper m oral conducts and action s
B. def ine w hat w as meant by good tas te in the dome stic interior
c. encomp ass Art s and Cra fts as well as orn am ent and decoration
D. def ine good chara cte r and con tribute to the bette rm ent o f so cie ty
6. The phr ase “new attitud e” refers t o _________.
A. includ ing th e ear ly Arts arid C raft M ovem ent as w ell
B. artistic rea wa kenin g
c . the Industrial Revolution
D. Ro coco and Renaissa nce Rev ival
7. T he wo rd “deno ted ” is clos est in meaning to •______ .
A. pro mo ted B. facilitated c. dev elo ped D. signified
8. Wh ich of the follow ing can be inferred from the pas sag e?
A. designers and decorators were m ainly responsible for starting the new attitude
B. the mov ement led to a higher standard of morality in late 19th cen tury Am erica
c. the Am erican s con sidered the Eng lish to be the arb iters of goo d taste
D. wo me n, as kee per s of the hom e, faced a heavy burden
9. Ac cording to the pas sage, which of the follow ing rem ain ed mo st important,
reg ard les s o f influenc es from oth er cou ntries?
A. surfac e pattern B. English opinions
c. good taste D. Pro per m oral cond uc t and actions
10. Which of the following is NO T mentioned as feature of the Aesthetic Movement?
A. sha pes becam e less flamb oyant
B. orn am entation b eca me ligh ter in appe arance
c. forms w ere a cce ntu ated by colored lines
D. decorat ion s focused mainly on furniture
PASSAGE 2 (5 points)
It is said tha t Geo rge Wa shington was one of the firs t to rea lize how
important the buildi ng of can als would be to the na tio n’s develop me nt. In fact,
before he becam e Preside nt, he headed the first com pan y in the Un ited Sta tes to
build a can al, which was to connect the Ohio and Potomac rivers. It was nev er
comp leted, but it sho wed the nation the fea sibility of can als. As the cou ntry
exp and ed westw ard , sett lers in wester n Ne w York, Pennsylva nia , and Ohio
needed a means to ship goo ds. Canals link ing natural wa terwa ys see me d to
sup ply an effect ive m ethod.
In 1791 engineers com miss ioned by the state of Ne w York investigated the
possibility of a canal between Albany on the Hudson River and Buffalo on Lake
Erie to link the Great Lakes area with the Atlantic seacoast. It would avoid the
mountains that served as a barrier to ca nals from the Delaware and Potomac rivers.
The first attem pt to dig the can al, to be called the Erie Canal, wa s made by
private comp anies but only a comp ara tively small portion wa s built before the
60
project was halted for lack o f funds. The cost of the project was an estimated $5
million, an enorm ous amount for those days. There was some on -aga in-offagain federal
funding, but this time the War of 1812 put an end to construction.
In 1817, DeWitt Clinton was elected Governor of New York and persuaded the
state to finance and build the canal. It was completed in 1825, costing $2 million
more than expected.
The canal rapidly lived up to its sponsors’ faith, quickly paying for itself
through tolls. It was far more economical than any other form of transportation at
the time. It permitted trade between the Great Lake region and the East coast,
robbing the Mississippi River of much of its traffic. It allowed New York to supplant
Boston, Philadelphia, and other eastern cities as the chief center of both domestic
and foreign commerce. Cities sprang up along the canal. It also contributed in a
number o f ways to the North's victory over the South in the Civil War.
An expansion of the canal was planned in 1849. Increased traffic would
undoubted ly have warranted its construction had it not been for the development
of the railroads.
1. Why does the author most likely mention George Washington in the first paragraph?
A. He was President at the time the Erie Canal was built
B. He was involved in pioneering efforts to build canals
c. He successfully opened the first canal in the United States.
D. He commissioned engineers to study the possibility of building the Erie Canal.
2. The word feasibility in paragraph 1 is closest in meaning to
A. profitability B. difficulty c. possibility D. capability
3. According to the passage, the Erie Canal connected the
A. Potomac and Ohio Rivers. B. Hudson River and Lake Erie
c. Delaware and Potomac Rivers. D. Atlantic Ocean and the Hudson River
4. The phrase on -aga in-off-ag ain in paragraph 3 could be replaced by which of
the following with the least change in meaning?
A. Intermittent B. Unsolicited c. Ineffectual D. Gradual
5. The completion of the Erie Canal was financed by
A. New York. B. private companies,
c. the federal government. D. DeWitt Clinton.
6. The actual cost o f building the Erie Canal was
A. $5 million. B. less than had been estimated,
c. $7 million. D. more than could be repaid.
7. The word tolls in paragraph 4 is closest in meaning to which of the following?
A Jobs B. Grants c. Links J D. Fees
8. Which of the following is NOT given as effect of the building of the Erie
Canal in paragraph 4?
A. It allowed the East coast to trade with the G reat Lakes area.
B. It took w ater traffic away from the Mississippi River.
61
c. It helped determine the outcome of the Civil War.
D. It established Boston and Philadelphia as the most important centers of trade.
9. What can be inferred about railroad s in 1849 from the information in the
last paragraph?
A. They were being planned but had not yet been built.
B. They were seriously underdeveloped.
c. They had begun to compete with the Erie Canal for traffic.
D. They were weakened by the expansion of the canal.
10. The word warranted in paragraph 5. is closest in meaning to
A. guaranteed. B. justified c. hastened D. prevented.
VI. GUIDED CLOZE TEST (10 pts)
CLOZE TEST 1 (5 points)
The knowledge and eloquence that people gain through travelling is usually
perceived as the best (1) _ ______ in life. It is the inquisitive human nature that
(2 )______ people to seek thrilling experiences and to set out on an exploration
trip. Those who travel frequently and to diverse places benefit from establishing
new relationships and (3 )________ a better knowledge about other cultures and
lifestyles.
However, there is a (4 )________ of truth in the assumption that people are
prone to (5) cliches and unfounded prejudices about other nations and
their characteristics. Sometimes, it is only the first-hand encounter that can help
change the ( 6 ) ________ towards the so-called ‘inferior communities'. This
direct contact with a different civilisation enables travelers to ( 7)________ their
baseless assumptions and get acquainted v»th the real concept of life in all four
corners of the globe.
(8 )________ question, travelling (9 )_______ _ friendship and makes it easier
for many individuals to acknowledge the true value of different traditions and
customs. Yet, it does not always mean enjoyment. It may also .involve coming
close with the atrocities of real existence as well as becoming aware of the
challenges and hardships that other people have to struggle with. Hence, a true
voyage is the one with a good deal of experience to (1 0)________ about, very
often combined with exposure to abhorrent sights and incredible ordeals. The
learning to be complete, thus, requires an ability to observe and analyse the
surroundings, both their glamour and brutality.
completion B. fulfilment c. conclusion D. resolution
impels B. involves c. entails D. pursues
acquiring B. educating c. learning D. exacting
speck B. grain c. scrana D. tip
persevering B. cherishing c. indulging D. persisting
prejudice B. manner c. outlook D. approach
drop B. cease c. fail D. quit
62
8. A. Apart B. Beyond
9. A. facilitates B. affords
10. A. commemorate B. reminisce
c. Unfailing
c. elicits
c. resemble
D. Beneath
D. incites
D. remind
CLOZE TEST 2 (5 points)
Stressful situations that ( 1 ) ________ almost every day in life seem to be
unavoidable. However, we can do little sometimes to avoid a misfortune or an
unpleasan t occurrence which may (2) _ _______US unexpectedly as only it can.
At such a moment, one may hit the (3 )________ , give in to the helplessness of
the situation or, ideally, put a brave face on it trying to (4) _______ the burden.
Can you (5 )________ in your mind an hour spent in a traffic jam, say, this
morn ing? Do you light one cigarette after another? Do you sound the horn every
few seconds like the other neurotics? Or do you take a different (6 )________
and make good use of the time drawing up a schedule for the days to come? To
withstand the stressful moment, you can also do a crossword puzzle, listen to
your favorite music or even compose a menu for your Sunday dinner.
In fact, whateve r way you (7 )________ to the annoying situation, you can
exert no impact on it as the traffic jam will only reduce in due (8 )________ .
Neve rtheless, your reaction might considerably influence your mood for the rest
of the day. The inability to confront a stressful occurrence like that with a deal of
composure and sensibility adds much more strain to your life and in this way
puts your well-being in ( 9 )________ . Surprisingly, it’s the seemingly negligible
hardships We stumble on daily that run double the risk of developing- serious
health disorders rather than our isolated tragedies however painful they may be.
(1 0)________ that so many of those wretched stresses inducing troubles affect
us in a day, we should, at best, try to avoid them or possibly make radical
altera tions in the way w e lead our daily lifestyles.
1. A. devise B. create c. originate D. emerge
2. A. arise B. happen c. befall D. occur
3. A. post B. ro of c. bottom D. wall
4. A. subsist B. remain c. cow D. bear
5. A. envision B. observe c. picture D. image
6. A. manner B. stance c. practice D. mode
7. A. strike B. deal c. respond D. challenge
8. A. course B. term c. timing D. period
9. A. risk B. weakness c. insecurity D. jeop ardy
10. A. Providing B. Given ■C. Hence D. As
B: WRITTEN TEST (70 points)
I. OPEN CLOZE TEST
OPEN CLOZE TEST 1 (1 0points)
Throughout our lives, right from the moment when (1 )______ _ infants we cry
to express hunger, we are engaging insocial interaction of one form or another.
Each and (2) _________time we encounter fellow human beings, some kind of
63
social interaction will take place, (3 )______ it’s getting on a bus and paying the
fare for the journey, or socializing with friends. It goes without (4 )________ ,
therefore, that we need the ability to communicate. Without some method of
transmitting intentions, we would be at a complete loss when it comes to
interacting socially.
Communication involves the exchange of information, which can be
(5 )________ from a gesture to a friend signaling boredom to the presentation of
à university thesis which may (6 )______ __ ever be read by a handful of others,
or it could be something in (7 )________ the two.
Our highly developed languages set US apart from animals. But for these
languages, we could not communicate sophisticated or abstract ideas. (8 )______
could we talk or write about people or objects (9 )_______ immediately present.
(1 0)________ we restricted to discussing objects already present, we would be
unable to make abstract generalisations about the world.
OPEN CLOZE TEST 2 (10 points)
Preserving organisms in museums is one way of retaining them (1)________
posterity, but most people agree that it would be nice to (2)________ a few of
them alive in the wild, too. At the moment, which species survive, which decline
to threatened or even status and which succumb to (3)________ is something of
a lottery. WORLDMAP is an easy-to-use software that (4)_______ geographical
patterns in diversity, rarity and conservation priorities. It can (5)________ a
range of specialist biological analysis for countless numbers of species, in order
to provide biodiversity data for research purposes. The program (6)________ the
surface area of the world into cells, usually arranged in a rectangular grid.
WORLDMAP can also (7)________ the likelihood of a hitherto unobserved
species being found in an area on the basis of its known distribution. (8)_______
the patchiness of most records, that is a useful trick. Furthermore, it can select
complementary areas for preservation. Those are not (9)____the cells with the
highest individual biodiversity, but (10)____those which, together, maximize
what is preserved by picking places with the least overlapping species.
IL WORD FORMS (20 pts)
PART 1 (10 points)
1. All the judges gave her ten out of ten for h er ______ _ (FLAW) performance.
2. A cost of $5,000 was on Monday imposed by a court in New Delhi on Chief
Minister Sheila Dikshit in a (F AM OUS)________ case lodged by her against
BJP leader Vijender Gupta.
3. He was in such a haste to return home that he left a t________(DAY) by the
first bus. (
4. His collection of plants contains ma ny________ (RARE).
5. For many people, social networking offers them a feeling o f ___________
(ESCAPE) from the real world.
6. If you can afford it, and still have lots of money left, buy it _______ (RIGHT).
64
7. We all take pride in our________ (DESTROY) friendship.
8. The ________ (ELECTRIC) of those mountainous villages is the greatest
achievem ent of this governm ent.
9. The little boy dreams of being a s ports ____ (COMMENT) for the TV station.
10. T he young girl is at a v er y________ (IMPRESS) age so we must make sure
that she goes with the right sort of person.
PART 2 (10 points )
Until comparatively recent times science and technology
performed different and separate functions, the progress of
one so often completely ( 1 )____ to the progress of the other.
(2) ________ have established thát, since the earliest
times, the improvements in our way of life have resulted from
an empirical approach, that is a process of trial and error, by
which equipment and tools are made to satisfy important
needs. It is to this approach that we owe the evolution of
technology. Our modern concept o f science, both (3 )______
and pragmatic in approach, stems from the seventeenth
century, when extensive investigations into the natural laws
governing the behavior of matter were (4) _ _______. It was
this (5) ________ style of thought which led to a sciencebased technology. Scientific
knowledge was not in itself seen
as a (6 )________ for the earlier system of trial and error, but
it did help the technical (7 )________ to see which path of
experimentation might be more (8) .. . With the
industrialization of the nineteenth century, the bond between
science and technology (9) ♦ In our own time, the
mutual (1 0 )________ of one discipline upon the other has
increased still further.
RELATE
HISTORY
PHILOSOPHY
TAKE
REVOLUTION
PLACE
INNOVATE
FRUIT
STRONG
RELY
III. ERROR IDE NTIFICATION (10 pts)
Each week Hilary Mullock, also known as Doctor Doppit, visiting London’s
General Hospital. Carrying balloons and magic tricks better than a stethoscope, she
administers her own special kind of medicine. Employed by the Theodora
Children’s Trust, Hilary brings fun and laughter for the patients in the. children’s
wards, making a hospital staying a less difficult experience for these young patients.
Having studied drama at university, Hilary later became interested in
childre n’s theatre. Seeing an advert with a clown doctor, she knew she had
founded the ideal job.. Before taking up the position, Hilary had to complete four
weeks of training, being instructed in balloon modeling and magic tricks.
According to a spokesperson for the Theodora Children’s Trust, hospitalized
children, having been excluding from their normal day-to-day routine and the
65
family environment, are likely to be frightening and homesick. Clown doctors
like Hilary has a valuable part to play in helping them forget their problems for a
while. Humour, it has been seen, has a positively impact on health. In fact,
certain chemicals produced in the body by laughter have even been shown to act
as natural painkillers. Laughter really is the best medicine, it seems.
3 .___________ __
1._______________
6 .___________ _
4 ._______________ 2. _______
9 ._______________
7.______________
10.
5 ._______________
8 ._______________
IV. S ENTENCE TRA NSF ORMATIO N (20 pts)
Rewrite the following sentences using the words given.
1. Sam was so crazy about stamps that he spent lots o f money on them every
month. (SPL ASHED)
—» So stron g___________________________________________________ .
2. The impression most people have o f Kathy is that he is an honest person.
—> Kathy com es________________________________________________ .
3. It’s impossible to predict how long it will take the business to make a great
deal of profit. (TELLIN G)
—» __________________________________________________ lucrative.
4. She cannot make her mind up about whether to stay up or sleep.
—» She was t or n_____ _________________________________________ in.
5. His argument was irrelevant to the case being discussed.
—» His argument had nothing_________________________ _______ _____ .
6. The father was over the moon when his son was bom.
-> The father was as pleased_______________________ _
birth o f his son.
7. I don’t mind if you stay at my house tonight. (PUTTING)
- » I’m not a verse _______________ ______________.._____ ■ ______
8. Seeing that Tuan was determined to get the job, I knew he is serious about his
future. (BUSIN ESS)
-> In light _ _______ __________________________________to his future.
9. If you want, you can leave. (LIBERTY )
-> You _________________________ _____ ___________________ wish.
10. The young lady was angry that no one confessed to having broken her vase.
(BRE AST)
-» T he youn g lady was b esi de________________________________ ___
up to hav ing brok en the vase.
66
TRƯỜNG THPT CHUYÊN LÊ QUÝ ĐÔN ■ KHÁNH HÔA
A. MULTIPLE CHOICE QUESTIONS (40 PTS)
I. WOR D CHOICE (5 PTS)
Choose the best options to complete the following sentences.
1. The vegetation on the island w as ______ .
A. exuberant B. chivalrous c. overcast D. ingenious
2. He became an outlaw by______ the law.
A. defying B. observing c. sticking to D. abiding by
3. The plague, otherwise known as the Black Death, was a ______ disease.
A. contagious B. contiguous c. contingent D. congenial
4. The Prime Minister will decide whether to release the prisoner or not; that’s
h is ______ .
A. prerogative B. derogatory c. abdication D. humanity
5. H e______ the illusion that he will live to be a hundred.
A. grows B. relishes c. develops D. cherishes
6. They continued fighting despite all the______ they met with.
A. adversities B. amenities c. properties D. liabilities
7. I’d love to live in th es e______ surroundings.
A. desolate B. bashful c. gloomy D. serene
8. The Secretary of State handled the m atter______ and prevented a war.
A. adroitly B. intensely C. abjectly D. slightly
9. Her c ondition seems to be______ . W e’ll have to take her to intensive care.
A. ameliorating B. deteriorating c. amputating D. imitating
10. No t only is little Jonny’s grammar incoherent and his spelling atrocious but
also his pronun ciation ______ .
A. slothful B. sluggish c. hazard D. haphazard
IL STRUCTURES AND GRAMMAR (5 PTS)
Choose th e best options to complete the following sentences.
1. • so incredible is that these insects successfully migrate to places they
have never seen.
A. That makes the monarch butterflies’ migration
B. The migration of the monarch butterflies is
C. W hat makes the monarch butterflies’ migration
D. The migration of the monarch butterflies, which is
2. It is mandatory th at smoking in public ______ .
A. prohibited B. prohibit C. be prohibited D. is prohibited
3. with about fifteen times its weight in air does gasoline allow the
carb uretor to run smoothly.
A. It is mixed B. Only when mixed C. When mixed D. To mix it
67
4. H e______ safety goggles, but he wasn’t and, as a result, the hot steel badly
damaged his eyes.
A. could have been wearing B. must have been wearing
c. should wearing D. ought to have been wearing
5. If the M oo n______ one orbit around the Earth and one complete revolution
on its axis at the same rate, we som etim es_____ _ the other side o f it.
A. won't complete - will see B. didn’t complete - would see
c. doesn 't complete - had seen D. wouldn’t complete - see
6. The two boys were ca ug ht______ the exam papers from the teachers' room,
so they definitely deser ved ______ from school.
A. to be stealing - having suspended B. to steal - being suspended
c. stealing - to be suspended D. having stolen - suspended
7. No whale has ever been known to attack human except in de fenc e;______L
stories have come down since the Biblical Jonah of men being swallowed by
whales.
A. consequently B. whatever c. inasmuch as D. nonetheless
8. After the First World War, the author Anais Nin became interested in the art
movement known as Surrealism and in psychoanalysis, both ____ _ her
novels and short stories.
A. in which the influence B. of which influenced
c. to have influence D. its influence in
9. Nebraska has floods in some y ears,______ .
A. in others drought B. droughts are others
c. while other droughts D. others in drought
10. _ ____ native to Europe, the daisy has now spread throughout most o f North
America.
A. Although B. If it were c. In spite o f D. That it is
III. PREPOSITIONS AND PHRASAL VERBS (5 PTS)
Choose the best options to complete the following sentences.
1. I need t o ______ your offer very carefully before I make a decision.
A. look over B. see out c. figure out D. mull over
2. There’s nothing t o ____ _ as it’s a general knowledge quiz.
A. come round to B. face up to c. swot up on D. come up with
3. The account of their journey has be en ______ together from personal letters
and diaries.
A. pieced B. set c. pulled D. got
4. Perhaps it is the almost universal use of flavourings that makes it so hard to
______ the p rod uct s______ .
A. tak e- ap ar t B. te ll -a par t c. co m e- ap ar t D. fa ll -a par t
5. We fin ally______ out a way to get the band into the hotel without the press
knowing.
A. tried B. set c. puzzled D. put
68
6. He has taken some painkillers but when the effects______ , his leg will hurt
quite badly.
A. wear away B. wear down c. wear off D. wea r out
7. Follow ing years of intense training, the accomplished athlete ______ the
medal triumphantly.
A. took o ff B. went off c. ran off D. carried o ff
8. Could you lend me some money to ' me over to the end of the month?
A. hand B. tide c. get D. make
9. My parents had a lot of children, so sometimes there wasn’t enough food to _____.
A. put on . B. fall back on c. give out D. go round
10. The noise from the unruly fans celebrating their team ’s victory didn’t _____
until early in the morning.
A. shut o ff B. give away c. let up D. fall over
IV. COLLOCATIONS AND IDIOMS (5 PTS)
Choose the best options to complete the following sentences.
1. How did they manage to keep me com pletely______ about this for so long?
A. in the dark B. under the shadow c. in the shade D. out o f shape
2. You’ve got t o ______ to succeed in advertising.
A. go to your head B. have your wits about you
C. have your head in the clouds D. gather your wits
3. It took Dane a long time to understand what was going on. He’s usually
than that, isn’t it?
A. round the bend B. harder and faster
c. easie r on the ear D. quicker on the uptake
4. Stop w asting time spl itting______ and look at the big picture.
A. hairs B. ears c. eyes D. noses
5. The country is an economic__with chronic unemployment and rampant crime.
A. lost cause B. basket case c. false dawn D. dark horse
6. My m othe r nearly ______ when I said I was going to buy a motorbike.
A. let the cat out o f the bag B. put the cat among the pigeons
C. had kittens D. got out of the rat race
7. Fresh evidence has re ce nt ly _____ _ which suggests that he didn ’t in fact
commit the murder.
A. come a long way B. come under fire
c. come into bloom D. come to light
8. We ’d been working hard for a month and so decided to go out a nd ______ .
A paint the town red B. face the music
c. read between the lines D. steal the show
9 The boss was ______ with the new employee over his qualifications and
ability to perform his job well.
A. at a loose end B. at a loss c. at a standstill D. at odds
10. I’m so ______ under with work at the moment - it’s awful!
A. iced B. rained c. fogged D. snowed
69
V. READING COM PREHENSION
Read the passages carefully and choose the best answer to each question.
READING PASSAGE 1 (5 PTS)
While most desert animals will drink water if confronted with Zz, for many of
them the opportunity never comes. Yet all living things must have water, or they
will expire. The herbivores find it in desert plants. The carnivores slave their
thirst with the flesh and blood of living prey. One of the most remarkable
adjustments, however, has been made by the tiny kangaroo rat, who not only
lives without drinking but subsists on a diet of dry seeds containin g about 5%
free water. Like other animals, he has the ability to manufacture water in his
body by a metabolic conversion of carbohydrates. But he is notable for the
parsimony with which he conserves his small supply by every possible means,
expending only minuscule amounts in his excreta and through evaporation from
his r espiratory tract.
Investigation into how the kangaroo rat can live without drinking water has
involved various experiments with these small animals. Could kangaroo rats
somehow store water in their bodies and slowly utilize these resources in the
long p eriods when no free water is available from dew or rain ? Th e simplest way
to settle this question was to determine the total water content in the animals to
see if it d ecreases as they are kept for long periods on a dry diet. If th ey slowly
use up their water, the body should become increasingly dehydrated, and if they
begin with a store of water, this should be evident from an initial high water
content. Results of such experiments with kangaroo rats on dry diets for more
than 7 w eeks showed that the rats maintained their body weight. There was no
trend toward a decrease in water content during the long period of water
deprivation. When the kangaroo rats were given free access to water, they did
not drink water. They did nibble on small pieces of watermelon, but this did not
change appreciably the water content in th eir bodies, which remained at 66.3%
to 67.2% during this period.
This is very close to the water content of dry-led animals (66.5%), and the
availability of free water, therefore, did not lead to any ‘storage’ that could be
meaningful as a water reserve. This makes it reasonable to conclud e that
physiological storage of water is not a factor in the kangaroo rat’s ability to live on
dry food.
1. What is the topic of this passage ?
A. K angaroo rats. B. Water in the desert,
c. Desert life. D. Physiological experiments.
2. The word 'expire' in the first paragraph is closest in meaning t o ______ .
A. becom e ill B. die c. shrink D. dehydrate
3. Which of the following is NOT a source of water for the desert animals?
A. Desert plants.
B. M etabolic conversion of carbohydrates in the body.
70
c. The blood o f other animals.
D. Streams.
4. The word ‘z7’ in the first paragraph refers t o ______.
A. a living thing B. the desert c. the opportunity D. water
5. The author states that the kangaroo rat is known for all of the following
EXCEPT___
A. the economy with which it uses available water
B. living without drinking water
c. breathing slowly and infrequently
D. manufacturing water internally
6. The word 'parsimony'’ in the first paragraph is closest in meaning t o ______ .
A. intelligence B. desire c. frugality D. skill
7. It is implied by the author that desert animals can exist with little or no water
because of______ .
A. - less need for water than other animals
B. m any opportunities for them to find water
c. their ability to eat plants
D. their ability to adjust to the desert environment
8. The word 'deprivation' in the second paragraph is closest in meaning t o ____.
A. preservation B. renewal c. examination D. withholding
9. According to the passage, the results of the experiments with kangaroo rats
showed that ______ .
A. kangaroo rats store water for use during dry periods
B. kangaroo rats took advantage of free access to water
c. there was no significant change in body weight due to lack of water or
accessibility to water
D. a dry diet seems detrimental to the kangaroo rat’s health
10. The word 'access' in the second paragraph is closest in meaning to ______.
A. right B. method c. opportunity D. entrance
READING PASSAGE 2 (5 PTS)
The end of the 19th century and the early years of the 20th century were
marked by the development of an international Art Nouveau style, characterized
by sinuous lines, floral and vegetable motifs, and soft evanescent coloration. The
Art Nouveau style was an eclectic one, bringing together elements of Japanese
art, motifs of ancient cultures, and natural forms. The glass objects of this style
were elegant in outline, although often deliberated distorted, with pale or
iridescent surfaces. A favored device of the style was to imitate the iridescent
surface seen on ancient glass that had been buried. Much of the Art Nouveau
glass produced during the years of its greatest popularity had been generically
termed ‘art g lass’. Art glass was intended for decorative purposes and relied folks effect on
carefully chosen color combinations and innovative technique’s.
71
Trance produced a number of outstanding exponents of the Art Nouveau
style; among the most celebrated was Emile Galle (1846-1904). In the United
States, Louis Comfort Tiffany (1843-1933) was the most noted exponent of this
style, producing a great variety of glass forms and surfaces, which were widely
copied in their time and are highly prized today. Tiffany was a brilliant designer,
successfully combining ancient Egyptian, Japanese, and Persian motifs.
The Art Nouveau style was a major force in the decorative arts from 1895 to
1915, although its influence continued throughout the mid-1920s. It was
eventually to be overtaken by a new school of thought known as Functionalism
that had been present since the beginning of the 20th century. At first restricted
to a small avant-garde group of architects and designers, Functionalism emerged
as the dominant influence upon designers after the First World War. The basic
tenet of the movement that function should determine fo rm - was not a new
concept. Soon a distinct aesthetic code evolved: form should be simple, surfaces
plain, and any ornament should be based on geometric relationships. This new
design concept, coupled with the sharp postwar reactions to the styles and
conventions of the preceding decades, created an entirely new public taste which
caused A ll Nouveau types of glass to fall out of favor. The new taste demanded
dramatic effects of contrasts, stark outline and complex textural surfaces.
1. Paragraph 1 of the passage mainly discusses______ .
A. design elements in the Art Nouveau style
B. the popularity o f the Art Nouveau style
c. production techniques for art glass
D. color combinations typical of the Art Nouveau style
2. The word “one ” in paragraph 1 refers t o ______ .
A. century B. development c. style D. coloration
3. Paragraph 1 mentions that Art Nouveau glass was sometimes similar to
______ of ancient buried glass.
A. the distortion o f the glass B. the appearance of the glass surface
- c. the shapes of the glass objects D. the size of the glass objects
4. The main purpose of paragraph 2 is to ______ .
A. compare different Art Nouveau styles
B. give examples of famous Art Nouveau artists
c. explain why Art Nouveau glass was so popular in the United States
D. show the impact Art Nouveau had on other cultures around the world
5. The word “pr ize d” in paragraph 2 is closest in meaning t o _____ _.
A. valuéd B. universal c. uncommon D. preserved
6. The word “ov ertaken” in paragraph 3 is closest in meaning to ______ .
A. surpassed B. inclined c. expressed D. applied
7. By stating that “funct ion shou ld deter mine for m” the author means th at ____,
A. a useful object should not be attractive
B. the purpose o f an object should influence its form
72
c. the design of an object is considered more significant than its function
D. the form o f an object should not include decorative elements
8. It can be inferred from the passage that one reason functionalism became
popular was that it ______ .
A. clearly distinguished between art and design
B. appealed to people who like complex painted designs
c. reflected a common desire to break from the past
D. was easily interpreted by the general public
9. Paragraph 3 supports the idea th at______ .
A. functionalism's design concept avoided geometric shapes
B. functionalism started on a small scale and then spread gradually
c. Functionalism was a major force in the decorative arts before the First
World War
D. Functionalism was not attractive to architects and designers
10. According to the passage, an object made in the Art Nouveau style would
most likely inc lud e______ .
A. a flowered design B. bright colors
c. modem symbols D. a textured surface
VI. GUIDED CLOZE TEST
Read the following passages and ch oose the options that best complete the blanks.
GUIDED CLOZE TEST I (5 PTS)
THE TRADE IN RHINO HORN
Last year thieves broke into a Scottish castle and stole only one thing:
a rhino horn, which is at 1.5 metres, was the longest in the world. In China,
pharmaceutical factories have been building up (1 )______ of antiques made
from rhino horn, for the sole purpose of smashing them to powder to make the
(2) ______ ingredient of many of their medicines. And in Africa poachers
continue to die in the ( 3)______ for the black rhino.
Recently, conservationists have met to (4 )______ a campaign to persuade
countries where rhino horn is still a part of the traditional medicine to switch to
substitutes. The biggest (5 )______ to the survival o f the rhinoceros is the refusal
of certain countries to enforce a ban on domestic (6 )______ in rhino horn.
The rhino horn is included in many aids for disorders ranging from fevers to
nosebleeds. Horn, like fingernails, is made of keratin and has no proven
medicinal (7) ______ . Traditional substitutes, such as horn from buffalo or
antelope, are regarded as second best.
The battle is (8) _ ____ to be winnable. But it may be harder than the battle
against the trade in ivory, for there is a (9 )_____ between the two commodities.
Ivory is a luxury, while rhino horn, people believe could (1 0)______ the life o f
their child.
73
1. A. bundles B. collections c. amounts D. groups
2. A. real B. actual c. essential D. true
3. A. tight B. chance c . search D. race
4. A. design B. plan c. programme D. form
5. A. threat B. danger c. disaster D. menace
6. A. business B. commerce c. selling D. trading
7. A. capacity B. property c. control D. powers
8. A. thought B. dreamed c. imagined D. viewed
9. A. variation B. comparison c . gap D. difference
10. A. make B.sav e c. help D.s urv ive
GUIDED CLOZE TEST 2 (5 PTS)
Read the following passage and decide which option A, B, c or D best fits
each space.
NOISE POLLUTION
Noise is more than a mere nuisance. At certain levels and durations of
exposure, it can cause physical damage to the eardrum, and (1 )___ in temporary
or permanent hearing loss. In addition to causing hearing loss, (2 )______ noise
exposure can also (3 )______ blood pressure and pulse rates, cause irritability,
anxiety, and mental fatigue, and interfere with sleep, recreation, and personal
communication. Noise pollution control is, therefore, (4 )______ importance in
the workplace and in the community.
Noise effects can be (5 )______ by a number of techniques, for example,
increasing the distance or blocking the path between the noise source and the
recipient, reducing noise levels at the source, and ( 6 ) ______ recipients with
earplugs or earmuffs. Increasing path distance is very effective because, as a
sound wave spreads outward from the source, the fixed (7 )'_ __ __ __ _ of energy
in the wave is dissipated over an ever-expanding wave front. Path barriers
(8 )______ walls, ceilings, and floors can be effective by absorbing as well as
reflecting sound energy. Special earmuffs are (9 )______ to protect industrial
and construction workers. The best way ( 10 )_____ _ noise pollution is to reduce
the sound levels at the source, for instance, by improving design, muffling
machinery and engines, and properly maintaining and lubricating machinery to
reduce vibrations.
1. A. bring B. result c. create D. affect
2. A. exceedingly B. excess c. excessively D. excessive
3. A. enlarge B. maximize c. raise D. rise
4. A. for B. of c . at D. within
5 .A. done B. created c. brought D. minimized
6. A. protecting B. protect c. protected D. protects
7. A. kind B. type c. amount D. number
8. A. as B.such as c. like D. or
9. A. available B. ready c. accessible D.enou gh
10. A. prevent B. preventing c. for preventing D. to prevent
74
B. W R IT TEN TE S T (70 PTS)
I. OPEN CLOZE TESTS
Fill in each of the numbered blanks with ONE suitable word.
OPEN CLOZE TEST 1 (10 PTS)
ELDEST CHILD
1 he first-born is an only child until the second child comes (1 )______ - when
they go from (2) • the centre of attention to sharing the care (3 )______
parents. This 'deth ronement' may be extremely traumatic and forever shape the
first-born's outlook (4 )_____ life. They may spend the rest o f their lives striving
(5) _______regain their parents' approval. They could even unconsciously feel
their parents had (6 )_______ child (7 )_________somehow they weren’t good
(8) _______ (9 )______ may lead to feelings of inadequacy and also contributes
to resentment of subsequent siblings. The first-born is often the least warm (10)
______ frequently the most hostile to their brothers and sisters.
(adapted from “Does your place in the family mould you forever? ” in Daily Mail)
OPEN CLOZE TEST 2 (10 PTS)
SOUTH AFRICA PLAGUED BY HIV VIRUS
Nearly one-in-four adults of working age in South Africa is believed to be
infected with (1 )______ HIV / Aids virus. (2)______ is a shocking statistic, and
one (3 )______ potentially catastrophic economic and social consequences for
the country. The limp response from the South African government has already
been condemned, and it has been left to the country’s financial community to
tackle what (4 )______ become an epidemic.
At the forefront of a campaign to recognise, report and raise awareness about
(5 )_____ disease (6 )____ the South African Institute o f Chartered Accountants
(Saica). It is drafting guidelines on Aids (7) _____ _ be presented to the
Johannesburg stock exchange. These might (8 )______day be incorporated into its
listings requirements. Thingle Pather, a chartered accountant and project director
at Saica, heads the HIV working group that is putting together the first draft.
Pather is working with the Global Reporting Initiative, an organisation
(9) ______ issues guidelines on non-financial reporting, and the South African
Actuarial Society to put together a document that will push (10) _________
voluntary disclosure of information about the prevalence levels of HIV I Aids
and the estimated financial impact.
(adapted fro m “Businesses count the cost o f Aids epidemic ” in The Guardian Weekly)
II. WORD FORMS
PART 1. Supply the correct word form of the word in parentheses. (10PTS)
1. He gave a __ _________ _______ shrug. He is always making his own
achievements seem unimportant. (DEPRECATE)
2. It’s a __ ________article which is fair to both sides o f the dispute. (ROUND)
75
3. My friends started going out late to nightclubs so I decided to ____________
myself from tile group. (SOCIAL)
.4. New immigrants have been successfully ____________________ into its
community. (SIMILAR)
5. She felt her husband con stantly ____________ her achievements. (LITTLE)
6. Steroids often help reduce th e__________ and itching in the skin. (FLAME)
7. The amounts o f radioactivity present w ere_____________ small. (FINITE)
8. The museum's collection includes ___________________ dating back to
prehistoric times. (FACT)
9. There is a strong smell o f___________________ in the hospital. (INFECT)
10. This docum ent needs ________________________ before sending to the
publishing house. (PROOF)
PART 2. Supply each gap with the correct form of the word given in the box. (lOpts)
-11 im cattle emit firm
live voice pose praise

In January 2001 the (1 )_______________ Panel on Climate Change (IPCC)


issued its latest report on climate change. Climate models worked out by giant
super-computers had become far more reliable since the previous report in 1995
and allowed them to ( 2 )___________ the earlier (3) .________ for global
warming. Their conclusions were that something very serious is happening and
that it cannot be a natural process. The 1990s was the hottest decade for 1.000
years and the Earth is warming faster than at any tim e in the last 10.000 years’
According to the report, human activities are (4 )______________ to blame for
the temperature rise. The burning of fossil fuels releases carbon dioxide and, due
to deforestation, there are fewer trees to absorb this gas and recycle it back into
oxygen. Methane concentrations have also gone up dramatically because of
increases in rice culture and (5) ___________both of which generate
methane from (6 )_______________ vegetation. These greenhouses gases trap
heat in the E arth’s atmosphere and cause the temperature to rise.
The IPCC reported that, in the worst case, the average temperature could rise
by 5.8°c this century, 2°c higher than their original predictions. The resulting
melting of ice-caps and glaciers would cause sea levels to rise by up to 88 cm,
endangering the homes and (7) _________________ of tens of millions of
people who live in low-lying regions.
Unfortunately, there is far greater (8 )_______________ _ among the world’s
scientists over the issue than among politicians. As long ago as 1990, the ỈPCC
recommended a 60% reduction in carbon dioxide (9 )_______________ , as the
basic level required to return the planet's climate to a healthy level.
Now that Governments globally failed to enact these proposals. Now that the
dangers have been (1 0) _________________ by the latest report, it is high time
that governments took an active interest in exploring alternative, renewable
energy sources.
76
III. ERROR IDENTIFICATION
Read the following passage. There are 10 errors. Identify the errors and
then correct them. (10 PTS)
OAK '
(1)
(5)
(10)
(15)
(20)
4.
7.
Oak wood has a density of about 0.75 g/cm3, great strong and hardness,
and is very resistant to insect and fungal attack because of its high tannin
content. It also has very appeal grain markings, particularly when quartersawn. Oak planking
was common on high status Viking long ships in the
9' 1and 10th centuries. The wood was hewn from green logs, with axe and
wedge, to produce radial planks, similar to quarter-sawn timber. Wide,
quarter-sawn boards of oak have been prized since the Middle Ages for use
in interior paneling off prestigious buildings such as the debating chamber
of the House of Commons in London, and in the construction of fine
furniture. Oak wood, from Quercus robur and Quercus petraea, was used in
Europe for the construction of ships, especial naval men of war, until the
19lh century, and was the principal timber used in the construction of
European timber-framed build.
Today oak wood is still commonly used for furniture making and
flooring, timber frame buildings, and for veneer production. Barrels in
which wines, sherry, and spirits such as brandy, Scotch whisky and
Bourbon whiskey are age are made from European and American oak.
The use of oak in wine can add many different dimensions to wine based
on the type and style of the oak. Oak barrels, which may be charred
before use, contribute to the colour, taste, and aroma of the contents,
imparting a desirable oaky vanillin flavour to these drinks. The great
dilemma for wine product is to choose between French and American
oakwoods. French oaks give the wine greater refinement and are chosen
for the best wines since they increase the price compared to those aged in
American oak wood. American oak contributes greater texture and resist
to ageing, but produces more violent wine bouquets. Oak wood chips are
used for smoking fish, meat, cheeses and another foods.
3 ._______________
____________ 2 ._______________
6 .___________ __
____________ 5 ._______ ;_______
9 ._______________
_ __________ 8 ._______ ________
1 0 ._ ____________
IV. SENTENCE TRANSFORMATION
PAR T 1: Use the word(s) given in brackets and make any necessary additions to
complete a new sentence in such a way that it is as similar as possible in
meaning to the original sentence. Do NOT change the form of the given
word(s). (10 PTS)
I. Immediately after winning the race. Sandy began training for the next one. (had)
-» No so on er ......................................................................................................
77
2. Going to and fro with all the cases is what I can’t stand about holidays, (toing)
—> I t's all ....... ..........................
about holidays.
3. As a result of the bad weather, there may be delay to some international
flights, (subject)
—» Due to the bad weather ........................ ................. .............. possible delay.
4. Bill changed his ways when he came out o f prison, (leaf)
—> Bill h a s ...........................................................’................................................
5. The committee had a long discussion but they could not make up their mind,
(reach)
—> Lengthy ....................................................... ............. .....................................
PART 2: Rewrite the sentences with the given words or beginning in such a
way that they are as similar as possible in meaning to the original
sentences. (10PTS)
1. The president's bodyguards stood behind him watching.
-> Watchfully .................................................... ................................................
2. Success in the academic field depends on your ability to amass qualifications.
-> The m ore ................................................ ........................................................
3. 1 find his clothes the most irritating about him.
—> What m ost ............. .........................................................................................
4. Richard only took over the family business because his father decided to
retire early.
—> But f o r........ ....................... ................. .....................................................
5. It was n’t Melanie’s fault that she ended up breaking the law.
-» Through no ............................................. ....................... ................................
TRƯỜNG THP T CHU YÊN LÊ QUÝ ĐÔN - NINH THUẬN
I. M U LTIP LE C H O IC E (80 sentences / 40 marks)
I. WORD CHOICE (5 marks)
Choose the word or phrase that best fits each space.
1. The modification has been t he______ on the cake for both of us, for a lot o f
hard w ork has gone into the design and development o f the course.
A. chilling B. cooling c. freezing D. icing
2. The media is supposed to a lways_______ on the side of revealing rather than
concealing.
A. blunder B. err c. misc hief D. transgress
3. Toxic ___________ disfigure black neighbourhoods, degrade property
values, and discourage investment.
A. eyesores B. gatecrashers c. keystones D. watersheds
78
4. You s hou ldn't have been . to your elders by raising those matters.
A. ill-mannered B. immaterial c. impertinent D. inapposite
5. An 18-year-old girl, of no f ix ed ____________ , was charged with robbery in
connection with the incident.
A. abode B. accommodation c. dwellings D. lodgings
6. Docum entation is required before the drug can get the seal o f_______ from
world health authorities.
A. approval B. assent c. consent D. permission
7. Many Americans refused to fall in with the idea that religion is a (n )_______
disreputable anachronism.
A. academically B. cerebrally c. cognitively D. intellectually
8. Most academ ic articles bounce around from topic to topic, really without
, but are informative.
A. common or garden B. hell or high water
c. let or hindrance D. rhyme or reason
9. It is up to the court to decide, in the exercise of the ir_____ , whether to grant
the favour or not.
A. circumspection B. discretion c.judiciousness D. prudence
10. Tome said something disgusting, from which and a heated argument
, the result o f which was that they broke up.
A. ensued B. eventuated c. supervened D. transpired
11. STRUCTU RES AND GRAMMAR (5 marks)
You are to choose one word or phrase that best completes the sentence or
has the same meaning as the original sentence.
11. _______ , our original thesis that we are in fact a lot more uptight about
sexuality than we ’d like to believe is going to be adapted for psychological
consultation.
A. As it was opulently vindicated B. As was amply vindicated
c. Bountifully vindicated as it was D. Much as it was copiously vindicated
12. Let alone being left out in the cold for an hour after a second 30-year-old lift
failed and security guards said they could not allow them to use the stairs, the
staff started t o _______ in their work.
A. exasperate, such an unwarranted interference it was
B. fume, such was an unwarranted interference
c. smoulder, so unwarranted an interference it was
D. seethe, so unwarranted an interference was which
13. About six million years ago, the Mediterranean sea is s ai d______ , _______
thick layers of salt in southern France.
A. to have evaporated - depositing B. to be evaporating - deposit
c. to evaporate - depositing D. having evaporated - to deposit
79
14. It turned out that w e ______ rushed to the airport as the plane was delayed
by several hours.
A. hadn't B. should have c. mustn't have D. needn't have
15. ________ as a masterpiece, a work of art must transcend the ideals of the
period in which it was created.
A. Ranking B. To be ranked
c. Being ranked D. In order to be ranking
16. ___________ the weather forecast, it will rain heavily later this morning.
A. On account of B. According to c. Because of D. Due to
17. 1 read the contract again and ag ai n__________ avoiding making spelling
mistakes.
A. in view of B. in terms o f c. with a view to D. by m eans of
18. Minh and Cuong were badly injured in the last match, s o ___of them can
play in this match.
A. neither B. none c. either D. both
19. When I arrived on the spot, I found out that I _______ the heavy gas bottle
on my back because the campsite now had electric cookers.
A. needn't have carried B. didn't need to carry
c. can't have earned D. might not have carried
20. We are pleased to announce that Keith Danish ________ replace Susan
Williams as Operations Manager from 24th September.
A. does B. is about to c. is to D. is due
n i. PRE POSITIONS AND PHRASA L VERBS (5 m arks)
Ch oose the word that best fits each gap.
21.1 ran into an unknown in the gap-filling exercise, but I didn 't try an d______
the gap.
A. held on B. hang out c. press on D. stuck at
22.T0 prevent soldiers from rebelling, the commander splits them into groups to
_________ one another.
A. let them off with B. pick up on them
c. play them off against D. splash out on them
23. The old lady's savings were considerable as she h ad ..................a little money
each week.
A. put by B. put in c. put apart D. put down
24. The po lic e_________ a good deal o f criticism, over their handling o f the
demonstration.
A. came in for B. brought about c. went down with D. opened up
25. It was too late to '______ of the contract.
A. back away B. back down c. back out D. back up
26. Helen managed to put _____ her point of view very successfully in meetings.
A. up B. over c. on . D. across
80
27. - If* no one will_______ misbehaving, all pupils will be kept in after school.
A. adm it off B. admit in c. own up in D. own up to
28. Even if you are miserable, th ere‘s no need to____________ on me
A. take it out B. take it down c. take it from D. take it over
29. Despite all the evidence to the contrary, the w itne ss __________ that his
story was true.
A. stuck up B. stood out c. stuck out D. held out
30. Everyone approved of the scheme but when we asked for volunteers they all
hu ng _______ .
A. back B. on c. about D. up
IV. COLLOCATIONS AND IDIOMS (5 marks)
Choose the word that best fits each gap.
31. The social condemnations of the bombing, therefore, focus not o n _______
the act but on lamenting the consequences.
A. execrating B. deploring
c. shedding tears over D. wringing hands over
32. If you want to have a good flat in London, you have to pay through the
for it.
A. mouth B. eye c. nose D. head
3 3 .1 was out o f__________ with you on this matter, but let’s not quarrel about it.
A. empathy B. harmony c. keeping D. sympathy
34. Eventually, a passer-by ._____and asked her what her problem was.
A. put her out of his misery B. was charitable towards
c. took pity on her D. threw him self on her mercy
35. She expects the political experience gained in this election will stand her in
good _______ in her future career, which, she suggests, could include another
campaign. '
• A. footing B. grounding c. precedent D. stead ■
36. Poor management brought village shops to tee ter_____ of collapse although
comm unity-owned shops, Internet retailing and home delivery schemes were
becom ing more popular.
A. in the teeth B. on the brink
c. on the razor’s edge D. on the threshold
37. A lot o f criticism and scorn has been hea ped _______ his options.
A. above B. beyond c. on D. up
38. Unanswered, the demands for nuclear deterrents ha ve ____ fears of civil war.
A. flashed up B. prognosticated c. sidetracked D. stoked up
39. , the people who come to this club are in their twenties and thirties.
A. By and large B. All together c. To a degree D. Virtually
4 0 .1 don’t feel like buying a ________ in a poke; we’d better check the content.
A. pig B. cattle c. buffalo D. ox
81
V. READINGCOMPREHENSION
RE AD ING 1: Read the following magazine article and choose the best
answer to each question. (5 marks)
In the course of its history, human inventions have dramatically increased the
average amount of energy available for use per person. Primitive peoples in cold
regions burned wood and animal dung to heat their caves, cook food, arid drive
off animals by fire. The first step toward the developing of more efficient fuels
was taken when people discovered that they could use vegetable oils and animal
fats in lieu QỈ gathered or cut wood. Charcoal gave o ff a more intensive heat than
wood and was more easily obtainable than organic fats. The Greeks first began
to use coal for metal smelting in the 4th century, but it did not come extensive
use until the Industrial Revolution.
In the 1700s, at the beginning of the Industrial evolution, most energy used in
the United States and other nations undergoing industrialization was obtained
from perpetual and renewable sources, such as wood, water streams, domesticated
animal labor, and wind. These were predominantly locally available supplies. By
mid-1800s, 91 percent of all commercial energy consumed in the United States
and European countries was obtained from wood. However, at the beginning of
the 20th century, coal became a major energy source and replaced wood in
industrializing countries. Although in most regions and climate zones wood was
more readily accessible than coal, the latter represents a more concentrate source
of energy. In 1910, natural gas and oil firmly replaced coal as the main source of
fuel because they are lighter and, therefore, cheaper to transport. They burned
more cleanly than coal arid polluted less. Unlike coal, oil could be refined to
manufacture liquid fuels for vehicles; a very important consideration in the early
1900s, when the automobile arrived on the scene.
By 1984, nonrenewable fossil fuels, such as oil, coal, and natural gas,
provided over 82 percent of the commercial and industrial energy used in the
world. Small amounts of energy were derived from nuclear fission, and the
remaining 16 percent came from burning direct perpetual and renewable fuels,
such as biomass. Between 1700 and 1986, a large number of countries shifted
from the use of energy from local sources to a centralized generation of
hydropower and solar energy converted to electricity. The energy derived from
nonrenewable fossil fuels has been increasingly produced in one location and
transported to another, as is the case with most automobile fuels. In countries
with private, rather than public transportation, the age of nonrenewable fuels has
created a dependency on a finite resource that will have to be replaced.
Alternative fuel sources are numerous, and shale oil and hydrocarbons are
just two examples. The extraction of shale oil from large deposits in Asian and
European regions has proven to be labor consuming and costly. The resulting
product is sulfur- and nitrogen- rich, arid large-scale extractions are presently
prohibitive, Similarly, the extraction of hydrocarbons from tar sands in Alberta
and Utah is complex.
82
Semi-solid hydrocarbons cannot be easily separated from the sandstone and
limestone that carry them, and modern technology is not sufficiently versatile for
a large-scale removal of the material. However, both sources of fuel may
eventually be needed as petroleum prices continue to rise and limitations in fossil
fuel availability make alternative deposits more attractive.
41. What is the main topic o f the passage?
A. Applications of various fuels B. Natural resources and fossil fuels
c. A history o f energy use D. A historical overview of energy rates
42. The phrase “per per son ” in line 2 is close in meaning to
A. per capita B. per year c. per family D. per day
43. It can be inferred from the first paragraph that
A. coal mining was essential for primitive peoples
B. the Greeks used coal in industrial production
c. the development of efficient fuels was a gradual process
D. the discovery o f efficient fuels was mostly accidental
44. The phrase “in lieu” in line 5 is closest in meaning to
A. in spite B. in place - c. in every way D. in charge
45. The author o f the passage implies that in the 1700s, sources o f energy were
A. used for commercial purposes B. used in various combinations
c. not derived from mineral deposits D. not always easy to locate
46. The phrase “the latter” in line 16 refers to
A. wood B. coal c. most regions D. climate zones
47. According to the passage, what was the greatest advantage o f oil as fuel?
A. It was a concentrated source o f energy.
B. It was lighter and cheaper than coal.
c. It replaced wood and coal and reduced pollution.
D. It could be converted to automobile fuel.
48. It can be inferred from the passage that in the early 20th century, energy was
obtained primarily fr om ...............
A. fossil fuels B. nuclear fission
c. hydraulic and solar sources D. burning biomass
49. The author o f the passage implies that alternative sources o f fuel are currently
A. being used for consumption B. being explored
c. available in few locations D. examined on a large scale
50. The word “prohib itive” in line 35 is closest in meaning to
A. prohibited B. provided c. too expensive D. too expedient
R E A D IN G 2: Read the following magazine article and choose the best answer
to eac h question . (5 marks)
In seventeenth-century colonial North America, all day-to-day cooking was
done in the fireplace. Generally large, fireplaces were planned for cooking as
well as for warmth. Those in the Northeast were usually four or five feet high,
and in the South, they were often high enough for a person to walk into. A heavy
83
timber called the mantel tree was used as a lintel to support the stonework above
the fireplace opening. This timber might be scorched occasionally, but it was far
enough in front of the rising column of heat to be safe from catching fire.
Two ledges were built across from each other on the inside of the chimney.
On these rested the ends of a "lug pole" from which pots were suspended when
cooking. Wood from a freshly cut tree was used for the lug pole, so it would
resist heat, but it had to be replaced frequently because it dried out and charred,
and was thus weakened. Sometimes the pole broke and the dinner fell into the
fire. When iron became easier to obtain, it was used instead of wood for lug
poles, and later fireplaces had pivoting metal rods to hang pots from.
Beside the fireplace and built as part of it was the oven. It was made like a
small, secondary fireplace with a flue leading into the main chimney to draw out
smoke. Sometimes the door of the oven faced the room, but most ovens were
built with the opening facing into the fireplace. On baking days (usually once or
twice a week) a roaring fire of "oven wood," consisting of brown maple sticks,
was maintained in the oven until its walls were extremely hot. The embers were
later removed, bread dough was put into the oven, and the oven was sealed shut
until the bread was fully baked.
Not all baking was done in a big oven, however. Also used was an iron "bake
kettle," which looked like a stewpot on legs and which had an iron lid. This is
said to have worked well when it was placed in the fireplace, surrounded by
glowing wood embers, with more embers piled on its lid.
51. Which of the following aspects of domestic life in colonial North America
does the passage mainly discuss?
A. Methods of baking bread
B. Fireplace cooking
c. The use of iron kettles in a typical kitchen
D. The types o f wood used in preparing meals
52. The author mentions the fireplaces built in the South to illustrate
A. how the materials used were similar to the materials used in northeastern
fireplaces
B. that they served diverse functions
c. that they were usually larger than northeastern fireplaces
D. how they were safer than northeastern fireplaces
53. The word "scorched" is closest in meaning t o ............... .........
A. burned •' B. cut c. enlarged D. bent
54. The word "it” refers t o ...................
A. the stonework B. the fireplace opening
c. the mantel tree D. the rising column of heat
55. According to the passage, how was food usually cooked in a pot in the
seventeenth century?
A. By placing the pot directly into the fire
B. By putting the pot in the oven
84
c. By filling the pot with hot water
D. By hanging the pot on a pole over the fire
56. The w ord ’’obtain” is closest in meaning t o ..................... ..
A. maintain B.' reinforce c. manufacture D; acquire
57. Which of the following is mentioned in paragraph 2 as a disadvantage of
using a wooden lug pole?
A. It was made of wood not readily available.
B. It was difficult to move or rotate,
c. It occasionally broke.
D. It became too hot to touch.
58. It can be inferred from paragraph 3 that, compared to other firewood, "oven
wood” produced
A. less smoke B. more heat c. fewer embers D. lower flames
59. According to paragraph 3, all of the following were true of a colonial oven
EXCEPT:
A. It was used to heat the kitchen every day.
B. It was built as part of the main fireplace.
c. The smoke it generated went out through the main chimney.
D. It was heated with maple sticks.
60. According to the passage, which of the following was an advantage of a
* ’’bake kettle”?
A. It did not take up a lot o f space in the fireplace.
B. It did not need to be tightly closed.
c. It could be used in addition to or instead of the oven.
D. It could be used to cook several foods at one time.
VI. CLOZE TEST
CLOZE TEST 1
Read the following passage and mark the letter A, B, c , or D on your
answ er sheet to indicate the correct word for each of the blanks. (5 marks)
THE BEG INN INGS OF FLIG HT
Th e sto ry of m an's ma ste ry o f the air is a lm ost as old as ma n himself, a
pu zz le in wh ich the essential (0 ).......... wer e not fou nd until a ver y late
sta ge. Ho we ver, to (6 1) .............. this we must firs t go bac k to the tim e
wh en primitive man hunted for his food, and only birds and insects flew. We
can not know with any certainly when man first deliber ately shaped weapo ns
for throwing, but th at (6 2) ... .,..... o f co ns ciou s de sig n marke d the first
ste p on a road that lea ds from the spear and the arrow to the ae ro-plane
and the gi an t ro ck et o f the pr es en t (6 3)........... It w ould see m, in f act, tha t
thi s (6 4) ........... to throw things is one of the mo st prim itive and deepseate d of o ur ins tin
cts , (6 5) .......... in childhood and persisting into old age.
The mo re ma ture ambition to thr ow thin gs sw iftly and accurat ely , wh ich is
85
the origin o f m ost ou tdoo r gam es, pro bably has its ro ot s in th e ag es whe n
the po ss es sion o f a (6 6)................. wea po n an d th e ability to throw it w ith
force and accu racy (6 7) ............the dif fer ence between ea tin g and sta rving.
It is sig nific an t that suc h we apons we re (68)....... . and brou gh t to the ir
final form at an early stage in history. If we were restricted to the same
(6 9 )........... , it is dou btful if we could pro duc e better bow s and arr ow s than
thos e th at de stro ye d the armies o f the pas t. Th e ar ro w was the fir st tru e
we apon ca pa ble o f main tai ning dir ection over considerab le (7 0) ............. .
It was to be centu ries bef ore m an himself cou ld fly.
0 A. clues B. k eys c . respon ses D. resolu tion
61. A. value B. a pprove c . und erst and D. realize
62. A. act B. deed c. action D. e ven t
63. A. instant B. day c .h o u r D. m om ent
64. A. feeling B. urge c. enc ourag em ent D. emotio n
65. A..com ing B. arrivin g c . appearing D. growing
66. A. suitable B. fitting c . related D. chosen
67. A. involved B. meant c. to ld D. showed
68. A. invented B. imagined c. planned D. produced
69. A. matters B. substances c. materials D. sources
70. A. length B. extents c. areas D. distances

CLOZETEST2:
Read the following passage and mark the letter A, B, c , or D on your
answer sh eet to indicate the correct word for each of the blanks. (5 marks)
A LACK OF COMMUNICATION
Recent research has (0) .....................that a third of people in Britain have
not met their (71) .................... neighbours, and those who know each other
(72) ........... speak. Neighbours gossiping over garden fences and in the street
was a common (73) .. .. .. .. .. . in the 1950s, says Dr. Carl Chinn, an expert on
local communities. Now, however, longer hours spent working at the office,
together with the Internet and satellite television, are eroding neighbourhood
(74) ..................... 'Poor neighbourhoods once had strong kinship, but now
prosperity buys privacy,' said Chinn.
Professor John Locke, a social scientist at Cambridge University, has
analysed a large (75) .............. of surveys. He found that in America and Britain
the (76) ..............of time spent in social activity is decreasing. A third of people
said they never spoke to their neighbours at (77) .................. Andrew Mayer,
25, a strategy consultant, rents a large apartment in west London, with two
flat-mates, who work in e-commerce. “We have a family of teachers in upstairs
and lawyers below, but our only contact comes via letters (78) .............. to the
communal facilities or complaints that we’ve not put out our bin bags properly,”
said Mayer.
86
The (7 9 )..............of communities can have serious effects. Concerned at the
rise in burglaries and (80) ............ . of vandalism, the police have re-launched
crime prevention schemes such as Neighbourhood Watch, calling on people who
live in the same area to'k eep an eye on each-other's houses and report anything
they see which is unusual.
0. A. exhibited B. conducted c. displayed D. reveal
71. A. side-on B. next-door c. close-up D. nearside
72. A. barely B. roughly c. nearly D. virtually
73. A. outlook B. view c. vision D. sight
74. A. ties B. joins c. strings D. laces
75. A. deal B. amount c. number D. measure
76. A. deal B. amount c. number D. measure
77. A. least B. once c. all D. most
78. A. concerning B. regarding c. applying D. relating
79. A. breakout B. break though c. breakdown D. breakawa
80. A. acts B. shows c. counts D. works
B. WR ITT EN TEST (60 senten ces/7 0 marks)
I. OPEN CLOZE TEST (20 marks)
CLOZE TEST 1
Read the text below and think of the word which best fits each space. Use
only one word in each space.
THE SLOW ARRIVAL OF THE WHEEL
It is nearly impossible in our post-industrial society to conceive of a world
without wheels. From clocks to huge machinery and from cars to computer
disks, (1 ).............. ............... employs cogs, wheels or other types o f cylindrical
components that spin on an axis. (2) ............... the wheel took a relatively long
time to be invented and several civilizations reached a relatively high level of
technological sophistication (3 )....... ...... .............. it. The most likely explanation
is that n either terrain nor climate suited the wheel. Until 10,000 BC, much of the
world was (4 ).................the grip o f the last vestiges of the lee Age. (5) . ............
was not under ice sheet was covered by desert, jungle or bog - conditions
obviously unsuited for something like the wheel. Most experts agree that the
wheel evolved from the fact that Neolithic man was familiar with moving heavy
objects (6) .. .. .. .. .. .. .: .. .. .. .putting a roller, such as a tree trunk, under the load.
Such techniques were used to move the huge stone blocks to build the pyramids
around 2980 BC and probably Stonehenge, which dates back to around 2000 BC.
(7) ................technique for moving large, heavy objects was to place them on
sledges and to put the sledges on rollers. In time, it is likely that the sledge wore
grooves into the rollers with the result that ancient man had a ratio - a small turn
of the inner edge of the worn groove generated a larger turn of the outer edge of
the roller. The next ( 8 ) ................ final step in the invention of the wheel was
87
to reduce the weight of the roller by cutting away the wood between the grooves,
in (9) ..................way creating an axle with a wheel at each end. At last man
(1 0).................. better indulge his passions for travel, speed and m ovement. .
CLOZE TEST 2
Head the text below and think of the word which best fits each space. Use
only one word in each space.
ENJOY THE BENEFITS OF STRESS!
Are you looking forward to another busy week? You should be according to
some experts. They argue that the stress encountered in our daily lives is not only
good for us, but essential to survival. They say that the response to (11)________ ,
which creates a chemical called adrenal in, helps the mind and body to act quickly
(1 2)___________ emergencies. Animals and human beings use it to meet the
hostile conditions which exist on the planet.
Whilst nobody denies the pressures of everyday life, what is surprising is that
we are yet to develop successful ways of dealing with them. (13) ________ the
experts consider the current strategies to be inadequate and often dangerous. They
believe that (1 4) ______, of trying to manage our response to stress with drugs
or relaxation techniques, we must exploit it. Apparently, research shows that
people (15) ________ create conditions of stress for (16) _______ by doing
exciting and risky sports or looking for challenges, cope much better with life's
problems. Activities of this type have been shown to create a lot of em otion;
people may actually cry or feel extremely uncomfortable. But there is a point
(1 7)_________ which they realise they have succeeded and know that it was a
positive experience. This is because we learn through challenge and difficulty.
That's (1 8) ______ _ we get our wisdom. Few o f US, unfortunately, understand
this fact. For example, many people believe they (1 9)_________ from stress at
work, and take time off as a result. Yet it has been found in some companies that
by far (2 0)__________ healthiest people are those with the most responsibility.
So next time you're in a stressful situation, just remember that it will be a
positive learning experience and could also benefit your health!
II. WORD FORMS (20 marks)
A. Give the correct form o f the word in brackets.
1. The act o f __________ ' should be prohibited under any circumstances.
Everyone all has equal rights to vote. (FRANCHISE)
2. The boss ordered his staff to b e ________ with each other in dealing with the
company's end-of-year burden. (LABOR)
3. It is such a _________ story that the mom lost" her child when her family
immigrated from Africa into Europe. (LUCK)
4. The mayor was determined that he would do everything in his power to _____
the murder case. (MYSTERY)
88
5. The government stressed it was not abandoning its tough
'______ policies.
(INFLATE)
6. The woman detected h er _______ conditions soon after she.felt ill in her body.
(CA NC ER)
7 .1 cast my e ye s________ and saw the geese flying in V-line. (HEAVEN)
8. _________ program s are offered to those who want to do professional training
while being paid to work. (SER VIC E)
9. It is inhumane to se ll__________ _ limbs o f the animals, such as rhinoceros or
elephants, in the market place. (ME MB ER)
10. He was regarded as a ____________ winner as he beat his opponents in three
straight sets. (RUN)
B. Use the correct form of each of the words given in the box to fill in the
blank in the following passages.
persist confer dispose lonely
part think out condition
More than half the world’s population consid er themselves shy, delegates to
the first international (1 1)..............on shyness, being held in Cardiff, will be told
today. One in 10 cases is severe. Effects include mutism, speech problems,
(1 2)............., blushing, shaking and trembling, lack of eye contact, (1 3)..............
in forming relationships and social phobia - the most extreme form of shyness,
defined by the Americ an Psychiatric Association as a pronounced and (14).........
fear of social or perform ance situations in which embarrassment may occur. Shy
people tend to blame themselves for social failure and attribute success to
(15) .................. . factors. They expect their behaviour to be inadequate
remember only negative information about themselves and accept without
challenge advers e comments from others.
The causes are complex and not fully understood. The latest theory is that it
can be traced to genes as well as to social (1 6)....................... . One estimate,
based on research with twins, is that around 15 per cent of the population are
bom with a ( 1 7).................to shyness. Some psychologists believe there are two
types: an early developing, fearful shyness and a later developing, (1 8).............
shyness. The fearful version emerges often in the first year of life and is (19)
...................... to be (2 0)......... .............. inherited. -
III ERROR IDENTIFICATION: (10 marks)
Ther e a re 10 erro rs in the following passage. Identify and corre ct them.
Line
Children who tell pop music does not interfere with their homework receive 1
support today, with the discovery that pay attention to visual stimuli and sounds 2
requires completely different brain pathways which can operate at the same 3
89

time with your appreciation o f either being damaged. Researchers have 4


founded that listening to car stereos does not create much interference 5
when you are driving. Similarly, pop music should not interfere with 6
children homework. T he affect o f pop music on their performance at it 7
is far outweigh by other factors, such as how happy they are to be doing it. 8
These findings could be applied for the design of places which people 9
have to take in large am ounts of information very quickly. They could, 10
for example, be relevantly to the layout of pilot cockpits on aircraft. 11
1.
4.
7.
2.
5.
8.
3.
6.
9.
10.
IV. SENTENCE TRANSFO RMATION (20 marks)
Rew rite the sentence with the given word or the given beginning so that the
new sentence has the same meaning as the previous one.
1. He contributed significantly to the success of the project, (import)
—> H is ____________________________________ the success o f the project.
2. He was terribly upset by the news, (blow)
—> The news ca m e________ ______________________________ ______ him.
3. The new town has made the town centre nicer, but public transport could still
be improved a lot. (room)
—> The new town has made the town centre nicer, but__________________ _
to public transport.
4. Trying desperately to compensate for his terrible behaviour, he bought her a
bunch of flowers, (amends)
—» In a __________________ • his terrible behaviour, he bough t her
a bunch of flowers.
5. We decided to stay for longer because we were so thrilled by the plane,
(prolonging)
—> O ur__________ _________________________________ we by the plane.
6. The north west o f Britain has more rain each year than the southeast.
—> The an nu al______ ____________________________ ________________
7. They declared war on the pretext of defending their territorial rights.
—»The
8. No one stands a chance of beating Mansell in this y ear’s championship.
—> It’s a foreg one____________________ •________ J_______________ _
9. This house is very different from the little flat we used to live in. (cry)
—►This ho us e_________ ___________________________________________
10. The spectators got so angry that they had to cancel the football match.
—* S uc h_____ ______________ ____________________________
90
- - _____ « TRƯỜNG THPT CHUYÊN LÊ QUÝ ĐÔN - BÀ RỊA VŨNG TÀU
A. MULTIPLE CHOICE
I. PHONOLOGY
Choose the word whose underlined part is pronounced differently from the others
brood B. broomstick c. foolscap D. brooch
massage B. pilgrimage c. pillage D. dosage
exist B. exhaustion c. explorer D. exhibit
accredit B. salamander c. majesty D. saliva
swarthy B. w reathe c. apothecium D. feather
Ch oos e the word which is stressed differen tly from the other three
6. A. interviewer
7. A. diplomacy
8. A. authoritative
9. A. communal
10. A. telecommuting
B. geophysical c.hydroelectric D. humanitarian
II. WORD CHOICE
B. encouragementc. acknowledge D. interpreter
B. synonymous c.elephantine D. petroleum
B. argumentative c.administrative D. initiative
B. delicacy c.peninsula D. pneumonia
Ch oos e the b est options to c omplete the following sente nces
11. The children in the audience we re___ by the show.
A. brightened B. advocated c. captivated D. cultivated
12 There’s a great deal of pressure in the newspaper industry; editors might
work a 12-hour day with n o___.
A. come-down B. letdown c. let-up D. crack-down
13. The difference between your estimate and mine is just too small to mention.
It makes no sense to __about it.
A. chew the fat B. split hairs c. talk shop D. brave the elements
14. It was predicted that business would be destroyed and the town’s economy
would be i n____.
A. fragments B. scraps c. segments D. tatters
15. In friendship w e___ the embarrassment o f a dissident disclosure.
A. are at pains to avoid B. are on pain of avoidance
C. are under pain o f avoidance D. avoid for one another’s pains
16. In some countries, confrontation between police and strikers o n ___has
become a feature of life in the eighties.
B. picket c. back benches D.
A dole queues
lines assembly lines
17. Believe it or not, he has ev
en___to swindling his own friends.
A reduced B. declined c. sunk D. connived
18. Even if they threaten you, you must be firm a nd _________ _ .
A. rub your shoulders B. get off scot-free
c. stick to your guns D. turn the tables on
91
19. A: Sarah is one of the most popular fashion designers in the country, isn’t she?
B: Indeed. She is _____________________at present.
A. riding the crest of a wave B. burning the m idnight oil
c. letting off steam D. making a mountain out of a molehill
20. The depleted column was barely able t o __ the enemy attack.
A. withstand B. expel c. withhold D. smash
III. G RAM MAR AND STRUCTURES
21. The idea is ____ the publicity department during peak periods.
A. Susan to assist B. Susan assisting
c. that Susan be assisted D. that Susan assist
22. My do ctor sai d___ the medication for six months.
A. I would have to take B. I must have taken
c. that I take D. me to take
23. Nebraska has floods in some ye ars,____ ;
A. in others drought B. droughts are others
c, while other droughts D. others in drought
24. Manjypeo ple regard ____ the police have started
carrying guns.
A\ assa-»reallỳybad idea when B. it a really bad idea if
G?.ittassadreally bad idea that D. it to be a really bad idea for
25. _____■ real work was done in the office while the boss was away.
A. Not a B.No c. Not only D. None
26. Social scientists believe th at ____ from sounds such as grunts and barks
made by early ancestors o f human beings.
A. language, which was very slow to develop
B. language, very slowly developing
c. the very slow developmen t of language
D. language developed very slowly
27. ________ , only people who agree with her are real Jews.
A. We can infer how she is concerned
B. We can infer that as far as she is concerned
c. Inferred from what she is concerned
D. We can infer what she is concerned
28. _____ the circling the globe faster than Jules Verne’s fictional Phileas Fogg.
A. A pion eer journalist, N ellie Bly’s exploits included
B. Also included in the exploits o f Nellie Bly, a pioneer journalist, was
c. The exploits of Nellie Bly, a pioneer journalist, were included
D. The pioneer journalist’s exploits o f Nellie Bly included
29. O wning and living in a freestanding house is still a goal of young a du lts ,__
earlier generations.
A. as did B. as it was of c. like that o f D. so have
30. Unsalted butter is best for this recipe, bu t___ that, m argarine will do.
A. except B. given c. for all of D. failing
92
IV. PHRASAL VERBS AND PREPOSITIONS
31. He do esn 't invest in arms industry . principle.
A. on B. in c. under
32. No agreem ent was reached. The strikers decided to
Ạ. keep B. watch c. hold
33. Marrying into such a rich family had always been _
A. under B. above
34. What the treasurer said virtually _
c. over
a confession,
c. stood by
D. within
_ out for a better deal.
D. draw
his wildest dreams.
D.beyond
B. came up
with your lies and excuses
B. palm me off c. back me up
_ a magazine to kill time.
B. thumbed through
D. muscled in
D. embodied with
D. hand me down
A. amounted to
35. Don’t try t o _ _
A. head me off
36. I was bored so I _
A.eye d up
c. knuckled down
37. Ever since they made her assistant manager, she’s been
A. looking up B. getting above c. getting out
38. She k nocke d_____ a meal in ten minutes.
A. up B. off c. out
39. The firm w ill______ unless business improves.
A. go on B. go under c. take up
40. The files are locked up and 1 can’t ___them.
A. put on B. take in c. get at
herself.
D. looking out
D. on
D. take over
D. pick up
V. GUIDED CLOZE
GUIDED CLOZE 1
Read the text below and decide which answer best fits each space:
I never seem to stop. I’m not so much a workaholic as th e (l )_____ _ ’girl
who can ’t say n o’. Not only have I never learned to organize my time, I need the
deadline to pass before I get going. Then I wonder why I feel ( 2) ____! I’m very
good at whipping up false energy. Without (3).____ rest, though, I start to look
grey and then, apart from a good night’s sleep, the only thing that brings me
back to life is meditation. I find that 20 minutes’ meditation is (4 )______ to a
nig ht’s sleep and that keeps me going. .'
Somehow though, I never get around to
meditating on a daily ( 5)______ .
I’m no good in the morning. By the time I’ve
managed to get up and repair
of the night, half the day’s gone. When it
the (6)
comes to food, I’m a
hostess’s nightmare. As 1 (7 )_____ from migraine, I avoid cheese and stimulants
like coffee, red wine and spirits, which are generally (8) ‘ to trigger an
attack. I only eat rye bread, as the gluten in wheat makes me feel bloated and
ropy, I’ve ( 9)______ countless migraine ‘cures’, from the herbal remedy
feverfew to acupuncture. Acupuncture (10)______ balance the system, but
nothing stops the attacks.
93
1. A innovative B first c foremost D original
2. A apathetic B exhausted c gruelling D spent
3. A sufficient B severe c acceptable D minimal
4. A compatible B alike c equivalent D proportion
5. A way B basis c routine D manner
6. A damages B wrecks c ravages D ruins
7. A complain X B suffer c experience D ache
8. A foreseen B maintained c regarded D thought
9. A tried B experimented c searched D proved
10. A assists B improves c restores D helps

GUIDED CLOZE 2
Read the text below and decide which answer best fits each space:
In the universally ( 1 )__ Pixar movie Inside Out, there is a scene where a dad
is having trouble getting his toddler to eat broccoli, a vegetable that (2)
disgust in m ost children of that age. For the mov ie’s international release, Pixar
replaced broccoli with whatever vegetable was most suitable for a particular
region’s picky-eating children. When food blogger Linda Miller Nicholson was
(3) ___ with the challenge of raising a son who turns his (4) ___ up at
vegetables, she readily accepted it by (5) __ _ them into pasta in (6) _ _
imaginative ways.
As a molecular gastronomist known as the Lady Gaga o f Food by the Cooking
Channel, Nicholson regularly ( 7)_______ food in an original way. However, she
has (8 )______ to fame for her out-of-this-world pasta (9 )_______. They aren’t
your ordinary pastas with bland colors and dull flavors; they’re zany pasta
designs with (1 0)_____ of colors, mesmerizing patterns, homages to art pieces
like Vermeer's Girl with a Pearl Earring, and other cultural icons using all
natural ingredients. (...)
1. A. extolled B. exalted c. acclaimed D. applauded
2. A. invokes B. provokes c. excites D. incites
3. A. presented B. acquainted c. displayed D. introduced
4. A. chin B. eye c. mouth D. nose
5. A. sneaking B. stitching c. stalking D. slipping
6. A. wildly B. passionately c. indigenously D. disorderly
7. A. reaches B. undertakes c. approaches D. tackles
8. A, increased B. ascended c. risen D.surged
9. A. creations B. productions c. constructions D. formation
10. A. outpourings B. outbreaks c. bursts D. flare-ups
VI. READING COMPREHE NSION
PASSAGE 1
LOST CIVILIZATION OF PERU
Two thousand years ago a mysterious and little known civilization ruled the
northern coast of Peru. Its people were called the Moche.They built huge and
94
bizarre pyramids that still dominate the surrounding countryside; some well over
a hundred feet talk Many are so heavily eroded they look like natural hills; only
close up can you see they are made up o f millions of mud bricks.Several of the
pyramids, known as 'huacas', meaning sacred site in the local Indian dialect,
contain rich collections of murals depicting both secular and sacred scenes from
the Moche world. Others house the elaborate tombs of Moche leaders.
Out in the desert, archaeologists have also found the 2,000-year-old remains
of an extensive system o f mud brick aqueducts which enabled the Moche to tame
their desert environment. Many are still in use today, indeed there are signs that
the Moche irrigated a larger area o f land than farmers in Peru do now.
But who.were the Moche? How did they create such an apparently successful
civilisation in the middle of the desert, what kind o f a society was it, and why did it
disappear? For decades it was one of the greatest archaeological riddles in South
America. But now at last, scientists are beginning to come up with answers.
As archaeologists have excavated at Moche sites they've unearthed some of
the most fabulous pottery and jewellery ever to emerge from an ancient
civilization. The Moche were pioneers of metal working techniques like gilding
and early forms of soldering. These skills enabled them to create extraordinarily
intricate artefacts; ear studs and necklaces, nose rings and helmets, many heavily
inlaid with gold and precious stones.
But it was the pottery that gave the archaeologists their first real insight into
Moche life. The Moche left no written record but they did leave a fabulous
account of their life and times jn paintings on.pots and vessels. Many show
everyday events and objects such as people, fish, birds and other animals. Others
show scenes from what, at first sight, look like a series of battles.
But as the archaeologists studied them more closely they realised they, weren't
ordinary battles; all the soldiers were dressed alike, the same images were
repeated time and again. When the battle was won, the vanquished were ritually
sacrificed; their throats cut, the blood drained into a cup and the cup drunk by a
God-like deity. It was, the archaeologists slowly realised, a story not of war but
ritual combat followed by human sacrifice.
But what did it mean. Was it a real or mythological scene; and, above all, was
it a clue to the Moche's life and times?
The first break through came when a Canadian archaeologist called Dr Steve
Bourget, of the University o f Texas in Austin, discovered a collection o f bones at
one of the most important Moche huacas. Examining them he realised he wasn't
looking at an ordinary burial site. The bodies had been systematically
dismembered and marks on neck vertebrae indicated they had had their throats
cut. Here was physical proof that the images o f combat and sacrifice on the pots
were depicting not a mythological scene but a real one.
Many of the skeletons were deeply encased in mud which meant the burials
had to have taken place in the rain. Yet in this part o f Peru it almost never rains.
95
Bourget realised there had to be a deliberate connection between the rain and the
sacrifices. It lead him to a new insight into the Moche world. The Moche, like
most desert societies, had practiced a form of ritual designed to celebrate or
encourage rain. The sacrifices were about making an unpre dictable world more
predictable. A harsh environment had moulded a harsh civilisation with an
elaborate set of rituals designed to ensure its survival.
These discoveries answered one question - what was the iconography all
about - but still left'a central riddle. What had gone wrong; why had Moche
society finally collapsed?The next clue was to come from hundreds of miles
away in the Andes mountains. Here climate researcher Dr Lonnie Thom pson, of
Ohio State University, was gathering evidence of the region's climatic history
using ice cores drilled in glaciers.
Almost immediately Thompson and his team noticed something intriguing.
The historic records showed that over the last one hundred years, every time the
ice cores showed drought in the mountains, it corresponded to a particular kind
of wet weather on the coast, a weather system known as an El Nino. In other
words drought in the mountains meant an El Nino on the coast. If Thompson
could trace back the climate record in the mountains he'd also get a picture of
what happened on the coast. .
The result was fascinating. The climate record suggested that at around 560 to
650 AD - the time the Moche were thought to have collapsed - there had been a 30-
year drought in the mountains, followed by 30 years or so of heavy rain and snow.
If the weather on the coast was the opposite, then it suggested a 30-year El
Nino - what climatologists call a mega El Nino - starting at around 560 AD,
which was followed by a mega drought lasting another 30 years.-Such a huge
series of climatic extremes would have been enough to kill off an civilizatio n -
even a modern one. Here, at last, was a plausible theory for the disapp earance of
the Moche. But could it be proved?
Archaeologists set out to look for evidence. And it wasn't hard to find. All the
huacas are heavily eroded by rain - but scientists couldn't tell if this was recent
damage or from the time of the Moche. But then Steve Bourget found evidence
of enormous rain damage at a Moche site called Huancaco which he could date.
Here new building work had been interrupted and torn apart by torrential rain,
and artefacts found in the damaged area dated to almo st exactly the period
Thompson had predicted there would have been a mega El Nino. Thom pson's
theory seemed to be stacking up.
Then archaeologists began to find evidence of Thom pson's mega drought.
They found huge sand dunes which appeared to have drifted in and engulfed a
number of Moche settlements around 600 to 650 AD. The story all fitted •
together. The evidence suggested the Moche had been hit by a doubly w hammy:
a huge climate disaster had simply wiped them out.
96
For several years this became the accepted version o f events; the riddle of the
Moche had been solved.
- There was only one problem. In the late 1990s American archaeologist Dr
Tom Dillehay revisted some of the more obscure Moche sites and found that the
dates didn't match with the climate catastrophe explanation. Many of these
settlements were later than 650 AD. Clearly the w eather hadn't been the cause of
their demise.
He also found something else. Many of the new settlements were quite unlike
previous Moche settlements. Instead of huge huacas, the Moche had started
building fortresses. They had been at war.
But who with? Searching the site for clues, Dillehays's team were unable to
find any non-M oche military artefacts. It could only mean one thing. The Moche
had being fighting amongst themselves.
Dillehay now put together a new theory. The Moche had struggled through
the climatic disasters but had been fatally weakened. The leadership - which at
least in part claimed authority on the basis of being able to determine the weather
- had lost its authority and control over its people. Moche villages and and/or
clan groups turned on each other in a battle for scare resources like food and
land. The Moche replaced ritual battles and human sacrifices with civil war.
Gradually they fought themselves into the grave.
‘ Yet even that's not , the whole story. Today, along the coast of Peru it's
impossible to escape the legacy of this lost civilization. Their art lives on in the
work of local craftsmen. And if you travel to the highlands, the Moche tradition
of ritualised combat is preserved in the Tinku ceremonies where highland
villages conduct ceremonial battles against each other in the hope of ensuring a
good harvest. ,
Today, after 1,500 years, the Moche, and their legacy are beginning to take
their place in world history. The story o f the Moche is an epic account o f society
that thought it could control the world and what happened to it when it found it
couldn't. It's a story of human achievement and natural disaster, human sacrifice
and war.
Do the following statements agree with the information given in the
Reading Passage? Write
TRU E if the statement agrees with the Information
FALSE if the statement contradicts the information
NOT GIVEN if there is no information on this
1. Chiefs are buried in some pyramids.
2. Moche water channels have lasted to the present day.
3. Archaeologists found evidence that the Moche used money.
4. Texts in the Moche language were discovered.
5. Pottery designs had scenes of the Moche fighting foreign armies.
ANSWER
1. _______
2 ._______
3.
4. _______
5. ____ __
97
Complete the notes. Choose NO MORE THAN THREE WORD S AND/OR
A NU MBER from the Reading P assage for each answer.
6. The aim of the killings and burials was to make it more likely there would be
7. The extremely dry weather led to some Moche sites being covered by______
8. It was thought their civilization had been destroyed by changes in t h e _______
9. Dillehay found evidence that Moche society had survived beyond__________
10. The first evidence of military activity was the discovery o f_________ _
Which THREE of these reasons does Dillehay suggest contributed to the
disappearance of the Moche civilization?
A. a disastrous war with an external enemy
B. six decades of extreme weather
c . people no longer obeying their leaders
D. declining religious belie f
E. the practice of sacrificing people
PASSAGE 2
FOOTBALL’S PAST
In the early 19th century, football was very popular in the top private schools
in England. Initially, each school had its own rules and while the pupils were still
at school the fact that they played by these particular rules hardly mattered.
When, however, they left for the universities or for business in the provinces, it
became clear that if they were to continue playing football they were going to
need a universal set o f rules, acceptable to all teams.
Up until the 1850s, two teams at, say, Oxford University, would only be
playing a familiar game if every player had been to the same school. As things
turned out, a major game was often preceded by a long correspondence with
lengthy arguments about the rules. Was handling to be allowed? How many
players on each side? How long should the pitch be? How wide the goals?
Would carrying the ball be permitted? (‘Yes’, would say all the ex-pupils of
Rugby school; ‘No’, would say alm ost everyone else.) And even when the game
got under way, confusion and protests would necessitate long midfield
conferences between the two captains.
In time it became usual for the ex-Rugby students and their small but growing
company of followers from other schools to play ‘rugb y’ football on their own.
and for the others to come to some agreement over the rules of the more popular
version. Few of these early codes of rules have come down to US complete, but
snatches from them give a clear idea of the patterns of the early game, and in
particular how boring it must have been to stand in the cold and watch.
The first serious attem pts at laying down the rules of football were made at
Cambridge University in 1848 and these were adapted and tightened up twice in
the 1850s. Then in No vem ber 1862 the Cambridge Rules were revised yet again
and specified 11-a-side, an umpire from each side plus a neutral referee, goals 12
feet (3.7m) across and up to 20 feet (6.15m) high and an hour and a quarter’s
98
play only. These rules were said to have worked well; in the following year they
formed a vital part o f the rules o f the newly formed Football Association.
The formation of the Football Association was bitter and often ill-tempered.
With neith er side willing to give way, the split between rugby players and the
rest becam e too wide ever to be mended. The real disagreement was not over
running with the ball, but over ‘hacking’. Rugby players felt it was manly and
courageous to tackle an opponent by kicking him on the leg: the others did not,
and voted against it. The rugby men called them cowards and walked out.
Rugby did, however, leave behind one significant innovation- the more precise
name for its competitor. The story may not be true, but the legend is that one Charles
Wreford-Brown, who later became a notable official of the Football Association,
was asked by some friends at Oxford whether he would join them for a game of
rugby or ‘rugger’, as it was now often called. Hẹ refused, claiming that he was going
to play ‘soccer' — evidently a play on the word ‘association’.It caught on.
1. In the middle of the 19th century, it became clear that .........
A. university football was played by a special set of rules.
B. not all football teams were prepared to accept standard rules
c. each school had different rules for football.
D. provincial footballers players by old-fashioned.
2. Before the mid-19th century, the rules for important games were mainly settled
by......
A. letters exchanged before the match. 1
B. players from the same school.
c. a special meeting between the two captains.
D. all the players on the pitch.
3. We know from the text that football before 1848 wa s.........
A. very sim ilar to modern rugby.
B. more strictly regulated than rugby.
c. always played with the same width goal..
D. not very interesting for spectators.
4. The word ‘snatche s’ probably m eans .........
A. detailed copies B. players’ descriptions
c. short extracts D. careful diagrams
5. The Cambridge Rules of 1862 laid down........
A. the minimum time for a game. B. the number of players and officials,
c. the exact size of the goal D. the maximum length of the pitch.
6. Rugby players could not agree with the Football Association ov er ... .’..
A. running with the ball
c. what to call the game
7. The word ‘it’refers to .......
A. the Football Association,
c. the game at Oxford.
B. kicking other players
D. the shape o f the goal
B. the use o f ‘rugger’ for rugby.
D. the new name for football.
99
B, WRITTEN TEST
I. CLOZE TEST: Read the text and fill in each gap with ONE suitable word
OPEN CLOZE 1
Chocolate has an intriguing history that goes way back in time to the Mayas
of Central America, who first discovered the secret of the cocoa tree. The Mayas
educated the Aztecs, who in turn revealed all to the Europeans.
The Aztecs called the drink they made from crushed cocoa beans (1 )____
with vanilla and sugar ’The Food of the Gods’, a term (2 )_____ changed to ’The
Food for Lov e’ in Europe. Many do swear that chocolate is an aphrodisiac and
event recent research (3) _____ proved that chocolate does help ÚS to relax and
feel more sensual (Casanova swore by it).
Chocolate has changed a great deal (4 )_____ the days of the Aztecs. For a
start, Europeans found (5 )_____ preferred the taste if they didn ’t include chilli
pepper ( 6 )_____ its production! Spain held a monopoly on chocolate (7 )______
1615, when the daughter of Phillip 11 of Spain married King Louis XIII of
France and took the secret of chocolate with her. But (8 )_____ so, chocolate
recipes were a closely guarded secret for hundreds o f years.
Although in great demand, the quality of chocolate at that time was very
changeable. It all changed in the early nineteenth century, (9)____ , when the first
Swiss chocolate factory was founded near Vevey, (10) __ the shadow of
beautiful Lake Geneva. The founder and the father of Swiss chocolate was
Fracois-Louis Cailler. You can still buy C ailler chocolate today; in fact, most of
the original chocolate ’names’ are still in existence centuries on.
OPEN CLOZE 2
THE WORLD OF COSMOPOLITAN
One magazine, (1 )____ than any other, symbolizes the global dominance of
Western youth culture and fashion. Cosmopolitan, the lifestyle and fashion guide
for women in the 18-34 age group, ( 2 ) ____now published in 28 languages.
From Latvia to India, women (3 )____to Cosmo for advice and a taste of adult
content in a lighthearted, upbeat style. Each local (4) ____follows the same
formula, which is strictly laid down in a secret 50-page instruction manual. The
cover, for example, of every issue must depict a woman who (5 )____ the spirit
of Cosmo, which is summed up in its slogan: fun and fearless female. But each
local editor ensures that Cosmo reflects local tastes and is respectful (6).____
local culture. Photographs are mostly of local celebrities who are easily
recognised, and relationship advice is geared to the values of the local
population. In China, for example, any mention of sex would be unacceptable,
(7) ____the maga zine still offers beauty advice amongst articles advoc ating
patriotism and good citizenship. With an international (8 )____ of 8.2 million,
Cosmopolitan can be said to (9 )____ succeeded in catering for the needs and
interests of women worldwide. But to others, Cosmopolitan is just another
American produ ct su ccessfully marketed to the (1 0) ____of the world.
100
IL WORD FORMATION
PAR T 1: Complete each sentence, using the correct form of the word in parentheses
1. For the first nig ht's performance th e_______________ had to be called in to
take the part because the leading lady was ill, (STUDENT)
2. She survived the crash with minor injuries, but the car was a _____ . (WRITE)
3. Some •____________ members left to form a new party. (AFFECT)
4. They are on a (n )_________________ mission to gather all data, surveys and
spec ialist reports to oppose the no smoking ban in Ireland. (FACT)
5. Paying children too much attention when they misbehave can b e__________ .
(DEFEAT)
to achieve but having an opposite effect
6. Prices may rise and consumption may fall as individuals refrain from
purchasin g new cars, computers and o ther_________ _______ . (ESSENCE)
7. The only slig ht ______________ in the painting is a scratch in the corner.
(PERFECT)
8. His life offered me no hope o f_____________ . (GOOD)
9. Catholic leaders have not___________ _ themselves from the am bivalent
statement emanating from last week’s funeral. (SOCIAL)
10. Native speakers of all written languages use context t o _____________
homophone pairs that have a single written form. (AMBIGUOUS)
PART 2: Complete the following passage using the words given in the box
below. You may change the form o f the word i f necessary.
decimation information weaker pic k
governmental capital lobby legislate
“ After decad es of cutting its own staff and research arms, much of Capitol
Hill’s (1 )______ memory and policy expertise now resides in the (2) __ ____
industry. ” This is the most important sentence in the article. Our penny-wise,
pound-foolish country has (3 )______ its expertise in the sectors that are the
obje ct of its (4 )______ , so now it relies on lobbyists to tell US how things work.
Unlike government experts who receive decent salaries to keep their congressmen
(5) ______ so that they can negotiate with lobbyists with knowledge, these
lobbyists use their monopoly on knowledge to manipulate the system, now
regularly writing the laws, only to get a rubber stamp from Congress. According
to our Constitution, demo cracy resides in our government. All this (6 )______
prop aganda, from the right and the Democratic centrists, (7) _ ____our republic.
There are many things markets do well. There are many things governments do
well. The only things the Constitution says about markets or (8 )______ is that
states should regulate their trade and that the federal government regulates
(9) ______ and international trade. Our republic is suppos ed to promote the
general welfare, not easy (1 0) ______ for global corporations.
101
III. ERROR CORRECTIO N.
The following passage contains 10 errors. Identify and correct them
LINE
1 People in Britain today still pay a great deal of attentions to the notion of
class. According to recent research by linguists, British people attach
much significance to accent and choice of words than anything else, even
wealth, when assessing oth er people social status. However, a new style of
5 English pronunciation has been adopted by people from all levels o f society
may soon make it impossible to judge somebody according to their speech.
The new standard English is known as Estuary English (EE) although it is
originated in the area round the River Thames estuary, but is now typical
throughout the south-east. A way of speaking is very popular with the young
10 in particular, who are keen to disguise their social origins.
The most character feature of EE is a tendency to weaken consonants,
particularly / and t sounds, so the word what is heard as wha and will
sounds something like wiw. Some vowels are voiced clearly so that the
wordsfull, fa ll and/òơ/ sound virtually the same, which can cause ambiguity.
75 EE speech is a controversial subject in England - some people welcom e it
as a sign that Britain is moving towards a free-class society, but others
consider it a lazy, ignorant way of speaking, and schools have told to
encourage their pupils to speak more correctly.
4.
7.
2.
5.
8.
3._
6. _
9. _
10.
IV. Rewrite the following sentences, using the words given
1. When confronted with his crime the accused was unrepentant. (REMORSE)
-> The _________________ .________________ crime he had committed.
2. My brother gets a terrible rash every time he eats seafood. (BRINGS)
—> E ating____________________________a terrible crash.
3. Journalists are expected to work until late at night. (PUT)
—> Journalists are expect ed_____________________ .
4. He had no idea what was going to happen to him when he walked into that
room. (STORE)
—> Little___________ ________________________________ him when he
walked into that room.
5 .1 was determined to take advantage of the experience. (MISSED)
-» 1 wo uld________________________such an experience for all the world.
6. Someone must have seen the thieves escaping with the jew els. (MAKING)
—» The thieves .____________________ _ _________with the jewels.
102
7. In order to discover how the disagreement had started, Mary talked to each
child separately. (ONE)
-> Mary talked to the children ______________________________ attempt to
discover how the disagreement had started.
8. Minnie meant well so you mu stn't be offended by her comm ents. (AMISS)
—> Please____________________________ because she meant well.
9. There are so many different styles of ethnic cuisine to choose from these days.
(SPOILT)
—> These days, w e___________________ when it comes to ethnic cuisine.
10. Ray agonized over whether he should tell his host that the chicken was
underdone. (QUANDARY)
—> Discovering that his chicken was unde rdone________________________
should he tell his host or not.
TRƯỜNG THPT CHUYẾN NGUYEN DU - ĐẮK LẮK
I. WORD CHOICE (5 pts)
Choose the best options to complete the following sentences.
1. The ma n's choice to run away virtua lly_______ to an admission of guilt.
A. resulted B. came c. amounted D. added
2. Environmental pollution has _______ many species to the verge of extinction.
A. sent B, thrown c. brought D. driven
3. After years of working together, the partners found them selves ______ linked.
A. permanently B. indelibly c. perpetually D. inextricably
4. It was decided that the cost of the project would b e _______ and so it was
abandoned.
A. repressive B. prohibitive c. restrictive D. exclusive
5. Living by the ocean really _ _ _ _ _ yo ur_______ . Once y ou've lived there,
you never want to leave.
A. came .in/heart B. get in/heart c. run in/blood D. came in/blood
6. Tamara has set h er _______ on becoming a ballet-dancer.
A. feet B. brain c. head D. heart
7. P aul 's been in Alice’s b ad _______ ever since he offended her at the party.
A. eyes B. books c. likes D. treats
8. Knowing the confidential details gave him a( n) ___over the other candidates.
A. edge B. possibility c. exertion D. fringe
9. He promised me an Oxford dictionary and to my great joy, h e _____ his word.
A. stood by B. stuck at c. went back on D. held onto
10. His new play is not only interesting butalso unique. It is really off the beaten
A. track B. road c. path D. route
103
II. STRUCTURE AND GRAMMAR (5 pts)
Choose the word or phrase which best completes each sentence.
1. John: “Our teacher, Mr. Jones, is not very flexible. He always requires US to
submit his assignments on time.”
Jack: “_______ . He should know that we have to learn many subje cts.”
A. I can ’t disagree with you more B. I c an ’t agree with you more
c. That can be true D. I am not with you here
2. The more expe nsive carpet is a good c ho ice_______ it will last longer.
A. by mean s of B. due to c. in that D. in view of
3. There was no one downstairs; so he turned off the lights again and decided
that she ' imagined things.
A. m ust have B. should have c. can’t have D. n eed n’t have
4. Little Deon: “This herb smells horrible!”
Mom my :_______ , it will do you a pow er o f good.
A. Be th at as it may B. Com e w hat may
c. How much horrible is it D. Whatever it smells
5. _______ I’ve told him not to go out with those people, but he w oul dn’t listen.
Just let him face the music now.
A. M any a time B. Many the time c. Quite a time D. For a time
6. _______ as taste is really a com posite sense made up o f both taste and smell.
A. That we refer to B. What we refer to
c. To which we refer D. What do we refer to
7. _______ the w ater clear but also prevent the river from overflowing.
A. Not only the hippo’s eating habits keep
B. Keep not only the h ippo’s eating habits
c. The hippo’s eating habits not only keep
D. Not only keep the hippo’s eating habits
8. Computer are said to b e_______ for the development of mankind.
A. here today B. here and there
c. here to stay D. neither here nor there
9. Did the minister approve the building plans?- Not really, he turned them down
_______ that the costs were too high.
A. in case B. provided c. on the grounds D. supposing
10. It was such a loud no ise_______ everyone in the house
A. as to w ake B. that to wake c. so as to wake D. th at waking
III. PREPOSITIONS AND PHRASAL VERBS (5 pts)
Choose the word or phrase which best completes each sentence.
1. H e_______ a big fortune when he was young, so he didn 't have to work hard.
A. came into B. came up c. came across D. came round
2. Are you t ak in g_______ all of these phrasal verbs?
A. for
B. down c. of f D. in
104

3. The medicine takes one hour t o _______ .


A. bear with B. kick in c. make out
4. Have you_______ with your homew ork yet?
A. gotten through B. taken over c. thought up
5. Je nn ife r_______ the invitation to join US for dinner.
A. called on B. come out c. got out of
6. If he's clumsy, he can b um p_______ the desk.
A. by B. over c. into
D. get by
D. check ed over
D. passed on
7. We can put you
A. on B. out
8. Mary was astonished that she w as __
A. got by B. turned down
9. After running up the stairs, I w as ___
A. without B. out of
D. through
for a few days if you have nowhere else to live.
c. up D. off
__ for the counselor's position.
c. caught on D. come to
breath.
c. no D. away from
10. She nearly lost her own life ___attempting to save the child from drowning.
A. with B. for c. at D. in
IV. COLLOCATIONS AND IDIOMS (5 pts)
1. I felt a b it _______ and seemed to have more aches and pains than usual.
A. out o f sorts c. on the mend
B. over the worst D. under the fever
2. A: ‘Oh, I'm exhausted! I've been doing homework all day.'
B: ‘Come and put your up for 5 minutes and I’ll make you a cup of tea.’
A. hands B. hair c. heart
3. His English was rou ghly_______ with my Greek, so com munication was
rather difficult!
A. levelled B. on a par c. equal D. in tune
4. Although she had never used a word-proceesor before, she soon got the
_____ of it.
A. feel B. touch c. move D. hang
5. I overslept this morning and caught the last bus to school by the skin of my
A. mouth B. leg c. neck D. teeth
6. If you want a flat in the centre of the city, you have to pay through the
for it.
A. teeth B. back of your head c. nose D. arm
7. You will be putting your life on th e_______ if you take up skydiving.
A. ground . B. line c. way D. lane
8. As far as her future goes, Olivia is _______ . She hasn't got a clue wha t career
to follow.
A. on the level B. all at sea c. behind the scenes D. in the know
9. Your husband w as a bit out of control at the party, to _______ mildly.
A. take it B. put it c. say D. tell
105
10. There is a large ef fo rt ________ to rebuild arts education in the New York
city public schools.
A. under way B. a long way c. out of the way D. in the way
V. REA DING COM PREHENSION (10PTS):
Read the pasages below and choose the best answer to each question.
PASSAGE 1
They are just four, five and six years old right now, but already they are
making criminologists nervous. They are growing up, too frequently, in abusive
or broken homes, with little adult supervision and few positive role models. Left
to them selves, they spend much of their time hanging out on the streets or
soaking up violent TV shows. By the year 2005 they will be teenager s-a group
that tends to be, in the view of Northeastern University criminologist James Alan
Fox, “temporary sociopa ths-impuisive and immature.” , if they also have easy
access to guns and drugs, they can be extremely dangerous.
For all the heartening news offered by recent crime statistics, there is an
ominous flip side. While the crime rate is dropping for adults, it is soaring for
teens. Between 1990 and 1994, the rate at which adults age 25 and older
committed homicides declined 22%; yet the rate jumped 16% for youths
between 14 and 17, the age group that in the early ’90s supplanted 18- to 24-
year-olds as the most crime-prone. And that is precisely the age group that will
be boom ing in the next decade. There are currently 39 million children under 10
in the U.S., more than at any time since the 1950s. “This is th e calm before the
crime storm,” says Fox. “So long as we fool ourselves in thinking that we're
winning the war against crime, we may be blindsided by this bloodbath of
teenage violence that is lurking in the future.”
Demographics do n't have to be destiny, but other social trends do little to
contradict the dire predictions. Nearly all the factors that contribute to youth
crim e-single-p arent households, child abuse, deteriorating inner-city schools -
are getting worse. At the same time, government is becoming less, not more,
interested in spending money to help break the cycle of poverty and crime. All o f
which has led John J. Dilulio Jr., a professor of politics and public affairs at
Princeton, to warn about a new generation of “super predators,” youngsters who
are coming of age in actual and “moral poverty,” w ithout “the benefit o f parents,
teachers, coach es and clergy to teach them right or wrong and show them
unconditional love.”
Predicting a generation's future crime patterns is, of course, risky; especially
when outside factors (Will crack use be up or down? Will gun laws be tightened?)
remain unpredictable. Michael Tonry, a professor of law and public policy at the
University of Minnesota, argues that the demographic doomsayers are unduly
alarmist. “There will be a slightly larger number of people relative to the overall
population who are at high risk for doing bad things, so th at's going to have some
106
effect, he concedes. “But it's not going to be an apocalyptic effect." Norval
Morris, professor of law and criminology at the University of Chicago, finds
Dilulio s notion of super predators too simplistic: “The human animal in young
males is quite a violent animal all over the world. The people who put forth the
theory o f moral poverty lack a sense of history and comparative criminology."
Yet other students of the inner city are more pessimistic. “All the basic elements
that spawn teenage crime are still in place, and in many cases the indicators are
worse," says Jonathan Kozol, author o f Amazing Grace, an examination o f poverty
in the South Bronx. “There's a dramatic increase of children in foster care, and that's
a very high-risk group of kids. We’re not creating new jobs, and we're not
improving education to suit poor people for the jobs that exist."
Can anything defuse the demographic time bomb? Fox urges “reinvesting in
children": improving schools, creating after-school programs and providing
other alternatives to gangs and drugs. Dilulio, a law-and-order conservative,
advocates tougher prosecution and wants to strengthen religious institutions to
instill better values. Yet he opposes the Gingrich-led effort to make deep cuts in
social programs. “A failure to maintain existing welfare and health commitment
for kids," he says, “is to guarantee that the next wave of juvenile predators will be
even worse than we’re dealing with today." Dilulio'urges fellow conservatives to
think o f Medicaid not as a health-care program but as “an anticrime policy.”
(Source: Time Magazine)
1. Young children are making criminologists nervous b ecause_______ .
A. they are committing too much crime
B. they are impulsive and immature
c. they may grow up to be criminals
D. they have no role models
2. The general crime rate in the US i s_______ .
A. increasing B. decreasing c. not changing D. difficult to predict
3. The age group which commits the highest rate of crime i s_______ .
A. 14—17 B. 1 8 -2 4 c. 24 + D. the old
4. James Fox believes that the improvement in crime figures c ou ld _______ .
A. make US complacent in the fight against crime
B. result in an increase in teenage violence
c. make US become fooled and blindsided
D. result in a decrease in teenage violence
5. According to paragraph 3, the g overnment_______ .
A. cutting down on the budget
B. is doing everything it can to solve the problem
c. is not interested in solving the problem
D. is not doing enough to solve the problem
6. In comparison with James Fox, Michael Tonry is _______ .
A. more pessimistic ■ B. less pessimistic
c. equally pessimistic D. indifferent
107
7. Jonathan Kozol believes t hat _______ .
A. there is no solution to the problem
B. employment and education are not the answ er
c. employm ent and education can improve the situation
D. people can solve the problem by improving schools
8. Professor Dilulio thinks that spending on social pro gra ms _______ .
A. should continue as it is B. should be decreased
c. 'is irrele vant to crime rates D. is a better solution to the problem
9. The word ‘lurk ing ” in the paragraph 2 can be best replaced b y _______ .
A. happe ning B. hiding c. impending D. looming
10. The sentence “This is the calm before the crim e storm. ” means "_ _____ .".
A. There will be population booming in in the next decade
B. The age group committing crime m ost in the next decade is now under io
c. The age group 1 4 -1 7 will commit the most crimes in the next decade
D. People will be successful in dealing with the war against crime in the next
decade
PASSAGE 2: For questions 1-6, read the text below and choose the correct
heading for each paragraph B-G from the list o f headings below (i-x). There
are more headings than paragraphs. Write your answers in the corresponding
numbered boxes. There is an example at the beginning. (5pts)
HOW DOES THE BIOLOG ICAL CLOCK TICK?
A. Our life span is restricted. Everyone accepts this as 'biologically' obvious.
‘Nothing lives for eve r!’ However, in this statem ent we think of artificially
produced, technical objects, products which are subjected to natural wear and
tear during use. This leads to the result that at some time or other the object
stops working and is unusable ('death' in the biological sense). But are the
wea r and tear and loss o f function of technical objects and the death of living
organisms really similar or comparable?
B. Our ‘dead’ products are ‘static’, closed systems. It is always the basic material
which cons titutes the object and which, in the natural course of things, is
worn down and becom es ’older’. Ageing in this case must occur accord ing to
the laws of physical chemistry and of thermodynam ics. Altho ugh the same
law holds for a living organism, the result of this law is not inexorable in the
same way. At least as long as a biological system has the ability to renew
itse lf it could actua lly become older w ithout ageing; an organism is an open,
dynam ic system through which new material continuously flows. Destruction
of old material and formation of new material are thus in pe rmanent dynamic
equilibrium. The material of which the organism is formed changes
continuously. Thus our bodies continuously exchange old substance for new,
just like a spring which more or less m aintains its form and movement, but in
which the w ater molecules are always different.
108
c . Thus ageing and death should not be seen as inevitable, particularly as the
organism possesses many mechanisms for repair. It is not, in principle,
necessary for a biological system to age and die. Nevertheless, a restricted life
span, ageing, and then death are basic characteristics of life. The reason for
this is easy to recognise: in nature, the existent organisms either adapt or are
regularly replaced by new types. Because of changes in the genetic material
(mutations) these have new characteristics and in the course of their
individual lives they are tested for optimal or better adaptation to the
environmental conditions. Immortality would disturb this system - it needs
room for new and better life. This is the basic problem of evolution
D. Every organism has a-life span which is highly characteristic. There are
striking differences in life span between different species, but within one
species the parameter is relatively constant. For example, the average
duration of human life has hardly changed in thousands of years. Although
more and more people attain an advanced age as a result of developments in
medical care and better nutrition, the characteristic upper limit for most
remains 80 years. A further argument against the simple w ear and tear theory
is the observation that the time within which organisms age lies between a
few days (even a few hours for unicellular organisms) and several thousand,
years, as with mammoth trees.
E. If a lifespan is a genetically determined biological characteristic, it is logically
necessary to propose the existence of an internal clock, which in some way
measures and controls the aging process and which finally determines death
as the last step in a fixed programme. Like the fife span, the metabolic, rate
has for different organisms a fixed mathematical relationship to the body
mass. In comparison to the life span this relationship is ‘inverted’: the larger
the organism the lower its metabolic rate. Again this relationship is valid not
only for birds, but also, similarly on average within the systematic unit, for all
other organisms (plants, animals, unicellular organisms)..
F. Animals which behave ‘frugally’ with energy become particularly old for
example, crocodiles and tortoises. Parrots and birds of prey are often held
chained up. Thus they are not able to ‘experience life’ and so.they attain a high
life span in captivity. Animals which save energy by hibernation or lethargy
(e.g. bats or hedgehogs) live much longer than those which are always active,
The metabolic rate of mice can be reduced by a very low consumption of food
(hunger diet) They then may live twice as long as their well-fed comrades.
Women become distinctly (about 10 per cent) older than men. If you examine
the metabolic rates of the two sexes you establish that the higher male
metabolic rate roughly accounts for the lower male life span. That means that
they live life ‘energetically’ - more intensively, but not for as long.
G. It follows from the above that sparing use of energy reserves should tend to
extend life. Extreme high performance sports may lead to optimal
cardiovascular performance, but they quite certainly do not prolong life.
109
Relaxation lowers metabolic rate, as does adequate sleep and in general an
equable and balanced personality. Each of US can develop his or her own
‘energy saving programme' with a little self-observation, critical self-control
and, above all, logical consistency. Experience will show that to live in this
way not only increases the life span but is also very healthy. This final aspect
should not be forgotten.
For question 1-6, choose the correct heading for paragraphs A-G from the list
of headings below.
Write the correct number, Ỉ-X, in the corresponding num bered boxes.
LIST OF HEADINGS
The biological clock
i
Why dying is beneficial
ii
The ageing process o f men and women
iii
Prolonging your life
iv
Limitations o f life span
V
Modes of development of different species
vi
A stable life span despite improvements
vii
Energy consumption
viii
Fundamental differences in ageing o f
ix
objects and organisms
X
Repair o f genetic material
Example answer: Paragraph A: V
answers:
__________
1. Paragraph B
2. Paragraph c •____
3. Paragraph D __________
4. Paragraph E ______ _
5. Paragraph F __________
6. Paragraph G __________
Questions 77- 90, complete the notes below
Choose NO MORE THAN TWO WORDS from the passage for each answer.
• Objects age in accordance with principles o f (7 )_______________ and o f (8)
• Through mutations, organisms can ( 9) __________ better to the environment.
• (1 0) ____________ would pose a serious problem for the theory of evolution.
VI. GUIDED CLOZE TEST (lOpts):
Read the following passages and decide which option A, B, c or D best
fits each space.
PASSAGE 1
The coolest kids in Europe share a single ( 1)______ they want to get married,
have children and live happily ever after. They know it means (2 )______their
110
children first and sticking with their spouses even if they slip out of love. This news
comes from the report o f a new study that ( 3)__________ out to find the answer to
the modem riddle: What will today's youth really, really want tomorrow?
Poignantly, one of the clearest answers is that they want to have happy
families. Even in the most (4 )_______ _ _ _ countries there was conde mnation
for divorce, demands that parents should keep their marriage (5 )___________
and admiration for stable couples.
It appears that among the middle classes, the quality of our child ren's lives
has suffered from the pressures on parents in high-stress professions. In the days
when the concept of 'quality time' first (6 )___________ , I reme mber seeing a
TV producer on (7 )_______ dial home on her mobile phone to read her son a
bedtime story. This is just not good enough.
Quality time cannot be time (8 )______ . Children need unconditional time in
the same way that they need unconditional love. This study found a generation
that had given up trying to (9 )______ its parents' attention but was (1 0)_______
to do better by its own children.
1. A. drive B. eagerness c. ambition D. yearning
2. A. putting B. keeping c. having D. wanting
3. A. made B. aimed c. looked D. set
4. A. loose B. relaxed c. generous D. liberal
5. A. oaths B. vows c. pledges D. promises
6. A. proceeded B. revealed c. emerged D. rose
7. A. site B. place c. situation D. location
8. A. organized B. managed c. controlled D. discipline
9. A. have B. make c. get D. take
10. A. determined B. firm c. persistent D. stubborn
PASSAGE 2
KIWI SURPRISE
When a work project gave me the opportunity to return to New Zealand, I
spent several weeks ( ! ) _______ a country I had left in my early twenties. Ed
forgotten about the petrol stations where men in smart uniform s (2) _ _ _ _ _ to
you. They fill your tank, check your oil and still (3 )____ you less than one third
of the British price for fuel. And the people rush to your assistance if they see
you (4 )______ over a map. Or the blissful absence of tips. Locals simply cann ot
understand why anybody should expect to pay extra for friendly efficient service.
Given that New Zealand has about 3,000 kilometers of co astline, it should
come as no (5) _______ that social life (6) _______ around the sea. When
Auckland office workers leave their desks at the end of the working day, they
don’t (7 )_______ home. Instead, they (8) _ ______a beeline for the marina and
spend the evening under sail on the Hauraki Gulf. There are more yach ts in
Auckland than in any other city in the world- no wond er it’s called the City of
111
Sails. Even those who can ’t of their own will always
afford a (9) know
, or at the very least, will
someone who has one. win dsurf the offshore
breezes at
speeds that make the
commuter ferries appear to .
stand (10)
c . refamiliarizing D.
1. A. regaining B. recapturing
rediscovering
2. A. assist B. attend c. supply D.ser ve
3. A. charge B. ask c. require D. dem and
4. A. pointing B. doubting c. clamoring D. puzzling
5. A. wonder B. surprise c. amazement D. news
6. A. centers B. revolves c. turns D. gathers
7. A. move B. aim c. head ’ D. divert
8. A. have B. do c. get D. make
9. A. vehicle B. hull c. vessel D. receptacle
10. A. still B. dead c. afloat D. upright
B. WRITT EN TE ST (70pts)
I. OPEN CLOZE TEST (20 pts):
Read the passages below and fill each blank with ONE word.
CLOZE TEST 1 (lOpts)
As a result of the recent discovery of lunar water, the moon has suddenly
become a far more interesting place for investors, (1 )_______ must now view
the long-term prospects with optimism. The last manned mission to the moon
drew (2) _______ a close in 1973, when two astronauts from Apollo 17 climbed
back into their lunar module, (3 )_______ collected a lot of moonrock, but bereft
(4 )_______ any future plans. Now the moon shines brighter for astronauts and
scientists alike, (5 )_______ to the existence of (6 )________might be billions of
tonnes of water at the poles.
There is (7 )_______ high-tech substitute for water in space exploration. To
support the international space station, (8 ))_________ has cost at least $100,000
a day to send water into orbit. Not only would lunar w ater cut these costs, but it
would additionally be used for rocket fuel, (9 )_______ two components, liquid
hydrogen and liquid oxygen, are the elements found in water.
Scientists are particularly excited (10) _______ , given the absence of an
atmosphere, lunar water has never been recycled and they believe, therefore, that
it could very well hold clues to the formation of the solar system itself.
CLOZE TEST 2 (lOpts)
For over two hundred years, scholars have shown an interest in the way
children learn to speak and understand their language. Several small-scale
studies were carried out, especially towards the end of the nineteenth century,
(1)_______ data recorded in parental diaries. But detailed, systematic
investigation did not begin until the middle decades of the twentieth century,
when the tape recorder (2 )_______ into routine use. This made it possible to
keep a permanent record of samples of child speech, so that analysts could listen
112
repeatedly to obscure ( 3 ) _______ , and thus produce a detailed and accurate
description. The problems that have (4) _______ when investigating child
speech are quite different from ( 5 ) _______ encountered when working with
adults. It is not possible to carry out certain kinds of experiments, because
aspects of children’s cognitive development, such as their ability to (6 )_______
attention or to remember instructions, may not be sufficiently advanced. (7)
_______ is it easy to get children to (8 )________ systematic judg ments about
language - a task that is virtually impossible below the age of three. Moreover,
anyone who has tried to make a tape recording of a representative sample of a
child’s speech knows how frustrating this can be. Some children, it seems, are
innately programmed to (9 )_______ off as soon as they notice a tape recorder
(1 0) _______ switched on.
II. WORD FORMATIONS (20 pts)
PAR T 1: Complete the sentence with the correct form of the given word. (10 pts)
1. The private, school feared losing its with the state's university system.
(CREDIT)
2. That thè child beh ave d_______ made the couple happy. (DEAR)
3. The candidate made a( n) _______ speech that incensed all those who heard it.
(FLAME)
4. The business is as it can no longer meet the repayments on its debt.
(SOLVE)
5. A _______ is a popular place for tourists and travellers to send and receive emails. (CAFÉ)
6. In Scotland, there is greater emphasis o n ___by individual schools. (VALUE)
7. Babies affected by the disease will be born small, _______ and braindamaged. (FORM)
8. She stood there completely , so I had no idea at all what she was
thinking. (EXPRESS)
9. “What if ’ questions inv olving_______ are familiar in historical speculations.
(FACT)
10. The Prime Minister warned thẹ people o f his country that they must be ready
for any_______ - even the possibility of war. (EVENT)
PART 2: Complete the passage with appropriate forms from the words
given in the box. (lOpts)
offend) respect irritate temper
ground construct organize compose

ANGER .
We’ve all felt anger at some time, whether as faint annoyan ce or blind rage.
Anger is a normal, sometime useful human emotion, but un controlled (1)
of temper can be destructive. People who give free rein to their anger, regardless
113
of the (2 )____ this may cause, haven’t learned to express themselves ( 3)_____ ,
says Martin Smolik, who runs weekend residential courses in anger management. ‘It
is important to maintain your (4 )______ and put your case in an assertive, not
aggressive manner without hurting others. Being assertive doe sn’t mean being
pushy or demanding; it means being (5 )_______ of yourself and other peop le.’
He adds that people who are easily angered1 are (6) _______ of frustration,
inconvenience or irritation and, not surprisingly find relating to other people very
difficult. But what causes people to behave like this? It seems there is evidence
to support the idea that some children may be born (7 )_______ and prone to
anger and this tendency is sometimes apparent from a very early age. However,
research also suggests that a person’s family (8 )_______ may have an influence.
Very often, people who are (9 )_______ and often find it difficult to express
their emotions come from (1 0) _______ and disruptive families.
III. ER ROR CO RREC TION (10 pts): Read the following text which
contains 10 mistakes. Underline the errors and write the corrections in the
corresponding numbered boxes. There is an example at the beginning (0).
Line
123456789 The arrival of satellite TV
10 has brought up a whole new
11 world of
12 viewing into our living
13 rooms - if we can afford to
pay for it, that is,
major sport events can now
be seen live. Beside this, a
wider variety o f
sports is now available. The
viewer can choose anything
from dog
racing and sumo wrestling.
Certain channels show 20
film every day,
again, the choice is
enormous-from old classics
to the latest Hollywood
releases. For them who like
to keep informed, 24-hour
news is available
for the touch of a button.
Children are not forgotten
neither. A special
jun ior channel broadcasts
cartoons and children’s
films. But do we
really need all this choice?
The danger is that we will
become a
population of passive couch-
potato with square eyes and
fingers glued
to the remote-control. Of
course choice is a good
thing, but viewers
should use his ability to
select the best and disregard
o f the rest.
Your answers: 0. up
1.______________ _
4.________________
7.
on (line 1)
2. ______
5 .______
8.
3._
6."
9."
io"
IV. SENTENCE TRAN SFOR MATION (20 PTS):
PART 1: Complete the second sentence in such a w ay that it means exactly
the same as the sentence printed before it. (10 points)
1 .1 was too scared to tell him what I really thought.
—> Hacked.............................................. ................................
114
2. It’s a widespread assumption that George was wrongly accused.
—* George................................................................................................................
3. He declared his disapproval o f the behaviour of some of his supporters.
—> He let it..............................................................................................................
4. The collision didn't damage my car much.
—> No t a g re at......................................................................................................
5. Although he is 8 years older than her, they were good friends.
—* Despite ,................................................................................
PA RT 2: Complete the second sentence so that it has a similar meaning to
the first one. Use the word given in block letter, and this word must not
be changed in any way. (10 points)
6.1 presum e you are coming to the party Miriam. READ
—> Can 1 .....................:........... ............................................. you are coming to
the party Miriam?
7. After a long hard journey, I cheered up when I saw my home. SIG HT
—> After a long hard journey, my spirits................................ of my home.
8. Yvonne did everything she could to ensure the trip was successful. LE NG TH S
—> Y vo nn e..... é............................................ Í................ to ensure the success.
9. Russ's opinions on the new management policies were very different from
those o f his fellow workers. ODDS
—> Russ ....... .................................................... ............................. ........ the new
management policies.
10.1 tried as hard as I could to make sure that this problem would not arise. PO WE R
—* I did ................................................................... this problem from arising.
TRƯỞNG THPT CHUYÊN NGUYỄN THỊ MINH KHAI - s ó c TRĂNG
A. MULT IPLE CHOICE (40 points)
1 .1-10 W OR D CH OI CE
Choose the best options to complete the following sentences.
1. Although she would have preferred to carry on working, my mum........... her
career in order to have children.
A. devoted B. repealed c . sacrificed D. abolished
2 .1 find the offer q uite............. . but I think I’d rather study at Oxford
A. tempting B. desirous c . inclined D. envious
3. I don ’t normally like noisy clubs, but I had a sudden .. .. .. .. .. to see wha t the
Blue Parrot was like.
A. force B. motive c . pressure D. impulse
4. Jerry loves snowboarding so much that it’s almost like a d ru g............
A. passion B. obsession c . addiction D. requirem ent
115
5. I do n't want to do the course in applied statistics, but i t 's .............
A. compulsory B. inevitable c. bound D.indecisive
6. Don't worry about me - I’m quite.......to sit here and wait for you to come back.
A. ecstatic B. delighted c.joyful D. con tent
7. When 1 was pregnant, 1 often got a su dd en .............for tinned sardines.
A. preference B. craving c. envy D. greed
8. Thank you for thinking of US, but I’m afraid we’re going to have to ............
your kind invitation.
A. decline B. deny c. condem n D. reject
9. I'v e never seen anyone s o ........... to their joy as Philip is.
A. eager B. Keen c. dedicated D. interested
10. I wish you would stop wasting so much on your computer games and do
something as a little more...........
A. welcome B. enviable c. feasible Đ. worthwhile
IL 11-20 GRAMMA R AND STRUCTURES
Choose the best options to complete the following sentences.
11.1 recommend that Miriam a boarding school. She’d be much more challenged
academically.
A. attends B. attend c. is attending D. were attending
12. Ambrose had to take a job at a fast-food res tau ran t;.... ........he wouldn’thave
been able to make his car payment.
A. otherwise B. if so c. had he done so D. were that the case
13. Many educators seem convinced students would respond favorably to a
change in the system.
A. what B. the fact that c. that D. whatever
14. ............... students wanted to go right into the workplace after graduation
would choose the vocational track.
A. Whatever B. However c. Whenever D. Whomever
15. He ..............us on the last day of the congress so his presence at the opening
ceremony was something of a surprise.
A. must have joined B. was to join c. had to join D. should join
16. In fact the crim inals....... in because the front door was wide open and so
they just walked in.
A. needn’t have broken B. didn’t need break
c. did n't need to break D. nee dn't to have broken
17. People whose main concern is ........... may eventually develop some serious
health problems.
A. not to be overfed B. not to be overfeeding
c. not to overfed D. not overfeeding
18................ of Albania was once forested, but ..........of the original vegetation
remains, due to centuries o f clearing and livestock grazing.
A. Much - little B. M an y- no ne c. Some - few D. M ost -a ny
19. Most critics agree that rather li tt le ............. done by the artist after his illness
has any great value.
A. of what was
B. of that was c. of that what was D. which has been
116

20............... between the two attacks, the comm ander tried to prevent the enemy
troops........... their strength.
A. Without delaying - to gather B. By not delaying - from gathering
c. In delaying - for gathering D. Having delayed - without gathering
III. 21-30 PREPOSITIONS - PHRASAL VERBS
Choose the best options to complete the following sentences.
21 .............being a little messy, she will be a good flatmate for you.
A. F ar away B. At length c. Beside D. Apart from
22 ......... working very long hours, John strives to exercise at least trice a week.
A. Regardless of B. On beha lf of c. Because of D. However
23. There is a rum our............. circulation that the director had a big argum ent
........... the manager at the last meeting.
A. o n-a b o u t B. u pon-i n c. fo r- o u t D. a ft e r- o ff
24. Factories which produce a lot of toxic fumes are contributing ................the
de str uc tio n..........th e environment.
A. f or- ar ound B. b y-a gai nst c. into - upon D. t o - o f
25. It’s so hot, 1 could d o ......... a nice cool drink.
A. up B. with c. out D. without
26. Once we 'd spent all our money, we f e ll ........... our credit card.
A. out B. for c. back on D. off
27.1 fe ll ..............one of my friends and have n’t spoken to her for weeks.
A. in with B. out with c. through D. behind
2 8 .1 c an’t g e t.............to Joan. I think there’s a fault on the line.
A. away B. at c. through D. along
29. The record shop has a promotion and is givi ng .............free CDs.
A. away B. of f c. in D. out
30. H e’s a good speaker and gets his views......... very well.
A. over B. across c. on D. through
IV. 31-40 COLLOCATIONS AND IDIOMS
Choose the best options to complete the following sentences.
31 I h aven’t had an accident yet but I’ve had a number o f .............shaves.
A. narror B. near c. close D. tiny
32. My father refused to eat meat that had been fried. He had............... in his
bonnet about it causing cancer.
A. a bug B. a bee c. a bull D. an ant
33. You really drop pe d..;...... ;...th e other day when you told Brian you ’d seen
his wife at the cinema. He thought she was at her mother’s.
A. a brick B. a stone ' c. a log D. a plank
34. I can’t stand Mr. Bryant. He’s always blowing his own ... - telling everyone
how good he is at everything.
A. balloon B. breath c. mind D. trum pet
35. The escaped prisoner f ou gh t....... ...before he was finally overpowered
A. head over heels B. tooth and nail
c. heart and soul D. foot and mouth
117
36. I didn’t suspect anything at first, but when I noticed her going through the
office drawers I began to s m el l.............
A. a rat B .a pig c. a th ief D. culprit
37. Jane looked ................ at the shop assistant who had been rude to her.
A. arrows B. needles • c. poison D. daggers
38. Peter was born and brought up in Hastings and knows it like the.......
A. nose on his face B. tip of his tongue
c. back of his hand D. hair on his head
39 I've heard that argum ent before and quite frankly it jus t d oesn’t ..............
A. face the music B. hit the nail on the head
c. carry weight D. hold water
40. John will never buy you a drink - h e’s far t o o ................
A. tight-fisted B. pigheaded c. highly-strung D. easy-going
V. 41-60 RE AD IN G CO MPR EH EN SION
41-50 READING 1
Read the passa ge and ch oose the best option for each qu estio ns below
Sharks have gained an unfair reputation for being fierce predators of large sea
animals. Humanity's unfounded fear and hatred of these ancient creatures is
leading to a worldwide slaughter that may result in the extinction of many
coastal shark species. The shark is the victim of a wa rped attitude of wildlife
protection; we strive only to protect the beautiful, non-threatening parts of our
environment. And, in our efforts to restore only non-threatening parts of our
earth, we ignore other important parts. A perfect illustration of this attitude is the
contrasting attitude toward another large sea animal, the dolphin. During the
1980s, environmentalists in the United States pro tested the use of driftnets for
tuna fishing in the Pacific Ocean since these nets also caught dolphins. The
environmentalists gen era ted enough political and economic pressure to prevent
tuna companies from buying tuna that had been caught in driftnets. In c ontrast to
this effort on behalf of the dolphins, these same environmentalists have done
very little to help save the Pacific Ocean sharks whose population has decreased
nearly to the point of extinction. Sharks are among the oldest creatures on earth,
having survived in the seas for more than 350 million years. They are extremely
efficient animals, feeding on wounded or dying animals, thus perfo rming an
important role in nature of weeding out the weaker animals in a species. Just the
fact that species such as the Great White Shark have ma naged to liv e in the
oceans for so many millions of years is enough pr oo f of their efficiency and
adaptability to changing environments. It is time for US humans, who may not
survive another 1,000 years at the rate we are damaging the planet, to cast away
our fears and begin considering the protection of sharks as an important part of a
program for protection o f all our natural environment.
11 8
/•
41. With which of the following topics is this passage primarily concerned?
A. Sharks are efficient creatures with bad reputations.
B. Sharks are some of the oldest creatures on earth,
c. Sharks illustrate a problem in wildlife protection
D. The campaign to save dolphins was not extended to save sharks.
42. Which of the following is most similar to the meaning of the word "warped"
in line 4?
A. distorted B. wasteful c. extravagant D. wãnton
43. In line 10, the word "protested" is closest in meaning to which of the following?
A. prescribed B. objected to c. protected D. reflected on
44. In line 11, the word "generated" could be best replaced by
A. consumed B. absorbed c. designated D. produced
45. How did environmentalists manage to protect dolphins?
A. They prevented fishermen from selling them for meat.
B. They pressured fishermen into protecting dolphins by law.
c. They brought political pressure against tuna companies.
D. They created sanctuaries where dolphin fishing was not allowed.
46. About how long have sharks lived on the planet?
A. 25 million years B. 150 million years
c. 350 million years D. 500 million years
47. In lines 19-20, the phrase "man aged to live" is used to infer that
A. surviving was difficult B. migration was common
c. procreation was expanding D. roaming was necessary
48. The word "proof' in line 20 could be best replaced by which of the following?
A. characteristic B. evidence c.praise D. customary
49. What is the author's tone in this passage?
A. explanatory B. accusatory c.gentle D. proud
50. Which o f the following best describes the organization of this passage?
A. order o f importance B. cause and effect
c. statement and example D. chronological order
51-60 R EA DI NG 2
Read the passa ge below and choose the cor rec t a nsw ers
WATER IN DESERT
Desert areas of the world are not completely void of precipitation, but rainfall
in these regions is known to vary greatly. Typically, scientists use an annual
rainfall amount o f four inches as the dividing line between desert and non-desert
areas. While these arid regions may receive a similar amount of rainfall each
year, how that rainfall affects surface water and groundwater resources is largely
dependent upon the area’s topographical characteristics. For example, landforms
such as flats and depress ion s common to deserts are known to collect water, but
these features constitute only a small percentage of the desert landscape. It is
119
interesting to note that these arid lands in fact contain some of the ea rth’s largest
systems o f rivers. In the Cases o f these river systems, the sources of the rivers lie
outside the arid region -itself, and hence they are known as “exogenous” rivers
and systems. These exogenous systems play a critical role in nourishing life in
the world's most parched regions. For example, annual flooding of the Nile,
Tigris, and Euphrates rivers has long supplied residents with water and brought
in fertile silt to help agriculture. With the advent of modern technology,
however, this flooding is largely managed by human hands, which has
required more governmental cooperation among nations in crafting treaties
and agreements regulating the use of these river basins.
In addition to human intervention, the flow of exogenous rivers is impacted
by the season. Ironically, it sometimes takes a number o f months for the effect of
a rainy season outside a desert area to be felt within it, so by the time the peak
flow finally arrives, it may in fact be the driest time of the year. While this
additional wate r does aid the irrigation for agriculture, weather changes during
the dry season like higher temperatures and lower humidity may make the
cultivation of some crops even more challenging than it is during the normal
growing season.
In contrast to exogenous rivers, those identified as “endogenous” systems
begin and end within the arid region. Water in these systems generally comes
from groundwater springs, but many of these rivers are fed by water coming
from limestone massifs, such as the Atlas Mountains in Morocco. Rivers of this
variety tend not to reach the sea, but instead end up draining into inland basins,
where most of it is simply lost to evaporation or disappears into the ground.
Though rivers and lakes are present in desert areas, groundwater makes up a
much larger percentage of the total water in these regions. However, only a small
portion of these underground deposits ever reach the hydrologic cycle; i.e., it
never enters the constant m ovement o f water on, above, and below the surface of
the earth that characterizes most of the planet’s other w ater resources. ■ A) The
groundwater that does enter into this cycle usually does so by feeding the flow o f
streams or by maintaining water levels in lakes. Water taken in this way from
groundwater stores is then refilled by surface flows and rainwater. ■ B)
International organizations such as the United N ations and the World Bank have
both funded efforts to survey existing groundwater in arid lands and to create
effective methods to draw usable water from these sources. ■ C) These efforts
are particularly necessary due to the lack of clear understanding in these regions
about jus t how much groundwater exists. ■ D) What is known, however, is that
these groundwater deposits are very unevenly distributed, and that much of this
water can be found deep beneath the earth’s surface.These groundwa ter deposits
are found underground in open spaces between, inside, and among rocks and
sediment. These water-laden layers of earth are known as “aquifers.” In general,
limestone and sandstone aquifers tend to be deep and large, which contributes to
120
their relatively high concentration of minerals. They are usually not replenished
because they became saturated more than 10,000 years ago and do not need to
be. These deep aquifers are sometimes called “fossil waters” to denote the fact
that this water has been present for so long. On the other hand, shallow aquifers
found in sand or gravel are much less extensive, but they can quickly be replenished.
51. According to paragraph 1, what do scientists use to determine whether or not
an area is labeled a desert?
A. The amount o f annual rainfall
B. The lack o f groundw ater resources
c. Topographical characteristics
D. The absence of flats and depressions in the landscape
52. The word “depressions” in line 7 is closest in meaning to
A. rifts B. dunes c. ranges D. basins
53. Which of the sentences below best expresses the essential information in the
highlighted sentence in the passage? Incorrect choices change the meaning in
important ways or leave out essential information.
A. Government involvement in the creation of treaties and agreements enhances
flood-controlling m odem technology o f river basins.
B. Technology has allowed for better human management o f floods,
makingriver basin regulation agreements largely unnecessary.
c. Humans have been able to use technology to control flooding, which hasmade
international cooperation on the use of river basins more necessary.
D. More modem technology will lead to better flood management
becauseintergovemmental treaties regarding river basins allow for cooperation.
54. in paragraph 3, the author mentions all of the following about the flow of
exogenous rivers EXCEPT:
A. The impact o f the flow is often felt in the driest seasons.
B. The flow can be an aid in the cultivation of crops,
c. The effects of the flow are not felt immediately.
D. The flow is unaffected by the season of the year.
55. According to paragraph 4, what distinguishes an endogenous river from an
exogenous river?
A. An endogenous river flows into a limestone massif.
B. The source of an endogenous river is within the desert region,
c. The endogenous river does not reach the sea.
D. Endogenous rivers have a higher saline content than exogenous rivers.
56. The word “stores” in line 39 is closest in meaning to
A. locations B. accumulations c. caves D. discoveries
57. Why does the passage mention in paragraph 5 the fact that the exact extent of
groundwater, resources is still unknown?
A. To highlight the lack of scientific research being done in desert areas.
B. To illustrate the importance of freshwater to the inhabitants of deserts.
121
c. 1o explain why the recent actions of the United Nations and others are
needed.
D. To show how groundwater can affect the flow of exogenous rivers.
58. The word "replenished" in line 51 is closest in meaning to
A. refilled B. rejuvenated c. reinvigorated D. rescinded
59. What can be inferred in paragraph 7 about shallow aquifers?
A. They tend to have less mineral content than deeper ones.
B. They can quickly be replenished because they were saturated as limestone
aquifers once were.
c. The water from shallow aquifers is unusable because of its proximity to
gravel.
D. They are more extensive than limestone aquifers.
60. Look at the four squares [ ■] that indicate where the following sentence
could be added to the passage.
In fact, these water deposits have recently become the focus of increased
attention as a source of freshwater for the inhabitants of desert areas.
Where would the sentence best fit?
ABCD
VI. 61-80 GUIDED CLOZE TESTS
Read the texts below and decide which answer best fits each space.
PASSAGE 1
Greenhouse gases are being released into the atmosphere 30 times faster than
the time when the Earth experienced a (61)............ episode of global wanning.
A study comparing the rate at which carbon dioxide and methane are being
(62)........... now, compared to 55 million years ago when global warming also
occurred, has found dramatic differences in the speed of release.
James Zachos, professor of earth sciences at the University of California,
Santa Cruz, said the speed of the present buildup of greenhouse gases is fatgreater than during
the global warming after the (63)................of the dinosaurs.
"The emissions that caused this past episode of global warming probably lasted
10,000 years," Professor Zachos told the American Association for
the Advancement of Science at a meeting In St. Louis. "By burning fossil
fuels, we are likely to emit the same amount over the next three centuries." He
warned that studies o f global warming events in the geological past (64)........... ".
the Earth's climate passes a (65).......... beyond which climate change accelerates
with the help of positive feedbacks - vicious circles of warming, professor
Zachos is a leading ( 66)..............on the episode o f global warming known as the
palaeocene-eocene thermal maximum, when average global temperatures
increased by up to 50C due to a massive release of carbon dioxide and methane.
His research into the deep ocean (67)............... suggests at this time that
abou t 4.5 billion tons of carbon entered the atmosphere over 10,000 years.
122
"This will be the same amount of carbon released into the atmosphere from cars
and industrial emissions over the next 300 years if present (68)................
continue", he said. Although carbon can be released suddenly and naturally into
the atmosphere from volcanic activity, it takes many thousands of years for it to
be removed permanently by natural processes. The ocean is capable of removing
carbon, and quickly, but this natural (69)................ can be easily (70 ).........
which is probably what happened 55 million years ago. “It will take tens of
thousands of years before atmospheric carbon dioxide comes down to
preindustrial levels," the professor said. "Even after humans stop burning fossil
fuels, the effects will be long-lasting
61 .A. prearranged B. premier c. previous D. fundamen
62. A. emitted B. exhaled c. incorporated D. digested
63. A. dementia B. demolition c. detachment D. demise
64. A. comment B. mark c. compliment D. indicate
65. A. Barricade B. verge c. threshold D. perimeter
66. A. autocrat B. authority c. administrator D. proprietor
67. A. dusts B. sediments c. dirt D. powder
68. A. trends B. gadgets c. fads D. crazes
69. A. capacity B. competence c. intelligence D. bulk
70. A. overcharged B. overstated c. overshadowed D. overwhelm
PASSAGE 2
On a yearly (71)....... , the United States is afflicted by hurricanes on the east
coast, flooding in the midwest, forest fires, earthquakes, and any number of
tornadoes, blizzards and storms. Historically, the weather has (72)..........as harsh
a chord in people's lives as any we feel today. Historians list the four major
events to wreak devastation on the country as the Chicago fire of 1871, the
Johnstown flood of 1889, the Galveston hurricane of 1900, and the San
Francisco earthquake o f Ĩ906. These disasters provide a(n) (73)...... . of the havoc
nature's storms have wrecked on humanity, and our reactions to them. Galveston,
Texas, located on the island of Galveston in the gulf of New Mexico, thrives on
industry and exports. Its over 30,000 citizens in the early 1900s were generally
economically successful and comfortable with (74)...........conditions. Until the
ill-fated morning of Sept. 7, 1900, the people of Galveston enioyed their life on
the remote island. However, weather watchers had been following a growing
storm in the Atlantic for several days and were warning of the impending danger
of a hurricane. Unfortunately, people ignored the warnings. A day after the
hurricane had hit the island, all that remained of the beautiful city was a mass of
crumbled buildings, debris and forlorn, hopeless survivors wandering aimlessly
with the stench of rotting flesh all around. It is (75)................ that over 8,000
people and most of the animals died that day, victims of one of the deadliest
natural disasters in u.s. history. Chicago's storm of 1871, although (76).............
123
to the hurricanes of the southeastern coast, was described by poet John Greenleaf
Whittier as a 'fiery hurricane' that struck the great city with such (77).............
that two days later, hal f of the city had been destroyed and 300 lives lost. Those
relating the events of that catastrophic 'hurricane' say that too little rain and a
strong wind blowing off the prairie had been the cause behind ihe destruction of
the Windy City. According to (78)...... . however, responsibility was placed on
one slovenly cow who kicked over a lantern in her barn somewhere in the
southwest corner of Chicago, igniting the dry and highly flammable barn and
ultimately bringing destruction to a large part of the city. Two days after the
sparks ignited, a steady rain on the burning city put out the flames and within
days hope had returned. Signs that the city was beginning to heal became
(79)........... everywhere - soon new buildings were (80)................the burned,
and the lifeblood of the ravaged city was being.
71. A. foundation B. theory c. basis D. institution
72. A. punched B. struck c. pinched D. played
73. A. scent B. flavour c. recipe D. ingredient
74. A. existing B. remaining c. enduring D. staying
75. A. anticipated B. predicted c. estimated D. forecast
76. A. disconnected B. unaccustomed c. unrelated D. discredite
77. A. stimulus B. force c. charisma D. constraint
78. A. tradition B. culture c. phenomenon D. legend
79. A. concealed B. notorious c. obscure D. evident
80. A. replacing B. rebuilding c. designing D. refurbishi
B. WRITTEN QUESTIONS (70 points)
I. 1-20 OPEN CLOZE TESTS
Read the texts below and think of the word which best fits each space. Use
only ONE WORD for each space.
PASSAGE 1
Most people I know (1) ............. never go to a martial arts movie, even if you
paid them, but I defy anyone not to enjoy seventy minutes in the dark with Jackie
Chan. For a start, Chan is more interested in apologetically evading the bullies
that want to do him in than in violent confrontation. His screen persona is never
(2) ............ to bombast. Chan is a likeable, bumbling. Everyman who tries to
extricate himself from scrapes with his astounding athletic (3) : as he leaps up
the side of a building, you (4) ............ swear he was on wires. With the kind of
agility (5 )......... limited to monkeys and flies, Chan seems (6).........o f scuttering
up any surface. But it is extremely (7) ............ for him to go on the offensive.
The films of lesser action stars like Jean-Claude Van Damme provide a diet of
relentless violence, punctuated (8)................ now and (9) ................. by some
semi-moronic ‘witticism1, but Chan's balletic altercations with his enemies are as
•a ( 10)..........oriented around the art of comic evasion.
124
PASSAGE 2
It was last May, while we were taking our annual late spring break on Lindos
that we (11 )........ our house broken into. All our TV and video equipmen t got
(12 )........... but what was worse was when we discovered that the final draft of
my husba nd's latest novel (13 )........... been torn into pieces and the disks he had
(14 )......... writing it on had (15) .......... Of course, you hear about people who
have had their properties vandalized and (16)................ whose most prized
possession have been (17 )........... . but it's a terrible shock when it (1 8)........... to
you, when you know that your home has been invaded, and that you have had
our (19)......... intimated belongings handled and examined (20 ).......... strangers.
II. 21-40 WORD FORM ATION
21-30 Use a word given in capitals to form a word that fits in the space
Students learning English as a second language are sometimes given a word
by the ir teacher and asked to give an ( 21).................. (EXPLAIN) as to what that
word means; in other words, to provide a (22) (DEFINE). The (23) ...................
(ASSUME) is that if you know a word, you can define it. (24) ........................
(LOGIC), that might make sense, but in reality it is not always (2 5 ).................
(REASON) to assume that. There are words and phrases that even native
speakers use in conversation without much (2 6).................. (THINK) which can
lead to (27) .................. (CONFUSE) when you ask a native speaker to define
them. Take the (2 8).................. (CONCEIVE) o f ‘Zeitg eist’, for example, which
has entered English from German. It’s ...............(DOUBT) much easier to use
than it is to define. With a word like ‘Zeitgeist', it may be more (3 0)...................
(SENSE) to test the student’s understanding in ways other than asking them to
define it.
31-40. W rite a word in the box in the correct form in each gap
columnist critic ghostwriter handbook
novelist Review er Subtitle supplement
FREELANCE WRITING
When you'r e a freelance writer, there are many varied opportunities open to
you. You may be working on that great work of fiction and know that within
couple of years you’ll be regarded as a hugely successful (31)........... .. but until
then there are bills to be paid. The Sunday color (32)....... are filled with feature
articles. They all have to be written, and they often written by freelancers. Offer
your servic es. Know two languages? Consider doing the (33 )......... for foreign
films and TV programs. They're usually commissioned on a film-by-film basis.
Enjoy politics? Offer to help a political party to produce its next (34)................-
for a small fee, of course. Like technology? Someon e has to be write the
instructions or (35)..............that comes with every piece of equipm ent we buy. It
125
could be you! Set y ourself up as a gossip (36 )............. . film (37 ).............or book
(38 )............... for a small local newspaper. Once you ’ve got some pieces behind
you, you can move on to a bigger paper, or even a national (39 )................... And
when you've made a few contacts and had a few things published, there’s a fair
amount of work out there for (40 )................ - people who write celebrities’
autobiographies for them.
III. 41-50 ERROR CORRECTION
The following passage contains 10 errors. Identify and correct them.
NO C ELLPHONE RESTRICTIONS!
It’s seeming that I
Line 1 constantly hear the same
thing: "Cell phones are
2 dangerous. We need to
severely restrict them.
People are dying
because of cell phones."
3 Well, I'm thinking cell
phones themselves
aren't the problem. I'm
4 completely opposed to
restrictions on them,
and here's why. First, people
5 say cell phones are
dangerous to health,
6 so they should be limited.
Supporters o f this idea say
7 there are being studies
showing that cell
phones produce radiation
8 that is harmful to users. 1
think this is
nonsense. There hasn't been
9 any real proof. It sounds jus
t another
study that ultimately doe sn't
10 mean anything. Second, a
lot of teachers
are proposing that we not
11 allow cell phones in classes
because they're
a distraction. 1 feel pretty
12 angrily about this. Here's a
good example.
Two weeks ago in my
13 history class, one of the
students was having
her cell phone on because
14 her mother was really sick
and might need a
ride to the hospital. The
15 ' student's mother couldn't
contact anyone else.
In fact, the student's mother
16 did call, and the student
found someone
to help her mother. What i f
17 her cell phone hadn't been
on? The teacher
18 would have felt pretty bad.
Third, people argue that
19 using a cell phone while
driving is dangerous.
I disagree. It's no more safe
20 than turning on the car radio
or eating a
sandwich. People do those
21 things when they drive. The
law says you
must have one hand on the
22 steering wheel. It's possible
to use a cell
phone correct with one
23 hand. I use my cell phone
carefully; I always
keep one hand on the wheel.
24 Maybe there should be
training in ways to
use a cell phone good, but
25 we shouldn't prohibit to use
cell phones in
cars. This has always been a
26 free country. I hope it stays
that way
1. _______________ 2 .__________________
4. _________________ 5 .__________________
7. ________________ 8 ._______________
3 ._______________
6 ._______________
9. _________
10. ______________
126
IV. 51-60 SENTENCE TRANSFORMATION
I. Finish each of the sentences in such a way that it means the same as the
given one.
51. The singer has strongly and publicly opposed the war.
The singer has been a............................. of the war.
52. We carried out market research to see whether the public would like our new car.
We carried out market research t o .......................... to our new car.
53. The rock star made an emotional request for the earthquake victims, which
produced a huge response.
The rock star’s ......................... for help for the earthquake victims produced a
huge response.
54. The ream will post accounts of their progress on their website every few days.
The team will post................................ on their progress on their website.
II. Rewrite the following sentences using the given words.
55. After 1 had introduced my guests to each other I made a long speech on the
current changes in the computer technologies.
Hav ing.......... .....................................................................................................
56. George won't lend his tape recorder to you if you don't promise to bring it
back by Saturday. (UNLESS)
57. Jerry had terrible problems with solving the riddle. (HARDLY)
58. Our representatives have been criticizing the new concept. (CRITICAL)
59. It doesn't make any difference if they paint the board white or yellow. (M ATTER)
60. Finally, the new prime minister has been appointed. (LAST)
TRƯỜNG T HPT CHU YÊN PHAN NGỌC HIỂ N - CÀ MAU
A. MULTIPLE CHOICE (40 PTS)
I. WORD CHOICE (5 pts)
Choose the best options to complete the following sentences.
1. The sky darkened and there was a dis tan t____________ of thunder.
A. clap B. clang c. groan D. flash
2. Vietnam ____________ war against French colonial empire has gone down
in history as one of the most epic, destroying the harrowing shackles of
tyranny after nearly a century.
A. emancipation B. remission c. liberation D. salvation
127
3. Everyone was shocked to hear that a politician of h is ____________ would
stoop so low.
A. credence B. stature c. guile D. affinity
4. A new treatment ______________ , it became less prohibitive and more
effective to deal with patients infected with tuberculosis.
A. tainted B. assorted c. repudiated D. effected
5. On 6th August 1945, an atomic bomb was dropped on Hiroshima, killing
thousands of citizens and alm ost____________ the city.
A. overruling B. annulling c. eradicating D. annihilating
6. His smirk suggested some viciou s_____________ , which terrified everyone
at the meeting.
A. subtleties B. allusions c. insinuations D. inertia
7. She w a s ____________ with guilt when she realized that the accident had
been her fault.
A. consumed B. ravaged c. devoured D. dazzled
8. That my br ot he r_____________ boasts about his achievements is absolutely
annoying!
A. inexorably B. incessantly c. intricately D. intrepidly
9. She lifted it over the fence and set of f across the little m ead ow ,____________
speed and thoroughly enjoying it.
A. gathering B. collecting c. consuming D. firing
10. One o f Kim's secretarial works include taking th e_________ of the meeting.
A. gist B. crux c. minutes D. fundamentals
11. GR AMM AR AND STRUCTURES (5 pts)
Choose the best options to complete the following sentences.
1. Everything looks very positive for the com pany,_____ the current investors
do not default on their agreements.
A. assuming that B. whether c. whereas D. as if
2. Probably no man had more effect on the daily lives of most people in the
United State s_____ .
A. as Henry Ford, a pioneer in automobile production.
B. rather than Henry Ford, a pioneer in automobile production,
c. than did Henry Ford, a pioneer in automobile production.
D. more than Henry Ford, a pioneer in automobile production.
3. ___________ the invention of the steam engine, most forms of transport were
horse-drawn.
A. With refere nce B. Akin c. Prior to D. In addition to
4. Suppose s h e _________ that outrageous story circulating around the office;
she'd be furious!”
A. has heard B. were heard c. would hear D. had heard
5. I'd rather y ou_____ _____ a noise last night; I couldn’t get to sleep.
A. w ouldn’t make B. did n’t make
c. have n’t made D. ha dn't made
128
6. A new generation o f performers,____those who by now had become household
name, honed their skills before following the same path onto television.
A. no less talented than B. together with talented with
c. along with talented with D. having been more talented than
7. We all wished to be tre ate d___________ .
A. as equal B. as equally c. as equals D. as equal as
8. _________ the US superiority at that time, it was probable that any threatened
US response would have deterred the Soviet Union.
A. If B. Given c. Although D. Since
9. Business has been thriving in the past year. Lo ng______ it continue to do so.
A. could B. does c. may D. might
10. The m an __________ of carrying out the burglary was relea sed__________
by police.
A. to be suspected - following questioning
B. having been suspected - following questioned
c. suspected - following questioning
D. being suspected - following questioned
III. PHRASAL VERB AND PREPOSITIONS (5 pts)
1 .The love of life shone______ the author’s book, giving me as much inspiration
as I could ever ask for.
A. through B. over c. into D. upon
2. You shouldn’t t ak e______ more than you can handle, otherwise you ’ll suffer
from stress.
A. on B. over c. out D. away
3. He’ s so stubborn and stupid. I just couldn’t g e t________ him that she can
never make money from gambling.
A. over B. out o f c. across to D. out of
4. The success of our project hi ng es _________ Mike’s ability persuade the
locals to move to the renewal quarter.
A. in B. on c. about D. over
5. The government’s plans to reduce crime c am e__________for a lot o f criticism
from freedom groups.
A. across B. around c.in D. with
6. Before they open the new factory, a lot o f the young people round here were
__________ the dole.
A. on B. in c. over D.abo ve
7. The two countries met at the conference to iron _ _____ their differences.
A. on B. out c. over D. into
8. He tried to p ap er_______ the country’s deep-seated problems.
A. over B. with c. into D. down
9. Linda chats so much, she could talk the hind le g_______ a donkey.
A. up to B. over c. off D. under
10. I'm going to put my he ad _______ for a while as I feel very tired.
A. down B. over c. up D. through
129
IV. CO LLOCATION AND IDIOMS (5PTS)
Choose the best options to complete the following sentences.
1. 1 had to get through a lot of____tape, but I finally got the documents I needed.
A. red B. blue C. link D. yellow
2. The hospital staf f pulled out all th e_______ to make sure the children had a
wonderful day.
A. roadblocks B. barricades C. plugs D. stops
3 .Neither side is prepared to _____ an inch in the negotiations.
A. stir B. budge C. push D. bend
4. Congressman Saunders fired the opening_______ during a heated debate on
capital punishment.
A. salvo B. barrage C. cannonade D. burst
5. I like the way people here always queue up. Back home we just push and
shove, and the devil take t he________ !
A. outermost B. foremost c. hindmost D. utmost
6. The manager attached himself to the luncheon club and became a
'
fixture there.
A. perennial B. enduring C. stable D. permanent
7. I was so hungry, and that meal was absolutely delicious! It was just what the
' ordered.
A. scientist B. doctor C. expert D. healer
8. Although the Government has increased allocations to the social sector by as
much as 40 per cent, State funding still falls ________ short of needs.
A. well B. totally C. severely D. abjectly
9. Last wee k's violence was _______ condemned by foreign governments.
A. grimly B. roundly C. roughly D. bitingly
10. She has n't had an accident yet but she has had a number o f........Shavers.
A. narrow B. near c. close . D. tiny
V. READING COMPREHENSION (10 pts)
PASSAGE A
Question 56 - 65. Read the following passage and mark the letter A, B, c, or
D on your answer sheet to indicate the best answer to each of the following
questions.
Color in textiles is produced by dyeing, by printing, or by painting. Until the
nineteenth century, all dyes were derived from vegetable or, more rarely, animal
or mineral sources. Since madder plants could be grown practically everywhere,
the roots of some species of the madder plant family were used from the earliest
period to produce a whole range of reds. Red animal dyes, derived from certain
species of scale insects, were also highly value from ancient times through the
Middle Ages. Blues were obtained from woad, a plant common in Europe and
130
also used in the Near East from the beginning of the Christian era Before the
first., nonfading “solid” green was invented in early nineteenth century, greens
were achieved by the overdyeing or overprinting of yellow and blue. However,
yellow dyes, whether from weld or some other plant sources such as saffron or
turmeric, invariably fade or disappear. This accounts for the bluish tinge of what
were once bright greens in, for example, women tapestry.
The range of natural colors was hugely expanded and, indeed, superseded by
the chemical dyes developed during the eighteen hundreds. By 1900, a complete
range of synthetic colors had been evolved, many o f which reached a standard of
resistance to fading from exposure to light and to w ashing that greatly exceeded
that of natural dyestuffs. Since then, petroleum industry has added many new
chemicals, and from these, other types of dyestuffs have been developed. Much
of the research in dyes was stimulated by the peculiarities of some of the new
synthetic fibers- Acetate rayon, for example, seemed at first to have no affinity
for dyes and a new range of dyes had to be developed; nylon and Terylene
presented similar problems.
The printing of textiles has involved a number of distinct methods. With the
exception of printing patterns directly onto the cloth, whether by block, roller, or
screen, all of these are based on dyeing; that is, immersion of the fabric in a dye bath.
1. The passage mainly discusses t he______ ' .
A. Development of synthetic colors for textiles during the nineteenth century
B. Advantages o f chemical dyes over dyes derived from plants and animals
c. Differences between dyeing textiles and printing items
D. History o f the use of natural and chemical dyes to color textiles
2. According to the passage, what was the source o f most textile dyes that were
used before the nineteenth century?
A. Animals. B. Minerals. c. Plants. D. Chemicals.
3. What was the advantage o f using madder plants for different shades of red?
A. It was possible to cultivate madder plants in almost every location.
B. Madder plants produced brighter colors than other plant sources.
c. Plant sources produced more lasting colors than other plant sources.
D. Dyes derived from the madder plants were easier to work with than other dyes.
4. One disadvantage o f green dyes before the nineteenth century w as ________ .
A. The yellow dyes were expensive
B. They lost their original color
c. The blue dyes involved lost their color
D. The final color varied
5. The green areas in women tapestries developed a bluish tinge because_____
A. A dark er color, like blue, dominates a light color, like yellow
B. Light changed some of the green dye used in the tapestries to blue
c. The yello w dye that was used in the tapestries had faded
D. The dyes used to color woven tapestries were made from minerals
131
6. Red dyes came mostly f ro m ___________ .
A. animals B. plants and insects
c. indigo D. minerals
7. How did chemical dyes compare to natural dyes?
A. The chemical dyes had less attractive colors.
B. The chemical dyes were less easy to use.
c. The chemical dyes lost their brightness more quickly when exposed to light.
D. The chemical dyes held up better after washing.
8. The chemical dyes keep color because they are less prone to _____________ .
A. sunshine B. wind c. restriction D. wear
9. According to the passage, what problem led to the development of the new
dyes after 1900?
A. Previously d eveloped dyes did not work on new types o f fibers
B. Dyes derived from petroleum caused damage to new synthetic fibers
c. New synthetic fibers lost required brighter colors than natural fibers did.
D. New fabrics easily lost the ir colors when washed.
10. W hat does the author mean by “block, roller, or screen” ?
A. To give examples of textile printing techniques that are based on dyeing.
B. To argue that all methods printing patterns onto textiles involve dyeing,
c. To emphasize the variety of special tools used the process o f dyeing textiles.
D. To give examples of textile printing techniques.that do not involve dyeing.
PASSAGE B:
MORE GOOD THAN EVIL?
It is a comm only held belief that today’s teens are in trouble. They spend
hours communicating via social network sites instead of socializing in person
and send countless text messages that are in a virtu ally unrecognizable
language. In a study analyzing today’s youth in the digital world, renowned
scientist, Susan Greenfield, laments: “We are raising a generation of children
who are shallow, thrill-seeking and in danger of detaching themselves from
reality.” In spite of such pessimistic beliefs, recent research seems to indicate
that social network sites may not be as detrimental as some may think.
A crucial point is that we need to maintain a broader perspective. New
technologies have always provoked generational panic, which usually has more
to do with adult fears than any real harm they may do. In the 1930s, parents
worried that radio was taking over the lives o f their children. In the 60s, the great
danger was the television and then in the 80s, the Sony Walkm an was claimed to
be turning teens into mindless zombies.
In fact, social scientists who study young people have found that technology
and the digital world can essentially benefit today's youth. It seems that if teens
use a lot of social media, it has no negative effect on their engag ing in face-toface contact.
Actually, the evidence suggests that the most avid texters are also
132
the kids most likely to spend time with friends in person. One form of socializing
doesn 't replace the other. It augments it. Then, as the young get older and are
given more freedom, they often ease up on social networking. Early on, the web
is their own personal space, but by their late teens, it is replaced as they acquire
greater independence.
But isn 't all this short-form writing eroding language skills? Studies of firstyear college
papers frorrj 1917 is show that this is not the case. The rates of
gram mar and vocabulary errors by these freshmen in their compositions were the
same as in the ones written by their modern counterparts. There is one essential
difference, however. Student essays have blossom ed in size and complexity.
They are now six times longer and offer arguments supported by a wealth of
evidence. Why? Because computers have vastly increased the ability o f students
to gather research, consider different points of view and write more analytically.
If truth be told, the online world also offers kids remarkable opportunities to
become literate and creative because young people can now publish ideas not
jus t to their circle of friends, but to the whole world. And it turns out that when
they write for strangers, it makes them w ork harder, push themselves further, and
create powerful new communicative forms.
1. What is meant by the phrase virtually unrecognizable in the first paragraph?
A. Teenagers use foreign languages when they speak or write.
B. Teenagers are very creative when they communicate with one another,
c. Teenagers enjoy taking on a different identity when they communicate.
D. The language used in text messages has been greatly altered.
2. Why is Susan Greenfield quoted in the first paragraph?
A. She has teenage children who use computers.
B. She strongly contradicts what society believes.
c. Her research supports that teens are negatively affected by today’s technology.
D. She believes more information is needed to draw conclusions.
3. According to Susan Greenfield, what may be a consequence, of teens using
technology?
A. They may become overly cynical about their lives.
B. They m ay not be connected to what is happening around them,
c. They may not find pleasure in exciting activities.
D. They may not grow up into mature adults.
4. Why does the author use examples from past decades?
A. to prove that technology has always been harmful
B. to argue that new inventions always seem suspicious
c. to show that teenagers will never obey th eir parents
D. to present a trend that is getting more dangerous
5. The author implies that a teen who uses a cell phone frequently is ____
A. less athletic B. a poor student
c. highly introverted D. more sociable
133
6. The word augments in the third paragraph is closest in meaning to______
A. substitutes for B. adds to c. explains D. decreases
7. What happens when teenagers get older?
A. They d on 't need personal space anymore.
B. Their studies become more important to them than their social lives,
c. The digital world becomes increasingly important to them.
D. They do not participate in social networks as much.
8. What does the word counterparts in the fourth paragraph refer to?
A. compositions B. first-year college students
c. grammar and vocabulary errors D. language skills
9. How does a modern college fresh man’s paper compare to one from 1917?
A. It has almost no linguistic errors.
B. It has the same amount of content.
c. It has a less complicated writing style.
D. It is based on more information. ’ >
10. What is the author’s view of social networks?
A. Th eir access must be strictly controlled by parents.
B. They play a positive role in young people’s lives.
c. They are detrimental to young people's relationships.
D. They have evolved faster than most people realize.
VI. GUIDED CLOZED TEST(1 OPTS)
PASSAGE 1
Read the passage carefully and choose the answer (A, B, c or D) that best
fits each space.
It only requires the completion of the reconstruction of the human genetic
map for a whole host of hereditary disease to b e ___________ (1). Originally,
it was forecast th at the venture would take until the beginning o f the 21st century
to b e ___________ (2). At present, it is clear that the task can be finished much
earlier. Hundreds of scholars have gone to ___________ (3) to help unravel the
mystery of the human genetic structure with an ardent hope for __________ (4)
mankind from disorders such as cancer, cystic fibrosis or arthritis.
The progress in this incredible undertaking is conditioned by an accurate
interpretation of the information contained in the chromosomes forming the
trillions of the cells in the human body. Locating and characterizing every single
gene may _________ (5) implausible an assignment, but very considerable
_________ (6) has already been made. What will you know my now is that the
hereditary code is assembled in DNA, some of which may be diseased and
________ (7) to the uncontrollable transmission of the damaged code from
parents to their children? Whereas work at the completion of the human genom e
may last for a few years more, notions like gene therapy or genetic engineering
d o n 't_________ (8) much surprise any longer. Their potential application has
already bee n_________ (9) in the effective struggle again st many viruses or in
134
the genetic treatment of blood disorders. The hopes are, then, that hundreds of
maladies that humanity i s _________ (10) with at the present might eventually
cease to exist in the not too distant future.
1. A. terminated. B. interfered c. eradicated D. disrupted
2. A. accomplished B. discharged c. dismantled D. extermina
3. A. maximum B. utmost c. supreme D. extremes
4. A. liberating B. surviving c. insulating D. averting
5. A. sound B. hear c. voice D. perceive
6. A. headline B. heading c. headway D. headship
7. A. amiable B. conceivable c. evocative D. conducive
8. A. evoke B. institute c. discharge D. encourage
9. A. examined B. inquired c. accounted D. corrobora
10. A. aggravated B. teased c. persecuted D. plagued
Passage 2
Very few of US would admit putting much trust in horoscopes and the fact that
the movements of astronomical bodies _________ (1) to earthly occurrences
affecting peoples' everyday lives.
We all know about the zodiac signs which reflect the position of the sun, the
moon and the planets at the moment of a man's birth and about the peculiar
characteristics_________ (2) to them by astrologers. We say we will take these
phenomena with a pinch of salt while we k ee p________ (3) our eyes over them
in every tabloid we lay our hands on. Most frequently, we expect horoscopes to
predict the future, t o ____________ (4) our optimistic mood with a piece of
comforting information or to _____________ (5) our ego by confirm ing the
superlative features that we tend to attribute to our zodiacs.
However, there's no scientific evidence t o ___________ (6) the assumption
that human existence is so clos ely__________ (7) with the parameters of the
celestial bodies. Our curiosity in horoscopes may, th en ,_________ (8) our sheer
fascination with the unexplained or the unpredictable as well as in the enticing
insight into the future that the horoscopes offer, thus establishing the sense of our
__________ (9) an extreme power over our own lives. An additional explanation
is that humans tend to have a so ft ____________ (10) for any form of flattery,
which is the fact to which astrologers and the horoscope writers seem to attach
the greatest deal o f weight.
1. A. rely B. correspond c. match D. compare
2. A. identified B. associated c. incorporated D. ascribed
3. A. running B. sending c. fixing D. putting
4. A. restore B. adjust c. upgrade D. reassure
5. A. boost B. escalate c. revitalize D. improve
6. A. conclude B.co ncede c. corroborate D. confound
7. A. fused B. adhered c. coalesced D. intertwine
135
8. A. stem B. crop c.r ear D. dawn
9. A. disposing B. wielding c. effecting D. committin
10. A. pit B. dot c.s pot D. nick
B. WRITTEN TEST (70 pts)
I. OPEN CLOZE TEST (20 pts)
Read the texts below and think of the word which best fits each space. Use
only ONE WORD for each space.
PASSAGE A
INDOOR AIR POLUTION
The citizens (0)....ơ/...our major European countries think the (1).................
of climate change such as severe floods and storms are already affecting them,
according to a major new polling study.The research dispels the idea that global
warming is widely seen as a future problem, and also shows strong support for
action to tackle global warming, (2) ........................ subsidies for clean energy
and big financial penalties for nations that refuse to be part of the international
climate deal signed in Paris in 2015 - (3 )............... US president Donald Trump
has threatened. There was also strong support for giving financial (4) ..................
to developing nations to cope with the impacts of climate change.Renewable
energy was viewed very positively in all nations, but fracking had little support,
with ju st 20% of people seeing it positively in the UK, 15% in Germany and 9%
in France. Nuclear power was also unpopular: only 23% of those in France,
(5).......................... it supplies the vast majority of electricity, have a favourable
opinion. Overw helming majorities of people in the UK, Germany, France and
Norw ay said climate change was at (6)........................ partly caused by human
activities, such as burning fossil fuels. But only a third thought the vast majority
of scientists agreed with this, despite about 97% o f climate scientists doing so.“It
is encouraging to see that most people in this very large study recognise that
climate change is happening, and that support for the need to tackle it remains
high (7) .............................. the people we surveyed,” said Pro f Nic k Pidgeon at
Cardiff University, who led the international project.He said the firm backing of
the public could be important in the light of Trump’s opposition to climate
action: “(8 )........................ the recently shifting political mood in some countries,
climate policy is now entering a critical phase. It is therefore even (9 )................
important that the public’s clear support for the Paris agreemen t is carried
(10 )....................by policymakers Europe and worldw ide.”
PASSAGE B
THE TRIUNE BRAIN
The first of our three brains to evolve is what scientists call the reptilian
cortex. This brain sustains the elementary activities of animal (1).............such as
respiration, adequate rest and a beating heart. We are not (2).................. to
consciously “think” about these activities. The reptilian cortex also houses the
136
“startle centre”, a mechanism that facilitates swift (3).............. to unexpected
occurrences in our surroundings. That panicked lurch you experience when a
door slams shut somewhere in the house, or the heightened awareness you feel
when a twig cracks in a nearby bush while out on an evening stroll are both
examples of the reptilian cortex at work. When it comes to our interaction with
others, the reptilian brain offers up only the most basic impulses: aggression,
mating, and territorial defence. There is no great difference, in this (4)........... .
between a crocodile defending its spot along the river and a turf war between
two urban gangs. (5)........... the lizard may stake a claim to its habitat, it exerts
total indifference toward the well-being of its young. Listen to the anguished
squeal of a dolphin separated from its pod or witness the sight of elephants
mourning their dead, however, and it is clear that a new development is at play.
Scientists have identified this as the limbic cortex. Unique (6 )........ mammals,
the limbic cortex impels creatures to nurture their offspring by (7) ...........
feelings o f tenderness and warmth to the parent when children are nearby. These
same sensations also cause mammals to develop various types of social relations
and (8)................ networks. When we are with others of “our kind” - be it at
soccer practice, church, school or a nightclub - we (9) ..................... positive
sensations of togetherness, solidarity and comfort. If we spend too long away
from these networks, then loneliness (10)........ . in and encourages US to seek
companionship.
II. WORD FORM ATION (20 pts)
PART 1: Complete each sentence, using the correct form of the word in
parentheses.
1. I'm not against_______ , but obviously we all want to avoid animals suffering
unnecessarily. (SECTION)
2. The administration announced that the Ư.S. would no longer produce
____________ landmines or acquire new ones, including replacing expiring
munitions in its stockpile. (PERSON)
3. And lastly, it provided the authorities with an opportunity to dispose of
troublesom e true believers or neighborhood__________ (CONTENT)
4. She hopes to _______ her success as a model into an acting career. (LAY)
5. M ohammed Zardah, 26, a slim ,_______ man with an academic mien, studied
computer engineering in Damascus, he says. (SPECTACLE)
6. Hunger and a _________ meal did not sit happily side by side. (SLAP)
7. The machine must b e ______________ , executing as fast as a mower is able
to cut. (EXPEDITE)
8. ________________are medicines that lower the chance o f blood clotting.
(COAGULATE)
9. With the great achievement of U-23 Vietnam team, many international
newspapers w ro te__________ praising our heroes. (COME)
137
10. In the countryside, farmers try to _______ their land with a view to keeping
out th eir neigh bours' cattle. (MARKER)
PART 2: Complete the passage with appropriate forms from the words given
in the box. 1
produce utile replace inf ect
lum inaire extract temp orate orient
The natural uses of bioluminesc ence vary widely, and organisms have learnt
to be very creativ e with its use. Fireflies employ biolumines cence primarily for
(1) ....................m eans - their flashing patterns advertise a firefly's readin ess to
breed. Some fish use it as a handy spotlight to help them locate prey. Others use
it as a lure; the anglerfish, for example, dangles a (2) ..................... flare that
draws in gullible, small er fishes which get snapped up by the anglerfish in an
automated reflex. Sometimes, biolumin escence is used to resist predators.
Vampire squids eject a thick cloud of glowing liquid from the tip of its arms
when threatened, which can be (3) ..................Other species use a single, bright
flash to (4) ................ blind their attacker, with an effect similar to that of an
oncoming car which has not dipped its headlights.
Humans have captured and (5) ............. bioluminescence by developing, over
the last decade, a technology known as Bioluminescence Imaging (BLI). BLI
involves the of a DNA protein from a bioluminescent organism, and then the
integration of this protein into a laboratory animal through transgeneticism.
Researchers have been able to use luminised pathogens and cancer cell lines to
track the (6) ........... spread of (7).............. and cancers. Through BLI, c ancers and
infections can be observed without intervening in a way that affects their
(8) ................ developm ent. In other words, while an ultra-sensitive camera and
bioluminescent proteins add a visual element, they do not disrupt or mutate the
natural processes. As a result, when testing drugs and treatments, researchers are
permitted a single perspective o f a therapy's progression.
Once scientists learn how to engineer biolumin escence and keep it stable in
large quantities, a number of other human uses for it will become available.
Glowing trees have been proposed as ( 9) .............. for ele ctric lighting along busy
roads, for example, which would reduce our dependen ce on (10).................. .
energy sources.
III. ERROR CORRECTION (10 pts)
The following passage contains 10 errors. Identify and correct them.
It is not easy to have a
rational discussion with
people about the nature
of their language. They feel
that the language belongs to
them, and
1
they are entitled to hold cut
and dry opinions about it.
And when
opinions differ, emotions
run high. Arguments can
easily stem from
138
5
10
15
20
minor points of usage as from major education policies. In English,
the origin of many popular misconceptions lies in the work of the
linguists of the eighteenth century who first attempted to codify the
English grammar. Unfortunately, they worked under the premise that
English gramm ar is derived from Latin grammar and that the rules of
the latter are to determine the former. It was this fundamentally
misunderstanding that resulted in the absurd but time-honored 'neverend-a-sentence-with-a-
preposition' type of rule that many people still
cling to. These days, many people complain that the Internet is the
source of much unforgivable distortion of English, and such the ease
and speed of email communication engenders a lazy approach to
writing. This is possibly a short-sighted view: perhaps we should be
more broad-minded and view such changes as potential enrichment
other than corruption of the language. Perhaps those who argue it is
only the latter are guilty of stick-in-the-sand mentality which is often
not confined to their own language, The American linguist Leonard
Bloomfield tells the story o f a doctor who was so firm in his view that
the American language Chippewa had only few hundred words. When
Bloomfield tried to dispute the point, the doctor had no thoughts of
losing down. He turned away and refused to listen.
1._______________ 2. • 3 .__________ _
4Ỵ_______________ 5. _________ 6._______________
7._______________ 8._______________ 9. _________
10.________ _
IV. SENTENCE TRANSFORMATION (20PTS>
For each of the sentences below, write a new sentence as similar as possible
in meaning to the original one, using the word given.
1. Going to and fro with all the cases is what I can’t stand about holidays.
-> It’s al l............... .-............................ .................................................................
2. The village shop is now being managed by a national supermarket chain.
—> A national supermarket chain has taken...... .................................................
3. This door is an emergency exit and must never be locked for any reason.
—> O n ...............................................be locked because it is an emergency exit.
4. Y olan da’s family persuaded her to enter the competition.
—» Yolanda was talked.......................................................... the competition by
her family.
5. So many people were really delighted when the government lost the election.
(JUBILATION)
139
6. The board had a secret meeting in order to discuss changes in company policy.
(DOORS) .
- > .................................................................................. ........................................
7. Peter was in trouble with his boss because he did n’t finish an important
project by the deadline. (HOT)
- > ..................................................................................................... .......................
8. She told everyone that she had been fired by the company. (SACK)
—> She let.................................................................................................................
9. Should there be a problem, contact USat all costs. (LINE)
—> In t h e .................................................................................................................
10. Although the manager is sluggish, he is a smooth speaker. (GIFT)
—> Sl ug gish ............................................................................................................
TRƯỞNG THP T CHUYÊN THĂNG LONG - OÀ LẠT - LÂM ĐỒNG
A. M U LTIP LE CH O IC E QUESTIONS
I. WOR D CHOICE
Choose the best answer to complete each o f the following sentences. (5 pts)
1. He was a bso lutely ___with anger when he found that I had scratched his car.
A. burned B. carmine c. fickle D. livid
2. He glanced at Juliet accusingly and she loo ked_____ abashed.
A. suitably B. completely c. utterly D. absolutely
3. Everyone was th er e_____ Linda. She was too sick to come.
A. save B. take «■ c. make D. have
4. She did six hours’______studying a day for her exam.
A solid B heavy c strong D big
5. Are you having a _______ exam before you sit the actual exam in June?
A. fake B. mock c. false D. dress
6 .1 was kept awake for most o f the night by th e ______o f a mosquito in my ear.
A. whine B. moan c. groan D. screech
7. He decided to withdraw from the powerboat race as he had a (n )___________
of danger.
A. interim B. foreboding c. dearth D. prediction
8. It was difficult for the lecturer to _______ his voice to the back of the hall.
A. shout B. raise c. project D. deliver
9. Many children who get into trouble in their early teens go on to become
offenders.
A. co nsistent B. insistent c. persistent D. resistent
10. He would ne ve r_______ his pride and say that he was sorry.
A. taste
B. swallow c. sip D. crunch
140

II. GR AMM AR & STRUCTURES


Choose the best answer to complete each of the following sentences. (5 pts)
1. “It is raining outside" — “_________ ”
A. So is it B. So it is
2. They must have gone a wa y,______
c. So it does D. Is it so?
A. shou ldn't they B. haven’t they
_______ the meeting began.
A. After we have sat down
c. Our having seated
c. m ustn't they D. didn't they
B. All o f us having taken the seats
D. Once we had seated
with being so busy at work and at home, she became increasingly
tired and bad-tempered.
A. How B. What
The police took him into custody
A. as long as
c. the moment when
c. Which D. Where
_ he disembarked from the plane.
B. while
D. the instant
6. _______ earning all that money if you do n't have time to enjoy it?
A. Is it to the good B. What's the good of?
c. Is it all to the good D. Is it for good?
7. “Was that the new schoolmaster who walked by?” - “_______ ”
A. It must be that
c. He must be
8. It is urgent that the b os s____
A. called B. call
B. It must have been
D. This must have been
a meeting to discuss the problem,
c. to call D. calls
____ the truth about his corporation.
B. the director did tell
D. that the director told
9. It was at the press c onference ____
A. did the director tell
c. where the director told
10. “I think we ought to see the rest o f the exhibition as quickly as we can,
_____. that it closes in half an hour.”
A. granted B. assuming c. given D. knowing
III. PREPOSITIONS & PHRASAL VERBS
Choose the best answer to com plete each of the following sentences. (5 pts)
1. Doctors are o fte n________ to accidents in rural areas.
A. called up B. driven out c. called out D. rung up
2. I f you never put oil in your engine, one day it will ________ .
A. flake out B. shut down c. go off D. seize up
3. To get his proposal accepted, the Finance Manager had t o __________ heavy
pressure from colleagues.
A. fend off B. laugh off C. send off D. push off
4. The new regime determined to _____ compulsory military service.
A. stop o f B. end up c. phase out D. break o ff
5. After a fall in profits, the company dec ide d_____ the hotel business.
A. to pull out o f B. to back off from
C. to take out o f D. to keep away from
141
6. S he’s ru ng _____ . I must have said something to upset her.
A. up B. round c. back D. off
7. Our ciass mon itor had a brilliant speech yesterday although he sp oke____ _
the cuff then
A. ab out B. with c. off D. on
8. Cali gambled away all his father’s legacy and finally h e _____ his family
_____ misery.
A. p u t/ in B. th re w /in to c. made / in D. led / to
9. The you nge r sons conside r them selves to have been ro bb ed ________ their
rightful inheritance.
A. by B. with c. around D. o f
10. He 's sometimes bad tempered but h e’s a good f ell ow _____ heart.
A. by B. at c. with D. in
IV. COLLOCATIO NS & IDIOMS
Choose the best answer to complete each of the following sentences. (5 pts)
1.1 d on 't trust him and it really goes against th e_______ to give him the money.
A. cloak B. grain c. rice D. grapevine
2. Did you see Jonathan this morning? He looked li ke_____ It must have been
quite a party last night. ?
A. a w et blanket B. a dead duck
C. death w armed up D. a bear with a sore head
3. In spite of working their fingers to the ___ , all the staff were made redundant.
A. nail B. edge c. flesh D. bone
4. The general was convinced that if his army could make the fi rs t________
strike, they would be able to win the battle.
A. enigmatic B. pre-emptive c. showdown D. rudimentary
5. Royce is unlikely t o ________ any surprises.
A. springs B. leaps c. sources D. traces
6. Jud ging from the noise it is making, the washing machine is on its las t______
A. grasp B. legs c. resort D. breath
7. As a poet, I think s he ________ comparison with the greatest this century.
A. m akes B. stands c. leads D. matches
8. It’s time we had a ________ talk with each other in an effort to clear the air.
A. heart to heart B. eye to eye c. face to face D. cheek to cheek
9. I don't know what our guests will be wanting to do this weekend. We'll have
to ________ .
A. play it by ear B. bend our ears about it
c. be our on our ear D. turn a deaf ear to it
10. Oil spills w ill_______ even the healthiest o f marine ecosystem.
A. play havoc on B: break ground with
c. pay the consequences for D. take their toll on
142
V. RE AD IN G CO MPR EH EN SION
READING 1
You are going to read an extra ct from a nov el. Fo r qu estio ns 1-1 0, choose
the a ns wer (A , B, c or D) w hich you think fits best according to the text.
A folk culture is small, isolated, cohesive, conservative, nearly self sufficient
group that is ho mo geneou s in custom and race, with a strong family or clan
structure and highly developed rituals. Order is maintained through sanctions
based in the religion or family, and interpersonal relationships are strong.
Tradition is paramount, and change comes infrequently and slowly. There is
relatively little division of labor into specialized duties. Rather, each person is
expected to perform a great variety of tasks, though duties may differ between
the sexes. Most goods are handmade, and a subsistence economy prevails.
Individualism is weakly developed in folk cultures, as are social classes.
Unaltered folk cultures no longer exist in industrialized countries such as the
United States and Canada. Perhaps the nearest modern equivalent in AngloAmerica is the
Amish, a German American farming sect that lar gely renoun ces
the products and labor saving devices of the industrial age. In Amish areas,
horse-drawn buggies still serve as a local transportation device, and the faithful
are not permitted to own automobiles. The Amish’s central religious concept of
Demut, 'humility ', clearly reflects the weakness of individualism and social class
so typical of folk cultures, and there is a corresponding strength of Amish group
identiy. Rarely do the Amish marry outside their sect. The religion, a variety of
the Mennonite faith, provides the principal mechanism for maintaining order.
By contrast, a popular culture is a large heterogeneous group, often highly
individualistic and constantly changing. Relationships tend to be impersonal, and
a pronounced division of labor exists, leading to the establishment of many
specialized professions. Secular institutions, of control such as the police and
army take the place of religion and family in maintaining order, and a moneybased economy
pr ev ails. Because of these contrasts, ‘ popular’ may be viewed
as clealy different from ‘ folk ’. The popular is replacing the folk in industrialized
countries and in many developing nations, Folk-made objects give way to their
popu lar equivalent, usually because the popular item is more quickly or cheaply
produced, is easier or time saving to use, or lends more prestige to the owner.
1. What does the passage mainly discuss?
A. Two decades in modern society. »
B. The ch aracteristics o f ‘fo lk ’ and ‘po pu lar’ societies.
c. The influence of industrial technology. ’
D. The specialization of labor in Canada and the United States.
2. The word ‘ho mog en eo us ’ in the first paragraph is closest in meaning t o .......
A. uniform B. general c. primitive D. traditional
3. Which of the following is typical of folk cultures?
A. There is a money-based economy.
143
B. Contact with other cultures is encouraged,
c. Social change occurs slowly.
D. Each person develops one specialized skill.
4. What does the author imply about the United States and Canada?
A. They value folk cultures. B. They have no social classes,
c. They have popular cultures. D. They do not value individualism.
5. The phrase ‘larg ely reno un ces' in paragraph 1 is closest in meaning t o .......
A. loudly declares B. greatly modifies
c. generally rejects D. often criticizes
6. What is the main source of order in Amish society?
A. The religion B. The economy
c. The clan structure D. The government
7. Which o f the following statements about Amish beliefs does the passage support?
A. A variery of religious practices is tolerated.
B. Individualism and competition are important,
c. Premodern technology is preferred.
D. People are defined according to their class.
8. Which of the following would probably NOT be found in a folk culture?
A. A carpenter B. A farmer c. A banker D. A weav er
9. The word ‘preva ils ’ in the second paragraph is closest in meaning t o ...........
A. develops B. provides c. dominates D. invests
10. Which of the follow ing is NOT given as a reason why folk-m ade objects are
replaced by mass-produced objects?
A. quality B. prestige c. cost D. convenience
RE AD IN G 2
Read the follo wi ng pa ssa ge a nd ma rk the letter A , B, c , or D to indicate the
co rr ec t an sw er fo r eac h of the question s
MORAL BEHAVIOR
The dictionary defines morality as “being in accord with standards of right or
good conduct.” The argum ent over whether our moral behavior is innate of
whether it is developed by our environment and culture has been raging for ages.
Many people feel morality is based on reason, while others feel it comes from
religion or one ’s own spirituality. Biologists believe that hum ans’ tendency to
obey the Golden Rule- “Do unto others as you would have them do unto you”- is
ạ product o f evolution.
■ A) At first, moral behavior seems to oppose the rules of Charles Darwin's
theory of survival of the fittest and natural selection. ■ B) However, researchers
in the field of biology feel that as animals evolved to live in groups, the
propensity to look only after the group’s success as a w hole, every member had
to look only after one's own needs had to fade in order for a group mentality to
emerge. ■ C) To ensure the grou p’s success as a whole, every member had to
look out for the interests of the majority, a concept known as utilitarianism . ■ D)
144
I his is a system of beliefs based on what does the greatest good for the greatest
number of people.
A researc her named Jonathon Haidt at the University o f Virginia believes that
morality is driven by two separate mindsets-one ancient and one modern. Dr.
Haidt declares that the human mind is unaware of the distinction between the
two. The ancient mental system is based on the emotion behind moral behaviors,
which is a type of intuitive sense of what is right and wrong that evolved before
language developed. These are the “gut reactions” people experience in tough
situations that call for quick action. The more modern system of thought came
with the development of language, as people became able to express verbally
why something was right or wrong. The two work together when we are put in
morally compromising situations. When confronted with a moral dilemma, o ne’s
intuition immediately decides what is right or wrong. Rational thought and
judgment about the morality o f an issue follow the decision that one ’s emotional
reaction already made.
Dr. Haidt identified five areas of moral conduct that are common in most
countries and systems throughout the world, and he describes these as the
foundation for all moral behavior. These moral components conceptualize how
people treat others and what is important in being part of a group. Regardless of
their background, religion, socioeconomic status, or educational level, Dr. Haidt
found that the majority of people hold to these moral concepts. The first moral
concept is the prevention of harm. Generally, people believe that it is wrong to
harm another human being or animal for cruel and needless reasons. The second
moral concept is fairness, which holds that all people should be treated fairly.
For instance, people should treat the poor in the same manner as the wealthy, and
the weak the same as the strong. The third moral concept is loyalty to one ’s
group. This entai ls a strong devotion to the values of the group as a whole.
People with strong loyalties believe that adherence to the laws of society is
important because it upholds the integrity of that society. The idea of loyalty is
closely interwoven with the fourth moral foundation: respect for authority.
People who value authority believe in the strength of a governing body and a
strong hierarchy with established roles and rules. The fifth concept involves
upholding high standards of purity. This deals with the way that members of a
group view their bodies. The idea of purity comes into play in the standards of
cleanline ss in society. Daily hygiene routines, eating food that has not been
contaminated in some way, and burial rules and rituals fall into this category.
Dr. Haidt's research concludes that these moral concepts are inherent in our
physical makeup and are learned behaviors, reinforced by our environments
from a very early age. The five moral foundations are interpreted differently
from socie ty to society, and people rate them differently in order o f importance.
While morality may take different forms across the many different cultures of
the world, it remains true that the basic task of morality, restraining selfishness,
is a part o f all humanity's moral behavior.
145
1. Why does the author mention the Golden Rule in paragraph 1?
A. To contrast moral behavior with immoral behavior
B. To prove that people generally know right from wrong
c. To suggest that evolution shaped morality
D. To define the idea o f moral behavior
2. Look at the four squares (■) that indicate where the follow ing sentence could
be added to the passage.
In other words, evolution appears to favour individuals who have learned
how to get what they need in order to survive.
Where would the sentence best fit?
A. 1st square B. 2nd square c. 3rd square D. 4th square
3. According to the passage, morality developed because_____ ______ .
A. people learned to comm unicate using spoken language
B. people had to learn to survive in groups
c. people were born with the ability to know right and wrong
D. people learned moral behavior from their ancestors
4. Which of the following best expresses the essential information in the
highlighted sentence. Incorrect answer choices change the meaning in
important ways or leave out essential information.
A. The earlier mindset based morality on an inborn, emotional understanding
of what is proper and acceptable.
B. Old-fashioned ways of thinking based goodness on the way a person felt
about a situation.
c. Theories about ancient attitudes claimed language was not necessary to
determine moral behavior.
D. Emotions governed the morality of people in ancient times before they
developed a way to communicate.
5. Which of the following is NO T correct about the five moral concepts?
A. They typically develop in sequential order.
B. They serve as the core o f all moral behaviors.
c. They vary in importance from country to country.
D. They explain the various ideas that drive moral behavior,
6. Which of the following can be inferred about Dr. Haidt’s five areas of moral
conduct?
A. They are disputed in various cultures.
B. Aspects o f them appear in the laws o f many countries.
c. Many leaders would likely disagree with their loyalty principles.
D. Th ey are based on innate human tendencies.
7. What can be inferred about humanity as a whole based on Dr. Haidt’s moral
concepts?
A. Morality is a universal characteristic that applies to the whole world.
B. The natural world plays a major role in the developm ent of morality.
146
c . Morality is an instinctive characteristic that humans have from birth.
D. People from different cultures will not value the same principles.
8. According to the passage, mo rality_____________ .
A. conflicts with Darwin’s theory of natural selection
B. goes against the dominant authority
c . seeks to restrict human selfishness
D. includes generosity as an important moral
9. The word entai ls in the passage is closest in meaning to____________
A. follows B. produces c . causes D. involves
10. The word Th is in paragraph 4 refers to____________ .
A. a strong hierarchy B. the fifth concept
c. a governing body D. upholding high standards
VI. CL OZ E TE ST S
Choo se the words or phrases that best fit the blanks to make a complete passage
CL OZ E TE ST 1
There’s extensive historical evidence that our ancestors may have witnessed a
massive invasion of Unidentified Flying Objects (1 )_______ on their territories.
These extraterrestrials are ( 2 )_________ to have come into ( 3 )__________with
the ancient earthly populations and helped them erect numerous magnificient
structures or even establish glamorous empires.
However, the present-day fascination with UFO was only (4 )_________ by
the first widely (5 )_________ American sighting in Idaho in 1947. Since that
time, other countless close encounters have been reported both by highly
credible witnesses such as top-class pilots and less credible ones such as ordinary
civilians. Thousands of people around the world maintain having come close to
the visitors from outer space or to have been ( 6)_________ for a scientific study
inside their flying saucers.
Although most of these accounts have been (7) _________ as fantasy or
hallucinations, there ’s mounting criticism from the public and media for ignoring
the subject for too long. To many people, rejecting even the most inexplicable
sightings or UFO encounters as luminous artificial objects, natural phenomena
like auroras or even as meteorological baloons and satellites seems to be an
irresponsible (8) _________ Most of US would prefer to believe that these
extraterrestrial guests are arriving from some remote galaxies to establish a peaceful
relationship and possibly give US a fair warning against the consequences of our
wasteful lifestyles. Yet, there’s another theory (9 )_________ that the visitors’
attitude towards mankind isn’t so conciliatory and that their sole aim might be
the unscrupulous annihilation of the terrestrial populations. Doubtless, flying
saucers still continue to be observed in many places o f the world ( 10 )_________
the imagination of UFO maniacs. But, a large percentage of such sightings will
remain beyond explanation until more convincing evidence is supplied by the
true experts.
147
1. A. surpassing B. approximating c. transgressing D. encroaching
2. A. reasoned B. alleged c. denoted D. inferred
3. A. touch B. grip c. face D. sight
4. A. discharged B. instigated c. constituted D. devised
c. communicated D.
5. A. proclaimed B. notified
conveyed
6. A. abolished B. abducted c .absconded D. abbreviated
7. A. speculated B. disposed c. repelled D. dismissed
8. A. approach B. bias c. encounter D. manner
9. A. consisting B. specifying c. meaning D. implying
10. A. startling B. triggering c. sparking D. arising
CLOZE TEST 2
Theodor Seuss Geisel, better known as ‘Dr. Seuss’, began writing for children
(1 .) _____ by chance. During a long sea voyage in 1936, Seuss amused himself
by (2.)_____ together a nonsense poem to the rhythm of the sh ip's engine. Later
he illustrated the rhyme and published it as And to think that I saw it on
Mulberry Street. Many critics (3 .)_____it as Seuss’ best work.
A later book, McETligot’s Pool, (4.) _____ the first appearance of Seuss'
famous fantasy characters, and Horton Hatches the E gg introduces an ( 5.)_____
of morality. Seuss’ reputation as a major children’s writer was sealed with the
publication of The Cat in the Hat. This book uses easy-to-read words to tell the
story of two children alone at home on a rainy day. A cat wearing a tall hat
arrives to entertain them, wrecking their house in the (6.) __________ . The
enthusiastic (7.) _ ___ of this book delighted Seuss and led him to found
Beginner Books, a publishing company specializing in easy-to-read books for
children. Some o f his books have been made into cartoons and one o f them, How
the Grinch stole Christmas, was also made into an ingenious and (8.) ,
successful feature film starring Jim Carrey.
gave up writing for
At one point in his career, Seuss (9.) •
children
and (10.) ______ his talents to making docum entary
films. One of these
attracted a great deal of attention and won an Academy
Award. .

1. A. fully B. quite c. extremely D. fairly


2. A. placing B. laying c. putting D. setting
3. A. look beyond B. look upon c. look through D. look towa
4. A. indicates B. shows c. means D. marks
5. A. amount B. ingredient c. element D. item
6. A. practice B. method c. process D. action
7. A. reception B. welcome c. greeting D. admission
8. A. highly B. intensively c. strongly D. widely
9. A. shortly B. momentarily c. temporarily D. presently
10. A. assigned B. allocated c. donated D. devoted
148
B. WRITTEN TEST
I. OPEN CLOZ E T EST
Fill in each blank with ONE word to make a complete passage
TEST 1
THE KARAOKE CULTURE
We live in a culture that values participation over ability: the karaoke culture.
In broadcasting, it seems we cannot escape the vogue for ‘ access TV ’, ‘ people
show s' and "video diaries’. (1) _________ is our apparent obsession with
docu menting our own lives (2) ______ in the future, programmes will be
replaced by ca meras in every room so that we can watch (3 )________ endlessly
on TV. In the countless shows that fill our daytime schedules, (4 )______-
audience has become the star. The (5),_________ with this ‘ inclusive’ culture is
that it knows (6 )_________ bounds. The public make program mes, the public
participate in programmes, the public become performers. Anybody (7 )_______
do it! But there is a world of (8 )______ ■ enjoying something and jo ining in.
if we all join in, (9 )_________ is the point of artists or experts? If everything is
accessible, (1 0)_________ can be no mystery, no mystique.
TEST 2
QUEST - THE N EXT BIG THING?
How often do you go along to a gig and see (1)_________ new? Well, Quest's
Friday night gig at the City Hall certainly caught my (2)_________ . Having
heard one or two tracks online, I was (3)_________ a group of about six
musicians. Imagine my surprise when just three young men walked on stage.
It was clear that the band already have a small but (4)_______ following. A
group of fans in front of the small stage were singing (5)_________ to at least
half o f the songs. And it was easy to see why. Quest have a cleve r combination
of catchy (6)._________ , an irresistible beat, and very much their own sound. All
three of the band members play with great energy and expertise (7)_________
their age.
The only downside was when it came to the encores. They (8)_________ up
repeating some of their material and giving US cover (9)_________ of early rock
classics. A bit disappointing, but give them time and I'm sure they'll be writing a
lot more.
I'm sure we'll be hearing a lot more from Quest. Check them out every Friday
at the City Hall until the end of the month. It's well (10)_____ ’
it.
II. WORD F ORMS
A. C omplete the sentences with the correct forms of the given word
1. Do you know that exceptionally successful entrepreneurs, such as Richard
Branson, used to be academically . by their peers when they were
at school? (SHIN E)
149
2. Efforts have been made to recover disaster-stricken an d __________ areas of
the country, hope fading day by day. (WAR)
3. I think saying that all black people are lazy is a ve ry __________ remark, to
be honest (RACE)
4. .In some countries, women can sue their husband for having _______ ■
affairs and opt for one-sided divorce. (MARRY)
5. New full-time students from lower income households will be able to apply
for a(n )__________ maintenance grant offered by the government, but have
to work in public sectors after they graduate. (PAY)
6. 1 don ’t think their marriage will last long. They’re___________________
quarrelling. (CONTINUE)
7. Six-core processor is one of the m os t__________ technologies yet invented
to revolutionise the way computers work. (CUT)
8. It’s __________ when people won’t believe things that they are obviously
true. (FURY)
9. Globalization is the ongoing process that deepens and broadens the
relationship an d__________ among countries. (DEPEND)
10. The board of directors pour scorn on th e __________ ’ dereliction of duties,
causing the company to suffer catastrophic quarterly losses. (SEE)
B. Fill in each blank with the correct form of one suitable word from the list given
colony advantage access evolve
establish plant sufficient wild
The principal difference between urban growth in Europe and in the North
American colonies was the slow ( 1)_____ of cities in the former and their rapid
growth in the latter. In Europe they grew over a period of centuries from
town economies to their present urban structure. In North America, they started as
(2)___ communities and developed to mature urbanism in little more than a century.
In the early ( 3)_____ days in North America, small cities sprang up along the
Atlantic Coastline, mostly in what are now New England and the Middle
Atlantic states in the USA and in the lower Saint Lawrence and France,
particularly England, from which most capital goods (assets such as equipment)
and many consumer goods were imported.
Merchandising (4 )_____ were, accordingly, (5 )______located in port cities
which goods could be readily distributed to interior (6 )_____. Here, too, were
the favored locations for processing raw materials prior to export. Boston,
Philadelphia, New York, Montreal, and, other cities flourished, and as the
colonies grew, these cities increased in importance.
This was less true in the colonial South, where life centered around large
farms, known as (7 )_____, rather than around towns, as was the case in the
areas further north along the Atlantic coastline. The local ( 8 )_____ and the
150
economic (9 )_____ of the plantations were antagonistic to the development of
the towns. The plantations maintained their independence because they were
located on navigable streams and each had a wharf (10) _____ to the small
shipping of that day. In fact, one of the strongest factors in the selection of
plantation land was the desire to have it front on a water highway
III. ERROR IDENT IFICA TION
There are 10 mistakes in this passage. Underline and correct them
First comes the PC, then the internet and e-mail; now the e-book is upon us, a
hand-held device similarly in size and appearance to a video cassette. The user
simply rings up the website on their PC, selects the desired books, downloads
them onto their e-book machine and sits down to read them. In turn a page, the
user simply taps the screen. E-book technology is evolving rapidly, and with
some of the latest handholds you will even get internet access. But why would
one want an e-book machine with preference to a book? Well, one selling point
companies emphasized, when these devices hit the market a few years ago.
which is the space they save when going on holiday. E-books enlighten the load,
literally. Ten large novels can be put onto a device that weighs less than the
average paperback. One can understand why commercial interests seem to want
us to change. After all, the whole production process at first plan by author until
delivery to the printer has been doing electronically for a while now, so why not
save a few million trees and cut out the hard copy?
4 ._______________
7 .______________ _
2 ._______________
5 ._________ _
8 ._______________
3 ._____________ _
6 ._______________
9. _______________
10.
IV. SENTENCE TRANSFORMATION
Rewrite each of the sentences with the given word or the given beginning so
that the new sentence has the same meaning as the previous one
1. Pop stars are corrupted by the adulation of their fans.
It's the w ay ................................................. .........................................................
2. What I understand from her words is that it seems neither Cole nor Ledley
King will be going to Japan.
Rea ding ...................................................................................................................
3. The crash victim was beyond help when em ergency services reach her.
WHA TSOEVER
T her e............................................................... do on reaching the crash victim.
4. The staff hated Frank’s new policies intensely and so went on strike.
So..... .. .. . ....... ................ ................................................... ..............................
5. The fourth time he asked her to marry him, she accepted.
Only o n ..................................................................................................................
151
6. He worked very hard but he was unable to earn enough for his living. ENDS
Hard-w orking
7. Competition is fierce, but Frozen is a firm favourite to win the award for Best
Animation of the year. LOOK S
In th e ......................................................................................................................
8. Her latest novel isn't as good as her previous one. PAR
Her latest novel ......................................................................................................
9. She was very angry when her son made changes to her computer without her
permission. TA MP ERING
Her s o n ....................................................................................................... stack.
10. She herself admitted to being rather selfish
O n ................................................................ ,................ ......................................
TRƯỜNG T HPT CHUY ÊN TIỀN GIANG - TIỂN GIANG
A. MULTIPLE CHOICE
I. WO RD C HO IC E
1. The hotel room w as__________ furnished with only a bed, a wardrobe and
an ancient armchair.
A. thinly B. sparsely c. lightly D. sketchily
2. The main disadvantage to our house is that the only__________ to the garden
is through a bedroom.
A. passage B. doorway c. access D. communication
3. I've had my car examined three times now but no mechanic has been able to
the problem.
A. pinpoint B. focus c. specify D. highlight
4. Our hosts had prepared a________ _ meal with seven courses to celebrate
our arrival.
A. generous B. profuse c. lavish D. spendthrift
5. Don't thank me for helping in the garden. It was;____________ pleasure to be
working out o f doors.
A. plain B. mere c. simple D. sheer
6. She had______ mind which kept her alert and well-informed even in old age.
A. an examining B. a demanding c. an enquiring D. a querying
7. The police decided to_____________ the department store after they had
received a bomb warning.
A. abandon B. evacuate c. evict D. expel
8. The company was declared bankrupt when it had_____ ____ more debts than
it could hope to repay.
A. inflicted B. incurred c. entailed D. evolved
152
9. After months of bitter arguing the couple had to accept that they were______ .
A. incongruous B. incompatible c. dissident D. disaffected
10. The BBC has__________ a young composer to write a piece of music for the
Corporation's centenary.
A. ordered B. consulted c. commissioned D. decided
11. STRUCTURES AND GRAMMAR
1 • Y ou.............wonderful! Is that a new perfume you're wearing?
A. are smelling B. smell c. will smell D. have been smelling
2. It gets on my nerves that way that C arol ........... about her job.
A. forever boasts B. is forever boasting
c. never boasts D. is never boasting
3. W e.............to the new Harry Potter film on Saturday. If we can get tickets,
would you like to come too?
A. were going to go B. were thinking o f going
c. had planned to go D. had been meaning to go
4. Given the chance, many children .............TV uninterruptedly for hours.
A. are watching B. will have watched
c. will be watching D. will watch
5.................nocturnal creature(s).
A. Owl is a B. An owl is a c. The owl is a D. The owls are
6. The engagement ring he gave her .............been, more beautiful.
A. couldn't have B. might have c. shouldn't have D. must have
7. That w as.party last night - I haven't had so much fun foryears!
A. any B. some c. one D. no
8. I would normally have been on that train that was derailed yesterday, but
told me I should take my car to work instead.
A. anything B. everything c. something D. nothing
9. I wish you.............when I'm telling you something important.
A. won't interrupt B. wouldn't interrupt
c. didn't interrupt D. hadn't interrupted
10. Oh no! It looks like my rucksack .... behind in the scramble to get on the bus.
A. has left B. has got left c. had left D. had been left
III. PR EPOSITIONS AND PHRASAL VERBS
1. Since smokers appear to be unable to act in a socially responsible way by
voluntarily re fraining_____ smoking in public, it seems there must be a law.
A. from B. into c. off D. behind
2. He's a little confused and a little __________ balance, but he knows his
routine and heads to his car.
A. under B. against c. off D. in
3. My client accepts the formal apolo gy________ prejudice to any further legal
action she may decide to take.
A. without B. onto c. in D. for
153
4. If you get g oo ds _______ the cheap; you get them for a low price, often from
someone you know who works in the company or business that produces them.
A. on B. for c. at D. within
5. A pay rise is not__________ the realms o f possibility, I'm afraid.
A. behind B. under c. within D. against
6. Although the ot he rs __________ him, and his lies, Kirk persists in keeping
the pretence going.
A. see through B. turn up c. look back on D. come across
7. He failed to recover from a leg injury a nd __________ on a trip to Barcelona
A. missed out B. came up c. turned down D. made up
8. Next year we intend to __________ several new products but at the moment
we're still testing them.
A. bring out B. cut back . - c. take over D. go ahead
9. This heat has made m e__________ an itchy red rash.
A. come out in B. take up c. tone up D. put on
10. When we discussed how much our wedding was going to cost, it _________
over two thousand pounds!
A. added up to B. moved on to
c. sent away for
D. went ahead with
IV. COLLOCATIONS AND IDIOMS
1. Mr Simkins is the big '■ in the company as he has just been promoted
to the position of Managing Director.
A. bread B. apple c. cheese D. meat
2. This is a good hair dye but the colour gradually_________ after a few weeks.
A. fades B. vanishes c. disappears . D. pales
3. Breaking his leg dealt a _________to his chances of becoming a professional
footballer.
A. thump B. strike c.hit D. blow
4. The judge's ruling_________ a wave o f protest campaigns across the country.
A. provoked B. instigated c.launched D. commenced
5. Despite her poor exam results, Alice put a __________ face on the situation.
A. tough B. brave c.courageous D. bold
6. My grandmother was a lovely person who____pleasure from helping others.
A. gathered B. derived c.deduced D. collected
7. For busy people in today's society, lifestyle management is gain ing _______ .
A. points B. speed c.ground D. terrain •
8. Once at the skating rink, Ivan was allowed to skate to his hear t's__________ .
A. happiness B. content c.contentm ent D. delight
9. Benjamin Britten, the composer, is probably m os t__________ for his opera
"Peter Grimes".
A. famous B. conspicuous c. remarkable D. distinguished
154
10. He slamm ed my hand in the car door and, to add insult to__________ , didn't
stop to apologise.
A. dam age B. harm c. injury D. infamy
V. READING COMPREH ENSION
READING 1
Printm aking is the generic term for a number o f processes, of which woodcut
and engraving are two prime examples. Prints are made by pressing a sheet of
paper (or other material) agáinst an image-bearing surface to which ink has been
applied. When the paper is removed, the image adheres to it, but in reverse.
The woodcut had been used in China from the fifth century A.D. for applying
patterns to textiles. The process was not introduced into Europe until the
fourteenth century, first for textile decoration and then for printing on paper.
Woodcuts are created by a relief process; first, the artist takes a block of wood,
which has been sawed parallel to the grain, covers it with a white ground, and then
draws the image in ink. The background is carved away, leaving the design area
slightly raised. The woodblock is inked, and the ink adheres to the raised image. It
is then transferred to damp paper either by hand or with a printing press.
Engraving, which grew out of the goldsmith's art, originated in Germany and
northern Italy in the middle of the fifteenth century. It is an intaglio process
(from Italian intagliare, "to carve"). The image is incised into a highly polished
metal plate, usually copper, with a cutting instrument, or burin. The artist inks
the plate and wipes it clean so that some ink remains in the incised grooves. An
impression is made on damp paper in a printing press, with sufficient pressure
being applied so that the paper picks up the ink.
Both woodcut and engraving have distinctive characteristics. Engraving
lends itself to subtle modeling and shading through the use of fine lines.
Hatching and cross-hatching determine the degree of light and shade in a print.
Woodcuts tend to be more linear, with sharper contrasts between light and dark.
Printmaking is well suited to the production of multiple images. A set of
multiples is called an edition. Both methods can yield several hundred goodquality prints
before the original block or plate begins to show signs of wear.
Mass production of prints in the sixteenth century made images available, at a
lower cost, to a much broader public than before.
1. What does the passage mainly discuss?
A. The origins of textile decoration
B. The characteristics of good-quality prints
c. Two types of printmaking
D. Types o f paper used in printmaking
2. The word "prime" in line 2 is closest in meaning t o __________ .
A. principal B. complex c. general D. recent
3. The author's purposes in paragraph 2 is to describ e__________ .
A. the woodcuts found in China in the fifth century
155
B. the use o f woodcuts in the textile industry
c. the process involved in creating a woodcut
D. the introduction of woodcuts to Europe
4. The word "incised" is closest in meaning to__________ .
A. burned B. cut c. framed D. baked
5. The word "distinctive" is closest in meaning to_________ _.
A. unique B. accurate c. irregular D. similar
6. According to the passage, all of the following are true about engraving
EXCEPT that it ■
A. developed from the art of the goldsmiths
B. requires that the paper be cut with a burin
c. originated in the fifteenth century
D. involves carving into a metal plate
7. The word "yield" is closest in meaning to__________ .
A. imitate B. produce c. revise D. contrast
8. According to the passage, what do woodcut and engraving have in common?
A;Their;designs are slightly raised.
BLTÌtey^a&Hiève.contrast through hatching and cross-hatching.
GE.TFfe^ywere-flrst used in Europe.
D. They allow multiple copies to be produced from one original.
9. According to the author, what made it possible for members of the general
public to own prints in the sixteenth century?
A. Prints could be made at low cost.
B. The quality of paper and ink had improved.
c. Many people became involved in the printmaking industry.
D. Decreased demand for prints kept prices affordable.
10. According to the passage, all o f the following are true about prints EXCEPT
that they__________ .
A. can be reproduced on materials other than paper
B. are created from a reversed image
c. show variations between light and dark shades
D. require a printing press
READING 2
Long before they can actually speak, babies pay special attention to the
speech they hear around them. Within the first month of their lives, babies'
responses to the sound of the human voice will be different from their responses
to other sorts of auditory stimuli. They will stop crying when they hear a person
talking, but not if they hear a bell or the sound of a rattle. At first, the sounds that
an infant notices might be only those words that receive the heaviest emphasis
and that often occur at the ends of utterances. By the time they are six or seven
weeks old, babies can detect the difference between syllables pronounced with
156
rising and falling inflection s. Very soon, these differences in adult stress and
intona tion can influence babies' emotio nal states and beh avior. Lon g before they
develop actual language com pre hen sion, babies can sense when an adu lt is
play ful or ang ry, attem pting to initiate or term inate new beh avior, and so on.
me rely on the basis of cues such as the rate, volume , and melody of adult speech.
Ad ults ma ke it as easy as they can for babies to pick up a langua ge by
exaggerat ing s uch cue s. One res ear che r observed babies and the ir mo the rs in six
div ers e cultures and found that, in all six lang uages, the mothers used sim plified
syntax, sho rt uttera nce s and nonsense sounds, and transform ed certain sou nds
into bab y talk . Other investigators have no ted tha t when mo the rs talk to babies
who are only a few m onths old, they exa gge rate the pitc h, loudne ss, and inte nsity
of the ir words. The y also exa gge rate the ir facial exp ressions, hold vow els
long er, and em ph as ize certain w ords.
Mo re sig nif ica nt for lang uage dev elo pm ent than the ir response to general
intona tion is observation tha t tiny babies can mak e rela tive ly fine distinctions
between speech sounds. In o the r w ords, babies enter the world with the ability to
make pre cisely tho se percep tual discriminat ion s tha t are nec essary if they are to
acq uire aural language.
Babies obvio usl y derive pleasure from soun d inpu t, too: even as young as
nine mo nth s the y will listen to songs or stories, alth oug h the wo rds themselves
are bey ond the ir und erstandin g. For bab ies, lang uage is a sensory -m oto r delight
rather than the route to prosaic meaning tha t it often is for adults.
1. Wh at does the pas sag e mainly di scu ss?
A. How bab ies differentiate betw een the sound of the hum an voice and oth er
sounds
B. The differences between a baby's and an adult's ability to comprehend language
c. How bab ies perceive and respond to the human voice in their earliest stages o f
language d eve lop me nt
D. The respon se of babies to sounds o the r than the hum an voice
2. Why does the autho r m ention a bell and a rattle?
A. To contr ast the reactio ns of babie s to hum an and non human sou nds
B. T o give exam ple s of sounds th at will cau se a baby to cry
c. To exp lain how b abies dist inguish betw een different n onhum an sou nds
D. To give exam ple s of typical toys th at babies do not like
3. Why does the autho r men tion syllables pro nounced with rising and falling
inflec tions?
A. To demo nstrate how diffic ult it is for babies to inte rpret emotions
B. To illustrate that a six-w eek-old baby can already distinguish som e language
dif fer ences
c. To pro vid e an exa mp le of way s a dults speak to babies
D. To give a reason for bab ies' difficulty in dis tinguish ing one adult from
another
4. The wo rd "noted" in line 18 is clos est in m ean ing t o ___________ .
A. the orized B. r equ ested c. disagr eed D. o bse rve d
157
5. The word "They" refers t o __________ .
A. mothers B. investigators c. babies D. words
6. The passage mentions all o f the following as ways adults modify their speech
when talking to babies EX CEPT__________ .
A. giving all words equal emphasis B. speaking with shorter sentences
c. speaking more loudly than normal D. using meaningless sounds
7. The word "emphasize" is closest in meaning t o __________ .
A. stress B. repeat c. explain D. leave out
8. Which of the following can be inferred about the Endings described in
paragraph 2?
A. Babies who are exposed to more than one language can speak earlier than
babies exposed to a single language.
B. Mothers from different cultures speak to their babies in similar ways,
c. Babies ignore facial expressions in comprehending aural language.
D. The mothers observed by the researchers were consciously teaching their
babies to speak.
9. What point does the author make to illustrate that babies are born with the
ability to acquire language?
A. Babies begin to understand words in songs.
B. Babies exaggerate their own sounds and expressions,
c. Babies are more sensitive to sounds than are adults.
D. Babies notice even minor differences between speech sounds.
10. According to the author, why do babies listen to songs and stories, even
though they cannot understand them?
A. They understand the rhythm.
B. They enjoy the sound.
c. They can remember them easily.
D. They focus on the meaning of their parents' words.
VI. CLOZE TEST
CLOZE TEST 1
THE TRUTH BEHIND A SMILE
People smile a great deal, and we seem to know instinctively that some smiles
are more genuine thán others. But is there any scientific (1 )__________ for this?
Recent research suggests that a mechanism in the brain can help US ( 2)________
whether a smile is really heartfelt - or whether it is just being (3 )__________ on
for show.
According to various long-held traditions, a genuine smile involves the eyes
as well as the mouth. In the nineteenth century, a French anatomist (4 )________
to prove this. He used electrodes to stimulate the facial muscles of volunteers,
(5 )____________ creating false smiles. He found that real smiles were always
158
(6 )_ ________ with the contraction of a muscle around the eye, but that his
artificially induced ones were not.
During more recent research, volunteers were shown a variety of human
facial (7 )________ and their reactions to these were monitored. When they were
shown a happy face, 35% of the volunteers immediately started looking at the
eye area, checking for tell-tale crinkles that would (8) _ _________that the smile
was genuine; but when shown a sad or neutral face, they did not. So why did the
human brain evolve to distinguish between real and false smiles? It could be that
this ability to (9 )__________ a quick assessment of a smile has an important
role to play in successful communication. A genuine smile (1 0) __________ as a
gesture of conciliation in conflict, and it’s important to know whether we are
really being offered a truce or not.
1. A. sign B. basis c. root D. fact
2. A. recollect B.a cce pt c. admit D. recognize
3. A. put B. brought c. created D. stuck
4. A. got down B. set out c. went off D. carried ou
5. A. despite B. thereby c. however D. nonethele
6. A. associated B. mixed c. joined D. accompan
7. A. exhibitions B. resemblances c. appearances D. expression
8. A. assure B. confirm c. justify D. approve
9. A. make B. earn c. do D. hold
10. A. aims B.serve s c. portrays D. applies
CLOZE TEST 2
HOW TO CONCENTRATE
Concentration is good in exams, bad in orange juice. Concentration happens
when you manage to focus on one thing to the (1)_J_________o f all others, andconcentrating
on that one thing (2)__________ you to stop worrying about a lot
of other things. Sometimes, of course, your mind concentrates when you do n't
want it to. Maybe you can't get something out of your head, such as a problem
you have to (3)__________ up to, or an embarrassing situation you’ve been in.
That's why collecting things as a hobby is popular; it (4)__________ your mind
off other things, indeed, some people seem to prefer looking after and
cataloguing their collections to actually doing anything with them, because this
is when the (5)__________ , single-minded concentration happens.
The natural span for concentration is 45 minutes. That’s why half an hour for
a television programme seems too short whilst an hour seems too long. But manv
peop le’s lives are (6)_________ of concentration. Modern culture is served up in
small, (7)__________ digestible chunks that require only a short (8)___________
span - although young people can concentrate on computer games for days at a
(9) •
Sticking out the tongue can aid concentration. This is because you can't
(10) __________ yourself with talking at the same time and other people won’t
dare to interrupt your thoughts, because you look like an idiot!
159
1. A. removal B. exclusion c. omission D. rejection
2. A. lets B. means c. makes D. allows
3. A. face B. confront c. tackle D. meet
4. A. brings B. puts c. holds D. takes
5. A. arresting B. gripping c. absorbing D. enthrallin
6. A. absent B. devoid c. lacking D. deficient
7. A. gently B. plainly c . surely D. easily
8. A. attention B. application c. consideration D. contempla
9. A. length B. stroke c. time D. sequence
10. A. sidestep B. distract c. sidetrack D. disturb
B. WRITTEN TEST
I. OPEN CLOZE TEST (20 PTS):
CLOZE TEST 1
AN EQUAL SHARE OF HOUSEWORK MAKES A HAPPY RELATIONSHIP
A recent study shows that an unequal share of household chores is still the
norm in many households, despite the fact that many more women now have
jobs. In a survey of 1,256 people (1)__________ between 18 and 65, men said
they contributed an average of 37 per cent of the total housework, while the
women estimated their share to be nearly double that, at 70 per cent. This ratio
was not (2)__ _ ______ by whether the woman was working or not.
When they were asked what they thought was a (3)__________ division of
labour, women with jobs felt that housework should be shared equally between
male and female partners. Women who did not work outside the home were
satisfied to perform 80 per cent - the (4)__________ of the household work - if
their husbands did the remainder. Research has shown that, if levels increase
(5)__________ these percentages, women become unhappy and anxious, and
feel they are unimportant.
After marriage, a woman is reported to increase her household workload by
14 hours per week, but for men the (6)______ is jus t 90 minutes. So the division
of labour becomes unbalanced, as the man's share increases (7)__________ less
than the woman's. It is the inequality and (8)______ ' of respect, not the
actual number of hours, which leads to anxiety and depression. The research
(9)__________ housework as thankless and unfulfilling. Activities included in
the study were cooking, cleaning, shopping, doing laundry, washing-up and
childcare. Women who have jobs report that they feel overworked by these
chores in (10)__________ to their professional duties. In contrast, full-time
homemakers frequently anticipate going back to work when the children grow
up. Distress for this group is caused by losing the teamw ork in the marriage.
CLOZE TEST 2
Chewing gum, until now considered the ultimate jun k food and the national
dish of the gormless, turns out to have a positive effect on cognitive performance. In
the first intellectual victory far supporters' of progressive education in many
years, it seems that the teacher telling the child at the back of the class to 'spit it
160
out' has (1)_________ it all wrong; The Human Cognitive Neuro-Science Unit at
the University of Northumbria has tested the thinking and memory of those who
chew gum. The results show that gum-chewers (2)__________ far better in
cognitive tests than those who did not partake.
This new information makes many things clear that were (3)__________
shrouded in mystery. It was said of Gerald Ford that he could not walk and chew
gum at the same time. That he proved so bad at walking and was constantly
falling over was seen as a (4)__________ that he was not up to the job. Now it is
apparent that faced with the choice between walking and masticating he picked
wisely. Even though he fell down the steps of many aircraft, his performance
When he reached the ground was (5)__________enhanced.
Certain mysteries of literature and. language are (6)__________ up, too.
Shakespeare has Brutus tell Cassius to “chew on this” when he wants him to think
about something. Chew on what? Now all is clear. Difficult problems which
(7)_________ cogitation are confusingly called “ sticky”. Be confused no longer.
The beneficial effects of gum may (8)___________ as a surprise to some, but
chewers themselves, being intellectually superior to everyone else, of course,
have been (9)___________ of gum's advantages for years.
Sadly, some politicians want to tax gum to pay for the (10)_ _________ of
scraping discarded pieces from the street. It can confidently be predicted that
such small-mindedness among non-chewers will leave the mouths of gum
consumers everywhere goping open.
II. WO RD FO RM S (20 PTS)
WORD FORM 1
1. These shoes are very nice, but they're te rribly__________ (PR ICE ).
2. The __(OU T) appearance of the building has not changed at all in 200 years.
3. Sadly, some economists think that full employment in Europe is a n ________
(ATT AIN) go al . .
4. The state must ensure the independence an d__ (PA RT) of the justice system.
5. Sales o f_______ _ (CO OK ) meals have risen sharply over the past few years.
6. He refused to say anything on the grounds that he might __________
(CRIME) himself.
7. The recent factory closures and job losses are just a __________ (TAS TE ) of
the recession that is to come.
8. Most of the errors were corrected at t h e___________ (RE AD ) stage before
the copies are finally printed.
9. You can accuse me of __________ (CO WA RD ), but 1 still wouldn't
volunteer to fight in a war.
10. When we re-examined the regulations, we realised that we had__________
(IN TE RP RE T) them.
161
WORD FORM 2
suffice terminate fo ot signifo
pow er nature resource submit
Man kind 's intuition of freedom, and our identification of freedom with
knowledge, sets US apa rt from animals. The anim al's grasp o f freedom is (1)____
in comparison, being only the freedom to respond to external stimuli. The
nearest creature to US on the (2)__________ tree of life, the chimpanzee, cannot
retain an image for a (3)____length o f time to be able to reflect on it. So animal
life is largely a matter of conditioned reflexes, performed in an (4)__________
present; in short, animals are little more than m achines with (5)__________ .
While the animal is carried along (6)_______ on the stream of time, mankind
has certain capacities that (7)__________ US to resist the current or look into the
future. Our (8)__________ invention of language was the first step towards this
‘conquest of tim e’. Language ‘fixes’ experiences, and places the experience of
the past on an equal (9)_______ with that of the present. Imagination was bound
to follow, as a (10)_________ progression from ‘labelling’ a past experience to
conjuring up its mental image.
III. ERROR IDENTIFICATION
Line ECO-TOURĨSM
15 Snacking on green ants is
JO not everyone’s idea of the
15 most delicious
holiday indulgently, but on a
recent walk through the
Daintree rainforest in
Queensland, Australia,
Aboriginal guide Kirsty
Norris assured a group
of uncertain guest that the
traditional food source of
her KukuYalanji
tribe was worth a try. She
might have been right - but
lucky for the
native ants and the tourists,
rain came streaming down
through the
canopy, sending any
possible food scurrying for
cover.
Connected with nature isn’t
a compulsory part of a stay
at an
environmental friendly
resort, but at the Daintree
Eco Lodge, where tree
house villas are set on stilts
above the compound’s
waterfall-fed creek,
many people find
themselves done a bit of
communing while they
relax.
Although ecotourism is
increasing on popularity,
recording heady
growth worldwide, it is still
difficult to define. For some
travellers,
ecotourism means
eavesdropping on nature
from the comfort of a plush
bed with a magnificent
view. To others, it's about
doing without hot
showers and trekking across
wildernesses. However,
industrial watchers
say the category’s basic
tenet is minimal
environmental impact
combination
with some contribution to
education and conservation.
1._______________ 2 ._______________
4. _______________ 5 ._______________
7._______________ 8 .________________
3 ._______________
6 ._______________
9. _______________
10. ______________
162
IV. SENTENCE TRANSFORM ATION (20 PTS)
1. Absolute secrecy was crucial to the success of the mission.
—> Without________________________________________________ _
2. The two sides never looked likely to reach an agreement.
—> At no time w as ___________________________________
3. He is a complete hypocrite; in public he condemns smokers, yet he smokes a
packet a day himself.
—> S o______________________________________________ _______
4. Ruth never asks anyone for a loan as she do esn't like to admit she has
financial problems.
—> Ruth is ________________________________________________ _
5. He loses his temper at all things, even the slightest one.
—» He flie s________________________________________________ _
6. Initially, losing one's job can seem awful; afterwards it can work out well, for
some people. BLESSING
—» Losing one's job has prov ed________________________________
7. He's a pleasant man socially, but he's a tough businessman. BARGAIN
-> He's a pleasant man _______________________________________
8. As a champion swimmer, she will never be better than she is now. PRIME
- > _______________________________________________________
9. A summary' cannot bring out the high quality of this book. JUSTICE
—»_____________________________________
10. The inspector showed US four potential health hazards. DREW
-> _______________________________________________________
TRƯỜNG THPT GIA ĐỊNH - TP. HQ CHÍ MINH
A. MUL TIPL E CH OI CE (4 0 PT S)
I. PHONOLOGY (5PTS)
Choose the word whose underlined part is pronounced differently from the others.
1. A. chauffeur
2. A. hypocrite
3. A. umpire
4. A. pizza
5. A. lounge
Choos e the w ord w hich is stressed different ly from the oth er three.
6. A. ridicule
7. A. legitimate
8. A. hotel
9. A. Aborigine
10. A. vigilance
B. chic
B. expertise
B. ultramarine
B. buzzard
B. mound
B. politics
B. comparison
B. caffeine
B. enthusiasm
B. manifest
c. chore
c. trịchologist
c. trumpet
C. muzzle
C. foul
C. dogwhistle
C. arithmetic
C. wholesale
C. simultaneous
C. magenta
D. Michigan
D. click
D. fumigate
D. fuzzy
D. gourd
D. cheerio
D. engineering
D. bereaved
D. paracetamol
D. interview
163
11. WORD CHOICE (5 PTS):
Choose the best options to com plete the following sentences.
11.1 could he ar __________ of the conversation from across the room.
A. shreds B. pieces c. patches D. snatches
12. They took c as h__________ the car previously offered as a prize.
A. in place o f B. on account o f c. with regard to D. in lieu o f
13. After the concert, everyone had t o _________ home through the thick snow.
A. trudge B. tread c. trace D. trickle
14.1 wish he w ouldn 't make such unkind an d__________ remarks.
A. lapse B. disparaging c. inconsolable D. rough
15. The little girls w er e______ . brightly colored hoops around their waists.
A. twirling B. curling c. swirling D. hurling
16. As the President was absent, I was asked to __________ the meeting.
A. officiate B. govern c. chair D. regulate
17. In th e __________ of security, personnel must wear their identity badges at
all times.
A. requirement B. interests c. demands D. assistance
18. Meg had a __________ escape when she was hang-gliding yesterday.
A. narrow B. close c. near D. slender
19. The doctor thought he had got over the worst, but his condition suddenly
A. deteriorated B. dismantled c. dissolved D. disintegrated
20. He had a momentary __ ________ of concentration and before he knew it the
car had spun out o f control.
A. lapse B. mistake c. slip D. error
III. G RAM MAR AND STRUCTURES (5PTS):
Choose the best options to complete the following sentences.
21. The Second World War was so terrible, _ ________ time a large number o f
families were separated.
A. during the B. during which c. at the D. on which
22. They turned down the proposal ____________ that it did n't fulfill their
requirement.
A. by reason B. on the grounds c. as a cause D. allow ing
23. We are prepared to overlook the error on this oc ca sion __________ your
previous good work.
A. in the light o f B. thanks to c. with a view to D. with regard to
24. We should leav e__________ 8.30.
A. any later than B. much later than c. no later than D. not later than
25. There was the most fan tastic__________ during the opening ceremony.
A. display o f fireworks B. firework display
c. firework’s display
D. displayed firework
164

26. “I bought you some flowers.” -“T hey're beautiful, but y ou __________
A. ne edn't B. sho uldn't have c. mustn't have D. didn't have
27. __________ a small creature that defends itself with lobster-like claws and a
poisonous sting.
A. Scorpions are B. Many a scorpion is
c. A scorpion, which is D. The scorpion is
28. She insisted that the rep orter__________ her as his source of information.
A. not to mention B. not mention c. don't mention D. not mentioning
29. The accountant broke the law, so it would be irresponsible of the firm
A. i f they won't fire him B. not to fire him
c. that they not fire him D. for not firing him
30. __________ that took American art out of the romanticism of the mid-1800’s
and carried it to the most powerful heights of realism.
A. Winslow Homers' paintings B. It was Winslow Homers’ paintings
c. When Winslow Homers' paintings D. Paintings of Winslow Homers'
IV. PHRASAL VERBS AND PREPOSITIONS (5 PTS)
31. When he's depressed, he needs someone like a good friend t o __________ .
A. ham mer on B. pour out c. dump on D. drum out
32. Her success can be__________ a combination o f luck, intelligence and sheer
hard work.
A. ranked among B. put together c. put down on D. lived up to
33. When the Chairman ran o ff with his secretary, the Board tried to _________
the matter.
A. switch o ff B. hush up c. calm down D. tuck away
34. The library is __________ people who lose their books.
A. cracking down on B. stepping up
c. going down with D. coming up against
35. The word “chaos” h as __________ a special scientific meaning.
A. included in B. taken on c. held o f D. gasped up
36. When the funds finally _ _________they had to abandon the scheme.
A. faded away B. clamped down
c. petered out D. fobbed off
37. The Press thought the football manager would be depressed by his dismissal
but he ju st__________ .
A. ran it down B. called it off c. turned it down D. laughed it off
38. She accidentally dropped her ring__________ a drain in the road.
A. into B. in c. down D. on
39. D on’t be too hard on yourself. Ju st__________ it up to experience and d on’t
do it again.
A. talk B. chalk c. stalk D. set
40. The murderer d id _______ all of his victims by poisoning them with cyanide.
A. away with B. with c. for D. without
165
V. GUIDED CLOZE 1 (5PTS):
Read the text below and decide which answer best fits each space.
FRIDAY THE THIRTEENTH
Police are hunting for a hit-and-run driver who knocked a teenage cyclist off
her bike in East Street. Sarah Tucker, 17, had a lucky escape on Friday, 13th May,
when she was sent reeling by a black Volvo on her way home from work.
She bruised her thigh and shoulder and her bicycle was (4 1) __________ The
driver stopped for a moment but then drove off without (42 )__________ a name
or address and before Sarah could get his number. “ I tried to get out of his way,
but I cou ldn 't,” she said.” Everyone at w ork kept going on about it being Friday
13th. I’m not a bit (4 3)_____________ and wouldn’t change any of my plans
jus t because Friday 13th is supposed to be unlucky, I don ’t usually take, any
(4 4) _________ of that sort o f thing but I will now. I think I’ll stay in bed.”
The accident (4 5)__________ at the junction with Westwood Road at about
6.30pm as Sarah was making her ( 46 )__________ home to the Harley Estate.
The Volvo pulled out of Westwood onto Henley Road in front of the
teen ager's bicycle. “He could at (47) __________ have helped her up. I d on 't see
why he should get away with it,” said her father, Derek, “Sarah was lucky. I
don’t know why the driver didn ’t see her. He can’t have been (4 8)__________
attention. It is unfortunate that nobody took down the number.” Though still too
(4 9) __________ to ride a bike, Sarah was able to go back to (5 0) ___________in
Marlow on Monday.
41. A. damaged B. harmed c. devastated D. crashed
42. A. noting B. presenting c. leaving D. suggesting
43. A. irrational B. superstitious c. unreasonable D. prejudices
44. A. notice B. consideration c. note D. care
45. A. came about B.turned up c. finished up D. took place
46. A. route B. way c. course D. path
47. A. once B. most c. least D. best
48. A. giving B. paying c. attracting D. providing
49. A. discouraged B. confused c. overcome D.s hak en
50. A. work B.job c. post D. employme
VI. GUIDED CLOZE 2 (5PTS):
Read the text below and decide which answer best fits each space.
THE TRUTH BEHIND A SMILE
People smile a great deal, and we seem to know instinctively that some smiles
are more genuine than others. But is there any scientific (5 1)________ for this?
Recent research suggests that a mechanism in the brain can help us (52 )_______
whether a smile is really heartfelt - or whether it is just being (5 3)_________ _
on for show.
(54) __________ to various long-held traditions, a genuine smile involves
the eyes as well as the mouth. In the nineteenth century, a French anatomist
166
(5 5) _ _______ to prove this. He used electrodes to stimulate the facial muscles
of volunteers, (5 6)__________ creating false smiles. He found that real smiles
were always (5 7) __________ with the contraction of muscle around the eye, but
that his artificially induced ones were not.
During more recent research, volunteers were shown a variety of human
facial expressions and the reactions to these were monitored. When they were
shown a happy face, 35% of the volunteers immediately started looking at the
eye area, checking for tell-tale crinkles that would (5 8 )__________ that the
smile was genuine, but when shown a sad or neutral face, they did not. So why
did the human brain evolve to (5 9) __________ between real and false smiles? It
could be that this ability to (6 0)__________ a quick assessment of a smile has
an important role to play in successful communication. A genuine smile serves
as a gesture of conciliation in conflict, and it’s important to know whether we are
really being offered a truce or not.
51. A. sign B. basis c. ro ot D. fact
52. A. recollect B. accept c. admit D. recognise
53. A. put B. brought c. created D. stuck
54. A. Providing B. Considering c. Relating D. According
55. A. got down B. set out c. went off D. carried ou
56. A. despite B. thereby c. however D. nonethele
57. A. associated B. mixed c. joined D. accompan
58. A. assure B. confirm c. justify D. approve
59. A. decide B. tell c. distinguish D. reckon
60. A. make B. earn c. do D. hold
VII. READING PASSAGE 1 (5PTS):
Read the text below and choose the best answer to each question.
Excerpted fro m What Video Games Have to Teach US about Learning and
, Literacy by James Paul Gee
When people learn to play video games, they are learning a new literacy. Of
course, this is not the way the word "literacy" is normally used. Traditionally,
people think of literacy as the ability to read and write. Why, then, should we
think o f literacy more broadly, in regard to video games or anything else, for that
matter? There are two reasons.
First, in the modern world, language is not the only important communicational
system. Today images, symbols, graphs, diagrams, artifacts, and many other
visual symbols are particularly significant. Thus, the idea of different types of
"visual literacy" would seem to be an important one. For example, being able to
"read" the images in advertising is one type of visual literacy. And, of course,
there are different ways to read such images, ways that are more or less aligned
with the intentions and interests o f the advertisers. Knowing how to read interior
designs in homes, modernist art in museums, and videos on MTV are other
forms of visual literacy.
167
Furthermore, very often today words and images of various sorts are
juxtaposed and integrated in a variety of ways. In newspaper and magazines as
well as in textbooks, images take up more and more of the space alongside
words. In fact, in many modern high school and college textbooks in the sciences
images not only take up more space, they now carry meanings that are
independent of the words in the text. If you can't read these images, you will not
be able to reco ver their meanings from the words in the text as was more usual in
the past. In such multimodal texts (texts that mix words and images), the images
often communicate different things from the words. And the combination of the
two modes communicates things that neither of the modes does separately. Thus,
the idea of different sorts of multimodal literacy seems an important one. Both
modes and multimodality go far beyond images and words to include sounds,
music, movement, bodily sensations, and smells.
None of this news today, of course. We very obviously live in a world awash
with images. It is our first answer to the question why we should think of literacy
more broadly. The second answer is this: Even though reading and writing seem
so central to what literacy means traditionally, reading and writing are not such
general and obvious matters as they might at first seem. After all, we never just
read or write; rather, we always read or write something in some way.
So there are different ways to read different types of texts. Literacy is
multiple, then, in the sense that the legal literacy needed for reading law books is
not the same as the literacy needed for reading physics texts or superhero comic
books. And we should not be too quick to dismiss the latter form of literacy.
Many a superhero comic is replete with post-Freudian irony o f a soil that would
make a modern literary critic's heart beat fast and confuse ãny otherwise normal
adult. Literacy, then, even as traditionally conceived to involve only print, is not
a unitary thing but a multiple matter. There are, even in regard to printed texts
and even leaving aside images and multimodal texts, different "literacies."
Once we see this multiplicity o f literacy (literacies), we realize that when we
think about reading and writing, we have to think beyond print. Reading and
writing in any domain, whether it is law, rap songs, academic essays, superhero
comics, or whatever, are not just ways of decoding print, they are also caught up
with and in social practices... Video games are a new form o f art. They will not
replace books; they will sit beside them, interact with them, and change them
and their role in society in various ways, as, indeed, they are already doing
strongly with movies. (Today many movies are based on video games and many
more are influenced by them.) We have no idea yet how people "read" video
games, what meanings they make from them. Still less do we know how they
will "read" them in the future.
61. According to the first paragraph, the broadest definition o f'’literacy” is
A. The ability to analyze literature
B. The ability to comprehend basic cultural cues
168
c. The ability to read and write
D. The ability to compose poetry
62. All are mentioned as being types o f’’visual literacy” EXCEPT
A. Musical tones B. Interior Design c. Diagrams D. Modern Art
63. An example from a science textbook o f the phenomenon the author describes
in the third paragraph could be
A. A genetic tree that coincides with the discussion of specific mammal
classes in the text
B. A diagram of a specific chemical reaction that is used to explain a broad
definition in the text
c. An illustration of a plant cycle that accompanies a chapter on photosynthesis
D. A cartoon that references the same methods discussed in the text about
laboratory safety
64. What is an example of a "m ultimodal” text?
A. Adictionary B. Amovie script c. Aphoto album
D. An art book that describes the art as well as reproduces images of the
original prints
65. The idiom in the sixth paragraph, "read aga inst the grain of the text" is
closest in meaning to
A. Reading to understand the underlying meanings and themes of the author's
words-not just a literal interpretation
B. Reading text that defines different types of wheat and grains
c. To read the text from right to left rather than left to right
D. To read books that use recycled paper and other green alternatives
66. In the seventh paragraph, the author suggests that literacy is multiple,
meaning that
A. To be "literate" can mean participating in any form of expression
B. One's literacy increases exponentially as greater mastery of reading and
writing is achieved
c. Different genres and modes of expression require different background
knowledge and perspectives to understand them
D. Literacy can only be gained by exploring every type of media and expression
67. Why does the author give the example of superhero comics to explain
multiple literacies?
A. To explain that comic books are written for children and purely for
entertainment. They require only a basic knowledge of the action that
occurs in the story
B. To once again refer to his earlier points about "m ultimodal” texts
c. To insist that even when an author may intend multiple meanings and
interpretations, they are rarely successful in conveying those to readers
D. Things that may seem on the surface to be only meant for a particular
group of people can actually have very profound meanings to those who
possess other types o f literacy
169
68. The author suggests that all of the following require different types of
literacy and the ability to decode meaning EXCEPT
A. Rap music B. Comic books c. Academic papers D. Symphonies
69. The author says that video games
A. Are not yet entirely understood in terms of literacy, but are already
impacting other forms o f expression such as filmmaking
B. Are unrealistic and should not fall into the same categories as the other
texts he describes
c. Are too violent to risk experimenting with for the purposes o f understanding
literacy
D. Are irrelevant in academic discussion because no one has yet determined
how to explain the ways that people understand them
70. What would be the most logical information for the next paragraph to contain
if the article continued?
A. A technological definition of video games, how they are made, and how
they are played
B. A historical explanation of the very first video game and its evolution
c. Examples of the way that some people currently interpret video games
and what they mean to them
D. A price comparison of video game consoles and whether or not quality
has a direct impact on literacy
READING PASSAGE 2 (5PTS):
Read the text below and choose the best answer to each question.
1. The craft of perfumery has an ancient and global heritage. The art flourished
in Ancient Rome, where the emperors were said to bathe in scent. After the
fall of Rome, much of the knowledge was lost, but survived in Islamic
civilizations in the Middle Ages. Arab and Persian pharmacists developed
essential oils from the aromatic plants of the Indian peninsula. They
developed the processes of distillation and suspension in alcohol, which
allowed for smaller amounts of raw materials to be used than in the ancient
process, by which flower petals were soaked in warm oil. This knowledge
was carried back to European monasteries during the Crusades.
2. At first, the use o f fragrances was primarily associated with healing. Aromatic
alcoholic waters were ingested as well as used externally. Fragrances were
used to purify the air, both for spiritual and health purposes. During the
Black Death, the bubonic plague was thought to have resulted from a bad
odour which could be averted by inhaling pleasant fragrances such as
cinnamon. The Black Death led to an aversion to using water for washing,
and so perfume was commonly used as a cleaning agent.
3. Later on, the craft of perfume re-entered Europe, and was centred in Venice,
chiefly because it was an important trade route and a centre for glass-making.
170
Having such materials at hand was essential for the distillation process. In the
late seventeenth century, trade soared in France, when Louis XIV brought in
policies of protectionism and patronage which stimulated the purchase of
luxury goods. Here, perfumery was the preserve of glove-makers. The link
arose since the tanning of leather required putrid substances. Consequently,
the gloves were scented before they were sold and worn. A glove and
perfume makers’ guild had existed here since 1190. Entering it required 7
years of formal training under a master perfumer.
4. The trade in perfume flourished during the reign of Louis XV, as the master
glove-and-perfume makers, particularly those trading in Paris, received
patronage from the royal court, where it is said that a different perfume was
used each week. The perfumers diversified into other cosmetics including
soaps, powders, white face paints and hair dyes. They were not the sole
sellers of beauty products. Mercers, Spicers, vinegar-makers and wig-makers
were all cashing in on the popularity of perfumed products. Even simple
shopkeepers were coming up with their own concoctions to sell.
5. During the eighteenth century, more modern, capitalist perfume industry
began to emerge, particularly in Britain where there was a flourishing
consumer society. In France, the-revolution initially disrupted the perfume
trade due to its association with aristocracy, however, it regained momentum
later as a wider range of markets were sought both in the domestic and
overseas markets. The guild system was abolished in 1791, allowing new
high-end perfumery shops to open in Paris.
6. Perfume became less associated with health in 1810 with a Napoleonic
ordinance which required perfumers to declare the ingredients of all products
for internal consumption. Unwilling to divulge their secrets, traders
concentrated on products for external use. Napoleon affected the industry in
other ways too. With French ports blockaded by the British during the
Napoleonic wars, the London perfumers were able to dominate the markets
for some time.
7. One of the significant changes in the nineteenth century was the idea of
branding. Until then, trademarks had had little significance in the perfumery
where goods were consumed locally, although they had a long history in other
industries. One o f the pioneers in this field was Rimme! who was nationalized
as a British citizen in 1857. He took advantage of the spread of railroads to
reach customers in wider markets. To do this, he built a brand which
conveyed prestige and quality, and were worth paying a premium for. He
recognised the role of design in enhancing the value of his products, hiring a
French lithographer to create the labels for his perfume bottles.
8. Luxury fragrances were strongly associated with the affluent and prestigious
cities of London and Paris. Perfumers elsewhere tended to supply cheaper
171
products and knock-offs of the London and Paris brands. The United States
perfume industry, which developed around the docks in New York where
French oils were being imported, began in this way. Many American firms
were founded by immigrants, such as William Colgate, who arrived in 1806.
At this time, Colgate was chiefly known as a perfumery. Its Cashm ere
Bouquet brand had 625 perfume varieties in the early 20th century.
71. The purpose o f the text is to
A. compare the perfumes from different countries
B. describe the history of perfume making
c. describe the problems faced by perfumers
D. explain the different uses of perfume over time
72. Which of the following is NOT true about perfume making in Islamic countries?
A. They created perfume by soaking flower petals in oil.
B. They dominated perfume making after the fall of the Roman Empire,
c. They took raw materials for their perfumes from India.
D. They created à technique which required fewer plant m aterials.
73. Why does the w riter include this sentence in paragraph 2?
During the Black Death, the bubonic plague was thought to have resu lted
front a bad odour which could be averted by inhaling p leasan t fragran ces
such as cinnamon
A. To explain why washing was not popular during the Black Death
B. To show how improper use o f perfume caused widespread disease
c. To illustrate how perfumes used to be ingested to treat disease
D. To give an example o f how fragrances were used for health purposes
74. Why did the perfume industry develop in Paris?
A. Because it was an important trade route
B. Because o f the rise in the glove-making industry
c. Because of the introduction of new trade laws
D. Because o f a new fashion in scented gloves
75. In paragraph 4, it is implied tha t...
A. Master glove and perfume makers created a new perfume each week.
B. Mercers, spicers and other traders began to call themselves masters,
c. The Royal Court only bought perfume from masters.
D. Cosmetics were still only popular within the Royal Courts.
76. How did the French Revolution affect the Parisian perfume industry?
A. The industry declined then rose again.
B. The industry collapsed and took a long time to recover,
c. The industry was greatly boosted.
D. The industry lost most o f its overseas customers.
172
77. London came to lead tile perfume industry because...
A. the French Revolution meant that there were fewer customers in France.
B. Napoleon's new laws affected the profitability o f perfume-making,
c. the production of perfume ceased during the Napoleonic wars.
D. the French were unaèle to export perfumes for a period of time.
78. Which of the following is NOT true o f Rimmel?
A. He was one of the first people to utilise trademarks.
B. He created attractive packaging for his products.
c. His products were more expensive than other brands.
D. He transported his goods to potential customers by train.
79. What is implied about the New York perfume industry?
A. It was the fastest-growing perfume industry in the world at that time.
B. It was primarily developed by immigrants arriving from France,
c. It copied luxury fragrances and sold them cheaply.
D. There was a wider range of fragrances available here than elsewhere.
80. Which city is being described in this sentence?
‘The perfum e industry developed here because the city pr odu ced materials
and equipment necessary for perfume production
A. Paris B. London c. Venice D. New York
B. WRITTEN TEST
I. CLOZE TEST (20 PTS): Read the text below and think of the word which
best fits each space. Use only ONE WORD for each space.
PASSAGE 1
Sir Alexander Fleming (1881 - 1955) is (1 )__________ with leading the way
in the use of antibiotics to treat bacterial infections. His discovery of penicillin in
1928 came at a time when many people died of tuberculosis, diphtheria and other
infectious diseases which made such a cure highly sought after.
After obtaining his medical degree in 1906 at St Mary's Hospital Medical
School in London. Fleming began to work (2) _____ • antibacterial
substances which could be used with humans. He was in the middle of his career
when the First World War began, but was fortunate to be able to continue his
research while serving in the Royal Army Medical Corps. He made his first
major discovery in 1921, when he identified and isolated lysozyme, an enzyme
found in human tears and saliva. (3) _________ _ antibiotic activity helps to
prevent infections.
It was not until seven years later, however, that Fleming became internationally
famous. He was working with the bacterium Staphylococcus aureus (4 )_______
he noticed that it was killed off by a green fungus, Penicillium notatum, which
has contaminated the culture. Further investigation showed that .there was a
substance in the fungus which prevented.the growth of the bacteria, even when
the substance was diluted 800 times.
173
The development of penicillin, which derives its name ( 5 )__________ the
fungus, must also be (6 )__________ to Ernst Chain and Howard Florey. 1 he
work of these two men revolved (7 )__________ isolating the active ingredient
in the fungus so that it could safely be administered to humans. They finally
achieved this, and in 1945 Chain, Florey and Fleming were jointly ( 8 )________
the Nobel Prize.
Since this ground-breaking work, scientists have discovered numerous further
antibiotics to treat a variety of bacterial diseases. All of these discoveries,
however, are grounded in the work o f Fleming, and even today he is (9 )_______
up to as a leading figure in the treatment of infectious diseases. Indeed, a
museum has now been opened at the (1 0) __________ of his old laboratory at St
Mary’s in Paddington, London.
PASSAGE 2:
Volcanic eruption has been a constant threat to our natural environment for
millions of years, but seldom in recent times (1 1) __________ a volcano erupted
with the ferocity o f Krakatoa.
Krakatoa. (1 2)__________ is a volcanic island group in Indonesia, erupted
on 27th August 1883. (1 3) __________ only was the explosion (1 4) ___________
loud that it was heard as far away (more than 3,000 km) as Perth in Australia, but
it is also recognised as (1 5)__________ the loudest sound (1 6 )___________
recorded.
Tens o f thousands of people in the region were killed many (1 7) __________
in the enormous tsunamis which the eruption produced - tsunamis which
eventually reached South Africa and the English Channel.
The explosion also had a major effect on the (18) __________ world's
weather system. The volcanic dust in the atmosphere reduced the (1 9) ________
of sunlight reaching the earth’s surface, reducing global temperatures by more
than one degree centigrade. Only after five years had passed (2 0)__________
global temperatures begin to return to normal.
II. WORD FORMATION: (20PTS)
PART 1: Complete each sentence, using the correct form of the word in
parentheses.
1. She also points out that mandatory .................................................................
fuel pumping creates jobs for all the people who pump the fuel. (SERVICE)
2. There is concern that the judges might .............................................................
their power. (USE)
3. Constant correction by a teacher is often ..........................................................
as the student may become afraid to speak at all. (PRODUCE)
4. It sees restructuring of Urenco as the means to convert a ................................
business into one with potential for growth and profit. (LOSS)
5. Like all tyrannical leaders, he demanded .........................................................
obedience from his followers. (QUESTION)
174
6. She's such a .............................................................................. that she notices
even the tiniest mistakes. (PERFECT)
7. Workers are fully exposed to chemical toxins and hazardous machines, and
suffer sickness, .......................................................... and death at the highest
rates in world history. (FIGURE)
8. By sheer luck, the gas released in Oklahoma City was blown into a mostly
..................... ....... ... ..........................area. (HABITAT)
9. His methodical ............................................................................. was another
point of divergence from the Impressionists and he devoted many studies to
creating the composition. (PAIN)
10. Hundreds of sailors and Marines yesterday loaded supplies and equipment
aboard th e .....................................................................ships. (AMPHIBIAN)
PART 2: Complete the passage with appropriate forms from the words given
in the box.
absen t observe variable describe
fa ir pre dict alternate enthuse
A live broadcast o f any public event, such as a space flight or sporting occasion,
is almost (11) ............... accompanied by the thoughts of a commentator. This
may be on television, along with the relevant pictures, or ( 12)...............on radio.
The technique involved differs between the two media, with radio broadcasters
needing to be more explicit and ( 1 3)............... . because of the (1 4)...............
of visual information. TV commentators do not need to paint a picture for th eir
audience; instead their various (1 5)........... s...........should add to the images that
are already there. There will sometimes be silences and pauses in TV commentary,
although these are becoming increasingly rare. Both types of commentator
should try to be informative, but should avoid sounding (16) ............... In sports
commentary, (17) .. ................ and impartiality to both sides is vital, but
spontaneity and ( 1 8)............... are valued by those watching or listening. Sports
commentators usually broadcast live in an essentially unscripted way, although
they may refer to previously prepared materials such as sports statistics. Because
of the ( 19)................... nature of live events, thorough preparation in advance is
vital. The internet has helped enormously with this aspect of the job. Anyone
interested in becoming a commentator should have excellent organisational
skills, the willingness to work (2 0)................... hours and a strong voice. .
III. ERROR CORRECTION: (10PTS)
The following passage contains 10 errors. Identify and correct them.
Line
1 Most astronomical events that influence the Earth, apart from the
2 occasional asteroid impact, do so in aregular fashion, such as day and
3 night, the tides and the seasons. There is, therefore, one event that has a
175
4 tremendous impact on the Earth - those of the total eclipse. For a few
5 minutes, broad daylight changes to complete darkness as the Moon
6 totally hide the Sun. This darkness is accompanied by many spectacular
7 effects, and it also provides a rare opportunity to physicists to make
8 observations that are impossible at any other time. However, as a total
9 solar eclipse is a sudden interruption of the day, it can also have an effect
10 on plants and animals that are used to the regular circle of day and night.
11 As total eclipses occur in average once every 360-years at any particular
12 location, there is little chance of any living thing becoming accustomed
13 to them. In fact, there are some amazing stories of the unusual behaviour
14 of animals as a total eclipse approaches. In Australia, for example, one
15 observer said, ‘I found myself have to calm a distressed parrot, which
16 fell to the ground a moment or so before the total eclip se’. Joanna Kale,
17 other observer, found her head surrounded by a cloud of insects that
18 dispersed when the Sun finally emerged from the eclipse. So, as these
19 examples show, the Suns presence has an astonishing influence on life
on Earth.
2. -____________
5 ._______________
8 . _ _____________
3 ._______________
6 ._______________
9. _______________
10. ______________
IV. SENTENCE TRANSFORMATION: (20 PTS)
Rewrite the following sentences using the words given.
1. It was not until five years had elapsed that the whole truth about the murder
came out.
-> Not for an othe r...............................................................................................
2. I have frequently made stupid mistakes like that.
—» M any's..................................... "........................................................................
3. Don’t tell the boss anything about this. (BREATHE)
- > ........................................................................................................................
4. It seemed the young man was feeling bitter about his family background.
(SHOULDER)
—» The young man appeared ...............................................................................
5. He has an obsession about the dishonesty of lawyers. (BEE)
—> H e ............................................................................. dishonesty o f lawyers.
6. He was exasperated with being criticized in public all the time. (TEETH)
-> He was fed.....................................................................................................
7. It’s possible the looming crisis won’t ever actually materialize.
-» It’s not beyond...............................................................................................
176
8. I do n't remember much about my mother, but I do remember she was very
kind and loving towards US.
—> What littl e..............................................................................................-a.........
9. You have said exactly the right thing. (NAIL)
-> You....................................................................................................................
10. They designed the stadium to make hooliganism impossible. (SUCH) z
—> The stad ium .......................................................................................................
TRƯỜNG THPT NGUYỄN THƯỢNG HIÉN - T P. Hổ CHÍ MINH
A. M U LTIP LE C H O IC E QUESTIONS
I. WORD CHOICE
1. In any profession, you have to learn to take t he______ with the smooth.
A. hard B. stodgy c. rough D. coarse
2. The new secretary seems a bit lazy, she doesn ’t rea lly _______ her weight.
A. push B. give c. act D. pull
3. A bad management de cis ion _______ disaster for the company.
A. drew B. spelt c. lay D. sprang
4. There was n o _______ difference between the original and the copy.
A. knowable •' B. discoverable c. discernible D. understandable
5. Jessica was scraping the bottom of th e______ for an excuse saying she had a
hairdresser’s appointment.
A. glass B. pitcher c.j u g D. barrel
6. Daisy says she didn’t want to be chosen, but it’s just sou r_____; she did really.
A. grapes B. lemons c. raisins D. milk
7. Harold realized too late that he had sold the van too cheaply: but there was no
point in ____ over spilt milk.
A. sobbing B. weeping c. screaming D. crying
8. I get so stressed at work it’s hard sometimes to _______ in the evenings.
A. unwind B. undo c. undergo D. untie
9. Going down white-water rapids in a canoe must be e xtrem ely _______ ! Does
your heart start beating really fast?
A. trivial B. mundane c. sedentary D. exhilarating
10. Was it always a n _______ of yours to play for France?
A. urge B. adoration c. anticipation D. aspiration
IL STRU CTURE AND GRAMMAR (1 Opts)
Choose the best answer A, B, c or D to complete each blank.
1. The engagement ring he gave her ______ been more beautiful.
A. co uld n't have B. might have c. shouldn’t have D. must have
177
2. He doesn't like children chatting in class.______ he will not put up with.
A. Such B. Those c. That D. This thing
3. Despite his disability he tried to lead _____ as possible.
A. as normal life B. a normal a life
c. as normal as life D. as normal a life
4. Since I ______ him, I have never seen him so angry like that.
A. have known B. knew c. know D. am know ing
5. Th ere 's no thing______ now - we 'll have to buy a new car.
A. to do B. to be done c. to have been done D. to be doing
6. ______ a high level o f blood cholesterol.
A . It is eggs that contains B. It is eggs that contain
c. Those are eggs they contain D. It'are eggs that contain
7. The work area______ cordoned off. Some passers-by could have been injured.
A. must have been B. might have been
c. could have been D. should have been
8. There was a sudden ba ng ______ me up during the night.
A. woke B. waking c. that woke D. that wakes
9. I f______ by hunger, you might consider eating a frog.
A. they are driven to desperation B. driving to desperate
c. you are driving to desperation D. driven to desperation
10. His story was so sad it almost had US______ .
A. to cry
B. cry c. crying D. have cried
III. PREPOSITION
Choose the best answer A, B, c or D to com plete each blank.
1 .1 really m us t______ up my Japanese before we visit Japan next year.
A. bottle B. brush c. dig D. tighten
2. It was supposed to be a private meeting but he ju st______ !
A. barged in B. broke o ff c. crowded around D. whiled away
3. An Industrial society which makes goods that are not designed to last is
known as a _____ society.
A. fallout B. takeaway c. set-aside D. thro waway
4. He'd threatened to divorce her but I never thought h e' d______ with it.
A. come through B. get through c. go through D. make through
5. The prime minister has said that the government is com mi tted_________ the
preservation of the country's national interests.
A. on B. with c. to D. in
6. I was about to pay for the shopping when it sudd en ly______ me that I'd left
my wallet at home.
A. dawned on B. dawn up to c. came up to D. came on
7. It's a good idea t o ______ people before taking them into yo ur confidence.
A. tumble to
B. size up c. bank on D. root out
178

8. These measures have been taken with a view______ increasing the company's
profits.
A. o ff B. in c. of D. to
9. All our household goods are ins ure d______ accidental damage.
A. just in case B. for c. from D. against
10. When the chairman ran o ff with his secretary, the Board tried to____the matter.
A. tuck away B. hush up c. shut down D. switch off
IV. COLLOCATIONS
Choose the best answer A, B, c or D to complete each blank.
1. John takes a _______ seat in his marriage. His wife makes all the decisions in
the family.
A. tail B. rear c. front D. back
2. Catherine Rhodes is considered to be one of the b ig ______ in the fashion
industry.
A. rifles B. guns c. pistols D. weapons
3. Martha didn’t get the job at the florist’s, but she’s got several ____ in the fire.
A. logs B. irons c. coals D. embers
4. He d idn 't intend to make a speech. If asked to, he would make a few remarks
_______ , but that was all he would do.
A. off the c uff B. on the off-chance c. off and on D. off his rocket
5. In a situation like this, there are n o _______ rules. You ju st have to use your
own discretion.
A. clear and cut B. hard and fast c. up-and-coming D. out-and-out
6. She demanded to be promoted to a post o f responsibility, otherwise she would
_____ about her affair with the Prime Minister.
A. reveal the cake B. spill the beans
c. let loose the pussycat D. spit out the bit
7. Her wages were so low that she had to take a second job just to ____ .
A. tie up loose ends B. meet her match
c. make ends meet D. be on the loose
8. “Why don ’t we go for a picnic this weekend?”
- “That’s what I w as just about to suggest. Great mi nds_____
A. think alike B. think the same
c. meet each other D. meet likewise
9. Two escaped prisoners are cur ren tly_____ the police.
A. at ease with B. out of respect for
c. on a par with D. on the run from
10. The studio didn ’t publicize the film but its reputation nonetheless spread
A. at a distance
c. by word of mouth
B. from memory
D. by force
179
IV. RE AD IN G (lO pts )
Rea d the follo wing pa ssa ge and ch oo se the best an sw er for eac h of the
qu estio ns below .
PASSAGE 1
Ocean wate r plays an indispensable role in supporting life. The great ocean
basins hold about 300 million cubic miles of water. From this vast am ount, about
80,000 cubic miles of water are sucked into the atmosphere each year by
evaporation and returned by precipitation and drainage to the ocean. More than
24,000 cubic miles of rain descend annually upon the continents. Th is vast
am ount is required to rep len ish the lakes and streams, springs and water tables
on which all flora and fauna are dependent. Thus, the hydrosphere permits
organic existence.
The hydrosphere has strange characteristics because wate r has properties
unlike those of any other liquid. One anomaly is that water upon freezing
expands by abou t 9 percent, whereas most liquids contract on cooling. For this
reason, ice floats on wate r bodies instead of sinking to the bottom. If the ice
sank, the hydrosphere would soon be frozen solidly, except for a thin layer of
surface melt wate r during th’e summ er season. Thus, all aquatic life would be
destroyed and the interchange of warm and cold currents, which moderates
climate, w ould be notably absent.
Another ou tst an ding characteristic of water is that w ater has a he at capacity
which is the highest of all liquids and solids except ammonia. This characteristic
enables the oceans to absorb and store vast quantities of heat, thereby often
preventing climatic extremes. In addition, water dissolves more substances than
any other liquid. It is this characteristic which helps make oceans a great
storehouse for minerals which have been washed down from the continents. In
several areas of the world these minerals are being commercially exploited. Solar
evaporation of salt is widely practised, potash is extracted from the Dead Sea,
and magnesium is produced from sea water along the American Gu lf Coast.
1. The author’s main purpose in this passage is to ___________ .
A. illustrate the importance of conserving water
B. describe the properties and uses o f water
c. comp are water with other liquids
D. explain how water is used in commerce and industry
2. The phrase “this vast amount” in line 4 of paragraph 1 refers to __________ .
A. 80,000 million cubic miles of water B. 24,000 cubic miles o f rain
c. 80,000 cubic miles of water D. 300 million cubic miles of water
3. The word “replenish” in paragraph 1 can best replaced by________ .
A. fill again B. replace c. evaporate D. form
4. According to the passage, fish can survive in the oceans b ecause________ .
A. evaporation and condensation create a water cycle
B. there are currents in the oceans
c. they do not need oxygen
D. ice floats
180
5. Which of the following is NOT mentioned as a characteristic of water?
A. Water can absorb heat B. Water is good solvent.
c. Water contracts on cooling D. Water expands when it is frozen
6. The word “outstanding” in paragraph 3 is closest in meaning t o ________ .
A. exceptionally good B. special c. amusing D. important
7. According the passage, the hydrosphere is NOT .________ .
A. the pail o f the earth covered by water B. responsible for all forms of life
c. in danger of freezing over D. a source o f natural resources
8. The author’s tone in the passage can best be described a s________ .
A. dispassionate B. speculative c. biased D. dogmatic
9. The autho r organizes the passage by_______ .
A. juxtaposition o f true and untrue ideas B. comparison and contrast
c. general statement followed by examples D. hypothesis and proof
10. Which of the following statements would be the most likely to begin the
paragraph immediately following the passage?
A. Water has the ability to erode land
B. Droughts and flooding are two types of disasters associated with w ater
c. Another remarkably property o f ice is its strength
D. Magnesium is widely used in metallurgical processes
PASSAGE 2
THE CHANGE IN ART AFTER WORLD WAR II
In the 1930s, before the onset o f war, rationing, and army drafts, art reflected
the somewhat serene lives of the people. Mundane scenes such as factory
workers or office settings were routinely painted to depict the era. They were
reminiscent of the people living a routine life in middle-class, ordinary settings.
Yet in 1939, fighting spread throughout the world. War and the subsequent
struggles for power, existence, and peace brought great unrest for countries
around the world following World War II. With the changes wrought by war,
many countries felt the need to convey a new, postwar image. It was from this
need that abstract expressionism evolved as a modem and recognized art form.
Abstract artist Jackson Pollock gave a clear picture of the emergence of
abstract art when he said, “The modern painter cannot express this age - the
airplane, the atom bomb, the radio - in the old forms of the Renaissance or of
any other past culture. Each finds its own technique.” It was the art of this
revolutionary painter that helped define the abstract movement. Postw ar artists
like Pollock developed free-form aesthetics by abandoning conventions of past
styles while maintaining focused, self-reflexive qualities and the feelings of each
individual artist. The method for creating abstract art involved painting free of
religious, political, and popular subjects. The paintings were instead comprised
of bright colors and shapes, characterized by personal expression rather than the
developm ent of a predictable art style. Much personal empowerment grew out o f
this profound freedom o f expression.
181
After World War II and during the uncertainty of the Cold War, the world
tottered back and forth between stability and instability. People felt great anxiety
amidst their growing prosperity. They viewed the modern art o f the time as bold,
triumphant, and self-assured. Although the work seemed to exude postwar
confidence, artists portrayed profound unease and viewed their work much
differently. Their images were the expression of desperation in the midst of a
tough reality inspired by unrest and contrasted with material growth. The
psychology of the abstract art form emerged from this altered mindset that was at
once strong and vulnerable, confident and subdued. Consequently, artists at the
time had the need to feel their experiences in ways that were intense, immediate,
direct, subtle, unified, and vivid. “Painting is a state of being ... painting is selfdiscovery.
Every good artist paints what he is,” stated Pollock. Abstract
expressionism, as the new art style became known, was a way to embody the
artist's yearning for stability in an unstable world as well as a way to emphasize
his own personal ideas and use those as expression.
Pollock's chi ef ambition in his art was to incorporate opposition. He did this
by pairing order with chaos, reason with passion, and modernism with
primitivism. Similar to other abstract artists, he preferred to portray notions of
the subconscious, giving free reign to forgotten personal memories and psychic
impulses. George Tooker, another artist o f the time, painted The Subway, which
illustrated postwar expectations of individuality and conformity. The affluence
of the nation’s newfound economic success combined with anxiety over political
instability to form a dual consciousness that is said to haunt America's identity
still. Each head a set of signature styles that expressed personal and societal
isolation of the artist in abstract ways.
During this time, modern art became identified widely as “American” art,
having its focal point primarily on the nation. The Museum of Modern Art in
New York began to ship abstract expressionistic works to be displayed in places
like Milan, Madrid, Berlin, Amsterdam, Paris, and London. Some critics
overseas were dismayed, stating that this type of abstract art was not new. As
this art was practiced elsewhere, they continued by saying it was not good
quality painting and was not purely American. One writer hailed typical
American abstract art as “heir of the pioneer and immigrant.” Another saw the
artists as heroic rebels, comparing them to movie stars of the same caliber as
James Dean and Marlon Brando or teen idols such as Elvis Presley.
As the US was celebrating a highly contradictory mix of freedom and
individuality, abstract expressionism became a political pawn of sorts. The art
reflected the ambiguity of the world at the time as war-ravaged countries worked
to recover their economy and people worked to achieve a normal state of life.
The artists of abstract expressionism effectively captured the emotion of the
nation as it emerged from a time o f stress and tried to form an updated image.
1. The author discusses art from the 1930s in order t o ______
A. demonstrate the drastic change in art
B. explain the change in America’s culture
182
c. describe the hardships of the people
D. list the events that transpired
2. The word ‘conv ention s’ in the passage is closet in meaning t o ______
A. perceptions B. agreements c. situations Đ. traditions
3. Which of the following can be inferred from paragraph 2 about the change in art?
A. Artists sought ways to distinguish their art from previous artists.
B. The painters used traditional design elements in whole new ways,
c. Consistency in art overpowered the need for originality.
D. Artwork reflected the personal empowerment of the artist.
4. The word ‘exude’ in the passage is closest in meaning to_____
A. discourage B. portray c. replace D. instruct
5. The word “afflu ence” in the passage is closest in meaning to _______
A. wealth B. learning c. position D. stature
6. According to paragraph 3, although the work of abstract artists appeared
confident, it was in fact
A. identical to the nation’s certainty
B. representative of the country’s wealth
c. reflective of the anxiety o f the era
D. expressive o f the artists’ low self-esteem
7. All of the following are mentioned in paragraph 4 as Pollock’s techniques
painting EXCEPT:
A. Stark displays of contrasts and opposition
B. Feelings and impulses from within the mind
c. Emotion mixed with ordinary scenes
D. Intense emotions from personal experiences
8. The word ‘each in the passage refers t o ____
A. artist B. dual consciousness
c. physic impulse D. notion of the unconscious
9. According to paragraph 5, abstract expressionism was critiqued for being
A. widely followed and admired B. labeled “American”
c. exhibited worldwide D. claiming to be modern
10. The word “ its” in the passage refers to
A. ambiguity B. nation c. modern art D. world
V. GUIDED CLOZE TEST
Read the following passage and choose the options tha t best complete the blanks.
PASSAGE 1
Our group (1 )_______ silent at the base of a narrow steel ladder that rose
vertically through the maze of girders at the south-east end o f Australia's Sydney
Harbour Bridge. We needn't have worried about the first part of the climb. IJp to
this (2) our guided tour had been little more than a stroll but now our
task was to face the ladder. It must have been at least fifty feet high. There were
handrails and our safety belts would be tethered to a cable to (3 )_______ a fall
183
but the water couldn't have been less than 250 feet below US and the (4 )_______
of climbing was daunting.
What lay at the top was stepping out on to the exposed uppe r arch of the
bridge, with blue sky all round and the water almost 262 feet below. We ought
to have found this out before embarking on what now seemed a singularly
(5 )_______ mission! My own (6 )________ was extreme, but, on this sparkling
morning, I saw no option but to climb to the summit of one of the world's bestloved icons - a
m iracle o f engineering recognized by people everywhere.
As I climbed the tension (7 )_____ out o f me; 1 was driven by an exhilarating
feeling of conquest. At the top, I dropped my ( 8)_______ to the vast pool of the
harbor below. It might ju st as well have been a mill pond from this height. We
stood on a small viewing deck in the warm sunshine, (9) _______ with
exciteme nt and arms raised as our guide took a (1 0) ______ photograph.
L A . fell B. came c. rendered
2. A. position B. period c. point
3. A. hold B. halt • c. reduce
4. A. perspective B. proposal c. probability
5. A. negligent B. reckless c. careless
6. A. acrophobia B. claustrophobia c. agoraphobia
7. A. exuded B. drained c. leaked
8. A. glare B. glimpse c. gaze
9. A. flushed B. burned c. drenched
10. A. celebrant B. celebratory c. celebrated
PASSAGE 2
THE VALUE OF WALKING
New research reveals that w alking just 9.5 kilometers (six miles) a we ek mảy
keep your brain sharp er as you get older. Research published in the October 13
online issue o f Neurology suggests that w alking may protect aging brains from
growing smaller and, in (1)_________ , preserve m emory in old age.
‘Brain size shrinks in late adulthood, which can cause memory problems,’
study author Kirk Erickson o f the University of Pittsburgh said in a news release.
‘Our findings should encourage further well-designed scientific (2 )________ of
physical exercise in older adults as a very (3) _______ approach for preventing
dementia and Alzheim er’s d isease.’ For the study, the team asked 299 dementiafree seniors
to record the (4 )_______ they walked each week.
Four years later, the participants were tested to see if they had developed
(5 )______ of dementia. Then after nine years had passed, scientists (6 )_______
the participants’ brains to measure size. At the four-year test, researchers
discovered subjects who walked the most had (7) their risk of
deve lopin g memory problems by 50 per cent. At the nine-year checkpoint, those
who walked at least 9.5 kilometers a week, had brains with a larger ( 8 )_______
than those who did n’t walk as much.
184
This is not the first study to (9 )_______ the benefits of walking in seniors.
For example, last spring, Harvard University found that women who walked
regularly at a (10)______ pace had an almost 40 per cent lower risk o f stroke.
1. A. result B.tur n c .sequence D.cas e
2. A. trials B. attempts c. searches D. courses
3. A. indicative B. promising c. fortunate D. ideal
4. A. distance B. length c. duration D. extent
5. A. signals B. factors c. signs D. features
6. A. skimmed B.scanned c. screened D.sounded
7. A. depressed B. declined c .reduced D. dropped
8. A. volume B. amount c. dimension D. quantity
9. A. advertise B. promote c .respect D. admire
10. A. brisk B. hard c. crisp D. brief
B. WRITTEN TEST
I. OPEN CLOZE (20pts): Fill in each gap with one suitable word
PASSAGE 1
THE SIGHTS AND SMELLS OF MOROCCAN MARKETS
As you approach the.city of Fes, you are overwhelmed by the incredible noise
of the traffic. Roads are unmarked, there are no traffic lights, and people are
sounding th eir car horns all the time, (1 )_______ a terrible din. The traffic noise
abates (2 )___ _ you enter the market, as the streets are too narrow for cars to
g e t( 3 ) _ ____.
The medina (market) of Fes has a unique smell that is hard to describe, and
even harder to forget. It smells (4 )______ raw meat that has been in the sun too
long, combined with the stench of rancid oil and olives. The sound of flies
buzzing is everywhere.
It is easy to escape from this smell if you enter a pharmacy. The shopkeeper
speaks both Arabic and English fluently. He listens intently to his customers and
then tells them (5)______ medicines they need. Giant bottles of herbs and spices
give (6 )_______ a fragrant scent o f fresh mint and lavender.
There is a restaurant nearby, where lunch is being served. Big platters of fresh
vegetables come out first, but many tourists (7 )______ clear of raw vegetables
in case they get ill from the water (8 )______ were washed in. Then, the main
course is served - curried chicken. It looks and smells appetizing and tastes
delicious. The diners eat it with relish, washed (9) _ _ _ _ _ with cola, even
though many of them have seen chickens being killed in the m arket (10)______ a
few minutes earlier.
PASSAGE 2
THE ORIGINS OF CRICKET . -
Cricket is often considered the quintessential English sport, but in fact, there
are many theories about how the game (I)-_______ . One theory suggested that
the game was devised by shepherds, who would hit balls of wool with their
185
crooks or ‘cricc es'. (2 )_______ speculates that it was started by children. The
idea was that the person holding the crook would (3 )_______ the wicket gate to
the sheep-pen from the person throwing the woolen ball. However, it is (4 )____
that the game we know today would have evolved from a single inspiration, and
cricket probably came (5) _______ as an amalgam ation of different games
played over the centuries.
The (6 )_______ reference in history to a game similar to cricket dates back
to 1300, when King Edward I, also (7 )_______ as Edward Longshanks, is noted
as playing a game called ‘creag’. In 1597 we come across a reference to the
game in Surrey, where a witness in a land dispute stated he remembered playing
‘kreke tt’ almost fifty years (8 )_______ . The first recorded match took place in
1646 and at the end of the century it was so popular that (9 )_______ missed
church to attend a game was (10)’______to a fine! The first written rules of
cricket w eren't drawn up until 1744.
II. WORD FORM (20pts)
PART 1. Supply the correct form of the word in the bracket in each sentence.
1. Eighty percent of people interview ed___________ declared that the use of
endangered wild animals such as elephants and tigers as a form o f entertainment
should be prohibited. (EQUIVOCAL).
2. There is evidence to indicate that most animals fa ce _______ on a daily basis.
(TREAT)
3. ______ at this school is severely punished, so remember to mind your
manners both during the lessons and outside the classroom. (OBEY).
4. There are various myths surrounding bats, th ose______ creatures of the night.
(ENIGMA)
5. Some species feed primarily on fruit and so aid in s eed_______ . (DISPERSE)
6. The draft law was passed by 134 votes to 19, with 5 _______. (ABSTAIN) .
7. For some young people, online games have b ecom e_______ addictive to the
extent that is threatening their mental and physical health. (RESIST)
8. You do not have to be an expert to grow cacti, and th e _______ need n't be
concerned about cultivating them because they are among the easiest of plants
to care for. (INITIATE)
9. Girls are almost invariably expected to express their feelings, but it would be
______ to believe that they are born weaker, more sensitive or with a greater
natural inclination to cry. (ERROR)
10. What children need is more encouragement in th ei r_______ years in order
not to succumb to feelings of depression later on. (FORM)
PART 2
Complete the following passage with the correct forms o f the given words.
fro nt sure intend affirm
solid labour except tend
186
We are (1 )________ by the experts that we are, as a species, designed for
face-to-face communication. But does that really mean having every meeting in
person? Ask the bleary-eyed sales team this question as they struggle (2 )______
through their weekly teambuilding session and that answer is unlikely to be in
the (3) ________ . Unless you work for a very small business or have a(n)
(4 )________ high boredom threshold, you doubtless spend more time sitting in
meetings than you want to. Of course, you could always follow business guru
Archie Norman ’s example. He liked to express (5 )________ with customers
queuing at the checkout by holding management meetings standing up.
Is email a realistic (6 )_____ ? I f s certainly a powerful tool for disseminating
information, but as a meeting substitute it’s seriously flawed. Words alone can
cause trouble. We're all full o f (7 )________ that can be ( 8)_________triggered
by others and people are capable of reading anything they like into an email.
There is also a (9 )________ for email to be used by people who wish to avoid
‘real’ encounters because they don’t want to be (1 0)___with any awkwardness.
III. ERROR IDENTIFICATION (lOpts)
There are ten mistakes in the following paragraph. Find them then give the
correction.
Even before the turn of the century, movies began to develop in two major
directions: the realistic and the formalistic. Realism and formalism are merely
general, rather than absolute, term. When using to suggest a tendency toward
either polarity, such labels can be helpful, but at the end they are still just labels.
Few films are exclusive formalist in style, and fewer yet are completely realist.
There is also an important difference between realism and reality, although this
distinct is often forgotten. Realism is a particular style, where physical reality is
the source of all the raw materials of film, both realistic and formalistic.
Virtually all movie directors go to the photographable world for their subject
matter, but what they do with this material - what they shape and manipulate it -
determines their stylistic emphasis.
Generally speaking, realistic films attempt to reproduce the surface of
concrete reality with a minimum of distortion. In photographing objects and
events, the filmmaker tries to suggest the copiousness of life himself. Both realist
and formalist film directors must select (and hence emphasize) certain details
from the chaotic sprawl o f reality. But the element of selectivity in realistic films
is less obvious. Realists, in short, try to preserve the illusion that their film world
is unmanipulated, an objective mirror of the actual world. Formalists, on the
other hand, make no such pretense. They deliberately stylize and distort their
crude materials so that only the very naive should mistake a manipulated image
of an object or event to the real thing.
1._______________
4 ._______________
7._______________
2.
5.
8 .________ ______
3.._______________
6 ._______________
9. _______________
10.
187
IV. SENTENCE TRANSFORM ATION (20pts)
Rewrite the sentences with the given words or beginning in such a way that
their meanings remain unchanged.
1. It was only when the film had ended that I remembered to switch off the oven.
. Not_______ _________________________________________________
2. They think the manuscript was written by a fourteenth century scholar.
(ATTRIBUTED)
The manuscript_________________________________________________
3. Have you seen my glasses anywhere by any chance? (HAPPEN)
You___________________________________________ _______ , do you?
4. Whatever happens, you must not go into my office without permission.
(ACCOUNT)
O n________________________________ ___________________________.
5. It's a good idea to take out a small loan to help start your business. (GROUND)
lfs_ _______________________________
6. I've become extremely good at missing the rush hour over the last few weeks.
(FINE))
Eveegat‘.missing the rush hour _________________________________
7. ARHcaigtr.Riidjyreally didn’t want to play cricket on Sunday, he agreed in the
endL(PEA D)'
D e s p it e__________________________________ ________
8 .1 tried to remember what has happened all those years before. (CAST)
1__________________ ________________________ ________ _
9. My parents are furious with me for getting a tattoo and so have grounded me.
(ARMS)
My parents, who________________________________________________
10. Since the company’s methods were exposed in a newspaper, people have lost
their good opinion of it. (DISREPUTE)
Since_________________________________________________________
TRƯỜNG THPT THỰC HÀNH CAO NGUYÊN - ĐẮK LẮK
A. MULTIPLE CHOICE (40 PTS)
I. PHONOLOGY (5PTS):
Choose the word whose underlined part is pronounced differently from that
of the others.
1. A. massage B. carriage c. voyage D. dosage
2. A. dimension B. expansion- c. confusion D. tension
3. A. increase B. ink c. pink D. thank
4.. A. apology B. classify c. testify D. verify
5. A. beloved B. naked c. ploughed D. learned
188
Choose the word which is stressed differently from the other three.
I. A. obvious B. notorious C. credulous D. numerous
2. A. dialect B. diagram C. diagonal D. diamond
3. A. Europe B. monument C. province D. minority
4. A, obsolete B. complete C. compete D. deplete
5. A. consent B. obstinacy C. condolence D. equality
II. WORD CHOICE: (5PTS)
1. Hoang Anh Tuan .... ...injury to win the Olympic
A. recovered silver.
B. suffered c. avoided D. overcame
it is essential to eat
2. In my f ath er's........ ,
vegetable every day.
A. idea B. opinion C. confidence D. principle
3. Last summer, we had ........ holiday in Spain
B. a two - weeks c. two -
A. a two week’s D. a two - week
week
4. He retired early........ ■ill - health.
A. on beha lf of B. on account of c. believe D. imagine
5. Their eventual choice of house w as _____by the time Peter would take to get
to the office.
B.consequent c. determined D. dependent
A. related
6. He set one alarm-
clock for five o
’clock and the other
for five past so as to
that he did not
oversleep.
A. assure B.ensure c. insure D. reassure
7. When Tim was eating a cherry, he accidentally swallowed th e_____ .
A. nut B. stone c. seed D. core
8. She loved tennis and
home.
could watch it till the came
A. she B. everyone c. horses D. cows
9. Could you close the
window? There is a bit of a
A. current B. wind c. draught D. breeze
were used as the framework
10. Thousands of steel
o f the new office block
A. beams B. girders c. stakes D. piles
III. STRUCTURE AND GRAMMAR: (5PTS)
Choose the best options to complete the following sentences.
1. I am going to have my ey e......... tomorrow.
A. to test B. test C. testing D.
2. Nobody could hear her.. ..... she spoke too quietly.
A. so B. however C. although D.
3. Keep silent! My father....... in his room.
A. is sleeping B. was sleeping C. has slept D.
4. He showed US the house............ he was born
A. which B. where c. in where D.
5. Life here is much easier than it...........
A. would be B. used to be c. was • D.
tested
because
will be slept
in that
had better
189
r~
6 . Amy...........car had broken down, was in a very bad mood.
A. who B. whom c. whose
7. He........ ever goes to bed before midnight.
A. almost B. hardly c. nearly
8. Y ou....... better be careful not to miss the train.
A. had B. would c. should
9.I'd rather you............. at my party last night.
A. be B. were c. had been
10. Everyone was going away on holiday this week,...............
A. wasn 't it B. wasn't be c. weren’t they
D. that
D. scarcely
D. did
D. have been
...........?
D. wasn’t she
IV. PREPOSITIONS AND PHRASAL VERBS: (5PTS)
1. I'd like to c ontrib ute_________ the school Red Cross fund.
A. on c. with
2. Madrid is famous
A. as
3. Will y ou _______
B. to D. for
________ its pulls
B. with c. for
_ after the party.
A. call me off B. get me on c. take me up
4. The ceiling fans were on, but unfortunately they only _
A. stirred up B. poured through c. turned into
D. of
D. pick me up
the hot, humid air.
D. cut back
5. Hardly had the van turned the corner when one of the back wheels_____
A. Broke away B. Turned around c. Came off D. Rolled down.
6. The government is thinking of bringing_____a law to make it compulsory for
cyclists to wear crash helmets.
A. on B. up c. in D. round
7. The businessman inspected the contract carefully before signing it.
A. looked over B. looked out c. counted on D. look for
8. What's happening, John? The smoke alarm is ringing but there's no fire!
A. putting out B. going on c. hanging up D. going up
9. The fire fighters worked hard to put the fire_______ .
A. off B. over c. on D. out
10. When do you think that we'll arrive at the hotel this evening?
A. get on B. go on c. get to D. get o ff
V. READING PASSAGE (10PTS)
Read the text below and choose the best answer to each question.
PASSAGE 1:
Continents and ocean basins represent the largest identifiable bodies on Earth.
On the solid portions of the planet, the second most prominent features are flat
plains, elevated plateaus, and large mountain ranges. In geography, the term
“continent” refers to the surface of continuous landmasses that together comprise
about 29.2% of the planet’s surface. On the other hand, another definition is
190
prevalent in the general use of the term that deals with extensive mainlands, such
as Europe or Asia, that actually represent one very large landmass. Although all
continents are bounded by water bodies or high mountain ranges, isolated
mainlands, such as Greenland and India-Pakistan areas are called subcontinents.
In some circles, the distinction between continents and large islands lies almost
exclusively in the size of particular landmass.
The analysis of compression and tension in the earth's crust has determined
that continental structures are composed of layers that underlie continental
shelves. A great deal of disagreement among geologists surrounds the issue of
exactly how many layers underlie each landmass because of their distinctive
mineral and chemical composition. It is also quite possible that the ocean floor
rests on the top of unknown continents that have not yet been explored. The
continental crust is believed to have been formed by means of a chemical
reaction when lighter materials separated from heavier ones, thus settling at
various levels within the crust. Assisted by the measurements of the specifics
within crust formations by means of monitoring earthquakes, geologists can
speculate that a chemical split occurred to form the atmosphere, sea water, and
the crust before it solidified many centuries ago.
Although each continent has its special features, all consist of various
combinations of components that include shields, mountain belts, intra-cratonic
basins, margins, volcanic plateaus, and block-vaulted belts. The basic differences
among continents lie in the proportion and the composition of these features
relative to the continent size. Climatic zones have a crucial effect on the
weathering and formation of the surface features, soil erosion, soil deposition,
land formation, vegetation, and human activities.
Mountain belts are elongated narrow zones that have a characteristic folded
sedimentary organization of layers. They are typically produced during
substantial crustal movements, which generate faulting and mountain building.
When continental margins collide, the rise of a marginal edge leads to the
formation of large mountain ranges, as explained by the plate tectonic theory.
This process also accounts for the occurrence of mountain belts in ocean basins
and produces evidence for the ongoing continental plate evolution.
1. What does this passage mainly discuss?
A. Continental drift and division
B. Various definitions of the term “continent”
c. Continental structure and crust
Đ. Scientific analyses of continental crusts
2. According to the passage, how do scientists define continents?
A. As masses of land without divisions B. As extensive bodies of land
c. As the largest identifiable features D. As surgical compositions and ranges
3. In line 8, the word “bounded” is closest in meaning to _________ _ .
A. covered B. convened c. delimited D. dominated
191
4. The author of the passage implies that the disagreement among scientists is
based on the fact that
A. each continent has several planes and shelves B. continents have various
underlying layers o f crust
c. continents undergo compression and experience tension
D. continents have different chemical makeup
5. The word “specifics" in the passage is closest in meaning to_____ .
A. specialities B. speculations c. exact details D. precise movements
6. The word “it" in line 23 refers to
Ạ. a chemical split B. the crust c. the atmosphere D. sea water
7. The author of the passage implies t ha t___________ .
A. it is not known exactly how the continental crust was formed
B. geologists have neglected the exploration of the ocean floor
c. scientists have concentrated on monitoring earthquakes
D. the e arth's atmosphere split into water and solids
8. According to the passage, what are the differences in the structure of continents?
A. The proportional size o f continents to one another
B. Ratios of major components and their comparative size
c. The distinctive features of their elements D. Climatic zones and their effect
on the surface features
9. The author of the passage implies tha t_______ ,
A. the process of mountain formation has not been accounted for
B. mountain ranges on the ocean floor lead to surface mountain building
c. faulting and continental margins are parts o f plate edges
D. the process of continent formation has not been completed
10. The word “evidence” in the last line is closest in meaning.to___________ .
A. eventually
B. confirmation c. exemplification D. challenge
PASSAGE 2:
In this era of increased global warming and diminishing fossil fuel supplies,
we must begin to put a greater priority on harnessing alternative energy sources.
Fortunately, there are a number of readily available, renewable resources that are
both cost- effective and earth - friendly. Two such resources are solar power and
geothermal power. Solar energy, which reaches the earth through sunlight, is so
abundant that it could meet the needs of worldwide energy consumption 6,000
times over. And solar energy is easily harnessed through the use of photovoltaic
cells that convert sunlight to electricity. In the US alone, more than 100, 000
homes are equipped with solar electric systems in the form of solar panels or
solar roo f tiles. And in other parts of the world, including many developing
countries, the use o f solar system is growing steadily.
Another alternative energy source, which is abundant in specific geographical
areas, is geothermal power, which creates energy by tapping heat from below the
surface of the earth. Hot water and steam that are trapped in underground pools
192
are pumped to tile surface and used to run a generator, which is produces
electricity. Geothermal energy is 50,000 times more abundant than the entire
known supply of fossil fuel resources. And as with solar power, the technology
needed to utilize geothermal energy is fairly simple. A prime example of
effective geothermal use is in Iceland, a region of high geothermal activity where
over 80 percent of private homes are heated by geothermal power. Solar and
geothermal energy are just two of promising renewable alternatives to
conventional energy sources. The time is long overdue to invest in the
development and use o f alternative energy on global scale.
1. What is the main topic o f this passage ?
A. The benefits of solar and wind power over conventional energy sources.
B. How energy resources are tapped from nature.
c. Two types o f alternative energy sources that should be further utilized.
D. Examples of the use o f energy sources worldwide.
2. According to the passage, why should we consider using alternative energy
sources?
A. Because fossil fuels are no longer available.
B. Because global warming has increased the amount of sunlight that reaches
the earth.
c. Because they are free and available worldwide.
D. Because conventional energy resources are being depleted, and they cause
environmental damage.
3. Which of the following words could best replace the word “ harnessing”
A. Capturing B. Harassing c. Depleting D. Exporting
4. According to the passage, what can be inferred about solar roof tiles?
A. They are being used in many undeveloped countries,
c. They are more expensive than solar panels.
B. They can convert geothermal energy to electricity.
D. They contain photovoltaic cells.
5. According to the passage, how is solar energy production similar to geothermal
energy production ?
A. They both require the use of a generator.
B. They both use heat from the earth’s surface,
c. They both require fairly simple technology.
D. They are both conventional and costly.
6. Where is the best place in the passage to insert the following sentence:
“Although the.US is not utilizing geothermal resources to this extent, the
Western US has a similar capacity to generate geothermal power ”
A. after the phrase “earth-friendly”
B. after the phrase “ growing steadily”
c. after,the phrase “by geothermal power”
D. after the phrase “global scale”
193
7. According to the passage, which of the following is true about solar power ?
A. There is very little o f it available in Iceland.
B. It is being used in 100, 000 private homes worldwide.
c. It is 6,000 times more powerful than energy from fossil fuels.
D. There is enough of it to far exceed the energy needs of the world.
8. What can be inferred about the use o f geothermal energy in Iceland?
A. It is widely used form o f energy for heating homes.
B. Twenty percent o f the geothermal energy created is used to heat businesses,
c. It is not effective for use in private homes.
D. It is 80 times more effective than traditional forms of energy.
9. What does the author imply about alternative energy sources ?
A. Many different types o f alternative energy sources exist.
B. Most alternative energy sources are too impractical for private use.
c. Alternative energy is too expensive for developing countries to produce.
D. Solar and geothermal energy are the effective forms of alternative power
10. What best describes the author’s purpose in writing the passage ?
A. To warn people about the hazards of fossil fuel use.
B. To describe the advantages and disadvantages of alternative energy use.
c. To convince people o f the benefits of developing alternative energy sources.
D. To outline the problems and solutions connected with global warming.
VI. GUIDED CLOZE (10 PTS)
Read the text below and decide which answer bẽst fits each space.
PASSAGE 1:
Clean freshwater resources are essential for drinking, bathing, cooking, irrigation,
industry, and for plant and animal (1)_______ . Unfortunately, the global supply
of freshwater is (2)_______ unevenly. Chronic water shortages (3)________ in
most of Africa and drought is common over much of the globe. The (4)_______
of most freshwater supplies - groundwater (water located below the soil surface),
reservoirs, and rivers - are under severe and (5)_______ environmental stress
because of overuse, water pollution, and ecosystem degradation. Over 95 percent
of urban sewage in (6)_____ countries is (7)______ untreated into surface waters
such as rivers and harbors. About 65 percent of the global freshw ater supply is
used in (8)_______ and 25 percent is used in industry. Freshwater (9)_______
therefore requires a reduction in wasteful practices like (10) irrigation,
reforms in agriculture and industry, and strict pollution controls
worldwide.

1. A. survive B. survived c. surviving D. survival


2. A. delivered B. distributed c. provided D. given
3. A. exist B. lie c. show D. stay
4. A. resources B. springs c. sources D. starting
5. A. increasing B. growing c. climbing D. ascending
6. A. growing B. miserable c. poverty D. developin
194
7. A. recharged
8. A. farming
9. A. reservation
10. A. ineffective
B. discharged
B. planting
B. conservation
B. illogical
c. charged
c. agriculture
c. preservation
c. irrational
D. discharging
D. growing
D. retention
D. inefficient
PASSAGE 2:
My wife and 1 have always enjoyed travelling by sea, and last year we
decided to go for a Mediterranean cruise (1)_________ our holiday was rather
expensive we thought that the high standard of accomadation, the first-class food
and the many interesting places we saw were well ( 2)______ the price we paid.
We found that most of the other passagers were friendly and interesting, but
there was a man, Mr James, who irrated and annoyed US, and not (3)_______ US
but all the others who shared our table at dinner. Whatever subject we talked
about, it seemed that he was an expert (4)_______ He, had, apparently, read more
books, visited more countries and studied more languages, than anyone else.
After a few days, we ate our meals in (5)_______, because nobody wanted to
begin a conversation that would soon be taken (6)_________ by this man.
Then my wife had an idea. Fortunately, the ship had a library, and she
suggested that we (7)___________ think of an unsual subject, look it up in
encyclopaedia and then talk about it at dinner. If it were unusual enough
Mr James could not (8)_________ know anything about it. The subject we chose
was “English Agriculture in the Eleventh Century” . At diner that night we
discussed this subject with (9) enthusiam. Mr James was silent. We
realised we had at last (10)________ to find something he knew nothing about.
B. W R IT TEN TEST (70PTS)
I. CLOZE TEST (70PTS):
Read the text below and think of the word which best fits each space. Use
only ONE WORD for each space,
PASSAGE 1:
THE WORLD OF DREAMS
Most people feel that when they dream, they (0) are carried off to another
world. (1)_________ the contrary, dreams are often connected to our daily lives.
(2 )________ our whole mind (3)____________filled with something, when we
are either very upset or when we are in good spirit, à dream will represent this
reality in symbols. It is often said that we benefit from dreams because they help
the spirit to heal itself, when things (5)_________ wrong. Dreams are therefore a
kind of escape, almost a holiday from (6) _________ life, with its fears and
responsibilities. It is, (7 )_______ , a strange (8 )_________ of holiday because
whether we have a wonderful time or whether it turns (9)_________ to be a
nightmare, we quickly forget it. Most dreams disappear forever, (1 0 )_______ .
you are one of those people disciplined enough to write them down as soon as
you wake up.
195
PASSAGE 2:
LOOKING FOR A JOB
1 finished university six month (11)________ , I've got a degree in business
administration. I enjoyed the course very much (12)_______ . I realize I should
have studied a lot harder! A few o f my friends have already got full-time jobs but
most, me, are still waiting (13)_______something suitable to turn up. Meanwhile,
1 make sure that I keep myself busy. I look through the job advertisements
(14)________ the newspapers every day and I also ask all the people I know to
tell me if they hear o f any vacancies (15)_______they work . What I am looking
for is something challenging and I would certainly be happy to move to another
city or even work abroad for a while. The (16)_______ is not so important at this
stage, provided I earn enough to live on, because I don’t want to continue.
having to depend on my parents, although they are (17)_______ generous to me.
At the moment, I'm working in a nearby restaurant two evenings (18)________
week, washing up and generally helping out, which brings a little money. The
other people working there are very friendly, and many of them are in the same
(19)________ _as me, so we have lots of to talk (20)
___________
II. WORD FORMATION (20PTS)
PAR T 1: Complete the sentences with the correct form of the word given:
1. She left school with g oo d______.______ (QUALIFY)
2. The bride w as ________ dressed in white. (STYLE)
3. The______ of the agriculture in our country is very necessary. (ELECTRIFY)
4. Jim ’s _________ of the crime only made matters worse. (DENY)
5. You can raise your hand to show that you need ______ _ _ (ASSIST)
6. We had the phone________ because we are moving tomorrow. (CONNECT)
7................... there are black holes in space . (THEORY)
8. One o f the aims of the organization is to provide_____ aid to the refugees.
(HUMAN)
9. Dozens o f_________ are injured on the city road each year. (CYCLE)
10. It was____o f him to leave the children on their own in the pool. (RESPONSE)
PART 2: Read the text below. Fill in each of blank space with the correct
form of the word from the box.
interrupt pleasant converse impress
mean research argue listen
Some recent research into (1 )................between men and women has
produced results which will surprise few women. Men are always interrupting
women when they talk. One ( 2 )........feels that men regard female talk as a kind
of conversational (3 ).............. They expect women to play a supporting role. So
a man interrupts in a display of (4)........... ...of control. Men also have a much
196
more (5) ________ listening style. Whereas a woman uses gestures or say:
“mm” , a man will say such thing as “ring” or “okay” thus settle the stage for an
(6)....... Conversation of this kind, then, hardly has (7 )_______communication.
Male talk is often (8)______ while women are more tentative, asking more
questions and tending to build up their replies on what the other person had said.
The research would indicate that women are better (9)___________ than men.
Yet, listening secretly to conversation between groups of women, one has the
(1 0)________ of several simultaneous monologues into man would be able to
get a chance to speak.
III. ERRORCORRECTIO N (10PTS):
There are ten mistakes in the following passage. Find and correct them.
Man has always depended on plants of food and many other useful products.
For this reason, farming is that of the world’s most important industries. At
firstly, early man did not know how to plant seeds and raise crops. He gathered
wild fruits and vegetables where he found them. Then man discovered how to
grow his own food. He planted seeds and waited for the crop to grow. For the
first time, he could be reasonable sure of his food supply. He could settle down
and build shelters in the places which he grew food. Population began to
increase; the demand for food became greater. Old-fashioned tools and farming
methods were inefficient in meeting the demand. So man cultivated more still
more land and invented more complicated machines to make his work easier.
Tractors replaced horses and other farm animals. Scientists studied and
experimented with plants. They told farmers how to control plant diseases, and
how too grow bigger and gooder crops. Now another man, with the knowledge
of plants and the help of machines, can cultivate hundred of acres. He can raise
plants which did not originally grow in the soil or climate of his community.
1.
4.
7.
2.
5.
8.
3._
6. _
9. _
10.
IV. SENTENCE TRANSFORMATION (20PTS)
* Finish the second sentence in such a way that it means exactly the same as
the sentence printed before it
1. Even though I admire his courage, I think h e’s foolish.
Much...................................................... ....................... .....................................
2. Experts think that all dogs evolved from wolves.
All dons....................................................
3. The university didn ’t prepare to consider his application due to his lack of
right qualifications.
H ad ... ....................................................................................................................
4. If you find it necessary, you can contact me on this number.
Should................................ ..................................................................................
197
* Write a new sentenc e similar in meaning to the one given, usin g the word
give n in brackets. Do not alter the word in any way.
5. Many people nowadays find it increasingly difficult to exist on the money
they earn (ENDS)
M an y...................................................................................................................
6. I've lost interest in going to the same place all the time. (FED UP)
I'v e ........................ .. .. .. .. ............................................... ............ .....................
1. Mary is proud of being a good cook. (PRIDES)
M ar y...................................................................................................................
8. The rain was coming down in torrents. (CATS)
The r ain ...............................................................................................................
9. They arrived at their destination alive and kicking. (SOUND)
They ...................................................................................................................
10. The police caught the burglar in the process of committing the crime. (RED)
The b ur gl ar .................................................................... .....................................
ĨRƯỞMG ĨH P Ĩ CHUYÊN NGUYỄN THIỆN THÀNH - TRÀ VINH
A. MULTIPLE CHOICE (40 pts)
I. WORD CHOICE (10 pts)
Choose the best options to complete the following sentences.
1. I asked him what to do but his instruction were so _ ___ that 1 still didn’t
understand.
A. inexplicable B. muddled c. garbled D. illegible
2. The police are looking for a sho rt,______ man with a beard.
A. average B. stocky c. petite D. tallish
3. Mary was so angry that she ______to see the manager.
A. requested B. required c. risked D. demanded
4. The whole building collapsed, but fortunately there were no_____
A. wounded B. casualties c. hurt D. victims
5. He left the meeting early on the unlikely_____ that he had a sick friend to visit.
A. claim B. excuse c. pretext D. motive
6. The weather is so awful that I don’t _____going out this evening.
A. fancy B. like c. try D. want
7. He sat in the corner,______with anger about the treatment he received from
his boss.
A. washing B. boiling c. simmering D. peeling
8 .1 w as___by his lack of intelligence. 1 couldn’t believe anyone could be so stupid.
A. shattered B. staggered c. speechless D. startled
9. This has been an incredibly ____month.
A. hectic B. flustered c. wound up D. harassed
198
ì 0. She may b e _____ but she seems to end up annoying a lot of people.
A. well-meaning B. ill-advised
c. smooth-tongued D. widely-known
2. STRUCTURE AND GRAMMAR (10 pts)
1.1 supposed, a s______ we all, that the meeting would be cancelled.
A. just B. would c. equally D. did
2. A study has been made o f the microbiology of rock surfaces a nd _____ stones.
A. weathered B. rugged c.jagged D. rough
3. Well, it_____ but will it taste the same?
A. is smelling good B. is smelling well
c. smells good D. smells well
4. Y ou __in this report to the managing director personally. Do you understand?
A. are to hand B. would like to hand
c. may have handed D. would sooner hand
5. _____________________ up early, he would have caught the early train.
A. Were he to get B. If he got
c. Had he not got D. Were he to have got
6. You should be very glad that Bob didn’t recognize you that night. Can you
imagine his fury if h e_____?
A. had B. did c. did not D. would have
7. The research suggests that infants____________ highly susceptible to allergy
at the first 3 months.
A. should be B. are c. will be D. being
8. When reaching the top of the hi ll,________ .
A. we suddenly caught sight of the sea B. it was the sea that extended below US
c. we extended the sea below US D. the sea came into view
9. Consumers should boycott all products_____ has involved the use of child
labour.
A. who manufacture B. whose manufacturers
c. which manufacture D. whose manufacture
10. Su ch_____ that the firefighters could not get near it.
A. hot was the fire B. the fire was hot
c. the heat was of the fire D. was the heat o f the fire
3. PREPO SITIONS AND PHRASAL VERBS (10 pts)
1 .1 jus t want to settle_____ and start a family before I got too old.
A. in B. through c. down D. up
2. She spent twenty years studying the history o f London. She knows it____out.
A. through B. inside c. all D. over
3. Capita! punishment was don e____ in Britain nearly hal f a century ago.
A. out for B. away with c. off by D. over from
199
4. He wants to pay the bill himself, but 1 won't h ea r_________ it.
A. from B. about c. of D. for
5. This situation is ana logous _____ the one we had faced last year.
A. for B. in c. from D. to
6. When John was arrested for drunken driving, he expected to lose his driving
license, but he w as _____with a fine.
A. let through B. let off c. got away D. kept away
7. My wife ch ec ks ___________ our elderly neighbour every few days to make
sure that he's alright.
A. on B. in c. at D. out
8. The rain ra n_____ the roof of the house.
A. on B. onto c. from D. into
9. You may co m e_____ different kinds of problems when you first settle in a
different country.
A. out with B. into c. up with D. up against
10. A Japanese company tried to ____________ the deal.
A. get in on B. get on with c. catch up on D. walk in on
IV. IDIOMS & COLLOCATIONS (10 pts)
Choose the right phrase in the box to complete each of the following sentences.
Use each phrase only ONCE and write your answer in the numbered box.
1. I'm sick a nd _______ of studying grammar.
A. cross B. ill C. tired D. bored
2. No, 1 d on't want to watch a war film. I don’t like to see too much blood an d____.
A. guts B. tears c. pain D. bone
3. M ice________ ! ,
A. give me creeps B. give me the creep
C. give me the creeps D. give me creep
4. When his parents are away, his eldest b rother___.
A. knocks it off B. calls the shots c. draws the line D. is in the same boat
5. I will do everything in m y_____ _____ to rescue the victims.
A. ability B. power c. mind D. effort
6. Be careful or he'll take you for a _________ .
A. trick B. cheat C. fraud D. ride
7. Just because Laura makes a lot o f money, everyone expects her t o _____ all
the bills.
A. foot B. stand C. run D. carry
8. My sister told me to pack some trousers and shirts for the trip ,_________ , not
to forget my toothbrush.
A. after all B. but more or less C. before long D. but above all
9. His English was roughly _ ___ with my Greek, so communication was rather
difficult!
A. level B. on a par c. equal D. in tune
200
10. I didn’t want to make a deci sio n_____ so I said I’d like to think about it.
A. on and o ff B. in one go c. there and then D. at a stroke
V. READING COMPREH ENSION (20pts)
RE AD ING 1: Read the following passage and choose the best answer
MAGINOT LINE
The Maginot Line was one of the largest military structures ever built, second
only to the Great Wall of China. It was a series of bunkers, forts, turrets, and
obstacles that spanned more than 450 miles of France's border with Germany.
Built between 1930 and 1940, it was one of the world's most impressive forts,
yet it proved to be almost useless.
The Maginot Line was named after the man who argued for its construction,
French Minister of War André Maginot. André Maginot had fought with the
French against the Germans in the First World War. Much of this conflict took
place along the. Western Front, which was a line of trenches across which the
two sides faced one another. Both sides dug in deep and each lost many men
over little ground. Conditions were horrid for all and there was a stalem ate for
many years as neither side was able to move the other.
Maginot never forgot these awful conditions. He wanted to build a line of
defenses that would give the French an advantage in a similar conflict. He
feared, rightfully so, that the Germans would attack France again. Germany's
population nearly doubled France's. The line of defenses that Maginot pictured
would allow a smaller French army to hold off a larger German force. In 1929,
Maginot convinced the French Parliament to fund his vision.
Though calling it a line makes it seem thin, the Maginot Line was in fact quite
deep. It was fifteen miles wide at some points and varied in structure. There were
outposts disguised as houses. These were manned by troops and rigged with
explosives. There were antitank rails and obstacles. These were planted in the
ground to prevent tanks and trucks from passing. There were bunkers armed with
mounted machine guns and anti-tank guns. These were for pushing back attackers.
And there were many large and small fortresses along the line. Each had mess
halls, lots of supplies, and air conditioning. The Maginot line would give the
French a supreme edge in the case of a head-on invasion by the Germans.
Unfortunately for the French, the Germans did not attack head-on. They
positioned a decoy army in front of the line to distract the French. While the
French waited, the Germans snuck a larger force through Belgium. Belgium is
France’s northeastern neighbor. The French did have some defenses along their
border with Belgium, but this part of the Maginot line was weak. The Germans
made quick work of these defenses. Within five days of their initial attack they
were well into France.
Once they were in France, the Germans attempted to seize the main forts
along the Maginot Line. They were not successful. The forts had proved to be
strong, but they failed to hold back the Germans. The Germans had taken Paris,
.201
France's cap ital city. Soon aft er the French comm ander ord ered his men to stand
dow n. He comm anded the French def end ers to leave the ir bas es along the
Ma gin ot Line. These soldie rs were then ta ken to POW c amps.
While the Ma gin ot Line did not wo rk in the way tha t the French had hop ed,
they did benefit by havin g built it. Belgium and Eng land were strong allies.
England had pledge d to pro tec t Belgium . Belgium dec lared its elf a neu tral
cou ntry, one which wante d to stay o ut of wars. When Germany invade d Belgium
to byp ass the Ma gin ot Line, the y vio late d Belgium 's neu trality . This led to
Eng land enter ing th e wa r sooner.
Tho ugh the Magino t Line is no longer used mil itar ily, many o f the buildings
rem ain. Som e o f the forts are now wine cellars or mu shr oom farm s. One was
turned into a disco club. Today the Maginot Line is ofte n used as a metaphor.
People may ref er to a faile d pro ject in which som eon e placed a lot of hop e as a
Ma gin ot Line. Also, the Ma gin ot Line lives on as the best-known symbol o f the
com mo n say ing th at "ge nerals always figh t the last war."
1. Which o f the follow ing s tate ments is tru e?
A. The Ma gin ot Line provided absolutely no ben efit to the French.
B. The Magino t Line is still used by the Fren ch arm y today.
c . Ma gin ot's e xperienc es dur ing WW1 led to his sup port for the line.
D. The French w ere b etrayed by the Belgians during World War II.
2. Which bes t e xpr esses the author's purpose in w riting the fourth par agr aph ?
A. To per sua de readers that the Maginot Line w as not at all thin.
B. To info rm reader s of the type s of def enses along the Ma gin ot Line,
c. To descr ibe w hat the inside of the Ma gin ot Line looked like.
D. T o com pare and con trast defenses along the Maginot Line.
3. Which best des cribes the weakn ess o f the Maginot Line?
A. It wa s built from che ap material and could not resist attack.
B. Poor screen ing let spies take control of the forts from within,
c. The line was not fortified along France's borde r with Belgium .
D. The French u nde restim ated the power o f new er German tanks.
4. Which of the f ollo win g is a fa lse statement?
A. André Maginot fought against the French in the First World War.
B. André Maginot served as the Minister o f War f or Fran ce.
G. T he Ma gin ot Line is named after And ré Magino t.
D. A fter WWI, André Maginot believed that Germ any would attack France again.
5. Wh ich best exp lain s why And ré Maginot wanted to build up Fra nce 's defens e
rather than the ir offe nse?
A. After W WI, Fran ce and Germany beca me history's gre ate st allie s.
B. Fra nce nee ded protection from the Germans and the hos tile B elg ian s.
c. Ma gin ot feared that French com manders might invade Ge rmany if they
had the pow er.
D. France had far fewer p eople than Germany and nee ded to save ma npow er.
202
6. Which best defines the meaning of the word stalemate as it is used in the
second paragraph?
A. When troops lose morale because they miss their families
B. When two sides are locked in a draw
c. When breathing is made difficult by foul odors
D. When one side has a large advantage over the other
7. Which best expresses the main idea o f the last paragraph of this text?
A. It describes how the Maginot Line is still used today.
B. It restates the main points of the text.
c. It discusses how the French used the Maginot Line during World War II.
D. It explains how the Maginot Line is used as a symbol.
8. How did the French lose control o f the Maginot Line?
A. The Germans overtook the fortresses with military power.
B. The Germans tunneled into the fortresses using new technology,
c. The French were starved out of the fortresses by German siege.
D. The French surrendered the fortresses.
9. Which was NOT pail of Germany's strategy to invade France in World War II?
A. Position a decoy force in front of the Maginot Line
B. Sneak troops into France by tunneling underneath the Maginot Line
c. Bring a large army through Belgium
D. Move very quickly
10. Which title best expresses the main idea o f this text?
A. Undefeatable: How the Maginot Line Became the World's Strongest Fortress
B. André Maginot: France's Greatest Patriot
c.Strong as the Weakest Link: The Mighty Maginot Wall and Its Shortcomings
D. Blood Feud: The History o f War Between France and Germany
READING 2: Read the fo llow ing passage and cho ose the best answer
THE DOT-COM ERA
The late 1990s was a volatile time for the Internet entrepreneur. There were a
large number of new high-tech and Internet-based businesses being started up,
some based on little more than dreams. And, nearly any entrepreneur with a
high-tech or Internet-based business plan could receive a large amount of money
from investors to develop the business. These businesses were called dot-coms”
after the last part of their Internet addresses, and that time in business history
has become known as the dot-com era. Most dot-com ventures failed as the
inc onsisten cies between high-tech dreams and realities clarified, but a handful
of entrepreneurs from that time remain successful today.
Many people look back at the dot-com era with a certain amount o f cynicism.
Some charge that there were entrepreneurs who deliberately mislead investors
about the potential of their business plans; that they had the ulterior motive of
203
getting their hands on large amounts of cash before anything else - a ploy that
some say was common in the dot-corn era. By 2001, most o f the dot-coms had
disappeared and were sarcastically referred to as 1 1 dot-bombs.” By the time the
hype died down, incredible amounts of money had been lost. This is an
unfortunate legacy of the dotcom era.
Why did companies like Microsoft, Apple, and Google find success, whilst so
many others disappeared? There is no concise answer, but one interesting
coincidence is that many of them were started by two or more entrepreneurs
working in conjunction with each other. At Microsoft, Bill Gates and Paul Allen
comp lemented each other; Apple had Steve Jobs and Steve Wozniak; and we
owe Google to the rapport between founders Larry Page and Sergey Brin.
Besides collaborating with others, today’s entrepreneurs need to be able to
think on. their feet because the business situation changes so quickly.
Entrepreneurs need to do constant research to keep track of changing trends.
Today 's entrepren eurs need to be hardworking and tenacious — the easy money
of the dot-corn era is history. Experimentation and innovation a critical in a
rapidly changing and highly competitive marketplace.
It seems that the mistakes of the dot-corn era have shown the world what the
high-tech economy truly is — and what it isn’t. It isn’t a place where anyone
with a dream can get rich. It is, however, a powerful and flexible business
environment where people with sound business ideas and effective business
models can work together to produce successful business innovations —
sometimes very successful ones indeed. But, as has always been true in the
business world, there is no substitute for thorough planning and hard work.
1. What is the author’s main purpose in writing this passage?
A. To expose the negative side o f the dot-com era
B. To explain the history and qualities of a type of entrepreneur
c. To convince the: reader to become an entrepreneur
D. To show how being an entrepreneur is easier today than ever before
2. According to the passage, it was easy to get money from investors in the dotcom era is
most probably b eca use ______
A. investors were mainly Internet entrepreneurs
B. investors d idn’t expect to get their money back in those days
c. investors were confused about the potential of high-tech business ideas
D. investors knew that any idea would succeed
3. According to the passage, many people look back on the dot-corn era with
cynicism because______
A. many companies misled their investors
B. they are jealous o f the success o f dot-coms
c. the level of technology was so much lower then
D. there were no true entrepreneurs in that era
204
4. According to the passage, some people referred to dot-corns as “dot bomb s’
by 2001 probably because______
A. most dot-corn companies were short-lived failures
B. dot-corn companies started up extremely quickly
c. some dot-corns became extremely successful
D. many people wished to eliminate dot-corn businesses
5. According to the passage, what do Microsoft, Apple, and Google all have in
common?
A, They all produce high-quality computers.
B. They are all cynical about dot-corns.
c. They all have a good rapport with each other.
D. They were all developed by more than one person.
6. Which quality of today’s entrepreneurs is NOT mentioned in the passage?
A. They need to be able to think on their feet.
B. They need to work in more than one company at a time,
c. They need to collaborate with others.
D. They need to do constant research.
7. According to the passage, what have we learned from the dot-com era?
A. That most high-tech arid Internet-based businesses will succeed
B. That anyone with a dream can get rich in today’s environment
c. That sound business ideas and effective plans are needed for success
D. That investors should give money freely to new companies
8. What does the word “their” in line 5 refer to?
A. Entrepreneurs B. Businesses c. Investors D. Ventures
9. The word “inconsistencies” in paragraph 1 almost me ans______
A. incompatibilities B. changeability c. inconstancy D. contradiction
10. The word “rapport” in paragraph 1 closely means ■
A. intimate relationship B. sympathy
c. bondage
D. mutual understanding
IV. CLOZE TEST: (20 pts)
CLOZE TE ST 1: Read the following passage, and choose the letter A, B, c,
or D on your answer sheet to indicate the correct word or phrase for
each of the blanks
A LACK OF COMMUNICATION
Recent research has revealed that a third of people in Britain have not met
their (1 )_____ neighbors, and those who know each other (2) ______ speak.
Neighbors gossiping over garden fences and in the street was a common
(3 )_____ in the 1950s, says Dr Carl Chinn, an expert on local communities.
Now, however, longer hours spent working at the office, together with the Internet
and satellite television, are eroding neighborhood (4 )____ . ‘Poor neighborhoods
once had strong kinship, but now prosperity buys privacy,’ said Chinn.
205
Professor John Locke, a sociai scientist at Cambridge University, has
analyzed a large (5 )_____ of surveys. He found that in America and Britain the
amount of time spent in social activity is decreasing. A third of people said they
never spoke to their neighbors at ( 6 ) _____ . Andrew Mayer, 25, a strategy
consultant, rents a large apartment in west London, with two flatmates, who
work in e-com merce. ‘We have a family of teachers upstairs and lawyers below,
but our only contact comes via letters (7 )_____ to the communal facilities or
complaints that w e'v e not put out our bin bags pro perly,’ said Mayer.
The (8)._____of communities can have serious effects. Concerned at the rise
in burglaries and (9) _____ of vandalism, the police have relaunched crime
prevention schemes such as Neighborhood Watch, (1 0 )_____ on people who
live in the same area to keep an eye on each others’ houses and report anything
they see which is unusual.
1. A. side-on B. next-door c. close-up D. nearside
2. A. barely B. roughly c. nearly D. virtually
3. A. outlook B. view c. vision D. sight
4. A. ties B. joints c. strings D. laces
5. A. deal B. amount c, number D. measure
6. A. least B. once c. all D. most
7. A. concerning B. regarding c. applying D. relating
8. A. breakout B. breakthrough c. breakdown D. breakawa
9. A. acts B. shows c. counts D. works
10. A. asking B. calling c. inviting D. trying

CLOZE TEST 2:
Read tile following passage, and choose the letter A, B, c, or D on your
answer sheet to indicate the correct word or phrase for each of the blanks
The joys and tribulations of being a pet owner! During our lifetime most o f US
have some experience of either owning a pet or being in (1) ___________
contact with someone who does. Is there such a thing as “the ideal pet”? If so,
what goes to make up the ideal pet? Various (2 )__________ influence one's
choice o f pet, from your reasons for getting a pet to your lifestyle. For example,
although quite a few pets are relatively cheap to buy, the cost o f (3 )_________
can be considerable. Everything must be (4 )____________ into account, from
food and bedding, tó vaccinations and veterinary bills. You must be prepared to
(5) ______________ time on your pet, shopping for it, cleaning and feeding it.
Pets can be demanding and a big responsibility. Are you prepared to exercise and
housetrain an animal or do you prefer a more independent pet? How much
(6) ___________ room do you have? Is it right to lock an energetic animal into a
(7) ____________ space? Do you live near a busy road which may threaten the
life of your pet? Pets (8 )_____________ turtles and goldfish can be cheap and
convenient, but if you prefer affection pets a friendly cat or dog would be more
206
(9) ___________ ____ . People get pets for a number of reasons: for company,
security or to teach responsibility, especially to children. Pets can be affectionate
and’ loyal and an excellent source of company as long as you know what pet
(10) _____________ you and your lifestyle.
- tribulation (n) /, tnbju' leijn/ nỗi đau khổ, nỗi khổ cực; sự khổ não
1.A. near B. close c. narrow D. tight
2. A. measures B. elements c. factors D. points
3. A. upkeep B. maintenance c. upbringing D. raising
4. A. considered B. held c. taken D. kept
5. .A. take B. waste c. occupy D.sp end
6. A. free B.spare c. empty D. vacant
7. A. confined B. detained c .reduced D. closed
8. A. so that B. as for c. as if Đ. such as
9. A. suited B. appropriate c. likely D. good
10. A. fits B. matches c. suits D. goes with
B. WRITTEN TEST (70pts)
1. OPEN CLOZE TEST (20 points)
OPEN CLOS E TEST 1: Read the texts below and think of the word which
best fits each space. Use only one word in each space.
MARKETS
Most people in Britain and the US now buy their fresh food in supermarkets
rather than traditional markets. But markets are still important (1 )______ the life
of many cities and towns and in recent years farmers' markets, where local
farmers and others sell (2 )_______ or home-made foods directly to the public,
have grown in popularity.
In Britain, most markets are held in the open air, in town squares or market
places. They usually take (3 )______only on market day, the same day each
week, and sometimes on Saturday, and the stalls are put (4 )_______ for each
occasion. Towns where markets have traditionally been held are called market
towns. Many still have a market cross, indicating where the market was
originally held, or an old market (5) ______, a covered area open at the sides.
Today, markets (6) _________ flowers, fruit and vegetables, fish and meat,
clothes and household goods.
Some towns and cities in Britain and the US have a (7) or indoor
market. These markets are usually open more days of the week than outdoor
markets and operate more like shops. Markets that sell cheap second-hand
goods, (8 )______ clothes, jewellery and books are called flea markets. In the
US, these are usually in buildings and open during normal shopping hours.
The word market is sometimes used in American English to (9 )______ to
any food shop. A (1 0)_________ of superstore in both Britain and the US is a
very large store or supermarket.
207
OP EN CL OS E TE ST 2: Read the texts below and think of the word which
best fits each space. Use only one word in each space.
Face-to-face conversation is a (1) _________ process: you speak to me, I
reply to you and so on. Two-way ( 2)______ depends on having a coding system
that is understood by both (3 )______ and receiver, and an agreed convention
about signalling the beginning and end of the (4 )______ . In speech, the coding
system is a language like English or Spanish; the convention that one person
speaks at a time may seem too obvious to (5 )______ . In fact, the signals that
people use in conversations and meetings are often non-verbal. For example,
lowering the pitch of the (6 )______ may mean the end of a sentence; a sharp
intake of breath (7 )______ signal the desire to interrupt, catching the chairman’s
(8 )______ may indicate the desire to speak in a formal setting (9 )___________ a
debate; a clenched fist may indicate anger. When these (10)
. signals are
not possible, more formal signals may be needed.
II. WORD FORMS (20pts)
PART 1: Complete the sentence with the correct form of the given word. (10 pts)
1. Janet had t o ________________ from team because of injury, (draw)
2. The mountain roads are o fte n_______________ in winter, (pass)
3. It is not possible for everyone to put their own______________ on the law.
(interpret)
4. Many people think that the famine was a result of the civil war, but it is
_________ . (truth)
5. You'll need to be a little more ____________ if you want to hold their
attention to your writings, (imagine)
6. Andi has co rre ctly___________ the order of a pack of cards in ju st 31.16
seconds, (memory)
7. A ____________ is a person who writes a book, etc. for another person,
under whose name it is then published, (write)
8. Don’t believe what they said. I think they a re __________ witnesses, (worth)
9. William Shakespeare is probably the most famous ___________ ■ in
history, (play)
10. T he ________________ answer to my question was “no”, (vary)
PART 2: Write the correct FORM of each bracketed word in the numbered
space provided. (10 pts)
use anxiety act emotion
treat respiration tense term
THE EFFECTS OF STRESS
There is a famous expression in English. “Stop the world, I want to get off .”
This expression refers to a feeling of panic, or stress, that m akes a person want to
stop whatever they are doing, try to relax, and become calm again. “Stress”
208
means pressure or (1 1)_______________ . It is one of the most common causes
of health problems in modern life. Too much stress results in physical,
(1 2) _____ _____ , and mental health problems.
Stress can affect the heart. It can increase the pulse rate, make the heart miss
beats, and can cause high blood pressure. Stress can affect the (13) ___________
system. It can lead to asthma. It can cause a person to breathe too fast, resulting
in a loss of important carbon dioxide. Stress can affect the stomach. It can cause
stomach aches and problems of (14) _________ food. These are only a few
examples of the wide range of illnesses and symptoms resulting from stress.
Emotions are also easily affected by stress. People suffering from stress often
feel (15) . They may have panic attacks. They may feel tired all the time.
When people are under stress, they often (1 6)_____ ___ to little problems. For
example, a normally gentle parent under a lot of stress at work may yell at a child for
dropping a glass of juice. Stress can make people angry, moody, or nervous.
(17) __________ stress can lead to a variety of serious mental illnesses.
Depression, an extreme feeling of sadness and hopelessness, can be the result of
continued and increasing stress. Alcoholism and other addictions often develop
as a result of ( 1 8 )________________ __________of alcohol or drugs to try to
relieve stress. Eating (1 9) _________ , such as anorexia, are sometimes caused by
stress and are often made worse by stress. If stress is allowed to continue, then
one’s mental health is put at risk.
It is obvious that stress is a serious problem. It attacks the body. It affects the
emotions. (2 0) _________ , it may eventually result in mental illness. Stress has a
great influence on the health and well-being o f our bodies, our. feelings, and our
minds. Then, try to reduce stress: stop the world and rest for a while.
JII. ER ROR IDENTIFICATION (lOpts)
Read the text below and find 10 words which should not be there. Underline
the 10 words and correct them.
The Statue of Liberty is the famous statue that stands in Liberty Island in
New York Harbour. At a height of 151 feet, it is extremely tall. The statue was a
gift to the United States from the people of France, and over the years it had
become a symbol of freedom. Many people think an American designed it, but it
was in fact created by a French artist, Frederic Bartholdi.
The designer tried many designs and models, included children and animals,
before he chose a female figure, wearing a gown and cloak, holding a torch high
above his head. The statue’s body looks like a Greek goddess. In the early drawings,
the statue's face was going to look like the president’s wife, and Bartholdi decided
to copy the face of Charlotte Beysser, Bartholdi’s mother.
Bartholdi decided to make the skin o f copper sheets. This was a good choice
because the workmen were able to lift and work with the sheets with no
difficulty. He considered other cheaper metals but thought they might be so hard
to work with because of its weight.
209
On October 28th 1886, the President of the United States opened the Statue of
Liberty in a ceremony in front of a huge, exciting crowd. The President gave a
long speech, and Bartholdi was asked to give one too but he refused because he
was a shy man. A fireworks display was delayed until November 1st because of
poor weather, but the parade went ahead as plan.
1.
4.
7.
2.
5.
8.
3.
6.
9.
10.
IV. SENTENCE TRANSFORMATION (20 pts)
A. Finish each of the following sentence in such a way that it is as similar as
possible in meaning to the sentence printed before it
1. Wash vegetables before eating them.
—> Vege tables__________________________ _______________________
2. New road developments pose a major threat to these ancient woodlands.
—> These ancient
You will have to give up the whole idea.
—* 1 have no
4. He had a strong impulse to open the letter.
■ —> He couldn’t re sis t______________________________________________
5. It can be conclude from the research, heart diseases correlate with smoking.
—> The research shows a _______________ ___________________________
B. For each of the following sentences, write a new sentence as similar as
possible in meaning to the original sentence, using the word given. This
word must not be altered in any way.
6. The diplomat has been arrested because it is believed he has been spying for
his government. (SUSPICION)
—> The diplom at_________________________________________________
7. He got up very early this morning. (CRACK )
—> H e_______________________
8. They remained close friends, despite having had many differences. (FALLEN)
—> Though . ________________________________________________
9. I resent the way that she clearly feels herself to be superior to me. (NOSE)
—>I a m ________________________________________________________
10. Fortunately his route took him very near where they lived. (LUCK)
—* A s________________________________________________________
210
TRƯỜNG THPĨ CHUY ÊN NGUYỄN QUANG DỈÊU - ĐỒNG THÁP
A. MULTIPLE CHOICE (40 pts)
I. WORD C HOICE (10 pts)
Choose the best options to complete the following sentences.
1. Whenever she feels stressful, she listens to Beethoven's n inth_____ .
A. classical music B. symphony c. folk song D. tunes
2. The transport strikes a r ea l_____ , which will make it very difficult for me to
get to work.
A. pain in the back B. nuisance c. last straw D. frustration
3. benefits include a new car and free health insurance,
A. Well B. Fringe c. Edge
4. Ex-smokers often show their
to smokers due to their great effort and
success.
D. Verge
A. pride B. gesture c. behaviour D. millitancy
5. Wouldn't it be nice if we all had the time and money to _____ off to France
for a week?
. A. swan B. worm c. wolf D. beaver
6. You really shouldn’t buy that car. I know the engine is fine, but most of the
bodyw ork has b ee n_____ away by rust.
A. eaten B. dissolved c. erased D. crumbled
7. Surgeons now can do miracles to their physical _____ which used to be
unbeatable.
A. conditions B. wrongdoings c. malfunctions D. abnormalities
8. As an eco tourism _____ , the park provides visitors with beautiful landscape.
A. site B. park c. place D. land
9. A trick in the mining industry is _____ to bring about a shortage o f coal in the
near future.
A. causing B. resulting c. threatening D. prospering
10. Because of an unfortunate _ ___ , your order was not dispatched by the date
requested.
A. hindrance B. oversight c. negligence D. transgression
II. STRUCTURE AND GRAMMAR (lOpts)
1. The early railroads w er e_____ the existing arteries of transportation: roads,
turnpikes, canals, and other waterways.
A. those short lines connected B. short lines that connected
c. connected by short lines D. short connecting lines
2. When wood, natural gas, oil, or any other fuel bu m s,____ _ with oxygen in
the air to produce heat.
A. combining substances in the fuel B. substances in the fuel that comb ine
D. a combination of substances in the fuel
c. substances in the fuel combine
211

3. If only the n eig hbor_____ a pet fish, not a dog which always barks.
A. is having B. were having c. had had D. would have
4. Not onl y_____ in the project, but healso wanted to become the leader.
A. did Jack involve B. had Jack been involved
c. was Jack involved D. was involved Jack
5. _____ , what would you bring back?
A. Think o f going to Africa
c. If you should go to Africa
6. Luisa does a full-time job and
A. looks after also the house
c. also looks after the house
7.
B. Imagine to go to Africa
D. Supposing you went to Africa
B. looks after the house either
D. looks after the house too
_____ dangerous the situation is, Jim always indulges himself in it head first.
A. In spite of B. No matter what c. However D. Though
8. Not until I received the letter o f announceme nt,____
A. I could recognize how bad the situation was
B. could I recognize how bad was the situation
c. how bad the situation was I recognized
D. did I recognize how bad the situation was
9. The com petitio n_____ ; but you failed to enter it.
A. must have provided you with opportunities
B. could have provided you with opportunities
c. should have provided you with opportunities
D. may have had provisions for your opportunities
10. _____ , we couldn't'make out the gist of the passage.
A. Carefully as Jack explained
B. As Jack carefully explained
c. Due to J ack ’s carefully explanations
D. Though carefully explained by Jack
III. PREPOSITIONS AND PHRASAL VERBS (lOpts)
1. The police___ a good deal of criticism over their handling of the demonstration.
A. came in for B. brought about c. went down with D. opened up
2. I can ’t _____ what h e’s doing; it’s so dark down there.
A. look into B. make out c. show up D. see through
3. Dave didn 't understand what Miss Smith was ge tting _____ so he asked her
to explain it again.
A. about B. for c. over D, at
4. We are _____ very good term s_____ _ our next-door neighbor.
A. in/ of B. in/ with c. on/ o f D. on/ with
5. I was t ake n1_____ when Michaela said I’d hurt her feelings.
A. out
B. along c. aback D. up
212

6. Eating goo d food is g ood in su ra nc e_____ sickne ss.


A. for B. again st c. with D. to
7. Don't com e_____ hard on the n ew work ers.
A. dow n B. up c . about D. with
8 My te ache r say s that I should sail ______ the exa m, but I ’m not so sure.
A. th rough B. to c. with D. in
9. I can’t re member the n ame o f the hotel w e stayed a t_____ the top of my head.
A. on B. in c. up D. off
10. I f it starts to r ain, m ake_____ a n earby cav e to wa it for it to pass.
A. out B. for c. up D. with
IV. IDIOMS & COLLOCATIONS
(lOpts)
Choose the right phrase in the box to
complete each of the following
sentences.
Use each phrase only ONCE and write
your answer in the numbered box.
word perfect
the bight lights a dark horse in a word
take it amiss
a shot in the dark
take it as read
word fo r word
reading between the lines the light in the end of the tunnel lost fo r words
1. My interv iew with the policem an too k longer than I had tho ught bec aus e he
wanted to cop y dow n my s ta te m en t______ .
2. My an sw er ______ is no.
3. ______ I sensed that she was trying to tell me tha t she wa s havin g pro ble m at
hom e.
4. He read the speech again and again until he was absolutely sure that he was
5. Wh en she was told she had won the priz e she was _ _ _ _ _ and tea rs welled up
in h er eyes.
6. I do n’t think we nee d to put ‘Inf orma l’ on the inv itations - mo st peo ple will
7. Gail is a bit of - she never talks abo ut her fam ily and no one kno ws
much ab ou t h er past. .
8. N o one had con sid ere d offering mail order before, so setting it up was a bit of
9. Af ter mo nth s of unem plo ym ent, the act or had been offered a sma ll part in a
TV film , and felt he could at least see _______.
10. P lease d o n 't _____ when I say that you r last repo rt nee ded a lot of rewo rking.
V. READING COMPR EHENSION (20pts)
RE AD ING 1: Read the following passage and choose the best answer
Just as optical fibers have transformed communication, they are also revolutionizing
me dicine. Th ese ultra-thin, flex ible fibers ha ve op en ed a win dow into the living
tissues of th e bod y. By inserting o ptical fibers through nat ura l opening s or sma ll
213
inc isio ns and threading them along the body’s established pathways, physicians
can look into the lungs, intestines, heart, and other areas that were formerly
inaccessible to them .
The basic fiber-optics system is called a fiberscope, which consists of two
bundles of fibers. One, the illuminating bundle, carries light to the tissues. It is
coupled to a high-intensity light source. Light enters the cor es of the high-purity
silicon glass and travels along the fibers. A lens at the end of the bundle collects
the light and focuses it into the other bundle, the imaging bundle. Each fiber in
the bundle transmits only a tiny fraction of the total image. The reconstructed
image can be viewed through an eyepiece or displayed on a television screen.
During the last five years, improved methods of fabricating optical fibers have
led to a reduction in fiberscope diameter and an increase in the number o f fibers,
which in turn has increased res olu tion.
Optical fibers can also be used to deliver laser light. By use. of laser beams,
physicians can perform surgery inside the body, sometimes eliminating the need
for invasive procedures in which healthy tissue must be cut through to reach the
site of disease. Many of these procedures do not require anesthesia and can be
performed in a physician's office. These techniques have reduced the risk and
the cost of medical care.
1. What is the main subject of the passage?
A. A revolution in communication B. New surgical techniques
c. The invention of optical fibers D. The role of optical fibers in medicine
2. In line 2, the author uses the expression have ope ned a wi nd ow to indicate
that the use of optical fibe rs____ .
A. has enabled scientists to make amazing discoveries
B. sometimes requires a surgical incision .
c. allows doctors to see inside the body without major surgery
D. has been unknown to the general public until quite recently
3. Which of the following is closest in meaning to the word incisio ns in line 4?
A. holes made on the body B. acts o f incising
c. wounds caused to the tissues • D. cuts made in surgery
4. The word them in line 6 refers to ____.
A. pathways B. optical fibers
c. physicians D. other areas of the body
5. According to the passage, what is the purpose of the illuminating bundle in a
fiberscope?
A. To carry light into the body B. To reconstruct images
c. To collect and focus light D. To perform surgery inside the body
6. Which of the following is closest in meaning to the word cor es in line 9?
A. Tips B. Centres c. Clusters D. Lines
214
7. Ac cording to the pas sage, how do the fiberscop es used tod ay d iffer from tho se
used five y ears a go?
A. T hey use brigh ter lights. B. T hey contain mo re fibers,
c . They are long er. D. T hey are larger in d iam ete r.
8. The word re so lu tio n in line 16 is closest in m eaning to which of the f ollowing?
A. Streng th B. Sha rpness c. Inc onven ience D. Eff iciency
9. Wh ich of the fol low ing is not me ntioned by the author as one of the
ad va ntages of laser surger y tec hniqu es?
A. They can b e performed in a physici an’s office.
B. They are safer than con ven tion al surgery.
c. They can often be perform ed witho ut a nesthesia.
D. T hey are relativ ely ea sy to teach to physicians.
10. W here in the passage does the author provide a basic description o f a fiberscope?
A. Line 1 -3 B. Line 6 - 7 c. Line 10-1 1 D. Line 1 7 -1 9
RE AD ING 2: Rea d th e fo llow in g pa ss ag e an d ch oo se th e bes t an sw er
SUBV ERSIV E ART
Subversiv e political and social messages are a m ainstay of the art comm unity.
In most modern dem ocr atic nations, where free speech is consi der ed an
inalien able right, such subversive art is easily found. Am erican film is a fine
exa mp le of this. Many Am eric an films are openly critica l of Am erican society
and its values. Apocaly pse Now, a film abo ut the Vietnam War, is a withe ring
attack on Am erica's involvement and con duc t in tha t conflic t. In a mor e modern
setting, African- Am erican film director Spike Lee makes film s which dep ict
Am erica as a racist and unjust society. Oth er arts are filled with exa mp les of
sub versive art as well. There are cou ntle ss paintin gs and scu lpture s which ope nly
attack Christian ity and oth er values which are con sidered to be core asp ects of
Am erica's identity. Punk Rock, which orig inated in Eng land, derided many of
England's mo st sacred of social valu es. Most fam ously, the 1970s pun k ban d The
Sex Pistols freq uen tly sang songs abo ut overthro wing the gover nm ent, eve n
goi ng so far as to describe the Queen of En gland as sub hum an.
Not all subversive art is as upfront in its social criticis m. Much of su bvers ive
art is quietly subversive. The re are a num ber of reasons for this. The first is, of
cou rse, eco nom ics. An artist who is too open or too veheme nt in his or her
criticis m of soci ety may find it hard to find an aud ience willin g to pay for suc h
art. In som e societi es there may be a fea r of re tr ib uti on, eit her by the
governme nt or by individual society members offe nded by the artist's me ssa ge.
No r are such fears unfoun ded. Author Salm an Rushd ie was forced to go into
hid ing after he wro te a book that Muslim s too k to be critica l o f their religion.
There fore, the sub versive messages in art are often hidden .
One com mo n method of hiding subversive me ssages in one's art is throug h
the use of an unr elia ble or unte nab le viewpoint. In this me tho d, the artist
215
actually presents a viewpoint that is in opposition to his or her own, but does so
in such a way as to render that viewpoint unbelievable in the eyes of the
audience. American author Mark Twain was a master at this. The narrator in his
most famous novel is a young boy named Huck Finn. Huck supports the values
of his society. For example, one of the justificatio ns used by Americans to keep
Africans enslaved was that they were somehow less human than whites. There is
a scene in which Huck finds his friend Jim, a runaway slave, crying to-himself
because he misses his children whom he has left behind in slavery. Huck is
surprised by th is, and comments that it seems remarkable that Jim, a slave, could
feel sadness for the loss of his children in the same way a white person would.
This observation is so absurd that it discredits the entire notion of slaves
somehow being less human than their masters.
Mark Twain was not the only artist to hide the subversive nature of his work.
In earlier times, prior to the printing press and the mass production of posters,
artists depended on the support of wealthy individuals to make their living. A
rich noble who appreciated the work o f an artist would become his or her patron
and support the artist, in return the artist was generally expected to create art that
the patron approved of, and herein was the problem.
These wealthy nobles represented the social elite, which in most cases was
exactly the element o f society which artists wished to criticize. One could hardly
expect a patron to pay for art that was openly critical of his social class, so artists
became experts in concealing the subversive message in their art. A close look at
any of the great artists of history, da Vinci, Shakespeare, Milton, etc. will reveal
many hidden elements o f social subversion.
1. In paragraph 1, all of the following are mentioned as examples of subversive
art EX CEPT______ .
A. anti-war films B. songs o f rebellion
c highly sexual poetry D. blasphemous sculpture
2. According to paragraph 2, what is the role of economics in subversive art?
A. It limits the candor with which artists express themselves.
B. It creates a market for increasingly shocking art.
c. It encourages people not to buy subversive art.
D. It encourages artists to be upfront in their social criticism.
3. Based on the information in paragraph 2, it can be inferred that subversive art
A. is often highly profitable for the artist
B. often evokes strong emotional reactions
c. is often banned by many governments
D. is mostly focused on religious topics
4. The word “retribution” in the passage is closest in m eaning to ______ .
A. criticism B. rejection c. revenge D. anger
5. The word “untenable” in the passage is closest in m eaning to ______ .
A. com plex
B. untrustworthy c. obscure D. ind efensible
216

6. According to paragraph 3, how did Mark Twain seek to subvert the values of
his society?
A. He used a highly unpopular narrator to discuss the topic o f slavery.
B. He presented the views o f his opponents in a ridiculous fashion.
c. He argued that slaves also held emotional attachments to their families.
D. He described much o f American society as evil and bitter.
7. Based on the information in paragraph 3, what can be inferred about Mark Twain?
A. He feared retribution from American society.
B. He was the first author to practice subversive art.
c. He opposed the American practice of slavery.
D. He was forced into hiding for his political views.
8. The word “this” in the passage refers to .
A. the fact that Jim misses his family
B. the fact that Jim would leave his family in slavery
c. the fact that slaves are considered less human than whites
D. the fact that Mark Twain opposed the values o f his society
9. Based on the information in paragraph 4, the term “p atr on” can best be
explained a s______ .
A. a person who loves art B. the target of subversive art
c. a person who supports artists D. a noble who represents the social elite
10. According to paragraph 4, why was it-especially important for early artists to
hide the subversive nature o f their art?
A. The governments o f that time were far less tolerant of criticism.
B. The people they criticized were their sole source of income,
c. They were unable to mass produce their art.
D. There were very few patrons who approved of subversive art.
IV. CL OZ E TEST: (20 pts)
CLOZE TEST 1:
If you've been told by your boss to improve your knowledge of a foreign
language you will know that success doesn’t come quickly. It generally takes
years to learn another language well and constant (1 )_____ to maintain the high
standards required for frequent business use. Whether you study in a class, with
audiocassettes, computers or on your (2) _____ sooner or (3) _____ every
language course finishes and you must decide what to do next if you need a
foreign language for your career.
Business audio Magazine is a new product designed to help you continue
language study in a way that fits easily into your busy schedule. Each audiocassette
(4 )_____ of an hour - long program packed with business news, features and
interviews in the language of your choice. These cassettes won ’t teach you how
to order meals or ask for directions. It’s ( 5 )_____ that you can do that already.
Instead, by giving you an opportunity to hear the language as it’s really spoken,
they help you to ( 6)_____ your vocabulary and improve your ability to use real
language relating to, for example, that all- important marketing trip.
217
The great advantage of using audio magazines is that they (7 )_____ you to
perfect your language skills in ways that suit your lifestyle. For example, you
can select a topic and listen in your car or hotel when away on business. No
other business course is as ( 8)_____ and the unique radio- magazine format is as
instructive as it is entertaining. In addition to the audiocassette, this package
includes a transcript with a business glossary and a study (9) _____ . The
components are structured so that intermediate and advanced students may use
them separately or together,
their ability.
(10) on
1. A. exercise B. performance c. practice D. operation
2. A. self B. individual c. personal D. own
3. A. after B. then c. later D. quicker
4. A. consists B. includes c. contains D. involves
5. A. insisted B. acquired c. asserted D. assumed
6. A. prolong B. extend c. spread D. lift
7. A. allow B. let c. support D. offer
8. A. adjustable B. flexible c. convertible D. variable
9. A. addition B. supplement c. extra D. manuscrip
10. A. according B. depending c. relating D. basing
* CLOZE TEST 2:
For Nigel Portman, a love of travelling began with wha t's called a ‘gap year’.
In common with many other British teenagers, he chose to take a year out before
(1) ___ __ to study for his degree.
After doing various job s to (2) _____ some money, he left home to gain some
experience of life in different cultures, visiting America and Asia. The more
adventurous the young person, the (3 )_______ the challenge they are likely to
(4 )_____ themselves for the gap year, and for some, like Nigel, it can (5 )_____
in a thirst for adventure.
Now that his university course has come to an end, Nigel is ju st about to
leave on a three-year trip that will take him right around the world. What's more,
he plans to make the whole journey using only means of transport which are
(6 )______ by natural energy. In other words, he'll be (7 )_______mostly on
bicycles and his own legs; and when there’s an ocean to cross, he won’t be
taking a (8 )_______ cut by climbing aboard a plane, he’ll be joining the crew o f
a sailing ship (9 )_________ _.
c. taking over
c. advance
c. greater
c. aim
c.cause
As well as doing some mountain climbing and other outdoor pursuits along
the way, Nigel hopes to (1 0) _____on to the people he meets the environmental
message that lies behind the whole idea.
1.
2.
3.
4.
5.
B. getting up
B. raise
B. wider
B. set
B. lead
A. settling down
A achieve
A. stronger
A. put
A. result
D. holding back
D. win
D. deeper
D. place
D. create
218
6. A. pulled B. charged c. forced D. powered
7. A. relying B. using c. attempting D. trying
8. A. quick B. short c. brief D. swift
9. A. anyway B. alike c. instead D. otherwise
10. A. leave • B. keep c. pass D. give
B. WRITTEN TEST (70pts)
I. OPEN CLOZE TEST (20 points)
Read the texts below and think of the word which best fits each space. Use
only one word in each space.
a. DANGERS OF TECHNOLOGY
Much has been heard recently (0) about possible health hazards, including
memory loss and brain tumors, from the use of mobile phones. With the possible
half a billion mobile phones in (1 )____ throughout the world, in Britain (2 )___ ,
one person in four owns one. (3 )_____ is worrying enough, even if, so far, no
concrete evidence has come to ( 4)_____ One study by Dr. Alan Preece and his
team at Bristol University has shown, however, in a report in the International
Journal of Radiation Biology, that tests on volunteers demonstrated no effect on
(5 )_____short-term memory or attention span. Subjects (6)______exposed to
microwave radiation for (7 )_____ to thirty minutes, but the one noticeable effect
was positive (8 )_____ than negative; the subjects reacted more rapidly in one
test (9 )_____ a visual choice. One explanation of (Ì0 )___ _ is that following
the transmission, a wanning of the blood led to increased blood flow.
b. LAUGHING IS GOOD FOR YOU - SERIOUSLY
It is a sad fact (0) that adults laugh far less than children, sometimes by as
much ( 1)_____ a couple of hundred times a day. Just take a (2 )_____ at peoples
faces on the way to work or in the office: you'll be lucky ( 3)_____ see a smile,
let alone hear a laugh. This is a shame — especially in view of the (4 )_____ that
scientists have proved that laughing is good for you. “When you laugh” says
psychologist David Cohen, “it produces the feel-good hormones, endorphins. It
counters the effects of stress (5 )_____ enhances the immune system.”
There are many (6 )_____ why we might laugh less in adult life: perhaps we
are too work-obsessed, or too embarrassed to (7 )_____our emotions show.
Some psychologists simply believe that children have more native responses,
and as adults we naturally grow (8) ______ of spontaneous reactions. Luckily,
(9 )_____ , it is possible to relearn the art of laughter. In India, “laughter clinics "
have been growing (1 0)_____ popularity over the last few years, thanks to the
efforts of Dr Madan Kataria, whose work has won him a devoted following.
Dr Kataria believes that his laughing techniques can help to strengthen the
immune system and lower stress levels, among other things. He teaches his
patients different laughs or giggles to relax specific parts of the body. In 1998,
when Dr Kartaria organized a World Laughter Day at Bombay racetrack, 10,000
people turned up.
2Ĩ9
II. WORD FORMS (20pts)
PAR T 1: Complete the sentence with the correct form of the given word. (10 pts)
1. _____ in wealth causes a great number of social problems. ( EQUAL)
2. The church is the site o f a number o f_____ manifestations. (NATURE)
3. The slight_____ in his left hand was corrected by surgery. (FORM)
4. Barack Obama is the first president of the United States with _____
background. (RACE)
5. They won the case because of the _____ in court of the defendant. (APPEAR)
6. The sun and the moon are o fte n_____ in poetry. (PERSON)
7. Tax exemption only applies to those with _____ status. (RESIDENT)
8. The station quickly fell in to _____ after it was closed. It is now in very bad
condition. (REPAIR)
9. Low income and little administrative support make tea chers _____ with their
profession. (HEART)
10. Do you think these children ar e_____ ? They look very thin. (NOURISH)
PART 2: Write the correct FORM of each bracketed word in the numbered
space provided. (10 pts)
THE CRUSADES
The medieval crusades, when Western European knights and (11. adventure)
_____ attempted to seize Jerusalem from the hands of the Seljuk Turks, are
widely (12. understand) _____ by most people in the West, who think of
them as glamorous and heroic. True, the level of heroism was occasionally
(13. i mp ress)___ , but in fact the crusaders were ignorant and (14. gu ide )_____ .
For example, they viewed the Byzantine Emperor, through whose lands they had
to travel, as an annoying (15. relevant) _ ___ , denying him even so much as a
(16. c onsult)_____ role in the proceedings. In reality, his long experience of the
Saracens had given him a(n) (17. conside r)_____ knowledge of their fighting
methods and politics. His advice, had the crusaders chosen to follow it, would
have been absolutely (18. va lu e)_______ . Instead, they (19. repe at)________
dismissed it as (20. wor th )_____ with the result that, despite initial success, the
crusades eventually dwindled to ignominious failure.
n i. ERROR IDENTIFICATION
Identify 10 errors'in the following passage and correct them
Normal houses are full of
hazadous waste. The most
important
hazardous waste in the home
is batteries. If you throw
them out with
your other garbage, they are
open at the landfill, the
poison inside
Line
moves through rain water
1
and other liquids to the
/
bottom of the
5
landfill. Then they can
pollute the natural' water in
the ground. We
should use rechargeable
batteries, other hadarhous
water in homes is
motor oil. Don’t throw old
motor oil in the ground and
throw it on the
220
garbage. It poisons the 10
environment. We should
recycle motor oil.
Painting is another kind of
hazarhous waste in homes.
Some cities have
"Paint exchange day". If you
bring in open, unused blue
paint and want
red, they give you red,
sometimes they mix the
paints together into
strange colours. If you paint
walls with them, you help
save the
environment

2. ________ 3.
147 5. _______ _ 6 ._______________ .
8. __________ 9 ._______________
IV. SENTENCE TRANSFORMATION (20 pts)
* Finish each of the following sentence in such a way that it is as similar as
possible in meaning to the sentence printed before it
1. Because of his conviction for fraud, the trainer lost his licence
His conviction for fra ud.................................................................. ...............
2. My liitle brother can be so annoying sometimes
—» I a m ................................................ ..................................................................
3. They declared war on the pretext of defending their territorial rights
—> The excuse................ ......................................................................... ............
4. Whenever you are on a bus, you hear someone talking about snobbery.
—> You can ’t ............................................................................... ............ ............
5. Pop stars are corrupted by the adulation of their fans
—» It’s the way ............................................... ......................................................
* For each of the following sentences, write a new sentence as similar as
possible in meaning to the original sentence, using the word given. This
word must not be altered in any way.
6. I don ’t think the television 's likely to blow up at any minute. LIKEHOOD
7. I’m afraid our problems are just beginning. ICEBERG
8. In the area, Thailand is much better than all other countries in football.
SHOULDERS
9. Thomas was not given details of the compan y’s new project. DARK
10 .1 was too scare to tell him what I really thought. LACKED
1
221
TRƯỜNG TH P Ĩ CHU YÊN HÙNG VƯƠNG - GIA LAI
A. MULTIPLE CHOICE QUESTIONS (40 pts)
I. WORD CH OICE (5 pts)
Choose the word or phrase from A, B, c or D that fits each blank.
1, I'm afraid Jenn ifer’s ve ry ______ She believes everything s he’s told.
A. gullible B. trustworthy c. credible D. honest
2. In the hands o f a reckless driver, a car becomes a _____ weapon.
A. lethal B. fatal c. mortal D. venal
3 .1 slept badly last night and am feeling p articularly ______ this morning.
A. slow-witted B. far-reaching c. off-hand D. top-heavy
4. Radio and television make it possible for the news to be widely______ .
A. disseminated B. dislocated c. dismantled D. expressed
5. You can ’t bury your h ea d______ and hope that this problem-goes away, you
know.
A. in the mud B. in the pool !C. in the sand D. in the water
6. Unanswered, the demands for nuclear deterrents h av e_____ fears o f civil war.
A. flashed up B. prognosticated c. sidetracked D. stoked up
7. This evidence should pro ve______ that he was telling the truth
A. once and for all B. from time to time c. over and above D. now and then
8. A new com puter has been produced, which w ill______ all previous models
A. overdo B. supersede C. excel D. overwhelm
9. My cousin was nervous about being interviewed on television, but she rose to
th e_____ wonderfully.
A. event B. performance C. incident D. occasion
10. My decision to drop out of university after a year is one I n ow _____ regret.
A. painfully B. harshly C. heavily D. keenly
H. STR UCTURES AND GRAMMAR (5pts)
Choose the answer which best complete each sentence
1 .1 supposed, a s______ we all, that the meeting would be cancelled.
A. did B. would c. only D.jus t
2. Although the Moon ap pe ars______ , it reflects on average only 7 percent of
the light that falls on it.
A. bright to the eye B. brightly to the eye
C. bright in the eye D. brightly in the eye
3. “What would you do in my place?” - “W ere ______ treated like that, I’d
complain to the manager.”
A. I to be B. I to had been c. I have been D. to 1 be
4. ______ the catfish is prized for its taste.
A. With ugly look B. As ugly looking
c. Ugly looking as it is
D. As it is ugly looking
222

I
5. His advice that it
A. should do
6. At no time
__ at once is reasonable.
B. be done c. needs to be done D. is done
A. did I ever promise
c. I have ever promised
Dick p u t______ ball in
you pay a rise.
net in
B. do ever I promise
D. had ever I promised
second half but goal
8.
was disallowed
A. a/a/a/a ' B. the/the/the/the c. the/the/a/a D. a/a/the/the
______ about what he had heard, John telephoned hia brother
A. Having worried B.To worry c. He worried D. Worried
______ given to the various types of microscopic plants and animals found in
water
A. Named plankton
c. Plankton’s name
10. You must asleep in the train.
A. have been tired if you fallen
c. have been tired if you fell
B. the name of plankton
D. Plankton is the name
B. be tired if you fallen
D. have been tired if you fall
III. PREPOSTIONS AND PHRASAL VERBS (5 pts)
Choose the most suitable options to complete each sentence
1. Both the favourite and then the second favourite pulled out. Naturally, we
thought we wer e______ a chance.
A in with B up for c in for D up with
2. His company went bankrupt, three years of hard wo rk______ the drain.
A. into B. in c. down D. low
3. Don’t quote me. What 1 am about to say is ______ the record.
A. on B. off c. without D. above
4 .1 was so tired that I ju st______ in the armchair
A. flaked out B. broke up c. dropped out D. fell over
5. The subject of sex equality seem to ____in every discussion lesson in my school
A. burst out B. zero in c. crop up D. harp on
6. Our teacher tendsto __certain subjects which she finds difficult to talk about
A. boil down B. string along c.skate over D. track down
7. It’s a good idea t o ______ people before taking them into your confidence
A. tumble to B. root out c.bank on D. size up
8. Some people can just _____ a cold,but my colds seem to linger for weeks
A. shrug off B. cough up c.pulfthrou gh D. stamp out
9. The man in the market was selling leather coats very cheaply: they were such
bargainsthat were soo n______ .
A. cleared off B. done for c. bought out D. snapped up
10. I couldn’t rem ember where I had left my car , when it s uddenly ______ me
that 1 didn’t have a car any longer!
A. dawned on B. ran into c. went through D. tumbled to
223
IV. COLLOCATIONS AND IDIOMS ( 5pts)
Choos e the a nswe r w hich best com plete each sen ten ce
1. It's great here. Have had ___________ ever since we arrived. We’re having a
wonderful time.
A. broken sunshine B. unbroken sunshine
c. unbroken light D. unbroken clouds
2. Palair Airlines offers exce llent______ entertainment, with the latest movies
and music.
A. on-flight B. during-flight c. in-flight D. over-flight
3. The cottage is in some wonde rful______ countryside on the edge o f a dense
forest.
A. far B. unspoiled c. great D. spoilt
4. As students, David, Kevin and William were as thick as______ .
A. thieves B. boy scouts c. a team D. thistles
5. Whenever I feel embarrassed I always go as red a s______ .
A. a rose B. lipstick, c. a raspberry D. a beetroot
6. He might look kind and sympathetic, but deep down he’s as hard a s______ .
A. nails B. a mountain c. a gangster D. an iceberg
7. Nagging Susan to stop smoking has no effect on her. It’s like water o ff ____.
A. a windmill B. a duck’s back c. a dripping tap D. an umbrella
8. My father _ _ _ _ _ when he found out that I’d damaged the car.
A. hit the roof B. saw pink elephants
c. made my blood boil D. brought the house down.
9. My m oth er______ for an anvironmental group. She helps raise money to
protect wildlife
A. indentifies B. encourages c. shares D. volunteers
10. At the 1952 Olympics, speed skaters who had taken amphetamines became
____ _ il l.
A grimly B. gravely c. solemnly D. rigorously
V. READING COMPREHENSIONS. (10ms)
PASSAGE 1: Read the follow ing passage and choose the best option to complete
the blank or answer the que stion. (5 pts)
Life originated in the early seas less than a billion years after Earth was
formed. Yet another three billion years were to pass before the first plants and
animals appeared on the continents. Life's transition from the sea to the land was
perhaps as much of an evolutionary challenge as was the genesis of life.
What forms of life were able to make such a drastic change in lifestyle ?
The traditional view of the first terrestrial organisms is based on megafossils —
relatively large specimens of essentially whole plants and animals. Vascular
plants, related to modern seed plants and ferns, left the first comprehensive
224
r> -
megafossil record. Bec ause of this , it has been com mo nly assumed that the
seq uen ce of terrestria lization reflected the evo lution of mo dem terrestrial
eco sys tem s. In this view, primit ive vascu lar p lants first colonized the margin s o f
continen tal waters, follow ed by ani mals tha t fed on the plants, and lastly by
ani mals tha t preyed on the plant-eaters. Moreover, the me gaf oss ils sug gest that
terrestria l life app eared and diversified exp losively nea r the boundar y betw een
the Silurian and the Devon ian periods, a little mor e than 400 mil lion years ago.
Recen tly, however, paleon tologi sts have been taking a clo ser look at the
sed iment s below this Silurian-Devon ian geological boundary. It turns out that
som e fossils can be ex tr ac te d from these sediments by putting the rocks in an
acid bat h.T he techniqu e has uncove red new eviden ce from sed iment s tha t were
dep osi ted near the shores of the anc ient oceans — pla nt mic rofoss ils and
micro sco pic pieces of small anim als. In man y in stan ce s, the spe cim ens are less
than one -tenth of a millim ete r in diam eter. Although th ey were en to m be d in the
rocks for hundre ds of millions of years, man y o f the fossils con sist o f the o rganic
rem ains of the o rganism .
These new ly disc ove red fossils have not only revealed the existence of
pre viously unk now n organisms, but have also pushed back these dates for the
inva sion of land by mu ltic ellula r organis ms. Our views abo ut the nature of the
early pla nt and animal com munities are now being revised. And with thos e
revisio ns com e new sp eculations a bou t the first terrestrial life- form s.
1. The w ord “d ra st ic ” in the second paragraph is clo sest in m ean ing to _______.
A. wides pread B. ra dical c. progressiv e D. r isky
2. Accor din g to the theory th at the author calls “the tr ad itio nal v ie w ”, wha t w as
the firs t form of life to app ear on land?
A. Bacteria B. M eat-eating an ima ls
c. Plant-eating anim als D. V ascular plants
3. According to the p assage, what happ ened a bou t 400 million years ago?
A. Ma ny terrestria l life-form s died out
B. N ew life-form s on land develop ed at a rapid rate,
c. The megafossils were destroyed by floods.
D. Life bega n to develop in the ancient seas.
4. The w ord “e xt ra ct ed ” in line 18 is closest in meaning to _____ .
A. located B. preserved c. remo ved D. studied
5. What can be inferred from the passage about the fossils mentioned in lines 17-20 ?
A. They have not been helpful in understanding the evolution o f terrestrial life.
B. They were found in approximately the same numbers as v ascular plant fossils,
c. They are old er than the meg afossils.
D. They con sist o f modem life-forms.
6. The w ord “in st an ce s” in line 21 is closest in meaning to ______ .
A. methods B. p rocesses c. cases D. reasons
225
7. The word “they” in line 22 refers t o ______ .
A. rocks B. shores c. oceans D. specimens
8. The word “entombed” in lime 22 is closest in meaning t o ______ .
A. crushed B. trapped c. produced D. excavated
9. Which of the following resulted from the discovery of microscopic fossils?
A. The time estimate for the first appearance of terrestrial life-forms was revised.
B. Old techniques for analyzing fossils were found to have new uses,
c. The origins o f primitive sea life were explained.
D. Assumptions about the locations o f ancient seas were changed.
10. With which of the following conclusions would the author probably agree?
A. The evolution of terrestrial life was as complicated as the origin of life itself.
B. The discovery of microfossils supports the traditional view of how terrestrial
life evolved.
c. New species have appeared at the same rate over the course of the last 400
million years.
D. The technology used by paleontologists is too prim itive to make
accurate determinations about ages of fossils.
PASSAGE 2: Read the following passage and choose the best option to
complete the blank or answer the question. (5 pts)
Animation traditionally is done by hand-drawing or painting successive
frames o f an object, each slightly different than the preceding frame. In computer
animation, although the computer may be the one to draw the different frames, in
most cases the artist will draw the beginning and ending frames and the
computer will produce the drawings between_the first and the last drawing. This
is generally referred to as computer-assisted animation, because the computer is
more of a helper than an originator.
In full computer animation, complex mathematical formulas are used to
produce the final sequence of pictures. These formulas operate on extensive
databases of numbers that define the objects in the pictures as they exist in
mathematical space. The database consists of endpoints, and color and intensity
information. Highly trained professionals are needed to produce such effects
because animation that obtains high degrees of realism involves computer
techniques for three-dimensional transformation, shading, and curvatures.
High-tech computer animation for film involves very expensive computer
systems along with special color terminals or frame buffers. The fra me buffer is
nothing more than a giant image memory for viewing a single frame. It
temporarily holds the image for display on the screen.
A camera can be used to film directly from the computer's display screen, but
for the highest quality images possible, expensive film recorders are used. The
computer computes the positions and colors for the figures in the picture, and
sends this information to the recorder, which captures it on film. Sometimes,
however, the images are stored on a large magnetic disk before being sent to the
226
reco rder. Once th is process is complete d, it is repe ated for the next, frame. When
the entire seq uen ce has been recorded on the film, the film must be develop ed
before the anim atio n can be view ed. If th e entire sequen ce does not seem right,
the motions must be correct ed, recomputed , redispl ayed, and rerecorded. This
approa ch can be very ex pensive and time co nsuming. Ofte n, com put er-animation
com panies first do motion tests with simple com put er-g enerate d line drawin gs
before selling their com puters to the task of calcul atin g the high-resolution,
realistic-loo kin g images.
1. Wh at asp ect of com put er anim ation does the passage mainly disc uss?
A. The production proc ess B. Th e equipm ent ne eded
c. The high cost D. Th e role of the artist
2. Ac cording to the passage , in com puter-assisted anim ation the role of the
com puter is to draw the____ .
A. first fr ame B. middle fram es
c. last fram e D. entire sequence of frames
3. The word ’’they ” in the second paragraph refers to____ .
A. form ulas B. da taba ses c. num bers D. o bjects
4. According to the passage, the frame buffe rs men tioned in the third paragrap h
are used to____ .
A. add colo r to the images B. ex pose several fram es at the same time
c. store indiv idual images D. create new fram es
5. A ccordin g to the passage, the positions and colours of the figu res in h igh-tech
anim ation are dete rmined by____ .
A. drawing several versions
B. enlar ging one frame at a lime
c. analyzi ng the sequence from different angles
D. using com puter calculations
6. The word "ca ptu res” in the fourth paragraph is close st in m ean ing to
A. separates B. registers c. describes D. numbers
7. The word "O nce ” in the fourth paragraph is closest in mea ning to____ .
A. before B. since C: after D. while
8. According to the.passage, how do computer-animation companies often test motion?
A. T hey ex per iment with comp uter-gen erated line draw ings.
B. T hey hand -draw successive frames.
c. The y c alcu late h igh-reso lutio n images.
D. T hey develop extensive mathematical formulas.
9. The word ’’tas k” in the 4th paragraph is clo sest in meaning to____.
A. possibility B. position ~ c. time D .job
10. Which of the following statements is sup ported by the passage?
A. Computers have reduced the costs o f anim ation .
B. In the future, traditional artists will no longer be n eeded.
c.Artists are unable to produce drawings as high in quality as computer drawings.
D. A nimation involves a wide range of technical and artistic skills.
227
IV. CLOZE TEST (10 PTS)
PASSAGE 1: Read the following passage and mark the letter A, B, c or D on
your answer sheet to indicate the correct word for each o f the blanks. (5pts)
Why did you decide to read this, and will you keep reading to the end? Do
you expect to understand every single part of it and will you remember anything
about it in a fortnight's
(1) _________ ? Common sense (2) _________ that the answers to these
questions depend on “readability”- whether the (3 )_______ matter is interesting,
the argum ent clear and the (4)___________ attractive. But psychologists are
trying to (5)__________ why people read - and often don ’t read certain things,
for example technical information. They also have examined so much the writing
as the readers.
. Even the most technically confident people often (6)__________ instructions
for video or home computer in favor of hands - on experience. And people
frequently take little notice of consumer information, whether on nutritional
labels or in the small print of contracts. Psychologists researching reading
(7)_______ to assume that both beginners and competent readers read everything
put in front of them from start to finish. There are (8)__________ among them
about the roles of eyes, memory and brain during the process. Some people
believe that fluent readers take in very letter or word they see; others (9)_______
that readers rely on memory or context to carry them from one phrase to another.
But they have always assumed that the reading process is the same: reading
starts, comprehension (10) then reading stops.
1. A. term B. gap c. period D. time
2. A. informs B. transmits c. suggests D. advises
3. A. text B. content c. subject D. topic
4. A. layout B. assembly c. pattern D. formation
5. A. value B. determine c. rate D.ensure
6. A. pass B. miss c. ignore D. omit
7. A. tend B. lead c. undertake D.cons ent
8. A. contests B. arguments c. objections D. separation
9. A. direct B. insist c. press D. urge
10. A. issues B. sets c. establishes D. occurs
PASSAGE 2: Read the following passage and mark the latter A, B, c or D on
your answer sheet to indicate the word or phrase that best fits each of the
numbered blanks. (5 pts)
Why is it that many teenagers have the energy to play computer games until
late at night but can’t find the energy to get out o f bed ( 1)________ for school?
According to a new report, today’s generation of children are in danger of
getting so (2)_______ sleep that they are putting their mental and physical health
at (3)______■ Adults can easily survive on seven to eight hours’ sleep a night,
(4)_______ teenagers require nine or ten hours. According to medical experts,
228
one in five youngsters (5)________ anything between two and five hours’ sleep
a night less than their parents did at their age.
This (6 )_________ serious questions about whether lack o f sleep is affecting
children's ability to concentrate at school. The connection between sleep deprivation
and lapses in memory, impaired reaction time and poor concentration is well
(7 )_______ . Research has shown that losing as little as hal f an hour’s sleep a
night can have profound effects (8 )______ how children perform the next day. A
good night's sleep is also crucial for teenagers because it is while they are asleep
(9)______ they release a hormone that is essential for their ‘growth spurt’ (the
period during teenage years when the body grows at a rapid rate). It’s true that
they can, to some (1 0)______ , catch up on sleep at weekends, but that won’t
help them when they are dropping off to sleep in class on a Friday afternoon.
By Tim Falla and Paul A.Davies, Solutions Advanced. OUP
1. A. behind time B. about time c. in time D. at time
2. A. few B. less c, much D. little
3. A. jeopardy B. threat c. risk D. danger
4. A. or B. because c. whereas D. so
5. A. puts B. gets c. brings D. makes
6. A. raises B. rises c. results D. comes
7. A. organized B. arranged c. established D. acquired
8. A. in B. on c. to D. at
9. A. at which B. which c. where D. that
10. A. rate B. extent c. level D. point
B. WRITTEN TEST
I. OPEN CLOZE TEST: (20 pts)
CLOSE TEST 1: Fill each blank with ONE word. (10 pts)
CELL PHONES
Cell phones have been popular in Japan since the early 1990s, but it was not
(1 )____ 1999 that their use really took off. The age of cell phones has emerged,
but with it come problems. Cell phones are used on buses and trains, in restaurants,
and in all (2 )______ of life. They cause problems when they ( 3)______ during
meetings, concerts, wedding, or even funerals. What’s more, people speak loudly
in public, and students read and text messages during lessons. More seriously,
when a cell phone is used near a person using a (4) ______ to regulate his
heartbeat, its radio waves may (5 )______ with the functioning o f the pacemaker.
Now, something is being done to (6 )______ these problems. In many places,
new technology is being used to block cell phone calls. Airline (7 )______ are
requested to stop using cell phones while on board. Concert halls ask their
audience to switch their phones to the (8 )______ mode. However, phone users
fear that if they do not answer their phones, they will (9) _ _____ valuable
business opportunities. That’s why many do not (1 0)______ off their phones
even when they are asked to.
229
CLOSE TEST 2: Fill each blank with ONE word. (10 pts)
THE IMPORTANCE OF BEING SPORTY
It is a multi-million dollar industry and that attracts some of the most talented
individuals alive, it generates further millions (1 )______ advertising revenue
and has spawned a whole celebrity culture of its (2)______ . But what role does
sport play in our ordinary lives?
Few people are untouched (3 )______ sport. We all have (4 )______favorite
football team or tennis player or, at least, support our national side in major sporting
events like the Olympics. How can it be, then, that so (5 )______ people actually
play sports and that obesity is becoming a major threat in the developed world?
Well, in part, the answer is (6 )______ the question. Obesity is not a problem
in the developing world (7 )______ participation in sport is still high. True, there
may be few organized leagues but children the world still go out to play games
like football with other children and this comes the supremacy of countries like
Brazil in the world of football. So, what is stopping the British or the
Americans? After (8) ______ , they still have organized sports in schools and a
myriad of clubs and teams to join in their spare time. What they lack is a public
space for children to play unsupervised outdoors. As a result, they are kept
indoors and encouraged to play online ( 9)______ of getting our in the fresh air.
They may o f course still develop a love of sport as spectators but this is unlikely
to do much to lower levels o f obesity (1 0)______ the young.
II. WORD FORMS (20 pts)
PART 1: Complete the sentence with the correct form of the given word. (10 pts)
1. Janet’s exam results turned out to b e__________ DISASTER
2. You look rath er_______. Are you worried about something? OCCUPY
3. There are very few ._______ places on earth today. Man has been nearly
everywhere. EXPLORE
4. The doctor gave him an injection to _______ the pain. DIE
5. Nowadays people may no longer suffer from lifetime physical .
NORMAL
6. Home-made ice-cream is one of ou r______ . SPECIAL
7. The meeting has b een_______ arranged for 3 p.m next Friday. PROVIDE
8. The government’s _____approach has brought criticism. COMPROMISE
9. Jim is one o f the m ost_______ members of the committee. SPEAK
10. Many newspapers today gave very sensation al______ about the private life
of that Hollywood star. CLOSE
PART 2: Complete the passage with appropriate forms from the words
given
in the box. (10 pts)
230

angry fashion foreign frustrat e


patient polite secret serve

“All things come to those who wait.” It’s a (1 )______ phrase that has kept
the British patiently in line for years. For (2 )______we’ve been waiting our
turn. This may sometimes have led to (3) _____ abroad, when (4) ______
hav en't always respected the rules of the orderly queue. But at home we have
(5) ______ enjoyed our reputation. It's rather (6) ______ , though, isn’t it?
Today’s highly developed culture of (7 )_____ to the public seems to have more
in common with the American “I want it and I want it now” philosophy. But our
(8 )_ ____ seems to be growing. We may wait with less (9) _ _____ than we
used to - you only have to observe the (1 0) ____ _ shoppers in a supermarket
queue. But wait we still do.
III. ERROR IDENTIFICATION (10 pts)
Identify 10 errors in the following passage and correct them.
It is very difficult to succeed in the music business; nine out o f ten bands that
release a first record fail to produce the second. Surviving in the music industry
requires luck and patience, but most of all it requires and intricate knowledge of
how a record company is functioned. The process begins when a representative
of a company's Artists and Reportoire (A & R) department visits bars and night
clubs, scouting for young, talented bands. After the representative identifies a
promised band, he or she will work to negotiate a contract with that band. The
signature of this recording contract is a slow process. A company will spend a
long time to investigate the band itself as well as current trands in popular music.
During this period, it is important that a band reciprocates with an investigation
of its own, learning as much as possible about the record company and making
personnel connections within the different departments that will handle their
recordings.
l . _ _____________
4. __________
7.________________
2 ._______________
5 ._ _______ _____
8 ._______________
3._____________ __
6. ____________
9. ____________
10.
V. SENTENCE TRANSFORMATION (20pts)
PART 1: Finish each o f the second sentences in such aw ay that they have the
same meaning as the original ones:
1. 1 left without saying goodbye as 1 didn’t want to disturb the meeting.
-> Ra ther_____ _______________ . _____________________________
2. For further information, please send a self-addressed envelope to the above
address.
—> Further information c an ______________________________________
3. It shou ldn’t have surprised me that my children didn’t like the new, cheaper
ice-cream..
—> I mi ght_________________ _______ _________
231
4. The noise made by the children didn’t prevent the baby from sleeping soundly.
—> W hat ev er _____ - ________________________________________
5. Don’t run away with the idea that this job is easy, (co nclusio n)
6. Ru ss's opinions on the new management policies were very different from
those of his fellow workers, (od ds) ,
—» R us s_____ __________________________ the new m anagement policies.
7. The young actress was very nervous before the audition, (butter flies)
8. Margaret is said to be a very good cook, (re putation )
9. She chooses the kind o f hotels she stays in very carefully.
She’s very fussy____________ __________________________________
10. The rocketing prices have proved too much for most salaried people, (cope)
TR ƯỜ NG THPT CH UY ÊN LÝ Tự TR ỌN G - CẦ N THƠ
A. MULTIPLE CHOICE
I. PH ON OL OG Y (lOp ts)
Choose the wo rd w hose underlined part is pronounced differently from the others.
1. A. picturesqu ely
2. A. phlebitis
3. A. macabre
4. A. paranoia
5. A. absinthe
B. antiqu eness
B. wily
B. chirom ancy
B. papist
B.sooth e
c. querulously
c. poliomyelitis
c. chemical
c. saffron
c. loathe
D. discotheque
D. philately
D. chivalrous
D. travail
D. wre athe
Ch oose the w ord tha t has the stress pattern differ ent from that o f th e othe rs:
6. A. euphemism
7. A. matriarchy
8. A. disinter
9. A. acupuncturist
10. A. entrepreneur
B. phenotype
B. epileptic
B. cadaver
B. vociferously
B. anonymity
c. veracity D. cantaloupe
c. rutherfordium D. diplomatic
c. cathedral D. attorney
c. antithesis D. extraordinary
c. representation D. encyclopedia
II. VO CA BU LA RY (lOpts):
Ch oose the correct w ord or phras e A , B , c or D to complete ea ch sen ten ce.
1. His strange behaviour aroused th e_____ of police.
A. doubt . B. suspicion c. disbelief D. notice
2. When his parents are away, his oldest brother_____ .
A. knocks it off B. calls the shots
c. draws the line
D. is in the same boat
232

3. After the concert, everyone had to _____home through the thick snow.
A. trudge B. tread c. trace D. trickle
4. Alaska boasts several climates due to it s__________ mountains, warm ocean
currents, and frozen seas.
A. adept B. pious c. ghastly D. lofty
5. When I finish writing this composition, I’m going to _____and go to bed.
A. make time B. hit the hay c. hit the big time D. call it a day
6. Certain kinds o f plants can grow only in _____soil.
A. loose B. slack c. sparse D. scarce
7. John’s got very___feelings about taking on more responsibility at the moment.
A. puzzled B. mixed c. jumbled D. muddled
8. Paul’s been in Alic e's bad ' ever since he offended her at the party.
A. eyes B. books c. likes D. treats
9. The car w as _____speed.
A. gathering B. collecting c. consuming D. firing
10. They’re having serious problems. Their relationship is on th e_____ .
A. cliffs B. rocks c. stones D. grass
III. STRUCTURES AND GRAMMAR (lOpts)
1. _____earning all that money if you don’t have time to enjoy it?
A. Is it to the good B. What’s the good of
c. Is it all to the good . D. Is it for good
2. S he_____ fainted when she heard that her child died.
Á. rather than B. nothing but c. all but D. near
3. _____to interfere in your affairs but I would like to give you just one piece of
4.
advice.
A. It is far from clear
c. Far and wide for me
_____we just have to wait.
A. At this point in time
c. At some point
5. He gripped his brother’s arm lest he _
A..would be trampled
c. were trampled
6 .1 don’t know _____ the new manager.
B. Far from it for me
D. Far be it from me
B. On the point of time
D. That’s not the point
_ by the mob.
B. could be trampled
D. be trampled
A. what to do o f
c. how to do of
7. Don’t tell me I _
B. what to make o f
D. how to make of
_ till 5 o’clock in the afternoon.
A. was to make them to stay B. must have made them stay
c. could have made them stay D. was to have made them stay
8. What I fear most o f all is that w e_____ in what refers to the building o f a new
market in West Crane. The local people seem to be very wary about it.
A. might misinterpret B. might have misinterpreted
c. won ’t be misinterpreting D. might be misinterpreted
233
9. Sam looked as if he ____ in the world.
A. wer en't cared
c. did n't have a care
10. W hat's all this c ryi ng_____ ?
A. without the aid o f
c. with the aid of
B. weren’t taken into care
D. w eren't in the care
B. in aid o f
D. within the aid of
IV. PREPOSITIONS AND PHRASAL VERBS (lOpts)
1. He is not a very outspoken person, so we may find it hard to ____his opinions.
A. leak out B. come o ff c. sound out D. bring round
2. Are you still _______ an illusion that Mr. Spike will agree to your conditions?
A. in B. under c. on D. of
3. We all tried to convince our teacher to change his mind about the school trip
and he finally______ _.
A. caught on B. set out c. got through D. came round
4. Our grandfather c an ________ his war experiences all the day. Sometimes, we
all get truly bored with it.
A. harp on B. vouch for c. figure out D, leaf through
5. Father says he’s to o ________ after working the nightshift and he won’t help
us prepare the barbecue.
A. run away B. worn out c. taken in D. pulled down
6. They said I'd b e _______—probation for the first two or three weeks as they
want to find out about my skill.
A. for B. on c. in D. at
7. Your re po rt ______ the side effects of the venture. Why not write more
about its good points, Mr. Hewitt?
A. backs down B. takes to c. dwells on D. makes out
8 .1 know P ete's conduct was intolerable, but don’t be too ha rd ________ him.
A. on B. with c. at D. to
9. It is said that the corruption can hardly be _ ______ especially among top
officials.
A. worn o ff B. rooted out c. gone back D. stringed along
10. Mrs. Black has promised tobe______ _ service on Monday. I hope she
keeps her word or else I won’t manage to prepare everything in time.
A. in B. on c. of D. within
V. READING COMPREHENSION
PASSAGE 1: Read the following passage and choose the best option to
complete the blank or answer the question. (10 pts)
Every day, millions of shoppers hit the stores in full force - both online and
on foot - searching frantically for the perfect gift. Last year, Americans spent
over $30 billion at retail stores in the month of December alone. Aside, from
234
purchasing holiday gifts, most people regularly buy presents for other occasions
throughout the year, including weddings, birthdays, anniversaries, graduations,
and baby showers. This frequent experience of gift-giving can engender
ambivalent feelings in gift-givers. Many relish the opportunity.to buy presents
because gift-giving offers a powerful means to build stronger bonds with one’s
closest peers. At the same time, many dread the thought of buying gifts; they
worry that their purchases will disappoint rather than delight the intended
recipients.
Anthropologists describe gift-giving as a positive social process, serving
various political, religious, and psychological functions. Economists, however,
offer a less favourable view. According to Waldfogel (1993), gift-giving
represents an objective waste of resources. People buy gifts that recipients would
not choose to buy on their own, or at least not spend as much money to purchase
(a phenomenon referred to as “ the deadweight loss of Christmas"). To wit,
givers are likely to spend $100 to purchase a gift that receivers would spend only
$80 to buy themselves. This “ deadweight loss” suggests that gift-givers are not
very good at predicting what gifts others will appreciate. That in itself is not
surprising to social psychologists. Research has found th at people often struggle
to take account of others ' perspectives - their insights are subject to egocentrism,
social projection, and multiple attribution errors. What is surprising is that giftgivers have
considerable experience acting as both gift-givers and gift-recipients,
but nevertheless tend to overspend each time they set out to purchase a
meaningful gift.
In the present research, we propose a unique psychological explanation for
this overspending problem - i.e., that gift-givers equate how much they spend
with how much recipients will appreciate the gift (the more expensive the gift,
the stronger a gift-recipient’s feelings of appreciation). Although a link between
gift price and feelings of appreciation might seem intuitive to gift-givers, such an
assumption may be unfounded. Indeed, we propose that gift-recipients will be
less inclined to base their feelings of appreciation on the magnitude of a gift than
givers assume.
Why do gift-givers assume that gift price is closely linked to gift-re cipients'
feelings of appreciation? Perhaps givers believe that bigger (that is, more
expensive) gifts convey stronger signals of thoughtfulness and consideration.
According to Camerer (1988) and others, gift-giving represents a symbolic ritual,
whereby gift-givers attempt to signal their positive attitudes toward the intended
recipient and their willingness to invest resources in a future r elationship. In this
sense, gift-giv ers may be motivated to spend more, money on a gift in order to
send a '‘stronger signal" to their intended recipient. As for gift- recipients, they
may not construe smaller and larger gifts as representing smaller and larger
signals o f thoughtfulness and consideration.
235
1. What is the main idea discussed in the passage?
A. Gift-giving, despite its uneconomical downsides, cultivates a positive
social process.
B. Gift- recipients are widely acknowledged as considerably experienced in
gift-giving.
c. Gifts can serve as implicit signals o f thoughtfulness and consideration.
D. Gift-giving may have certain drawbacks alongside its positive qualities.
2. The author most likely uses the examples of gift-giving occasions in paragraph 1
to highlight the_____ .
A. regularity with which people shop for gifts
B. recent increase in the amount of money spent on gifts
c. anxiety gift shopping causes for consumers
D. number o f special occasions involving gift-giving
3. In paragraph 1, the word “ambivalent” most nearly means_____ .
A. unrealistic B. apprehensive c. conflicted D. supportive
4. The author indicates that people value gift-giving because they feel it_____.
A. functions as a form of self-expression B. is an inexpensive way to show
appreciation
c. requires the gift-recipient to reciprocate D. can serve to strenghten a
relationship
5. The “social psychologists'’ mentioned in paragraph 2 would likely describe the
“deadweight loss” phenom enon as_____.
A. predictable B. unprecedented c. disturbing D. questionable
6. In paragraph 3, the authors indicate that the assumption made by gift-givers
may be •
A. insincere B. unreasonable c. incorrect D. substantiated
7. The word “convey” in paragraph 4 most nearly means_____ .
A. transport B. countera ct c. exchange D. communicate
8. In paragraph 4, the author refers to works by Camerer and others in order
to_____ .
A. offer an explanation X B. introduce an argument
c. question a motive D. support a conclusion
9. In paragraph 4, the word “construe” most probably means_____ .
A. view B. understand c. consider D. take
10. Which of the following best characterizes the tone of the author?
A. authoritative B. pessimistic c. ironic D. informative
PASSAGE 2; Read the following passage and choose the best option to
complete the blank or answer the question. (lOpts)
According to sociologists, there are several different ways in which a person
may become recognized as the leader of a social group in the United States. In
the family, traditional cultural patterns confer leadership on one or both of the
parents. In other cases, such as friendship groups, one or more persons may
236
gradually emerge as leaders, although there is no formal process of selection. In
larger groups, leaders are usually chosen formally through election or recruitment.
Although leaders are often thought to be people with unusual personal ability,
decades of research have failed to produce consistent evidence that there is any
category of "natural leaders.” It seems that there is no set of personal qualities
that all leaders have in common; rather, virtually any person may be recognized
as a leader if the person has qualities that meet the needs of that particular group.
Furthermore, although it is commonly supposed that social groups have a
single leader, research suggests that there are typically two different leadership
roles that are held by different individuals. Instrumental leadership is leadership
that emphasizes the completion of tasks by a social group. Group members look
to instrumental leaders to “get things” done. Expressive leadership, on the other
hand, is leadership that emphasizes the collective well-being of a social grou p’s
member. Expressive leader are less concerned with the overall goals of the group
than with providing emotional support to group members and attempting to
minimize tension and conflict among them. Group members expect expressive
leaders to maintain stable relationships within the group and provide support to
individual members.
Instrumental leaders are likely to have a rather secondary relationship to
other group members. They give orders and may discipline group members who
inhibit attainment of the group’s goals. Expressive leaders cultivate a more
personal or primary relationship to others in the group. They offer sympathy
when someone experiences difficulties or is subjected to discipline, are quick to
lighten a serious moment with humor, and try to resolve issues that threaten to
divide the group. As the differences in these two roles suggest, expressive
leaders generally receive more personal affection from group members;
instrumental leaders, if they are successful in promoting group goals, may enjoy
a more distant respect.
1. What does the passage mainly discuss?
A. The problems faced by leaders
B. How leadership differs in small and large groups
c. How social groups determine who will lead them
D. The role o f leaders in social group
2. The passage mentions all o f the following ways by which people can become
leaders EXC EPT _____.
A. recruitment B. formal election process
c. specific leadership training D. traditional cultural patterns
3. In mentioning “natural leaders” in line 7, the author is making the point
that'_____. , .
A. few people qualify as “natural leaders”
B. there is no proo f that “natural leaders” exist
c. “natural leaders' are easily accepted by the members of a social group
D. “natural leaders” share a similar set of characteristics
237
4. Which of the following statements about leadership can be inferred from
paragraph 2?
A. A person who is an effective leader of a particular group may not be an
effective leader in another group.
B. Few people succeed in sharing a leadership role with another person.
c, A person can best learn how to be an effective leader by studying research
on leadership.
D. Most people desire to be leaders but can produce little evidence of their
qualifications.
5. The passage indicates that instrumental leaders generally focus on_____ .
A. ensuring harmonious relationships
B. sharing responsibility with group members
c. identifying new leaders
D. achieving a goal
6. The word “co llec tive” is closest in meaning to_____.
A. necessary B. typical c. group D. particular
7. The word “them’’ refers to_____.
A. expressive leaders B. goals of the group
c. group members D. tension and conflict
8. A “secon dar y relatio nsh ip” between a leader and the members of a group
- could best be characterized a s_____.
A. distant B. enthusiastic c. unreliable D. personal
9. The word “re solve” is closest in meaning to_____.
A. avoid repeating B. talk about
c. avoid thinking about D. find a solution for
10. Paragraphs 3 and 4 organize the discussion of leadership primarily in terms
of____ .
A. examples that illustrate a problem B. cause and effect analysis
c. narration of events D. comparison and contrast
VI. GUIDED CLO ZE
PAS SAG E 1: Read the following passage and choose the options tha t best
com plete the blank. (lOpts)
THE VACUUM CLEANER
Until about 250 years ago, households did not take dirt as seriously as they do
now - it was a fact of life, and that was that. Cleaning often consisted of an
annual (1 )______ called 'spring cleaning' when the furniture was moved aside,
and all the linen products in the house were cleaned. Carpets and rugs were taken
outside, hung on ropes and had the dust (2)
.__ out of them - an exhausting
and messy process.
The industrial revolution brought about a major change - as new products
became available to make homes cleaner, a corresponding interest in 'domestic
238
hygiene' appeared in households. This in turn led to the (3) .______o f further
products, one o f which was the vacuum cleaner.
(4) _____ has it that when one o f the first vacuum cleaners was demonstrated,
a kindly scientist took the proud inventor (5 )______ , and offered a bit of advice
that was to become crucial to the future evolution of the product - 'make it suck,
not blow'.
The first vacuum cleaners appeared in the 1860s in the United States. They
were operated by hand pumps and were almost as ( 6)______ as spring cleaning.
It was only when electric motors had become sufficiently (7 )______ to become
portable that vacuum cleaners became common household items. Most o f today's
major (8 )______ - including Electrolux and Hoover - were born in the 1920s.
The household dirt that vacuum cleaners suck up is mostly dead skin cells -
humans (9) ______ millions of cells every day. A much smaller proportion
comes from dust and soil carried into the house from (10 )______ .
1. A. ritual B. result c. resolution D. scrub
2. A. cleaned B. taken c. beaten D. sucked
3. A. fabrication B. appearing c. recreation D. developm
4. A. Story B. Epics c. Legend D. Tale
5. A. away B. aside c .aback D. along
6. A. laborious B. hard c. nefarious D. straining
7. A. scientific B. forward c. technological D.advanced
8. A. brand B. marks c. makes D. trademark
9. A. lose B. outgrow c. omit ■. D.shed
10. A. external B. outside c.b ey on d D. indoors
PASSAGE 2: Read the following passage and choose the options that best
complete the blank. (10 pts)
CARNIVOROUS PLANTS
All plants rely on nutrients taken from the soil in order to survive. However,
in areas where the soil does not contain enough vital nutrients, some plants have
adapted to (1 )_____ their diets from another source: living organisms. Though
they are few in number, carnivorous plants are nonetheless fascinating beings
that eat anything from one-celled organisms to insects in order to survive. They
are commonly found in marshlands. Carnivorous plants feature one of several
types of traps to'e nsn are prey, which they consume to (2 )_____ nutrients that
may be missing from the soil. While there are over 400 species of carnivorous
plants in the world today, some are more (3) _____than others.
The most well-known of these plants are the snap traps, which include the
Venus flytrap. Snap traps are easily identified by their leaves, which are
separated into two lobes that have the ability to fold together. Inside the lobes,
the surface is covered with tiny hairs that are (4) _____ to movement. When the
plant's prey brushes against the hairs, it triggers a closing mechanism that
rapidly brings the two lobes together, trapping the prey (5 )__________ inside.
239
The response of the traps is phenomenal (6) _____ speed: the time between
triggering the hairs and snapping shut is less than a second. As the prey struggles
inside the trap, it only triggers more hairs, causing the leaves to tighten their
(7 )_____
The plant then secrets liquid chemicals from special glands into the trap to
dissolve the prey and absorb all of its nutrients. Besides the Venus flytrap, only
one other type of snap trap exists today, (8 )_____ to as the waterwheel plant.
The two share a common ancestor and differ only in a few ways. (9 )_____ , the
waterwheel is an aquatic plant, while the flytrap is exclusively terrestrial. In
on arthropods like
addition, the flytrap feeds primarily
spiders, while the
simple invertebrates,
waterwheel lives (10) like certain types o f
plankton.
1. A. feed B. enlarge c. supplement D. modify
2. A. break in on B. make up for c. get through with D. drop out of
3. A. dominant B. prevalent c. recessive D. popular
4. A. prone B. liable c. vulnerable D. sensitive
5. A. closely B. furiously c. steadily D. securely
6. A. in accordance
c. in preference to D. on merits of
with B. in regard to
7. A. grip B. snail c. fist D. seizure
8. A. implicated B. indicated c. referred D. denoted
9. A. For instance B. As a result c. In contrast D. Otherwise
10. A. onto B. with c. for D. off
B. WRITTEN TEST
I. OPEN CLOZE TEST
PASSAGE 1: Fill in each gap with ONE suitable word.(lOpts)
ALTERNATIVE FAST FOOD
It is generally agreed upon that fast food is a great solution to the problem of
what to eat. However, it’s (1 )______as less nutritional than a healthy meal,
which is what growing children need. For years, nutritionists have been trying to
get children into the (2 )______ of eating healthy food. Without a doubt, this has
not been easy, getting in (3 )______ that they have to compete with numerous
fast food restaurants which are very popular with young people.
Nutritionists have lately come up with an idea which could (4 )______ to be
successful. This involve (5 )______ children how to cook and in the process to
enjoy the taste of fresh food. In the long (6 )______ , this may keep them off fast
food. There are already a number of school projects throughout Britain that are
experimenting with the idea. They begin making vegetarian pizza - both the
dough and the topping. Many children were impressed with their own homemade pizzas.
(7 )____ _ they compared their pizzas with ready-made ones, the
differences were noticeable. They found the (8) ______ greasy and not as
delicious as the homemade ones. Throughout the lesson, the children learn about
the quality and become skillful in the different techniques involved in cooking.
240
They become interested in cooking and in no ( 9)______ they even learn how to
look after themselves by eating nutritious food.’
Finally, nutritionists would like to get children cooking at home, too. It may
be messy, but, it's also an (1 0)______ for parents to spend quality time with
their children.
PASSAGE 2: Fill in each gap with ONE suitable word. (lOpts)
Every spring and fall, you can probably look up into the sky and see large
flocks of birds either coming or going. This (1)2_____ of flying south for the
winter and north for the summer is called migration. Birds migrate for a number
of reasons. One reason is of course to escape the (2)______conditions o f winter.
Although they have feathers, most birds are poorly (3)______ to deal with
extremely cold temperatures. Their (4) are open to the wind and snow,
and do little to keep the birds warm. Furthermore, birds often find it extremely
difficult to gather enough food during the winter. By flying south (5)______
warmer climates during winter, birds avoid the cold temperatures and food
shortages that other animals have to (6)______ during the winter months.
Another reason is mating. Many birds will only lay their eggs in certain places
and at certain times o f the year. So, migration also occurs at mating times.
One of the amazing things about birds is the distances and the accuracy
(7) ______which they migrate. Many birds are able to find their way back, to the
exact same spots year after year despite having flown thousands of kilometres.
The bar goose, for example, migrates over 30,000 kilometres each year without
(8) ______ its way. In order to travel such long distances without getting lost,
bird s-use a variety of methods to help them (9)______. One of the most
important of these methods is the use of landmarks. From high in the air, birds
have an excellent view of the land below them. They can see mountains, rivers
and other (10)______land features and use them to find their way.
II. WORD FORMS
PART 1: Give the correct forms of the words in brackets.
1. There is little hope'that Maurice’s behavior will ever improve. It will probably
remain so ____________________ till he grows up. (CORRECT)
2. The ____________________ project has just been launched in that university.
(DEPARTMENT)
3. Matt says you must be good a t ____________________ responsibilities of a
restaurant owner. (TASK)
4. Everyone marveled at h is ____________________ idea of organizing more
field trips for students. (OBJECT)
5. He gets _____________________ whenever he hears that song. (EYE)
6. She looked absolutely _____________________ when I told her what had
happened. She could hardly say a word. (DUMB)
7. New immigrants have been successfully ____________________ into the
community. (SIMILAR)
241
8. This school was once ____________________ as a military hospital during
the war. (REQUIRE)
9. In Scotland, there is greater emphasis o n_____________ .______ by individual
schools. (VALÙE)
10. He stood at the door to make sure that no o n e____________________ the
party. (GATE)
PART 2: Complete the following passage with the correct forms o f the given words.
fro nt excep t secure la'oour
solid tend affirm alter
EXTRACT FROM A BOOK ABOUT MEETING
We are (1 )___________ • by the experts that we are, as a species, designed
for face-to-face communication. But does that really mean having every meeting in
person? Ask the bleary-eyed sales team this question as they struggle (2 )______
through their weekly teambuilding session and that answer is unlikely to be in
the ( 3)_______________ . Unless you work for a very small business or have an
(4 )___________ high boredom threshold, you doubtless spend more time sitting
in meetings than you want to. Of course, you could always follow business
Norm an’s example. He liked to express (5 )__________ with customers queuing
at the (6 )____________ by holding management meetings standing up. Is email
a realistic (7 )_____ __________? It’s clearly a powerful tool for disseminating
information, but as a meeting substitute, it's seriously flawed. Words alone can
cause trouble. We’re all full of (8)_______________ that can be unintentionally
triggered by others and people are capable of reading anything they like into an
email. There is also a ( 9)_______________ for email to be used by people who
wish to avoid ‘real’ encounters because they do n’t want to be (1 0) ___________
with any awkwardness.
III. ER RO R IDEN TIFICA TION
Ide ntify 10 e rro rs in the follow ing pas sage and corre ct them .
BEETHOVEN
Ludwig Van Beethoven, one Line
of the most popular classical 1 5
composer
of all time, is sometimes
known as 'the Shakespeare o
f music'. He was
bom in Bonne in 1770, and
publishing his first work at
the age of
thirteen. Mozart was his
teacher for a short time and
they did not get on
very we'll together;
Beethoven was a difficult
student.
Like a composer, Beethoven
was a genius - but as a
person, he was
not very easy to like. He was
a passionate man who
controlled his
temper very easily. He was
also arrogant. The upper
classes of Vienna
used to invite him to parties,
when he was often quite
rude. He was once
242
heard to say to a prince:
‘There will always be many
princes, but there
is uniquely one Beethoven.’
Nevertheless, Beethoven
played some of the most
beautiful
symphonies the world has
never heard. How was such
an arrogant, bad
tempered man inspired to
10
the write such romantic
15.
music? Perhaps the
answer lies in the three
letters that were found after
his death. They
were addressed to his
‘immortal Beloved’.
Nobody knows that this
woman was, but it appears
that Beethoven was deeply
in love with her
for most o f his adult life.

1. 2. . 3.
4. 5. 6.
7. 8. 9.
IV. S EN TE NC E TR AN
10,
SFOR MA TION
Rewrite the sentences with the given words or beginning in such a way that
their meanings remain unchanged.
1. My new job is much more satisfying than any job I’ve ever had.
—» My new job is far __________________________ ______ I’ve ever had.
2. If Tom hadn ’t acted promptly to extinguish the fire, there might have been
more damage to the house.
—> B u t___________________ ____________________ the fire out, there
might have been more damage to the house.
3. A child o f his age is too young to be deceitful.
—> S o________________________ _________________________ deceitful.
4. I’ve become extremely good at missing the rush hour over the last few weeks.
(FI NE )
—> I’ve got_______________________ __________over the last few weeks.
5. I feel that to brand her ideas unworkable at this stage would be wrong.
(W RITE )
—> I don’t think _________ ______________________ _______ at this stage.
6. It’s crucial for us to control the juvenile inmates in a very strict way. (KEE P)
—> It’s crucial for us _______________________________________ inmates.
7. You’ve done nothing but look miserable all day. (AR OU ND )
—> Y ou’ve d on e______________ ____________________________ all day.
8. We don’t expect that the tourists in that accident have survived. (HOL D)
—> We don ’t ________________ ______________________ in that accident.
9. Your story is different from the facts. (DO ES)
—> Your story _____________________ __________________ •
the facts.
10. His excuse for such bad behavior has little effect on her. (NO )
—> His excuse _____________________________________ ________her.
243
LÓ P 111
ĐỀ CHÍNH THỨC
A. multiple choice (40 PTS)
I. GRAMM AR AND STRUCTURES (5PTS):
Choose the best options to complete the following sentences.
1. - Jack: Did you know Jim’s car broke down on the highway late at night?
- Jane: Unfortunately, that's a situation anyone______ .
A. might have to confront with ' B. might be confronted with
c. might be confronted D. might have been confronted
2. ______, 1 haven’t seen Catherine all day.
A. Come to think of it B. To come to think of it
c. Coming to think it D. Coming to think of it
3. ______ earning a great deal of money as a painter, Ivan Aivazovsky opened
an art school and gallery in his hometown of Feodosiya.
A. Rather B. Since c. Upon D. Until
4. In your place, I - to others for help. There’s no way of getting the
project finished yourself.
A. would have turned B. must have turned
c. will turn D. would turn
5. ______chair the meeting.
A. John was decided to
B. It was decided that John should
c. There was decided that John should
D. John had been decided to
6. We all wished to be treated______.
A. as equals B. as equally c. as equals D. equals
7. The great French novelist Honoré de Balzac would make _____ revision
when he was proofreading his work th at ______was left o f the original text
in the end.
A. too many I not enough B. such a lot of / a few
c. plenty of / not many D. so much / little
8. She *______fainted when she heard that her son had died.
A. rather than B. nothing but c. all but D. near
9. I'll be kind to her.______ she decide to leave me.
A. in case B. whereas c. so as not D. lest
10. Jasmine’s parents died when she was just a baby, so I ______ family she
ever had.
A. was all the - B. was the whole
c. have the whole D. have all the
244
II. PHRASAL VERBS AND PREPOSITIONS (5 PTS)
11. H e’s a tough politician - he knows how to ______the storm.
A. run down B. keep up c. push back D. ride out
12. D on't tak e______ your bad friends any more or you'll regret it.
A. up with B. in for c. down at D. for
13. He was unaware that he was being______ with out-of-date stock.
A. faded away B. clamped down c. petered out D. fobbed off
14. I was so tired that I ju st ______ in the armchair.
A. flaked out B. broke up c. dropped out D. fell over
15.1 thought she was being serious, but she was only having m e______ »
A. up B. on c. over D. round
16. Peter: How old do you think Perkins is?
Patrick: Oh, he must be getting______ eighty, I’d say.
A. away with B. about at c. down to D. on for
17. Nobody from the President down should imagine they a re _____ the law.
A beyond B over c within D above
18. At first the children enjoyed the game but quite soon the novelty______ .
A. went off B. died out c. died down D. wore o ff
19. When attacked by his opponents, the g ene ral___2. with a strong justification
for his policy.
A. hit back B. struck up c. leapt up D. pushed forward
20. The free tickets for the band’s concert w er e______ within seconds by the
enthusiasts flocking at the hall door.
A. run up B. broken up c. drawn up D. snapped up
III. VO CABULARY (10 PTS):
Choose the best options to complete the following sentences.
21.1 found the information for the project in the encyclopedia but 1 couldn't give
and verse on it.
A. chapter B. unit c. poem D. extract
22. During the riots, hundreds of people broke into the city's main department
store a nd_____it. There was almost nothing left after the night had finished.
A. mugged B. looted c. conned D. stole
23. When his manager went on a business trip, Smith stepped into th e ______
and chaired the meeting.
A. hole B. pool c. breach D. crack
24. The country has few natural resources and its economy has be en ______ for
some time now.
A. diseased B. unwell c. sickening D. ailing
25. Every time the government meets their demands, the union leaders shift the
A. goalposts B. lamp-posts c. bus stops D. roadblocks
26. They worked from dawn to dusk with s uch_____ that they were exhausted.
A. boredom B. zeal c. crack D. debility
245
Tl. The Prime Minister will decide whether to release the prisoner or not; that 's
his ______ .
A. prerogative B. contortion c. derogatory D. abdication
28. The sheep were huddled into a _____ to protect them from overnight frosts.
A. pen B. hutch c. kennel D. barn
29. That human rights ar e______ is unacceptable in a civilized society.
A. infringed B. impeached c. abrogated D. quashed
30. Ì overslept this morning and caught the last bus to school by______ .
A. the hair o f my head B. the skin of my teeth
c. the nail of my finger D. the skin of my heels
31. Since he is too old to be a porter, they have decided to put him out t o ____.,
A. the door B. grounds c. grass D. the kerb
32. Andrew Johnson was the first president of the US ever to be ______ ,
primarily because of his violent temper and unyielding stubbornness.
A. impeached B. overthrown c. overpowered D. dethroned
33. If she wins the prize again this year, it'll be a rea l______ in her cap.
A. nutshell B. gemstone c. feather D. landmark
34. Maggie is so moody and unpredictable. She’s apt to fly off th e ______
without any real cause.
A. handle B. strap c. catch D. belt
35. Despite a string o f_____ performances he retained his place in the side.
A. promising B. reasonable c. satisfactory D. moderate
36. The earthquake_____ 6.5 on the Richter scale.
A. weighed B. measured c. achieved . D. counted
37. It w as ___clear to me what they meant by their cold manner.
A. blatantly B. fully c. abundantly D. acutely
38. The international conference of the Cardiological Association has been
_____ in Cairo to discuss the revolutionary discovery of doctor Gonzales
from Mexico.
A. deployed B. collected c. mobilized D. summoned
3 9 .1 had a small accident with the car. One of the wings is a li ttle _____ _.
A. stained B. jamm ed c. dented D. cracked
40. I've only been here for a week so 1 haven't got into th e_______ yet.
A. swings of things B. groove o f things
c. rut of things
D. swing o f routine
IV. GUIDED CLOZE (10 PTS):
Read the text below and decide which answer best fits each space.
PASSAGE A:
Plants are (41) ______ to attack and infection by a remarkable variety of
symbiotic species and have evolved a diverse array of mechanisms designed to
246
frustrate the potential colonists. These can be divided into preformed or passive
defense mechanisms and (4 2)______ or active systems. Passive plant defense
comprises physical and chemical barriers that prevent entry of pathogens, such
as bacteria, or (43) ____ _ tissues unpalatable or toxic to the invader. The
external surfaces of plants, in addition to being covered by an epidermis and a
waxy cuticle, often carry spiky hairs known as trichomes, which either prevent
feeding by insects or may even puncture and kill insect (4 4 )______ . Other
trichom es are sticky and glandular and effectively trap and (4 5) ______ insects.
If the physical barriers of the plant are breached, then preformed chemicals may
inhibit or kill the intruder, and plant tissues contain a diverse array of toxic or
potentially toxic substances, such as resins, tannins, glycosides, and alkaloids,
many of which are highly effective (4 6) ______to insects that feed on plants.
The success of the Colorado beetle in infesting potatoes, for example, seems to
be correlated with its high tolerance (4 7)______ alkaloids that normally repel
potential pests. Other possible chemical defenses, while not directly toxic to the
parasite, may inhibit some essential step in the establishment of a parasitic
relationship. For example, glycoproteins in plant cell walls may inactivate
enzymes that degrade cell walls. These enzymes are often produced by bacteria
and fungi. Active plant defense mechanisms are comparable to the immune
system of vertebrate animals, although the cellular and molecular bases are
(4 8 )______ different. Both, however, are triggered in reaction to intrusion.
implying that the host has some means of recognizing the presence of a foreign
organism. The most dramatic example of an inducible plant defense reaction is
the hypersensitive response. In the hypersensitive response, cells undergo rapid
necrosis — that is, they become diseased and die — after being penetrated by a
parasite ; the parasite itself ( 49)__________ ceases to grow and is therefore
restricted to one or a few cells around the entry site. Several theories have been
(50)______ to explain the bases o f hypersensitive resistance.
41. A. likely B. inclined c. flexible D. subject
42. A. inducible B. causative c. influential D. medicinal
43. A. convert B.render c. alternate D. reimburse
44. A. embryos B. larvae c. larva D. caterpillar
45. A. immobilize B. demobilize c, deactivate D. depreciate
46. A. hindrances B. repellents c. deterrents D. expurgatio
47. A. io B. for c. within D.u nder
48. A. efficaciously B. phenomenally c. fundamentally D. originally
49. A. following B. substantially c. procedurally D. subsequen
50. A. brought about B. worked out c. put forward D. laid down
247
PASSAGE B:
MOUND-BUILDER THEORIES
Some o f the most impressive geographical features in North America are the
many earth mounds (5 1 )______ around the continent. These earthworks are
enormous artificial hills constructed by various Native American civilizations for
(52) _ _ _ _ _ purposes, such as burials worship, and they are thousands of years
old, with the latest ones being finished hundreds of years before Europeans
explored America. Upon their arrival, European explorers were impressed with
the structures, but prejudice against the native tribes prevented them from (53)
______Native Americans with their construction. Instead, American settlers
developed several theories that claimed a superior but (54) ______ "moundbuilder"
civilization made the earthworks. Moreover, particular details of
different mound-builder theories reflected the specific prejudices of the people
who supported them. For example, devout Christian groups like the Mormons
argued that a (5 5)______ Israeli society was responsible, and white Americans
argued that only the Vikings could have built such mounds because they
believed that their European (5 6) ______was far superior to Native Americans.
The (57 )______ of these myths showed how for centuries Americans selectively
examined evidence and distorted science in order to support their own agendas
(5 8) ______ the natives. Prior to the 20th century, many Americans accepted the
mound-builder theories as despite the dubious evidence that supported them. For
instance, most believers argued that the presence of metal artifacts beneath the
mounds showed that the natives couldn't have them because they had no
knowledge of metallurgy. Some tribes did in fact possess such skills, and the
presence of defensive walls around tribal lands indicates that Native Americans
could indeed construct structures such as earth mounds. Nonetheless, most
Americans (5 9)______ such evidence and instead considered other potential
candidates for the mound builders. Popular choices were ancient Chinese, Greek,
or African civilizations, none of which were in prehistoric America. Other
people argued that mystical (6 0) ______, such as God or people from mythical
Atlantis, built the mounds.
51. A. scattered B. extended c. stretched D. covered
52. A. philosophical B. infinite c. ritualistic D. opportuni
53. A. commemorating B. accrediting c. recognizing D. endowing
54. A. bygone B. disrupted c. exhausted D. extinct
55. A. secretive B. divine c. celestial D. sacred
56. A. forefather B. ancestors c. foregoer D. ancestry
57. A. persistence B. perseverance c. insistence D. determina
58. A. for B. towards c. against D. from
59. A. missed B. refuted c. dispatched D. disposed
60. A. rites B. vigors c. powers D. motivatio
248
V. READING COMPREHENSION (10 PTS):
Read the texts below and choose the best answer to each question.
PASSAGE A
MUSICAL TALENT
Among all the abilities with which an individual may be endowed, musical
talent appears earliest in life. Very young children can exhibit musical precocity
for different reasons. Some develop exceptional skill as a result of a well -
designed instructional regime, such as the Suzuki method for the violin. Some
have a good fortune to be bom into musical family in a household filled with
music. In a number of interesting cases, musical talent is part of an otherwise
disabling condition such as autism or mental retardation. A musically gifted child
has an inborn talent, however, the extent to which the talent is expressed
publicly will depend upon the environment in which the child lives.
Musically gifted children master at an early age the principal elements of
music, including pitch and rhythm. Pitch - or - melody - is more central in
certain cultures, for example, in Eastern societies that make use of tiny quarter -
tone interval. Rhythm, sounds produced at certain auditory frequencies and
grouped according to a prescribed system, is emphasized in sub- Saharan Africa,
where the rhythmic ratios can be very complex.
All children have some aptitude for making music. During infancy, normal
children sing as well as babble, and they can produce individual sounds and
sound pattern. Infants as young as two months can match their mother's songs in
pitch, loudness, and melodic shape, and infants at four months can match
rhythmic structure as well. Infants are especially predisposed to acquire these
core aspects of music, and they can also engage in sound play that clearly
exhibits creativity.
Individual differences begin to merge in young children as they learn to sing.
Some children can match segments of a song by the age of two or three. Many
others can only approximate pitch at this age and may still have difficulty in
producing accurate melodies by the age o f five or six. However, by the time they
reach school age, most children in any culture have a schema of what a song
should be like and can produce a reasonably accurate imitation of the songs
commonly heard in their environment.
The early appearance of superior musical ability in some children provide
evidence that musical talent may be separate and unique form of intelligence.
There are numerous tales of young artists who have a remarkable "ear" or
extraordinary memory for music and a natural understanding of musical
structure. In many of these cases, the child is average in every other way but
displays an exceptional ability in music. Even the most gifted child, however,
takes about ten years to achieve the levels of performance or composition that
would constitute mastery o f the musical sphere.
249
Every generation in music history has it famous prodigies - individuals with
exceptional musical powers that emerge at a young age. In the eighteenth
century, Wolfgang Amadeus Mozart began composing and perform ing at the age
of six. As a child, Mozart could play piano like an adult. He had perfect pitch,
and at the age of nine he was also a master of the art o f modulation - transitions
from one key to another - which became one of the hallmarks o f his style. By the
age of eleven, he had composed three symphonies and 30 other major works.
Mozart's well - developed talent was preserved into adulthood.
Unusual musical ability is a regular characteristic o f certain anomalies such as
autism. In one case, an autistic girl was able to play "Happy Birthday" in the
style of various composers, including Mozart, Beethoven, Verdi, and Schubert.
When the girl was three, her mother called her by playing incomplete melodies,
which the child would complete with the appropriate tone in the proper octave.
For the autistic child, music may be the primary mode of communication, and
the child may cling to music because it represents as a haven in a world that is
largely confusing and frightening.
61. The word "precocity" in paragraph 1 is closest in meaning to ■
Ạ. strong interest B. good luck c. a
62. Which sentence below best expresses the essential information in the
sentence printed in bold in paragraph 1?
A. Children may be born with superior musical ability, but their environment
will determine how this ability is developed.
B. Every child is naturally gifted, and it is responsibility o f the public schools
to recognize and develop these talents.
. c. Children with exceptional musical talent will look for the best way to
express themselves through music - making.
D. Some musically talented children live in an environment surrounded by
music, while others have little exposure to music.
63. The author makes the point that musical elements such as pitch and rhythm '
A. distinguish music from other art forms
B. vary in emphasis in different cultures
c. make music difficult to learn
D. express different human emotions
64. The word "predisposed" in paragraph 3 is closest in meaning to ______ .
A. inclined B. gifted c. pushed D. amused
65. According the passage, when does musical talent usually begin to appear?
A. When infants start to babble and produce sound patterns
B. Between the ages of two and four months
c. When children learn to sing at two or three years old
D. Between ten years old and adolescence
250
66. According the passage, which o f the following suggests that musical talent is
the separate form o f intelligence?
A. Exceptional musical ability in an otherwise average child
B. Recognition of the emotional power o f music
c. The ability of all babies to acquire core elements o f music
D. Differences between learning music and learning language
67. Why does the author discuss Mozart in paragraph 6?
A. To compare past and present views o f musical talent
B. To give an example of a well - known musical prodigy
c. To describe the development of individual musical skill
D. To list musical accomplishments of the eighteenth century
68. All of the following are given as examples of exceptional musical talent
EXCEPT
A. a remarkable "ear" or perfect memory for music
B. ability to compose major works at a young age
c. appreciation for a wide variety o f musical styles -
D. playing a single song in the style of various composers
69. The word "haven" in paragraph 7 is closest meaning t o ______ .
A. beautiful art B. safe place c. personal goal D. simple problem
70. Which of the following can be inferred from the passage about exceptional
musical ability?
A. It occurs more frequently in some cultures than in others.
B. It is evidence of a superior level of intelligence in other areas,
c. It has been documented and studied but is little understood.
D. It is the result o f natural talent and a supportive environment.
PASSAGE B
THE BALANCE IN THE OCEANS
The oceans' predators come in all shapes and sizes. For example, one of the
less infamous ones is the colorful starfish, which feeds on plant life, coral, or
other shellfish such as mussels for sustenance. A more bloodcurdling example,
especially to human beings and most other species of fish, is the shark, though
most scientists agree that only ten per cent o f the 450 plus species of sharks have
been documented as actually attacking a human. Still, there is another predator
lurking invisibly in the bodies of water of the world, one which poses one o f the
greatest threats to all species o f ocean life - bacteria. Though many types o f fish
are continually stalking and evading one another for survival, they all band
togethe r in an attempt to keep bacteria levels at bay in order to allow, their
own existenc e to continue.
Bacteria play a dual role in the ecosystems of the oceans. On the one hand, ■
they are beneficial as they stimulate plant life through food decomposition,
which releases the necessary chemicals for the growth of plant life. This is called
251
nutrient recycling and helps keep the oceans alive. But, on the other hand,
bacteria are a major predator for all fish because the y'attack fragile, weaker
individuals. If they are allowed to run rampant and not kept in check, they could
virtually suffocate the oceans. Ill water, bacteria prove to be an even greater
threat than on land because, as they proliferate, they reduce the oxygen levels
necessary for organisms in the oceans to live. Further, when fish populations
become depleted due to factors like overfishing, microbes such as algae expand
and threaten the fragile ecosystems of the ocean. Therefore, ocean predators play
a critical role by thwarting bacteria growth and maintaining the oceans'
equilibrium by reducing vulnerable links in the food chain.
In many ways, the balance within the oceans’ ecosystems mirrors the human
body. That is, all of their components must work in harmony for them to stay
healthy, efficient, and alive. If one of them is missing or deficient, an entire
system can be placed in jeopardy. In both the human body and the ocean,
bacteria play a vital role because, at manageable levels, they aid in protecting
and cleaning each system of foreign agents that can be of harm. On the other
hand, if bacteria levels increase and become out of control, they can take hold o f
a system, overrun it, and become debilitating. Therefore, both oceans and the
human body have a kind of custodian that maintains bacteria levels. In the
human body, it is called a phagocyte. Phagocytes eat up sick, old, or dying cells,
which are more prone to bacterial invasion, and thus keep the body healthy. Like
in the human body, bacteria can prove fatal to the living organisms in the ocean.
Like phagocytes in the human body ocean .predators work as antibacterial
custodians of the seas. In essence, they are the immune system and a vital link in
the food chain because they remove small, injured, or sickly fish from the ocean
environment before bacteria can become too. comfortable and multiply. By
ridding the ocean of weaker fish, predators allow the stronger ones to multiply,
making their species stronger and more resilient. Without their services and with
their declining numbers, bacteria will blossom to levels that will eventually
overpower and kill even the strongest species off ish because of the depletion of
their number one source of life, all important oxygen.
- While the greatest battle in the ocean may seem on the surface to be the
survival of the fittest fish, a closer look reveals something completely different:
fish versus microorganisms. Clearly, most living organisms in the oceans are
hunters by nature, but this way o f life does not merely provide a food source for
a dominant species, It also maintains a healthy level of bacteria in an ocean's
ecosystem, thus ensuring the continuation of all species of life within. Major
predators are necessary, like the antibacterial cells of the human body, to keep
this delicate balance in synch. If their numbers continue to decline and humans
ignore their vital role in the ocean, dire consequences will definitely result.
71. The word “lurking ” in the passage is closest in meaning to ■
A. attacking B. increasing c. waiting D. approaching
252
72. According to paragraph 1, which of the following is true of ocean predators?
A. The shark is the deadliest one for all other kinds of life in the oceans.
B. One o f the most threatening to all fish populations is bacteria.
c. Starfish do little damage to the population of mussels and shellfish.
D. Most o f the killers that hide in the oceans are unknown to humans.
73. Which ofthe following can be inferred from paragraph 1 about bacteria?
A. They can be extremely detrimental to fish if their numbers increase.
B. They are able to feed off themselves when other food sources are limited,
c. They stimulate plant life, which in turn releases oxygen into the water.
D. They present themselves in numerous shapes and forms as well as colors.
74. The author discusses “nutrien t recycling” in paragraph 2 in o rderto______ .
A. show how bacteria act similarly in the ocean and the human body
B. explain the different roles of nutrients and oxygen for species of fish
c. indicate that bacteria do have a positive impact in the oceans
D. note how chemicals from bacteria are able to stimulate plant growth
75. Which o f the sentences below best expresses the essential information in the
sentence in bold (“Though... con tinue”)?
A. Evasion tactics help fish escape from the threats posed by an increasing
number o f bacteria.
B. Various species of fish prey upon one another in order to lower bacteria
levels in the ocean.
c. High bacteria levels in the ocean help most species of fish to survive by
providing them with food.
D. Rivals or not, all fish help one another survive by preventing bacteria from
proliferating.
76. The word “thwarting” in the passage is closest in meaning to ______.
A. encouraging B. preventing c. slowing D. sustaining
77. According to paragraph 2, bacteria are dangerous to ocean life bec ause____.
A. they have the capability to attack both strong and weaker fish
B. they could monopolize the critical breathable gas in the ocean
c. they get rid of vulnerable links, like dying fish, in the food chain
D. they blossom out of control when overfishing becomes dominant
78. The word “deb ilitating” in the passage is closest in meaning to ______.
A. stimulating B..hindering c. elevating D. weakening
79. The author’s description of phagocytes mentions all of the following
EXCEPT
A. They rid the human body of potentially dangerous organisms.
B. They act in a similar manner as the predators of the ocean,
c. They dispose o f bacteria to make weakened cells revive.
D. They are cleaning agents in humans to maintain bacteria levels.
80. The word “It” in the passage refers t o ______.
A. nature B. way of life c. food source D. dominant species
253
B. WRITTEN TEST
I. CLOZE TEST (20 PTS):
Read the texts below and think of the word which best fits each space. Use
only ONE WORD for each space.
PASSAGE A
BRAIN GEL
A gel that helps brains recover from traumatic injuries has been developed by
scientists at the Clemson University in South Carolina. The gel, which is injected
in liquid (1 )______at the site where the injury was sustained, stimulates (2)
______cell growth in the affected area. In terms of circumstances (3 )_______
which it might be applied, the gel has the potential to treat a wide range of head
injuries, including those arising from car accidents, falls and gunshot wounds.
Serious brain injuries are notoriously difficult to recover from on (4 )______
of the fact that the affected tissue can swell ( 5 )______ considerably, which
causes additional collateral damage to the surrounding cells. Existing treatments
do (6 )______more than attempt to limit secondary damage and are relatively
ineffective, certainly when it comes to repairing the damaged cells, so the
discovery of a gel which stimulates cell repair is being heralded (7) ______
revolutionary.
(8 )______ the wave o f excitement now running through medical circles, it is
important to note that results so far are based solely on observations of the
effects of the gel on (9 )__ ___ rats. The development of the treatm ent is very
much still in its preliminary stage and human testing is expected to be some three
years or more (10) ______ yet.
PASSAGE B
The Dolby family gift is the largest philanthropic (1 1)______ ever made to
UK science, and will support the Cavendish Laboratory, the world-leading centre
for physics (1 2) ______where Ray Dolby received his PhD in 1961. Thanks to
this exceptional gift, the University has now surpassed the £1 billion milestone
in its current £2 billion fundraising campaign. This is the second generous gift to
Cambridge (1 3) ______the Dolby family, who donated £35 million to Pembroke
College, Cambridge in 2015. The Dolby family is now the largest donor to the
fundraising campaign, and the (1 4) ______-largest donor to the University in its
808-year history.
Ray Dolby, who died in 2013 at the age of 80, came to Cambridge as a
Marshall Scholar in 1957. He received his PhD from the Cavendish in 1961, and
was a student and later a Fellow of Pembroke College. In 1965, he founded
Dolby Laboratories in London and invented the Dolby System, an analogue
audio encoding system that forever improved the (15) _ _ _ _ _ of recorded sound.
He moved the company in 1976 to San Francisco, where it has been ( 16 )______
ever since.
254
The new Cavendish Laboratory will be its third home since its founding in
1874, and was first announced by the government in its 2015 spendin g Review.
it (12) ______ a £75 million investment in the Cavendish, which has been
confirmed today, (1 8)______ maintain Britain's position at the (1 9)______ of
physical sciences research. The funding will be delivered by the Engineering and
Physical Sciences Research Council (EPSRC). Work (20) ______ the new
facility is expected to begin in 2019.
IL WORD FORMATION: (20PTS)
PART 1: Complete each sentence, using the correct form of the word in
parentheses.
1. Since his bad habits were never broken when he was a child, they are now
______ . (CORRECT)
2. The ending was rather______ considering the fact that the film had been so
intense throughout. (CLIMAX)
3. Environmental quality will be th e______ for the 21 St century. (WORD)
4. 1 found the last scene extremely moving and particularly______ . (DIRECT)
5. At that time, package holidays to tropical resorts were pretty m uc h_____ _.
(HEAR)
6. There is a craze f or______ foods in our country these days. (CANCER)
7. With free-market globalization, investment funds can mov e______ from the
rich countries to the developing countries. (IMPEDIMENT)
8. Airport officials said there had been no hint of trouble until radio contact was
suddenly lost three minutes fro m______ . (DOWN)
9. Copyright is the _ ____ , legally secured right to publish, reproduce, and sell
the matter and form o f a literary, musical, dramatic, or artistic work. (ALIEN)
10. ______ pain is the reason for around 5% of all emergency department visits.
(ABDOMEN)
PART 2: Complete the passage with appropriate forms from the words given
in the box.
in conscientious industry emergence
periphery development nation abound
The issues for (1 1)________ economies are a little more straightforward.
The desire to build on (12). _____ land is not born out of desperation or.
necessity, but is a result of the (1 3)________ march of the progress. Cheap
labour and a relatively highly-skilled workforce make these countries highly
competitive and there is a flood of (14) ________ investment, particularly from
(15) looking to take advantage of the low wages before the cost and
standard of living begin to rise, it is factors such as these that are making many
Asian economies extremely attractive when viewed as investment opportunities
255
at the moment. Similarly, in Africa, the relative (16) ________ of preciousmetals and natural
resources tends to attract a lot o f exploitation companies and a
whole sub-industry develops around and is completely dependent on this
foreign-direct investment. It is understandable that countries that are the focus o f
this sort of attention can lose sight of the environmental (1 7) ________ of largescale
industrial development, and this can have devastating consequences for the
natural world. And it is a vicious cycle because the more (1 8) ________ active a
nation becomes, the greater the demand for and harvesting of natural resources.
For some, the environment issues, though they can hardly be ignored, are viewed
as a ( 19)________ concern. Indeed.'having an environmental (2 0) _________or
taking environmental matters into consideration when it comes to decisions on
whether or not to build rubber-tree plantations or grow biofuel crops would be
quite prohibitive indeed. For those involved in such schemes it is a pretty blackand-white
issue. And, for vast tracts of land in Latin America, for example, it is
clear that the welfare of the rainforests matters little to local governm ent when
vast sums o f money can be made from cultivating the land.
III. ERROR CORRECTION: (10PTS)
The following passage contains 10 errors. Identify and correct them.
SPORTS PHOTOGRAPHY
1 Sport as a spectacle and photography as a way of recording action have
developed together. At the arrival of the 20th century, Edward Muybridge
was experimenting with photographs of movement. His pictures of a
runner feature in every history of photography. Another milestone was
5 when the scientist and the photographer Harold Edgerton extended the
limits of photographic technology with his study of a drop of milk
hitting the surface of a dish. Another advance was the development of
miniature cameras in the late 1920s, that made it possible for photographers
to put their cumbersome cameras behind.
10 The an-ival of television was a significant development in the transmission
of sport. Paradoxically, it was having benefit to still photographers.
People who watched a sport event on TV, with all their movement and
action, valued the still image as a reminder of the game.
Looking back, we can see how radically sports photography has changed.
15 Early sports photographers were so interested in the stories behind the
sport as in the sport itself. Contemporary sports photography emphasizes
the glamour of sport, the colour and the action. But the best sports
photographers today do more than just simply tell the story of the event,
or take a record of it. They capture in a single dramatic moment the real
20 emotions of the participants, emotions which people looking at the
photographs can identify.
256
1.
4.
7.
2.
5.
8.
3.
6.
9.
0.
IV. SENTENCE TRANSFORMATION: (20 PTS)
Rewrite the following sentences using the words given.
1. Frankly speaking, I do not think that nature and nurture are equally significant.
(SUBSCRIBE)
To p u t............................................. ............................................... significance.
2. In the area, Thailand used to be much better than all other countries in
football. (HEAD)
In the area, Thailand doesn't ............ .................................................... longer
3. He hid the truth about the matter because he didn’t want to lose his high
position. (CARPET)
For fear that ......... ..............................................................................................
4. He indicated very clearly that he would do anything to obtain the contract.
(LENGTHS)
He g ave................................................................................................ -...............
5 .1 d idn 't dare to tell the truth because my father was there. (GUTS)
Had it ....... ..........................................................................................................
6. It was not until five years had elapsed that the whole truth about the murder
came out.
- Not f o r........................... ............................................................... .................... .
7. She was surprised when he suddenly asked her to marry him. (TOOK)
H is .................................................................................................. ....................
8. He seems to find the way Mimi behaves more a source of amusement than
embarrassment.
Far from .............................................................. ......... ............. amused by it.
9. After the expedition, I realized that she was not an efficient researcher.
(SHAKES)
It dawned .............................................. .......................................a researcher.
10. The accused may imagine a plan of killing the witness. (AWAY)
The accused may conceive .............. .................................... ............................
257
*•' «r.a*.» »wrci<:'»■*' »V f e « ~ t j « w W
TRƯỜNG T HPĨ CHUYÊN NGUYỄN BỈNH KHIÊM - VĨNH LONG
A. MULTIPLE CHOICE
I. WORD CHOICE:
Choose the best options to complete the following sentences.
1. He cannot.................. .. ignorance as his excuse; he should have known what
was happening in his own department.
A. Insist B. defend c. refer D. plead
2. Employees who have a ........................ are encouraged to discuss it with the
management.
A. Hindrance B. grievance c. disadvantage D. disturbance
3. The police have been ordered not to ............. if the students attack them
A. Combat B. challenge c. retaliate D. rebuff
4. In spite of his poor education, he was a most....................... speaker.
A. Attentive B. ambiguous c. articulate D. authoritarian
5. Sparkling pools o f water lay trapped among the rocks as the tide.....................
A. removed B. refilled c. retired D. receded
6. ...........through the attic and see i f you can find anything for the jumble sale
A. forage B. ravage c. rummage D salvage
7. They began constructing the bridge in 1960, but several years........ .. ......
before the project was completed.
A. elapsed B. advanced c. proceeded D. progressed
8. People still haven’t ..................how dangerous pollution can be.
A. remarked B. realised c. noted D. minded
9. Competitive ................. is an essential requirement for success in the
entertainment industry.
A. ,mind B. thought c. spirit . D.soul
10. You must complete the Business................... course satisfactorily before
you can progress to the third year.
A. Morals B. Values c. Rights D. Ethics
11. STRUCTURES & GRAMMAR
Choose the best option A, B, c or D.
1. He tried to make out that the fake painting he had ____ __ genuine.
A. to be B. being c. been D. was
2. The city libraries present a gloomy picture of th e ___________ who used to
flock the libraries every evening.
A. gradual reduction of readers B. gradual readers reduction
c. gradual readers of reduction D. reduction gradual readers
258
3. Centuries of erosion have exposed____ rock surfaces in the Painted Desert of
northern Arizona.
A. in colors o f the rainbow B. colored like a rainbow
c. rainbow-colored D. a rainbow's coloring
4. The temperature_______ takes place varies widely from material to material.
A. which melting B. at which melting
c. at which they melt D. which they melt
5. L egend_____________ that Robin Hood fired an arrow from his dead-bed and
was buried where the arrow landed.
A. tells it B. says it c. makes it D. has it
6. A m ajor problem in the construction of new buildings_________ .
A. is that windows have been eliminated while air conditioning systems have
not been perfected.
B. is they have eliminated windows and still don’t have good air conditioning,
c. is because windows are eliminated but air conditioners don’t work.
D. is dependent on the fact that while they have eliminated windows, they are
not capable to produce efficient air conditioning systems.
7. There’s no point in telephoning him. He’s certain___________ by now.
A. to leave B. to have left c. left D. having left
8. _____________ , the catfish is prized for its taste.
A. With ugly look B. As ugly looking
c. Ugly looking as it is D. As it is ugly looking
9. Mr. Gump sup pos es,_____________ , that he will retire at 60.
A. like most people did B. as most of people
c. like most people do D. as do most people
10. On the battle f ield ______________________ .
A. the tanks lay B. did the tanks lie
c. lay the tanks D. lied the tanks
III. PREPOSITIONS & PHRASAL VERBS
Choose the word or phrase which best completes each sentence.
1. This is the time of the year when stores____their prices, so you can get good deals.
A. mark on B. mark through c. mark up D. mark down
2. You shouldn’t have sent Sebastian that Valentine’s card. 1 think you’ve scared
him_____ .
A. back ■ B. down c. off D. through
3. His bad bahaviour was put_________ his upbringing.
A. down to B. with c. off D. up
4. hl the end, the detective managed to ______down the dangerous criminal.
A. find B. track c. get D. hit
5. He is not exactly rich but he certainly earns enough to ______ .
A. get through B. get by c. get on D. get in
259
6. The robbers packed the money into a suitcase an d______ in a van that waited
for them in the street.
A. put through B. rolled over c. carried away D. made o ff
7. The weather was fine, and everyone was _ ____ the coast.
A. going for B. making for c. joining in D. seeing about
8. Learning English isn’t so difficult once you .
A. get down to it B. get of f it c. get on it D. get down with it
9. Sometimes a postman ______ some terrible handwriting and did n't know
where the letter should go.
A. ran away with B. ran up with c. ran up against D. run without
10. Beaches were ■ as police searched for canisters o f toxic waste fr.om the
damaged ship.
A. sealed off B. cut off c. washed up D. kept out
IV. COLLOCATIONS & IDIOMS
Choose the word or phrase which best-completes each sentence.
1. Peter is a librarian but this job is not suitable for him because he has chances
of traveling______. He should be a tour guide.
A. in a mind B. on the head c. on the brain D. in the heart
2.1 think people who help the old, poor, sick and homeless ar e______ .
A. the sugar of the sea B. the salt of an ocean
C. the salt of the earth D. the sugar o f the ocean
3. The te acher__when she knew that more than ten students cheated in her test.
A. angry B. crazy c. hit the ro of D. beat the desk
4. We don't want to continue our business any longer. All the goods will
'
so that we can close at the end o f this month.
A. be sold out B. be solved thoroughly
c. go for a song D. go with colour
5. He lo oke d___ at the security guard of the supermarket when this man asked
him to open his bag.
A. daggers B. angry c. up to D. attentively
6. Don’t be angry with Sue. All that she did in good______
A. hope B. belief c. idea D. faith
7.1 overslept this morning and caught the last bus to school by______ .
A the hair of my head B. the skin of my teeth
c. the nail o f my finger D. the skin of my heels
8. The sixth time he called me at night was t he ____
A. last cause ,B. last straw c. touch and go D. ho tai r
9.1 do not think there is so much as a ____o f truth in that rumor.
A. crumb B. speck c. grain D. pebble
10.____ the public concern about the local environment, this new road scheme
will have to be abandoned.
A. As regards B. In the event of c. In view of D. However much
260
V. READING COMPREHENSION
PASSAGE 1: Read the passage and choose the best answer to each question
Rocky Mountain Spotted Fever, an acute febrile illness, is transmitted to man
by ticks. Prevention is attained primarily by avoidance of tick-infested areas.
When this impractical, personal prophylactic measures include the wearing of
clothing which interferes with attachment of ticks, i.e., boots and a one-pie ce
outer gar me nt, pre ferably impregnated with a tick repelle nt, and daily
inspec tion of the entire body, inclu ding the hairy parts, to detect and
rem ove atta che d ticks.
In removing attached ticks, great care should be taken to avoid crushing the
arthropod, with resultant contamination of the bite wound. Touching the tick
with gasoline or whisky encourages detachment, but gentle fraction with
tweezers applied close to the mouth parts may be necessary. The skin area
should be disinfected with soap and water or other antiseptics. Similarly,
precautions should be employed in removing engorged ticks from dogs and other
animals, since infection through minor abrasions on the hands is possible.
Vaccines are available commercially and should be used for those exposed to
great risk, viz., persons frequenting highly endemic areas and laboratory workers
exposed to the agent. Since the broad-spectrum antibiotics were shown to be
such excellent therapeutic agents in Spotted Fever, there has been less impetus
for vaccination o f persons who run only a minor risk of infection.
1. What is the main purposd o f the passage?
A. To describe the symptoms of Spotted Fever.
B. To explain how to treat Spotted Fever,
c. To warn o f the danger of Spotted Fever.
D. To outline the types of Spotted Fever.
2. The word “acute” in line 1 is closet in meaning to
A. fatal B. violent c. serious D. incurable.
3. From the passage, it can be inferred that ticks are
A. prickly plants B. biting animals
, c. poisonous reptiles
D: blood-sucking insects.
4. The word “this” refers to
A. prevention B. avoidance c. attachment D. clothing
5. The author suggests all of the following as preventative measures against
Spotted Fever EXCEPT
A. removing hair from body c. staying away from infested areas.
B. dressing in suitable clothes D. using appropriate sprays.
6. The word “fraction” could be replaced by
A. dissection B. examination c. investigation D. removal
7. The author states that most people become ill with Spotted Fever through
A. squeezing the body o f a tick B. playing with dogs
c. working in laboratories D. not washing with soap and water.
261
8. According to the passage, if whisky is applied to a tick, it
A. attaches itself to the mouth B. becomes very confused
c. bites the person D. falls off the body
9. The word “im petus'1 is closest in meaning to
A. haste B. need c. chance D. thought
10. The passage supports which of the following conclusions?
A. There is no known cure for Spotted Fever.
B. All people in the Rocky Mountains should vaccinated
c. Most medicines are ineffectual against spotted Fever.
D. Spotted Fever is controllable with the use o f suitable antibiotics.
PASSAGE 2: Read the passage and choose the best answer to each question
THE CREATORS OF GRAMMAR
No student of a foreign language needs to be told that grammar is complex.
By changing word sequences and by adding a range of auxiliary verbs and
suffixes, we are able to communicate tiny variations in meaning. We can turn a
statement into a question, state whether an action has taken place or is soon to
take place, and perform many other word tricks to convey subtle differences in
meaning. Nor is this complexity inherent to the English language. All languages,
even those of so-called 'primitive' tribes have clever grammatical components.
The Cherokee pronoun system, for example, can distinguish between 'you and r,
'several other people and I' and 'you, another person and I'.. In English, all these
meanings are summed up in the one, crude pronoun 'we'. Grammar is universal
and plays a part in every language, no matter how widespread it is. So the
question which has baffled many linguists is - who created grammar?
At first, it would appear that this question is impossible to answer. To find
out how grammar is created, someone needs to be present at the time of a
language's creation, documenting its emergence. Many historical linguists are
able to trace modern complex languages back to earlier languages, but in order
to answer the question of how complex languages are actually formed, the
researcher needs to observe how languages are started from scratch.
. Amazingly, however, this is possible.
Some o f the most recent languages evolved due to the Atlantic slave trade. At
that time, slaves from a number of different ethnicities were forced to work
together under colonizer's rule. Since they had no opportunity to learn each
other's languages, they developed a make-shift language called a pidgin. Pidgins
are strings of words copied from the language o f the landowner. They have little
in the way of grammar, and in many cases it is difficult for a listener to deduce
when an event happened, and who did what to whom. ỊAỊ Speakers need to use
circumlocution in order to make their meaning understood. [BJInterestingly,
however, all it takes for a pidgin to become a complex language is for a group of
children to be exposed to it at the time when they learn their mother tongue.
262
|CỊ Slave children did not simply copy the strings of words uttered by their
elders, they adapted their words to create a new, expressive language.
[D| Complex gramm ar systems which emerge from pidgins are termed creoles,
and they are invented by children.
Further evidence of this can be seen in studying sign languages for the deaf.
Sign languages are not simply a series of gestures; they utilise the same
gramm atical machinery that is found in spoken languages. Moreover, there are
many different languages used worldwide. The creation of one such language
was documented quite recently in Nicaragua. Previously, all dea f people were
isolated from each other, but in 1979 a new government introduced schools for
the deaf. Although children were taught speech and lip reading in the classroom,
in the playgrounds they began to invent their own sign system, using the gestures
that they used at home. It was basically a pidgin. Each child used the signs
differently, and there was no consistent grammar. However, children who joined
the school later, when this inventive sign system was already around, developed
a quite different sign language. Although it was based on the signs of the older
children, the younger children's language was more fluid and compact, and it
utilised a large range of grammatical devices to clarify meaning. What is more,
all the children used the signs in the same way. A new creole was born.
Some linguists believe that many of the world's most established languages
were creoles at first. The English past tense - ed ending may have evolved from
the verb 'do'. 'It ended' may once have been 'It end-did'. Therefore it would
appear that even the most widespread languages were partly created by children.
Children appear to have innate grammatical machinery in their brains, which
springs to life when they are first trying to make sense of the world around them.
Their minds can serve to create logical, complex structures, even when there is
no grammar present for them to copy.
1. In paragraph 1, why does the writer include information about the Cherokee
language?
. A. To show how simple, traditional cultures can have complicated grammar
structures
B. To show how English grammar differs from Cherokee grammar
c. To prove that complex grammar structures were invented by the Cherokees.
D. To demonstrate how difficult it is to learn the Cherokee language
2. What can be inferred about the slaves' pidgin language?
A. It contained complex grammar.
B. It was based on many different languages.
c. It was difficult to understand, even among slaves.
D. It was created by the land-owners.
3. All the following sentences about Nicaraguan sign language are true EXCEPT:
A. The language has been created since 1979.
B. The language is based on speech and lip reading.
263
c. The language incorporates signs which children used at home.
D. The language was perfected by younger children.
4. In paragraph 3, where can the following sentence be placed?
It included standardised word orders and grammatical markers that existed
in neither the pid gin language, nor the language o f the colonizers.
ABCD
5. 'From scratch' in paragraph 2 is closest in meaning to:
A. from the very beginning B. in simple cultures
c. by copying something else D. by using written information
6. 'Make-shift' in paragraph 3 is closest in meaning to:
A. complicated and expressive B. simple and temporary
c. extensive and diverse D. private and personal
7. Which sentence is closest in meaning to the highlighted sentence?
Grammar is universal and plays a p art in every language, no matter how
widespread it is.
All languages, whether they are spoken by a few people or a lot o f people,
contain grammar.
B. Some languages include a lot of grammar, whereas other languages
contain a little.
c. Languages which contain a lot of grammar are more common that
languages that contain a little.
D. The grammar o f all languages is the same, no matter where the languages
evolved.
8. All of the following are features of the new Nicaraguan sign language EXCEPT:
A. All children used the same gestures to show meaning.
B. The meaning was clearer than the previous sign language,
c. The hand movements were smoother and smaller.
D. New gestures were created for everyday objects and activities.
9. Which idea is presented in the final paragraph?
A. English was probably once a creole.
B. The English past tense system is inaccurate.
c. Linguists have proven that English was created by children.
D. Children say English past tenses differently from adults.
10. Look at the word 'consistent' in paragraph 4. This word could best be
replaced by which of the following?
A. natural
B. predictable c. imaginable D. uniform
IV. CLOZE TEST
CLOZE TEST 1: Read the passage below and choose A, B, c, or D which
best fits each space.
Film directors usually make the least promising subjects for biography. They
tend to stay behind the camera and get on with making films, emerging only to
264
make the particular promotional statement. Only rarely is a film-m aker
interesting enough to (1)_____ biographical interest, and some pay off the
attention handsomely. What biographer could (2) ____ analyzing Hitchcock,
Woody Allen or Polanski? These directors, in any case, were themselves
sufficiently absorbed in their own (3 )____to cross over the other side of the
camera and (4 )____ themselves to the public.
Much of Jean Renoir's public profile is ( 5 )____ on his appearance in his final
film. But judg ing by the most recent biography, by Ronald Bergan, the man was
simply not interesting. He grew up in the benevolent ( 6 ) ____of his painter
father, against whom he appears not to have (7 )____in any way, emerged to
make his own (8 )____ in the early life and went on making films for most o f the
rest of his life. Even when (9 )____ by war, Renoir seems to have (1 0)____ his
career with.
1. A. award B. prize c. value D. merit
2. A. oppose B. resist c. confront D. expel
3. A. view B. image c. trend D. expression
4. A. display B. examine c. allow D. distinguis
5. A. described B. based c. imagined D. gathered
6. A. memory B. vision c. shadow D. regard
7. A. rebelled B. related c. referred D. resisted
8. A. spot B. field c. mark D. point
9. A. faced B. tackled c. charged D. opposed
10. A. flown B. sailed c. wandered D. run
CLOZE TEST 2: Read the passage below and choose A, B, c , or D which
best fits each space.
It's a sleepy village, whose main features are a central square with a fountain
and an unpretentious resta urant.(l)_____ the place for an internationally famous
exhibition attracting 15000 visitors, one would think. Yet Bussière-Badil has just
that reputation in the world of ceramics.(2________, when a pottery fair was
first held there over 30 years ago, it was only one in all of France, and it is still
the country's only ceramics fair that (3)________ four days.
But why here? There is a seam of clay which runs through the area, but it is
red clay of the type used to make tiles and bricks as (4)______ pots, so there is
no (5)_________ tradition of art pottery. The idea of the fair started when a
Portuguese potter by the name of Miguel Calado (6)_______ a studio in the
village at the (7)_________ of the mayor, himself a local tile-maker, who was
determ ined to put the region on the map.
And he has certainly succeeded. Every year, up to 40 potters from all over
France and beyond (8)________ on the village to display their wares in a huge
purpose-built shed. (9)______ _ on the show range from the utilitarian to the
decorative, with every nuance in between. And the crowds come to look, to
(10)_________ at the potters’ art, and to buy.
265
1. A. Barely B. Seldom c. Hardly D. Unlikely
2. A. Nevertheless B. However c. Indeed D. Otherwise
3. A. perseveres B. endures c. continues D. lasts
4. A. opposed to B. rather than c. instead o f D. apart form
5. A. certain B. particular c. exact D. individual
6. A. turned up B. took up c. made up D. set
7. A. instigation B. advice c. encouragement D. persuasion
8. A. gather B. assemble c. converge D. collect
9. A. Issues B. Items c. Matters D. Topics
10. A. astonish B. fascinate c. amaze D. marvel
B. WRITTEN TEST
I. OPEN CLOZE TEST
OPEN CLOZE TEST 1: Fill in each numbered space with one suitable word:
Everyone wants to reduce pollution. But the pollution is (1) ..................
complicated as it is serious. It is serious. It is complicated (2 ).................much
pollution is caused by things that benefit people. For example (3)........................
from automobiles cause a large percentage of all air pollution. But the automobile
(4 )................... transportation for millions of people.
Factories (5)............... much of the material that pollutes air and water, but
factories give employment to a large number o f people.
Thus, to end (6)...................... greatly reduce pollution immediately, people
would have to stop using many things that (7)..................them. Most people do
not want to do that, of course. But pollution can be (8)................. reduced in
several ways. Scientist and engineers can work to find ways to lessen the amount
of pollution that such things as automobiles and factories cause. Governments
can pass and enforce laws that (9)................... businesses and (10).......................
to stop, or cut down on certain polluting activities.
OPEN CLOZE TEST 2: Fill in each numbered space with one suitable word
Copyright is the inalienable, legally secured right to publish, reproduce, and
Sell the matter and form of a literary, musical, dramatic, or artistic work.
Copyright is designed (1)..................... to protect an artist, publisher, or other
owner against any unauthorised copying of his works - as by reproducing the
work in any material form, publishing it, performing it in public, filming it,
broadcasting it, causing it to be distributed to (2 ) ................ or making any
adaptation of the work. A copyright supplies a copyright holder with a kind of
ownership over the created material, (3 )................. assures him o f both control
over its use and the monetary benefits derived from it. Historically, copyrights
grew (4) ..................... of the same system as royal patent grants, by which
certain authors and printers were given the exclusive (5)................... to publish
books and other materials. The (6 ).........................purpose of such grants was
not to protect authors' or publishers' rights but to (7) .....................government
266
revenue and to give governing authorities control over publicised contents. 1 he
Statute of Anne, passed in England in 1710, was a (8 )...................in the history
ol copyright law as it recognised that authors should be the ( 9 ).........................
beneficiaries of copyright law. Today, the Berne Convention of 1886 and the
Universal Copyright Convention of 1955 protect rights (10) ..................... an
international level recognised in all countries.
II. WORD FORMS
WORD FORM 1: Supply the correct forms of the words in brackets
1. 1 was late because 1......................how much time 1 would need, (estimate)
2. 1 don't care if y ou 'd had too much to drink. Your behaviour last night was
qui te ........... (defend)
3. The cat is ...............in the sunshine, (luxury)
4. Don't judge the matter by my op po ne nt 's.................. statement, but wait till
you have heard the other side, (lateral)
5. Some phones can recognize your voice and b e ........................by it. (act)
6. Are all th os e....................they put in food really necessary? (add)
7. The bicycle I lent Tom had been in good condition, but he returned it in
.....................(repair)
8. She c om pla ine d.................. about his rudeness, (cease)
9. 1 try to ......................■ an occasion when I might be 600 feet below a level.
(vision).
10...................and overpopulation are two of the most difficult problems in the
Third World countries, (forest)
WORD FORM 2:
Read tile text below. Use the word given in capitals at the end of some o f the
lines to form a word that fits in the space in the same line,
FLAMENCO DANCE
The essence of flamenco is song, often accompanied by the guitar and
improvised dance. Music and dance can be placed into specific groups.These
categories are usually located across a continuum with subjects dealing with the
profound to those that are light-hearted. (1. T YP E) .........................., the themes
of death, anguish and despair, in contrast to love, gaiety and the countryside are
(2. DR AM A)...... ............ In flamenco dance, the men's steps are intricate, with
toe and heel clicking. Footwork in women's dancing is o f less importance, with
the (3. GR AC E)............. use of hands and body taking (4. PRECEDE )...............
In the dance, the arm, hand and foot movements closely resemble those of
classical Hindu dance. Essential to traditional flamenco is the performer's
interpretation of the dance (5. HIN DER)................ by the emotion of the music.
Performances are often accompanied by rapid hand clapping, finger snapping
and (6. CO UR AG E)............. shouts. The dancers themselves frequently employ
linger snapping in complex rhythms including the use of castanets. This dance
267
form was (7. PROF ESSIO N)..............in the 19th century, when Romany people
first began to perform in cafes. In this environm ent, (8. DE PA RT ).......................
from the traditional form occurred. Unfortunately, the pressures of the
(9. COMM ERCE) ................. stage meant that rehearsed routines replaced the
(10. S PONT AN EO US).................... of the original flamenco performances.
III. ERROR IDENTIFICATION
The passage below contains 10 errors. Underline and correct them. Write
your answers in the space provided on the right. (0) has been done for you
as an example.
1 Whirlwind, any rotating air mass, include the tornado and the large
cyclonic and anti-cyclonic storm. In meteorology, the term whirlwind is
more strictly application to the smaller swirling atmospheric phenomenon
commonly known as dust devil or dust whirl, which occurs mostly over
5 deserts and semiarid plains during hot, calm days. The principle cause of
whirlwind is intense insulation, or incoming solar radiation received from
the earth, which produces an overheated air mass just above the ground.
This air mass arises, usually in the form of a cylindrical column, sucks up
loose surface material, just as dust, sand, and leaves. Whirlwinds vary with
10 height from 30 to 152 m, but exceptionally vigorous dust devils may
exceed 1,524 m in height. The vortices of whirlwinds range in size from a
few meters to several hundred meters and, depend on their force and size,
dust devils may disappear in seconds and last several hours. Brief
whirlwinds are erratic in motions, but the longer-lasting ones move
15 slowly with the prevailing winds.
1._______________ 2 ._______________
4 .__________ • 5 ._______________
7 ._______________ 8 ._______________
3 .__________ _
6 ._______________
9. _______________
10. ______________
IV. SENTENCE TRANSFORMATION
Finish each of the sentences in such a way that it means the same as the
given one.
1. Can I speak to someone about my problem?
—* Would i t .........................................................................................................
2. They declared war on the pretext o f defending their territorial rights.
—+The e xc us e........................................................................ ..............................
3. The play is so popular that the theater is likely to be full every night.
—» S uch....... .. .. .. .. .. .. .. .. .. .. .. .. .. .. .. .. .. .. .. .. .. .. .. .. .. .. .. .. .. .. .. .. .. .. .. .. .. .
4. They will not announce the decision formally.
—>No fo rm a l........................................................................................ I .............................
268
5. The boy was about to cry when he was reprimanded by his mother.
—> The hoy was o n ........................................................................................
Rewrite the following sentences using the given words. The given word must
not be altered in any way.
6. He maintained his position against his adversary. (GROUND)
7.1 sudd enly realized the meaning o f a “freebie” . (DAWNED)
....................... .......... *••••••••••••••................ .. ... ......... .................... ...........
8. Most stores will accept a credit card instead of cash. (ALTERNATIVE)
9. The bank robbers escaped in a stolen car. (GETAWAY)
10. Do n't panic about something so trivial. (MOUNTAIN)
TRƯỞNG THPT CHUYÊN BẢO LỘC - TP. BẢO LỘC - LÂM ĐỔNG
A. MULTIPLE CHOICE (40 PTS)
I. PH ONO LOG Y
Choose the word whose underlined part is pronounced differently from the
others.
1. A. apathy B. unanimous c. catalyst D. attorney
2. A. tarantula B. restitution c. congratulation D. Portugues
3. A. hierarchy B. machete c. moustache D. niche
4. A. immunity B. inundated c. munificent D. dubious
5. A. audit B.r auc ous c. draught D. manslaug
Choose the word which is stressed differently from the other three.
6. A. ubiquitous B. infuriating c. promotional D. hypocritical
7. A. meningitis B. anaesthetic c. congenital D. promiscuous
8. A. homicide B. internship c. incumbent D.s yco pha nt
9. A. tyrannical B. subservient c. obstentably . D. taxidermy
10. A. zealot B. hermit c. morbid D. concoct
II. WORD
CHOICE (5 pts):
Choose the best
options to com plete
the following
sentences.
into other's people
1. You sho uld n't lives.
private
A. prowl B. prod c. proceed D. pry
2. he will be sued fo r________ of contract if he does not do what he promised.
A. fracture B. crack c. rupture D. breach
3. S an dr a's__________ ;_______ sin is pride.
A. re deem ing B. overriding c. besetting D. overwhelming
269
4. In spite of working their fingers to th e _______________ , all the staf f were
made redundant.
A. nail B. edge c. flesh D. bone
5. It was h er___________________ powers at the keyboard rather than her age
which eventually forced her retirement.
A. meteoric B. waning c. slippery D. negative
6. He has be en _____________ for gross misconduct.
A. impressed B. im pou nded' c. impeached D. impelled
7. It seems õur application has been refuse d__________________ _.
A. point blank B. bull's eye c. carte blanche D. about face
8 .1 don't believe ther e's a _______of evidence that could be held against him.
A. shred B. grain c. drop D. strain
9 .1 find the prose style of many American writers virtua lly________________ .
A. illegible B. impenetrable c. impermeable D. incorrigible
10 .1 thought the bill for just two people was sim ply ___________________.
A. exhaustive B. exorbitant c. execrable D. exalted
11. Mozart's mature compositions ar e____________ better than his juvenilia.
A. significantly B. utterly c. flatly D. strenuously
12. Mr Parris said he'd l ik e___________________ by Monday, if tha t's possible,
A. finished the report B. the report finished
c. the report will be finished D. have the report finished
13. Were_________________ my dad, I would never have started playing tennis
in the first place.
A. it for B. it to be for c. it had n't been for D. it not for
14. It's amazing how Jenny acts as though she and Darren _____________
serious problems at the moment.
A. aren't having B. hadn't had
c. weren't having D. hadn’t been having
15. The rents in this area are _____________________ the highest in the city.
A. far from away B. away by far c. far and away D. far to away
16. _____________________ I’d like to help you out, I’m afraid I j ust hav en't
got any spare money at the moment.
A. Even B. Despite ' c. Much as D. Try as
17. This government would destroy all our civil lib ertie s,______ __________
half a chance.
A. given B. being given c. giving D. having been given
18. So_____________ that they actually finished three weeks early.
A. were the builders fast B. fast the builders were
c. the builders were fast D. fast were the builders
19. I feel _________________ to inform the committee that a number of
members are very unhappy with the decision.
A. my duty B. this my duty c. it my duty D. that my duty
270
20. I 'm __________________________my brother is.
A. nowhere like so ambitious B. nothing as ambitious than
c. nothing near as ambitious as D. nowhere near as ambitious as
III. PHRASAL VERBS AND PREPOSITIONS. (5 pts)
1. You can b an k________ our support any time you are in need.
A. at B. in c. on D. from
2. Her method of fishing for comments will certainly b e ________siege.
A. at B. for c. under D. with
3. After 10 years in business, the Browns hav e________ 2 million dollars.
A. called up B. collected up c. pulled up D. racked up
4. His classmates are always________ him for his oversized nose.
A. lazing around B. picking on c. running down D. shrinking from
5. It seems a lot o f US are putting ourselves_____the mercy of material comfort.
A. at B. in c. on D. under
6. It wasn't clear to me what they we re________by those sentences.
A. driving at B. getting on c. stabbing at D. opting out
7. That your son keeps himself aloof________ people is quite worrying.
A. from B. for c. of D. to
8. What I hate about Alice is that she’s always trying to ____teachers for favors.
A. crying out for B. making up for c. pinning down to D. sucking up to
9. The Prime Minister's declaration that taxes will be raised has put the public
_______ an uproar.
A. at B. in c. to D. with
10 .1 personally never________ the idea that to be attractive you have to be thin.
A. dream up B. hold back c. buy into D. fall out
IV. GUIDED CLOZE READING (10 pts):
Choose the word/phrase that best fits each blank in the following texts.
TEXT 1
Parent-teen relationships are among the most important in a youth's life.
Whether a parent is (1)___________love, criticism, or old-fashioned rulemaking,
the structure and stability of a healthy parental relationship with teens can make a
huge impact (2 )____________ their stressful adolescent lives. A healthy parentteen
relationship may mean the difference between swift justice and months of
harassment. Mothers, fathers, and family communication ( 3 )___________ are all
important in unique ways.
The mother-teen relationship is one o f the most important relationships a.child
can have. Research has ( 4)____________ that a mother has great influence over
their child's decisions about sex, drugs, alcohol, body image, ... Fathers also'have
a special ( 5 ) ___________ in the lives of healthy teens. Fathers are the backbone
of American family life, and it’s often on them that the foundations for their
271
children's future lives they are (6 )____________ . Involved fathers are present in a
child’s life to give everything from advice to independence. For example, teens
with fathers who are active in their lives are more independent as adults, have
(7 )____________ self-esteem, are less likely to be (8 )_____________ or commit
suicide, and are often happier. For daughters, a "daddy's little girl" relationship is a
large pail of what (9 )__________ happy teenage girls. Dads build self-confidence
in young women and give them a special (1 0) ____________ of achievement that
can only come when given by an authoritative male figure.
1. A. donating B. offering c. providing D. supplying
2. A. in B. on c. und er D. to
3. A. in general B. in all c. in contrast D. in full
4. A. expressed B. hinted c. implied D. suggested
5. A. emphasis B. part c. place D. status
6. A. lay B. lied c. laid D. lain
7. A. longer B. deeper c. higher D. larger
8. A. depressed B. desperate c. hurt D. stressed
9. A. brings up B. makes up c. sets up D. turns up
10. A. emotion B. feeling c. impression D. sentiment
TEXT 2
Adult education is a practice in which adults engage in systematic and
(1)___________ self-educating activities in order to gain new (2)___________of
knowledge, skills, attitudes, or values. It can mean any form of learning adults
engage in beyond traditional schooling, (3)_______________ basic literacy to
personal (4)____________as a lifelong learner.
In particular, adult education reflects a specific philosophy about learning and
teaching based on the (5)____________ that adults can and want to learn, that
they are able and willing to take responsibility for that learning, and that the
learning itself should respond to their needs.
Driven by what one needs or wants to learn, the opportunities (6)_______ ,
and the manner in which one learns, adult learning is affected by demographics,
globalization and technology. The learning happens in m any ways and in many
(7)____________ jus t as all adults' lives differ.
Educating adults differs from educating children in several ways given, that
adults have (8)____________ knowledge and work experience which can add to
the learning experience. Most adult education is voluntary, therefore, the
participants are (9)____________ self-motivated, unless required to participate,
by an employer. The science and art o f helping adults learn, the practice of adult
education is referred to as andragogy, to distinguish it from the traditional
school-based education for children pedagogy. Unlike children, adults are seen
as more (10)____________ , rather than relying on others for help.
272
1. A. attain B. pertain c. retain D. sustain
2. A. forms B. means c. patterns D.s hap es
c. encompassing D.
3. A. decom posing B. decompressing
encroaching
4. A. contentment B. fulfilment c. recognition D. realization
5. A. assumption B. notion c. resumption D. speculation
6. A. at will B. in hand c. in store D. on offer
7. A. contexts B. manners c. occurrences D. situations
B. accommodated c.
8. A. accomplished D. accredited
accumulated
9. A. all in all B. by and large c. in principle D. on the whole
10. A. self-absorbed B. self-accessed c. self-directed D. self-possessed
V. READING COMPREHENSION
Read the texts below and choose the best answer to each question.
READING 1
A DANCER'S LOT
All across London, they emerge from underground stations and buses; bags
slung over their shoulders and taut stomachs beneath thick winter overcoats.
Nobody recognises them, as they head for freezing upstairs rooms in tatty
gymnasiums or slink into backstage theatre doors, even though they appear
regularly in sold-out musicals and favourite television shows. They earn precious
little, even those who perform live with famous singers, and have no real
prospects, doing what they're doing, despite having hustled and sweated
themselves to the 10 heights of one of Britain's most demanding professions. But
still they go, every morning, to their grim upstairs rooms in gyms and their
backstreet backstage doors, to dance.
Most have left behind worried parents in faraway towns and villages; made
repeated promises to look after themselves and is taken trains, in-their late
teenage years, for London. There's much to despise about the city, where talent
and a reptilian grade of resilience, although prerequisites, provide no guarantee
of success. Even auditions are becoming rare. Conscious of deadlines and
financial constraints, choreographers call in talent from the blessed pool of their
own chosen. If you aren't the right height, don’t have the right face, hair or
sartorial style, then don't expect a look in. Although choreographers occasionally
seek out the beautiful, they’re mostly instructed to hunt the bland: those least
likely to outshine the stars. And, as many dancers will tell you, it’s getting to the
point where mediocrity is acceptable; there’ll be someone over there out of
sync, someone over there who can't hold her arm still.
And if they get a part, increasingly dancers are turning up for jobs where the
choreographer just stands there and works them endlessly, fingers clicking:
‘Again, again, again’. As one dancer Melanie Grace says, 'You dance for the
love and the passion, and keep your mouth shut because you don't want to get a
reputation.' It’s not always easy though. You think the television shows provide
273
changing rooms? For dancers? Even the big budget ones have them disrobing in
a comer o f the canteen - and the pay's lousy. But you have to ignore it, keep your
head down. You're in London now. You’re one of many; one of nothing. The
sooner you accept that, the better you'll get on. Of the fleets of talented dancers
who try only a q uarter make it, the rest simply can't process the ruthlessness —
to dance in London is hard on the soul.
Yet most of the dancers have agents, who you might think would negotiate a
better fee or conditions for their dancers, but no. You'll never meet a dancer who
thinks their agent deserves their twenty percent cut o f the fee. Mostly you ’ll just
get a text or email notifying you of an audition and a single agent might have as
many as two hundred dancers on,their books. As Melanie says, ‘It's catch-22,
because you won’t hear about the auditions without one .’ Here’s the job, take it
or leave it, and if so you leave it, they 'll ju st hire someone straight out of college
and pay them even less.
Oh, the annual churn of the colleges. The dancers hear it constantly the sound
of the machine in the distance, its ceaselessly grinding gears that, with every
coming year; push out hundreds of new dancers, each one younger and hungrier
and less jaded than you. And with every release of fresh limbs into the stew o f the
city things get harder. The worst thing the kids can do is accept a job for no pay.
They do it all the time. One website has become notorious for television and popvideo
production companies scrounging for trained people to work for nothing but
exposure. And if the youngsters are fresh out o f dance school, despairing of their
blank cv and craving the love of those ranks of sparkle-eyed strangers, they’ll
leap at the chance. It’s the reason things are getting harder. How to describe the
London dance scene today? The word Melanie chooses is 'savage'.
1. In the first paragraph, the writer paints a picture o f dancers who are
A. careful not to be recognised by fans in the street..
B. deserving of the fame they have achieved.
c. unlikely to be making further advances in their careers.
D. hoping to find work on stage alongside established stars.
2. What do we learn about auditions in the second paragraph?
A. Increasingly higher standards are expected of dancers.
B. The best dancers do not necessarily get the jobs on offer.
c. It's difficult for dancers to find the time to attend very many.
D. Dancers with family connections in the business get invited to more.
3. The word “tatty” in the passage is closest in meaning to _______ ______ .
A. shabby B. modern c. abusive D. well- designed
4. What US implied about choreographers in the third paragraph?
A. They expect dancers to do as they are told.
B. They dislike it when dancers criticise each other,
c. They are intolerant of dancers who make mistakes.
D. They are sensitive to the pressures-that dancers are under.
274
5. What point is made about agents in the fourth paragraph?
A. Dancers are largely satisfied with their service.
B. Most dancers recognise that they are essential.
c. They tend to represent only the less experienced dancers.
D. They make every effort to get the best deal for dancers.
6. The word “mediocrity” in the passage is closest in meaning to __________ .
A. average B. excellence c. extravagance D. extremity
7. The writer uses the image of a machine in the last paragraph to underline
A. the attitude o f training institutions. B. the dubious activities of a website,
c. the constant supply o f new talent. D. the exploitation of young people.
8. The word “lousy” in the passage is closest in meaning to _______________.
A. wonderful B. awful c. terrific D. satisfying
9. In the text as a whole, the writer is suggesting that dancers in London
A. should demand much better pay and working conditions.
B. have to regard the experience as useful for the future,
c. should be rewarded for dedication and perseverance.
D. have to accept the realities o f a competitive industry.
10. What does the phrase “do it all the time” refer to:
A. accept a job for no pay B. hear the sound o f the annual churn
c. leave the job
D. push out hundreds of new dancers
READING 2
IMAGE AND THE CITY
In the city, we are barraged with images of the people we might become.
Identity is presented as plastic, a matter o f possessions and appearances; and a very
large proportion of the urban landscape is taken up by slogans, advertisements,
flatly photographed images of folk heroes - the man who turned into a
sophisticated dandy overnight by drinking a particular brand of drink, the girl
who transformed herself into a femme fatale with a squirt of cheap scent. The
tone of the wording of these advertisements is usually pert and facetious,
comically drowning in its own hyperbole. But the pictures are brutally exact:
they reproduce every detail of a style of life, down to the brand of cigarettelighter, the stone in
the ring, and the economic row o f books on the shelf.
Yet, if one studies a line of ads across from where one is sitting on a tube
train, these images radically conflict with each other. Swap the details about
between the pictures, and they are instantly made illegible. If the characters they
represent really are heroes, then they clearly have no individual claim to speak
for society as a whole. The clean-cut and the shaggy, rakes, innocents, brutes,
home-lovers, adventurers, clowns all compete for our attention and invite
emulation. As a gallery, they do provide a glossy mirror of the aspirations of a
representative city crowd; but it is exceedingly hard to discern a single dominant
style, an image of how most people would like to see themselves.
275
Even in the business of the mass-production of images of identity, this shift
from the general to the diverse and particular is quite recent. Consider another
line of stills: the back-lit, soft-focus portraits of the first and second generations
of great movie stars. There is a degree of romantic unparticularity in the face of
each one, as if the y were communal dream-projections of society at large. Only
in the specialised genres of westerns, farces and gangster movies were stars
allowed to have odd, knobbly cadaverous faces. The hero as loner belonged to
history or the underw orld: he spoke from the perimeter of society, reminding US
of its dangerous edges.
The stars o f the last decade have looked quite different. Soft-focus photography
has gone, to be replaced by a style which searches out warts and bumps, emphasises
the uniqueness not the generality of the face. Voices, too, are strenuously
idiosyncratic; whines, stammers and low rumbles are exploited as features of ‘star
quality'. Instead of romantic heroes and heroines, we have a brutalist, hard-edged
style in which isolation and egotism are assumed as natural social conditions.
In the movies, as in the city, the sense of stable hierarchy has become
increasingly exhausted; we no longer live in a world where we can all share the
same values, the same heroes, (ft is doubtful whether this world, so beloved of
nostalgia moralists, ever existed; but lip-service was paid to it, the pretence, at
least, was kept up.) The isolate and the eccentric push towards the centre of the
stage; their fashions and mannerisms are presented as having as good a claim to
the limelight and the future as those of anyone else. In the crowd on the
underground platform, one may observe a honeycomb of fully-worked-out
worlds, each private, exclusive, bearing little comparison with its nearest
neighbour. What is prized in one is despised in another. There are no clear rules
about how one is supposed to manage one’s body, dress, talk, or think. Though
there are elaborate protocols and etiquettes among particular cults and groups
within the city, they subscribe to no common standard.
For the new arrival, this disordered abundance is the city ’s most evident and
alarming quality. He feels as if he has parachuted into a funfair o f contradictory
imperatives. There are so many people he might become, and a suit of clothes, a
make of car, a brand of cigarettes, will go some way towards turning him into a
personage even before he has discovered who that personage is. Personal identity
has always been deeply rooted in property, but hitherto the relationship has been
a simple one - a question of buying what you could afford, and leaving your
wealth to announce your status. In the modern city, there are so many things to
buy such a quantity of different kinds of status, that the choice and its attendant
anxieties have created, a new pornography of taste.
The leisure pages of the Sunday newspapers, fashion magazines, TV plays,
popular novels, cookbooks, window displays all nag at the nerve o f our uncertainty
and snobbery. Should we like American cars, hard-rock hamburger joints,
Bauhaus chairs ...? Literature and art are promoted as personal accessories: the
paintings of Mondrian or the novels of Samuel Beckett ‘go’ with certain styles
276
like matc hing handbags. There is in the city a cre eping imper ialism of taste, in
which mo re and mo re comm oditie s are made over to being mere expressio ns of
per son al identity. The piece of furnitur e, the pair o f shoes, the boo k, the film, are
important not so much in the ms elv es but for wh at they comm unica te about the ir
ow ners; and ow nersh ip is stre tched to include what one likes or believes in as
well as wh at one can buy.
1. Wh at does the wr ite r say about a dvertiseme nts in th e first parag rap h?
A. Ce rtain kin ds are consid ere d more effective in cities than others.
B. Th e w ay in which some o f them are worded is cleverer than it might appear,
c. They often depic t p eop le that m ost othe r p eople wou ld not care to be like.
D. The pictu res in them accurately reflect the way th at some people really live.
2. The word “pe rt ” in th e first paragraph is cl ose st in m ean ing t o ____________ .
A. che erful B. ple asa nt c. serious D. arg um entative
3. The wr ite r says tha t if you look at a line of adv ertiseme nts on a tube train , it is
cle ar that
A. city dwellers have very diverse ideas about what image they would like to have.
B. som e ima ges in adv ertiseme nts have a general appeal tha t oth ers lack.
c. city d we llers are mo re influence d by ima ges on adv ertiseme nts th an oth er
peo ple are.
D. som e ima ges are inte nded to be representativ e of eve ryon e’s asp irations.
4. Wh at does the w riter imply abo ut portraits of old mov ie stars?
A. They tried to disguise the less attractive features of their subjects.
B. Most people did not think they were accurate representations of the stars
in the m.
c. They ma de peo ple feel tha t th eir ow n face s w ere rather un attr active.
D. They reflected an era in which people felt basically safe.
5. The w ord “the y ” in the third paragraph refers to:
A. the a spiration s B. the portraits of old m ovies stars
. c. gangste r mo vie s D. gen eration s
6. Wh at doe s the w riter sugges t a bout the stars o f the last d eca de?
A. Som e of them may be uncom fortable abo ut the w ay they com e across.
B. T hey m ake an effort to spe ak in a way that may not be ple asa nt on the ear.
c. They make peo ple wonder wh eth er they shou ld bec ome more selfish.
D. Mo st people acc ept that they are not typical of soc iety as a whole.
7. The w riter uses the crowd on an underground platform to exemplify his belie f that
A. no single attitude to life is m ore com mon than a nothe r in a city.
B. no one in a city has strict attitu des to wards the beh aviou r o f others,
c. vie ws o f what s oci ety w as like in the past are often inaccurate.
D. people in citie s would like to have more in common with each other.
8. T he w riter implies that new arriv als in a city m ay
A. change the ima ge they wish to have too freq uently.
B. underes tim ate the imp ortanc e of wea lth.
277
c . acquire a certain image without understanding what that involves.
D. decide that status is of little importance.
9. The word “commodities” in the last paragraph is closest in meaning to _____ .
A. products B. companies c . entrepreneurs D. luxuries
10. What point does the writer make about city dwellers in the final paragraph?
A. They are unsure as to why certain things are popular with others.
B. They are aware that judgments are made about them according to what
they buy.
c . They want to acquire more and more possessions.
D. They are keen to be the first to appreciate new styles.
B. WRITTEN TEST (70 pts)
I. OPEN CLOZE TEST (20 pts):
Read the texts below and think of ONE word which best fits each blank.
TE XT 1
Obesity is a medical condition in which excess body fat has accumulated to
the (1 )_____________ that it may have an adverse effect on health, leading to
reduced life expectancy or increased health problems. Body mass index (BMI), a
measurement which compares weight and height, defines people (2 )_________
overweight (pre-obese) when their BMI is between 25 kg/m2 and 30 kg/m2, and
obese when it is greater than 30 kg/m2.
Obesity' increases the (3) _____________ of various diseases, particularly
heart disease, breathing difficulties during sleep, certain types of cancer, and
osteoarthritis. Obesity is most commonly caused by a combination of excessive
dietary calories, (4 )___________ of physical activity, and genetic susceptibility,
although a few cases are caused primarily by genes, endocrine disorders,
medications or psychiatric illness. Evidence to support the view that some obese
people eat little (5 )__________ gain weight due to a slow metabolism is limited;
(6)_________ average obese people have a greater energy expenditure than their
thin counterparts due to the energy required to maintain an increased body mass.
The primary treatment for obesity is dieting and physical exercise. To
supplement this, or in case of failure, anti-obesity drugs may be (7 )________ to
reduce appetite or inhibit fat absorption. In severe cases, surgery is performed or
an intragastric balloon is placed to (8)_________ stomach volume and/or bowel
length, leading to earlier satiation and reduced ability to absorb nutrients from
food.
Obesity is a leading preventable cause of death worldwide, with increasing
prevalence in adults and children, and authorities (9 )_____________ it as one o f
the most serious public health problems of the 21st century. Obesity is
stigmatized in much of the modern world (particularly in the Western world),
(10) _____________ it was widely perceived as a symbol of wealth and fertility
at other times in history, and still is in some parts of the world.
278
TEXT 2
In consecutive interpreting (CI), the speaker is required to pause to allow
interpretation. (1)____________ , the time needed is much greater (possibly
double the time needed). Traditionally, the interpreter will sit or stand near the
speaker. Consecutive interpretation can be (2) ____________ in a pattern of
short or long segments according to the interpreter's preference. In short CI, the
interpreter relies mostly on memory (3) ____________ , in long CI most
interpreters will rely on note-taking. The notes must be clear and legible in order
to not waste time on reading them. Consecutive interpreting of whole thoughts,
(4 )____________ than in small pieces, is desirable so that the interpreter has the
whole meaning before rendering it in the target language. This (5 )__________ _ a
truer, more accurate, and more accessible interpretation than where short CI or
simultaneous interpretation is used.
A consensus about lengths of segments may be (6 )____________ prior to
commencement, depending upon complexity of the subject matter and purpose
of the interpretation, though speakers generally face difficulty adjusting to
unnatural speech patterns.
(7 )_______ occasion, document sight translation is required of the interpreter
during consecutive interpretation work. Sight translation (8) __________
interpretation and translation; the interpreter must render the source-language
document to the target-language as i f it were written in the target language. Sight
translation occurs usually, (9) ____________ not exclusively, in judicial and
medical work.
Consecutive interpretation may be the chosen mode when bilingual listeners
are present who wish to hear both the original and interpreted speech or where,
as in a court setting, a (10)____________ must be kept o f both.
II. WORD FORMATION
PART 1: Complete each sentence, using the correct form of the word in
parentheses.
1. It’s absolute ly__________________ why the council decided to close down
the youth club. (EXPLAIN)
2. I’m scanning my gran’s photos onto my computer because they’d be________
if anything happened to them. (PLACE)
3. Astronomers at t he_____________ have announced that they have discovered
a new comet. (OB SERV E)
4. The man was later charged with .____________ a police officer. (PERSON)
5. If you can respond calmly an d______________________ _, then I think that
would be the best way forward. (AGGRESSIVE)
6. Em looking for a flatmate so Em going to put an ad in th e_______ . (CLASS)
7. The positive relationship between a business and a customer, often referred as
‘ ____________ ’, is difficult to quantify financially. (GOOD)
279
8. After len matches, we had th e____________________ record of played ten,
won none, lost ten. (ENVY)
9. Don't you think it's a bit early to ______________ blame? (PORTION)
10. The rumours are com pletely _________________ and 1 would urge everyone
not to believe them. (SUBSTANCE)
PART 2: Complete the passage with the appropriate forms from the words
given in the box.
argue be direct heart
intelligence marriage reach reduction
Historically, most advocates o f correspondence theories have been ontological
realists; that is, they believe that there is a world external to the minds of all
humans. This is in contrast to metaphysical (1)_________ who hold that
everything that exists is, in the end, ju st an idea in some mind. However, it is not
strictly necessary that a correspondence theory be (2)________________ to
ontological realism. It is possible to hold, for example, that the facts o f the world
determine which statements are true and to also hold that the world (and its facts)
is but a collection of ideas in the mind of some supreme (3)__________ ;______ .
One attack on the theory claims that the correspondence theory succeeds in its
appeal to the real world only in so far as the real world is (4)__________by US. The
direct realist believes that we directly know objects as they are. Such a person can
(5)_______adopt a correspondence theory of truth. The (6)______ theorist believes
that there are no real objects. The correspondence theory appeals to imaginary
undefined entities, so it is incoherent. The skeptic believes that we have no
knowledge. The correspondence theory is simply false.
Other positions hold th at we have some type of awareness, perception, etc. of
real-world objects which in some way falls short of direct knowledge of them.
But such an (7)________________ awareness or perception is itself an idea in
one's mind, so that the correspondence theory of truth (8) _____________ to
a correspondence between ideas about truth and ideas of the world, whereupon it
becom es a coherence theory o f truth.
If no theory of the world is offered, the (9) ______________ is so vague as
to be useless or even (10)________________ : truth would then be supposed to
be correspondence to some undefined, unknown or ineffable world. It is difficult
to see how a candidate truth could be more certain than the world we are to judge
its degr ee of correspondence against.
III. ERROR CORRECTION (10 pts):
The following passage contains 10 errors. Identify and cor rect them.
1 Brain drain, which is the action of having highly skilled and educated
people leaving their country to work abroad, have become one of the
developing countries concern. Brain drain also refers as human capital
280
flight. More and more Third World science and technology educated
5 people are heading for more prosperous countries seeking higher wages
and better working conditions. This, of course, has serious consequences
on the sending countries.
While many people believe that immigration is a personal choice that
must be understood and respected, others look at the phenomenon at a
10 different perspective. What makes those educated people leave their
countries should be seriously considered and a distinct between push and
pull factors must be made. The push factors include low wages and lack
of satisfactory working and living conditions. Social unrest, political
conflicts and wars may also be determined causes. wThe pull factors,
15 however, include intellectual freedom and substantial funds for research.
Brain drain has negative impact on the sending countries economic prospects
and competitiveness. They will reduce the number of dynamic and creative
people who can contribute to the development of their country. However,
with more entrepreneurs taking their investments abroad, developing
20 countries are missing an opportunity of wealthy creation. This has also
negative consequences on tax revenue and employment.
Most of the measures made so far have not had any success in alleviating
the effects o f brain drain. A more global sight must take into consideration
the provision of adequate working and living conditions in the sending
25 countries. Another option should involve encouraging the expatriates to
contribute their skill to the development of their countries without
necessarily physically relocating.
1.
4.
7.
2.
5.
8.
3._
6._
9. _
10.
IV. SENTENCE TRANSFORMATION (20 pts):
Rewrite the following sentences using the words given.
1 .1 d idn’t dare to tell the truth because my father was there. (GUTS)
—* Hadn’t it__________________________________________________ .
2. Looking back, we could have managed things in a more organized way. (SHIP)
-> W ith _______________________________________________ .
3. It was a shame I couldn't come to his fa ther's funeral. (RESPECT)
—* I'd lik e___________________________ ________________________ .
4. Despite the effort we made, our plan just wouldn 't come off. (PIE)
—> H ard______ _________________________________________ -
5. When it comes to his political view, his comments are sometimes inappropriate.
(PALE)
281
6. I hate it that I. always have to attract the boss’s criticism when things go
wrong. (FIRE)
—> I object t o _________________ ________________________________ .
7. It was bad the adults pretended to be unaware o f the kids' problems. (SAND)
—> The adults sh ou ldn't_________________ _______________________ .
8.1 haven’t been told exactly what I will have to do in my new job. (REQUIRED)
—* It hasn't b ee n___________________________ _____________ ______.
9. Andrew's determination to become a doctor despite his poor health should
receive fervent support from his father.
—> It is crucial that Andrew’s pa ren ts________ __ ___________________ .
10. A committed’ atheist, Stephen Hawking believes that the universe is only
governed by the laws o f science.
—> Stephen Hawking hold s____________________ due to h is ______
TRƯỜNG THPT CHUYÊN BẾN TRE - BẾN TRE
A. MULTIPLE CHOICE (4 0 PT S)
I. WORD CHOICE (5 PTS):
Choose the best options to complete the following sentences.
1. Mr. Minh is held in high ___________ for his dedication to the school.
A. respect B. esteem C. honor, D. homage
2. Marine construction technology like this is very complex, som ewhat _______
to trying to build a bridge under water.
A. analogous B. correlative C. synchronized D. incumbent
3. The car w heels __________ as they curved and bounced over the rough
broken ground.
A. squeaked B. squished C. screeched D. sputtered
4. He was disappointed by his result, but he is now ___________ to having to
re-take the exam.
A. composed B. submitted C. reconciled D. subdued
5. Adults exchange drinks around a pub bar in _______ of temporary friendship.
A. commemoration B. breach c. reference D. token
6. Fighting among rebel soldiers_________ last night and a curfew has now
been imposed on the city.
A. enhanced B. aggravated c. heightened D. intensified
7. Our journey w as ____ by the icy roads, which forced, us to drive very slowly.
A. obstructed B. impeded c. barred D. blocked
8. Many poets h av e_______ the beauties o f the countryside.
A. feared B.excited c.ex tolled D. inhibited
9. Mary is a( n) ______ _ liar. She was even arrested for lying to a police officer.
A. physical B. congenital c. naive D. abnormal
282
10. The Prime Minister will decide whether to release the prisoner or not; tha t’s
his
A.pre rogative B.derogatory c. abdication D. humanity
II. G RAM MAR AND STRUCTURES (5PTS):
Choose the best options to complete the following sentences.
11. He started hammering on the adjoining wall but Mary carried o n ________
A. notwithstanding B. regardless
c. irrespective D. nevertheless
12. ___________ hundreds of people dead during the evacuation from the wartorn area in the
Middle East.
A. It is estimated to be B. There is estimated to be
c. There are estimated to be D. It is estimated that
13. Jane was not_________ the landlady had imagined she was.
A. what
14.
B. that
the party began.
c. which D. at all
B. The music having been turned on
D. Having turned the music on
A. The music turning on
c. Turning on the music
15. Would you be________
A. kind enough in order to answer the phone for me
B. so kind as to answer the phone for me
c. too kind to answer the phone for me
D. that too kind and answer the phone for me
16. _______ , I could attend the conference now.
A. Were the flight to have taken off on time*
B. Were it not for the abrupt delay
c. Were the flight not have been postponed at last minutes
D. Were it to be a punctual flight
17. ___________ , the youngest mountain climber got lost with the rest o f his
team when his walkie-talkie malfunctioned.
A. As luck would have it B. By a stroke of luck
c. Be it lucky or not D. To put it down to luck
18. ' _______ , the meeting stops here.
A. I f no question being asked B. Without any question, however
c. No questions asked D. There being no question
19.Two weeks___________ , we will celebrate our coronation.
A. yet B. hence c. hitherto D. albeit
20. In your place, I ___________ to others for help. The re’s no way of getting
the project finished yourself.
A. would have turned
c. will turn
B. m ust have turned
D. would turn
283
III. PHRASAL VERBS AND PREPOSITIONS (5 PTS)
3 1. ‘‘Don't be a l aw __________ yourself!” - the mom shouted.
A. onto B. up after c. in for D. unto
32. Linda w as ________ herself with joy when her friends threw a surprise party
for her birthday.
A. beyond B. on and o ff c. beside D. at
33. Until this evening, Hannah is st ill __________ away at her unfinished report.
A. ham mering B. grinding c. whiling D. axing
34. After a four-day holiday, they began to work_________
A. on end B. out of bounds c. in proportions D. in earnest
35. The fact that a member holding authoritative power rides roughshod
_____ •
innocent residents exists in any society, even the democratic one.
A. on
B. over c. at
36. O ne's childhood heavily rubs ____________ one’s character traits.
A. off on B. together c. in with D. along
37. You don't _ __________ all this nonsense, do you?
A. cordon off B. buy into c. edge out D. hit on
38. After our busy day, we both sat an d____________ in front o f the TV.
A. nodded o ff B. slipped out c.flaked off D. knocked out
39. The story draws heavily__________ how strict the writer’s upbringing was.
A. toward B. in c.on D. into
40. Jacobin cuckoo is a _____________ of monsoon.
A. forerunner B. harbinger c.premonition D. herald
IV. COLLOCATIONS AND IDIOMS
31. This lovely new dress fist like a __________ .
■ A. treat B. gauntlet c. gown D. glove
32. The designer refuses to gild th e_________ , preferring clean, simple lines for
his creations.
A.lily B. flower c. rose D. daisy
33. Gina fina lly__________ and admitted that she had broken the vase.
A. threw in the towel B. made a clean breast o f it
c. read her the riot act D. let her hair down
34. There are no hills around and the area is as flat as a ____________ .
A. stick B coin c. feather D. pancake
35. It's time we changed t he___________ regarding the roles in the family.
A. idee fixe B. carte blanche c. hoi polloi D. social milieu
36. During pioneer days a lot of land in the United States was up fo r________ .
A.gain B. promotion c. taking D grabs
37.1 warned her not to say sensitive things
but she was like a bull in a(n) __ shop
A. India B. Nepal c. China D. Japan
38. The argument drove a ___________ between party members.
A. wedge B. linchpin c. needle D. sword
284
39. The government is making lit tle _________ in its fight to beat inflation.
A. advance B. headway c. forward D. forward
40. It must be true. I heard it s traigh t from th e _____ mouth.
A. dog's B. horse's c. came l's D. cat's
V. G UIDED CLOZE I (5PTS):
Read tile text below and decide which answer best fits each space.
INDUSTRY 4.0
Digitally connected manufacturing, often (4 1)____________ “Industry 4.0” ,
(4 2 )__________ a wide variety of technologies, ranging from 3D printing to
robotics, new materials and production systems.
A move towards Industry 4.0 would benefit the private sector. Large,
integrated manufacturers would find in it a way to (4 3) __________ and shorten
their supply chain, for example via flexible factories. A more digitalized
manufacturing would also open new market ( 44 )_________ for SMEs providing
such specialized technologies as sensors, robotics, 3D printing or machine-tomachine
communications.
For developed nations, Industry 4.0-a term initially (45) _ _________ in
Germany-could be a way to regain manufacturing competitiveness. This is
particularly relevant in the case of Western Europe, which, unlike the US, does
not currently enjoy reduced energy costs.
As for (4 6) _________ markets, Industry 4.0 could provide the much-needed
route to moving up the value chain, something that has become increasingly
important to achieve in the ( 47 )_______ _ of rising labor costs. For example,
Chin a’s new ten-year plan, issued last May and (4 8) ___________ named “Made
in China 2025”, targets key sectors such as robotics, information technology and
energy (4 9 )_______ turning the country from a “manufacturing giant” into a
“world manufacturing power.
As promising as it is, much more work remains to be done to make Industry
4.0 a large scale reality. On the regulatory side, for instance, policy makers will
have to ensure that data—the ($0) ___________ of Industry 4.0— can move
freely and securely throughout the supply chain, including across borders. This is
an effort that is likely to take some time.
41. A. deferred to as B. referred to as c. deferred to D. referred to
42. A. involves B. supports c. encompasses D. necessitat
43. A. optimize B. maximize c. customize D. legitimize
4 4 .A. chances B. possibilities c. opportunities D. places
45. A. yielded B. pieced c. lauded D. coined
46. A. novel B. emerging c. premature D. immature
47. A. loop B. teeth c. feat D. wake
48. A. aptly B. particularly c. unwittingly D. distinctive
49. A. in the interests of B. in lieu of c. in awe of D. in the hop
50. A. pins and needles B. nuts and bolts c. root and branch D. part and p
285
GUIDED CLOZE 2 (5PTS):
Read the text below and decid e w hic h an sw er b est fits eac h spa ce.
BITCOIN
The UK and other EU governments are planning a ( 51 )________ on bitcoin,
also known as a cryptocurrency that is not (52) _________ by banks, amid
growing concerns that the digital currency is being used for money laundering
and tax (53) _________ ?
The Treasury plans to regulate bitcoin and other cryptocurrencies to bring them
(5 4)_______ anti-money-laundering and counter-terrorism financial legislation.
Traders will be forced to reveal their identities, ending the (5 5)_________ that
has made the currency attractive for drug dealing and other illegal activities.
Under the EU-wide plan, online platforms where bitcoins are traded will be
required to carry out (56) __ _______ diligence on customers and report
suspicious transactions. The UK government is negotiating amendments to the
anti-money-laundering directive to ensure firms’ activities are (5 7)_________
by national authorities.
Economists have compared bitcoin’s meteoric rise with past ( 58 )_______ ,
such as the tulip mania of the 17th century and the dotcom that began in the late
90s with the Nasdaq index in New York and burst in 2000. Both examples
foreshadow a painful collapse for a currency that has no (59)_________ value to
those who hold it beyond that (60) _ _______ to it by a community of owners.
Should they realise the emperor has no clothes en masse, there could be a rude
awakening.
51. A. crackdown B. walk-out c. lowdown D. toss-up
52. A. underscored B. underwritten c. upholstered D. underpinn
53. A. invasion B. elusion c. indictment D. evasion
54. A. in line with B. in line for c. on line with D. on line fo
55. A. animosity B. anonymity c. pseudonym D. misnomer
56. A. preemptive B. tender c. prudish D. due
57. A. overlooked B. overseen c. overdriven D. overthrow
58. A. insinuations B. economies of scale c. bubbles D. junctures
59. A. intrinsic B. intricate c. civic D. immacula
60. A. preordained B. ascribed c. etched D. conferred
VI. READING COMPREHENSION: .
PASSAGE 1
If you think that we are the only creatures on Earth with a moral sense, then
you're in. good company. Most experts in behavior believe that morality is a
uniquely human trait, without which our complex social life would never have
emerged - yet I'm convinced that many animals can distinguish right from
wrong. Decades spent watching wild and captive animals have persuaded me
that species living in groups often have a sense of fair play built on moral codes
286
of conduct that help cement their social relationships. The notion of Nature being
naturally ruthlessly and selfishly competitive doesn't hold true for those of US
who have observed and analyzed animal relationships.
That's not all. I suspect that herein lies the origin of our own virtue. Biologists
have had real problems trying to explain why people are frequently inexplicably
nice to each other. It just doesn't make sense in evolutionary terms, unless there
are ulterior motives behind Olir seemingly altruistic actions. Perhaps we expect a
payback somewhere down the line, or maybe our good deeds are directed only
towards kin, with whom we share a biological heritage. Nobody has really
considered the possibility that being considerate to your neighbors might
sometimes be the best way to survive. But I'm starting to find evidence that a
well-developed sense of fair play helps non-human animals live longer, more
successful lives.
I'm particularly interested in social play amongst youngsters because it has its
own special rules of engagement, allowing participants to reinterpret acts that
might otherwise seem aggressive. My studies of infant dogs, wolves and coyotes
reveal that they use a special signal to prevent misinterpretation of playful
actions. They perform a 'bow' - which entails crouching on the forelimbs while
keeping the rear upright - when initiating play, or in association with aggressive
actions such as biting, to modify their meaning. And role reversal is common, so
that during play a dominant animal will often allow a subordinate to have the
upper hand. Such behaviors reduce inequalities in size, strength and dominance
between playmates, fostering the co-operation and reciprocity that are essential
for play to occur. Indeed, on the rare occasions when an animal says 'Let's play'
and then beats up an unsuspecting animal, the culprit usually finds itself
ostracized by its former playmates.
My belief is that a sense of fairness is common to many animals, because
there could be no social play without it, and without social play individual
animals and entire groups would be at a disadvantage. If I'm right, morality
evolved because it is adaptive. It helps many animals, including humans, to
survive and flourish in their particular social environment. This may sound like a
radical idea, particularly if you view morality as uniquely human and a sort of
mystical quality that sets US apart from other animals. But if you accept my
argument that play and fairness are inextricably linked, you're halfway there. •
[A] As with any behavioral trait, the underlying genetics is bound to be
complex, and environmental influences may be large. [B] No matter. Provided
there is variation in levels o f morality among individuals, and provided virtue is
rewarded by a greater number o f offspring, then any genes associated with good
behavior are bound to accumulate in subsequent generations. [C] And the
observation that play is rarely unfair or uncooperative is surely an indication that
natural selection acts to weed out those who don't play by the rules. [D]
287
What does this tell US about human morality? First, we didn't invent virtue -
its origins are much more ancient than our own. Secondly, we should stop seeing
ourselves as morally superior to other animals. True, our big brains endow US
with a highly sophisticated sense of what's right and wrong, but they also give US
much greater scope for manipulating others - to deceive and try to benefit from
immoral behavior. In that sense, animal morality might be 'purer' than our own.
We should accept our moral responsibility towards other animals, and that means
developing and enforcing more restrictive regulations governing animal use.
While animal minds may vary from one species to another, they are not so
different from our own, and only when we accept this can we truly be moral in
our relations with nature as a whole.
61. In the first paragraph, what does the writer state about morality?
A. Humans are the only creatures that demonstrate true emotional behavior.
B. A well-developed moral code does not lead to civilization.
c. Humans and animals share the same selfish instincts for survival.
D. There is a comm on misconception that animals are not moral.
62. What point does the writer make in the second paragraph?
A. People who are generous to others are not always sure why they behave
that way.
B. People who do not possess good social skills achieve less in life.
c. People who behave considerately to others have selfish reasons for doing so.
D. People who treat acquaintances better than relatives are unusual.
63. Neighbors are mentioned as an example to ___________ .
A. illustrate the fact that another aspect of the driving force may have been
overlooked
B. reiterate the notion that being nice to others is not hereditary
c. strengthen his belief that longevity does come into play concerning how
kind we are
D. report that many folks are becoming increasingly disillusioned with their
proclivity to helping others
64. The word altruistic in paragraph two most likely m eans___________ .
A. strange B. democratic c. selfless D. immaterial
65. What has the writer deduced about social play from his observation of animals?
A. It provides an opportunity for physically weaker animals to develop
survival skills.
B. It allows animals to prove who is dominant in the group without using real
aggression.
c. It requires animals to abide by the rules or they will be excluded from the
group.
D. It demonstrates that certain animals possess a large range of emotions.
288
66. Which of the following best summarizes the writers argum ent in the fourth
paragraph?
A. There are different degrees o f morality between various cultures.
B. Hum ans adopted moral behavior as a means of survival,
c. Groups benefit from social play more than individuals do.
D. Spirituality and morality are inseparable.
67. What does the w riter state about the evolution of morality?
A. There may be a particular gene responsible for morality.
B. Moral development depends on physical hardships.
c. There is little point in seeking the origin of moral behavior.
D. Animals that behave fairly are more likely to breed.
68. In the final paragraph, according to the writer, p eo pl e_____________ .
A. must treat animals on equal terms with humans.
B. should be less arrog ant in their view o f themselves.
c. are more advanced as they use immorality to their advantage.
D. should discriminate between which animals display morality and those that
do not.
69. The w riter concludes that in our relations with nature as a w ho le, ______ .
A. an open mind is of paramount importance
B. any exploitation should be dispensed with
c. bilateral responsibility is not always conducive to a long-standing relationship
D. virtue should be perceived with a new level of intricacy
70. “I am not putting the case forward for a specific gene for fair or moral
behavior” .
The sentence is extracted from somewhere in paragraph five. Where should
this sentence be located?
A. [A]
B. [B] c. [C] D. [D]
PASSAGE 2
THE MORECAMBE & WISE SHOW
It happened one night. It happened, to be precise, at 8:55 p.m on the night of
25 Decem ber 1977, when an estimated 28,835,000 people - more than half o f
the total population of the United Kingdom - turned their television sets to BBC
1 and spent the next hour and ten minutes in the company of a rather tall man
called Eric and a rather short man called Ernie. It was an extraordinary night for
British television. It was - at least as far as that catholic and capacious category
known as “light enterta inm enf’ was concerned - as close as British television
had ever come, in some forty - one years of trying, to being a genuine mass
medium . None of the usua l'rigid divisions and omissions were apparent in the
broad audience of that remarkable night; no stark class bias, no pronounced
gender imbalance, no obvious age asymmetry, no generalized demographic slant.
It was also, of course, an extraordinary night for the two stars of the show: Eric
Morecambe and Ernie Wise - by far the most illustrious, and the best - loved,
289
double - act that Britain has. ever produced. Exceptionally professional yet
endearingly personable, they were wonderful together as partners, as friends, as
almost a distinct entity: not “Morecambe and Wise” but “Morecam bewise”.
There was Eric and there was Ernie: one of them an idiot, the other a bigger
idiot, each of them half a star, together a whole star, forever hopeful of that
“brand new, bright tomorrow” that they sang about at the end of each show.
True, Eric would often slap Ernie smartly on the cheeks, but they clearly thought
the world of each other, and the world thought a great deal of them, too.
There show succeeded in attracting such a massive follow ing on that
memorable night because it had, over he course o f the previous night years or so,
established, and then enhanced, an enviable reputation for consistency,
inventiveness, unparalleled professional polish and, last but by no m eans least, a
strong and sincere respect for its audience. The Morecambe & Wise Show stood
for something greater, something far more precious, than mere first-rate but
evanescent entertainment; it had come to stand - just as persuasively and as
proudly as any earnest documentary or any epic drama - for excellence in
broadcasting, the result not just of two gifted performers (great talent, alas, does
not of itself guarantee that great television) but also of a richly proficient and
supremely committed production team.
The show, culminating in the record-breaking triumph of that 1977 special,
represented an achievement in high-quality popular program-making that is now
fast assuming the aura of a fairy tale - destined, one fears, to be passed on with
bemused fascination from one doubtful generation to its even more disbelieving
successor as the seemingly endless proliferation o f new channels and novel forms
of distraction continue to divide and disperse the old mass audience in the name of
that remorseless quest for “quality demographics" and “niche audiences”. The
Morecambe & Wise Show appeared at a time before home video, before home
satellite dishes and cable technology, before the dawning of digital revolution, a
time when it was. still considered desirable to make a television program that
might - jus t might - excite most o f the people most o f the time.
Neither Morecambe nor Wise ever looked down on, or up at, anyone (except,
of course, each other); both of them looked straight back at their audience on
level terms. No celebrated was ever allowed to challenge this comic democracy:
within the confines of the show, the rich and famous went unrecognized and
frequently unpaid (a running gag); venerable actors with grand theatrical
reputations were mocked routinely by Eric's sotto voce comments; and two
resolutely down-to-earth working-class comedians gleefully reaffirmed the
remarkably deep, warm and sure relationship that existed between themselves
and the British public.
“It was,” reminisced Ernie Wise, “a sort of great big office party for the
whole country, a bit of fun people could understand." From the first few seconds
of their opening comic routine to the final few notes and motions of their closing
290
son g and danc e, Mo rec am be and Wise did the ir very bes t to draw peo ple
toge ther rat her than drive them apart. Instead of pan dering subm iss ive ly to the
sm ug e xc lusiv ity o f the cog nosce nti (they were flatter ed by a w ell -re garded critic
prais ed the sly “o eil lad e” tha t acc om panie d Er ic's sar castic asides , but the y still
mo cked him me rciles sly for his use of th e word), and instead of set tlin g - as so
ma ny o f the ir sup posed suc ces sor s would do with unsee mly has te - f or the easy
security of a “c ult following”, Mo recambe and Wise alw ays aim ed to ent ertain
the w ho le nat ion .
Wh en vie we rs watch ed tha t show at the end of 1977, they wi tne sse d a rare
and rich com pendium of th e very bes t in popular culture: the hap py summ ation
o f a j oin t career tha t had traversed all of the key develop me nts ass ociated with
the rise o f mass enter tainm ent in Britain, encomp ass ing the faint but still
dis cernibl e traces of V icto rian mu sic-ha ll, the crowded ani ma tion of Edwardian
Va riety, the wo rdy popul ism of the wir eless, the spectac ula r impact of the
mo vie s and, finally , the more intimate per vas ivenes s of televi sion. Whe n tha t
career was all over, it wa s sorely m issed. They were sim ply irre placea ble.
71. Th e wr ite r imp lies in the first paragraph tha t one reason why the sho w on 25
De cemb er 1977 was e xtraordinary was th a t_____________
A. ligh t enter tainm ent prog ram s had been the target s of critici sm before th en.
B. no one had thought th at a British television program could appeal to all classes,
c. its au dien ce included people who might not have been expected to watc h it.
D. pe ople tuning into it knew that they were taking part in a phenomenal event.
72. The word cap aci ous in paragraph 1 is closest in m ean ing t o __________.
A. larg e B. spa cious c. roo my D. restric ted
73. In the sec ond paragraph , the writer imp lies th at Mo rec am be and W is e_____ .
A. would pro bab ly not have been successful had the y been solo p erform ers
B. had a d iffe ren t r ela tionsh ip in real life from the one they had on televi sion
c. we re keen for aud ien ces to r ealize how profession al they w ere
D. probably did not kn ow h ow popula r they were
74. Th e wr ite r says in the third paragraph tha t one reason w hy th e More camb e &
Wise Show r emained so popular wa s t h at_____________
A. It adapted to cha nge s in aud ien ce atti tud es to wh at constitu ted goo d
entertainm ent.
B. It appealed to peo ple w ho normally p referred oth er kinds of program ,
c. The peo ple w ho made it knew th at its p opula rity w as guara nte ed.
D. T he contribu tion of people oth er than its s tars w as a key e lem en t in it.
75. Th e wr ite r susjpects tha t The Morecambe & Wise Show will in the fut ure be
reg ard ed a s _________;_____ .
A. some thi ng w hich might only catch on with c ertain a udien ces
B. some thi ng w hic h has acquired an exa ggera ted reputation
c. the kind o f program that program - makers will asp ire to
291
D. the kind of program that illustrates the disadvantages of technological
advances
76. The word evanescent in paragraph 3 most likely mea ns __________ .
A. tran sient B. contemptible c. malevolent D. cynical
77. According to the writer, one feature of The Morecambe & Wise Show was
A. the way in which it reflected developments in British society
B. its inclusion of jok es that only certain people would understand
c. the consistent way in which other stars were treated on it
D. its careful choice o f other stars to appear on it
78. In the sixth paragraph, the writer implies t ha t_________ '
A. other comedians have attempted to appeal to only a particular group o f people.
B. Morecam be and Wise usually disregarded what critics said about them. •
c. other comedians have not accorded Morecambe and Wise the respect they
deserve.
D. Morecambe and Wise realized that there were some people who would
never like them.
79. In the last paragraph, the writer implies that one remarkable feature of the
show was t hat _____________
A. it exceeded even the expectations of its audience.
B. it contained elements that could have been regarded as old-fashioned,
c. it showed the similarities between earlier forms o f entertainment.
D. it contained a hint of sadness despite being so entertaining.
80. The word compendium in paragraph 7 is closest in meaning t o _______ .
A. performance B. perfection c. attempt D. combination
B. WRITTEN TEST
I. CLOZE TEST: Read the text below and think o f the word which best fits
each space. Use only ONE WORD for each space.
OPEN CLOZE 1 (10PTS)
Resembling real bank notes but not an officially (1 )__________ currency,
the custom of burning Hell Money is also believed to distract evil spirits
(2) ■________ __ that they chase the cash instead of the departed soul o f the dead.
Hell Money, also known as Ghost Money and Spirit Money, has been
discovered in archaeological (3 )_________ and its use can be (4) _______
back to the first millennium BCE—however, money was not in (5 )_________
form then, but instead made of stone, bones and later metal. The design of Hell
Money banknotes is intrinsically an art form. They are printed on jo ss paper in
vivid colors and elaborate designs, with the King o f Hell featuring on their front
side. The size can (6 )__ but in most cases they all have huge denominations.
(7 )_ _____ ___most banknotes display values in the millions and billions,
there are a few notes with values of tens or hundreds. The 100 dollar note is
particularly interesting with the King of Hell (8 )__________ four-fold because
the Chinese word for “four” is a homonym of the word “death”.
292
On most bills, there are phrases like "Hell Bank Note", while some are designed
in ( 9) _________a way that is similar to known, existing currencies, say, American
dollars or British- pounds. Some Hell Money banknotes contain signatures, serial
numbers and other elements that make them look like real money.
The Hungry Ghost Festival and Dongzhi Festival are two common occasions
that sees many Chinese commemorate their ancestors by offering ( 10 )________
including the burning of Hell Money.
OPEN CLOZE 2 (10PTS)
(1 )_______ along the way, Black Friday made the giant leap from congested
streets and crowded stores to fevered shoppers fist-fighting over parking spaces
and pepper-spraying each other as they tussle (2 )_____ _____the last Tickle Me
Elmo. When did Black Friday become the (3 )__________ frenzy, over-the-top
shopping event it is today?
That would be in the 2000s, when Black Friday was officially designated the
biggest shopping day of the year. Until then, that (4) __________ had gone to
the Saturday before Christmas. Yet as more and more retailers started touting
"can’t miss” post-Thanksgiving sales and the /Black Friday discounts grew deeper
and deeper, American consumers could no longer (5 )__________ the pull of
this magical shopping day.
Retailers may spend an (6) __________ year planning their Black Friday
sales. They use the day as an opportunity to offer rock-(7) '_______prices on
overstock inventory and to offer doorbusters and discounts on seasonal items,
such as holiday decorations and typical holiday gifts. Retailers also offer
significant discounts on big-ticket items and top-selling brands of TVs, small
devices and other electronics, luring customers in the hope that, (8 )__________
inside, they will purchase higher-margin goods. The contents of Black Friday
advertisements are often so ( 9)__________ anticipated that retailers go to great
(10)_________ to ensure that they don't leak out publically beforehand.
II. WORD FORMATION: (20PTS)
PART 1:
Complete each sentence, using the correct form of the word in parentheses.
1. ____________ value of quinine in treating malaria has been proved by science.
(THERAPY)
2. We all go out for a ______ meal after the two-week project. (CELEBRATE)
3. This out-of-the-way region in China suffers an annual ___________ _
outbreak o f cholera. (VIRUS)
4. The sprawling factory compound, all grey dormitories and _____________
warehouses, blends seamlessly into the outskirts of the Shenzhen
megalopolis. (WEATHER)
5. A ____________ species is a plant or animal that plays an indispensable role
in the way an ecosystem functions. (KEY)
293
6. To the technical philosophers, who strictly confine them selves to the logical
collation and criticism of scientific methods, he has, _____________ , not
seemed philosophic enough. (CONT RARY)
7. Insidious 4, albeit a horror movie, has so m e_____________ scenes that stem
from two auxiliaries, whose lines are particularly conducive to uproarious
laughter. (SIDE)
8. It was ju st_____ _______ that I m et my high school friend on my second day
visiting Paris. (HAPPEN)
9. ‘Thank God you ’re here’ has attracted excessive favorable publicity thanks to
the imaginative and unorthodox plays, tailored to whet th e _____________
appetite for originality from the audience. (SATISFY)
10. Beethoven, despite not winning any prize during his time, was awarded
. (POST)
PART 2: Complete the passage with the appropriate forms from the words
given in the box.
genius grip practice perc eive
document timid grass pass
The cinema, though, does have something in common with the confessional,
video blog aspect of Youtube. The popularity o f the horror film th e Blair Witch
Project was inflamed by a vast, (1 )____________ Internet campaigns which
( 2 ) ____________ suggested that the film’s horrors were real. Plus there’s a
cousin to this blurring of fact and fiction - confessional blogs which turn out to
be faked by (3) __ __________actors. In the past, some (4 )____________ that
you could see on TV or at the cinema had Youtube qualities, in that the footage
was shot by the participants themselves, although they needed a professional
cinem a ( 5 )___________ to bring it to light. If the unhappy heroes of these
films are making their videos now, they would probably ( 6)___________ these
directors and take them straight to Youtube.
Where straight cinema and Youtube come more closely into parallel is the use
of continuous shot: the persistent, unjudging, (7) _ __________gaze; an unedited,
deep-focus scene in which our attention as audience is not (8 )__________ or
directed. The true Youtube gems are not the digitally (9) _ ___________minifeatures. The
most (1 0)________ _ material is raw, unedited footage in one
continuous take.
III. ERROR CORRECTION: (10PTS)
The following passage contains 10 errors. Identify and corr ect them.
The sacred Tet, intrinsically
a traditional festival, occurs
some time
between late January and
Line
early February, depending
1
on the Lunar
Calendar. Though officially
a three-day affair, festivities
continue, more
294
5
10
15
20
25
likely than not, for a week or more with every effort made to indulge in
eating, drinking, and enjoyable social activities. It is also an opportunity
for family reunions, and for memorizing ancestors. Gifts of food are
made to friends, neighbors and relatives on the days before Tet.
Everyone is in a rush to get a haircut, buy new clothes, spring up their
homes, visit friends, settle standing debts, and stock up on traditional
Tet delicacies. Businesses hang festive red banners printed ‘Happy New
Year' and city streets are festooned with colored lights. Stalls commence
to sell candied fruits and jams, traditional cakes, and fresh fruit and
flowers. Certain markets sell nothing but cone-shape kumquat bushes.
Others sell flowering peach trees, symbols of life and good fortune
which people bring into their homes to celebrate the coming of spring.
As vendors pour into the City with peach trees strapped to their
bicycles, the streets look alike moving pink forests.
On the last day of the old year, the preparation of food to offer to the
ancestors is o f special significance. Dishes to offer the ancestors differ
in the Northern, Central and Southern parts of the country, depending
on their respective weather conditions at the time and on different fresh
products available locally. What is common in all regions o f the country
during Tet holidays are the varieties of soups, fried, boiled, or stewed
dishes ,... The foods that the Vietnamese eat at Tet are various and
diverse, but the people throughout the country all want to have the best
and the most beautiful looking food on this occasion to offer their
decease ancestors and to treat their friends and guests.
1. ________________
4 ._____ T_________
7 ._______________
2 ._______________
5 ._______________
8 .___________
3 . _ _____________
6 ._______________
9. _______________
10.
IV. SENTENCE TRANSFORMATION: (20 PTS)
Rewrite the following sentences using the words given.
1. Your story is different from the facts.
—> Your story d oesn’t tie in with the fa cts.
2. Martine cannot go any higher in his career. (PINNACLE)
-» M artin e...................................................................................................... —
3 .1 will appreciate it if you didn ’t pretend to support my view. (LIP)
—> I would so oner .......................................................... *........... ........................
4. Sorry, you can ’t do whatever you want.
—> Sorry, you are not in a ................................................................ and choose
5. You can eat as much as you like for $5 at the new lunch-bar. .
-» T her e..................................... ............................................ .............................
295
6. My father became extrem ely angry when he discovered someone had
damaged his car.
-> My fa th er............................. ............................................................................
7. Addicts of computer games struggle to distinguish the virtual world from the
real world. (DRAW)
—> Those obsessed ...............................................................................................
8. Have a look at this picture. It may help you remember something. (JOG)
—> Have a ............................................................................................................
9. Apparently, a lot of employees will be made redundant when the 21st Century
Fox is taken over. (HEAP)
-> Apparently, many a n ......................................................................................
10. Although the beginning may be smooth for a business, one has to think about
thè worst-case scenario. (PREOCCUPIED)
-> ....................... ............................................................... ........................... ......
TRƯÒNG THPT CHUYÊN BÌNH LONG - BÌNH PHƯỚC
A. MULTIPL E CHOICE (40PTS)
1. WO RD CHOICE (5PTS)
Choose the best alternative to fill the gap in each of the following sentences
1 .When asked if she had ever been in trouble with the police before, she replied
in th e________ .
A. aggressive B. assertive c. affirmative D. abrasive
2. It is hard to know whether to believe such _________ of doom, possibly
because what they are saying seems too terrible to be true.
A. champions B. warriors C. prophets D. giants
3. Furiously, she banged down the ________ and immediately dialed the
complaints department.
A. headset B. handle c. recipient D. receiver
4. New restaurants often try to attract customers by u sing ________ to homes in
the surrounding area.
A. postal adverts B. mailshots c. mail flyers D. mailboxing
5. The young lad was t he__________ of his father.
A. image B. likeness c. picture D. portrait
6. The salesman demonstrated how a push o f the button would cause the aerial to
A. rebound B. retreat c. recoil D. retract
7. Patrick is to o_____ _ a gambler to resist placing a bet on the final game.
A. instant B. spontaneous c. compulsive D. continuous
8. Several soldiers of the squad were taken ________ by the enemy forces.
A. capture B. hostage c. kidnap D. torture
9. The newspaper is well-known for being __ ______ against trade unions.
A. biased B. skewed c. twisted D. distorted
296
10. We have always tried to peace with our unfriendly neighbors.
A. gain B. reserve c. live D. preserve
11. GRAMM AR AND STRUCTURES (5PTS)
Choose the best alternative to fill the gap in each of the following sentences
1 .Peter: Why were those three men arrested?
Tom: They were suspected of being at Nairobi National Park.
A. poaching B. poached c. poacher D. poach
2. Not to waste time, searching, Alex kept everything ju st_______ he could get
his hand on it. <;
A. where B. as c. when D. if
3. You can no more swim than I can fly. The sentence means:__________ .
A. Both of us can’t swim or fly B. You can never swim, which I am sure of.
c. Either you or I can fly D. You can swim as I can fly
4. Her father insisted that she_seriously ill and that a doctor for at once.
A. was / was sent B. w as /b e sent c. b e/b e sent D. being / sending
5. What a terrible thing to happen! Just think, if we ha dn’t missed the plane, we
. dead for certain.
A. were B. would have been c. would be D. are
6. What surprised me most.was to find ________ little boys could make_______
much progress in ________ a short time.
A. such, so, so B.su ch ,so ,su ch c.so,so , such D.so, such,such
7. I won’t pay 80 dollars for the coat; it is not worth_______ .
A. all that much B. that all much c. much that all D. that much all
8. With profits at record levels, the company’s certainly sit tin g______ this year.
A. pretty good B. pretty well c. prettily D. pretty
9. Because we hadn’t seen each other for ages, we sa t______ for hours.
A. to have talked B. to have been talking
c. talking D. to be talked
10 Jo hn :_________ what he has said be true?
Mary: Yes, it______ _ be true.
A. May/may B. Can/can c. May/must D. Can/must
IILPREPOSITIONS AND PHRASAL VERBS (5 PTS)
Complete each of the following sentences with a suitable preposition or particle.
1. The front page was full of the reports given by the reporters________ the
scene of the road accident.
A. in B. on c. at D. of
2. The winds changed and it seemed our sailing team might _______ _out on top.
A. float B. come c. reach D. get
3. Let’s row over__ _____ the other side of the lake. .
A. on B. along c. to D. off
4. I f you feel bad about what she said, then it up with her - don’t let it go.
A. get B. sit c. put D. take
297
5. He ga ve ______ his seat on the bus to an elderly woman.
A. over B. in c. up D. back
6. The d oc tor_____ all night with the patients in the hospital.
A. sat out B. sat up c. sat in D. sat on
7. I was allowed to s it ______ on an executive meeting.
A. about B. in c. at D. up
8. His latest book deals with the social problem______ the day.
A. on B. in c. for D. of
9. Unfortunately, some really ill animals have to be________ by our center.
A. put down B. turned over c. passed away D. taken out
10. The new regulations h av e______ up a number of problems for the company.
A. come B. thrown c. got D. moved
IV. IDIOMS AND COLLOCATIONS. (5PTS)
Choose the best option to complete the following sentences.
1. Those stones have been here since ti m e_____ _.
A. immemorable B. immemorial c. immortal D. innumerable
2. The art teacher gave the children a fre e______ in their creative compositions.
A. offer B. hand c. gift D. kick
3. Once at the skating rink, Ivan was allowed to skate to his he ar t's ______ .
A. happiness B. contentment c. content D. delight
4. After making several bad business deals the company was losing money hand
ov er______ .
A. finger B. thumb C.wrist D. fist
5. The government is making little ______ in its fight to beat inflation.
A. headway B. improvement c. advance D. forward
6. We at Buyrite fling down th e ______ to competitors to match us for price,
quality and service.
A. mitten B. sword c. gauntlet D. hat
7. The drama grou p______ a sketch about a couple buying a new house.
A. created B. carved c. did D. painted
8. Has the show finally jump ed th e______ ?
A. salmon B. herring c. shark D. dolphin
9. I can't tell you______ the population of Prague, but there’s an encyclopedia
in the cupboard.
A. in hand B. at hand c. off-hand D. on hand
10. During pioneer days a lot of land in the United States was up f or______ .
A. gain B. taking c. promotion D. grabs
V. READING COMPREHENSION (10PTS)
1. READING COMPREHENSION 1 (5PTS)
Read the passage and choose the right answer for each question:
Musicals are a wonderful form of entertainment. While plays are theatrical
productions which tell a story through the written words of a script and are
presented by actors, musicals are stories told with lyrics and music. In New York
298
many of the theaters where the famous and well-known musicals are performed
are on or near Broadway and for that reason the musicals are often referred to as
“Broadway Musicals” . Some o f the lesser known productions, whether theatrical
or musical, are also performed in New York but these are usually performed in
other areas o f the city and are referred to as “Off-Broadway” productions.
Some of the same musicals being performed in New York are also being
performed throughout the world in major cities. And not all musicals are first
performed in New York. Sometimes shows open in Lon don's West End which is
home to numerous theaters where musicals, as well as plays, are performed with
great frequency and regularity to full-house audiences.
One o f the world’s most prolific and well-known songwriters is an Englishman,
Andrew Lloyd Webber. Webber grew up in a family whose interests centered on
music. Andrew’s father was a music professor and his mother a singer and musician,
and Andrew's talents seemed to take center stage when he was very young. He
began playing the violin when he was three years old and by the time he was nine he
was composing music for the small theater which had been built in his home.
One of Webber’s first musicals was a short production about a religious topic
and was written for a school’s end of the year concert. The production later
became a musical known to the world as Joseph and the Amazing Technicolor
Dreamcoat. Based on the biblical story o f Jacob and his son Joseph, the musical
tells the story of Jacob giving his son, Joseph, a beautiful multi-colored coat and
the resulting jealousy of Joseph’s brothers who have Joseph sold as a slave.
As the story continues in song, Joseph is jailed, explains his dreams to other
prisoners, and is later asked by the Egyptian pharaoh to help explain the pharaoh’s
dream. The dream, signifying an impending famine, is correctly interpreted by
Joseph and as a result Joseph is held in high esteem by the ruler. The musical
concludes happily when Joseph’s brothers, who need food from Joseph as a result of
the famine, are no longer jealous o f him and he forgives them.
Cats, which is based on an English author’s poems about cats, is a favorite of
nearly everyone. It tells the story of cats who are beautiful and of cats who are
old. Some of the cats are magical and others are mischievous. One cat reminisces
about her youth and the others also become person-like as their stories are told.
And the cats come in all shapes and sizes, too. Some are chunky while others are
lean. Some have seen better days and some are waiting for better days. To make
the musical even better, in some of the productions the cats-all human sizes and
shapes, of course - find their ways down the aisles to the stage to begin each act.
The lyrics in Cats follow T. s. Eliot’s poetry and as the songs are sung by cats
they are about cats. And one of the most interesting aspects of the production is
that the human-like personalities of the cats are readily apparent to the viewers.
The award-winning Evita was another Webber’s masterpiece. Evita is the
story of Eva Peron, and tells of her rise to power and the influence she had
among the people of Argentina. The play begins with her death. This musical,
299
unlike som e oth ers, begin s with the imme dia te - Evi ta 's dea th - and the n flashe s
back to the be ginn ing o f her life, sho wing her rise in im porta nce in her South
Am erican ho me lan d. Hi sto ric all y sig nif ica nt, Evita has brou gh t to the wor ld 's
attention the pli ght o f A rgen tin a and its people as well as presen tin g som e o f the
story o f th e dic tat orship of Juan Peron and the lasting im pact Ev a Peron had on
the people o f he r co untry . Th is mu sic al, wh ile ser iou s in subje ct ma tter, leaves
the audience with a greater un derst andin g of a country and its peopled The lyrics
and mu sic o f "D on ’t C ry for Me, Ar genti na," are hauntin gly u pli fting rath er than
sad and de mon str ate Evi ta’s conti nuall y encouraging att itu de to the stru ggl ing
poor in her h om ela nd.
Th roug ho ut the ye ars Web ber has wr itte n lyrics to a varie ty o f pro ductions.
The rock mu sic al, Jesus Ch ris t Super star, beg an its run in 1969 and has been a
fav orite o f to uring companie s since then. Sunse t Boule var d is set in Los An geles
and tell s the sto ry of a silent mo vie star w ho despera tely - a nts to r evive her fame
and repu tat ion with a come back in the talkies. And perhaps one o f the all-rim e
gre ats is the hig hly accla im ed pro ductio n, Pha ntom of th e Op era . The setting, in
the bo we ls o f a great opera house , is sig nifica nt to the story line bec aus e the
phantom o f the opera has est abl ish ed his dom ain the re. Lov e, lost love , and
in tr ig ue are all facets of the ph antom’s stor y and his role in ass isting a talent ed
yo un g wo ma n who asp ires to be one o f the op era’s gre ats.
An drew Lloyd Webber has made significant contr ibu tio ns to the arts and
especia lly to the mu sical the ate r through the nume rou s lyrics he has writte n for
mu sicals. His pro lifi c pro ductivity continues as he has, for th e past sev eral yea rs,
wr itte n a new mu sic al aboilt eve ry two years. And in 1992 he was recogniz ed
and knigh ted for his services to the arts.
1. Wh y does the autho r disc uss Jo sep h in th e passage?
A. A phara oh is the cen tral chara cte r and is a necessa ry inc lus ion in the
mu sic al to convey th e me ani ng o f the story.
B. Usual ly a bib lical story is n ot set to music as Lloyd We bber saw fit to do.
c. Th e plo t o f the mu sical involve d a fam ine wh ich had dire effect s on the
peo ple of the Mi ddle East.
D. Egypt is the setting for the musical and is important to the story line o f the musical.
2. Wh at is the main ide a about Sir An dre w Lloyd Web be r’s mu sic als pre sen ted
in the pas sag e?
A. He is an Au stralian me mb er of P arl iam ent who decid ed to write son gs in
his spa re time.
B. As a New Ze ala nder. Webber felt it important to prom ote the British
Co mm onwe alt h o f Nations.
c. As an Englishm an hon ored for his mu sical talents, the mu sic als pre sen t a
wide var iety o f settin gs, lyrics, and intere st areas for th eater attendee s.
D. He is o f Scottish descent and wante d to sho w his appreciat ion to the
Britis h roy alty for s howing p ositive response s to his wo rks.
300
3. Musicals are different from plays in that_________ .
A. musicals are stories produced with scripts whereas plays can be scripted or
unscripted.
B. while plays and musicals both tell stories, in a play the stories are not set to
music.
c. plays are noted for their seriousness while musicals only provide entertainment
to the attendees.
D. plays are only informative while musicals are never informative.
4. Webber, who is noted for the variety o f subject matter in his famous musicals,
has chosen to write music for existing literature in which of the following
musicals?
A. Evita B. Sunset Boulevard
c. Joseph and the Amazing Technicolor Dreamcoat D. Cats
5. What does the author mention about Webber's other musical talents other than
composing music?
A. His singing ability gave him opportunities to try new verse ideas on people
who were inclined to appreciate his work.
B. Playing the piano provided the necessary background for understanding
how the piano played an important part in musical, Sunset Boulevard,
which was the story of asilent movie star.
c. Playing the violin helped Webber understand the different musical
qualities an instrument could provide, which in turn helped him understand
the importance of variety in lyrics.
D. Dancing, an important part of all musicals, was better appreciated by
Webber when he began to write lyrics.
6 Which of the following is mentioned about a famine in one of Webber's
musicals in the passage?
A. People in England understood the famine in connection with the British
rule o f its Commonwealth of Nations.
B. The famine was central to the plight of the Egyptians during the time
period in which the musical was set.
c. The Middle East had never experienced a famine and the reader needed to
be aware of the ramifications of such an event.
D. Americans did not understand the significance of a famine other than the
potato famine in Ireland which resulted in the migration of many settlers to
America.
7. Evita is the beautifully told story of___________ .
A. Argentina's havoc-wrecked rural society during the 1930s
B. dictators and rulers in a power hungry nation during the 1940s
c. the leadership in Buenos Aires at the turn of the century
D. Eva Peron’s influence over the masses in a poverty-stricken country
301
8. In what aspect was the introduction of Jesus Christ Superstar to the stage and
to the world a monumental undertaking?
A. It was able to put a serious subject matter to a type of music which many
found offensive or unacceptable.
B. It became highly acclaimed and extremely popular with the younger adults
during the 1980s.
c. It has the controversial nature of the subject matter and the widely
accepted musical basis
D. It was first shown in Los Angeles even though most US musicals are first
shown on stage in New York City.
9. The word •plig ht’ in paragraph 3 is closest in meaning to________ .
A. serious epidemics B. difficult situation
c. natural disasters D. poor cooperation
10. The word ‘intrigu e’ in paragraph 8 is closest in meaning to________ .
A. security B. plot c. betrothal D. surety
2. READING COMPREHENSION 2 (5PTS)
Rea d the p ass age and c hoose the righ t answer for each que stio n:
In February 2001, at the age of 24, Ellen Mac Arthur became the youngest
and fastest ever woman to sail round the world. After 94 days alone on board her
yacht Kingfisher, she finished second to Michel Desjoyaux of France in the
single-handed Vende Globe event.
In sport, like life, the winner is usually feted, and runners-up quickly forgotten.
This time the roles were reversed and it was Ellen, weighing just 50 kilos and barely
lm60 tall, that really captured people's imaginations and emotions. One newspaper
in France, where she was and is a real heroine, summed up the national mood there
with the headline 'Well done, Michel, brave Ellen'.
As with many spectacular achievers, the signs were there from an early age,
even in the unpromising nautical terrain of landlock ed Derbyshire. Her greatgrandparents
were sailing people and a great-uncle was a merchant seaman, but
any real link with the sea is tenuous. There was, however, an Auntie Thea who
lived on the east coast of England and had a 26-foot sailing boat called Cabaret.
It took j ust one trip on the open sea with her aunt to spark off Ellen's lifelong
passion. She was eight years old. She began saving her pocket money and spent
all her spare time reading sailing books in the library, absorbing information like
a sponge. With her savings and the help of her grandmother she bought an 8-
£00t fibreglass dinghy, and from that moment on there was no keeping her away
from the water.
Sailing round Britain sin gle-ha nded at the age of 18 was just the start; Ellen
had long since set her sights on the Vende. But finding the money to undertake
round-the-world voyages is no easy feat. She wrote 2,000 letters requesting
sponsorship and received jus t two replies, from the Kingfisher company who
302
were looking to expand into France. And in terms of race preparation, if
thoroughness was the key to success, Ellen could certainly be considered one of
the favourites. In the eight months leading up to the start of the race, she sailed
no fewer than 60,000 miles at the helm o f her 6 0-£001 Kingfisher, far more than
the rest o f the fleet put together in the same period.
During her three months at sea MacArthur negotiated deadly icebergs,
gigantic waves and gale-force winds. She endured the freezing cold of the
Antarctic and suffered the blistering heat of the windless doldrums. Racing
conditions meant sleeping in 10-minute bursts, a survival suit that stayed on for
weeks at a time and hands and wrists covered in sores and cuts. Food was dried
or frozen. Water came from a desalinator, which passes sea water through a
membrane. 'You don't really wash in the icy waters of the southern ocean,' she
laughs. 'Anyway, there's no one to tell you that you smell.'
As K ingfisher crossed the finishing line Ellen was surrounded by hundreds of
spectator boats and a cheering crowd of 200,000 lined the shore. Stepping off her
yacht she looked remarkably composed and seemed to take the change from
solitude to public adulation very much in her stride. Her thoughts, she later
confessed, were on the realization that she had fulfilled the ambition that had
dominated her life for the previous four or five years. 'Throughout that time my
sole focus had been crossing the finishing line, and in the fastest possible time.'
Now she could savour that moment.
But despite MacArthur's belief that everyone who finishes the Vende is a
winner, she still feels a sense of disappointment that, having taken the lead from
the eventual winner Michel Desjoyaux 10 days from the finish, she did not quite
have the energy or good fortune to turn her advantage into victory. 'You have to
believe you can win from the start,' she asserts. 'Deep down you're a competitor,
you don't climb the mast and come back black and blue just for a cruise. You do
it because it's a race.'
The public will now be hoping to see a suitable encore, some new feat of
endurance to ju stify her celebrity status. For Ellen can no longer claim, as she
did in her post-race press conference, to be the simple Derbyshire girl with 'no
mobile, no credit cards, no money, no nothing'; she is a heroine and an
inspiration to others of her generation. As if to reinforce this, and despite her
reluctance to take on this role, she later commented: 'If there's one thing I've
learned in this past year, it's that deep down in your heart, if you have a dream,
then you can and must it happen.'
1. The word feted in the second paragraph me ans__________ .
A. to make somebody pleasant. B. to praise somebody.
c. to make somebody happy. D. to give somebody a reward.
2. At the time o f her achievement we learn that Ell en____________ .
A. enjoyed only short-lived success.
303
B. was more famous in France than anywhere else,
c. attracted more attention than Michel Desjoyaux.
D. became popular because of her size.
3. The word landlocked in the third paragraph me ans______________ .
A. having no seaport. B. having no fresh w at er .
c. having no land. D. having no sea coast.
4. Where did Ellen's initial interest in sailing come from?
A. She came from a family of sailing enthusiasts:
B. She went to see one o f her relatives,
c. She read widely on the subject.
D. She lived near the sea.
5. The word single-handed in the fourth paragraph means____________ .
A. without any help from anyone else.
B. using only one of one's hands to row.
c. on a boat with only one paddle.
D. on a boat with only one sail.
6. What do we learn about Ellen at the start o f the race?
A. People thought she had a very good chance o f winning.
B. She was a more experienced sailor than the other racers,
c. She had been waiting for this moment since she was 18.
D. She had gone to great lengths to achieve her ambition.
7. The writer suggests that one cause o f discomfort for Ellen at sea was_______ .
A. the shortage of water. B. her failure to sleep.
c. extremes o f temperature. D. a lack of cooking facilities.
8. According to the writer, when Ellen finished the race, she w as ___________ .
A. overwhelmed by her new-found fame.
B. surprised by the number of people who came to greet her.
c. able to reflect on her achievement.
D. delighted to be amongst people again.
9. According to the writer, Ell en_____________ .
A. thinks she deserved to win the race.
B. has mixed feelings about the outcome of the race,
c. knew she would win the race.
D. thinks Michel Desjoyaux was lucky to beat her.
10. Which of the following views does the writer express in the last paragraph?
A. She has the power to motivate. B. She has no right to fame yet.
c. Her comments lack depth. D. She needs to change her lifestyle.
VI. CLOZE TEST
1. CLOZE TEST 1 (5PTS)
Read the text below and decide which answer best Fits each space.
Lee Kuna Yew was appointed as the first prime minister of Singapore in 1959
when the nation became a self-governing state. At that time, Singapore per
304
(1 )______ income was only about USD 400. When Lee (2 )_________ down in
1990. the figure was USD 11,000 and in 2014 it went up to over 56,000, even
higher than that of its former (3 )_______ , Great Britain. In his (4 )________
published in 2000, he wrote ‘They laughed at US, but I was confident that we
would have the last laugh.'
Born into a middle-class Chinese family in Singapore, Lee was (5 )_______
by the language and institutions of the British rulers. He was (6 )_________ to
go to the UK to study law, but the Second World War broke out and he had to go
to the local Raffles College where he learnt some economics. It was not until
1946 when he talked his way to Cambridge and graduated three years later with
a starred first. It was during this time that he nurtured ambitions ( 7 ) ______
starting a legal career back home.
As a prime minister, he has always been described as an efficient and inventive
person. He was one of the greatest statesmen who proved to the whole world that
human (8 )______ , not natural resources, is the essential element of prosperity.
Lee is also believed to be very ( 9) _______ . When Singapore gained independence
in 1965, he decided to keep English as the main language to increase economic
benefits. Although many of his policies are still considered (1 0 )______ , they
helped Singapore to overcome many obstacles and become one of the most
admired international business and financial centers around the world.
l.A . personal B. capita c. capital D. national
2. A. sat B. settled c. stepped D. passed'
3. A. colonizer B. colonial c. colony D. colonizati
4. A. journals B. agenda c. autography D. memoirs
5. A. rooted B. affected c. influenced D. educated
6. A. scheduled B. ordered c. made D. pushed
7. A to B. beyond c. over D. of
8. A. ability B. sources c. intelligence D. ingenuity
9. A. open-minded B. innovative c. forward-looking D. imminent
10. A. skeptical B. disbelieving c. controversial D. contradict
2. CLOZE TEST 2 (5PTS)
Read the text below and decide which answer best fits each space.
Sunday May 4th will be World Laughter Day. Dr Madan Kataria, who introduced
this annual event, says we need more laughter in our lives to (1 )_________ the
global rise of stress and loneliness. But surely that strange sound that we make
periodically can ’t be the ( 2)_____ to such problems.
If an alien were to land on our planet and (3 )_____ a stroll among a crowd of
earthlings, it would hear a lot of ‘ha-ha’ noises. It might wonder what (4 )_____
this strange habit served. If we ask ourselves what (5 )_____ a good laugh, the
obvious answer is that it is a response to something funny. But one scientist,
Robert Provine, says humour has surprisingly little to do with that. Instead, it lies
at the (6 )_____ of such issues as the perception of self and the evolution of
language and social behaviour.
305
Provine realised that you cannot capture (7 )_____ laughter in the lab because
as soon as you (8 )_____ it under scrutiny, it vanishes. So, instead, he gathered
data by hanging around groups of people, noting when they laughed.
He collected 1,200 laugh episodes - and episode being defined as the
comment immediately preceding the laughter and the laughter itself. His analysis
of this data revealed some important facts about laughter. “It's a message we
send to other people - it (9 )___________ disappears when we’re by ourselves,’'
he says. “And it's not a choice. Ask someone to laugh and they 'll either try to
(1 0) _____ a laugh or say they can't do it on command.”
1. A. struggle B. combat c. threaten D. contest
2. A. way B. answer c. end D. response
3. A. make B. get c. walk D. take
4. A. reason B. purpose c. idea D. meaning
5. A. results B. leads c. prompts D. concludes
6. A. root B. stem c. head D. back
7. A. complete B. authentic c. contemporary D. current
8. A. place B. lay c. stand D. keep
9. A. absolutely B. constantly c. undoubtedly D. virtually
10. A. offer B. pretend c. fake D. imagine
B. WRIT TEN TE ST (70PTS)
I. OPEN CLOZE TEST (20PTS)
1. OPEN CLO ZE TEST 1 (10PTS)
Fill in each blank with ONE suitable word
Have you ever given any thought to the concept of the protection of our
natural resources and the significance it (1)_______ on our sound existence? It
may have been recognized by only a few of US what consequences our wasteful
life may lead to unless we undertake some proper measures to conserve our
natural habitats and their key (2 )_ _____ - wildlife, vegetation, soil and water
supplies. This question requires still more publicity, (3)_______ sure. (4)_______
ever do we realize how much effluent gets discharged into water or how many
tons of waste our populations can (5)______ out daily. In our hectic life, we
seldom think of the vast area of woodlands, including the rain forests, that get
(6)_______ every minute. We aren't usually conscious of the fact that the ozone
layer is being depleted due to the greenhouse effect. How much do we know
about the animal species being on the (7 )_______ of extinction? Lastly, who is
to (8) ______ for our abysmal ignorance? One possible response is the
incredible intensity of life that we are living almost all the time. Statistically, an
average couple has more to acquire in the 20th century than their ancestors did
several decades ago - education, the financial means for securing the family with
a flat or a house, a car, a sterero and other variety of rudimentary accessories that
306
the civilized world has to older and (9)________ which our earthly existence
seems unimaginable. Therefore, the answer is simple. It is ourselves that should
lace the charges of devastating the natural environment that we originate from,
but lor w hich we don't give much consideration (10)______principle.
2. OPEN CLOZE TEST 2 (10PTS)
Fill 111 each blank with ONE suitable word
How otten do you go along to a gig and see (1)_________ new? Well, Quest's
Friday night gig at the City Hall certainly caught my (2)_________ . Having
heard one or two tracks online, I was (3)_________ a group of about six
musician s. Imagine my surprise when just three young men walked on stage.
It was clear that the band already have a small but (4)_________ following. A
group of fans in front of the small stage were singing (5)_________ to at least
halt of the songs. And it was easy to see why. Quest have a clever combination
ot catchy (6)_________ , an irresistible beat, and very much their own sound. All
three of the band members play with great energy and expertise (7)_________
their age.
The only downside was when it came to the encores. They (8)_________ up
repeating some of their material and giving US cover (9)_________ of early rock
classics. A bit disappointing, but give them time and I'm sure they'll be writing a
lot more.
I'm sure we'll be hearing a lot more from Quest. Check them out every Friday
at the City Hall until the end of the month. It's well (10)_________ it.
II. WOR D FO RM AT IO N (20PTS)
1. WORD FORM 1
Complete each sentence, using the correct form of the word in parentheses.
(lO PTS)
1. Students carried out a ______ at the governor's office to protest higher tuition
costs. (SIT )
2. It was a very idea _________ completely impractical and unworkable. (FAR)
3. Sometimes the way we view life seems to be determined not by what really
happe ns to US, so much as by our perception of what happens, which is
sometimes called thinking. (FA CT)
4. Bill Gate is a totally millio nai re_________ he started his own business with
no financial help at all. (SE LF)
5. we often forget we are inextricably linked to nature, and by doing so,
___________contribute to its slow destruction. (ADV ER T)
6. The drinking w ater had a ______ oily taste. (AGR EE )
7. The students ranged from people who already had some expertise in the
kitchen, to totally (3 )___________ people like myself. (SK ILL )
8.It was going to take some deft p olitica l______ to save the situation. (W OR K)
307
9. What a terrible film! It's really_________ in my views. (RATE)
10.1 he reason for Bruce Lee 's death was sleeping pill_______ (DOSAGE)
2. WORD FORM 2
Complete the passage with appropriate forms o f the words given in the box.
(10PTS)
revere rebel single love
atop recognize demographic timid
Shigeru Miyamoto is the most successful artist of the last 50 years. He has
(1)______ laid the foundations for the world's largest entertainment industry.
Sales of his video games, from Super Mario Bros to Wii Sports, have (2)______
an incredible 500 million, and show no signs of slowing down. Miyamoto, who
join ed Nintendo in 1977, has created eight of the top-10-selling video games of
all time. His latest smash, Wii Fit, has sold 23 million copies, becoming the
second bigg est-selling game in history. It's made video games a daily part o f life
for a huge (3)______ of people who previously dismissed them as child's play.
Yet over and above his phenomen al sales success, what really sets Miyamoto
apart from anyone else in any creative field is his marrying of genius with
astonishing modesty. He cuts a(n) (4)______ , diminutive figure, sitting
obediently in his chair. A B eatles T-shirt and mop top haircut are the only signs
of cultural (5)______ , smoothed at the edges by a quiet (6)______ and
politeness. It's immediately apparent that unlike many of the Bafta Fellows to
whose club he now rightly belongs, impassioned (7)
and theatrical
outbursts are not his style.
Neither, as it turns out, are the glitz and glamour to which some may suggest
he is entitled. "I'm not envious of the attention of movie stars. I enjoy not
(8)______ ," he says. "It allows me to get on with my life. All I want is to be
recognized through my work. It's funny - in America and the UK, they say I'm
famous in Japan. In Japan, they say I'm famous in America and the UK."
Miyamoto's placid temperament and genial (9)______ form a perfect fit with the
univers ally (10)______ nature of his work. Miyamoto has never produced a title
that wasn't suitable for families to play together, even in the days when video
games were the unique preserve of teens in darkened bedrooms.
III. ERROR CORRECTION (10PTS)
Read the following text which contains 10 mistakes. Identify the mistakes
and write the corrections in the corr esponding numbered boxes.
I trotted along country toward the den, and I w as within half a mile o f
it where there was a thunderous roar behind me. It was so loud and i
unexpected that I involuntarily flang myself down on the moss. The
Norsem an came over at ab ou t fifty feet. As it roared past, the plane
308
5
I
' 10

15
i waggled its wings gaily in salution, then lifted to skim the crest of the
• wolf esker, sending a blast of sand down the slope with its propeller
ị wash. 1 picked it up and quieted my thumping heart, thinking black
thoughts about the humor in the rapidly vanishing aircraft.
The den ridge was, as 1 expected, wolfless. Reaching the entrance to the
ị burrow, I shed my heavy trousers, tunic and sweater, and taking a flashlight
i (whose batteries were very nearly dead) and measured tape from my pack, 1
ị began the difficult task of wigling down the entrance tunnel.
The flashlight was so dim (that) it cast only an orange glow - barely
I sufficient to enable me to read the mark on the measuring tape. 1
Ị squirmed on, descending at a 45-degree angle, for about eight feet. My
ị mouth and eyes were soon full of sand and I was beginning to suffer
: from claustrophobia, for the tunnel was just big enough to admit me.
1._______________ 2. ______________ t 3 .__________ __
4 ._______________ 5 . _ _____________ 6 ._______________
7 ._______________ 8 ._______________ 9._______________
10.______________
IV. SENTENCE TRANSFORMATION (20PTS)
1. Finish each of the following sentences in such a way that it is as similar as
possible in meaning to the sentence printed before it.
1. Managing the company will probably be much more complicated than they say.
-> Managing the company should n o t.................................... easy as they say.
2. Because my father went away for the weekend, his secretary was in charge of
the business.
—> H ad ...................... .............................................. store in terms of business.
3. The family members find it hard to accept their father’s fortune will go to
charity, (resistance)
-» There ...............................................................................................................
4. If she fails to accomplish her mission, her job will be put at risk, (line)
—» S hould...............................................................................................................
2. For questions 5-10, complete the second sentence so that it has a similar
meaning to the sentence, using the word given. Write only the missing
words on the separate answer sheet.
5. The day after I lost my job, 1 did not feel like getting out of bed. (inclination)
-> ...........................................................................................................................
6. It is necessary to free our business from the influence of the bureaucracy right
now. (hand)
—> It is necessary to fr ee...............................................................................
309
7. The train is too crowded with the commuters during the morning rush hour,
(sardines)
—> The commuters on the train were..............................................................
8. The club has been very successful since the beginning o f the season, (on)
-> ....................................................... .............................................................. .
9. The boys fixed all their attention on what the teacher was saying in order not
to miss any small details, (zeroed)
-> .............................................................................................................................
10. He causes so much trouble that we can do nothing besides from leaving him
to worry and suffer the unpleasant effect of his own actions, (juice)
-> ............................................................................................................................
TRƯỜNG THPT CHUYÊN HỪNG VƯƠNG ■ BỈNH DƯƠNG
A. MULTIPLE CHOICE QUESTIONS (40pts)
I. WOR D CHOICE (5ps)
Choose the best options to complete the following sentences.
1. Large UK-owned companies are .................. their European rivals when it
comes to creating wealth, a study has concluded.
A. outstripping B. catching up c. overruning D. exceeding
2. Donald Trump's pledge to rip up existing trade deals with Mexico would
..................... substantial damage on the US economy and kill the region's
competitiveness on the world stage, according to the Mexican economist who
led the country's trade talks with the US.
A. induce B. inflict C. impose D. wreak
3. Discontent among the ship's crew finally led to t h e ..........................
A. riot B. rebellion C. mutiny D. strike
4. The inconsiderate driver w as .............for parking his vehicle in the wrong place.
A. inflicted B. harassed c. condemned D. confined
5. Another, perhaps inspired by the nursery rhyme, is that a(n )................... . for
sweet things during pregnancy makes you more likely to be having a girl, while
a boy will make you crave savoury, salty foods, meat and cheese in particular.
A. inspiration B. affinity c. predilection D. propensity
6. The proposal w ou ld ........................... a storm of protest around the country
A. spark B. sparkle c. ignite D. trigger
7. She refused to answer questions th at...................... her private affairs
A. infringed on B. imprinted in c. impinged on D. indulged in
8. The book is such a .....................that I can not put it down
A. page-turner B. best-seller c. duvet-cover D. mind-reader
310
9. The rates of extreme poverty have halved in recent decades. But 1.2 billion
people still live________________
A. on the house B. on the tenterhooks
c. on the breadine D. on the cards
10. Obama expressed regret as a US drone strike has__________ killed innocent
hostages.
A. incongruously B. vehemently c. inadvertently D. graciously
II. STRUCTURE AND GRAMMAR (5pts)
Choose the best options to complete the following sentences.
11 .______________ , we missed our plane.
A. The train is late
c. To be late
12. "Mind your business"
A. What, again?
c. Thank you
13. My friend h as ______
A. a sharp ear
B. The train was late
D. The train being late
for a bargain.
B. I don't mind.
D. All right. I didn't mean to be noisy
B. a keen eye
c. a strong head D. a keen ear
14. _________ _ they to open fire without a warning.
A. On condition that B. Under no conditions that are
c. On no condition are D. On no conditions that are
15. His jean s and checked shirt,___________ , looked clean and of good quality.
A. though it old and well-worn
B. though old and well-worn
c. even though are they old and well-worn
D. although them old but well-worn
16.1 hope you w on’t take it______
A. offence B. amiss
17. Claims for compensation could
A. far B. much
18. Td prefer to go skiing this year
A. to going
c. rather than go
19.
if I suggest an alternative remedy,
c. upset D. most
_______run into billions o f pounds.
c. well D. most
______ on a beach holiday.
B. than going
D. rather than going
the public’s concern about the local environment, this new road
scheme will have to be abandoned.
A. as regards B. In view o f
c. In the event of D. However much
20. We are prepared to overlook the error on this occ asion__ your previous
good work.
A. with a view to
c. in the light of
B. thanks to
D. with regard to
311
IV. PREPOSITION AND PHRASAL VERBS (5pts)
Choose the best options to complete the following sentences.
21. My fingers are tired! I've been hammering aw ay __________ this keyboard
for hours.
• A. on B. at c. onto D. in
22. There tactics were_strongly influenced by reliance on line formation and firepower, as
against attack close quarters.
A. in B. on c. to. D. at
23. His jokes seemed to be go in g__________ very well with his audience, if
their laughter was any indication. •
A. of f B. along c. by D. down
24. The communist government has res ign ed__________ the background of the
widespread social unrest.
A. against B. from c. to D. as
25. The prisoner was recaptured as he made a dash__________ the gate.
A. at B. to c. for D. from
26 .1 __________ a small fortune when my uncle died but 1 managed to squander
most o f it. I'm ashamed to say.
A. came into B. came about c. came round to D. came down with
27. At the meeting some one__________ the idea that there should be a student
representative on the committee.
A. put forward B. put across c. put about D. put out
28. Do you have trouble obtaining your copy of Teacher's Weekly? Why not
__________ a regular subscription and be sure of receiving each edition as
soon as it's published?
A. Take over B. take out c. take on D. take to
29. He originally __________ to beat the land speed record, but weather
conditions were unfavourable.
A. Set to B. set in c. set up D. set out
30. The jo b itself is well paid and interesting, but commuting to the City every
day really __________ m e_________
A. g et s- do w n B. get s-ro un d c. gets -f or D. get s- fr om
III. COLLOCATIONS AND IDIOMS (5pts)
31. The waiter tried to be friendly to his customers but a ___________ can't
change its spots and he was still very rude.
A. leopard B. lion c. cheetah D. wo lf
32. Denise has be en ___________ the midnight oil trying to finish this report, so
she must be exhausted.
A. lighting B. brightening c. burning D. burnt
33. For some pec uliar___________ , they didn't tell US they were going away.
A. cause B. effect c.justification D. reason
312
34. Monsoon Wedding was described as a cinematic jewe l when it hit the
screen.
A. golden B. silver c. bronze D. diamond
35. How much to tell terminally ill patients is _____________ _ the discretion of
the doctor.
A. leaving to B. left to c. led to D. lead to
36. The British prime minister is too apt to cling to W ash ing ton 's_____ strings.
A. apron B. violin c. heart D. taut
37. After I was nearly expelled in my first year of college, 1 decided to stop
fooling around, put m y ____________ to the wheel, and get as much out of
my degree as I could.
A. shoulder B. arms c. shoulders D. leg
38. Trying to find her way home through the woods in the dead of night, Sarah
lost her _ __________when the clouds obscured the stars overhead.
A. bearings B. rag c. heart D. marbles
3 9 .1 was struggling with the reading before, but now I've d efinitely _________ .
A. get the wrong end o f the stick B. got hold of the wrong end o f the stick
c. got the wrong end o f the steel D. get hold of the wrong end o f the steel
40. Cubans who reach the u.s. are u sua lly_______________ asylum.
A. protested B. protected c. found D. granted
VI. GUIDED CLOZE TEST (lOpts)
Read the text below and decide which answer best fits each space
PASSAGE A
LONDON’S BLACK CABS
Black cabs, officially known as Hackney Carriages, are synonymous with
London and are special for a number of reasons. For a start, they are the only
taxis in the city that can be (41) from the kerb with a raised hand signal
to get the driver's attention. Currently, it is
estimated that there are 20,000 black
on the capital's streets. Their origin, in
cabs (4 2)______ ■
fact, can be
(43) ’ the name ‘Hackney Carriage’ said to derive from the French word
haquenée referring to the type of horse used to pull the carriages in the days of
horse-drawn carriages. The first horse-drawn Hackney coaches appeared on
London 's streets in the 17th century during the (4 4) _______ of Queen Elizabeth
I. As transport developed and motor cars were (45) _______ , motor cabs
replaced the horse-drawn carriages. Since the end o f the 19th century, various car
manufacturers ' vehicles have been used as motor cabs but it was not until the
mid-20lh century that the cabs we have been (4 6) _______ over the last decades
first appeared.
It is such a(n) realisation of becoming a black cab driver in London and it is
(47) _______ . If you want to gain this honour you will need to have passed the
(48) _______ test known as ‘the Knowledge’, which was first introduced in 1851
313
following hordes of com plaints by passengers whose cab drivers got lost. This
incredibly difficult test can take around three or four years to prepare for and you
can often catch a(n) (49) _______ of those drivers who are doing jus t this
zipping around London oh their mopeds, with a map fastened to a clipboard on
their handlebars. These people are essentially trying not only to master the
25,000 or so streets within a six-mile radius of Charing Cross, but also to work
out the most direct routes from place to place. They must know thousands of
‘points of interest' such as hotels, hospitals, places of worship, theatres, stations,
sports and leisure facilities, to name but a few. Practically everywhere and
anywhere that a potential passenger would wish to be taken to or from must be
known, so a(n) (5 0 )_______ , for a black cab driver, is beyond the pale and
perhaps this is the most difficult part, knowing the quickest way to get from one
place to another. Little wonder so few people are successful.
41. A. acclaimed B. acknowledged c. hailed D. summoned
42. A. hereabouts B. hither and thither c. or thereabouts D. there and then
43. A. ferreted out from
c. mapped out to
44. A. incumbency B. leadership
45. A. all the time B. of high standing
46. A. clued in on
c. in the know about
47. A. beyond you
48. A. abominable
49. A. gander
50. A. nodding acquaintance
c. profundity of London
B. no brainer
B. infamous
B. geek
B. hunted down from
D. traced back to
c. reign
c. of repute
B. genned up on
D. no stranger to
c. no mean feat
c. loathsome
c. glance
B. insight into London
D. smattering knowledge
D. sovereignty
D. in vogue
D. over your head
D. unscrupulous
D. glimpse
PASSAGE B
If you are an environmentalist, plastic is a word you tend to say with a sneer
or snarl. It has become a symbol of our wasteful, throw-away society. But there
seems little (51 )______ it is here to stay, and the truth is, of course, that plastics
have brought enormous (5 2)______ , even environmental ones. It's not really
the plastics themselves that are the environmental evil - it's the way society
chooses to use abuse them.
Almost all the 50 or so different kinds o f modern plastics are made from oil,
gas, or coal-non-renewable natural (5 3 )______ . We (5 4 )_______ well over
three million tons of the stuff in Britain each year and, sooner or later, most of it
is thrown away. A high (5 5)___________ of our annual consumption is in the
(5 6 )________ of packaging, and this (5 7)_________ about seven per cent by
weight, of our domestic (5 8)________ . Almost all of it could be recycled, but
very little o f it is, though the plastic recycling (59)_______ is growing fast.
314
The plastics themselves are extremely energy-rich. They have a higher caloric
value than coal and one method of “recovery" strongly favored by the plastic
manufacturers is the (6 0) _______ of waste plastic into fuel.
51. A. doubt B. evidence c. concern D. likelihood
52. A. pleasures B. benefits c.sa ving s D. profits
53. A. processes B. products c. fuels D. resources
54. A. remove B. consume c.ch an ge D. import
55. A. proportion B. portion c. amount D. rate
56. A. way B. form c. kind D. type
57. A. takes B. makes c. carries D. constitute
58. A.goods B. refuse ' c. rubble D. requireme
59. A. industry B. manufacture c. factory D. plant
60. A. melting B.change c. conversion D. replaceme
V. READING COMPREHENSION (lOpts)
Read the text below and,decide which answer best fits each space
PASSAGE A
THE CREATORS OF GRAMMAR
No student of a foreign language needs to be told that gramm ar is complex.
By changing word sequences and by adding a range of auxiliary verbs and
suffixes, we are able to communicate tiny variations in meaning. We can turn a
statement into a question, state whether an action has taken place or is soon to
take place, and preform many other word tricks to convey subtle differences in
meaning. Nor is this complexity inherent to the English language. All languages,
even those of so-called 'prim itive' tribes have clever grammatical components.
The Cherokee pronounce system, for example, can distinguish between 'you and
r , 'several other people and T and ‘you, another person and T. In English, all
these meanings are summed up in the one, crude pronounce ‘we’. Gramm ar is
universal and plays a part in every language, no matter how widespread it
is. So, the question which had baffled many linguists is - who created grammar?
At first, it would appear that this question is impossible to answer. To find out
how grammar is created, someone needs to be present at the time o f a language’s
creation, docum enting its emergence. Many historical linguists are able to trace
modern complex languages back to earlier languages, but in order to answer the
question of how complex languages are actually formed, the researcher needs to
observ e how languages are started from scratch. Amazingly, however, this is
possible.
Some of the most recent languages evolved due to Atlantic slave trade. At
that time, slaves from a number of different ethnicities were forced to work
together under colo nizer’s rule. Since they had no opportunity to learn each
oth er's languages, they developed a make-shift language called a pidgin.
Pidgins are strings of words copied from the language of the landowner. They
315
have little in the way of grammar, and in many cases, it is difficult for a listener
to deduce when an event happened, and who did what to whom. IAJ Speakers
need to use circumlocution in order to make their meaning understood. [B|
Interestingly, however, all it takes for a pidgin to become a complex language is
for a group of children to be exposed to it at the time when they learn their
mother tongue. |C] Slave children did not simply copy the strings of words
uttered by their elders, they adapted their words to create a new, expressive
language. |D| Complex grammar systems which emerge from pidgins are termed
creoles, and they are invented by children.
Further evidence of this can be seen in studying sign languages for the deaf.
Sign languages are not simply, a series of gestures; they utilize the same
grammatical machinery that is found in spoken languages. Moreover, there are
many different languages used worldwide. The creation of one such language
was documented quite recently in Nicaragua. Previously, all dea f people were
isolated from each other, but in 1979 a new government introduced schools for
the deaf. Although children were taught speech and lip reading in the classroom,
in the playgrounds they began to invent their own sign system, using the gestures
that they used at home. It was basically a pidgin. Each child used the signs
differently, and there was no consistent grammar. However, children who joined
the school later, when this inventive sign system was already around, developed
a quite different sign language. Although it was based on the signs of the older
children, the younger children's language was more fluid and compact, and it
utilized a large range of grammatical devices to clarify meaning. What is more,
all the children used the signs in the same way. A new creole was born.
Some linguists believe that many of the world's most established languages
were creoles at first. The English past tense -ed ending may have evolved from
the verb ‘do’. ‘It ended’ may once have been ‘It end-did’. Therefore, it would
appear that even the most widespread languages were partly created by children.
Children appear to have innate grammatical machinery in their brains, which
springs to life when they are first trying to make sense of the world around them.
Their minds can serve to create logical, complex structures, even when there is
no grammar present for them to copy.
61. In paragraph 1, why does the writer include information about the Cherokee
language?
A. To show how simple, traditional cultures can have complicated grammar
structures
B. To show how English grammar differs from Cherokee grammar
c. To prove that complex grammar structures were invented by the Cherokees.
D. To demonstrate how difficult it is to learn the Cherokee language
62. What can be inferred about the slaves’ pidgin language?
A. It contained complex grammar.
B. It was based on many different languages.
316
c. It was difficult to understand, even among slaves.
D. It was created by the land-owners.
63. All the following sentences about Nicaraguan sign language are tme EXCEPT:
A. The language has been created since 1979.
B. The language is based on speech and lip reading.
c. The language incorporates signs which children used at home.
D. The language was perfected by younger children.
64. In paragraph 3. where can the following sentence be placed?
It included standardized word orders and grammatical markers that
existed in neither the pidgin language, nor the language of the colonizers.
A [A] B.[B] c. [C] 8 D. [D]
65. ‘from scratch’ in paragraph 2 is closest in meaning to:
A. from the very beginning B. in simple cultures
c. by copying something else D. by using written information
66. ‘mak e-shift’ in paragraph 3 is closest in meaning to:
A. com plicated and expressive B. simple and tem porary
c. extensive and diverse D. private and personal
67. Which sentence is closest in meaning to the highlighted sentence?
Grammar is universal and plays a part in every language, no matter how
widespread it is.
A. All languages, whether they are spoken by a few people or a lot of people,
contain grammar.
B. Some languages include a lot of grammar, whereas other languages
contain a little.
c. Languages which contain a lot of gramm ar are more common that
languages that contain a little.
D. The grammar of all languages is the same, no m atter where the languages
evolved.
68. All of the following are features of the new Nicaraguan sign language EXCEPT:
A. All children used the same gestures to show meaning.
B. The meaning was clearer than the previous sign language,
c. The hand movements were smoother and smaller.
D. New gestures were created for everyday objects and activities.
69. Which idea is presented in the final paragraph?
A. English was probably once a creole.
B. The English past tense system is inaccurate.
c. Linguists have proven that English was created by children.
D. Children say English past tenses differently from adults.
70. Look at the word ’consistent' in paragraph 4. This word could best be
replaced by which o f the following?
A. natural B. predictable c. imaginable D. uniform
317
PASSAGE B
Fish that live on the sea bottom benefit by being flat and ỉtUỊỊỊỊÌHỊỊ the
contours. There are two very different types of flatfish and they have evolved in
very separate ways. The skates and rays, relatives o f the sharks, have become flat
in what might be called the obvious way. Their bodies have grown sideways to
form great “wings” . They look as though they have been flattened but have
remained symmetrical and “the right way up”. Conversely, fish such as plaice,
sole and halibut have become flat in a different way. There are bony fish which
have a marked tendency to be flattened in a vertical direction; they are much
“taller” than they are wide. They use their whole, vertically flattened bodies as
swimming surfaces, which undulate through the water as they move. Therefore,
when their ancestors migrated to the seabed, they lay on one side than on their
bellies. However, this raises the problem that one eye was always looking down
into the sand and was effectively useless. In evolution this problem was solved
by the lower eye “moving” around the other side. We see this process of moving
around enacted in the development of every young bony flatfish. It starts life
swimming near the surface, and is symmetrical vertically flattened, but then the
skull starts to grow in a strange asymmetrical twisted fashion, so that one eye,
for instance the left, moves over the top o f the head upwards, an old Picasso-like
vision. Incidentally, some species of 20 flatfish settle on the right side, others on
the left, and other on either side.
71. The passage is mainly concerned wi th_____.
A. symmetrical flatfish B. bony flatfish
c. evolution of flatfish D. different types o f flatfish
72. The phrase “hugging the contours” m ean s_____ .
A. swim ming close to the seabed
B. hiding in the sand at the bottom of the sea
c. floating just below the surface
D. lying still on the sea bottom
73. In lines 2- 5, the author mentions skates and rays as examples of fish that
A. become asymmetrical B. appear to fly
c. have spread horizontally D. resemble sharks
74. It can be inferred from the passage that horizontal symmetrical fish _____ .
A. have one eye each side of the head B. have one eye underneath the head
c. have two eyes on top of the head D. have eyes that move around the head
75. The word “conversely” is closest in meaning to _____ .
A. sim ilarly B. alternatively c. inversely D. contrarily
76. According to the passage, fish such as plaic e_____ .
A. have difficulties in swimming B. live near the surface
c. have poor eyesight D. have distorted heads
318
D. leap
77. '1 lie word “undulate” is closest in meaning to _____
A. sway B. flutter c. flap
78. The word “this" refers to _____ .
A. the migration of the ancestors
B. the practice o f lying on one side
c. the problem of the one eye looking downwards
D. the difficulty of the only one eye being useful
79. According to the passage, the ability of a bony'fla tfish to move its eye
around is _____ .
A. average B. weak c. excellent D. variable
80. It can be inferred from the passage that the early life of a flat fish i s_____ .
A. often confusing B. pretty normal
c. very difficult D. full of danger
B. WRITTEN TEST
I. CLOZE TEST: (20pts)
Read the texts below and think of the word which best fits each space. Use
only ONE WORD for each space.
PASSAGE A:
IS HONESTY THE BEST POLICY?
Radical honesty (1)................,as it is know n in the US, is the latest thing
to be held up as the key to happiness and success. It involves telling the truth
all the time, with no exceptions for hurt feelings. But this is not as easy as it may
sound. Altruistic lies, rather than the conniving, self-aggran dising (2).................
are an essential part of polite society.
We all lie like mad. It wears US out. It is the major source of all huma n
(3)................ ,'says Brad Blanton, psychotherapist and founder of the Centre for
Radical Honesty. He has become a (4)................ name in the US, where he
spreads his message via day-time television talk shows. He certainly has his
work (5) ................ out for him. In a recent survey of Americans, 93 per cen t
admited to lying regularly and (6)................ in the work place. Dr Blan ton is
typically blu nt a bout the consequences o f being deceitful. ‘Lying (7) ................
peo ple ,'he says.
Dr Blanton is adam ant that minor (8 )........... are nothing at all com pare d
with the huge benefits of tru th telling. ‘Telling the tru th, especially after
hiding it for a long time, takes guts. It isn 't easy. But it is better tha n the
(9)................. ’.This, he believes, is the stress of living in the prison of the m ind;
which results in depression and ill health. ‘Your body stays tied up in (10)
.............and is susc eptib le to illness; he says. ‘Allergies, high blood pressure
and insomnia are all made worse by lying. Good relationship skills, parenting
skills and managem ent skills are also depend ent on telling the tru th .’
319
PASSAGE B:
It is not uncommon for people returning to visit their parents to react in this
way. Although there are obviously real joys in ( I I ) ....... with one's loved ones,
the downside is that it can stir up (1 2) .............. memories. These are memories
that exist deep down and can (1 3)..... ............. without conscious awareness,
(14).................. triggering feelings we had in the past. Dr Daniel Siegel, author
of The Mindful Brain, says that ‘When W'e (15)................ an element of implicit
memory into awa rene ss,'w e just have the (1 6) ..................... response without
knowing that these are (1 7) ................ related to something we've experienced
before. This is distinct from an explicit memory - a (1 8) ................ experience
that exists in our minds, such as learning to ride a bike.' It is implicit memories
that make it possible to experience even a seemingly (19)................. visit home
while at the same time unconsciously taking on the (20)................ and attitudes
we had as children and reacting as we would have done then.
II. WORD FORMATION: (20ps)
PART 1: Complete each sentence, using the correct form of the word in parentheses.
1. I W'ould say that yo u'd b e ........... to resign without having found a new job.
(ADVICE)
2. Two people have been arrested for illegal possession o f ................ in a police
raid. (ARM)
3. The comedy Bringing Up Baby presents practically non-stop dialogue
delivered a t ................ speed. (NECK)
4. This use o f dialogue underscores th e ................of the film itself and thus its
humor. (ABSURD)
5. People are increasingly becoming........... to violence on television. (SENSE)
6. They were now faced with see mingl y........... technical problems (MOUNT)
7. He is currently standing trial for al lege d................(PRACTICE)
8. All points on a circle a re ................(DISTANT) from the centre.
9. The organization insists that it i s ................and does not identify with any
one particular party (POLITICS)
10. The new regulations will b e ................ for small businesses. (BURDEN)
PART 2: Complete the passage with appropriate forms from the words given
in the box
content resign leisure ship
tax hold ponder atlantic
At one time, for the (11)................of people needing to make the journey, the
only way to travel from Europe to America was by ship, but the days of a (12)
................five-day (13) ................. crossing in a large liner have long gone.
Today, everything is about speed, and long distance travel has become almost
exclusively the business of the airlines.
320
Although air travel is admittedly fast, passengers are still subject to the
unexpected (1 4 )................ which seem to be inherent in any form of travel, but
peop le's re actions to such delays seem to be far more (1 5 )................ than in the
days of the great liners. Then, passengers might have shrugged their shoulders
(16) .......... — and relaxed with a book in their cabins, knowing that little could
be done about the situation, but today’s traveller is more likely to make his
(17) ................ known forcibly to any official who is unfortunate enough to
appear on the horizon.
It is easy to see why this might be the case since airport lounges are ( 1 8 ).......
clearly inadequately equipped to cope with large numbers of (19) ................
passengers. Small wonder that people’s tempers begin to fray and their nerves
are (2 0 )................ when faced with the prospect o f a sleepless overnight stay in
what is, after all, no more than a vast public hall.
III. ERROR IDENTIFICATION (lOpts)
The following passage contains 10 errors. Identify and correct them
£ z>7<? Tornadoes are one of the
15 most severe types of
10 weather phenomenon.
15 While many people fear
20 tornadoes and their
destructive power, few
people understand their real
causes and effects, nor are
they aware of
how to protect themselves
from their devastating force.
Tornadoes, violently
rotating columns of air,
occur when a change in
wind direction, couple with
an increase in wind speed,
results from a
spinning effect in the lower
atmosphere. These whirling
movements,
which may not be visible to
the naked eye, are
exacerbated when the
rotated air column shifts
from a horizontal to a
vertical position. As the
revolving cloud draws in the
warm air that surrounds it at
ground level,
its spinning motion begins
to acelerate, thereby create a
funnel that
extends from the cloud
above it to the ground
below. In this way,
tornadoes become pendent
from low pressure storm
clouds.
When a tornado comes into
contact the ground, it
produces a strong
upward draft known as a
vortex, a spiraling column of
wind that can
reach speeds in excess of
200 miles per hour.
Traveling on the
landscape, the tornado
wreaks a path of
concentrated destruction. It
is
not common for these
twisters to lift heavy objects,
like cars or large
animals, and throw them
several miles. Houses that
succumb to the
force of the tornado seem to
explode if the low air
pressure inside the
vortex collides with the
normal air pressure inside
the buildings.
Í. ____________
4. _ _ _____________
7. _ ___________ __
2 ._______________
5 ._______________
8 ._______________
3 .___________ •
6 ._______________
9. _______________
10.
321
IV. SENTENCE T RA NSFO RM AT ION (20p ts)
1. Were Jack not so affluent a man, she would not be dating with him.
-» But.................................................................................
2. It is likely that they forgot about the extra class.
—> They...............................................................................
3. Do you have any scarves? I'm looking for one that's woolen, green and fairly long.
-> I'm looking for a..................................................... .......
4. You should not lock this door for any reason when the building is open to the public.
-> Under n o .......................................................................
5. A bee sting is more likely to cause death these days than a snake bite.
-» Death..............................................................................
6. If someth ing's worrying you, you should tell me about it now. CHEST
—> If you have a problem, why don’t you............................................... now?
7 .1 think you should confront your boss right now with how you feel about this.
OUT
-> Why d on’t you............................................................. your boss right now?
8. Anna had decided Tom was not telling the truth about what happened.
TEETH
-> Anna was sure T om .................................................. about what happened.
9. Perhaps my sister was bad-tempered because she was tired. FACT
—> I put my sister’s...................................... ........................she was tired.
10. When confronted with his crime the accused was unrepentant. REMORSE
—» The accu sed............................................................ crime he had committed.
TRƯỜNG THPT CHU YÊN HÙNG VƯƠNG - GIA LAI
A. MULTIPLE CHOICE (40 points)
I. WORD CHOICE (5 points)
Choose the word or phrase which best completes each sentences.
1. Mark Stringly is rarely interviewed as he hates being in t he______ .
A. flashlight B. limelight c. headlight D. floodlight
2. I tried to push my way through the standing passengers to get to the door and,
failing to keep m y____ about me.
A. head B. soul c. wits D. mind
3. We need a more_______ assessment of the chances o f ending this war.
A. moral B. sober c. plain D. vivid
4. The mobile phone company planned to publicly • its latest technology
to upgrade its already strong operating system.
A. underwrite B. undervalue c. undress D. unveil
5. She has drawn ______praise for her charitable works.
A. abundant B. luxurious c. massive D. lavish
322
6. When times are good, people can spend freely, but during bad times we have
to ____ our spending.
A. multiple B. disperse c. curtail D. obstinate
7. The vagrant came out of the sh op ,___________ for about 10 paces before
falling against a car.
A. stammered B. tripped c. staggered D. skidded
8. James did n't ta ke _____ to your suggestion that she was mean with money.
A. kindly B. pleasantly c. cheerfully D. agreeably
9. When his manager went on a business trip, Smith stepped into t h e_____ and
chaired the meeting.
A. hole B. pool c. breach D. crack
10. The manager spoke highly of such___________ as loyalty, courage and
truthfulness shown by his employees.
A. haracteristics B. properties c. features D. virtues
IL GRAMMAR AND STRUCTURES (5 points)
Choose the word which best completes each sentence.
1. I know you didn’t want to upset me but I’d sooner you______ the whole truth
yesterday.
A. could have told B. told c. have told D. had told
2. ______him in York during your holiday, please give him my number.
A. Were you to meet B. Had you not met
c. Having met D. Should you meet
3. The scientists______ the festival of Ramadan, but they were too busy with
their research in the laboratory.
A. would have liked to commemorate
B. would have loved to have observed
c. would prefer to have obeyed
D. would sooner have celebrated
4. 1 haven’t got the time to do my own w ork ,______ help you with yours.
A. not counting B. let alone
c. apart from D. leaving aside
5. A new generation of performers,______ those who by now had become a
household name, honed their skills before following the same path onto
television.
A. no less talented than B. along with talented as
c. together with talented as D. having been more talented than
6. I gave my last Mars bar to Jake, because he was feeling hu ng ry;______ , he
likes them far more than 1 can do.
A. besides B. whereas c. nonetheless D. hence
7. My mother always told me that I shouldn’t judge people______ . The way they
look says nothing about their character.
A. by their appearance B. with their outlook
c. through appearance D. by their face
323
8. Owning and living in a freestanding house is still a goal of young adults,
_________ earlier generations.
A. as did B. as it was o f
9. ______ , we can take another road.
A .I f need be
c. We m ight as well
10. They turned down the proposal___
A. by reason
c.as a cause
c. like that of D. so have
B. When it may
D. Come w hat must
that it didn 't fulfill their requirement.
B. on the grounds
D. allow ing
III. PREP OSITIO NS AND PHRASAL VERBS (5 points)
Choose the w ord which best completes each sentence.
1. The latest novel by Grant is hilarious. It had me______ stitches.
A. in B. at c. on D. with
2. The com puter has______ a long way over the last thirty years.
A. taken on B. come on c. broken down D. put in
3. Steve threw______ his chances of passing by spending too much time on the
first question.
A. on B. off
4. Did you notice Bob trying t o ______
A. get up to B. break out of c. get out of
5. This p esticide kills insects______ contact.
A. with
c. away D. in
doing the washing-up.
D. get along with
B. at c. for
6. Eva doesn 't care what she says. People are often_
comm ents.
A. set aside B. taken aback c. stood over D. taken o ff
7. Steve used to be easy to work with, but since his promotion he's begun
D. on
_b y her outspoken
to
A. throw his weight around
c. throw him o ff balance
8. The new regim determined to ___
A. stop o f B. end up
9. Terry's quite nice really. Don’t be_
A. checked off B. put o ff
10. The court scot-free.
B. throw in the towel
D. turn up trumps
compulsory military service,
c. phase out D. break off
_by his appearance,
c. set on D. taken over
A. let her off B. took her away c. set her up D. kept her out
IV. COLLOCATIONS AND IDIOMS (5 points)
Choose the word which best completes each sentence.
1. It takes a very determined person to achieve results as good as this; you really
have to put your______ to it.
A. mind B. head c. brain D. thoughts
2. They are always on the______ young and versatile people.
A. search for B. look out for c. need for D. terms o f
324
3. There's a
A. small
balance between success and failure.
B. narrow c. fine D. light
D. made
exaggerated.
4. My camera was stolen from my bag at the airport so 1_________ a claim on
my insurance.
A. had B. did c. took
5. Reports that he earns more than $5m a year are___
A. grossly B. perfectly c. absolutely D. extremely
6. The manager hesitated to assign the job to the newcomer as he was________ .
A. wet behind the ears B. feeling your ears burning
c. ringing in your ears D. keeping your ears open
7. If you w ant to solve the problem for ever, you must face a difficulty or danger
boldly, that is, you must_________ .
A. pull the bull's horns B. take the bull by horns
c. blow your own horn D. be on the horns of a dilemma
8. Mrs Ro y_____ on Saturday evening parties-you’ll find all kinds o f people there.
A. keeps the doors of the house open B. sets her house in order
c. keeps an open house
9. Terry______ of the matter long ago.
A. was good with his hands
c. had his hands full
10. People are always critical. One needs to_
D. brings the house down
B. got his hands dirty
D. washed his hands
to cope with criticism.
A. have a thick skin
c. get under their skin
B. have a thin skin
D. make their skin crawl
V. READING COMPREHE NSION
READING PASSAGE 1 (5 points)
Continents and ocean basins represent the largest identifiable bodies on Earth.
On the solid portions of the planet, the second most prominent features are flat
plains, elevated plateaus, and large mountain ranges. In geography, the term
“continen t” refers to the surface of continuous landmasses that together comprise
about 29.2% of the planet’s surface. On the other hand, another definition is
prevalent in the general use o f the term that deals with extensive mainlands, such
as Europe or Asia, that actually represent one very large landmass. Although all
continents are bounded by water bodies or high mountain ranges, isolated
mainlands, such as Greenland and India-Pakistan areas are called subcontinents.
In some circles, the distinction between continents and large islands lies almost
exclusively in the size o f particular landmass.
The analysis of compression and tension in the earth’s crust has determ ined
that continental structures are composed of layers that underlie continental
shelves. A great deal of disagreem ent among geologists surrounds the issue of
exactly how many layers underlie each landmass because of their distinctive
mineral and chemical composition. It is also quite possible that the ocean floor
325
rests on the top of unknown continents that have not yet been explored. The
continental crust is believed to have been formed by means of a chemical
reaction when lighter materials separated from heavier ones, thus settling at
various levels within the crust. Assisted by the measurements of the specifics
within crust formations by means of monitoring earthquakes, geologists can
speculate that a chemical split occurred to form the atmosphere, sea water, and
the crust before it solidified many centuries ago.
Although each continent has its special features, all consist of various
combinations of com ponents that include shields, mountain belts, intracratonic
basins, margins, volcanic plateaus, and blockvaulted belts. The basic differences
among continents lie in the proportion and the composition of these features
relative to the continent size. Climatic zones have a crucial effect on the
weathering and formation of the surface features, soil erosion, soil deposition,
land formation, vegetation, and human activities.
Mountain belts are elongated narrow zones that have a characteristic folded
sedimentary organization of layers. They are typically produced during
substantial crustal movements, which generate faulting and mountain building.
When continental margins collide, the rise of a marginal edge leads to the
formation of large mountain ranges, as explained by the plate tectonic theory.
This process also accounts for the occurrence of mountain belts in ocean basins
and produces eviden ce for the ongoing continental plate evolution.
1. What does this passage mainly discuss?
A. Continental drift and division
B. Various definitions of the term “continent”
c. Continental structure and crust
D. Scientific analyses of continental crusts
2. According to the passage, how do scientists define continents?
A. As masses of land without divisions »
B. As extensive bodies o f land
c. As the largest identifiable features
D. As surgical compositions and ranges
3. In paragraph one, the word ’’bounded” is closest in meaning to______ .
A.covered B. convened c. delimited D. dominat
4. The author o f the passage implies that the disagreement scientists is based on
the fact that______ .
A. each continent has several planes and shelves.
B. continents have various underlying layers o f crust.
c. continents undergo compression and experience tension.
D. continents have different chemical makeup.
5. The word “sp ecifics”_in the second paragraph is closest in meaning to_____ .
A. specialities B. speculations
c. exact details D. precise movements
326
6. The author o f the passage implies that______ .
A. it is not known exactly how the continental crust was formed
B. geologists have neglected the exploration of the ocean floor
c. scientists have concentrated on monitoring earthquakes
D. the ear th's atmosphere split into water and solids
7. According to the passage, what are the differences in the structure o f continents?
A. The proportional size o f continents to one another.
• B. Ratios o f major components and their comparative size,
c. The distinctive features of their elements.
D. Climatic zones and their effect on the surface features.
8. In paragraph four, the phrase “This process” refers t o ______ .
A. continental collision B. mountain ranges
c. the rise o f margins D. plate tectonic theory
9. The author o f the passage implies that______ .
A. the process of mountain formation has not been accounted for •
B. mountain ranges on the ocean floor lead to surface mountain building
c. faulting and continental margins are parts of plate edges
D. the process of continent formation has not been completed
10. The word “ev iden ce” in paragraph four is closest in meaning to______ .
A. eventually B. confirmation c. exemplification D. challenge
READING PASSAGE 2 (5 points)
MOUND-BUILDER THEORIES
Some of the most impressive geographical features in North America are the
many earth mounds scattered around the continent. These earthworks are
enormous artifioial hills constructed by various Native American civilizations for
ritualistic purposes, such as burials worship, and they are thousands of years old,
with the latest ones being finished hundreds of years before Europeans explored
America. Upon their arrival, European explorers were impressed with the
structures, but prejudice against the native tribes prevented them from
accrediting Native Americans with their construction. Instead, American settlers
developed several theories that claimed a superior but extinct "mound-builder"
civilization made the earthworks. Moreover, particular details of different
mound-builder theories reflected the specific prejudices of the people who
supported them. For example, devout Christian groups like the Mormons argued
that a sacred Israeli society was responsible, and white Americans argued that
only the Vikings could have built such mounds because they believed that their
European ancestors were far superior to Native Americans. The persistence of
these myths showed how for centuries Americans selectively examined evidence
and distorted science in order to support their own agendas against the natives.
Prior to the 20th century, many Americans accepted the mound-builder theories
as despite the dubious evidence that supported them. For instance, most
believers argued that the presence of metal artifacts beneath the mounds showed
that the natives couldn't have them because they had no know ledge of
í
327
metallurgy. Some tribes did in fact possess such skills, and the presence of
defensive walls around tribal lands indicates that Native Americans could indeed
construct structures such as earth mounds. Nonetheless, most Americans dismissed
such evidence and instead considered other potential candidates for the mound
builders. Popular choices were ancient Chinese, Greek, or African civilizations,
none of which were in prehistoric America. Other people argued that mystical
forces, such as God or people from mythical Atlantis, built the mounds.
A com mon presupposition for all of these theories was that the natives were
too unskilled and primitive to build these intricate and complex structures.
Ironically, by proclaiming the natives' ignorance, these theorists often displayed
their own ignorance: they didn't recognize that the writings on many artifacts
excavated from the mounds were Native American, particularly because they
didn't know that these tribes had written languages. However, the popularity of
these myths wasn't solely the result of racism and ignorance: these theories also
served the Americans' agenda of seizing native lands. During the 19th century,
when these theories were most popular, Americans expanded throughout the
continent and eventually gained total control of all native territory. The settlers
often justified their conquests by claiming that the natives themselves had stolen
the land after they eradicated the mound- builders, so the Americans were
essentially avenging these vanisfied peoples.
In order to support these agendas, proponents of the mound-builder theories
would point to any bit of evidence, no matter how weak, that lent credence to
their daims, and some even planted false evidence. Throughout the 19th century,
excavations at many mound sites produced forged tablets, and these supported
theories that either the natives had killed the original builders of the earth mounds or
that the mounds were indeed built by a sacred people. All of these were inscribed
with different languages, such as Mayan, Chinese, Hebrew, and Egyptian, and this
supposedly proved that non-native civilizations built these mounds.
A While these findings were eventually exposed as frauds, the moundbuilder theories
persisted, even this day, especially among groups that claim
racial superiority over the natives. B
Regardless of the misinformation and domineering prejudice that supported
mound- builder theories, there has always been support for the idea that natives
constructed the earthworks, c The early American president Thomas Jefferson
reached this conclusion when he performed excavations at these sites and
recognized similarities between native burials and mound burials. D Also, early
evidence includes accounts by Spanish and French explorers who stayed with
various native tribes and learned o f the construction of many mounds. The myths
finally ceased to be the dominating view after ethnologist Cyrus Thomas proved
that native tribes constructed the mounds. At the end of the 19,b century, this also
became the United States government's official position.
328
1. According to paragraph 1, the earth mounds
A. still pose many questions as to who built them
B. show that an American people preceded the natives
c. were scoffed at by the first European settlers
D. were misunderstood for hundreds of years
2. In paragraph 1, what does the author imply about the mound-builder theories?
A. They were based on native history.
B. They didn't find many supporters,
c. They confirm ed pre-existing beliefs.
D. They are widely accepted today.
3. The word dubious in the passage is closest in meaning to
„ A. solid B. technical c. confusing D. doubtful
4. The word resupposition in the passage is closest in meaning to
A. assumption B. conclusion c. contradiction D. question
5. Which of the sentences below best expresses the essential information in the
highlighted sentence in the passage? Incorrect choices change the meaning in
important ways or leave out essential information.
A. Many Native American tribes could actually write in their own languages,
which several believers of the mound-builder theories wouldn't realize
until decades later.
B. By claiming that the natives were too ignorant to write language, the
myths' believers actually showed that they were too ignorant to recognize
native writing.
c. The people who argued in favor of the mound - builder myths showed that
the discovery of inscribed artifacts proved no Native American tribe ever
built earthworks.
D. Proponents of myths surrounding the earth mounds lacked crucial
knowledge of the various native cultures, and this showed in their support
for such ignorant theories.
6. The phrase these vanished peoples in the passage refers to
A. mound-builders B. Americans c. natives D. settlers
7. In paragraph 3, the author describes the American conquest of native lands in
order to
A. show how important the mounds were to white Americans
B. explain how settlers were able to excavate the mounds
c. prove that natives couldn't build effective walls or mounds
D. discuss how the mound-builder theories were exploited
8. According to paragraph 4, how did the fake tablets allegedly prove that Native
Americans d idn 't build the earth mounds?
A. Th ey supposedly existed before any native society.
B. They featured writing from other civilizations,
c. They were metal, which natives couldn't make.
D. They resembled sacred tablets described in the Bible.
329
9. According to the passage, all of the following are true about mound-builder
theories EXCEPT:
A. Most expressed racist attitudes. B. They became government policy,
c. Some involved fictional societies. D. They relied on shaky evidence.
10. Look at the four choices A, B, c, or D that indicate where the following
sentence could be added'to the passage.
For instance, some black nationalist groups believe that an African civilization
responsib le for con str uctin g the pyram ids also bui lt the earth mounds;
like the Viking theory, this myth serves to stress pride in a p articular race.
Where would this sentence best fit?
VI. GU IDED CI^OZE TE ST
CLOZE TEST 1
Read the follow ing passage car efu lly and then choose best option to fit each
space (5 points)
Oxford is a city with such a mind-blowing reputation that many who come
here find themselves intimidated by the place and can’t wait to leave, while
others, taking to it like a duck to water, find themselves returning again and
again. The college lawns provide a gorgeous (1)______ to serious study, and in
the right light, on a sunny w inter’s morning say, one feels as if one is (2)______
on air. such is the sense of unreality. Oxford may like to (3)______ that it is at
the intellectual hub of things, but in many ways it is no more than a sleepy
(4)______ where, to mix metaphors, transitory students, the (5)______ of their
generation, wait in the (6)______ , allowing their talents to (7)_______before
moving off into the industrial or political fast-lane. Much of this, is a myth, of
course. Hardship and hard work are very much part and (8)______ of student
life. The (9)______ get through the three years' hard grind by simply putting
their shoulders to the wheel before going on to fairly average jobs. Only for the
tiny minority is Oxford the first (10)______on the ladder to fame and fortune.
1.A. backdrop B. curtain c.sc en e . D.screen
2. A flying B. gliding c. floating D. swimming
3. A. pretend B. act c. dissemble D. produce
4. A. backwater B. stream c. tributary D. watershed
5. A. froth B. cream c. fat D. caviar
6. A. pavilion B. dressing room c. wings D. foyer
7. A. flourish B. open c. spread D. float
8 .A. package B. section c. province D. parcel
9. A. level-headed B. hot-headed c. hot-blooded D. kind-hear
10. A. step B. position c. elevation D. ascent
CLOZE TEST 2
Read the follow ing passage car efu lly and then choose best option to fit each
space (5 points)
Viewed from the outside (1) ________ , the Houses of Parliament look
impressive. The architecture gives the place a traditional look, and the buildings
330
are sandwiched between a busy square and the river, making them a (2 )_______
between the country house of an eccentric duke .and a Victorian railway station.
You have only to learn that the members ( 3 ) ______ to each other as ‘The
Honorable Member to (4 )______ the picture of a dignified gentlemen's club,
with of course a few ladies to ( 5)_______ the numbers. Sadly, over the past few
years first radio, and now television, have shown the general public, who are
(6) ______ the electorate, what in fact goes on when bills are discussed and
questions are asked. The first obvious fact is that the chamber is very rarely full,
and there may be only a handful of members present, some of whom are quite
clearly asleep, telling jokes to their neighbor, or shouting like badly-behaved
schoolchildren. There is not enough room for them all in the chamber in any
(7) _______ , which is a second worrying point. Of course, television does not
follow the work of committees, which are the small discussions groups that do
most of the real work of the House. But the (8 )______ impression that voters
receive o f the workings of government is not a good one. To put it (9)_______ ,
parliament looks disorganized, is clearly behind the time and seems to be filled
with bores and comedians. This is presumably why members (1 0)_______ for
so long the efforts of the BBC to broadcast parliamentary matters on television.
1. A. likewise B. at least c. nevertheless D. as well
2. A. mixture B. combination c. cross D. match
3. A. call B. refer c. speak D. submit
4. A. finalize B. end c. conclude D. complete
5. A. take away B. bring about c. make up D. set in
6. A. after all B. anyway c.ev en D. furthermo
7. A. point B. way c. matter D.c ase
8. A. total B. broad c. overall D. comprehe
9. A. bluntly B. shortly c. directly D. basically
10. A. prevented B. checked c. defied D. resisted
B. WRITTEN TEST (70 points)
I. OPEN CLOZE TESTS
Read the text below and think of the word which best fits each space. Use
only ONE WORD for each space.
CLOZE TEST 1: (10 points)
Throughout our lives, right from the moment when as infants we cry to
express our hunger, we are engaged in social interaction of one form or another.
Each and ( 1 ) ______ time we encounter fellow human beings, some kind of
social interaction will take place, (2)_______ it’s getting on a bus and paying the
fare for the journey, or socializing with friends. It goes without (3) ______ ,
therefore, that we need the ability to communicate. Without some method of
(4) intentions, we would be at a(n) (5)_______ loss when it came to
interacting socially.
331
Communication (6)_______ the exchange of information which can be
anything from a gesture to a friend signalling boredom to the presentation of a
university thesis which may only ever be read by a (7)_______ of others, or it
could be something in (8)_______ the two. Our highly developed languages set
us (9)______from animals. But for these languages, we could not communicate
sophisticated or abstract ideas. Nor could we talk or write about people or objects
not immediately present. (10)_______we restricted to discussing objects already
present, we would be able to make abstract generalizations about the world.
CLOZE TEST 2: (10 points)
MY NEW FRIEND’S A ROBOT
In fiction robots have a personality, (1 )_________ reality is disappointingly
different. Although sophisticated (2) _________ to assemble cars and assist
during complex surgery, modern robots are dumb automatons, (3 )_________ of
striking up relationships with their human operators.
However, change is (4) _________ the horizon. Engineers argue that, as
robots begin to make (5 )_________ a bigger part of society, they will need a
way to interact with humans. To this end they will need artificial personalities.
The big question is this: what does a synthetic companion need to have so that
you want to engage (6 )_________ it over a long period of time? Phones and
computers have already shown the (7 )_________ to which people can develop
relationships with inanimate electronic objects.
Looking further (8 )_________ , engineers envisage robots helping around the
house, integrating with the web to place supermarket orders using email.
Programming the robot with a human-like persona and (9 )_________ it the
ability to learn its users’ preferences, will help the person feel (1 0)________
ease with it. Interaction with such a digital entity in this context is more natural
than sitting with a mouse and keyboard.
II. WORD FORMATION (20 points)
PART 1: Complete each sentence, using the correct form of the word in
parentheses.
1. Look ! I know you’ve lost your wallet, but there’s no use in keeping on______
the fact. Someone has stolen it and tha t's all there is to i t ! (MOAN)
2. Paul is a good employee, and is very__________ . (CONSCIENCE)
3. He must be drunk. He is making__________ remarks. (SENSE)
4. You can ask a__________ for advice on what kind of food you should eat to
keep you healthy. (DIET)
5. Few active steps were taken to measure, understand or manage the occurance
of__________ . (ABSENCE)
6. ________ fans crowd near the stage the moment the pop star appears. (AWE)
7. She finally left her husband because o fhis _______ . (ADULT)
8. Liskeard and Callington are basically__________ from here, so it’ll take about
the same time to get to either o f them. (DISTANT)
332
9. You wo n’t persuade him to change his mind. His decision is_________ .
(REVOKE)
10. Since his bad habits were never broken when he was a child, they are
now__________ . (CORRECT)
PART 2: Complete the following passage with the appropriate forms from
the words given in the box.
access appear commit deny
immerse infant institute lonely
{
One of the most challenging aspects o f the science anthropology comes from
its fieldwork. Certainly, in its (1)________ as a profession, anthropology was
distinguished by its concentration on so-called primary societies in which social
(2)________ appear to be fairly limited and social interaction to bẹ conducted
almost (3)______________ face - to - face. Such societies, it was felt, provided
anthropologists with a valuable (4)________ into the workings of society that
contrasted with the many complexities of more highly developed societies. There
was also a sense that the way of life represented by these smaller societies were
rapidly (5)____________ and that preserving a record of them was a matter of
some urgency.
The (6) _____ of anthropologists to the first - hand collection of data led
them to some of the most (7)________ places on earth. Most often they worked
alone. Such lack of contact with other people created feelings of intense
(8)________ in some anthropologists, especially in the early stages of fieldwork.
Nevertheless, this process of (9)________in a totally alien culture continues to
attract men and women to anthropology, and is (10)_________ the most
effective way o f understanding in depth how other people see the world.
III. ERROR IDENTIFICATION (10 points)
The passage below contains 10 mistakes. Identify and correct them.
Until recently, we have confined ourselves for our own solar system in the
search for life, partly because we have not had evidence for the existence of
other solar systems. Furthermore, our telescopes have not been powerful enough
to detect planets. But not long ago, a technique was developed that could
ascertain reliably whether stars have planets orbiting it. Basically, this technique
relies upon our ability to detect with some degree of precision how much light a
star is giving. If this change for a brief period, it is probably because a large
object - a planet - is passing in front of it. At first, the technique could only
establish the existence of a very large planet with an elliptical orbit that brought
it in close proximity to the star. This was one of the limits of the technique: life
could not exist on such large planets. Furthermore, the orbit of the planet would
333
preclude the possibility of other, smaller planets orbiting the same star.
Therefore, that particular planetary system could be effectively ruled out in terms
of the search for life.
However, astronomers using an Anglo-American telescope in New South
Wales now believe they have pinpointed a planetary system which resembles to
our own. For the first time, they identified a large planet, twice the size of
Jupiter, orbiting a star like the sun, at much the same distance from its parent star
as Jupiter is to the sun. And this is the vital point about their discovery: there is
at last a theoretical possibility that smaller planets could be orbiting inside the
orbit of this planet.
1.
4 .' 5 ._______________
;>
7.' 8 ._______________
_______________
_______________
6 .________________
9 .________________
IV
10.______________
. SENTENCE TRANSFORM ATION. (20 points)
Rewrite the following sentences using the words given.
1. The mob went away when the police came along.
—» A long ____________ _______________________________ .
2. The inevitability of unemployment was something nobody cared to admit.
—> The adm issi on________________ _____________________ .
3. Absolute secrecy was crucial to the success of the mission.
—> Witho ut________ ________________________ __________ .
4. Something must be done quickly to solve the problem of homelessness.
-> Urgent_________________ ___________________________ .
5. An pretends not to know about Minh's absence.
—» An t ur ns ________________________ __________________ .
6. Minnie meant well so you mustn't be offended by her comm ents. (AMISS)
-> Please____________ because she meant well.
7. It’s urgent that the new principal start to deal with the problem s fa cing the
school. (GRIPS)
-> The new principle will have to ____________ the problem facing the
school straight away.
8. My supervisor did not even hint to me that he was about to resign. (INDICATION)
-> I_ ________________________________ _ ________________ .
9. When I saw the exam questions, I couldn't remember anything. (MIND)
-» When I saw the exam questions, my_______________________ .
10. We have recently noticed that you have not been attending staff training
sessions. (ATTENTION )
—> It has __________ that you have not been attending sta ff training sessions.
334
TRƯỜNG THP T CHUY ÊH LONG AM - LONG flW
A. M ULTIPLE CHOICE (40 pts)
I. WOR D CHOICE (5.0 pts)
Choose the best options to complete the following sentences.
1. She n early _____ on her sandwich as she was eating very fast.
A. suffocated B. drowned c. choked D. fainted
2. She wasn’t allowed into the country ;_____ because her papers aren’t in order.
A. subsequently i B. presumably c. admittedly D. paradoxically
3. H e’s _____ his health, and takes dozens o f vitamins every day.
A. fanatical about B. fascinated by
C. obsessed in D. enthusiastic about
4. Andrew is a very disciplined employee. H e_____ great importance to coming
to work in time.
A. affixes B. attributes c. admits D. attaches
5. His public announcement of the secret plans was dealt a sheer _____ of
confidence and was heavily criticized by the other members of the council.
A. damage B. breach c. crash D. fracture
6. Nobody, even the best doctors, expected Frank’s _____ to be so quick and
successful after so severe an accident.
A. renewal B. resumption c. recuperation D. remuneration
7. Unanswered, the demands for nuclear deterrents h av e_____ fears o f civil war.
A. flashed up B. prognosticated c. sidetracked D. stoked up
8. Little did 1 imagine The Amazing Race would entail long-winded journeys and
ups and do wn s_____ .
A. aplenty B. inexhaustibly c. profusely D. superabundant
9. Researchers have made a( n) _____plea for more sponsorship so that they can
continue their project.
A. com passionate B. dispassionate c. encompassed D. impassioned
10. The game keeper was on the alert as there has been s om e____ on the estate.
A. poaching B. punching c. pouching D. pushing
11. GR AMMAR AND STRUCTURES (5.0 pts)
Choose the best options to complete the following sentences.
1. The growth of two-income families in the United Sta te s_____ of people
moving to a new social class.
A. has resulted in millions B. resulting in millions
c. results of millions D. millions of results
2. Stars differ fundamentally from pla net s_____ they are self-lum inous whereas
planets shine by reflected light.
A. from which B. when c. and D. in that
335
“Did you know Jim's car broke down on the highway late at night?"
“Unfortunately, that's a situation a ny on e_____ .”
A. might have to confront with
c. might be confronted
D. might have been confronted
Although he acts t ou gh ,_____ .
A. his bark is worse than his bite
B. might be con fronted with
B. he 's a tough nut to crack
D. he isn’t up to scratch
_ have been killed!
D. might
c. he’s learned the hard way
What a dangerous thing to do! You ___
A. may B. can c. must
On his de sk _____ , which he usually sits in front o f and I oaks at.
A. stood the picture of US B. it is the picture o f US
c. standing the picture of US D. stands the picture o f US
7. The sc ientists_____ the festival of Ramadan, but they were too busy with
their research in the laboratory.
A. would have liked to commem orate
B. would have loved to have observed \
c. would prefer to have obeyed
D. would sooner have celebrated
8. The m an _____ of carrying out the burglary was r ele ased____ _ by police.
A. to be suspected/followed questioning
B. having been suspected/following questioned
c. suspected/following questioning
D. being suspected/followed questioned
9. A new generation of performers, those who by now had become
a household name, honed their skills before following the same path onto
television.
A. no less talented than B. along with talented as
c. together with talented as D. having been more talented than
10. In some countries, acupuncture is used as an anesthesia, permitting patients to
have major surgery_____ .
A. while fully conscious B. what is fully conscious
c. that fully consciousness D. which is fully conscious
III. PH RASAL VERBS AND PREPOSITIONS (5.0 pts)
Choose the best options to complete the following sentences.
1. Towards the end of the film Thelma and Louise, a large truck is _____ by the
main characters when they shoot its tank, which is full of oil.
A. shot off B. struck in c. poured out D. blown up
2. In the novella, Hadji Murat is an important figure among the Chechens, but he
_____ the main leader Sheikh Shamil following an argument.
A. sets apart from B. falls out with c. cuts back on D. keeps ou t of
3. I thought the movie was going to an end, but it ju st_____ .
A. dragged in B. dragged on c. dragged into D. dragged up
336
4. When he saw her with John, J ac k_____ anger.
A. showed up B. bristled with c. pricked with D. brushed with
5. Ben’s so unlucky in love. Why does h e_____ the type of woman who brings
trouble?
A. fall for B. catch on c. put before D. set out
6. My father decides to apprentice m e_____ a lawyer although I d on’t agree.
A. with B. to c. for D. into
7. China, which has been producing cars _____ joint ventures with foreign
partners, is now launching its own bra nd _____ the global market.
A. through - on B. to - along c. of - throughoutD. among - for
8 .1 m uddle d_____the jigsaw pieces and the children did the puzzle again.
A. up B. off c. over D. for
9. We flic ked___ a few magazines while we waited in the doctor’s waiting room.
A. through . B. on c. at D. over
10. Karen was terribly nervous before the interview but she managed to pull
herse lf_____ and act confidently.
A. through B. over c. together D. off
IV. COLLOCATIONS AND IDIOMS (5.0 pts)
1. If you want a flat in the centre o f the city, you have to pay through t he_____ '
for it.
A. teeth B. head c. nose D. arm
2.1 like tny new job; the only fly in t he_____ is the fact that I have to work
every other weekend.
A. fat B. porridge c. soup D. ointment
3 .1 caught the last bus by the skin o f my____ .
A. mouth B. leg c. neck D. teeth
4. Since he started his own business, he has been making money hand over____ .
A. fist B. heel c. head D. palm
5. He was wearing very shabby, dirty clothes and looked very_____ .
A. easy-going B. down to teeth c. out o f shape D. down at heel
6. I’m afraid you’ve caught me on th e_____ . 1 wasn’t expecting you until this
afternoon.
A. stove B. grapevine c. spot D. hop
7. The escaped prisoner fo ug ht ____ before he was finally overpowered.
A. head over heels B. tooth and nail c, heart and soul D. foot and mouth
8 .1 d idn’t suspect anything at first, but when I noticed her going through the
office drawers I began to sme ll_____ . .
A. a rat B. a pig c. a thief D. a culprit
9. Nagging Susan to stop smoking has no effect on her. It’s like water off _____ .
A. a windmill B. a du ck’s back c. a dripping tap D. an umbrella
10 .1 hope the computer course stalls this term. We’re all as keen as . _____
to get going.
A. coffee B. mustard c. a gigolo D. cornflakes
337
V. READING COMPREHE NSION
Read the following passages and choose the best option to complete the
blank or answer the question.
PASSAGE 1: (10 pts)
THE BALANCE IN THE OCEANS
The oceans' predators come in all shapes and sizes. For example, one of the
less infamous ones is the colorful starfish, which feeds on plant life, coral, or
other shellfish such as mussels for sustenance. A more bloodcurdling example,
especially to human beings and most other species of fish, is the shark, though
most scientists agree that only ten per cent o f the 450 plus species o f sharks have
been documented as actually attacking a human. Still, there is another predator
lurking invisibly in the bodies of water o f the world, one which poses one o f the
greatest threats to all species of ocean life - bacteria. Though many types of fish
are continually stalking and evading one another for survival, they all band
together in an attempt to keep bacteria levels at bay in order to allow their
own existence to continue.
Bacteria play a dual role in the ecosystems of the oceans. On the one hand,
they are beneficial as they stimulate plant life through food decomposition,
which releases the necessary chemicals for the growth of plant life. This is called
nutrient recycling and helps keep the oceans alive. But, on the other hand,
bacteria are a major predator for all fish because they attack fragile, weaker
individuals. If they are allowed to run rampant and not kept in check, they could
virtually suffocate the oceans. In water, bacteria prove to be an even greater
threat than on land because, as they proliferate, they reduced the oxygen levels
necessary for organisms in the oceans to live. Further, when fish populations
become depleted due to factors like overfishing, microbes such as algae expand
and threaten the fragile ecosystems of the ocean. Therefore, ocean predators play
a critical role by thwarting bacteria growth and maintaining the oceans'
equilibrium by reducing vulnerable links in the food chain.
In many ways, the balance within the oceans’ ecosystems mirrors the human
body. That is, all of their components must work in harmony for them to stay
healthy, efficient, and alive. If one of them is missing or deficient, an entire
system can be placed in jeopardy. In both the human body and the ocean,
bacteria play a vital role because, at manageable levels, they aid in protecting
and cleaning each system of foreign agents that can be of harm. On the other
hand, if bacteria levels increase and become out of control, they can take hold of
a system, overrun it, and become debilitating. Therefore, both oceans and the
human body have a kind of custodian that maintains bacteria levels. In the
human body, it is called a phagocyte. Phagocytes eat up sick, old, or dying cells,
which are more prone to bacterial invasion, and thus keep the body healthy. Like
in the human body, bacteria can prove fatal to the living organisms in the ocean.
Like phagocytes in the human body ocean predators work as antibacterial
custodians of the seas. In essence, they are the immune system and a vital link in
338
the food chain because they remove small, injured, or sickly fish from the ocean
environment before bacteria can become too comfortable and multiply. By
ridding the ocean of weaker fish, predators allow the stronger ones to multiply,
making their species stronger and more resilient. Without their services and with
their declining numbers, bacteria will blossom to levels that will eventually
overpower and kill even the strongest species of fish because of the depletion of
their number one source o f life, all important oxygen.
While the greatest battle in the ocean may seem on the surface to be the
survival of the fittest fish, closer look reveals something completely different:
fish versus microorganisms. Clearly, most living organisms in the oceans are
hunters by nature, but this way o f life does not merely provide a food source for
a dominant species, h also maintains a healthy level of bacteria in an ocea n's
ecosystem, thus ensuring the continuation of all species of life within. Major
predators are necessary, like the antibacterial cells of the human body, to keep
this delicate balance in synch. If their numbers continue to decline and humans
ignore their vital role in the ocean, dire consequences will definitely result.
1. The word “lur kin g” in the passage is closest in meaning t o _____
A. attacking B. increasing c. waiting D. approaching
2. According to paragraph 1, which of the following is true of ocean predators?
A. The shark is the deadliest one for all other kinds of life in the oceans.
B. One o f the most threatening to all fish populations is bacteria.
c. Starfish do little damage to the population of mussels and shellfish.
D. Most of the killers that hide in the oceans are unknown to humans.
3. Which of the following can be inferred from paragraph 1 about bacteria?
A. They can be extremely detrimental to fish if their numbers increase.
B. They are able to feed o ff themselves when other food sources are limited,
c. They stimulate plant life, which in turn releases oxygen into the water.
D. They present themselves in numerous shapes and forms as well as colors.
4. The author discusses '“nut rient rec ycling ” in paragraph 2 in orde rto...........
A. show how bacteria act similarly in the ocean and the human body
B. explain the different roles o f nutrients and oxygen for species o f fish
c. indicate that bacteria do have a positive impact in the oceans
D. note how chemicals from bacteria are able to stimulate plant growth
5. Which of the sentences below best expresses the essential information in the
sentence in bold (“Th oug h... c ontinue”)?
A. Evasion tactics help fish escape from the threats posed by an increasing
number o f bacteria.
B. Various species of fish prey upon one another in order to lower bacteria
levels in the ocean.
c. High bacteria levels in the ocean help most species of fish to survive by
providing them with food.
339
D. Rivals or not, all fish help one another survive by preventing bacteria from
proliferating.
6. The word “th war tin g” in the passage is closest in meaning t o _____
A. encouraging B. preventing c. slowing D. Sustaining
7. According to paragraph 2, bacteria are dangerous to ocean life bec ause_____
A. they have the capability to attack both strong and weaker fish
B. they could monopolize the critical breathable gas in the ocean
c. they get rid of vulnerable links, like dying fish, in the food chain
D. they blossom out of control when overfishing becomes dominant
8. The word “de bilitating” in the passage is closest in meaning t o _____
A. stimulating B. hindering c. elevating D. weakening
9. The author's description of phagocytes mentions all of the following EXCEPT:
A. They rid the human body o f potentially dangerous organisms.
B. They act in a similar manner as the predators of the ocean,
c. They dispose of bacteria to make weakened cells revive.
D. They are cleaning agents in humans to maintain bacteria levels.
10. The word “I f ’ in the passage refers t o _____
A. nature B. way o f life c. food source D. dominant species
PASSAGE 2: (10 pts)
ARTISANS AND INDUSTRIALIZATION
Before 1815 manufacturing in the United States had been done in homes or
shops by skilled artisans. As master craft workers, they imparted the knowledge
of their trades to apprentices and journeymen. In addition, women often worked
in their homes part-time; making finished articles from raw material supplied by
merchant capitalists. After 1815 this older form of manufacturing began to give
way to factories with machinery tended by unskilled or semiskilled laborers.
Cheap transportation networks, the rise of cities, and the availability of capital
and credit all stimulated the shift to factory production.
The creation of a labor force that was accustomed to working in factories did
not occur easily. Before the rise of the factory, artisans had worked within the
home. Ap prentices we re conside red par t of the fam ily, and ma sters we re
responsib le not only for tea ching the ir apprentices a tra de but also for
providing them som e education and for supervising the ir mo ral beh avior.
Journeymen knew that if they perfected their skill, they could become respected
master artisans with their own shops. Also, skilled artisans did not work by the
clock, at a steady pace, but rather in bursts of intense labor alternating with more
leisurely time.
The factory changed that. Goods produced by factories were not as finished
or elegant as those done by hand, and pride in craftsm anship gave way to the
pressure to increase rates of productivity. The new methods of doing business
involved a new and stricter sense of time. Factory life necessitated a more
340
regimented schedule, where work began at the sound of a bell and workers kept
machines going at a constant pace. At the same time, workers were required to
discard old habits, for industrialism demanded a worker who was alert, dependable,
and self-disciplined. Absenteeism and lateness hurt productivity and, since work
was specialized, dis rupte d the regular factory routine. Industrialization not only
produced a fundamental change in the way work was organized; it transformed
the very nature o f work.
The first generation to experience these changes did not adopt the new
attitudes easily. The factory clock became the symbol of the new work rules.
One mill worker who finally quit complained revealingly about "obedience to
the ding-dong of the bell-just as though we are so many living machines." With
the loss of personal freedom also came the loss of standing in the community.
Unlike artisan workshops in which apprentices worked closely with the masters
supervising them, factories sharply separated workers from management. Few
workers rose through the ranks to supervisory positions, and even fewer could
achieve the artisan's dream of setting úp one's own business. Even well-paid
workers sensed their decline in status.
In this newly emerging economic order, workers sometimes organized to
protect the ir rights and traditional ways of life. Craft workers such as carpenters,
printers, and tailors formed unions, and in 1834 individual unions came together
in the National Trades' Union. The labor movement gathered som e mo mentu m in
the decade before the Panic of 1837, but in the depression that followed, labor's
strength collapsed. During hard times, few workers were willing to strike or engage
in collective action. And skilled craft workers, who spe arh ead ed the union
movement, did not feel a particularly strong bond with semiskilled factory workers
and unskilled laborers. More than a decade o f agitation did finally bring a workday
/ shortened to 10 hours to most industries by the 1850's, and the courts also
recognized workers' right to strike, but these gains had little immediate impact.
Workers were united in resenting the industrial system and their loss of status,
but they were divided by ethnic and racial antagonisms, gender, conflicting
religious perspectives, occupational differences, pol itical par ty loyalties, and
disagreements over tactics. For them , the factory and industrialism were not
agents of opportunity but reminders of their loss o f independence and a measure
of control over their lives. As United States society became more specialized and
differentiated, greater extremes of wealth began to appear. And as the new
markets created fortunes for the few, the factory system lowered the wages of
workers by dividing labor into smaller, less skilled tasks.
1. Which of the following can be inferred from the passage about articles
manufactured before 1815?
A. They were primarily produced by women.
B. They were generally produced in shops rather than in homes.
341
c.They were produced with more concern for quality than for speed of production.
D. They were produced mostly in large cities with extensive transportation
networks.
2. Which of the sentences below best expresses the essential information in the
highlighted sentence in the passage? Incorrect answer choices change the
meaning in important ways or leave out essential information.
A. Masters demanded rhoral behavior from apprentices but often treated them
irresponsibly.
B. The responsibilities of the master to the apprentic e went beyond the
teaching of a trade.
c. Masters preferred to maintain the trade within the family by supervising
and educating the younger family members.
D. Masters who trained members of their own family as apprentices
demanded excellence from them.
. 3. The word “dis rupte d” in paragraph 3 is closest in meaning to
A. prolonged B. established c. followed D. upset
4. In paragraph 4. the author includes the quotation from a mill worker in order
to _____
A. support the idea that it was difficult for workers to adjust to working in
factories
B. to show that workers sometimes quit because of the loud noise made by
factory machinery
c. argue that clocks did not have a useful function in factories
D. emphasize that factories were most successful when workers revealed their
complaints
5. All of the following are mentioned in paragraph 4 as consequences o f the new
system for workers EXCEPT a loss o f_____
A. freedom
B. status in the community
c. opportunities for advancement
D. contact among workers who were not managers
6. The phrase “gathe red some momen tum ” in paragraph 5 is closest in meaning
to _________
A. made progress B. became active c. caused changesD. combined forces
7. The word “spe arh ead ed” in paragraph 5 is closest in meaning to _____
A. led B. accepted c. changed D. resisted
8. Which of the following statements about the labor movement of the 1800's is
supported by paragraph 5?
A. It was most successful during times of economic crisis.
B. Its primary purpose was to benefit unskilled laborers,
c. It was slow to improve conditions for workers.
D. It helped workers o f all skill levels form a strong bond with each other.
342
9. The auth or identifies “poli tic al party loy alt ies , and di sa gr ee men ts ov er
ta ctics” as two o f several factors t hat _____
A. encouraged w orkers to demand higher wages
B. created divisions among workers
c. caused work to becorr/e more specialized
D. increased workers' resentment o f the industrial system
4. The word “th em ”, in paragraph 6 refers to _____
A. Workers B. political party loyalties
c. disag reements over tactics
D. agents o f opportunity
VI. GUIDED CLOZE TEST:
Re ad the pa ssa ges below a nd de cid e w hic h an sw er b est fits eac h space.
PASSAGE 1: (10 pts)
Over the past thirty years or so, the methods used for collecting money from
the public to (1 )_____ the developing world have changed out o f all recognition,
along with the gravity of the problems faced, and the increasing awareness
among the population that something must be done. At the beginning of this
period, it would have been common to put ( 2)_____ in a collecting box, perhaps
on the street or at church, or to receive a small ‘flag’ to wear in the lapel. The
1960s saw the development o f shops which sold secondhand goods. (3) '
by the public, and which also began to sell articles manufactured in the
developing world in projects set up by the parent charity, to guarantee a fair
income to local people.
The next development was probably the charity 'event', in which participants
were (4 )_____ to run, cycle, swim and what have you, and collected money
from friends and relatives according to how far or long they managed to keep
going. The first hint of what was to become the most successful means of
(5 )_____ money was the charity record, where the artists donated their time and
talent and the proceeds from sales went to a good cause. This was perhaps a
reflection of the fact that young people felt increasingly concerned about the
obvious differences between (6 )_____ in Europe and the United States, and that
in most o f Africa and Asia, and this concern was reflected in songs besides being
clearly shown on television. The problems were becoming hard to (7 )_____ , but a
feeling of frustration was building up. Why was so little being done? The huge
success of Band Aid, and subsequent televised concerts, reflected the (8 )_____ of
the media, and of music in particular but also differed in style from other events.
People phoned up in their thousands on the day and (9 )_____ money by quoting
their credit card numbers. After all, if you have enough money to buy CDs arid a
stereo player, you can afford something for the world's (10)_____ children.
1. A. finance B. aid c . pay D. loan
2. A. this B. money c. them D. funds
3. A. donated B. freed c. offered D. awarded
343
4. A. sponsored B. invited c . required D. used
5. A. borrowing B.suc h c. furthe r D. raising
6. A. being B.life . c. them D. lifestyles
7. A. avoid B. understand c. define * D. implemen
8. A. mass B. ability c. style D. power
9. A. loaned B. handed in c. pledged D. raised
10. A. famine B. underdeveloped c. starving D. own
PASSAGE 2: (10 pts)
The money that some professional sportsmen earn sho uld n't impress anyone
when you take into (1 )_____ the fact that only a few of them manage to attain
immortality and everlasting fame. And once they reach their ( 2)_____ and display
their talent at their best, they are fully conscious that their brilliant careers won ’t
last forever. They live under a constant of being (3) _____ and subsequently
replaced by someone who is younger, faster and more accomplished. For that
reason, objectives like retirement benefits and pensions are (4) _____ great
concern to all professional athletes.
Some of the retired competitors go as far as to organize strikes and rallies to
voice their protest against any policy unresponsive to their demand (5 )_____ the
younger professionals seek more upgrading solutions to the problem as more and
more of them attach a proper significance to ( 6 )_____ a solid education, even at
university level. Such an approach should help them find interesting and wellpaid jobs
(7 )_____ their sports career is over.
A completely new strategy has been devised by the schools priding
themselves (8 )_____ supporting their own teams. Their authorities insist that the
sports clubs members achieve high academic standards or else they are debarred
from partaking certain sports events, which may lead to further disruption in
their professional careers.
By these practical and most- effective (9 )_____ , combining education with
sports activity, the (1 0)_____ of the professional athlete as being brainless and
unintelligent may eventually be changing to the sportsmen’s benefit.
1. A. reflection B. attention c. examination D. considera
2. A. prime B.shap e c. best D. capacity
3. A. outcast B. outshone c. outstayed D. outgrown
4. A. with B. in c. at D. o f
5. A. whereby B. whereas c. whereupon D. wherein
6. A. mastering B. learning c. receiving D. attending
7. A. right away B. promptly c. barely D. once
8. A. with B. on . c. for D. in
9. A. grounds B. results G. factors D. means
10. A. vision B. outlook c. image D. judgment
'344
B. WRITTEN TEST:
I. OPEN CLOZE TEST (20pts)
Fill each o f the following numbered blanks with ONE suitable word.
PART 1:
PSYCHOLOGY: THE SCIENCE OF THE MIND
Psychology is the study of the mind, by far the most sophisticated machine
(1) _____ _ Earth. But how can something as inaccessible as the mind be
studied? Even (2 )_______ we were to open someone’s skull and look inside, we
would only see the brain, not the mind in action. (3 )_______ we cannot observe
the mind directly, it controls everything we do. Therefore, psychologists study
human behaviour in order to discover (4 )______the mind works. The behaviour
that interests them ranges from simple acts such as feeding, to much more
complex skills (5) _______ language. Psychologists measure behaviour, and
often use statistics to show that what they find is reliable evidence and not just
down to chance. The scientific knowledge gained from this research is then used
by practising psychologists. For example, clinical psychologists- (6 )_______
make up the largest group of specialists - help people with emotional problems
(7) _______ with their difficulties. Research findings are not only used by
psychologists, but also by other professionals who are concerned with the ways
people interact-doctors, teachers and judges, to name just a (8) _____________ .
A substantial body of psychological knowledge has been built up since the
nineteenth century. Nevertheless, the enormous complexity of the mind
(9 )_______ that there will always be more to learn about it, (10 )________much
research is undertaken.
PART 2:
GRANDPARENTS: THE NEW GENERATION
Always a sure source of affection, my grandparents were hugely important
figures in my life. They (1 )_______ shower my sisters and me with sweets,
indulgences and stories, telling tales about my parents as naughty children.
When the last of (2) _______ died, we all wondered who would hold the family
together.
People have relied on grandparents in Britain since the Industrial Revolution,
(3 )_______ whole families moved into cities from the country to get work in the
new factories, taking grandmother along to look after the children. Despite the fact
that more grandmothers are working now, grandparents are still the backbone of
childrencare in Britain. They provide 44% o f full-time care for pre-school children,
which makes you wonder how the country would manage (4 )_____ _ them.
The traditional image of a grandparent is a smiling old person surrounded by
a cohort of happy children, but ( 5)_______ doesn't match the facts. (6 )_______
we have now is the so-called ‘‘beanpole family”, thinly stretched (7 )_______
several generations, with fewer family members in each and with growing
345
nu mbe rs o f sin gle-pa rent fam ilie s. Gr andpare nts are getting yo un ge r-m ore than
50% o f grand pa rents hav e a lre ady had the ir firs t grandchild by the a ge o f 54.
Fo r man y o f the m, grandp arenthoo d me ans jug gling a job , invo lvem en t with
gran dc hildren and, som eti me s, the care of (8) _ _ _ _ _ _ _ _ own parents. It 's up
to us to ba lanc e the dema nds we ma ke on them if we don't wan t to wea r them
( 9 ) ________. Gr andpare nts are (1 0 )________a valua ble pa rt o f the fam ily that
we ju st can no t d o witho ut them.
II. WORD FORM S (20pts)
Fill in each blank with the most suitable form o f the word in brackets.
PART 1:
1. So me people wake up every mo rning with a sen se o f ease, quiet , and
spaciou sness in the ir mind, while oth ers tend to feel anxiou s, fearful and
tow ard the day ahea d. (PA TH ET IC)
2. We (judge)______ how long it would tak e US, so the ceremo ny wa s ov er by
the ti me we arr ive d, m is ju dg ed ..
3. Up to a point, peo ple can imp rove th e ir ______ skills with pract ice , though
ma ny sci entists bel ieve it's not pos sible for the brain to car ry ou t two or m ore
processes sim ultaneou sly . (TA SK )
4. In ______ talk (that is, talk in which the par ticipants do not h ave equal pow er,
sta tus , res ponsibility or con trol), we should app ly the 'one rule for one and
one for anothe r' princi ple cons iste ntly. (SY MM ET RY )
5. Af ter the Sec ond World War, there was inde ed a/an _______o f develop ing
natio ns tha t we re able to think up political institu tions with a vie w to
achie vin g ind epend enc e from the ir prior col oni al and sus tain mas s
en ga geme nt the rea fter. (RISE)
6. By pausi ng for a mo me nt or tw o before spe aki ng you will sim ply b e ______ a
nervo us tendency to rush headlon g into eve rythin g. (AC T)
7. Ma ny in no ce nt ______ we re killed in the cro ssfire. (ST AN D)
8. A cro wd of cu ri ous_____ soon gather ed to see what was hap penin g. (LO OK )
9. The oth er team com ple tely (play)______ US and we lost 6-0.
10. Wo men who are slimm ing can never enjoy a meal wi tho ut bei ng afraid of
the ir diet. (ORG AN ISE )
PART 2:
Recently, researchers have been kept busy picking lettuces under cov er o f darkness,
but the re is a perfe ctly rea son abl e exp lan ation for th is _________(1. APPEAR)
biz arre experim ent. Tests have shown tha t the vegetab les pic ked at nig ht stayed
fr esh _________(2. CONSIDER) longer than those picked durin g the d ay, tho ugh
the r eas on for this im pr ov ed ________ _(3. LONG) is u nclear.
Lunar garde ners claim to have kno wn about the _________(4. BENEFIT)
effects of noc turnal veg etable ma nageme nt for yea rs, and tho se gardeners who
belie ve in envir onme nta lly frie ndly org anic metho ds see the idea of working
346
with the moon's influence as a continuation of their principles. They claim to be
following a tradition, long-established in various parts of the world, of working
in harmony with the moon’s _____________ (5.GR AV ITY) pull. In England,
lunar gardening reached its zenith in the 16th century, but the vagaries of the
______________ (6. PR ED ICT) climate meant that it survived only as part of an
oral folklore tradition.
_________ (7. CO NF US E), several different and sometimes contradictory
system s are practised today. Although all of them focus on the effects of
moonligh t and the moo n's pull on the Earth's water, the exact science remains
controversial. There are so m e________ (8. HO RT ICUL TU RE ) who regard the
ideas w it h ______ (9. SCEP TIC). Others, however, are more encouraging and
le ss _________ (10. DISM ISS ), and advocate further research, even though no
discernible results have been forthcoming in support o f any particular theory.
Hl. ERR OR IDENT IFIC ATION (1 Opts)
Read the tex t, find th ei o mis takes and cor rec t them. You sho uld ind ica te in
wh ich line the mistake is.
4.
7.
123456789
10
11
12
13
14
15
16
17
18
19
20
The market for tourism in remote areas is booming as never before.
Countries all across the world are active promoting their ’wilderness'
regions - such as mountains, Arctic lands, deserts, small islands and
wetlands - to highly spending tourists. The attraction of these areas is
obvious: by defining, wilderness tourism requires little or no initial
investment. But that does not mean that there is no cost. Like the 1992
UN Conference on Environment and Development recognized, these
regions are fragile (i.e. highly vulnerable of abnormal pressures) not just
in terms of the culture of their inhabitation. The three most significant
types of fragile environment in these respects are deserts, mountains and
Arctic areas. An important character is their marked seasonality.
Consequently, most human acts, including tourism, are limited to clearly
defined parts o f the year.
Tourists are drawn to these regions by their natural beauty and the
unique culture of its people. And poor governments in these areas have
welcomed the 'adventure tourists', grateful for the currency they bring.
For several years, tourism is the prime source of foreign exchange in
Nepal and Bhutan. Tourism is also a key element in the economics of
Arctic zones such as Lapland and Alaska and in desert areas such as
Ayres Rocks in Australia and Arizona's Monument Valley.
2 ._______________ 3 .__________________
5 ._______________ 6. _ _ _ _ _ _ _
8. ■__________ 9 ._______________
10. __ ____________
347
IV. SENTENCE TRANSFORMATION (20pts)
Rewrite the following sentences with the given words or beginning in such a
way that their meanings remain unchanged. You MU ST NO T change the
given words in any way.
1. We only came to this restaurant because you insisted that we did so.
—» It was ____________ •_____________ _______________ __________
.
2. Melis sa's father was very busy, but he still played with her.
—> Bu sy ________________________________________________ _ ______.
3. Success in academic field depends on your ability to amass qualifications.
—> The m or e__________________________________________________•
4. It was pure chance that a policeman was passing as I was attacked.
—> O nly________________________________________________________.
5 .1 c ouldn 't make Julie give a definite answer.
—> I c oul dn't p in _____________________ ____________________ .______.
6. I hope his story will help US to understand what happened. (SHED)
—> H opefully when he tells US his Story, it w ill_________________________.
7. He was finally able to adjust himself to the new working condition. (SWING)
—> He finally got___________________________ ’______________________ ;
8. Her role in the plot was of secondary importance. (SECOND FIDDLE)
—>_________________________________ ;___________________________ .
9. The offer to go and work in Brazil was a complete surprise. (BLUE)
—>
10. This liver condition is common in those who drink a lot. (DRINKERS)
- > __________________________ 2____________ ’__________ ._________ .
TRƯỜNG THPT CHUYÊN LƯŨNG THÊ VINH - ĐỔNG NAI
A. MU LT IPLE -CHO ICE QUES TI ONS (4 0 PTS)
I. PHONOLOGY (5 PTS)
Sort out the word with the underlined part pronounced differently from
that of the others.
1. A. confusedly B. allegedly c. supposedly D. wickedly
2. A. sizzle B. why c. quits D. pidgin
3. A. comet B. simile c. starlet D.sequence
4. A. sociable B. species c. specimen D. specialist
5. A. pneumonia B. Portugese c. suitable D.n eph ew
Pick out the one word with a different stress pattern from the others.
6. A. mollify B. dissipate c. tentative D. panacea
7. A. armchair B. innate c. accent D. datum
8. A. bigotry B. adherent c. asylum D. flamboyan
9. A. frivolous B. hibernate c. jeopardize D. procastina
10. A. morale B. moustache c. contour D. disease
348
II. WORD CHOICE (5 PTS):
Read the sentences and choose the best answer.
11. That human rights ar e___________ is unacceptable in a civilized society.
A. infringed B. impeached c. abrogated D. quashed
12. I've heard that argument before and quite frankly it just doesn 't __________!
A. face the music B. hold water c. carry weight D. hit the roo f
13. That ancient car of his i s___________ joke among his friends.
A. standing B. steady c. settled D. stable
14. He’s regarded as the___________ favourite to win the prize
A. warm B. fiery c. hot D. ardent
15. When the police examined the house, they found that the lock had been
A. touched B. broken c. hindered D. tampered
16. We need to ___________ the trend towards centralized power.
A. repeal B. reverse c. overturn D. undo
17. Financial worries gra dually___________ his health and he was obliged to
retire early.
A. disabled B. exhausted c. undermine D. invalidated
18. He looks very aggressive and threatening, and so his soft, gentle voice is
rathe r___________ .
A. disembodied B.disconcerting c. dismissive D. discordant
19. After congratulating his team, the coach left, allowing the players to let their
__________ _ for a while.
A. souls B. hearts c. heads D. hair
20. Grandma says there wasn’t a _________ __ o f truth in that story Grandpa told
last night about being a war hero.
A. speck B. grain c. crumb D. dot
III. GR AMMAR AND STRUCTURES (5PTS):
Read the sentences and choose the best answer.
21. ___________ worry about our teenagers getting into trouble.
A. We adults B. We are adults c. Adults we D. Adults us
22. What happened ___________ their car broke down on the motorway so they
did n't get to Jo’s wedding on time.
A. to be that B. being that c. was that D. to that
23. Public television stations are different from commercial sta tio ns________ .
A. because they receive money differently and different types o f shows
B. for money and program types
c. in the areas o f funding and programming
D. beca use the form er receives money and has programs differently from
the latter
349
24. Ì don't think it would be wise to try to make Max change his mind about
divorcing Barbara. Well, in his place 1___________ her at all.
A. would never have married B. needn’t have married
c. would never marry D. must never have married
25. Once known as the “Golden State” because of its gold mines, _ ________ ■
A. North Carolina today mines few metallic minerals
B. few metallic minerals are mined in North Carolina today
c. there are few metallic minerals mined in North Carolina today
D. today in North Carolina few metallic minerals are mined
26. Unsalted butter is best for this recipe, bu t_________ that, m argarine will do.
A. except B. failing c. for all of D. given
27. Smith had a lucky escape. H e___________ killed.
A. would have been B. must have been
c. could have been D. should have been
28. It is imperative that your facebook passwo rd___________ confidential.
A.need keeping B. need to keep
c. needs to be kept D. needed keeping
29. “Which is more important: luck or effort?” “Luck i s___________ effort.”
A. of the same importance B. the same importance as
c. as the same importance as D. of the same importance as
30. This year will be difficult for this organization bec aus e___________ .
A. they have less money and volunteers than they had last year
B. it has less money and fewer volunteers than it did last year
c. the last year it did not have as few and little volunteers and money
D. there are few er and volunteers that in that last year there were
IV. PHRASAL VERBS AND PREPOSITIONS:
Read the sentences and choose the best answer.
31.1 have a predilection for clothes that jum ps ___________ you.
A. up on B. out at c. in for D. down under
32. The company had to cough___________ a lot o f money to indemnify for the
victims in the accident.
A. up B. out c. into D. away
33. Picasso bequeathed most o f his paintings____ _____ France and Spain.
A. for B. over c. to D. in
34. Don 't remind her about that. She has jus t stopped crying and you may risk,
setting her ___________ with that remark.
A. up B. against c. away D. off
3 5 .1 can only hold hi m ___________ contempt after what he said in the speech.
A. to B. in c. with D. for
36. She soon has complete contempt___________ misogynists.
A. on B. for c. at D. with
350
1
37. 1 got the wrong number because my fingers sli pp ed ___________ the button
as 1 was punching in the number.
A. away B. on c. off D. under
38. Tom has set his heart on jac ki ng _____ his job and travels around the world.
A. up B. off c. out D. in
39. The figures give the lie______ the rumours that the company is on the rocks.
A. at B. with c. up D. to
40. John turned ___________ trumps, finally finding a job to sustain his life.
A. up B. to c. in D. at
V. GUIDED CLOZE (1 OPTS):
Read the passages and choose the best answer.
PASSAGE 1
Childhood is the time when there are (1 )______ responsibilities to make life
difficult. If a child has good parents, he is fed, looked after and loved,
(2 )______ he may do. It is improbable that he will ever again in his life be given
so much without having to do anything in return. In ( 3 )______, life is always
presenting new things to the child - things that have lost their interest for older
people because they are too well-known. A child finds pleasure playing in the
rain, or in the snow. His first visit to the seaside is a marvellous adventure.
(4) ______ , a child has his pain. He is not so free to do as he wishes as he thinks
older people are: he is continually told not to do things, or being punished
(5) ______ what he has done wrong. His life is therefore not perfectly happy.
When a young man starts to ( 6)______ his own living, he becomes free from
the discipline of school and parents, but at the same time he is forced to accept
responsibilities. He can no longer expect (7 )_______to pay for his food, his
clothes and his room, but he has to work if he wants to live comfortably. If he
spends most time playing about in a day he used to as a child, he will be hungry.
And if he breaks the laws o f society as he used to break the laws of his parents,
he may go to prison. If, however, he works hard, keeps out of troubles and has
good health, he can have the greatest happiness of seeing himself make steady
(8 )______ in his job and o f building up for himself his own position in society.
Old age has always been thought of as the worst age to be, but it is not
necessary for the old to be unhappy. With old age (9 )______ wisdom and the
ability to help others with advice wisely given. The old can have the joy of
seeing their children making progress in life: they can watch their grandchildren
growing up around them, and perhaps best o f all, they can, if their life has been a
useful one, feel the happiness of having come (1 0 )______ the battle of life
safely and of having reached a time when they can lie back and rest, leaving
others to continue the fight.
351
41. A. little B. few c. a little D. a few
42. A. whenever B. wherever c. whatever D. whosoever
43. A. short B. term c. reality D. addition
44. A. H owever B. But c. Therefore D. Moreover
45. A. because B. for c. at D. by
46. A. take B. have c. create D.earn
47. A. other B. another c. others D. someone
48. A. progress B. achievement c. improvements D. accomplishments
49. A. coming B. come c. came D. had come
50. A. out B. across c. through D. back
PASSAGE 2
United Parcel Service (UPS) believes that its em ployees should give the firm
a fair day 's work for a fair’s day pay. The package delivery firm seems willing to
give more than a fair's day pay. But in (1) ______ , UPS expects maximum
output from its employees.
Since 1920s, the firm 's industrial engineers have been studying every detail
of every task (2 )______ by most UPS employees. From their studies have come
time and motion standards that (3 )______ how those tasks are performed and
how long they should take. Drivers, for example, are expected to walk to a
custom er's door at a speed of exactly three feet per second. They are told to
knock as soon as they get there, rather than (4 )____ time looking for a doorbell.
Work engineers are (5 )______ riding with drivers, timing everything from
stops at traffic lights, to wait at customers’ doorway, to stairway climbs, to
coffee break. And they are not (6) ______ to pointing out the occasional
inefficiency. Additionally, supervisors ride with the least good drivers, noting
how they work and constantly ( 7)______ them until their work is up to standard.
The (8) _ ____ of all this work engineering is efficiency, and UPS has been
called one of the most efficient companies anywhere. It's also a'highly profitable
company. Most drivers take the regimentation in stride: many show (9 )______
in meeting the UPS standards each day. Others, however, feel that they are
constantly being pushed, that it is impossible for them to (10)______ at work.
UPS officials claim that the standards provide accountability. And, they say,
employees who work according to UPS standards should feel less tired at the end
of the day.
51. A. fact B.ex change c. return D. short
52 A. hold B. performed c. accepted D.under
53. A. indicate B. govern c. demonstrate D. tell
54. A. wasting B.spend c. spending D. waste
55. A. consistently B. continually c. constructively D. chronicall
56. A. impolite B. brave c. intimate D. averse
57. A. scolding B. criticizing c. encouraging D. correcting
58. A. task B. reason c. object D. target
352
59. A. pride
60. A. rest
B. passion c. interest D. pleasure
B. relieve c. relax D. restrain
VI. RE AD IN G CO MPR EH EN SION (10 PTS ):
Read the passa ges and choose the b est answer.
PASSAGE 1
A major American bank is being sued by the US authorities for allegedly
discriminating against its female employees. It has been alleged that women in
the firm do not have as good promotion prospects as the men and that their
salaries are not on a par with those o f their male counterparts.
Sexism in the workplace is against the law in America and in many other
countries. However, there are still obstacles in many firms which prevent women
from achieving the promotion that their talents and qualifications deserve. Many
women are promoted quite rapidly through the junior jobs in a firm, but face the
gla ss ce ili ng , when they apply for any of the most senior posts. So it is that,
although there are many women in middle management positions in many firms,
there are very few in top management posts.
Many women blame this situation on the old-boy network. Others see it as
evidence that many men are resistant to change and are still chauvinists at heart,
while some may feel that failure to promote women to top jobs is a result of
feeling of insecurity in the men who are making the appointments. Perhaps they
are afraid of letting women become too powerful.
Not many years ago the power which women have today would have seemed
an impossible dream to many women. Before the rise of the wo men’s movement
there were no such things as equal rights or equal opportunities for women. For
the most part, women were expected to get married and then be responsible for
childcare and for carrying out all the household tasks. If they worked, it was
probably in a part-time, often rather menial, job.
Feminists saw this as sexism and a waste of wome n’s talents, and set out to
change things. Although some people, women as well as men, now do not have a
high regard for feminism, women owe to feminists many of the improvements in
their work situation. Without the m, there would be no positive discrimination,
no job-sharing and no parity with men in the workplace.
61. Which of the following statement is NOT true, according to the passage?
A. Women can easily be appointed to high management positions nowadays.
B. In spite o f their ability and qualifications, women are hard to be promoted,
c. Discrim ination against women in the workplace is illegal in America.
D. There are very few women who hold top management posts in many firms.
62. The pronoun “them” in the penultimate line of the last paragraph refers to ____ .
A. women B. men c. people D. feminists
63. What does “the glass ceiling” used in the second paragraph mean?
A. the boss es’ indifferent attitude towards women in the workplace.
353
B. the unsta ted thi ng th at pre vents w om en from gettin g a prom oti on,
c . the law th at does not allow fe ma le wo rke rs to get p rom ote d.
D. the top inside sur fac e of a room tha t is tr ansparent a nd inv isible .
64. The phrase “on a p ar wi th" a s used in th e first par agr aph can be bes t rep laced
b y ______ .
A. so low as B. better than c. comp are d with Đ. equ al to
65. It can be inferred from the pas sage th a t______ .
A. havin g po we r is still an imp oss ible dream for many wo me n nowadays.
B. wom en in the past could not do any jobs whether they were part-time or menial,
c. since the w om en's liberation movement, women have had equal opportunities.
D. wom en's curren t tasks include taking care o f children and doing housew ork.
66. The word “p ari ty” as used in the last paragraph is closest in me aning t o ___?
A. relati onship B. comp etition c. equal ity D. responsib ility
67. Wh at cou ld be the b est title of the p assage?
A. Sex Equality B. Ge nder discrimin atio n
c. Men and Wo men D. Job s for W omen
68. In wh ich par agr aph does the writer give an exa mp le of a busin ess tha t
comm its se xism?
A. Par agr aph 3 B. Paragraph 2 c. Par agraph 1 D. Parag rap h 4
69. The wo rd “th is" in the last paragraph refe rs to the fact that w om en ______ _.
A. wasted the ir talent s and time doing unimportant jo bs.
B. cou ld not do f ull-tim e jobs , ju st hous eho ld chores,
c. exp ected to get married and give birth to childre n.
D. fou ght for the ir equal rights and e qual opportunitie s.
70. Which of the follow ing is NO T mentio ned as a reason for sex ism at the
wo rkp lac e?
A. Male cha uvi nism B. M en 's feelings of insec urity
c. W om en's we akness D. T he old-boy net wo rk
PASSAGE 2
Over the last cen tury the world has become inc rea singly sm aller. No t
geo gra phically, of cou rse, but in the sense tha t media, tec hnolo gy and the
ope nin g of borde rs has ena bled the world's citi zen s to view, sha re and gain
acc ess to a much wider range of cultures, soc ieti es and wo rld vie ws. In this
me ltin g pot tha t the world has become, toda y’s child is pr iv y to facets of the
hum an exp erienc e tha t his imm ediate pre dec ess ors had no ink ling eve n existed.
It stands to reas on tha t in order, to absorb, con figure and fina lly form opinions
about this inform ation- lad en planet, childre n must be supplie d with cer tain tools.
Included in this list o f ‘too ls' are: edu cation, social skil ls, cultural aw are ness and
the acquisition of languages, the mo st imp ortant of these being the latter. Until
recently, a child who had the ability to spe ak more than one lan gua ge would
have been consi der ed a very rare entity. This one-language phenom enon could
354
be attributed to a combination of factors. Firstly, the monolingual environment
in which a child was raised played a strong role, as did the limited, biased
education of the past. With regard to immigrants, the sad fact was that nonnative parents
tended to withhold the teaching of the mother tongue so that the
child would acquire the ‘more prestigious’ language of the adopted country.
Nowadays, the situation has undergone an almost complete reversal. In the
majority of North American and European countries, most children are given the
opportunity to learn a second or even a third language. Children acquire these
foreign languages through various and diverse means. In many countries,
learning a foreign language is a compulsory subject in the state school
curriculum. Other children rely on language schools or private tuition to achieve
their goal. In other instances, children are born to bilingual parents, who. if they
so desire, may teach the children two languages.
Bringing up one's child bilingually is not a decision to be taken lightly. Both
parents must consider long and hard the implications involved in raising a child
in a two-language home. This decision is one of those all-important choices
which will affect not only the parents’ lives but also the life of the child. Raising
a child bilingually has a two-fold effect. Firstly, o f course, the child learns the
two languages of the parents. Secondly, the parents' decision will influence
factors which will have a far-reaching effect on the child's life. Some of these
factors include: style and place of education; diameter of social circle;
employment potential and preference; and, most importantly, the way in which
the child views himself and his global environment.
One o f the more advantageous by-products of being a member of a bilingual
family is the inherent awareness of two different cultures. This bicultural child
inherits a wealth of knowledge brought about by an exposure to: historical
backgrounds; traditional songs and folklore; rituals o f marriage; models o f social
interaction; and therefore, two varying interpretations of the world. The
monolingual child seems to be at a disadvantage in comparison to the bilingual
child, who has a set of languages and an accompanying set of abstract cultural
ideas. Practically speaking, when a child comes from a two-language family, he
must be taught both languages in order to communicate with the extended family
members. When, for example, the grandparents speak a language which differs
from that of the child's locale, a monolingual child would be deprived of the
interaction which occurs between grandparents and grandchildren. On the other
hand, a bilingual child will not only be able to speak to grandparents, but will
also comprehend where these people have Nome from’. There will be a shared
cultural empathy within the family. Because all family members can
communicate, on both a verbal and cultural level, no one will feel excluded and
the child will develop a sense o f rootedness.
On a more abstract level, it has been said that a bilingual child thinks
differently from a monolingual child. Current research in linguistics indicates
355
that there may be a strong correlation between bilingualism and cognitive skills.
This new research concerns itself with the fact that a bilingual child has two
lexical structures for any given physical or abstract entity. This leads logically to
the assumption that the child also has two associations for many words, as a
word can mean different things in different languages. For example, the word
‘fire' in many western hemisphere languages conn ote s warmth and survival. For
the bilingual child, then, vocabulary items and the abstract idea behind them are
both dual in nature and more elastic. Researchers maintain that this elasticity of
ideas may allow the child to think more flexibly and, therefore, more creatively.
71. Ill the a uthor's view, the world is becoming a _____ .
A. more culturally diverse place
B. place where only privileged children will prosper
c. less complex place to live in
D. much more integrated place
72. According to the first paragraph, which of the following was true of immigrants?
A. Children were reluctant to use their mother tongue.
B. The mother tongue was considered less important.
c. Parents encouraged children to use their mother tongue.
D. Most parents made it a priority for children to grow up bilingual.
73. The phrase “privy to" in paragraph 1 mostly m eans_____ .
A. acquainted with B. advised of c. apprised of D. in the know about
74. The phrase “ attributed to’’ mostly m ean s_____ .
A. ascribed to B. associated with
c. connected with D. held responsible for
75. According to the writer, second or foreign language learning is something__.
A. people are still apathetic towards
B. mainly associated with private sector education
c. that few people take seriously
D. about which general attitudes have evolved considerably
76. According to the article, the decision to raise bilingual children is difficult
bec aus e______ .
A. it may limit the child’s choice o f friends
B. though simple for parents, it can impact negatively on children
c. it may cause children to lose their sense of identity
D. it needs to be considered from many different angles
77. With regard to the ‘extended family’ in immigrant situations, the writer feels
it is important tha t_____ .
A. adults try to understand the child’s difficult cultural situation
B. children are not pressured to speak their parents’ native language
c. adults recognise the child’s need to identify more with local culture
D. children can relate to all aspects of their parents’ native culture
78. The word “by-products” in paragraph 4 mostly me ans_____ .
A. entailments B. knock-on effects
c. side effects Đ. spin-offs
356
79. The word “connotes” in paragraph 5 mostly means_____ .
A. underpins B. implies c. signifies D. smacks of
80. According to current research, the benefit of learning two languages is that
A. different types of knowledge can be accessed in different languages
B. bilinguals become more aware the origin of words in languages
c. it helps to develop different capabilities of the mind
D. bilinguals develop a greater sense o f the value o f culture
B. WRITTEN QUESTIONS (60 PTS)
I. OPEN CLOZE (20 PTS):
Read the passsages and fill each gaps with ONE word.
PASSAGE 1
MALNUTRITION GOALS IN AFRICA 'VERY FAR AWAY’
A new report shows that no country in Africa will meet goals (8 1 )_________
to end childhood malnutrition by the year 2030. That target was set by the
United Nations in 2015 (8 2)_________ a Sustainable Development Goal. The
UN adopted a set of goals, "to end poverty, protect the planet and ensure
prosperity for all as part of a new sustainable development agenda". The new
report is published in the journ al Nature. It identifies poor child nutrition and
(83) _____________ levels o f education across 51 African countries. These were
(84) _________ factors in countries battling to (8 5 )__________ children with
sufficient food. Researcher Simon Hay said the goal of ending childhood
malnutrition was always an "aspirational" target. He said: "This aspiration is
very, very far away."
There was some good news in the report. It highlighted the fact that many
African nations, (8 6) ________ Ghana and Nigeria, have shown (8 7) __________
of improvem ent in childhood development since the year 2000. However, it is a
different story for countries like Chad, Central African Republic and Eritrea. The
report indicates that malnutrition remained "persistently high" in 14 countries
between Senegal in the west and Somalia in the east. Many of these countries
have (8 8 )_________ war, famine and mass migration, all of ( 8 9 )__________
have put massive strains (9 0)________ _ health and agriculture. One researcher
said considerable investment was needed in health and infrastructure in order to
address "serious inequalities".
PASSAGE 2
THAILAND TO CLOSE FAMOUS BEACH TO PROTECT CORAL
Thailand will soon close one of its world famous beaches in an (91) _______
to reverse the damage done to its coral by mass (9 2) _________ . The beach is in
Maya Bay on the tiny island o f Koh Phi Phi Leh. It garnered worldwide attention
after it featured heavily in the Leonardo DiCaprio blockbuster movie The Beach.
357
The film (9 3)_________ the Maya Bay beach on the bucket lists of millions of
travellers worldwide. The result has been an influx of up to 5,000 sun
worshippers a day to its emerald shores. They arrive (9 4)_________ hundreds
of boats, which have caused irreversible damage to the bay's coral. Officials say
that most of the coral in the bay has died. The beach will close between June and
September to let the coral (9 5) _________ .
Experts predict that almost 80 per cent of Thailand's coral (9 6 )_________
have been destroyed, and that once pristine beaches have been damaged by
tourism. The deputy dean of the Faculty of Fisheries at Bangkok's Kasetsart
University is an outspoken critic of the level of tourism Thailand's coastline
(9 7)_________ to endure. He said the biggest culprits were beachfron t hotels,
boat anchors (9 8) _________ into and breaking the coral, and plastic waste being
(9 9) _________ in the sea. He believes the "ideal solution" is for Maya Bay to be
closed permanently. He said: "While it's a very pretty bay, visiting it with a
flotilla of boats and hordes of other tourists rather detract from the magic."
He welcomed the temporary (100)_________ .
II. WORD FORMATION (20 PTS)
Complete each sentence, using the correct form o f the word in parentheses.
101. British Prime Minister Margaret Thatcher called the hanging an act of
_____ which is deeply repugnant to all civilized people. (BARB ARIC)
102. Students mustn’t be _______________ and illiterate when they leave
school. (NUMBER)
103. Very soon, ‘Third World’ came t o _______________ poverty. (NOTE)
104. Scientists are convinced that, because crocodiles have such a long
_______________ , they must have a natural ability to combat infection.
(LIFE)
105. University professors both teach an d_______________ research. (TAKE)
106. Treat them with this kind of spray to make t he m_____ ______ . (WATER)
107. Steroids often help reduce th e ______________ and itching in the skin.
(FLAME)
108. An NGO is helping to make ozone-friendly n atu ral ____________ designed
to replace R-22 in existing air conditioning systems. (FRIDGE)
109. In my opinion, this book is ju st ________________rubbish. (INTELLECT)
110. He was standing in the middle, in the of the
picture. (GROUND)
Complete the passage with the appropriate forms from
the words given in
the box.
convince
search
estimate
decide
cohere perceive rhythm
argue controversy electric
358
FAST BRAIN WAVES
Over half a century ago, scientists found they could record the (1 11) _____
signals of the brain at work. What at first appeared a random hotchpotch of
activity becam e a pattern of elegant waves (112 )______ determined. Ever since,
scientists have wondered whether the secrets of our thoughts, (1 13 )______ and
even consciousness itse lf might be hidden in the patterns o f our brain waves.
The question of why we have brain waves is, (11 4)______ , as hotly debated
today as it was when the patterns were discovered. But the meaning, and even
the existence, of fast rhythms in the alert brain is highly (115 )______ .
What is problematic is that you can ’t perceive these rhythms directly, they are
so well hidden in the noise created by other brain activity, but many (116)
' now hold the (117) • that the significance of these brain waves
should not be (1 Ỉ
,
8)
The latest suggestion is that the rhythms could be (1 19 )______ in detecting
processes going on in different regions of the brain. Some believe that these
rhythms might even interact, and in doing so help the brain to package
information into (1 20)______ thoughts. How we bring together these related
signals in the brain is a puzzle as yet unresolved.
III. ERROR CO RRECTION (10 PTS):
Identify and correct 10 errors in the passage. Find and correct them.
Underline the mistakes and write the answers in the blanks provided.
CALLS FOR EVERYONE IN ENGLAND TO SPEAK ENGLISH
(19TH MARCH, 2018)
A top British official has said the UK government should set a targeted date
for everybody in England to speak English. Dame Louise Casey is an expert on
social welfare and community interaction. She said a "common language" would
help to "heal rifts across Britain". Ms Casey has been a long-time criticism of
successive governments, whom she deems have failed to focus on integration in
an ever-increasing multicultural Britain. She said politicians had continually
failed to keep up with the "unprecedented pace and scale of immigration" over
the past decade. She said that many communities were becoming increasingly
divided, and a lack o f ability in English was a key factor to creating division.
The UK's Communities Secretary, Sajid Javid, revealed that 770,000 people
who live in England neither speak no English whatsoever or hardly any. He
warned that up to 70 per cent of those whose English skills were lacking in were
women, mostly from Pakistani and Bangladeshi communities. He said these
women were at great risk from equality and discrimination. Ms Casey said:
"Everybody of working age and of school age should be able to speak one
language, and I think the public in particular would feel some,relief." Opponent
of Ms Casey's views say England should be proud to be a multi-lingual country
and not to force people to learn English if they have no desire to.
359
3 .________________
4.
7.
_______________ 2 ._______________
_______________ 5 .________________
_______________ 8 ._______________
6 .________________
9. ________________
10. ______________
IV. SENTENCE TRANSFO RMATION (20 PTS)
Complete the second sentence so that it has a similar meaning to the first
sentence, using the word given. Do not change the word given. You must use
between three and eight words, including the word given.
131. The journalis t pretended that she was a parent of one of the children, false
The Journalist ................................................................... ............................. .
that she was a parent o f one o f the children.
132. You have to use logic and lateral thinking in equal measure in this job. strike
You have to ......................................................................................................
logic and lateral thinking in this job.
133. Being inexperienced was a disadvantage to her when she applied for
promoton. counted
H e r.......................................................................................... when she applied
for promoton.
134. The man over there lost his tem per in the public meeting, handle
T h at's............................................................................ in the public meeting.
135. We honestly all found it almost impossible not to laugh when we saw
Josh's new haircut, face
Honestly, ............................................................................... .......................
almost impossible when we saw Josh ’s new haircut.
136. The phone bill was so expensive that Dave was furious, arms
Dave was ........... ............................................................................................... .
an expensive phone bill.
137. ‘I’ve decided I really want to go on a cruise round the Med this summer,’
said Molly, heart
Molly says ............................................................................................................
on a cruise round the Med this summer.
138. He told US that getting a later plane was our only option, but
‘Y ou................................................................ ................a later plane,’ he said.
139. Inefficient treatment of customers creates a bad inpression of the company,
reflects
Treating customers with a la ck .....................................................the company.
140. The organizers planned everything as carefully as they could possibly have
done, utm ost
Everything was p lann ed .......................... .............................. by the organizers.
360
TRƯỜNG THPT CHUYÊN LƯƠNG VÃN CHÁNH ■ PHÚ YẼN
A. MULTIPLE CHOICE
I. WORD CHOICE (5ms)
Choose the word which best completes each o f the following sentences.
1. Many people have written to the newspaper to show t he ir______________ of
such behavior.
A. antipathy B. abhorrence c. aversion D. animosity
2. The regular appearance of sex and violence on television undoubtedly has a
____________ influence on teenagers.
A. pernicious B. parsimonious c. precipitous D. propitious
3. It is disquieting that there has been an increased____________ of cancer near
the nuclear power station.
A. occurrence B. âccidence B. persistence D. incidence
4. “The Song o f Ice and Fire’' is the first volume o f a series, wh ich ___________
six books in all.
A. assembles B. totals c. compiles D. amasses
5. From this year on, under-fives will get tra vel_____________ , which certainly
makes huge savings for family trips.
A. consolations B. concessions c. contortions D. constrictions
6. Unfortunately, his report does not ■ with what we have learnt from
other sources.
A. ally B. pally c. tally D. rally
7. H er _______ _____display of tears at work did not impress her new boss, who
felt she should try to control her emotions.
A. maudlin B. meritorious c. precarious D. plausible
8. The authority is going to great lengths t o _________ war on dangerous driving.
A. launch B. battle c. stage D. wage
9. China fueled fears that it s__________ economy is about to slow further after
Beijing cut its main interest rate by 0.25 percentage points.
A. diseased B. ailing c. sickening D. unwell
10. Robert is a well-behaved child, but his younger brother is a bit o f a(n) _____
A. Eyeful B. skinful c. armful D. handful
11. GRAMM AR AND STRUCTURES (5ms)
Choose the word or phrase that best completes o f the following sentences.
1. Three candidates will be short-listed for the post but we do not know ........
■ A. whom B. those c. which D. what ones
2. In this instance, there is some d isagre ement.................Dubai is in breach of
international law. ■
A. as to whether B. whether c. as if D. as w hethe r or not
361
3. After a while, the flat was silent again. The baby had cried hi m se lf.........His
mother sighed with relief.
A. sleeping B. to sleeping c. into sleeping D. to sleep
4. Naturally, there is a great deal of anxiety about o ng oing ..................in the area.
A. jobs'losses B. job's losses c. job losses D. jobs lost
5. — These are the screws you're looking for. - ................ ! You have really
sharp eyesight !
A. So are they B. It's so c. So they are D. So are there
6. Don't worry, Albert.............., you can depend on me. I'll never let you down.
A. CGmes what comes B. Come what may
c. Whatever it comes D. Comes what might
7. ... the governm ent's pre-election promises, taxes have been raised yet again.
A. Despite of B. For all c. But for D. Regardless
8. They had.................. that it was invisible from the road.
A. so designed it B. designed it to be
c. designed it such as D. designed it like
9. Most ideas ne ve r.......... the top brass, being quietly shelved by middle managers.
A. rise to B. make it to c. do with D. reach up to
10......................, I haven't seen Catherine all day.
A. Come to think of it B. To come to think o f it
c. Coming to think it D. Coming to think o f it
III. PREPOSITIONS AND PHRASAL VERBS (5ms)
Choose the word which best completes each o f the following sentences.
1. Our teacher tends to..............:.... certain subjects which she finds difficult to
talk about. •
A. boil down B. string along c. skate over D. track down
2. I was so tired that I just. ...............in the armchair.
A. flaked out B. broke up c. dropped out D. fell over
3. He could n’t have been very hungry. He just.................. ..at his food.
A. worked B. nibbled c. got D. marked down
4. Simon hasn’t got a job, and isn’t trying to get one: he jus t............. his friends.
A. meddles with B. drags on c. sponges on D. tarts up
5. At first, the managing director insisted that he was right and everyone else was
mistaken, but in the end was forced to.................
A. ease off B. gulp down c. scrape through D. climb down
6.1 need twenty pounds t o............... me.............until the end of the month.
A. turn in B. tide over c. dig into D. dry up
7. You must try not to ............... your brother’s death.
A. roll on B. make out c. dwell on D. wipe out
8. The patient was very ill indeed. The doctors were w orking............ time.
A. against B. with c. on D. for
9. The boxer lost the fight because he hit his op ponent............... the belt.
A. at B. by c. around D. below
362
10. 1 telt pleased 1 no longer had the responsibility. It was a weight.....................
my shoulders.
A. on B. with c. off D. from
IV. COLLOCATIONS AND IDIOMS (5ms)
Choose the word which best completes each of the following sentences.
1. The last candidate for the managerial position is a pretty unk nown_________ .
A. quality B. quantity c. quandary D. qualification
2. The couple tied the knot two months after they met - it was indubitably a
_______ _ romance.
A. typhoon B. hurricane c. whirlwind D. cyclone
3. The introduction of new immigration laws is th e ___________ of the town
these days.
A. gossip B. story c. talk D. tale
4. Although it was initially regarded as a ________________ , Leicester City
finished 2014-2015 Premier League being the champion.
A. rank interloper B. rank outsider c. rank newcomer D. rank foreigner
5. I don't expect the entry test this year will be more demanding, bu t,______
1 do n't think it will be simpler either.
A. over and above B. on the side
c. by the same token D. for good measure
6. 1 love hanging out with Mary - she is really a __________ of sunshine
A. beam B. ray c. glow D. light
7. The teacher, by dint of draconian disciplinary measures, h a s ___________
terror among his students.
A. stimulated B. inspired c. engaged D. stirred
8. The new superconductors look set to ____________ a useful niche for
themselves in the world's electrical industries.
A. etch B. incise c. carve D. lacerate
9. Many people, for all the repeated warnings, re m ain _______________
ignorant of the danger o f using the mobile phones at night.
A. blissfully B. delightedly c.jubilantly D. ecstatically
10. The authority should not have made those cutting comments, no matter how
you _____ it.
A. cut B. slice c. chop D. clip
V. READING COMPREHENSION. (10ms)
READING 1
Read the following passage and choose the correct answer to each of the
questions.
CLINICAL DEPRESSION AND ANTIDEPRESSANTS
Clinical depression, or Major Depressive Disorder (MDD), is clinically
defined as a period of sadness or melancholia severe enough to interrupt the
patterns of one 's everyday life and lasting for a period of several weeks or
363
longer. Although there has been an awareness of clinical depression for
thousands of years, an understanding of its causes, and effective means of
treatment, have only come in the last fifty years.
Clinical depression is an extremely widespread illness, with an estimated 16%
of the population suffering from an episode of MDD at least once in their
lifetime. Currently, clinical depression is the second leading cause of disability
in the United States behind heart disease. Traditionally, females have reported
higher rates of clinical depression than males, but this is thought to be due to the
greater stigm as against expressing one’s feelings that are typical among males in
most cultures rather than to any true difference in the occurrence of MDD among
the two genders. Sufferers of clinical depression may exhibit an array of
symptoms, some of the most common being increased apathy, disruptions in sleep
and eating patterns, a withdrawal from pleasurable activities, and suicidal ideation.
While not completely understood, clinical depression is known to have both
environmental and biological causes, and the most effective treatments involve a
two-pronged approach of medication and emotional therapy. The mean age for
the onset of clinical depression is in the mid to late twenties, and the pressure of
transitioning to independent adult life is often as the primary trigger mechanism.
While specific bou ts of clinical depression may be attributable to a certain event
or trauma in a people’s life, neurochemistry, which is largely genetic, seems to
predispose some people to MDD. Clinical depression has been linked in
numerous studies with imbalances in neurotransmitters, the chemicals that relay
information between cells in the brain.
Medication to treat clinical depression first became available in the 1950s,
and today there are two majors classes of widely prescribed antidepressants:
selective serotonin reuptake inhibitors (SSRIs) and serotonin norepinephrine
reuptake inhibitors (SNRIs). Both classes of drugs operate by limiting the
absorption (reuptake) of neurotransmitters, thereby increasing their levels in the
brain, but SSRIs operate exclusively on serotonin, while SNRIs effect
norepinephrine as well, making them slightly more effective. Both classes of
drugs take several weeks to build up to effective levels in the bloodstream, and
their effectiveness may be com pro mis ed by the use of alcohol or other
medications. For this reason they are carefully controlled by psychiatrists. Used
alone, these antidepressants have proven to be about as effective as counseling,
with reductions of depressive symptoms averaging about 55% and full remission
at about 22%, but when used in conjunction with counseling they often result in
reduction and remission rates closer to 85% and 45% respectively. Furthermore,
continued use of antidepressants after the conclusion of the counseling has been
proven to greatly decrease the risk of relapse.
As effective as antidepressants have proven in treating clinical depression,
they are not without drawbacks and criticisms. Typically, the first few weeks of
364
treatment are when a depressive patient is at the greatest risk of suicide. There is
some evidence that during this period, antidepressants may actually contribute to
this risk because they reduce mental lethargy, and a more active mind may lead
to increased suicidal ideation. Thus, it is critical that a patient be closely watched
during the first few weeks of an antidepressant regimen. A further criticism of
antidepressants is that they may have unpleasant sexual side-effects which may
make them unpalatable to patients, given the long duration of their intended use. A
final criticism concerns their role in the ever growing field of psychopharmacology.
Antidepressants are by far the most commonly prescribed of
psychiatric drugs, and some mental health professionals worry that their profusion
is leading to an over-reliance on chemical solutions to emotional issues.
1. The passage discusses all of the following aspects of clinical depression
EXCEPT .
A. its characteristic symptoms
B. its official medical definition
c. methods of counseling used to treat it
D. available medications for treatment
2. The word “stigmas” in the passage is closest in the meaning t o __________ .
A. cultures B. taboos c. preventions D. inabilities
3. According to paragraph 2, the actual rates of clinical depression are most
likely ______ .
A. much higher than they were in the past
B. quickly catching up with those of heart disease
c. about the same between men and women
D. impossible to accurately calculate
4. According to paragraph 3, the pressures of becoming an independent adult
often_______ .
A. are the root causes of clinical depression
B. are irrelevant to clinical depression
c. precede the onset of clinical depression
D. only serve to increase the symptoms of depression
5. The word “bouts” in the passage is closest in meaning to ___________ .
A. symptoms B. episodes c. types D. diagnoses
6. Based on the information in paragraph 3, it can be inferred that clinical
depression is ___________ .
A. largely an untreatable condition
B. often a hereditary condition
c. entirely chemical in nature
D. often accompanied by other mental problems
7. The word “compromised” in the passage is closest in meaning to ________ .
A. negotiated B. questioned c. altered D. endangered
365
8. In paragraph 4, why does the author discuss rates of reduction and remission
in clinical depression?
A. To better demonstrate the efficacy of antidepressants when used with
counseling.
B. To suggest that medication is secondary in importance to counseling in
treating depression.
c. To discuss the chances for full recovery for people who suffer from
clinical depression.
D. To illustrate the vast superiority of SNRIs to SSRIs in the treatment o f
clinical depression.
9. According to paragraph 5, antidepressants may lead to an increased risk o f
suicide in the first weeks o f treatment because.....
A. that is the time when their side-effects are strongest.
B. they may stimulate an overproduction of serotonin.
c. they increase mental activity, leading to more thoughts o f suicide.
D. they draw attention away from much needed counseling.
10. Why do some mental health workers oppose the overuse of antidepressants ?
A. They fear that their side effects are too poorly understood.
B. They feel the benefits o f antidepressants are overstated.
c. They know that antidepressants do not prevent the risk o f relapse.
D. they fear that the reliance on antidepressants draws away from counseling.
READING 2
Read the following passage and mark the letter A, B, c or D to indicate the
correct answer to each of the questions,,
No longer is asthma considered a condition with isolated, acute episodes of
bronchospasm. Rather, asthma is now understood to be a chronic inflammatory
disorder of the airways-that is, inflammation makes the airways chronically
sensitive. When these hyper-responsive airways are irritated, airflow is limited,
and attacks of coughing, wheezing, chest tightness, and breathing difficulty occur.
Asthma involves complex interactions among inflammatory cells, mediators,
and the cells and tissues in the airways. The interactions result in airflow
limitation from acute broncho-constriction, swelling of the airway wall,
increased mucus secretion, and airway remodeling. The inflammation also
causes an increase in airway responsiveness. During an asthma attack, the patient
attempts to compensate by breathing at a higher lung volume in order to keep the
air flowing through the constricted airways, and the greater the airway limitation,
the higher the lung volume must be to keep airways open. The morphologic
changes that occur in asthma include bronchial infiltration by inflammatory
cells. Key effector cells in the inflammatory response are the mast cells,
T lymphocytes, and eosinophils. Mast cells and eosinophils are also significant
participants in allergic responses, hence the similarities between allergic
366
reactions and asthma attacks. Other changes include mucus plugging of the
airways, interstitial edema, and microvascular leakage. Destruction of bronchial
epithelium and thickening of the subbasement membrane is also characteristic.
Il l addition, there may be hypertrophy and hyperplasia of airway smooth muscle,
increase in goblet cell number, and enlargement of submucous glands.
Although causes of the initial tendency toward inflammation in the airways of
patients with asthma are not yet certain, to date the strongest identified risk
factor is atopy. This inherited familial tendency to have allergic reactions
includes increased sensitivity to allergens that are risk factors for developing
asthma. Some of these allergens include domestic dust mites, animals with fur,
cockroaches, pollens, and molds. Additionally, asthma may be triggered by viral
respiratory infections, especially in children. By avoiding these allergens and
triggers, a person with asthma lowers his or her risk of irritating sensitive
airways. A few avoidance techniques include: keeping the home clean and well
ventilated, using an air conditioner in the summer months when pollen and mold
counts are high, and getting an annual influenza vaccination. Of course, asthma
sufferers should avoid tobacco smoke altogether. Cigar, cigaiette, or pipe smoke
is a trigger whether the patient smokes or inhales the smoke from others. Smoke
increases the risk of allergic sensitization in children, increases the severity of
symptoms, and may be fatal in children who already have asthma. Many of the
risk factors for developing asthma may also provoke asthma attacks, and people
with asthma may have one or more triggers, which vary from individual to
individual. The risk can be further reduced by taking medications that decrease
airway inflammation. Most exacerbations can be prevented by the combination
of avoiding triggers and taking anti-inflammatory medications. An exception is
physical activity, which is a common trigger o f exacerbations in asthma patients.
However, asthma patients should not necessarily avoid all physical exertion,
because some types of activity have been proven to reduce symptoms. Rather,
they should work in conjunction with a doctor to design a proper training
regimen, which includes the use of medication.
In order to diagnose asthma, a healthcare professional must appreciate the
underlying disorder that leads to asthma symptoms and understand how to
recognize the condition through information gathered from the patient's history,
physical examination, measurements of lung function, and allergic status.
Because asthma symptoms vary throughout the day, the respiraĩồry system may
appear normal during physical examination. Clinical signs are more likely to be
present when a patient is experiencing symptoms; however, the absence of
symptoms upon examination does not exclude the diagnosis of asthma.
1. According to the passage, what is the name for the familial inclination to have
hypersensitivity to certain allergens?
A. interstitial edema B. hyperplasia c . hypertrophy D. atopy
367
2. Why does a person suffering from an asthma attack attempt to inhale more air?
A. to prevent the loss of consciousness
B. to keep air flowing through shrunken air passageways
c. to prevent hyperplasia
D. to compensate for weakened mast cells, T lymphocytes, and eosinophils
3. The word triggered is closest in meaning to
A. Caused B. taken place c. eliminated D. cured
4. The passage suggests that in the past, asthma was regarded as which of the
following?
A. a result o f the overuse o f tobacco products
B. a hysterical condition
c. mysterious, unrelated attacks affecting the lungs
D. a chronic condition
5. Which of the following would be the best replacement for the underlined word
exacerbations in this passage?
A. allergies B. attacks c. triggers D. allergens
6. The passage mentions all of the following bodily changes during an asthma
attack except
A. severe cramping in the chest. B. heavy breathing,
c. airways blocked by fluids. D. constricted airways.
7. Although it is surprising, which of the following triggers is mentioned in the
passage as possibly reducing the symptoms of asthma in some patients?
A. using a fan instead o f an air conditioner in summer months
B. exposure to second-hand cigarette smoke
c. the love of a family pet
D. performing physical exercise
8. Why might a patient with asthma have an apparently normal respiratory
system during an examination by a doctor?
A. Asthma symptoms come and go throughout the day.
B. Severe asthma occurs only after strenuous physical exertion,
c. Doctor's offices are smoke free and very clean.
D. The pollen and mold count may be low that day.
9. Who might be the most logical audience for this passage?
A. researchers studying the respiratory system
B. healthcare professionals
c. a mother whose child has been diagnosed with asthma
D. an antismoking activist
10. What is the reason given in this article for why passive smoke should be
avoided by children?
A. A smoke-filled room is a breeding ground for viral respiratory infections.
B. Smoke can stunt an asthmatic child's growth.
c. Smoke can heighten the intensity of asthma symptoms.
D. Breathing smoke can lead to a fatal asthma attack.
368
VI. CLOZE TEST (10ms)
CLOZE TEST 1
Read the following passage and decide which answer (A, B, c, or D. best fits
each gap.
THE YOUNG VIOLINIST
Chloe Hansip is (1)_________ for a busy week; today she (2)_________her
first album, then she flies off to play the violin in Vienna - and on Friday she
celebrates her 14th birthday. The child prodigy has been playing since she was
two. And in five years since, she has been (3)__________ with praise by some o f
the greatest (4)__________ in classical music.
Chloe has played to enraptured audiences in places as (5)_________afield as
Kyoto in Japan and St Petersburg in Russia, (6)__________numerous awards
and even appeared in a film. These may be (7)_______distinctions, especially
for a 13-year-old, but Cloe is decidedly down-to-earth (8)_________ .
Like any other teenager, she assiduously keeps in (9)______contact with her
best friends through text messages, and she admits to have a soft (10)_________
for one or two o f the pop groups the typical British teenager is keen on.
1. A. fixed B. assured c. promised D. set
2. A. unveils B. launches c. inaugurates D. publishes
3. A. showered B. sprinkled c. sprayed D. scattered
4. A. numbers B. figures c. digits D. variables
5. A. long B. distant c. far D. way
6. A. carried out B. carried on c. carried off D. carried ov
7. A. irregular B. rare c. sparse D. scarce
8. A. in all modesty B. all the same c. in all fairness D. at all time
9. A. close B. narrow c. intimate D. cordial
10. A. taste B. affection c. room D.s pot

CLOZE TEST 2
Read the following passage and decide which answer (A, B, c, or D. best fits
each gap. ,'
Cairo, despite its contrasts, is.similar to any other huge (1 )...............jungle
with its high-rise tower blocks and evidence of urban development. It is a
melting (2) ............ of people from all over Africa and, in parts, resembles a
huge marketplace with little space to move, as street vendors and customers
(3)............ over the price o f goods. Cairo, however, is by no means a cheap city
to visit, and accommodation, while not costing the ( 4 )........... , can be expensive.
This is especially true if you want to stay in one o f the hotels along the banks of
the River Nile where there are long ( 5 )...........o f unspoilt beauty. (6) ............in
Cairo is (7)............... easy and cheap, as admission to most cities is well
(8)............... the average tourist's (9 )..............All visitors to Cairo take pleasure
in experiencing the city and its sounds, and this is best done on foot. (10)...........
cars, as this city o f almost 16 million suffers from chronic traffic congestion and
369
the inevitable smog which motor vehicles help to produce. For a trip to the
Pyramids, it is better to catch a bus, provided you can put up with the crowded
and noisy conditions, as taxis can be expensive.
LA . concrete B. brick c. stone D. wooden
2. A. box B. can c. pot D. pan
3. A. bend B.rake c. flick D. haggle
4. A. earth B. world c. globe D. money
5. A. layers B. areas c. stretches D. line
6. A. Viewing B. Sightseeing c. Journeying D. Sighting
7. A. loosely B. significantly c. relatively D. tolerably
8. A. away B. within c off D. up
9. A. economics B. cost c.ch arge D. budget
10. A. Utilize B. Avoid c. Abandon D. Forget
B. WRITTEN TEST
I. OPEN CLOZE TEST (20ms)
OPEN CLOZE TEST 1
Fill each of the numbered blanks in the following passage. Use only ONE
word in each space.
Unicorns, dragons and centaurs feature (1)________ in the world of Harry
Potter, and while JK Rowling wasn’t the founding (2)________of these magical
beasts herself, she certainly gave age-old myths a new (3)________ of life. She
re-imagines the phoenix, a bird from Greek (4)________, in her second Potter
book. The Chamber of Secrets. It's a swan-sized scarlet bird with a gold beak
and talons, who endearingly looks a bit like a half-plucked Christmas turkey
when it gets near its 'Burning Day' - the day the bird extinguishes and is reborn
as a new, baby phoenix from its ashes.
What JK Rowling (5)_______ invent is the idea that phoenix feathers can
feature as constituent (6)_______ of a wizard’s wand, such as those of Harry
and Lord Voldemort. A phoenix’s tears can also heal wounds, as Fawkes,
Professor Dumbledore’s bird, does to Harry after he falls (7)________ to the
Basilisk in the second book. (8)________difficult to domesticate, they can
nevertheless become a loyal pet for life; when Professor Dumbledore is
(9)________to the sword later in the series, Fawkes sings the ‘Phoenix Lament’
- a mournful song so heartbreaking that it feels like it comes from inside the
listener. There also exists a book manuscript from the 13th Century showing a
phoenix rising from its ashes, proving that JK Rowling’s predecessors were
possessed o f (10)________ vivid imaginations.
OPEN CLOZE TEST 2
Fill each of the numbered blanks in the following passage. Use only ONE
word in each space.
If you were to hazard a (1)...................... at when and where the next major
earthquake will strike , there's a chance that you'd be almost as (2 )............... as
370 Í
the experts. The science of earthquake prediction is still in its (3 )....... .. which is
uncomfortable, considering the threat (4 )............... to human civilization . Even
with vast resources at their ( 5 )............... . often the best scientists can do is say
that the (6 )............... are that an earthquake will strike where one (7 ).................
before . One reason the (8) ............... of making an accurate prediction are so
low is the nature of the forces involved. It seems to be impossible to tell the
difference between a small earthquake and a warning ( 9 ) ......... Scientists need
to decide whether to put lives at risk by not issuing a warning, or constantly issue
warnings about the threat of an earthquake that may be (10) ................ In the
end , the odds seem to be against US solving this difficult problem any time soon.
II. WORD FORMS. (20ms)
WORD FORMS 1
Complete the sentences by changing the form o f the word in capitals.
1. T h e________________ (PERVADE) reservation and donation system too
adds to the youths' inquietude.
2. Cattelan himself, tall an d__________ ___ (SATURN) in slim jeans and a
salmon-colored shirt, was on hand for last-minute activities on the day before
the press opening.
3. Sadly, Vietnamese U-23 team's tireless efforts.to win the AFC Championship
were_______________ (AVAIL).
4. The splendid onomatopoeia of "hoary roaring sea" reminds US how
well____________ (SOUND) and alliteration work throughout the poem.
5. This recipe is ____________ (FOOL) - it works every time.
6. I must admit that it is time the organizers did away with th e_____________
(ANNUAL) computer system and bought a new one.
7. The philanthropist does not do any thing________________ (HEART), and
that means a lot to her to help others.
8. ____________ (GET) candidates who actively contribute to the development
of the company stand a better chance o f being recruited.
9. The leader's second ____________ (TURN) on the matter of the keynote
presenter met with strong disapproval from other group members.
10. Avid descriptions of sophisticated technologies gave the science-fiction novel
"The War of the Worlds" a sense o f_______________ (SIMILAR).
WORD FORMS 2
Choose and write the correct form of the words in the box to finish the
following passage.
astronomy polar center sphere
analog align guard distance
371
Not only was William Gilbert in the (1)_________ of the 17th-century science
movements but he also questioned the traditional (2)_____________ beliefs.
Despite being a Copernican, he didn’t express in his (3)________ beliefs whether
Copernicus's (4)_____________ model was true or not. However, he believed
that stars are not (5)________ from the earth but have their own earth-like planets
orbiting around them. The earth itself is like a giant magnet, which is also why
compasses always point north. They spin on an axis that is in (6)________ with
the earth's polarity. He even likened the (7)_____________ of the magnet to that
of the earth and built an entire magnetic philosophy on this (8)_____________ .
In his explanation, magnetism is the soul of the earth. Thus a perfectly
(9)__________ Iodestone, when aligned with the earth 's poles, would wobble all
by itself in 24 hours. Further, he also believed that the sun and other stars wobble
jus t like the earth does around a crystal core, and speculated that the moon might
also be a magnet caused to orbit by its magnetic attraction to the earth. This was
perhaps the first proposal that a force might cause a (10)_ ________ orbit.
III. ERROR IDENTIFICATIO N (10ms)
The passage below contains 10 mistakes. Identify and correct them.
Hardly a day goes by without a fresh demonstration of the ignorance of
America's first video generation. Illiteracy is growing, and a new poll shows
what a quarter of university students have no idea when Columbus reached
America, leading pessimistic academics to prophesy a new barbarism.
Some institutions, at least, have from now been presumed to be above decay. It
was imagined, for example, that they were still reading books in the English
faculty at Harvard. But that illusion, too, has been shattered by Professor Sven
Birkerts, aging 38, who teaches Creative Writing to undergraduates there.
“Mostly none of my students reads independently,” he says in a powerful lament
that has jus t been published by Harvard Magazine. Every year, he says, he
conducts the survey among his students, and the responses are heartbroken.
The Harvard undergraduates, who have enrolled to study the arts of expository
writing under Professor Birkerts are, to put it not higher, reluctant readers.
“The printed page taxes and wearies them. They find few pleasure there.
What hope does a teacher have for getting them to write? initially, I confess, I
always despair. I read through their first papers, so neatly word-proce ssed ... but
my heart sinks”, he writes, adding: The writing is almost always flatting,
monotonous prose.
1.
4.
7.
_______________ 2 ._______________
_______________ ■ 5 ._______________
_______________ 8 ._ _ ____________
3 ._______________
6 ._______________
9. _______________
10. ______________
372
IV. SENTENCE TRANSFORMATION. (20 ms)
Rewrite the following sentences with the given words in such a way that the
second sentence has the same meaning as the first one. Do not change the
form o f the words in brackets.
1. She seems to find the way Tom behaves more a source of amusement than
embarrassment.
Far from ...............................................................................................................
2. They declared war on the pretext o f defending their territorial rights.
The exc use ............................................................................................................
3. She passed the word around that she was looking for a flat.
She le t................................... ...............................................................................
4. They will try Abrams for murder at the High Court next week.
Abrams will s tand................................................................................................
5. You may be disqualified if you do n't obey the regulations.
Failure...................................................................................................................
6. It is extremely important that the company rules are observed. UTMOST
Ob servation...........................................................................................................
7. Although it was not his fault, the personnel manager resigned from his
position. HANDED
Thr ou gh ...............................................................................................................
8. Frankly speaking, I do not think that nature and nurture are of equal
significance. SUBSCRIBE.
To put............................................................. ....................................................
9. The business is hardly making a profit at the moment.
The bu sin ess.................................................................. ....................................
10.1 do n't really know why, but 1 don 't trust him..FINGER
I c a n 't ..................................................................................... I don’t trust him.
11. I felt vaguely that something was wrong, but what was it? BACK
1 ...............................................................................................but what was it?
TR ƯỞ NG THPT CHUYÊN LÝ Tự TR ỌNG ■ CẦ N THƯ
A. MULTIPLE CHOICE
I. PHONOLOGY (lOpts)
Choose the word whose underlined part is pronounced differently from the
others.
1. A. picturesquely
2. A. phlebitis
3. A. macabre
4. A. paranoia
5. A. absinth e
B. antiqueness
B. wily
B. chiromancy
B. papist
B.s oothe
c. querulously
c. poliomyelitis
c. chemical
c. saffron
c. loathe
D. discotheq ue
D. philately
D. chivalrous
D. travail
D. wreathe
373
Choose the word that lias the stress pattern different from that of the others:
6. A. euphemism
7. A. matriarchy
8. A. disinter
9. A. acupuncturist
10. A. entrepreneur
B. phenotype
B. epileptic
B.cadaver
B. vociferously
B. anonymity
c. veracity D. cantaloupe
c. rutherfordium D. diplomatic
c. cathedral D. attorney
c. antithesis D. extraordinary
c. representation D. encyclopedia
II. VOCABULARY (lOpts):
Choose the correct word or phrase A, B, c or D to complete each sentence.
1. His strange behaviour aroused th e_____ of police.
A. doubt B. suspicion c. disbelief D. notice
2. When his parents are away, his oldest brother_____ .
A. knocks it off B. calls the shots c. draws the line D. is in the same boat
3. After the concert, everyone had to _____home through the thick snow.
A. trudge B. tread c. trace D. trickle
4. Alaska boasts several climates due to its _____ mountains, warm ocean
currents, and frozen seas.
A. adept B. pious c. ghastly D. lofty
5. When 1 finish writing this composition, I'm going t o _____ and go to bed.
A. make time B. hit the hay c. hit the big time D. call it a day
6. Certain kinds o f plants can grow only i n_____soil.
A. loose B. slack c. sparse D. scarce
7. Joh n's got v ery_________ feelings about taking on more responsibility at the
moment.
A. puzzled B. mixed c.jumbled D. muddled
8. Paul's been in A lice's b ad _____ever since he offended her at the party.
A. eyes B. books c. likes D. treats
9. The car w as _____speed.
A. gathering B. collecting c. consuming D. firing
10. They're having serious problems. Their relationship is on th e_____ .
A. cliffs B. rocks c. stones D. grass
III. STRUCTURES AND GRAMMAR (lOpts)
1. ._____earning all that money if you don't have time to enjoy it?
A. is it to the good B. What's the good o f
C. Is it all to the good D. Is it for good
2. S he _____fainted when she heard that her child died.
A. rather than . B. nothing but C. all but D. near
3. _________ to interfere in your affairs but 1 would like to give you just one
piece o f advice.
A. It is far from clear
C. Far and wide for me
B. Far from it for me
D. Far be it from me
374
4. we jus t have to wait.
A. At this point in time
c . At some point
5. He gripped his brother’s arm lest h e_
A. would be trampled
c. were trampled
6. I do n't k no w_____ the new manager.
A. what to do o f
c. how to do of
7. Don’t tell me I
B. what to make of
D. how to make of
till 5 o'clock in the afternoon.
A. was to make them to stay
c. could have made them stay
8. What 1 fear most of all is that we
market in West Crane. The local people seem to be very wary about it.
A. might misinterpret
c. wo n't be misinterpreting
9. Sam looked as if he_____in the world.
A. were n't cared
c. did n't have a care
10. W hat's all this cry ing _____ ?
A. without the aid of
c. with the aid of
B. On the point o f time
D. That's not the point
_ by the mob.
B. could be trampled
D. be trampled
B. must have made them stay
D. was to have made them stay
in what refers to the building o f a new
B. might have misinterpreted
D. might be misinterpreted
B. weren’t taken into care
D. were n’t in the care
B. in aid of
D. within the aid of
ĨV. PREPOSITIONS AND PHRASAL VERBS (lOpts)
1. He is not a very outspoken person, so we may find it hard to ____ his opinions.
A. leak out B. come off c. sound out D. bring round
2. Are you st ill _______ an illusion-that Mr. Spike will agree to your conditions?
A. in B. under c. on D. o f
3. We all tried to convince our teacher to change his mind about the school trip
and he finally________ .
A. caught on B. set out c. got through D. came round
4. Our grandfather c an________ his war experiences all the day. Sometimes, we
all get truly bored with it.
A. harp on B. vouch for c. figure out D. leaf through
5. Father says he’s to o ________ after working the nightshift and he won ’t help
us prepare the barbecue.
A. run away B. worn out c. taken in D. pulled down
6. They said I'd b e ________ probation for the first two or three weeks as they
want to find out about my skill.
A. for B. on c. in D. at
7. Your re port ________ the side effects of the venture. Why not write more
about its good points, Mr. Hewitt?
A. backs down B. takes to c. dwells on D. makes out
375
8 .1 know Pete’s conduct was intolerable, but do n't be too h ar d________ him.
A. on B. with c. at D. to
9. It is said that the corruption can hardly b e______ especially among top officials.
A. worn off B. rooted out c. gone back D. stringed along
10. Mrs. Black has promised to b e ________ service on Monday. I hope she
keeps her word or else I won't manage to prepare everything in time.
A. in B. on c. of D. within
V. READING COMPREHENSION
PASSAGE 1: Read the following passage and choose the best option to
complete the blank or answer the question. (lOpts)
Every day, millions of shoppers hit the stores in full force - both online and
on foot - searching frantically for the perfect gift. Last year, Americans spent
over $30 billion at retail stores in the month of December alone. Aside from
purchasing holiday gifts, most people regularly buy presents for other occasions
throughout the year, including weddings, birthdays, anniversaries, graduations,
and baby showers. This frequent experience of gift-giving can engender
ambivalent feelings in gift-givers. Many relish the opportunity to buy presents
because gift-giving offers a powerful means to build stronger bonds with one 's
closest peers. At the same time, many dread the thought of buying gifts; they
worry that their purchases will disappoint rather than delight the intended
recipients.
Anthropologists describe gift-giving as a positive social process, serving
various political, religious, and psychological functions. Economists, however,
offer a less favourable view. According to Waldfogel (1993), gift-giving
represents an objective waste of resources. People buy gifts that recipients would
not choose to buy on their own, or at least not spend as much money to purchase
(a phenomenon referred to as “ the deadweight loss of Christmas"). To wit,
givers are likely to spend $100 to purchase a gift that receivers would spend only
$80 to buy themselves. This “ deadweight loss” suggests that gift-givers are not
very good at predicting what gifts others will appreciate. That in itself is not
surprising to social psychologists. Research has found that people often struggle
to take account of others' perspectives - their insights are subject to egocentrism,
social projection, and multiple attribution errors. What is surprising is that giftgivers have
considerable experience acting as both gift-givers and gift-recipients,
but nevertheless tend to overspend each time they set out to purchase a
meaningful gif t..
In the present research, we propose a unique psychological explanation for
this overspending problem - i.e., that gift-givers equate how much they spend
with how much recipients will appreciate the gift (the more expensive the gift,
the stronger a gift-recipient's feelings of appreciation). Although a link between
gift price and feelings of appreciation might seem intuitive to gift-givers, such an
376
assumption may be unfounded. Indeed, we propose that gift-recipients will be
less inclined to base their feelings of appreciation on the magnitude of a gift than
givers assume.
Why do gift-givers assume that gift price is closely linked to gift-recipients'
feelings of appreciation? Perhaps givers believe that bigger (that is, more
expensive) gifts convey stronger signals of thoughtfulness and consideration.
According to Camerer (1988) and others, gift-giving represents a symbolic ritual,
whereby gift-givers attempt to signal their positive attitudes toward the intended
recipient and their willingness to invest resources in a future relationship. In this
sense, gift-giv ers may be motivated to spend more money on a gift in order to
send a “stronger signal " to their intended recipient. As for gift- recipients, they
may not construe smaller and larger gifts as representing small er and larger
signals of thoughtfulness and consideration.
1. What is the main idea discussed in the passage?
A. Gift-giving, despite its uneconomical downsides, cultivates a positive
social process.
B. Gift- recipients are widely acknowledged as consider ably experienced in
gift-giving.
c. Gifts can serve as implicit signals of thoughtfulness and consideration.
D. Gift-giving may have certain drawbacks alongside its positive qualities.
2. The author most likely uses the examples of gift-giving occasions in paragraph
1 to highlight the_____ .
A. regularity with which people shop for gifts
B. recent increase in the amount of money spent on gifts
c. anxiety gift shopping causes for consumers
D. num ber o f special occasions involving gift-giving
3. In paragraph 1, the word “am biv ale nt'’ most nearly means___ _ .
- A. unrealistic B. apprehensive c. conflicted D. supportive
4. The author indicates tha t people value gift-giving because they feel it_____.
A. functions as a form of self-expression
B. is an inexpensive way to show appreciation
c. requires the gift-recipient to reciprocate
D. can serve to strenghten a relationship
5. The “so cia l psycholog ists” mentioned in paragraph 2 would likely describe
the “d eadw eig ht loss” phenomenon as_____.
A. predictable B. unprecedented c. disturbing D. questionable
6. In paragraph 3, the authors indicate that the assumption made by gift-givers
may be_____ .
A. insincere B. unreasonable c. incorrect D. substantiated
7. The word “conv ey” in paragraph 4 most nearly means_____ .
A. transport B. counteract c. exchange D. comm unicate
377
8. In paragraph 4, the author refers to works by Camerer and others in order
to .
A. offer an explanation B. introduce an argumen t
c. question a motive D. support a conclusion
9. In paragraph 4, the word “construe” most probably means_____ .
A. view B. understand c. consider D. take
10. Which of the following best characterizes the tone of the author?
A. authoritative B. pessimistic c. ironic D. intormative
PASSAGE 2: Read the following passage and choose the best option to
complete the blank or answer the question.(lOpts)
According to sociologists, there are several different ways in which a person
may become recognized as the leader of a social group in the United States. In
the family, traditional cultural patterns confer leadership on one or both of the
parents. In other cases, such as friendship groups, one or more persons may
gradually emerge as leaders, although there is no formal process of selection. In
larger groups, leaders are usually chosen formally through election or recruitment.
Although leaders are often thought to be people with unusual personal ability,
decades of research have failed to produce consistent evidence that there is any
category QỈ “natural lead ers” It seems that there is no set of personal qualities
that all leaders have in common; rather, virtually any person may be recognized
as a leader if the person has qualities that meet the needs o f that particular group.
Furthermore, although it is commonly supposed that social groups have a
single leader, research suggests that there are typically two different leadership
roles that are held by different individuals. Instrumental leadership is leadership
that emphasizes the completion of tasks by a social group. Group members look
to instrumental leaders to “get things” done. Expressive leadership, on the other
hand, is leadership that emphasizes the collective well-being of a social group's
member. Expressive leader are less concerned with the overall goals of the group
than with providing emotional support to group members and attempting to
minimize tension and conflict among them. Group members expect expressive
leaders to maintain stable relationships within the group and provide support to
individual members. Instrumental leaders are likely to have a rather secondary
relationship to other group members.
They give orders and may discipline group members who inhibit attainment
of the group's goals. Expressive leaders cultivate a more personal or primary
relationship to others in the group. They offer sympathy when someone
experiences difficulties or is subjected to discipline, are quick to lighten a serious
moment with humor, and try to resolve issues that threaten to divide the group.
As the differences in these two roles suggest, expressive leaders generally
receive more personal affection from group members; instrumental leaders, if
they are successful in promoting group goals, may enjoy a more distant respect.
378
1. What does the passage mainly discuss?
A. The problems faced by leaders
B. How leadership differs in small and large groups
c. How social groups determine who will lead them
D. The role of leaders in social group
2. The passage mentions all o f the following ways by which people can become
leaders EX CE PT _____ .
A. recruitment B. formal election process
c. specific leadership training D. traditional cultural patterns
3. In mentioning “n atu ral lea ders” in line 7, the author is making the point
that_____ .
A. few people qualify as “natural leaders”
B. there is no p roof that “natural leaders” exist
c. “natural leaders' are easily accepted by the members o f a social group
D. “natural leaders” share a similar set of characteristics
4. Which of the following statements about leadership can be inferred from
paragraph 2?
A. A person who is an effective leader of a particular group may not be an
effective leader in another group.
B. Few people succeed in sharing a leadership role with another person.
c. A person can best learn how to be an effective leader by studying research
on leadership.
D. Most people desire to be leaders but can produce little evidence of their
qualifications.
5. The passage indicates that instrumental leaders generally focus on_____.
A. ensuring harmonious relationships
B. sharing responsibility with group members
c. identifying new leaders
D. achieving a goal
6. The word “co lle cti ve” is closest in meaning to_____ .
A. necessary B. typical . c. group D. particular
7. The word “the m” refers to_____ .
A. expressive leaders B. goals of the group
c. group members D. tension and conflict
8. A “se cond ary relationship” between a leader and the members of a group
could best be characterized a s_____.
A. distant B. enthusiastic c. unreliable D. personal
9. The word “r eso lve ” is closest in meaning to_____ .
A. avoid repeating B. talk about
c. avoid thinking about D. find a solution for
10. Paragraphs 3 and 4 organize the discussion of leadership primarily in terms
of_____.
A. examples that illustrate a problem B. cause and effect analysis
c. narration of events D. comparison and contrast
379
VI. GUIDED CLOZE
PASSAGE 1: Read the following passage and choose the options that best
complete the blank. (lOpts)
THE VACUUM CLEANER
Until about 250 years ago, households did not take dirt as seriously as they do
now - it was a fact of life, and that was that. Cleaning often consisted of an
annual (1 )______ called 'spring cleaning' when the furniture was moved aside,
and all the linen products in the house were cleaned. Carpets and rugs were taken
outside, hung on ropes and had the dust (2 )______ out of them - an exhausting
and messy process.
The industrial revolution brought about a major change - as new products
became available to make homes cleaner, a corresponding interest in 'domestic
hygiene' appeared in households. This in turn led to the (3 )______ of further
products, one o f which was the vacuum cleaner.
(4 )______________ has it that when one of the first vacuum cleaners was
demonstrated, a kindly scientist took the proud inventor ( 5)______ , and offered
a bit of advice that was to become crucial to the future evolution of the product -
'make it suck, not blow'.
The first vacuum cleaners appeared in the 1860s in the United States. They
were operated by hand pumps and were almost as ( 6)______ as spring cleaning.
It was only when electric motors had become sufficiently (7 )______ to become
portable that vacuum cleaners became common household items. Most of today's
major ( 8)______ - including Electrolux and Hoover - were born in the 1920s.
. The household dirt that vacuum cleaners suck up is mostly dead skin cells -
humans (9) ______ millions of cells every day. A much smaller proportion
comes from dust and soil carried into the house from (10 )______ .
1. A. ritual B. result c. resolution D. scrub
2. A. cleaned B. taken c. beaten D.sucked
3. A. fabrication B. appearing c. recreation D. developm
4. A. Story B. Epics c. Legend D. Tale
5. A. away B. aside c. aback D. along
6. A. laborious B. hard c. nefarious D. straining
7. A. scientific B. forward c. technological D.advanced
8. A. brand B. marks c. makes D. trademark
9. A. lose B. outgrow c. omit D.s hed
10. A. external B. outside c.be yo nd D. indoors

VI. G UIDED CLOZE


PASSAGE 2: Read the following passage and choose the options that best
complete the blank. (lOpts)
CARNIVOROUS PLANTS
All plants rely on nutrients taken from the soil in order to survive. However,
in areas where the soil does not contain enough vital nutrients, some plants have
380
adapted to (1 )_____ their diets from another source: living organisms. Though
they are few in number, carnivorous plants are nonetheless fascinating beings
that eat anything from one-celled organisms to insects in order to survive. They
are commonly found in marshlands. Carnivorous plants feature one of several
types of traps to ensnare prey, which they consume to (2 )_____ nutrients that
may be missing from the soil. While there are over 400 species ofcarnivorous
plants in the world today, some are more (3 )_____ than others.
The most well-known of these plants are the snap traps, which include the
Venus flytrap. Snap traps are easily identified by their leaves, which are
separated into two lobes that have the ability to fold together. Inside the lobes,
the surface is covered with tiny hairs that are (4) _____ to movement. When the
pla nt's prey brushes against the hairs, it triggers a closing mechanism that
rapidly brings the two lobes together, trapping the prey (5 )_____ inside. The
response of the traps is phenomenal (6 )_____ speed: the time between triggering
the hairs and snapping shut is less than a second. As the prey struggles inside the
trap, it only triggers more hairs, causing the leaves to tighten their (7) .
The plant then secrets liquid chemicals from special glands into the trap to
dissolve the prey and absorb all of its nutrients. Besides the Venus flytrap, only
one other type of snap trap exists today, (8 )_____ to as the waterwheel plant.
The two share a common ancestor and differ only in a few ways. (9 )_____ , the
waterwheel is an aquatic plant, while the flytrap is exclusively terrestrial. In
addition, the flytrap feeds primarily on arthropods like spiders, while the
waterwheel lives (10 )___ _ simple invertebrates, like certain types o f plankton.
feed B. enlarge c. supplement D. modify
c. get through with D.
break in on B. make up for
drop out o f
dominant B. prevalent c. recessive D. popular
prone B. liable c. vulnerable D. sensitive
closely B. furiously c. steadily D. securely
in accordance with B. in regard to c. in preference to D. on merits of
grip B. snail c. fist D. seizure
implicated B. indicated c. referred D. denoted
For instance B. As a result c. In contrast D. Otherwise
.. onto B. with c. for D. of f
B. WRITTEN TEST
I. OPEN CLOZE TEST
PA SSAG E 1: Fill in each gap with ONE suitable word.(lOpts)
ALTERNATIVE FAST FOOD
It is generally agreed upon that fast food is a great solution to the problem of
what to eat. However, it’s (1 )______ as less nutritional than a healthy meal,
which is what growing children need. For years, nutritionists have been trying to
381
get children into the (2 )______ of eating healthy food. Without a doubt, this has
not been easy, getting in (3 )______ that they have to compete with numerous
fast food restaurants which are very popular with young people.
Nutritionists have lately come up with an idea which could (4 )______ to be
successful. This involve (5 )______ children how to cook and in the process to
enjoy' the taste of fresh food. In the long (6)______ , this may keep them off fast
food. There are already a number o f school projects throughout Britain that are
experimenting with the idea. They begin making vegetarian pizza - both the
dough and the topping. Many children were impressed with their own homemade pizzas.
(7 )______ they compared their pizzas with ready-made ones, the
differences were noticeable. They found the (8) ______ greasy and not as
delicious as the homemade ones. Throughout the lesson, the children learn about
the quality and become skillful in the different techniques involved in cooking.
They become interested in cooking and in no (9 )______ they even learn how to
look after themselves by eating nutritious food.
Finally, nutritionists would like to get children cooking at home, too. It may
be messy, but, it’s also an (1 0)______ for parents to spend quality time with
their children.
PASSAGE 2: Fill in each gap with ONE suitable word. (lOpts)
Every spring and fall, you can probably look up into the sky and see large
flocks of birds either coming or going. This (1) of flying south for the
winter and north for the summer is called migration. Birds migrate for a number
of reasons. One reason is of course to escape the (2)______conditions o f winter.
Although they have feathers, most birds are poorly (3)______ to deal with
extremely cold temperatures. Their (4)______are open to the wind and snow,
and do little to keep the birds warm. Furthermore, birds often find it extremely
difficult to gather enough food during the winter. By flying south (5)______
warmer climates during winter, birds avoid the cold temperatures and food
shortages that other animals have to (6)______ during the winter months.
Another reason is mating. Many birds will only lay their eggs in certain places
and at certain times o f the year. So, migration also occurs at mating times.
One of the amazing things about birds is the distances and the accuracy
(7) ______ which they migrate. Many birds are able to find their way back, to the
exact same spots year after year despite having flown thousands of kilometres.
The bar goose, for example, migrates over 30,000 kilometres each year without
(8) its way. In order to travel such long distances without getting lost,
birds use a variety of methods to help them (9)______ . One of the most
important of these methods is the use of landmarks. From high in the air, birds
have an excellent view of the land below them. They can see mountains, rivers
and other (10)______ land features and use them to find their way.
382
ÌĨ. WORD FORMS
PART 1: Give the c orrect forms o f the word s in b rackets.
I. There is little hope that M aurice’s behavior will ever improve. It will probably
remain so ____________________ till he grows up. (CORRECT)
-• T he________________ •
project has just been launched in that university.
(DEPARTMENT)
3. Matt says you must be good a t ___________ _________responsibilities of a
restaurant owner. (TASK)
4. Everyone marveled at h is ____________________ idea of organizing more
field trips for students. (OBJECT)
5. He g et s______________ _____ whenever he hears that song. (EYE)
6. She looked absolutely ; __________________when I told her what had
happened. She could hardly say a word. (DUMB)
7. New immigrants have been successfu lly_____________ into the community.
(SIMILAR)
8. This school was once ____________________ as a military hospital during
the war. (REQUIRE)
9. In Scotland, there is greater emphasis o n___________________ by individual
schools. (VALUE)
10. He stood at the door to make sure that no one __________ the party. (GATE)
II. WOR D FO RM S
PART 2: Comple te the following passage with the correct forms o f the given words.
fro nt except secure labour
solid tend affirm alter

EXTRACT FROM A BOOK ABOUT MEETING


We are (1) ________ by the experts that we are, as a spe
for face-to-face communication. But does that really mean having every meeting
in person? Ask the bleary-eyed sales team this question as they struggle
(2).______________ through their weekly teambuilding session and that answer
is unlikely to be in the (3) _______________ . Unless you work for a very small
business or have an (4 )_ ________ ' high boredom threshold, you doubtless
spend more time sitting in meetings than you want to. Of course, you could
always follow business Norm an’s example. He liked to express (5) _ _ _ _ _ _ _
with customers queuing at the (6) _ _____________ by holding management
meetings standing up. Is email a realistic (7 )__________ ? It’s clearly a powerful
tool for disseminating information, but as a meeting substitute, it’s seriously
flawed. Words alone can cause trouble. We’re all full of (8 )_________ that can
be unintentionally triggered by others and people are capable of reading anything
they like into an email. Th ereJs also a (9 )________ for email to be used by
people who wish to avoid ‘real’ encounters because they don’t want to be (10)
,_________ with any awkwardness.
383
III. ERROR IDENTIFICATION
Identify 10 errors in the following passage and correct them.
BEETHOVEN
Ludwig Van Beethoven, one
of the most popular classical 1.
composer
of all time, is sometimes
known as 'the Shakespeare o 2.
f music'. He was
bom in Bonne in 1770, and
publishing his first work at 3.
the age of
thirteen. Mozart was his
teacher for a short time and 4.
they did not get
on very well together;
Beethoven was a difficult 5.
student.
Like a composer, Beethoven
was a genius - but as a 6.
person, he was
not very easy to like. He was
a passionate man who 7.
controlled his
temper very easily. He was
also arrogant. The upper 8.
classes of Vienna
used to invite him to parties,
when he was often quite 9.
rude. He was
once heard to say to a
prince: ‘There will always
be many princes, but 10.
there is uniquely one
Beethoven.’
Nevertheless, Beethoven
played some of the most 11.
beautiful
symphonies the world has
never heard. How was such 12.
an arrogant,
bad-tempered man inspired
to the write such romantic 13.
music? Perhaps
the answer lies in the three
letters that were found after 14.
his death. They
were addressed to his
‘immortal Beloved’. ■ 15.
Nobody knows that this
woman was, but it appears 16.
that Beethoven was deeply
in love with her
for most o f his adult life.

1. 2. 3.
4. 5. 6.
7. 8. 9.
IV, SENTENCE
10,
TRANSFORMATION
Rewrite the sentences with the given words or beginning in such a way that
their meanings remain unchanged.
1. My new job is much more satisfying than any job I’ve ever had.
—» My new job is fa r____________________________ _____ I’ve ever had.
2. If Tom hadn’t acted promptly to extinguish the fire, there might have been
more damage to the house.
—> But ______________________________________the fire out, there might
have been more damage to the house. .
3. A child of his age is too young to be deceitful.
—> S o______________ _ __________________ _______________ deceitful.
4. I've become extremely good at missing the rush hour over the last few weeks.
(FINE)
—> I've g o t________________________ ______________________ •
over
the last-few weeks.
384
5. I feel that to brand her ideas unworkable at this stage would be wrong.
(WRITE)
—> 1 don't th ink_______________________________ _ at this stage.
6. It’s crucial for us to control the juvenile inmates in a very strict way. (KEEP)
—> It's crucial for US_____________________________________ inmates.
7. You’ve done nothing but look miserable all day. (AROUND)
—> You'v e do ne ___________________________________________all day.
8. We don’t expect that the tourists in that accident have survived. (HOLD)
—> We d on’t _______ _______________________________ in that accident.
9. Your story is different from the facts. (DOES)
—> You r st ory ________ ___________________________________ the facts.
10. His excuse for such bad behavior has little effect on her. (NO)
—> His e xcuse_______________________________________________her.
TRƯỜNG THPT CHUYÊN NGUYỄN CHÍ THANH - ĐĂKNÔNG
A. MULTIPLE CHOICE
I. WORD CHOICE
Choose the best option A, B, c, or D to complete the following sentences and
write your answers in the corresponding numbered boxes.
1. Of all entries received, his w as _______out for special raise.
A. isolated B. brought c. opted D. singled
2. I-was in a _______ as to what to do. If I told the truth, he would get into
trouble, but if I said nothing I would be more in trouble.
A. doubt B. quandary c. hitch D. complexity
3. The matter has been left i n __________until the legal ramifications have been
explored.
A. recess B. suspension c. abeyance D. criticism
4. The eng ineer_______ the machine with a hammer and, miraculously, it roared
back to life.
A. slapped B. smacked c. whacked D. punched
5. The Oscar winning actress sim ply _____ _charm and professionalism in her
acceptance speech.
A. exuded B. excluded c. expunged D. extricated
6. The carrots are ready; could you drain them in the______ and put them in the
serving dish?
A. casserole B. colander c. whisk D. blender
7. Sue went for a ______ .when she injured her foot and broke several toenails.
A. manicure B. pediment c. foot massage D. pedicure
8. What I find m os t________ about it is that he didn’t even have a decency to
say that he was sorry.
A. galling B. furious c. touchy D. blazing
385
9.1 know it’s difficult but yo u’ll just have t o ................... and bear it.
A. laugh B. smile c. grin D. chuckle
10. The comp uter has changed the w or ld ......................
A. irreplaceably B. irrevocably c. irredeem ably D. irreparably
11. STRUCTURES AND GRAMMAR
11. Considered America’s first great a rch itect,______ .
A. many of the buildings at Harvard University were designed by Henry
Hobson Richardson
B. it was Henry Hobson Richardson who designed many of the buildings at
Harvard University
c. Henry Hobson Richardson designed many of the buildings at Harvard
University
D. Harvard University has many buildings that were designed by Henry
Hobson Richardson.
12. “ Frankly, Ms. Adamson wo rks______ her figures never ne ed _______,” said
the General Manager.
A. such efficiently that/ to check B. so efficient that/ checking
c. such an efficient that/ to be checked D. so efficiently that/ to be checked
13. Thick fog across much of the UK has resulted in dozens of flights_______
and many more delayed.
A. being cancelled B. to be cancelled
c. are cancelled D. cancelled
14. Although there is some truth to the fact that Linux is a huge threat to Microsoft,
predictions o f the Redmond company's demise ar e,______ , premature.
A. saying the most B. to say the least
c. to say the most D. to the least degree
15. David: Would you like fish or meat? Mary: 1 '_____fish, please.
A. would rather B. would prefer c. suppose D. believe
16. In his anxiety to make himself_______ ■, he spoke too loudly and too slowly.
A. understand B. understanding
c. understood D. to understand
17 .1 ________ with the performances but I got flu the day before.
A. was to have helped B. helped c. was to help D. had helped
18.________ chair the meeting.
A. John was decided to B. It was decided that John should
c. There was decided that John should D. John had been decided to
19 .1 thought about the problem but 1 couldn’t ________ a solution.
A. come in for B. come across
20 .________ , they slept soundly.
c. come up with D. come out
A. Hot though was the night air
c. Hot as was the night air
B. Hot though the night air was
D. Hot although the night air was
386
III. PR EPOSITIONS AND PHRASAL VERBS
21. The unions made it clear that they would not settle _ _ ______anything less
than a 20% increase in salaries.
A. to B. for c. down D. up
22. She d oesn't eat crisps or chocolate; she’s _________ health foods.
A. into B. in c. up for D. under
23. Housing in some cities is so expensive that some people cannot even afford
to p ut_________ the two months’ deposit that is required.
A. away B. down c. out D. about
24. The sky was heavy with black clouds. We hurried, hoping the rain would
ho ld _________ until we got the tents up.
A. through B. on c. up D. off
25. Several members of the public ste pp ed _______________ with information
pertaining to the inciden t
A. back B. forward c. backward D. onto
26. Ali was ....................an inch o f reaching "his goal when Ben stepped into the
picture and messed up his plans.
A. into B. within c. on D. for
27. We hav en't m et........ .......... a month of Sundays and I can hardly remember
what he looks like.
A. in B. up c. off D, out
28. Tim distrusts his new friends and prefers to keep them ........... arm 's length.
A. at B. to c. under D. in
29. As the day w ore ................... . I began to feel more and more uncomfortable
in their company.
A. with B. on c. out D. at
30. That car must have set J o e ................... quite a bit; it’s top of the range.
A. back B. away B. in D. from
IV. COLLOCATIONS AND IDIOMS
31. Most people feel a slig ht _______ of nostalgia as they think back on their
schooldays.
A. feeling B. surge c. pang D. chain
32. The cost of a new house in the UK has become _______ high over the last
few years.
A. totally B. astronomically c. blatantly D. utterly
33. The entire staff was thrown off _______ when the news of the takeover was
announced.
A. composure B. disarray c. stable D. balance
34. Mr Simkins is the big _______ in the company as he has just been promoted
.to the position of Managing Director.
A. bread B. cheese c. apple D. meat
387
35. It was an extremely hostile article which ca st _______ on the conduct of the
entire cabinet.
A. criticism B. aspersions c. disapproval D. abuse
36. He's so lazy! We all have to work harder because he's always____ his duties.
A. evading B. shirking c. ducking D. dodging
3 7 .1 found the information for the project in the encyclopedia but I couldn't give
______and verse on it.
A. chapter B. unit c. poem D. extract
38. I’m hoping that this work experience will stand me in ____in my future career.
A. good grounding B. good stead c. fine precedent D. stable footing
39. A meal in that restaurant costs t he_______ but still it is always packed.
A. world B. soil c. earth D. moon
40. What's that horrible noise downstairs?'
'It's only Sam. He always scream s______ _ murder when we take him to the
dentist.'
A. red B. black c. yellow D. blue
V. READING
READING 1 '
Read the following passage and choose the best answer to each question.
Write your answers in the corresponding numbered boxes provided below
the passage (10 pts)
Before 1815 manufacturing in the United States had been done in homes or
shops by skilled artisans. As master craft workers, they imparted the knowledge
of their trades to apprentices and journeymen. In addition, women often worked
in their homes part-time, making finished articles from raw material supplied by
merchant capitalists. After 1815 this older form o f manufacturing began to give
way to factories with machinery tended by unskilled or semiskilled laborers.
Cheap transportation networks, the rise of cities, and the availability of capital
and credit all stimulated the shift to factory production.
The creation of a labor force that was accustomed to working in factories did
not occur easily. Before the rise of the factory, artisans had worked within the
home. Apprentices were considered part of the family, and masters were
responsible not only fo r teaching their apprentices a trade but also for providing
them some education and for supendsing their moral behavior. Journeymen knew
that if they perfected their skill, they could become respected master artisans with
their own shops. Also, skilled artisans did not work by the clock, at a steady pace,
but rather in bursts of intense labor alternating with more leisurely time.
The factory changed that. Goods produced by factories were not as finished
or elegant as those done by hand, and pride incraftsmanship gave way to the
pressure to increase rates of productivity. The new methods of doing business
involved a new and stricter sense of time. Factory life necessitated a more
388
regimented schedule, where work began at the sound of a bell and workers kept
machines going at a constant pace. At the same time, workers were required to
discard old habits, for industrialism demanded a worker who was alert,
dependable, and self-disciplined. Absenteeism and lateness hurt productivity
and, since work was specialized, dis rupte d the regular factory routine.
Industrialization not only produced a fundamental change in the way work was
organized; it transformed the very nature o f work.
The first generation to experience these changes did not adopt the new
attitudes easily. The factory clock became the symbol of the new work rules.
One mill worker who finally quit complained revealingly about "obedience to
the ding-dong of the b ell -j ust as though we are so many living machines." With
the loss of personal freedom also came the loss of standing in the community.
Unlike artisan workshops in which apprentices worked closely with the masters
supervising them, factories sharply separated workers from management. Few
workers rose through the ranks to supervisory positions, and even fewer could
achieve the artisan's dream of setting up one's own business. Even well-paid
workers sensed their decline in status.
In this newly emerging economic order, workers sometimes organized to
protect their rights and traditional ways of life. Craftworkers such as carpenters,
printers, and tailors formed unions, and in 1834 individual unions came together in
the National Trades' Union. The labor movement gathered som e momentum in the
decade before the Panic of 1837, but in the depression that followed, labor's strength
collapsed. During hard times, few workers were willing to strike or engage in
collective action. And skilled craftworkers, who spea rheaded the union movement,
did not feel a particularly strong bond with semiskilled factory workers and
unskilled laborers. More than a decade of agitation did finally bring a workday
shortened to 10 hours to most industries by the 1850's, and the courts also
recognized workers' right to strike, but these gains had little immediate impact.
Workers were united in resenting the industrial system and their loss o f status,
but they were divided by ethnic and racial antagonisms, gender, conflicting
religious perspectives, occupational differences, political party loyalties, and
disagreements over> tactics. For them, the factory and industrialism were not
agents of opportunity but reminders of their loss of independence and a measure
of control over their lives. As United States society became more specialized and
differentiated, greater extremes of wealth began to appear. And as the new
markets created fortunes for the few, the factory system lowered the wages of
workers by dividing labor into smaller, less skilled tasks.
41. Which of the following can be inferred from the passage 1 about articles
manufactured before 1815?
A. They were primarily produced by women.
B. They were generally produced in shops rather than in homes.
389
c.They were produced with more concern for quality than for speed of production.
D. They were produced mostly in large cities with extensive transportation networks.
42. Wh ich o f th e sen tences bel ow best exp res ses the essential inform ation in the
hig hlighted sen ten ce in the pas sag e 2? Inc orr ect ans wer choic es cha nge the
' me aning in imp ort ant w ays o r leav e out essential informatio n.
A. Ma sters dem and ed moral beh avior from app rentices but ofte n tre ate d them
irre spo nsibly .
B. Th e res ponsi bilitie s of the ma ster to the app ren tic e we nt bey ond the
tea ching o f a trad e.
c. Ma sters preferred to maintain the trad e within the fam ily by sup erv ising
and edu cating th e younger family members.
D. Ma sters who trained memb ers of the ir own fam ily as app rentices
dem and ed exc elle nce from them.
43. The w ord ’’dis rupted” in the passage 3 is close st in m ean ing to
A. pro lon ged B. established c. follow ed D. upset
44. In paragraph 4, the auth or includes the quotation from a mill worker in order to
A. sup port the idea tha t it was diff icult for workers to adjus t to working in
fac tories
B. to show tha t workers som etim es quit bec ause of the loud noise made by
factory m ach inery
c. argue that clocks did not have a useful function in facto ries
D. empha size tha t facto ries w ere most succ essful whe n wo rke rs rev ealed their
com plaints
45. All of the follow ing are men tioned in paragraph 4 as con seq uen ces of the
new system for workers EXCEP T a loss of
A. freedom
B. statu s in the c ommunity
c. opp ortunities fo r adva nce ment
D. conta ct among wo rkers who w ere not ma nagers
46. The phrase "g at he re d some m om en tu m ” in the pas sage 5 is clo ses t in
meaning to
A. made progress B. became active
c. caused c han ges D. combined forc es
47. The w ord "s pear hea ded” in the passage 5 is closest in m ean ing to
A. led B. accepte d c. changed D. resisted
48. Which of the f ollowing statements abo ut the labo r mo vem ent o f the 1800's is
supported by para graph 5?
A. It was m ost succ essful during tim es of econom ic crisis.
B. Its prim ary purpose w as to benefit unskilled laborers,
c. It was slow to improve con ditions for workers.
D. It he lped workers of all skill levels form a s tron g bond w ith each yea r
390
49. The author identifies political,party loyalties and disagreements over tactics
as two of several factors that
A. encouraged workers to demand higher wages
B. created divisions among workers
c. caused work to become more specialized
D. increased workers' resentment o f the industrial system
50. The word "them" in the passage 6 refers to
A. Workers B. political party loyalties
c. disagreements over tactics D. agents of opportunity
READING 2
Read the passage and choose the best answer to each o f the questions. (10 pts)
HISTORY OF THE CHICKENPOX VACCINE
Chickenpox is a highly contagious infectious disease caused by the Varicella
zoster virus; sufferers develop a fleeting itchy rash that can spread throughout
the body. The disease can last for up to 14 days and can occur in both children
and adults, though the young are particularly vulnerable. Individuals infected
with chickenpox can expect to experience a high but tolerable level of
discomfort and a fever as the disease works its way through the system. The
ailment was once considered to be a “rite of passage” by parents in the u.s.
and thought to provide children with greater and improved immunity to
other forms of sickness later in life. This view, however, was altered after
additional research by scientists demonstrated unexpected dangers associated
with the virus. Over time, the fruits of this research have transformed attitudes
toward the disease and the utility of seeking preemptive measures against it.
A vaccine against chickenpox was originally invented by Michiaki Takahashi, a
Japanese doctor and research scientist, in the mid-1960s. Dr. Takahashi began
his work to isolate and grow the virus in 1965 and in 1972 began clinical trials
with a live but weakened form of the virus that caused the human body to create
antibodies. Japan and several other countries began widespread chickenpox
vaccination programs in 1974. However, it took over 20 years for the chickenpox
vaccine to be approved by the Ư.S. Food & Drug Administration (FDA), finally
earning the u.s. government’s seal of approval for widespread use in 1995. Yet
even though the chickenpox vaccine was available and recommended by the FDA,
parents did not immediately choose to vaccinate their children against this disease.
Mothers and fathers typically cited the notion that chickenpox did not constitute a
serious enough disease against which a person needed to be vaccinated.
Strong belief in that view eroded when scientists discovered the link between
Varicellazoster, the virus that causes chickenpox, and shingles, a far more
serious, harmful, and longer-lasting disease in older adults that impacts the
nervous system. They reached the conclusion that Varicella zoster remains
dormant inside the body, making it significantly more likely for someone to
develop shingles. As a result, the medical community in the u.s. encouraged the
391
dev elo pm ent , ado ption, and use of a vac cine aga inst chick enpox to the public.
Alth oug h the app earan ce of chick enpox and shin gles within one pers on can be
many yea rs apart - gener ally ma ny dec ade s - the increased risk in dev elo pin g
shin gles as a younger adult (30-40 yea rs old rather th an 60-70 years old) proved
to be enough to con vin ce the medica l com mu nity tha t immu nization should be
preferred to the tradition al alte rnative.
An oth er reason tha t the chick enp ox vaccine was not imme dia tely acc epted
and used by paren ts in the Ư.S. centere d on obs erv ations made by scientists that
the vac cine sim ply did not last long enough and did not confe r a lifetime of
imm unity. In oth er words, scie ntis ts con sidered the ben efits of the vac cin e to be
tem porary when give n to you ng childre n. They also feared that it increased the
odds t hat a person could bec ome infected with chi cke npox l ater as a yo ung adult,
when the rash is more painful and prevalent and can last up to three or four
weeks. Hen ce, allo wing you ng childre n to develop chick enpox rather than take a
vac cine aga ins t it w as believed to be the “lesser o f two evils.” This idea changed
ove r time as booster shots of the vaccine elongate d imm unity and cou nte red the
perceived limits on the strength of the vaccine itself.
Tod ay, use of the chickenpo x vaccine is com mon throughout the world.
Pediatric ians suggest an initial vac cinatio n shot afte r a child turns one yea r old,
with boo ster shots recommende d after the child turn s eigh t. The vac cine is
estima ted to be up to 90% effective and has reduced worldwide cases of
chicke npo x infection to 400,000 cases per yea r from over 4 ,000,000 cases befo re
vac cinatio n became wid espread . ■ (A) In light of such statistics, mo st doctors
insist that the pote ntial risks of dev eloping shin gles outweigh the ben efits of
avoidin g rare com plic atio ns associated with inoc ulations . ■ (B) Of course,ma ny
parents con tinue to think o f the disease as an innocuous ailm ent, refusing to take
preemptiv e steps aga inst it. ■ (C) As increasing numb ers of studen ts are
vac cinated and the viru s becomes increasingl y rarer, how eve r, even this trend
amo ng parents has failed to halt the decline of chick enp ox among the most
vulnerable pop ulation s. B (D)
51. The word “tol er ab le ”in the 1st pas sage is clo sest in me ani ng to
A. sudd en. B. bearable, c. infre quent. D. une xpe cted.
52. According to paragraph 1, which of the following is true of the chicken pox virus?
A. It leads to a potentially d ead ly dise ase in adu lts.
B. It is associate d with a pos sibly permanen t rash.
c. It is easily tran smitta ble by an infecied individual.
D. it has been virtually eradicated in the modern world.
53. Which of the follow ing best express es the esse ntial inform atio n in the
hig hlighted sentence?
A. Ư.S. paren ts believed tha t ha ving c hicken pox benefited the ir c hildren.
B. Ư.S. parents believed that chickenpox led to immunity ag ainst most sickness.
392
c. u.s. parents wanted to m ake sure that their children developed chickenpox.
D. Ư.S. parents did not think that o ther vac cinations were needed af ter chickenpox.
54. Which of the follow ing can be inferred from paragraph 2 about the clin ical
tria ls for the chick enpox vaccine?
A. They too k lo nger than expected. B. They cost a lot o f mone y to complete,
c . They to ok a long time to finish. D. They were ultim ately successful,
55. The word “notion” in th e 2nd passage is clos est in me ani ng to
A. history. B. findings . c . fact. D. belief.
56. Ac cording to paragraph 3, w hich of the follow ing is true o f Varicella Zo ster l
A. It typica lly attac ks adults who are over 60 years old.
B. It is linked to a serious disease tha t occu rs more com mo nly in adults.
c . It like ly is n ot a serious e nou gh threat to hum an health to require a vaccine.
D. It is c om ple tely era dic ated from the body after chi cke npox occurs.
57. Ac cording to paragraph 3, all of th e follow ing is true about the chi cke npo x
virus EX CE PT: . .
A. It causes two distinct yet re lated ailm ents .
B. Peopl e did not view it as a serious public health threat.
c . It te nded to quick ly bec om e dormant and rem ain inoperativ e ove r time.
D. Va ccina tion again st it would help prevent the ons et of shingles.
58. The a uth or uses “booster shots”as an example of
A. a w ay to increase the effe ctiv ene ss of the c hickẹn pox vaccine.
B. a preferred method of chicke npo x rash and fever trea tme nt,
c . a s cie ntifically app roved medicine to e liminate chicke npox.
D. a strategy for parents to avoid vac cinatin g the ir child altoge ther.
59. According to paragraph 4, man y parents did not cho ose the chicke npo x
vaccine b eca use
A. the y bel iev ed that the virus was weak and not especially harm ful.
B. the y thought tha t scientis ts did not have enough d ata to reach a conc lusion,
c. they were unsure about the utility o f the vaccine given its expected duration.
D. they were convinced it was potentially very toxic, particularly for older
children.
60. Look at the fou r squ ares [■] that indic ate where th e follow ing sentenc e could
be add ed to the p assa ge.
Meanwhile, some continue to remain unconvinced, citing a supposed
potential of the vaccine to do harm.
Wh ere w oul d the sen tence fit best?
A. A B. B c . c D. D
VI. CLOZE TEST
CLO ZE TEST 1
For questions 61-70. Read the following passage and decide which answer
A, B, c or D best fits each gap. Write your answers in corresponding
numbered boxes. (10 pts)
393
The coolest kids in Europe sh are,a single (61) ______ they want to get
married, have children and live happily ever after. They know it means putting their
children first and sticking with their spouses even if they (6 2) ______ out of love.
This news comes from the report of a new study that set out to find the answer to the
modem riddle: What will today's (63)______really, really want tomorrow?
Poignantly, one of the clearest answers is that they want to have happy
families. Even in the most (6 4 )______ countries there was condemnation for
divorce, demands that parents should keep their marriage (65) ______ and
admiration for stable couples.
It appears that among the middle classes, the quality of our children's lives
has suffered from the (6 6)______ on parents in high-stress professions. In the
days when the concept of'quality time' first emerged , I remember seeing a TV
producer on (67) ... dial home on her mobile phone to read her son a bedtime
story. This is just not good enough.
Quality time cannot be time (68)______. Children need unconditional time in
the same (6 9) ___________ that they need unconditional love. This study found
a generation that had given up trying to get its parents' attention but was
(70) to do better by its own
children.
61. A. drive B. eagerness c. ambition D. ye'arning
62. A. drop B. fall c. slip D. jump
63. A. youth B.young c. juniors D. juveniles
64. A. loose B. relaxed c. generous D. liberal
65. A.oaths B. vows c. pledges D. promises
66. A. pressures B. weights c. burdens D. loads
67. A. site B. place c. situation D. location
68. A. organized B. managed c. controlled D. discipline
69. A. system B.respect c. way D. method
70. A. determined B. firm c. persistent D. stubborn

CLOZE TEST 2.
For each gap, choose the correct answer A, B, c or D which best fits the
context. (10 pts)
The thought of taking a year out from studying is (71)................... Fun,
freedom, adventure, perhaps even money seem to be on offer for a year. So why
not go (72 )............... it? A year out can provide all of these things. But before
you make a firm decision, take some time to think about whether a year out is
really what you want. If it is, then do make sure you know what you want to do
with it. A year with nothing planned can be depressing and a (73)............ .
waste of time.
You need to take a positive attitude if you are to benefit. You need to start
preparing for it well before the time comes to take the (74).................. Most
universities and employers look favourably on such applicants if they have
shown good reasons for taking a year out and have not just (75 )............... into it.
394
They value the maturity, confidence, social and other skills.a gap year can_bring.
Colleges find that gap year students (76)............... to be more mature and
motivated. They mix better with their fellow students and work harder because
they have a better idea of where they want to go. They also (77)........ . more
to college life in general, because they are used to being a part of a team, either
at work or study.
Employers also look kindly on (78) .............. gap year students. They want
employees who are experienced and mature and can deal better with the
unexpected. If their year out has also equipped them with skills which other
applicants may not have, such as a better (79)...............o f a foreign language,
then that too is a plus.
Your family may well have (80 )............... feelings. On the one hand, they
will be proud of,y our independence; on the other hand, there will be worries
about your safety.
71. A. disparaging B. intimidating c. alarming D. enticing
72. A. at B. for c. in D. off
73 .A. deep B. large. c.sh ee r D. full
74. A. plunge B. dive c. jump D. spin
75. A. drifted B. floated c. flopped D. flown
76. A. happen B. look c. tend D. end
77. A. present B. participate c. contribute D. involve
78. A. old B. former c. later D. present
79. A. grasp B. grab c. grapple D. hold
80. A. different B.crossed c. muddled D. mixed
B. WRITTEN TEST •-V
I. O PEN C LO ZE
L•
TEST
OPEN CLOZE TEST

1
For questions 1-10,
read the text below
and think of the word
which best
one word in each
each space. Use only your answers in
space. Wr ite
corresponding n umbered boxes provided below the passage.
Interest in mythology has grown steadily throughout the last hundred years,
assisted by the realization that myths are not childish stories or mere prescientific
explanations of the world, ( I ) ________serious insights into reality.
They ( 2)__________ in all societies, in the present as well as the past. They are
(3) ________ of the fabric of human life, embodying beliefs, moulding behavior
and justifying institutions, customs and values. Myths are imaginative traditions
about the nature, history and destiny of the world. Definitions of myth
(4) ________ /story ’ fail because a good many myths are not stories at all. The
mythology of some (5) ._____ includes the assignment of different functions
to the (6 )_____ ■ gods and goddesses: one deity presides over agriculture,
(7) ________________ over war and so on. Beyond this, the term myth is also
(8) ____________ to the religious and secular traditions which exert a powerful
395
influence on attitudes to life, but the literal accuracy of which there is
(9 )________ to doubt. However, because myths are woven into the fabric of a
society where they are accepted as true, the impact of new discoveries, new
attitudes and new ways of life on myths is usually to undermine them. When old
myths are lost, new ones are needed. No society seems ever to have flourished
without a set of myths containing its vision of its past, its ( 10)_____________ _
and its purposes.
OPEN CLOZE TEST 2
For questions 11-20, read the text below and think of the word which best
fits each space. Use only one word in each space. Write your answers in the
corresponding numbered boxes provided below the passage.
Does the thought of making a presentation to a group of peers bring you
(11) ........... ............ in a cold sweat? If so, you’re not alone. The mere idea of
having to “stand and deliver” in front o f others is enough to strike dread into the
(12) ........................ of even the most experienced business person, let alone
students. Yet effective spoken communication is an essential (13 )........................
for career success in today ’s business and academic environments.
So what can people do to add sparkle to their speaking skills and (14)...........
this understandable but unfounded fear of speaking, in public? The bad news is
that presentation nerves are quite normal and you will probably always suffer
from them. The good news is that interesting speakers are (15).............. .........
and not born. You can learn the techniques that will (16)........................you into
a calm and convincing speaker.
The first step is to persuade yourself you can do it. Just like an actor waiting
in the wings, or an athlete (17).......................up for the big race, you need to get
yourself on a confidence high. Try focusing your thoughts on moments of
particular success during your life to (18)......'.................. Remember that the
physical symptoms of nerves are most obvious to you. The audience won’t see
your knees knocking or your hands trembling so don’t worry about it. Some of
the worst presentations are those where the speaker clearly hasn’t (19)....... ......
enough time to it beforehand." Let’s face it; a presentation that’s slung together
half an hour before it’s going to be (20)........... isn’t going to impress anyone.
II. WORD FORMS
Supply each gap with correct form of the word given in brackets.
1. The scandal su rely ..................... the end of his political career. (SIGN)
2. He g aine d......... for being difficult to work with as an actor. (NOTORIOUS)
3. Tre Transformer is quite intriguing. It i s ............ . one of the best movies of
the year. (DOUBT)
4. Her hip has been _____... for quite a while, and she'll probably need surgery
_ on it. (TROUBLE) g
5. Her latest novel is a ......................... thriller, set some time in the late 2 1 St
century. (FUTURE)
396
6. The new policy only serves to .. ....................the inadequacy of help for the
homeless. (ACCENT)
7. Successful candidates will be required to .................an induction programme.
(TAKE)
8. Zom bies are archetypal monsters from the bottom of the uncanny valley, with
their dead eyes a nd................... faces (EXPRESS)
9. The terrible scenes were ind elibly ............ . on his mind (PRINT)
10. Phyllis Schlafly, the ................ conservative activist who helped defeat the
Equal Rights Amendment in the 1970s, has died. (SPEAK)
Supply each gap with correct form of the word given in the box. (lOpts)
new courtesy caress acquaint
relate disdain comfort ceremony
DON 'T JUDGE A BOOK BY ITS COVER
The magical world of literature was first revealed to me when I was still
young enough to be held (1 ) __________________ on my f ather’s lap. To my
(2 )___________ young mind, my father’s reading aloud to me could only be
bettered by the process of preparation which immediately preceded it. Choosing
the book, holding it (3 )___________ in one 's hands, admiring the ja cket, and,
finally, opening it to the first page to begin, seemed to me akin to following the
rites of some sort of (4) ___________ occasion.
To the dedicated reader, every book has its own distinct smell. A book just
purchased from one’s favorite book shop exudes a wonderful bouquet of printer's
ink, leather and binding glue. Because of its “yet to be discover ed” content, I
tend to treat a new book as I would a (5 ) _____________ acquired (6) _ ______ .
That is, while both new books and new friends have to be treated (7 )________
one still feels comfortable with them due to an ( 8 ) _______r
sense that
whispers that warm, lasting (9 )___________ are sure to follow.
An old book, on the other hand, is a horse of a different color. On opening it,
one's nostrils are assailed by an odor that a non-book lover would ( 10)_________
describe as simply ‘musty’..To the true book connoisseur, however, this scent
elicits enchanting memories of Christmases past, pressed roses, cedar chests and
autumn leaves burning in the back yard. To the devout reader, an old book
smells of nostalgia.
III. ERROR IDENTIFICATION (10 PTS)
The passage below contains 10 mistakes. Identify the errors and write the
corrections. (10 points)
Line
1 Human and primates, the family of apes, gorillas, and chimpanzees,
2 among others, divide many common traits. While primates are deemed
3 the most intelligent of anim als,.most researchers believed they lack the
397
4 capacity to produce language. However, a research project in the 1970s at
5 University of Georgia showed promise that chimpanzees have the ability
6 to learn a certain language, just as human children. The project used
7 several chimpanzees as test subjects in which Lana , a female chimp was
8 the study focus. Though the primates lack the vocal constructions to
9 make human speech patterns, the researchers created a language called
10 Yerkish, using lexigram made up of symbols that represent sounds and
11 words. 125 symbols were placed on a keyboard, which Lana was taught
12 how to use the board to communicate with the researchers. She successfully
13 expressed her thoughts by pressing different keys in succession. In some
14 cases, she used up to seven at times.
IV. SENTENCE TRANSFORMATION
Complete the second sentence so that it has a similar meaning to the first
sentence using the word given. Do not change the word given.
1. After a long hard journey, 1 cheered up when I saw my home. (SIGHT)
After a long hard journey, my spirits.............................. ......... ...o f my home.
2. He d idn't want to get into a position where he might lose all his money.
(POSSIBILITY)
He didn't want t o ....................... ...................... ............. .......... all his money.
3. The company received an enormous number of calls responding to the advert.
(DELUGED)
The company , ............ ................................................. response to the advert.
4. Children enjoyunwrapping their Christmas presents. (A KICK)
5. He hid the truth about the matterbecause he didn't want to lose his high position.
(CARPET)
6. You don't have to come and see the new house if you don't wish.
=> You are un de r...............................................................................
7. He threatened the officers with violence.
He made............................... .. .. .. .. .. .. .. .. .. .. .. .. .. .. .. .. .
8. The headmaster has a very good opinion of the physics teacher.
=> The headmaster holds..............................................................................
9. He himself admits to a fear of spiders.
=> On his................................... ...............................................
10. Suzanne is far superior to me in terms of technical knowledge.
=> When it.................... ............................................................................. ..
398
TRƯỜNG TH PT CHUY ÊN NGUYỄN ĐÌNH CHIỂU - ĐỔNG THÁ P
A. MULTIPLE CHOICE (40 PTS)
I. WORD CHOICE (5 PTS)
Choose the best options to complete the following sentences.
1. When Mary rented her apartment through a broker, she did n't have enough
money to pay both rent and broker
A. rentals B. reductions c. listings D. fees
2. They worked from dawn to dusk with su ch ______ that they were exhausted. ‘
A. boredom B. zeal c. detraction D. debility
3. She .______agreed to go with him to the football match although she had
no interest in the game at all.
A. apologetically B. grudgingly c. shamefacedly D. discreetly
4. The police couldn’t persuade the man to ______- the source of his
information.

A.u nco ver B. discover c. divulge D.concede


had nothing to
5. Due to many
, the Smiths fall back on
years o f
when it
was time for
them to retire.
A. illiteracy B. impunity c. inflexibility D. imprudence
6. The room is obvious.
A. blindlessly B. blindedly C. blindly . D. blindingly
7. All his hard
in success.
work
A. accounted B. merged c. culminated D. succumbed
8. The bedroom
with battles.
was
A. infested B. inflated c. infected D. infused
9. If things are going well. In fact, business is __________ ,
A. soaring B. rolling c. blooming D. leaping
from the burning
0. The smoke tyres could be seen
for miles
A. sweeping B. billowing c. radiating D. bulging
II. GRAMMAR AND STRUCTURES (5PTS)
Choose the best options to complete the following sentences.
11. ________ relatively inexpensive, the metal pewter can be fashioned into
beautiful and useful objects.
A. Even it is B. Despites c. Although D. Nevertheless, it is
12. ______ in front of a camera lens changes the color of the light that reaches
the film.
A. A filter placed B. A filter is placed
c. Placed a filter D. When a.filter placed
13. Do you play an instrument?” I have an accordion, but
A. 1 haven't played it in years
c. it has years to play
14. Scientists cannot agree on
A. that fleas are
B. I have years to play it
D. it hasn’t played for years
related to other orders o f insects.
B. how fleas are C. how are fleas D. fleas that are
399
15. Superconductivity will revolutionize the way that energy is used for the next
millennium, and__________ the first truly superconductive substance will be
remembered as a technological hero.
A. what the discovery of B. the discovery of
c. whoever discovers D. whose discovery
16. Before they go to the university, most senior high school graduates have
______ _ idea o f what college life is like.
A. no less
17. I’m
B. no least
my brother is.
c. not less D. not in the least
A. nowhere like so ambitious
c. nothing as ambitious than
18. He seem s______ _ for the experience.
B. nowhere near as am bitious as
D. nothing near as ambitious as
B. none the worse
D. none the worst
A. none worse
c. none worse at all
19. Betty says she cannot stand looking at the r at,______ touching it.
A. even so B. what i f c. let alone D. as far as
20. Country life differs from urban life in ________ a sense of community
generally binds the inhabitants o f small villages together.
A. it B. what c. which D. that
III. PHRASAL VERBS AND PREPOSITIONS (5PTS)
Choose the best options to complete the following sentences.
21. He’s a tough politician - he knows how to ______the storm.
A. ride out B. run down c. keep up D. push back
22. I w as ________ _ by the beautiful music.
A. carried up B. carried out c. carried away D. carried on
23. That sneaky real estate agent cons m e______ selling her my house for a lot
less than it was worth.
A. out B. on c. at D. into
24. The ringleader was extremely lucky to get r a suspended sentence.
A. off with B. away c. throught to D. through
2 5 .1 got impatient waiting for my turn to _________.
A. go out B. set out c. come out D. work out
26. Over the centuries the feet o f many visistors have _ _ the steps to the castle.
A. worn out B. worn down c. worn through D. worn in
27. We’re going to ________Mark since he moved into a new house.
A. call on B. call at c. call into D. call for
28. Fireworks__________ the attraction of the festival night.
A. appeared to B. increased to c. added up to D. added to
29. Susan’s fac e________ when she heard the good news.
A. lit up B. lightened up c. is bright D. brightene
30. Your plan doesn’t all ow _______ changes in the weather.
A. with B. on c. for D. into
400
IV. COLLOCATIONS AND IDIOMS (5PTS)
Choose the best options to com plete the following sentences.
31. If she gets a ________ of anger, don’t panic, that’s her uasual self.
A. tit B. breath c, period D. look
32. Scientists think that resource shortages will cause an escalation of conflicts
during this century, and will widen t he______ between the rich and the poor.
A. blank B. gulf C. difference D. hole
33. The government should take measures to prevent nuclear waste from being
_______ at sea.
‘ A. dropped Bi discharged c. dumped D. disposed
34. The actor was so nervous that he could
only remember sm al l______ _ of
dialogue. .
A. shreds B. pieces c. patches D. snatches
3 5 .1 was kept awake for most of the night by th e_____ of a mosquito in my ear.
A. whine B. moan c. groan D. screech
36. H e___________ a yawn as the actor began yet another long speech.
A. squashed B. suffocated c. stifled D. submerged
37. Tam afraid you’ve caught me on th e_____ . I wasn’t expecting you until this
afternoon.
A. stove B. grapevine c. spot D. hop
38. Every delicacy Miss Cook produces is do ne _________
A. to a turn B. at will c. there and then D. sooner or later
39. Richard started the race well but ran out o f__________ in the later stages.
A. pow er B. force c. effort- D. steam
40. He might look kind and sympathetic, but deep down he's as hard a s____
A. nails B. a mountain c. a gangster D. an iceberg
V. GUIDED CLOZE TEST: (10 PTS)
Read the following passages and choose the options that best complete the
blanks.
PASSAGE A
It is an accepted part of everyday nostalgia to assume that in the past, food
was somehow better than it is today. The fruit and vegetables were more
(41)...'............. grown and this was not seen as an extra bonus which added ten
percent on to the price. (42)...;....... food was fresh, not frozen, and you had the
chance to (43 )............ it to see whether you wanted it. When you went shopping,
you could ask for exactly what piece of meat you wanted and see the butcher
cutting it, instead of finding it ready-wrapped in plastic. And our local tradesman
soon got to know what you wanted, and provided it for you; otherwise he would
have gone (44)......... business. Of course, unless we invent time - travel we shall
never know whether this is all true. Survivor from those (45)...... ........ days
obviously tend to dislike today’s convenience foods, and to prefer the Good Old
401
Days when a joint of beef filled the oven, (46)............ thick red juice instead of
water when cooked, and cost the same (47)......:..a can of Coke.
What is always forgotten is that then as now the quality of your food depended
very much (48 )........... who you were, how well-off you happened to be, and
where you lived. Shopping then (4 9) ... .......considerable skill, and shoppers had
to be able to (50 )....................the fresh from the nor so fresh. There was no shellby date to act
as a guide. If you were hard up then frozen meat and canned foods
would have been on the menu, just as they are today.
41. A. nature B. natural c. naturalness D. naturally
42. A.Most B. Several c. Plenty D. Few
43. A. examine B. experiment c. distinguish D. detect
44. A. in to B. on to c. away from D. out of
45. A. far B. outlying c. distant D. further
46. A.manufactured B.
c.reprod uced D. resulted
produced
47. A. with B. to c. as D. than
48. A. upon B. from c .o f D off
49. A. ordered B. demanded c. commanded D. prepared
50. A. say B. talk c.sp ea k D. tell
PASSAGE B
If asked who (51) the game of baseball, most
would probably reply that it was Abner Doubleday. At the (52)beginning o f this
century, there was some disagreement over (53)___________ the game of
baseball had actually originated, so sporting-goods manufacturer Spaulding
inaugurated a commission to research the (54)____________ . In 1908 a report
was published by the commission _________ (55) which Abner Doubleday, a
Ư.S. Army officer from Cooperstown, New York, was (56)______ _____ credit
for the intervention of the game. The National Baseball Hall of Fame was
established in Cooperstown in (57)__________ of Doubleday.
Today, most sports historians are in agreement that Doubleday (58)________
did not have much to do with the development of baseball. (59)__________ ,
baseball seems to be a close (60)____________ to the English game of rounders
and probably has English rather than American roots.
51. A. discovered B. founded c. invented D. found
52. A. Americans B. American c The US D. America
53. A. what B. how c. when D. that
54. A. problem B. question c. matter D. issue
55. A. at B. to c .o f D. in
56. A. provided B. passed c. given D. delivered
57. A. honor B. reward c. award D. ceremony
58. A. truly B. really c. as well D. probably
59. A.But B. Moreover c. Instead D. Additiona
60. A. relate B. relationship c. relative D. relation
402
VI. READING COMPREHENSION (10PTS)
Read the texts below and choose the best answer to each question.
PASSAGE A
Glass is a remarkable substance made from the simplest raw materials. It can
be colored or colorless, monochrome or polychrome, transparent, translucent, or
opaque. It is lightweight impermeable to liquids, readily cleaned and reused,
du rable yet fragile and often very beautiful Glass can be decorated in multiple
ways and its optical properties are exceptional. In all its myriad forms as table
ware, containers, in architecture and design-glass represents a major achievement in
the history of technological developments.
Since the Bronze Age about 3,000 B.C., glass has been used for making
various kinds of objects. It was first made from a mixture of silica, line and an
alkali such as soda or potash, and these remained the basic ingredients of glass
until the development o f lead glass in the seventeeth century. When heated, the
mixture becomes soft and malleable and can be formed by various techniques
into a vast array of shapes and sizes. The homogeneous mass thus formed by
melting then cools to create glass, but in contrast to most materials formed in this
way (metals, for instance), glass lacks the crystalline structure normally
associated with solids, and instead retains the random molecular structure of a
liquid. In effect, as molten glass cools, it progressively stiffens until rigid, but
does so without setting up a network of interlocking crystals customarily
associated with that process. This is why glass shatters so easily when dealt a
blow. Why glass deteriorates over time, especially when exposed to moisture,
and why glassware must be slowly reheated and uniformly cooled after
manufacture to release internal stresses induced by uneven cooling.
Another unusual feature of glass is the manner in which its viscosity changes
as it turns from a cold substance into a hot, ductile liquid. Unlike metals that
flow or “freeze” .at specific temperatures glass progressively soften as the
temperature rises, going through varying stages o f malleability until it flows like
a thick syrup. Each stage of malleability allows the glass to be manipulated into
various forms, by different techniques, and if suddenly cooled the object retains
the shape achieved at that point. Glass is thus amenable to a greater number of
heat-forming techniques than most other materials
61. Why does the author list the characteristics o f glass in paragraph 1?
A. To demonstrate how glass evolved
B. To show the versatility of glass
c. To explain glassmaking technology
D. To explain the purpose of each component of glass
62. The word “durable” in paragraph 1 is closest in meaning to _________
A. lasting B. delicate c. heavy D. plain
403
63. What does the author imply about the raw materials used to make glass?
A. They were the same for centuries. B. They are liquid.
c. They are transparent. D. They are very heavy.
64. According to the passage, how is glass that has cooled and become rigid
different from most ther rigid substances?
A. It has an interlocking crystal network.
B. It has an unusually low melting temperature,
c. It has varying physical properties.
D. It has a random molecular structure,
65. The word “customarily” in paragraph 2 could best be replaced by “______”
A. naturally B. necessarily c. usually D. certainly
66. The words “exposed to” in paragraph 2 most likely mean________ _
A. hardened by B. chilled with c. subjected to D. deprived of
67. What must be done to release the internal stresses that build up in glass
products during manufacture?
A. The glass must be reheated and evenly cooled
B. The glass must be"cooled quickly
c. The glass must be kept moist until cooled
I D. The glass must be shaped to its desired from immediately
68. The word “induced” I paragraph 2 is closest in meaning t o _________
A. joined B. missed c. caused D. lost
69. The word “it” in paragraph 3 refers to _________
A. feature B. glass c. manner D. viscosity
70. According to the passage, why can glass be more easily shaped into specific
forms than can metals?
A. It resists breaking when heated
B. It has better optical properties
c. It retains he at while its viscosity changes
D, It gradually becomes softer as its temperature rises
PASSAGE B
The best way to learn is to teach. This is the message emerging from
experiments in several schools in which teenage pupils who have problems at
school themselves are tutoring younger children - with remarkable results for
both sides.
According to American research, pupil - tutoring winds “hands down” over
computerized instruction and American teachers say that no other recent
innovation has proved so consistency successful.
Now the idea is spreading to Britain. Throughout this term, a group of 14-
year olds at Trinity comprehensive in Leamington Spa have been spending an
hour a week helping children at nearby primary school with their reading. The
younger children read aloud to their tutors (who are supervised by university
students of education) and then play word games with them.
404
All the 14 year-olds have some of their own lessons in a special unit for
children who have difficulties at school. Though their intelligence is around
average, most of them have'fallen behind on reading, writing, and maths and, in
some cases, this has led to truancy or bad behaviour in class.
Jean Bond, who is running the special unit in Warwick university education
department, says that the main benefit of tutoring is that it improves the
adolescents' self-esteem. The younger children come rushing up every time and
welcome them. It makes the tutors feel important whereas, in normal school
lessons, they often feel adequate. Everyone benefits. The older children need
practice in reading, but, i f they had to do it in their own classes, they would say it
was kid 's stuff and be worried about losing face. The tutors are struggling at
school themselves so, when the younger ones ca n’t learn, they know exactly why.
Jean Bond, who describes pupil tutoring as an ‘‘educational conjuring
trick'’, has run two previous experiments. In one, six persistent truants, aged 15
upwards, tutored 12 slow-learning infants in reading and maths. None of the six
played truant from any of the tutoring sessions. “The degree of concentration
they showed while working with their tutees was remarkable for pupils who had
previously showed little ability to concentrate on anything related to school work
for any period of time”,'says Bond. The tutors became “reliable, conscientious
caring individuals”.
Their own reading, previously mechanical and monotonous, became far more
expressive as the result o f reading stories and to infants. Their view o f education,
which they had previously dismissed as a “crap” and “a waste of time” was
transformed. They became firmly resolved to teach their own children to read
before starting school, because, as one of them put it, “if they go for a job and
they can't write, they are not going to employ you, are they?” . The tutors also
became more sympathetic to their own teachers' difficulties, because they were
frustrated themselves when the infants “mucked about”.
In the seven weeks of the experiment, concludes Bond, “these pupils received
more recognition, reward and feelings of worth than they had previously
experienced in many years of formal schooling.” And the infants, according to their
own teachers, showed measurable gains in reading skills by the end of the scheme.
71. The majority of the tutor in the Trinity experiments are pupils who _______ .
A. cause discipline problems for their teachers
B. frequently stay away from school
c. are below standard in basic skills
D. are unable to read and write
72. What is true according to American teachers on pupil-tutoring?
A. It has been more successful than other recent innovations.
B. It is much better than computerized instruction.
405
c. The best way to learn is to teach other people.
D. Tutoring benefits both tutors and tutees.
73. Which skill is used in the experiment at Trinity?
A. Reading B. Speaking
c. Writing D. Reading and writing
74. What, according to Jean, makes tutoring interesting to tutors?
A. They feel the younger children need their help.
B. They feel being a tutor is more appealing than being a student,
c. They don’t have to go to class anymore.
D. They feel they are more like an adult.
75. According to the writer, the tutor wouldn’t normally practise reading in class
bec aus e__________________ .
A. their teachers would get impatient with them.
B. they wouldn’t be able to concentrate
c. their teachers wouldn't consider it necessary;
D. they would find it humiliating.
76. Which is NO T TRUE about the tutors?
A. They become more sympathetic to their own teachers.
B. Their reading skills have greatly improved.
c. They are never frustrated with their little children.
D. Their view of education becomes more positive.
77. Pupil tutoring is describes as “an edu cational con jur ing trick” because
A. no one understands why it works so well
B. it has caught the attention of the media
c. educational authorities are suspicious of it
D. it is a simple idea with extra ordinary results
78. What does the word “muck ab out” NOT mean?
A. treat someone badly B. behave badly
c. mess around D. fool around
79. The most significant result of the experiments so far carried out seemed to
have been that tuto rs____________ .
A. learnt to overcome their fear o f reading aloud
B. improved their pupils’ ability to concentrate
c. benefited from an increase of self-respect
D. came to see the importance o f writing skills
80. What is NO T mentioned in the text?
A. The educational benefits of pupil-tutoring
B. The experiments about pupil-tutoring
c. Agood way to motivate weaker students to learn better
D. Comparison between tutoring and other teaching method
406
B. W R IT TE N TE S T (70 PTS)
I. CLOZE TEST: (20PTS)
Read the texts below and think of the word which best fits each gap. Use
only ONE WORD for each space
PA SS AG E A
LONDON
London is a big city, but many people who live there (1)_____ . it as a
number o f small towns put together. Each one district has its (2)_______ identity
and atmosphere and some of parts are even described by their inhabitants as
'villages ’. Much of the centre of the city consists (3)_______ _ shops and
businesses and the majority of people they live in the suburbs. A (4)_______
many of them travel to work in the city every day (5)_____a train, bus, tube or
car; this is call commuting. Commuters might spend as (6)_______ as two hours
every morning getting to work and (7)_________two hours getting home again.
The cost of living in London is higher (8)_________so in most other parts of
■Britain, and many people are paid extra money on the top of their salaries
because of this. Millions of visitors come to London every year from all over the
world for to see the famous sights, such as Buckingham Palace, (9)________ the
Queen lives, and many other historic buildings. London is also very famous for
including its theatres, red buses, and black taxis. Some people find (10)________
a noisy, dirty place but it has many large, pleasant parts where everyone can
enjoy themselves some peace and quiet. London has many attractions there, both
for people from overseas and for people from other parts of Britain.
PA SSAG E B
The cinema is today a favourite place of entertainment to most people, young
and old. Hundreds of people visit the cinema today. There are many things that
we can learn (1)______ _______our visits to the cinema. The pictures we can
(2)_______ in the cinema often show US many useful things. They show how
people in (3)_________ . lands live and think. Sometimes they also show how
people in the past lived and died. There (4)________also pictures which show
many things about the world of birds and animal. Some pictures, ( 5 ) _______ ,
sometimes spoil the minds of many people especially (6)____________ of small
boys and girls. Pictures which show how people steal, kill or kidnap, for
example, have done a lot o f harm, but these (7)__________ pictures also show
how, in the end the criminals are caught and punished, and many people have
learnt to obey the law, as a result. Thus, whether such pictures are good or bad
depends on (8)________ . lesson the person has learnt from them. On the
(9 )___________ most pictures try to teach many useful lessons. The cinema
may therefore be described as a school where people can learn a lot of god
lessons. It is for all these reasons that cinema is still the (20)__________
favourite place o f entertainment.
407
II. WO RD FO RM AT IO N (20 PTS)
PART 1:
Complete each sentence, using the correct form of the word in parentheses
1. Gazprom, whose boss Alexei Miller is a close political ally of Russian President
Vladimir Putin, is almost a s______ in world football. (PR ES EN T)
2. Some data showed vitamin D had a protective role against _________
diseases such as type 1 diabetes. (IM MU NE)
3. Hunger and a _________ meal did not sit happijy side by side. (SLAP)
4. All around a re __________ mountains which provide the walkers or naturelovers with
magnificent opportunities to savor the Alpine flora. (SN OW )
5. Britain's nuclear power program began with a lie: it was a __________ for the
nuclear weapons program. (SC REEN)
6. T he _______ vigor of man-made things, as suggested by these metaphors, is at
the same time ungainly and unnatural. (HU MAN)
7. H is ____________ whether poets, novelists, producers or scholars, tailored
their books to his minutely worked-out requirements. (LIBR AR Y)
8. I've never heard such ____________ nonsense in my life! (AD ULT)
9. He was discovered to have been________ company funds. (PR OP ER TY )
10. After 2012. the base shifted the event to a(n) __________ schedule, but
budget cuts forced it to cancel the 2014 show. (AN NU AL)
PART 2: Complete each passage, with the appropriate form of the words
given in the box.
fi t hot class new
illusion surface dismount land
SEOUL, South Korea — Divorced and out of money, Kwon Chol-nam fled
North Korea for China in 2014 by wading across a river border at night and then
crawling over a (1)__________ fence. After a perilous trek that included walking
through a jung le in Laos, he reached Thailand, where he was allowed to fly to
South Korea and start life (2) _________. After all that trouble and danger.
Mr. Kwon now wants South Korea to allow him to return home to the North.
“You have to ride a horse to know whether it’s the right (3) _ _ _ _ _ for you,”
Mr. Kwon said in an interview in Seoul. “ I have tried, and the South ( 4)_______
of me. I want to go home to the North to reunite with my (5)___________ and
16-year-old son.”
Although North Korea is one of the world's most politically repressive
countries, Mr. Kwon says he has grown (6)________with life in the capitalist
South, where he says North Korean defectors like him are treated like
(7)________ citizens. “They called me names, treating me like an idiot, and
didn't pay me as much as others doing the same work, just because I was from
the North,” Mr. Kwon (8)_____________ said. To press his unusual demand, he
408
has held news conferences, submitted petitions to the United Nations and
demonstrated with signs in front of government buildings in Seoul. Apart from
. him, more than 30,000 North Koreans have fled to South Korea since a famine
hit their (9)_________________ in the 1990s. Of them, 25 have mysteriously
(1 0) ___________ back in the North in the past five years.
(Source: adaptedfrom "The New York Times")
in . ERROR CORRECTION: (10PTS)
The follow ing passage contains 10 errors. Identify and correct them. Choose
the best options to complete the following sentences.
10
15
20
5
The role of the traditional zoo, inheriting from the 19th century, has
undergone a dramatic shift. A growing recognition that zoos ought to be
in the vanguard of the fight for the devastation of our natural world has
begun a zoologic revolution.
The change occurred in the 1960s, when the Jersey zoo was set off to
breed endangered species. As a result, the breeding o f animals in captive
has become a complex science, with zoos around the world co-coordinating
their efforts to avoid the genetic dangers of in-breeding small populations.
The answer for the question of whether zoos can have very much impact
on the preservation of endangered species is, probably, minimally. Zoos
do not focus their education efforts on those people in the strongest
positions to affect the future of the wildlife exhibited. For the most part,
conservation education is targeted at children and other non-decision
makers in a process too slow or too far away to address the extinction
crisis which exists now. Furthermore, the efforts of zoos to inform
lawmakers and government authorities are usually low-key or un-existent.
Campaigns are more likely to be for an animal exhibit rather than for
the existence of the animal itself.
Nevertheless, it does not do to address the future from a foundation of
pessimism. A vision of the future is embraced in which the human
population has leveled off at about 8.8 billion and where in human
effects upon the environment have been tethered and considerable
wildlife remain. It certainly will not be as rich or abundant as toda y’s
wildlife, but with substantial diverse and numbers of more or less wild
ecosystems, and the zo os' work, this vision can become reality.
4 ._______________
7 ._______________
2 ._________ _
5. __________
8.
3 .________ -
6 ._______________ ;
9. _______________
10.
409
TV. SENTENCE TRANSFORMATION (20PTS)
Rewrite the following sentences using the words given.
1. David hadn’t expected that he would feel so weak after the operation.
—» The operation left . __________________________________________ .
2. The president is the statesman I admire most o f all.
-> There is no ___ _______________________________________________ .
3. You’re far more practical than I am.
—» Now here ____________________ ______________________________ .
4. Could you watch my bag while I'm away?
-» Could you k ee p_______________________________________________?
5. The staff hated Frank’s new policies intensely and so went on strike.
-> So inten se______________ __________________ ;___________________ .
6. The family members find it hard to accept their father’s fortune will go to
charity. (RESISTANCE)
-> Th ere _________________________________________ go to the charity.
7. His analysis of the situation was far too complex for me to grasp. (HEAD)
—» His analysis of the situation____________ ____________________ .
8. Start doing some work now if you want to become a rich and powerful man.
(PULL)
—>_________________________________ __________________ substance.
9. Had it not been for John, she couldn’t have got the job. (AGENCY)
—> O nly _____ __________________________________ ______________ .
10. Peter never makes any effort to help his team when it is time for action.
(LIFTS)
-> Peter ne ver_____________________________________ -time for action.
TRƯỞNG ĨH PT CHUYÊN THOẠI NGỌC HẦU - AN GIANG
A. MULTIPLE CHOICE QUESTIONS
I. VOCABULARY: WORD CHOICE
Choose the best option to complete each of the following sentences.
1. Although the patient received intensive treatment, there was n o .......................
improvement in her condition.
A. decipherable B. legible c. discernible D. intelligible
2. The dentist told him to open his mou th.........
A. broad B. much c. greatly D. wide
3. Only one person who can provide the best'solution to the question will be
promoted and ..........a financial grant.
A. served B. awarded c. entitled D. equipped
410
4. You may feel sure the casting will be done perfectly, j u s t......... your trust in
me and you will see I’m right.
A. allow , B. forward c. grant D. lay
5. His invention changed th e ......... of history
A. way - B. line c. course D. route
6. The country has few natural resources and its economy has bee n............... for
some time now.
A. diseased B. unwell c. sickening D. ailing
7. In her speech the Prime Minister .................... tribute to the valuable
contributions to society made by voluntary organizations.
A. paid B. gave c. sent D. brought
8. Unfortunately, his report doesn’t ....................... with what w e’ve learnt from
other sources.
A. pally B. rally c. ally D. tally
9. We'd all been at school together for 12 years and at our leaving celebrations
w e ......................... eternal friendship.
A. assured B. pledged c. undertook D. confided
10. Maria and Jean had a ......................... romance - they met and married
within two months.
A. hurricane B. whirlwind c. cyclone D. typhoon
11. G RA MM AR AN D STR UCTURES
Choose the best word or phrase which best completes the follow ing sentences.
Write y ou r answer in the co rrespo nding num bered boxes.
1.................................... are poor observers of their child 's behavior so deviant
behavior reaches unmanageable proportions.
A. Parents that B. When parents
c. If parents D. Parents
2. Food is, after all, an important part of Chinese culture and mission controllers
say it is im portant................... China's space pioneers do not go hungry.
A. so that B. make sure
c. that food is provided for D. to ensure that
3. Crushed ice is used to cool drinks, and is often applied to injuries where there
is swelling, to remove excess h ea t................... in the tissues.
A. generated B. to generate c. generating D. generates
4. With ................... 5,000 tigers remaining in the world today, time is quickly
running out for this beautiful animal.
A. as fewer than B. so few as c. as few as D. not fewer than
5........................................ films about writers are so dull is that writers don't
dress up to practice their craft.
A. One o f the reasons B. Everybody says
c. It is often claimed D. Because
411
6. It's a horrible irony, but a bully has to know his victim well .........................
effective; it's almost impossible to bully strangers.
A. if being B. to be c. in order to D. if they are
7. Desks can often show aspects of character otherwise hidden from general
vie w ........ if analyzed in detail, will reveal their owner’s innermost secrets.
A. and which B. but where c. these D. however
8. Motor vehicle collisions a re .......................... of death in infants and children.
A. prim ary reason B. frequently resulting
c. often blamed D. a leading cause
9. Smokers who try to ................cigarettes can double their chances of success
by using patches, gum, lozenges, inhalers or nasal sprays containing nicotine.
A. stop B. quit the c. stop to use D. give up
10. For humans, running a give distance requires 50 to 80 percent more energy
than walki ng ......................... distance does.
A. equivalent B. equal c. the same D. identical
III. PREPO SITIO NS AND PHRASAL VERBS
Choose the best word or phrase which best completes the following
sentences. Write your answer in the corresponding numbered boxes.
1. He.mustered '..................................... enough to go to the horror film.
A. on B. up c. in D. together
2. You have to d o ................ ... .th e matter. You have to do what he says.
A. in B. about c. toward D. with
3. The child showed no anim osi ty........................her new stepbrother.
A. toward B. with c. about D. on
4. I would appreciate it if you could .......................... me the next time we see
your mother.
A. catch up with B. stick up for c. come up with D. live up to
5. As I had never worked with autistic children, I found it rather difficult to get
.................to them.
A. on B. up c. off D. through
6. She fixed US..................... a violin teacher. We’re really grateful to her.
A. to B. up with c. along with D. together
7. Your request for greater financial support has to be ......................the claims
.
from other departments.
c. balanced against D. brushed up
A. banked on B. blown up
8. There are a few things that I don't like about my job, but ........; ...................
it’s very enjoyable.
A. by and large B. here and there
c. black and blue all over D. near and far
9. I m ust ........ ...............my Italian before going to that meeting in Rome.
A. bear out B. blow over c. brush up D. break through
10................................ .. it seems to be quite a good suggestion.
A. All in all B. All the same c. For all I care D. All but
412
IV. C OLLOCATIONS AND IDIOMS
Choose the best word or phrase which best completes the following sentences.
Write your answer in the corresponding numbered boxes.
1. A ................................... of dirt and oil lay over the surface of the pond.
A. piece B. film c. charm D. flock
2. You ca n't s im pl y..................... the blame on the government whenever things
go wrong.
. B. give c. lay D. press
A. put
3. The company offered to pay i n ................................. , so in this way we would
not have to wait until work is done to get our money.
A. advance B. arrears c. purchase D. full
4. Janet-gushed over the sculpture in the antique shop, but Kevin thought that the
craftsmanship was merely run of the mill.
A. He thinks that it is o f exceptionally high quality.
B. He thinks that it is marked by creativity or originality.
c. He thinks that it is too expensive for the average person too afford.
D. He thinks that it is low quality, common or ordinary.
5. After Michael missed three shots in a row and passed the ball to a player on
the other team, the boys were only up by two and they suspected that Michael
would be the Achilles' heel of the team.
A. Michael performs well under pressure.
B. Michael is a valuable contributor to the team.
C. Michael is a weakness in an otherwise strong entity.
D. Michael inspires others to succeed.
6. Lyle chose Marco for his partner, thinking that it would be fun to work with
his best friend, but now that the project was dlie tomorrow and the boys had
nothing done, Lyle realized that he should have hitched h is ............................
to a different wagon.
A. dog B. horse c. camel D. bull
7. While Kristie's cake pops are both delicious and artistic, nobody can .. .. .. .. ..
a candle to her pecan pie.
A. hold B. lit . c. blow D. make
8. Don't be such a.... .........Thomas. It will work. There will be no problems,"
Mary said to her husband as she tried to convince him to buy laminate
flooring for the back bedroom.
A. doubting B. quirky c. mistrusting D. wavering
9. The school staff needs n ew ..................... in order to bring in novel ideas.
A. thing B. blood c. employees D. offices
10. Could I pick your................... on the subject before the meeting?
A. intellect B. mind c. head D. brains
413
V. RE AD ING CO MPR EH EN SION
READING 1:
Read the passage and choose the best option A, B, c or D to answer the
following questions. W rite you r an swers in the co rresponding num bered boxes.
According to the controversi al sunspot theory, great storms on the surface of
the sun hurl streams of solar particles into the atmosphere, causing a shift in the
weather on earth.
A typical sunspot consists of a dark center umbra surrounded by a lighter
penumbra o f light and dark threads extending out from the center like a spoke a
wheel. Actually the sunspots are cooler than the rest of the photosphere, which
may account for their color. Typically, the temperature in a sunspot umbra is
about 4000 K. Sunspots range in size from tiny granules to complex structures
with areas stretching for billions of square miles. About 5% o f the pots are large
enough so that they can be seen without instruments: consequently, observations of
sunspots have been recorded for several thousand years.
Sunspots have been observed in arrangements of one to more than one
hundred spots, but they tend to occur in pairs. There is also a marked tendency
for the two spots of a pair to have opposite magnetic field associated with any
given sunspots is closely related to the spot’s size.
Although there is no theory that completely explains the nature and function
of sunspots, several models attempt to relate the phenomenon to magnetic fields
along the lines o f longitude from the north and south poles o f the sun.
1. What is the author’s main purpose in the passage?
A. To argue for the existence o f magnetic fields in sunspots
B. To describe the nature of sunspots
c. To propose a theory to explain sunspots
D. To compare the umbra and the penumbra
2. The word con troversial is closest in meaning to .
A. open to debate B. very complicated
c. widely accepted D. just in traduce
3. Solar particles are hurled into space by .
A. small rivers on the surface of the sun
c. changes the earth's atmosphere
4. The word par ticles refers to .
A. gas explosions in the atmosphere
c. liquid streams on the sun
B. underdetermined causes
D. disturbances o f wind
B. small pieces
D. light ray from the sun
5. How can we describe matter from the sun that enters the ea rth’s atmosphere?
A. Very small B. Very bright c. Very hard D. Very hot
6. The sunspot theory is .
A. relatively new B. widely accepted
c. subject to disagreement D. not considered important
7. The word they in line 7 refers to .
A. po ts' B. miles c. granules D. Structures
414
8. The word consequently could best be replaced by .
A. Nevertheless B. Il l this way c. Without doubt D. As a result
9. In which configuration do sunspots usually occur?
A. hl a configuration of two spots
B. In groups of several thousand spots
c. In one spot o f varying size
D. In arrangements of one hundred or more spots
10. How are sunspots explained?
A. Sunspots may be related to magnetic fields that follow longitudinal lines
on the sun.
B. Sunspots are explained by storms that occur on the earth,
c. Sunspots have no theory or model to explain them.
. D. Sunspots appear to be related to magnetic fields on the earth.
READING 2:
Read the passage and choose the best option A, B, c or D to answ er the
following questions. Write your answers in the corresponding numbered boxes.
How a man uses money - makes it, saves it, and spends it - is perhaps one o f
the best tests of practical wisdom. Although money ought by no means to be
regarded as a c hief end o f man’s life, neither is it a trifling matter, to be held in
philosophic contempt, representing as it does to so large an extent, the means of
physical comfort and social well-being. Indeed, some of the finest qualities of
human nature are intimately related to the right use of money; such as
generosity, honesty, justice, and self- sacrifice; as well as the practical virtues o f
economy and pro vid enc e. On the other hand, there are their counterparts of
avarice, fraud, injustice, and selfishness, as displayed by the inordinate lovers of
gain; and the vices of thriftlessness, extravagance, and improvidence, on the part
of those who misuse and abuse the means entrusted to them. “So that,” as is
wisely observed by Henry Taylor in his thoughtful ‘Notes from Life,’ “a right
measure and manner in getting, saving, spending, giving, taking, lending,
boiTOwing, and bequeathing, would almost argue a perfect man.”
Comfort in worldly circumstances is a condition which every man is justified
in striving to attain by all worthy means. It secures that physical satisfaction,
which is necessary for the culture of the better part of his nature; and enables
him to provide for those of his own household. Nor. ought the duty to be any the
less indifferent to us, that the respect which our fellow-men entertain for us in no
slight degree depends upon the manner in which we exercise the opportunities
which present themselves for our honourable advancement in life. The very
effort required to be made to succeed in life with this object, is of itself an
education; stimulating a man’s sense of self-respect, bringing out his practical
qualities, and disciplining him in the exercise o f patience, perseverance, and such
like virtues. The provident and careful man must necessarily be a thoughtful
415
man, for he lives not merely for the present, but with provident forecast makes
arrangements for the future. He must also be a temperate man, and exercise
the virtue of self-denial, than which nothing is so much calculated to give
strength to the character. John Sterling says truly, that “the worst education
which teaches self denial, is better than the best which teaches everything else,
and not that.’' The Romans rightly employed the same word (virtus) to designate
courage, which is in a physical sense what the other is in a moral; the highest
virtue of all being victory over ourselves.
1. What is the main idea o f this passage?
A. Wealthy people and poor people can both be virtuous.
B. Money is insignificant.
c. Money is the most important thing in the world.
D. The way a person handles money indicates his or her general character..
2. The author's purpose in writing this essay is ......... ........
A. to teach people how to earn a great deal of money.
B. to warn readers about the dangers of greed,
c. to describe the life of a wealthy person.
D. to convince the reader that proper money management is a sign of good
character.
3. Which is the best synonym for the word “providence”?
A. prudence B. fate c. sustenance D. doom
4. What would be the author’s response to those who say that poverty is noble?
A. The author would agree with this statement.
B. In order to cultivate other virtues a person must have money.
c. Once a person gets rich, they can start worrying about self-discipline.
D. The Romans believed that poor people are evil.
5. Which word best describes the author’s attitude to Henry Taylor?
A. admiring B. condescending c. skeptical D. disgusted
6. What does the author imply by saying that money provides 'physical satisfaction,
which is necessary fo r the cultivation o f the better part o f his nature' ?
A. People are more friendly after they have had a hot bath.
B. In order to improve oneself in more lofty ways, one must attain the basic
necessities.
c. The most important thing in life is physical pleasure.
D. Money can only provide physical pleasure.
7. What does the author mean by the comment, 'The very effort required to be
made to succeed in life with this object, is o f itself an ed uca tion?
A. In order to earn money a person needs to go to college.
B. Money makes people seem smarter than they are.
c. That learning to manage money effectively entails learning a number of
other valuable skills.
D. Only intelligent people can earn money.
41 6
8. Why must the ‘provi den t and car eful man’ be a thoughtful man?
A. because he has earned a great deal of money
B. because he is familiar with the works of Henry Taylor
c. because he gives most of his money to charity
D. because he must always be planning for the future
9. The author brings up the Roman word for cou rage to illustrate ...-................
A. his knowledge o f the classics
B. that people throughout history have valued money
c. that self-discipline is less important than physical bravery
D. that self-control is similar to physical bravery
10. What is the detin ition of the word “tem perate'’ as it is used in this essay?
A. moderate B. irritated c. self-denying D. warm
VI. CL OZ E-TE ST
CLOZE-TEST 1.
Read the passage below and decide which answer (A, B, c or D) best fits
each spa ce. Write y our answers in the correspon ding num bered boxes.
In addition to the challenge to be excellent, American schools have been facing
novel problems. They must (1 )______with an influx of immigrant children,
many o f whom speak little or no English. They must respond to demands (2 )___
the curriculum reflect the various cultures of all children. Schools must make
sure that students develop (3 )______ skills for the job market, and they must
consider the needs of nontraditional students, such as teenage mothers.
Schools are (4)______ these problems in ways that reflect the diversity of the
US educational system. They are hiring or training large numbers o f teachers of
English (5)______ a second language and, in some communities, setting up
bilingual schools. They are opening (6)______the traditional European-centered
curriculum to embrace material from African, Asian, and other cultures.
Schools are also teaching cognitive skills to the (7 )______40 percent of
American students who do not go on to higher education. In the (8 )_______ of a
recent report by the Commission on Achieving Necessary Skills, “A strong back,
the willingness to work, and a high school diploma were once all that was
necessary to ( 9)_____________ a start in America. They are no longer. A welldeveloped
mind, a continued willingness to learn and the ability to put
knowledge to work are the new keys (10) _ ____ the future o f our young people,
the success of our business, and the economic well-being of the nation.”
(Extractedfrom InfoUSA - CD Version)
1. A. fight
2. A. that
3. A. basics
4. A. distributing
5. A. as
B.cope c. stay D. do
B. whether
B. basic
B. delivering
B. from.
c. what
c. basis
c. discharging
c. with.
D. who
D. base
D. addressing
D. like
417
for B. up c. into D. on
fairly B. nearly c. mostly D. slightly
ways B. minds c. words D. directions
take B. get c. bring D. make
. to B. for c. in D. at

CLOZE-TEST 2.
Read the passage below and decide which answer (A, B, c or D) best fits
each space. Write your answers in the corresponding numbered boxes.
TECHNOLOGY
When faced with some new and possibly bewildering technological change,
most people react in one of two (1) ............. They either recoil from anything
new, claiming that it is unnecessary, or too complicated or that it somehow
makes life less than (2)................. Or they learn to (3) ...................... to the new
invention, and eventually (4)............... how they could possibly have existed
without it. Take computers as an example. For many of US, they still represent a
(5)............ to our freedom, and give US a frightening sense of a future in which
all (6) ............ will be taken by machines. This may be because they seem
mysterious, and difficult to understand. Ask most people what you can use a
home computer for, and you usually get (7)............ answers about how They
give you information'. In fact, even those of US who are familiar with computers,
and use them in our daily work, have very little idea of how they work. But it
does not take long to learn how to operate a business programme, even if things
occasionally go wrong for no apparent reason. Presumably much the same
happened when the telephone and the television became (8) ............. What seems
to alarm most people is the speed of technological change, rather than change
itself. And the (9)............ that are made to new technology may well have a
point to them, since change is not always an improvement. As discover during
power cuts, there is a lot to be said for the oil lamp, the coal fire, and forms of
entertainment, such as books or board games, that d on’t have to be (10) .............
in to work.
1. A. moments B. kinds c. ways D. types
2. A. formerly B. lively c. personal D. human
3. A. adapt B.react c. conform D. use
4. A. decide B. wonder c. suppose D. admit
5. A. hazard B. risk c. control D. threat
6. A. measures B. decisions c.chances D. instruction
7. A. vague B. such c. up with D. hundreds
8. A. in existence B widespread c. through D. extensive
9. A. objections . B appliances c. criticisms D. fears
10. A. wired B batteries c. plugged D. connected
418
B. WRITTEN TEST:
1. OPEN CLOZE TEST : Read the text below and think of the word which
best Fits each space. Use only ONE WORD for each space.
CLOZE TEST 1
One of the greatest problems with holidays, (1)___________ from the usual
travel complications and accommodation difficulties, is the expectations people
have of (2)____________ . When we go on holiday we expect to leave all the
stresses and strains of our daily lives (3)___________ US. We imagine we will be
able to escape to such a degree that we even tend to believe, consciously or not,
that we can change our own personalities and become completely (4)_________
people. The average business-person, tense, preoccupied, short-tempered,
(5)_________ to relax, envisages herself I himself (6)______________ , from the
moment of locking the office door, a radically different (7)_________ of person:
carefree, good-humoured, ready to relax and enjoy whatever adventures present
themselves. In practice, we take ourselves with US wherever we go, and the
personality that is shaped (8)____________ years of stress and tension is almost
impossible to shake off (9 )______________a mom ent's notice. It is no wonder
so many holidays are a disappointment, no matter how smoothly they go or how
lovely the weather is. In fact, the frequent problems that crop (10)____________
during the average holiday are probably a welcome distraction from the nagging
feeling that we are not enjoying ourselves as much as we should.
CLOZE TEST 2
Much has been heard recently about possible health hazards, including
memory loss and brain tumours, from the use of mobile phones. With the
possible half a billion mobile phones in (1)_________ throughout the world, in
Britain alone, one person in four owns one, (2)__________is worrying enough,
even if, so far, no concrete evidence has come to (3)______________ . One study
by Dr. Alan Preece and his team at Bristol University has shown, however, in a
report in the International Journal of Radiation Biology, that tests on volunteers
demonstrated no effect on their short-term memory or attention (4)___________ .
Subjects were exposed to microwave radiation for (5)______________ to thirty
minutes, but the one noticeable effect was positive rather than negative; the
subjects reacted more rapidly in one test (6)______________ a visual choice.
One explanation of this is that following the transmissions, a warming of the
blood led to increased blood flow. For the experiment, places were chosen where
the signal was good and the microwave dose light, and then where the signal was
poor and the dose (7)______________ higher. The subjects were tested for recall
and mental alertness (8)______________ exposure to microwaves characteristic
of analogue phones, digital phones or no phones at all, without knowing
(9)_______________ they were exposed to. It is, of course, early days yet and the
sample may not be large enough to generalise (10)_____________ . More
research needs to be done.
419
II. WORD FORM
PARTI:
Complete each sentence, using the correct form o f the word in parentheses.
1. She w or ke d.................................. to help homeless people. (STINT)
2. Apart from so m e.................... .........efforts, the government have not yet made
a serious attempt to effectively address the Scottish fisheries issue. (HEART)
3. As Tet holiday is coming, we’ve got an ......................................... weekend
ahead of US. (ACTION)
4. The effects of alcohol misuse spill over from private life into the workplace,
causing inefficiency and accidents as well as ............:.................... (ABSENT)
5. There was far too much light and all my photos w er e.......................................
(EXPOSURE)
6. The organization works in many war-torn and .................................. countries.
(POOR)
7. As an environmentally aware consumer, it is important for you to know in
what way you can help to make sure that le ss .....................................material
is left on the planet. (DEGRADE)
8............................................... has left a lot of workers with the risk of losing
their jobs, as they were recruited on a temporary basis. (CASUAL)
9. The waitress came round with a tray o f.............. .......cream cakes. (MOUTH)
IQ. He was the perfect son and brother and the pride in him from the family
i s ..............................*.......immense. (JUSTICE)
PART 2: Complete the passage with appropriate forms from the words
given in the box.
immune form invade introduce
success assist place effect
UNWANTED VISITORS
Loss of habitat poses the single greatest threat, endangering indigenous
species. The second largest threat to native flora and fauna would have to be the
(1 )......... ................. of alien species into an environment other than their own.
Alien species are able to cause such cataclysmic damage because they are usually
more ( 2)........................ in competing for food. They introduce diseases to which
the local inhabitants do not possess (3 )..........................Interbreeding has caused
the destruction of entire species because the first hybrid generation will eventually
(4 )............ . the parent stock. Hybrid individuals tend to possess greater vigour
and will therefore compete more (5) ...................... with the remaining pure
stock. Their offspring may also be infertile, resulting in the (6 )...... ....................
of an entire species because of a reduction in the number of breeding animals.
The (7) ......................... of guidelines has been called on to exclude non-native
wildlife, contain it where it has a foothold, and eliminate it if possible. The
principles call for border controls, (8) ............... . in international trade and
technical and financial ( 9).................................. to help poor countries detect and
combat (1 0).................................
420
III. ERROR IDENTIFICATION
The following passage contains 10 errors. Identify and correct them.
Severe hurricane force winds buffeted their way across Kent and much of the
south-east of England last night, leaving a trail of devastation from their wake.
Vast tracts of the county have been flattened, and damage to property estimated
at hundreds of millions of pounds.
Yesterday evening there was little warning of what was about to come. Wind
began to pick up just after 11 p.m. and by 2 a.m. storm force winds had hit towns
on the south coast. Later, the winds increased violence until they eventually
reached hurricane force in the early hours of the morning, lashing the country with
gusts of over 130 miles an hour. Hospitals were warned to expect casualties.
Kent has been very badly hit. In Sevenoaks, an entire forest was flattened by
the blast and uprooted trees were littered over the countryside as matchsticks.
Elsewhere there has been considerable damage to crops and buildings. David
Hart, from Lamberhurst, awoke to find that his roof had been blown off: “ I was
woken up by an incredible noise - it was just like a bomb going off. 1 could see
the roo f flapping about in the wind. Then it just flew off, and slates and bricks
came to crash down through the ceiling.”
Concern has been risen at the failure of the authorities to provide adequate
warning. A spokesperson from the Meteorological Office explained that storm
force winds had been forecast, that the hurricane itself had been caused by a
combination of freak weather conditions.
The first priority for Kent County Council is the restoration o f communications,
and the government is considering giving emergency aid. Therefore, insurance
companies are bracing themselves to a deluge of claims that could well run to
nearly a billion pounds.
1. 2. 3.
4. 5. 6.
7. 8. 9.
10.
IV. SENTENCE
TRANSFORMATION
Rewrite the following
sentences, using the given
words.
1. The house shouldn’t be left unlocked for any reason .
N o t. . .................................. ......................................................................
2. The restoration o f communications and essential services is o f prime importance
for the council.
The f ir st ................................................................................................................
3. The government have been reviewing their immigration policy for some time.
The g overnment’s.............................................. .................. .................................
4. He declared his disapproval of the behavior of some of his supporters.
He let i t .................................................................. ...............................................
421
5. A new flu vaccine has been on trial since the beginning of the year.
The y.................................................. ....................................................................
6. They may have escaped through the backdoor. GETAWAY
The y........................................................................................................................
7. You've been looking miserable all day. MOON
You’v e ....................................................' ..............................................................
8. Jim knew he would have to go on the business trip at a moment's notice.
POISED
Jim ...................................... .................................................................................
9. The waiters in the new restaurant were very inefficient. SHODDY
We re ceived.............................. ...........................................................................
10. Sean did all the illustrations for the book but no one acknowledged his work.
CREDIT
Sean w as n't.........................................................................................................
TRƯỜNG THPT CHUYÊN TRẦN ĐẠI NGHĨA - T P. Hồ CHI MINH
A. MULTIPLE CHOICE (40 PT S)
I. WORD CHOICE (5 PTS)
Choose the best options to complete the following sentences.
1. The trial has been adjourned because no new an d _________ evidence has
been uncovered.
A. damning B. flattering c. withering D. prevailing
2. The former leader of the organization has expressed a wish to _________ for
his past sins.
A. atone B. annex c. adore D. abridge
3. The ultramodern printers can reproduce photographs with amazing_______ .
A. felicity B. frugality c. fidelity D. futility
4. With a wide range of qualifications and a wealth of experience, Rosa is
the most suitable candidate.
A. haphazardly B. begrudgingly c. jovially D. indubitably
5. The _________ soil absorbed the rain almost as rapidly as it fell.
A. sinewy B. porous c. fibrous D. prevailing
6. As a dietician, she of ten _________ the virtues of eating less fat.
A. exiles B. extols c. exudes D. exceeds
7. Salespersons often try to please their customers by using artif icial_________ .
A. eulogy B. kudos c. flattery D. homage
8. This area is very dangerous. In order not to g e t_________ on their way,
women are advised not to go out alone at night.
A. uplifted B. stonewalled c. browbeaten D. waylaid
9. In the morning sermon, the missio nary_________ talks about his religious
beliefs, his face glowing with enthusiasm.
A. dolefully B. insidiously c. zealously D. spasmodically
422
10. Even though my mother despised her twin sister, she always spoke about her
without __________.
A. conferment B. sublime c. rancour D. ignominy
11. GR AMMAR AND STRUCTURES (5 PTS)
Choose the best options to complete the following sentences.
11 •______ , I would give a party.
A. Were she to come next month B. She were to come next month
c. If she comes next month D. Should she come next month
12. On the battle fie ld_____ .
A. the tanks lay B. did the tanks lie c. lay the tanks D. lied the tanks
13. ________ , the catfish is prized for its taste.
A. With ugly look B. As ugly looking
c. Ugly looking as it is D. As it is ugly looking
14. The estranged husband desperately tried to get his message ac ro ss,______
without success.
A. despite B. however c. even though D. albeit
15. Janine's parents died when she was just a baby, so 1___ family she ever had.
A. was all the B. was the whole c. have the whole D. have all the
16. Despite many sophisticated techniques, the simple magnifying glass and gut
tools for unmasking fake paintings.
A. instinct remains the best B. instinct remain the best
c. instinctive remain the best D. instinctively remains the best
17. Mr. Gump sup poses,________ that he will retire at 60.
A. like most people did B. as mo st of people
c. like most people do D. as do most people
18. _______ that Mr. Jones is planning to run for governor this year.
A. Word is it B. Word has it
c. Word has D. The words are
19. The endless parade o f________ on television has made today’s young girls
obsessed with their bodies.
A. celebrities enhancing surgically B. surgically- enhanced celebrities
c. surgical celebrities enhanced D. enhanced surgically celebrities
20. At the stroke of midnight, I’m afraid t he re_________ .
A. wo n’t shops be left open B. won’t be shops left open
c. will be shops left not open D. will be no shops left open
il l. PHRASAL VERBS AND PREPOSITIONS (5 PTS)
Choose the best options to complete the following sentences.
21. The rebellion t oo k____ the reins of government immediately after the coup.
A. out B. over c. against D. in
22. Such a savage punishment must be abolished as it is abhorrent_________ a
civilized society.
A. with B. for c. to D. in
423
23. Dave is not a good boss because he's always palm the worst jo b s_________
his assistant.
A. out with . B. up for c. off on D. into
24. Steve Harvey brought the house down again. He_______ his s ho w_______
a lot o f jokes.
A. peppered - with B. salted - down c. seasoned - with D. sugared - 0
25. Her husband walked out_________ her and their three children.
A. on B. of c. to D. from
26. When the book was made into a screenplay, we decided to _________ some
of the comical anecdotes.
A. miss out B. amount to c. cutout D. tot up
27. It's hard to find accommodation during the high season, so the whole family
wer e_________ up in a small room.
A. penciled B. penned c. papered D. booked
28. A gang o f young men were b ooted _________ the pub for fighting.
A. away from B. up with c. down on D. out o f
29. My teacher advises me to juice _________ my presentation with more
colorful illustrations.
A. down B. up c. over D. off
30. We are fed up with him. He k eeps_____, his new girlfriend.
A. snaking away into B. horsing around with
c. rabbiting on about D. ducking out of
IV. COLLOCATIONS & IDIOMS (5 PTS)
31. She is a typical modern woman who _ __
the double burden o f
childcare and full-time work.
A. shoulders B. falls on
32. I'm going to call George’s ___
- I don't believe he’ll really carry out
his threat.
A. blood B. bluff
33.
c. lifts D. lightens
c. bid D. brave
by their new finds, the archeologists say they had made dramatic
new finds of Viking settlements.
A. Buoyed up B. Irked out c. Sounded off D. Perked up
34. Sarah looks back at her childhood on a tiny island throug h________
she
refuses to remember the difficulties she experienced.
A. wishful thinking B. a sweeping statement
c. rose-coloured spectacles D. a mixed blessing
35. Mr. and Mrs. Jones are such permissive parents that their son ca n____
no matter what he insists.
A. jump on the bandwagon
c. make their blood boil
36. The tailor found a
B, go against the grain
D. wind them around his little finger
in making high-quality suits for very tall and
very heavy gentlemen.
A. legal tender B. backing store c. niche market D. rummage sale
424
37. You sho uld n't tell the group about Dad's problems because you don't_____ .
A. blow your own trumpet B. go out on a limb
c. air your dirty laundry D. steal his thunder
38. After being woken up by the sound of someone moving around, she returned
to bed and wen t_________ .
A. under a cloud B. out like a light c. as the driven snow D. like a wind
39. All o f these knock-off superhero movies are clearly ________ on the back o f
the original franchise.
A. taking B. getting c. riding D. waking
40. Living apart from each other, Mike and Tara have been talking______ for a
year now.
A. off and on B. as a matter of course
c. till the cows come home D. in the least
V. G UIDED CLOZE (10 PTS)
Read the texts below and decide which answer best fits each space.
PASSAGE A:
Is Prince William an embryonic bard? A research team in Germany claim
they have found evidence that he is descended from Shakespeare and may thus
have inherited literary genius.
Frustrated by a lack of first-hand evidence, researchers trying to (41) _ _ _ _ _
together details o f the Bard's life have long turned to his sonnets as the only
words of his that might be autobiographical. For centuries, academics have been
trying to solve.the (4 2) _________ riddle of the ‘Dark Lady', the mystery person
to whom Shakespeare addressed his sonnets. Those involved in the most recent
detective (43) ______- have come up with some evidence that the Bard's
bloodline is linked to the youngest generation of the royal family.
This bold claim is (4 4 )_________ by clues hidden in paintings of a
previously unidentified noblewoman, to be named by a German academic team
as Shakespeare’s dark-haired lover. They were assisted by forensic experts from
the German police.
But who was the Dark Lady? So many (45) ._______ 2 theories have been
advanced that some scholars have abandoned the search. In fact, the answer may
be (4 6) _________ us in the face. According to one eminent academic, a portrait
of the mystery woman is on show in Hampton Court Palace in London,
(47) _________ it is known as The Persian Lady. She argues that the pregnant
woman depicted there is Elizabeth Vernon, a lady-in-waiting to Queen Elizabeth,
who, after an illicit affair with Shakespeare, went on to marry his patron.
(48) _________ this woman, Elizabeth, third Countess of Southampton, bore
Shakespeare a daughter, Penelope, who grew up to tie the (4 9)_________ with
William, second Baron spencer, and their descendant was the father of Diana,
Princess o f Wales and grandfather o f Prince William.
425
So far. Prince William 's talents have shown themselves in the sports field.
But who knows? His uncle, Earl Spencer, did after all receive world-wide
acclaim for his (5 0) _________ at the funeral of his sister.
41. A. part B. mold c. piece D. weld
42. A. enticing B. appalling c. tantalizing D. petrifying
43. A. series B.hunt c. wave D. thread
44. A. held up B. rallied round c. backed up D. stuck up f
45. A. rival B. no-nonsense c. holistic D. orthodox
46. A. staring B. looking c. glaring D. ogling
47. A. lest B. where c. for D. while
48. A. Supposedly B. Strangely c. Knowingly D. Seemingly
49. A, loop B.bond c. hitch D. knot
50. A. monologue B. elocution c. oratory D. address
PASSAGE B:
THE CONCEPT OF EVOLUTION
Charles Darwin may well be the most controversial scientist who ever lived.
Although ‘On the Origin o f Specie s’was his highly acclaimed book, he did not
invent the concept o f evolution. When he was a student in Edinburgh in the late
1820’s, evolution was already the (5 1)_________ of the town. But evolution
was rejected by the establishment. Those who (5 2)_________ to evolutionary
thinking were called Lamarckists, after the French scientist Jean-Baptiste
Lamarck first proposed that species are not static but change over time and give
(5 3) _________ to new species. Lamarck had presented this ( 54 )__________in a
book published in 1809. However, a correct mechanism for (55) _________
species change into each other was missed out. Later on, the mechanism was
discovered first by Charles Darwin and independently by Alfred Russel Wallace.
From reading the economist Thomas Malthus’s works, Darwin was aware of the
consequences of (56)_________ growing populations. Once resources become
limited, only a (57 )_________ of individuals can survive. Additionally, Darwin,'
as an (5 8) _________ observer of animal breeders, analysed their (59) -
and studied their results. Slowly but (6 0)_________ , he understood that nature
is like a gigantic breeder.

51. A. talk B. topic c. gossip D. subject


52. A. combined B. cohered c. cleaved D. complied
53. A. occasion B. evidence c. raise D. rise
54. A. standpoint B. mindset c. panorama D. vista
55. A. which B. how c. way D. mode
56. A. expressly B. exponentially c. exquisitely D. exclusivel
57. A. plethora B. division c. portion D. fraction
58. A. agile B. arduous c. ardent D. amorous
59. A. methodologies B. methods c. manners D. behaviors
60. A. surely B. definitely c. ultimately D. clearly
426
IV. READING COMPREHENSIO N (10 PTS)
Read the texts below and choose the best answer to each question.
PASSAGE A
“PLUM PUDDING” MODEL OF ATOMS
Througho ut history, scientists have believed that the atom was the most
fundamental unit of matter. However, at the end o f the 19th century, new studies
in atomic theory revealed that atoms were composed of even more fundamental
units, in order to understand these units, scientists first had to construct a new,
more comprehensive model of the atom. Because tiny atoms defy any direct
observations, even with the most advanced technology or instruments, scientists
conducted experiments in which they sought to observe the effects of sub-atomic
particles, rather than observing the atoms themselves. The results of such
demonstrations eventually helped scientists develop an atomic model with the most
specific details about matter’s structure and composition, but it was a process oLtrial
and error; there were many revisions and corrections made along the way. One .of
the scientists involved in this process of trial and error was J.J. Thomson.
Thomson, an Englishman, developed what came to be known as the “plum
pudding” model of atomic structure in 1900 while experimenting with cathode
rays. In previous experiments with cathode rays, scientists would propel gases
through vacuum tubes and then observe flashes of light; they thought this light
came from some nebulous substance that saturated all things. After several
similar tests, Thomson determined, that these flashes of light were actually
electrified particles, which he first called “corpuscles.” All gases that passed
through the tubes gave off these particles, which had negative charges that
caused them to glow. Through his observations, Thomson also discovered that,
like atoms, these corpuscles were found in all forms of matter, yet they were
even smaller than atoms. Essentially, Thomson had found out that the
fundamental units of matter were composed of even smaller particles
themselves, and Thomson’s corpuscles came to be known as electrons.
Thomson now knew that atoms contained electrons, but he was not certain
how electrons were arranged within the atom. Thomson proposed a “plum
pudding" model of atoms where several electrons were scattered randomly in a
positively charged field, like plums in a pudding. In this model, atoms
themselves were mostly empty space because the charged field was not solid, so
other particles could pass through it without obstruction. Later studies confirmed
that atoms were mostly empty space, but the plum pudding model still had some
inaccuracies. For instance, Thomson proposed that atoms were uniform for all
elements. He also thought that electrons could move in elliptical orbits that were
stabilized by the positively charged field. The further an electron moved from
the center, the more the field would pull it back in place.
A decade later, another scientist, Ernest Rutherford, performed experiments
that would debunk the “plum pudding” model and coiTect many of its inaccuracies.
427
Rutherford fired several particles through a thin sheet of gold foil, speculating
that they should all pass through or be slightly deflected. While most of the
particles did pass, a small number were directly reflected off the sheet.
Rutherford concluded that there were small clusters of matter that reflected the
particles, so an atom wasn’t completely empty space. It also had a nucleus, or a
concentrated center. He also deduced something else: the positive charge wasn't
in an insubstantial energy field but in a relatively dense, central cluster of
particles - this meant that electrons were on the outside.
Rutherford's critical discovery established a different model of the atom, one
in which the atom has a nucleus and electron orbits. Rutherford constructed this
atomic model based on both his own discoveries and previous atomic models.
This new model also compared the nucleus’s pull on electrons to the sun’s
gravitational pull on Earth and other planets. Future models would expand upon
this concept and figure in quantum factors to electron orbits;- this ultimately led
to the realization that only certain amounts of electrons could be placed in
certain orbits. Further improvements to the atomic model also established that
different elements had different number of electrons, protons, and neutrons. The
various models would build upon previous discoveries and inaccuracies in order
to advance understanding of atomic theory.
61. The phrase “such demonstrations” in the passage refers to
A. observations B. experiments c. movements D. results
62. According to paragraph I. what can be inferred about atomic models?
A. They are no longer considered to be of much importance to science.
B. Many early models had to be proven incorrect before a workable model
was found.
c. They are inherently inaccurate because they must be based on indirect
observations.
D. They developed only as more powerful microscopes became available.
63. The word “nebulous” in the passage is closest in meaning to
A. blazing B. indistinct c.fractured D. limited
64. The word “them ’■in the passage refers to
A. particles B. gases c.tubes D. charges
65. According to paragraph 2, what was J J. Thomson's discovery?
A. the electron B. the cathode rayc. the atom . D. the vacuum tube
66. The word “obstruction” in the passage is closest in meaning to
A. osmosis B. detonating c.interference D. accelerating
67. According to the information in paragraph- 3, what aspect of Thomso n's
theory was confirmed by later scientists?
A. Electrons are scattered in atoms. B. Atoms have an outer positive field,
c. Atoms are uniform for all matter. D. Atoms are mostly empty space.
428
68. The word “debunk” in the passage is closest in meaning to
A. support B. disprove c. amend D. ignore
69. According to paragraph 4, Ernest Rutherford discovered that
A. atoms had mass in their centers. B. electrons had quantum factors,
c. all matter emitted electrons. D. particles always went through atoms.
70. Based on the inform ation in paragraph 5, what can be inferred about
Ruthe rford's model for the atom?
A. It heavily borrowed details from the “plum pudding” model.
B. It is currently the scientifically accepted model for the atom,
c. It is controversial and competing with other atomic models.
D. It also had flaws and inaccuracies that were later corrected.
PASSAGE B
SET ASIDE TIME EACH DAY
Most of us can fine 15 minutes or half an hour each day for some specific
regular activity. It may be a free period or a regular wait, say in the queue for a
bus or meal- even while eating breakfast. One famous surgeon always made it a
rule to spend at least 15 minutes on general reading before he went to sleep each
night. Whether he went to bed at 10 p.m. or 2-30 a.m. made no difference. Even
if you cannot keep to this kind of discipline, it is a good idea to make sure you
always have a general interest book in your pocket. Don 't forget it should be a
book which entertains you and the English must not be too difficult for you.
Check your progress through pacing
Nearly all speed reading courses have a “pacing” element - some timing
device which lets the student know how many words a minute he is reading. You
can do this simply by looking at your watch every 5 or 10 minutes and noting
down the page number you have reached. Check the average number of words
per page for the particular book you are reading. How do you know when 5
minutes have passed on your watch if you are busy reading the book? Well, this is
difficult at first. A friend can help by timing you over a set period, or you can read
within hearing distance of a public clock which strikes the quarter hours. Pace
yourself every three or four days, always with the same kind of easy, general
interest book. You should soon notice your habitual w.p.m. rate creeping up.
Check comprehension
Obviously there is little point in increasing your w.p.m. rate if you do not
understand what you are reading. When you are consciously trying to increase
your reading speed, stop after every chapter (if you are reading a novel) or every
section or group of ten or twelve pages (if it is a textbook) and ask yourself a few
questions about what you have been reading. If you find you have lost the thread
of the story, or you cannot remember clearly the details o f what was said, re-read
the section or chapter.
429
“Lig htning speed” exe rcise
Try this from tim e to time. Tak e fou r or five pages of the gen eral interest
boo k you hap pen to be readin g at that time. Read them as fast as you pos sibly
can. Do not bother abo ut whether you unde rstand or not. No w go bac k and read
them at wha t you feel to be your “no rmal” w.p.m. rate, the rate at which you can
com fortably understand. After a “lightning speed” read thro ugh (proba bly around
600 w.p .m.) you will usually find that your “no rmal” speed has increas e - perhaps
by as muc h as 50 -100 w.p.m. This is the technique athlete s use when they
habitually run further in training than they will have to on the day o f the big race.
71. The p assage recom me nds setting as ide for reading pra ctice
A. two h ours a day B. one hou r a day
c. 15 minutes or h alf an hour a day D. three tim es a day before meals
72. One fam ous surgeo n alw ays m ade it a rule to read
A. for 15 minutes at 10 p.m. each night.
B. a t least 15 min utes at bedtime
c . at least 15 min utes at eith er 10 p.m. or 2.30 a.rn.
D. w hethe r h e had a spare mom ent.
73. It is a good idea alwa ys to carry in yo ur poc ket
A. a book you will nev er forget B. a serious book
c. seve ral boo ks of various kinds D. an e asy and ent ertaining English book
74. A “ pac ing ” d evic e
A. tim es a s tud ent’s reading speed
B. is not included in m ost speed reading courses
c. is an aid to voc abu lary learning
D. shou ld be used w hen eve r we read alone.
75. Loo king at your watch every 5 or 10 minutes
A. avo ids the need f or reading faster
B. is no t the sam e as pacing
c. is not easy at first
D. help you to rem em ber the page num ber you were at last time
76. The passage recommends pacing yours elf
A. eve ry two days w ith different kinds o f book
B. every thre e or four days with the. same kind of boo k
c. every week with the same kind of book
D. as o ften as you read a book
77. When you are reading a novel the passage advises you to pause to check the content
A. every chapter B. every hour
c. every thre e or four p ages D. after every p age
78. The p urpose of pausing for though t every chap ter so often is to
A. rest th e eyes
B. make sure you have not missed any pages
c. mak e sure you really und erstand what you have read
D. pre ven t brain fatigue
430
79. If you have lost the thread of a story you are reading, the passage recommends
A. choosin g an easier book
B. glancing back over the chapter you*have just read
c. asking a friend to help you with the difficult words
D. learning the previous chapter by heart
80. The purpose of the lightning speed exercise is to
A. increase your normal speed by practicing at a very high rate
B. get through the book in half the time so that you can go on to the next
c. help you understand more of the content o f the book
D. enable you to win reading races against your friends
B. WRITTEN TEST
I. CLOZE TEST (20 PTS)
Read the text below and think of the word which best fits each space. Use
only ONE WORD for each space.
PASSAGE A
Chewing gum, (1 )_________ now considered the ultimate junk food and the
national dish of the gormless, turns out to have a positive effect on cognitive
performance. Ịn the first intellectual victory for supporters of progressive
education in many years, it seems that the teacher (2 )___________ the child at
the back of the class to ‘spit it ouf has got it (3)_________ wrong. The Human
Cognitive Neuro-Science Unit at the University of Northumbria has tested the
thinking and memory of those who chew gum. The results show that gumchewers perform far
better in cognitive tests than those who did not partake.
This new information makes many things clear that were previously
(4 )________ in mystery. It was (5 )__________of Gerald Ford that he could
walk and chew gum at the same time. That he proved so bad at walking and was
constantly falling over was seen as a (6 )________ that he was not up to the job.
Now it is apparent that faced with the choice between walking and masticating
he picked wisely. Even though he fell down the steps of many aircraft, his
performance ( 7)_________ he reached the ground was greatly enhanced.
The beneficial effects of gum may come as a surprise to some, but chewers
themselves, (8 )_________ intellectually superior to everyone else, of course,
have been aware of gum 's advantages for years. Sadly, some politicians want to
tax gum to pay for the price of. scraping discarded pieces from the street. It can
confidently be predicted that such small-mindedness (9 )_____ ___ non-chewers
will leave the mouths of gum consumers everywhere gaping (1 0) _________ .
PASSAGE B
KINGSTON
Overw helming and fascinating in equal (1) _________ , Kingston is quite
unlike anywhere else in the Caribbean. (2 )_________ its troubled reputation, it’s
hardly surprising that few tourists visit, and while the scare stories are absurdly
431
exaggerated, Kingston is certainly not a place for the faint-hearted. In the 1950’s,
Ian Fleming called it a "tough city", and that still (3 )____________ true today.
Jamaica's capital is rough and (4) ___ _____ , a little uncom promising, but
always exciting and absorbing. With just under 600,00 residents, Kingston
(5 )_________ with life, noise and activity, and if you venture downtown, you'll
see the (6 )_________ edges. Nonetheless, the capital offers a look at a side of
Jamaica that couldn't be more different from the resorts. As well as being the
seat of government and the island's administrative centre, Kingston is Jamaica's
cultural heart, the city that spawned Bob Marley, Buju Banton, Beenie Man and
countless other reggae stars. If you do decide to visit - and it's well worth the
effort for anyone with even a ( 7 )_________ interest in Jamaican culture - you'll
find that not only is it easy to steer clear (8 )____________ the troubled areas,
but that there's ( 9 ) _________• of the persistent harassment that bedevils parts
(1 0) ______' the north coast.
II. WOR D FO RM AT IO N (20 PTS)
PART 1:
Comp lete each sentence, using the correct form o f the word in parentheses.
1. Copernicus justified his innovation by citing ________ authorities. (PE AC H)
2. The strike yvas engineered by a handful o f _________ who whined about the
smallest issues. (CON TE NT )
3. The attorney 's rambling________ _ in court proved that he was unprepared
for the trial and was flying by the seat of his pants. (TEM PO RA RY )
4. While the restaurant's kitchen is not overly attractive, it s _________ layout
allows the cooks to prepare meals quickly and competently. (UTILIZE )
5. Inside the American Museum of Natural History w er e_________ preserved
specimens of wild animals, which beckoned every tourist to enter and take
photographs. (TA XI)
6. It’s very difficult to tell th e_________ difference between the identical twins.
(M EA SU RE )
7. The president visited a local community on the outskirts of Da Nang in the
immed iate_________ _ of Typhoon Damrey. (MAT H)
8. In order to raise funds for impoverished musicians, an auction of Beatles
_________ is to be held next week. (MEM OR Y)
9. The doctor prescribed _________ drugs to reduce the old man's arthritis.
(FLA ME )
10. Immigration has played a significant role in the development of the US
economy; however, this ever increasing __ _____ _ of the nation 's population
will bring about a whole new set of challenges. (GEN ER OU S)
PART 2:
Complete tile passage with appropriate forms from the words given in the box.
ascend deny fulfill long
odd possess prove right

432

DICKENS AND HIS WORLD


With the circulation of Pickwick Papers in 1836, young Dickens enjoyed an
unprecedented (1 1)_________ into the favour of the British reading public. He
magnificently (1 2 )_________ a theory that his fame would disappear just as
quickly as it had come. He remained until his death 34 years later ( 13 )_____ __
the most popular novelist the'English-speaking world had ever known.
The public displayed an insatiable appetite for his works, and there was also a
great diffusion of them through (1 4) _________ dramatic adaptations (nearly all
completely unauthorized, the (15 )_________ laws being weaker).
His immense popularity was based on the widespread perception of him as a
great champion o f the poor and (16)_________ against all forms o f injustice and
abuse of power. In his personal life, however, he was incapable of achieving the
level of (17) _________ that he enjoyed with the public, and all his close
emotional relationships with women (18)_________ ended in failure. Yet out o f
his needs and fears, his disappointments and his (19) _________ , Dickens
created an extraordinary range and variety of female characters. They live on in
our minds and our culture in all their (2 0) _________ and distinctiveness, unlike
any other female characters created by Victorian novelists, no matter how well
they may have understood women.
III. ERROR CORRECTION (10 PTS)
The following passage contains 10 errors. Identify and correct them.
AN UGLY HABIT
Smoking is one of the most common and dead habits in the world. You have
probably seen thousand of cigarettes smoked in your life time, but perhaps not
by your family. Even though few people smoke today than in the past, one in
every four adults still smokes, and there are parts of the world where smoking is
increasing. Most people who become regular smokers started when they were
young. This is the time to get the facts straight: smoking does no one not good,
and it does a great number o f harm to your health. It also often means giving up
a lot later in life, such as the chance to excel in sports, extra spending money,
and even years of one's life. There is much to lose. Most smokers have a hard
time explaining why they started - and why they continue. They know it is
harmful, and much even know someone who has died for a smoking-related
illness, like lung cancer or heart disease. But at the same time, these smokers
continue lighting up when they go out for a drink, take a break from work, or
hear alarmed news. Some smokers even light up when they learn about the
dangers of smoking, because they become worried. Whatever the reason people
start smoking, the habit soon loses its attraction. Apart from the obvious health
risks, smoking is an ugly, unpleasant habit. Most people would prefer avoiding a
room that someone was filling with smelly smoke.
4 ._______________
7 ._______________
2 ._______________
5 .__________ _____
8._______________
3 ._______________
6 .______________
9. _______________
10.
433
IV. SE NT EN CE TR AN SF OR MAT IO N (20 PTS)
Rewrite the following sentences using the words given.
1. I'm not in the habit of sleeping in the afternoon.
—> R are ly_______________________________________________________
2. So smart and flexible was the coach’s tactics that Vie tnam's national football
team achieved a great success in the AFC Championship
-> It w as _________________________________ Vietnam ’s national football
team to the victory in the AFC Championship.
3.1 know he has tried hard; but his work is unacceptable.
-» I know he has tried hard; b e_______________ _______________________
were good enough.
4. it is common knowledge that a cooked meal is far more nutritious than junk food.
—» Junk food can nowh ere ___________________________________ belief.
5. 1 didn ’t have the faintest idea about the quirks of the antiques business, so I
couldn’t give you any advice. (KE N)
-» H ad _________________________________
6. His parents allowed him to buy an iPhone X after they had considered it for
three weeks. (GO-AH EA D)
-> Having mulled ___________ •________________________________ • .
7. Although the people here are of the working class, they are kindness and
honesty personified. (SA LT )
—> Notwithstanding th eir________________________________________
8. Tom is forever teasing Daisy for having spoonerism. (M ICKE Y)
—» On acc ount_________________________________________________
9. It was about time I went over there and made up with her. (OLIVE )
—> It was about time
10. Remem ber to check out our thirteen language learning tips! (BAK ER )
—> Remember t o _______________ ________________________________
TRtfflNG THPT MẠC ĐĨNH CHa ■ TP. Hồ CHÍ MINH
A. MULTIPLE CHOICE QU ESTION S
I. P HO NO LO GY (5 P TS)
Choose the word whose underlined part is pronounced differently from that
of the others.
fathom B. feather c. anthem D. southern
blizzard B. puzzle c . pizza D. drizzle
libel B. revival c . Hable D. liberal
saboteur B. amateur c. entrepreneur D. hauteur
tabulate B. draconian c . sanctuary D. plateau
434
Choose the word
that of the others.
6. A. tuberculosis
7. A. infamous
8. A. expertise
9. A. sanctuary
10. A. peninsula
whose main stressed syllable is placed differently from
B. mathematician c
B. impotent c
B. hypocrisy c
B. perseverance c
B. pneumonia c
psychopathology D. complimentary
affidavit D. inventory
D. paralysis
D. conscientious
D. euphemism
respiratory
practicality
metropolis
II. WORD CHOICE (5 PTS)
Choose the best answer to complete each of the following sentences.
11. The dro ught___________ considerable problems for farmers.
A. instigated B. posed c. flexed D. pressed
12. Simo n_______ in me on the understanding that 1 wouldn’t tell anyone else.
A. confided B. trusted c. confessed D. disclosed
13. Would passengers plea se___________ from using any electronic equipment
until the plane is airborne?
A. exclude B. resist c. restrain D. refrain
14. We talked to each other all night and resolved some of our problems. It'
good to have a proper _______sometimes.
A. head-to-head B. heart-to-head c. head-to-heart D. heart-to-heart
15. My father___________ when he found out that I’d damaged the car.
A. hit the roo f B. saw pink elephants
c. made my blood boil D. brought the house down
16. The optician says you have to wear glasses, like it o r_________ it.
A. jump B. lump c. dislike D. loathe
17. You will be putting your life on th e_________ if you take up skydiving.
A. ground B. way c. lane D. line
18. His friends and family left him in the __________ when he went bankrupt.
A. church B. lurch c. end D. street
19. Hotel rooms must b e __________ by noon, but luggage must be left with the
porter.
A. vacated B. evacuated c. abandoned D. left
20. Tara was really laying it on __________ about her accident at work.
A. fine B. broad c. thick D. thin
HI. STRUCTURES AND GRAMMAR (5 PTS)
Choose the best answer to complete each of the following sentences.
21. Unsalted butter is best for this recipe, b ut _________ that, margarine will do.
A. except B. failing C. for all of D. given
22. In front o f the doo r___________.
A. a dog sat B. was seated a dog c. seated a dog D. was a dog seating
435
23. Every Christmas of my childhood was the same. My f athe r_______ late for
lunch, weighed down with presents for the family.
A. would arrive B. had arrived
c. was arriving D. was used to arriving
24. I would never have entered the woods so sli gh tly _______________ I was
trespassing on his domain.
A. did I know B. for all I knew c. 1 had known D. had I known
25. The sudden expansion of heated air associated with lightning produces
often heard during a storm.
A. thunder is the rumbling sound B. the rumbling sound, thunder is
c. the rumbling sound, thunder, that
26. We all decreed th at __________ .
A. there be an end to their quarrel
c. they ended their quarrel then
27. I'd rather yo u_________
D. thunder, the rumbling sound
B. their quarrel should put an end to
D. their quarrel be coming to an end
a noise last night; 1 couldn't get to sleep.
A. wouldn't make B. hadn't made c. didn ’t make D. have n't made
28. Sally would prefer to pursue her stud ies__________ to look for a job.
A. rather than start B. to starting
c. rather than starting D. than to start
29. This car i s_________ of the two models in the showroom.
A. the more modern B. the most modern
c. more modem
30. ‘Well, I first came here last June,
‘So by the end of May you
with us for almost a year.’
A. will have been working
c. will have working
D. one of the most modem
B. are going to have worked
D. will work
IV. PREPOSITIONS AND PHRASAL VERBS (5 PTS)
31. She pl ay ed ___________ the fact that I’d enjoyed studying Shakespeare atschool and
suggested that I audition for a part.
A. off B. over c. by D. up
32. I was so tired that I ju st__________ in the armchair.
A. flaked out B. broke up c. dropped out Đ. fell over
33. We hadn’t _________ such heavy traffic; and we were delayed for hours.
A. gone in for B. set about ■ c. worked out D. bargained for
34. Two of the students in our class are identical twins and most of the teachers
can’t __________ .
A. see between them B. tell them apart
c. see them through D. find them between
35. I’ve never met my boss's wife bu t,____________ , she’s a very nice woman.
A. by all accounts B. under discussion
c, out o f kindness D. on bail
436
36. During the lead-up to Christmas, many teachers have so much work that they
get co mp letely _________ .
A. covered up B. washed off c. snowed under D. rubbed out
37. I'm afraid that the facts don't_________ your theory.
A. pull off B. check out c. show o ff' D. bear out
38. How did you come '_ _ _ _ _ that wonderful painting over there?
A. about B. in for c. by D. out with
39. I w ould n't__________ too much into her comments, I’m sure they were
meant as a joke.
A. draw B. see c. take D. read
40. He refused to ________ on why he took such an unexpected decision.
A. elaborate B. amplify c. account D. clarify
V. RE AD ING CO MP RE HE NS IO N (10 PTS)
Rea d the t ext s below and cho ose the best a nsw er to each question.
PASAGE 1:
How a man uses money - makes it, saves it,’and spends it - is perhaps one of
the best tests of practical wisdom. Although money ought by no means to be
regarded as a chief end of mail's life, neither is it a trifling matter, to be held in
philosophic contempt, representing as it does to so large an extent, the means of
physical comfort and social well-being. Indeed, some of the finest qualities of
human nature are intimately related to the right use of money; such as
generosity, honesty, justice, and self - sacrifice; as well as the practical virtues of
economy and pro videnc e. On the other hand, there are their counterparts of
avarice, fraud, injustice, and selfishness, as displayed by the inordinate lovers of
gain; and abuse the means entrusted to them. "So that", as it wisely observed by
Henry Taylor in his thoughtful "Notes from Life", "a right measure and manner
in getting, saving, spending, giving, taking, lending, borrowing, and bequeathing,
would almost argue a perfect man".
Comfort in worldly circumstances is a condition which every man is justified
in striving to attain by all worthy means. It secures that phy sical sat isfaction,
wh ich is neces sar y for the cultur e of the better part of his nat ure ; and
, enables him to provide for those of his own household. Nor ought the duty to be
any the less indifferent to US, that the respect which our fellow men entertain for
us in no slight degree depends upon the manner in which we exercise the
opportunities which present themselves for our honorable advancement in
life. The very e ffo rt req uired to be mad e to succeed in life with this objec t, is
of it self an edu cation; stimulating a man's sense o f self-respect, bringing out his
practical qualities, and disciplining him in the exercise of patience, perseverance,
and such like virtues. The pro vident and careful man must necessarily be a
thoughtful man, for he lives mot merely for the present, but with provident
forecast makes arrangements for the future. He must also be a tem perate man,
437
and exercise the virtues of self-denial, than which nothing is so much calculated
to give strength to the character. John Sterling says truly, that "the worst
education which teaches self denial is better than the best which teaches
everything else, and not that." The Romans rightly employed the same word
(vzr/M.s') to designate courage, which is in a physical sense what the other is in
moral; the highest virtue of all being victory over ourselves.
41. What is the main idea of his passage?
A. Wealthy people and poor people can both be virtuous.
B. Money is insignificant.
c. Money is the most important thing in the world.
D. The way a person handles money indicates his or her general character.
42. The author's purpose in writing this essay is________.
A. to teach people how to earn a great deal of money
B. to warn readers about the dangers o f greed
c. to describe the life of a wealthy person
D. to convince the reader that proper money management is a sign of good
character
43. Which is the best synonym for the word “pr ov iden ce”?
A. prudence B. fate c . sustenance D. doom
44. What would be the author's response to those who say that poverty is noble?
A. The author would agree with this statement.
B. In order to cultivate other virtues a person must have money.
c. Once a person gets.rich, they can start worrying about self-discipline.
D. The Romans believed that poor people are evil.
45. Which word best describes the author's attitude to Henry Taylor?
A. admiring B. condescending c . skeptical D. disgusted
46. What does the author imply by saying that money provides "physical satisfaction,
which is necessary fo r the cultivation o f the better par t o f his nature"?
A. People are friendlier after they have had a hot bath
B. In order to improve oneself in more lofty ways, one must attain the basic
necessities.
c. The most important thing in life is physical pleasure
D. Money can only provide physical pleasure
47. What does the author mean by the comment "The v ery effort requ ire d to be
ma de to su cc eed in life with this o bject, is o f itse lf an education"?
A. In order to earn money a person needs to go to college.
B. Money makes people seem smarter than they are.
c. That learning to manage money effectively entails learning a number of
other valuable skills.
D. Only intelligent people can earn money.
438
48. Why must the "provident and careful man" be a thoughtful man?
A. Because he has earned a great deal o f money.
B. Because he is familiar with the works of Henry Taylori
c. Because he gives most o f his money to charity. ’.
D. Because he must always.be planning for the future.
49. The author brings up the Roman word for courage to illustrate , .
A. his knowledge of the classics
B. that people throughout history have valued money •
c. that self-discipline is less important than physical bravery
D. that self-discipline is similar to physical bravery
50. What is the definition of the word temperate as it is used in this essay?
A. moderate
B. irritated c. self-denying D. warm
PASSAGE 2:
MUSICAL TALENT
Among all the abilities with which an individual may be endowed, musical
talent appears earliest in life. Very young children can exhibit musical precocity
for different reasons. Some develop exceptional skill as a result of a well -
designed instructional regime, such as the Suzuki method for the violin. Some
have a good fortune to be bom into musical family in a household filled with
music. In a number of interesting cases, musical talent is part of an otherwise
disabling condition such as autism or mental retardation. A musically gifted child
has an inborn talent, however, the extent to which the talent is expressed
publicly will depend upon the environment in which the child lives.
Musically gifted children master at an early age the principal elements of
music, including pitch and rhythm. Pitch - or - melody - is more central cultures,
for example, in Eastern societies that make use of tiny quarter - tone interval.
Rhythm, sounds produced at certain auditory frequencies and grouped according
to a prescribed system, is emphasized in sub - Saharan Africa, where the
rhythmic ratios can be very complex
All children have some aptitude for making music. During infancy, normal
children sing as well as babble, and they can produce individual sounds and
sounds pattern. Infants as young as two months can match their mother's songs
in pitch, loudness, and melodic shape, and infants at four months can match
rhythmic structure as well. Infants are especially predisposed to acquire these
core aspects of music, and they can also engage in sound play that clearly
exhibits creativity.
Individual differences begin to merge in young children as they learn to sing.
Some children can match segments of a song by the age of two or three. Many
others can only approximate pitch at this age and may still have difficulty in
producing accurate melodies by the age of five or six. However, by the time they
reach school age, most children in any culture have a schema of what a song
439
should be like and can produce a reasonably accurate imitation of the songs
commonly heard in their environment.
The early appearance o f superior musical ability in some children providence
that musical talent may be separate and unique form of intelligence. There are
numerous tales of young artist who have a remarkable "ear" or extraordinary
memory for music and a natural understanding of musical structure. In many of
these cases, the child is average in every other-way but displays an exceptional
ability in music. Even the most gifted child, however, takes about ten year to
achieve the levels of performance or composition that would constitute mastery
of the musical sphere.
Every generation in music history has it famous prodigies - individuals with
exceptional musical powers that emerge at a young age. In the eighteenth
century, Wolfgang Amadeus Mozart began composing and performing at the age
of six. As a child, Mozart could play piano like an adult. He had perfect pitch,
and at the age of nine he was also a master of the art of modulation - transition
from one key to another - which became one o f the hallmarks of his style. By the
age of eleven, he had composed three symphonies and 30 other major works.
Mozart's well - developed talent was preserved into adulthood.
Unusual musical ability is a regular characteristic of certain anomalies such as
autism. In one case, an autistic girl was able to "Happy Birthday" in the style of
various composers, including Mozart, Beethoven, Verdi, and Schubert. When the
girl was three, her mother called her by playing incomplete melodies, which the
child would complete with the appropriate tone in the proper octave. For the
autistic child, music may be the primary mode of communication, and the child
may cling to music because it represent as a haven as a world that is largely
confusing and frightening.
51. Which sentence below best expresses the essential information, in the
highlighted sentence in paragraph 1?
A. Children may be born with superior musical ability, but their environment
will determine how this ability is developed.
B. Every child is naturally gifted, and it is responsibility o f the public schools
to recognize and develop these talents.
c. Children with exceptional musical talent will look for the best way to
express themselves through music - making.
D. Some musically talented children live in an environment surrounded by
music, while others have little exposure to music.
52. The author makes the point that musical elements such as pitch and rhythm
A. distinguish music from other art forms
B. vary in emphasis in different cultures
c. make music difficult to learn
D. express different human emotions
440
53. The word "predispo sed" in paragraph 3 is closest in meaning to -
A. inclined B. gifted c. pushed D. amused
54. According the passage, when does musical talent usually begin to appear?
A. When infants start to babble and produce sound patterns
B. Between the ages of two and four months
c. When children learn to sing.at two or three years old
D. Between ten years old and adolescence -
55. According the passage, which of the following suggests that musical talent in
the separate form of intelligence?
A. Exceptional musical ability in an otherwise average child .
B. Recognition of the emotional power o f music
c. The ability of all babies to acquire core elements o f music
D. Differences between learning music and learning language
56. Why does the author discuss Mozart in paragraph 6?
A. To compare past and present views of musical talent
B. To give an example of a well - known musical prodigy
c. To list musical accomplishments o f the eighteenth century
D. To describe the development of individual musical skill
57. In music, the change from one key to another is known a s________ .
A. Rhythm B. Prodigy c. perfect pitch D. modulation
58. All of the following are given as example of exceptional musical talent
EXCEPT
A. a remarkable ’’ear" or perfect memory for music
B. ability to compose major works at a young age
c. appreciation for a wide variety o f musical styles
D. playing a single song in the style o f various composers
59. The word "haven" in paragraph 7 is closest meaning t o _________ .
A. Beautiful art B. Safe place c. personal goal D. simple problem
60. Which of the following can be inferred from the passage about exceptional
musical ability?
A. It occurs more frequently in some cultures than in others.
B. It is evidence of a superior lever of intelligence in other areas,
c. It has been documented and studied but is little understood.
D. It is the result o f natural talent and a supportive environment.
VI. CL OZ E TESTS: (10 PTS)
Rea d the t ext s below and dec ide which an sw er best Fits eac h spa ce.
PASSAGE 1:
There can be no doubt at all that the internet has made a huge difference to
our lives. Parents are worried that children spend too much time playing on the
internet, hardly ( 61).________doing anything else in their spare time. Naturally,
parents are (6 2)_______ to find out why the internet is so attractive, and they
441
want to know if it can be harmful to their children. Should parents worry if their
children are spending that much time ( 63 )_________ their com puter?
Obviously, if children are bent over their computers for hours, (6 4)______ _
in some game, instead of doing their homework, then somethin g is wrong.
Parents and children could decide how much use the child should (6 5)________
of the internet, and the child should give his or her (6 6)________ that it won’t
interfere with the homework. If the child is not (6 7)______ to this arrangement,
the parents can take more drastic steps. Dealing with a child's use of the Internet
is not much different from negotiating any other sort of bargain about behaviour.
Any parent who is seriously alarmed about a child's beha vior should make an
appointment to discuss the matter with a teacher. Spending time in front of the
screen does not (6 8)_______ affect a child's performance at school. Even if a
child is ( 69)________ crazy about using the internet, he or she is probably ju st
(7 0) __________ through a phase, and in a few months there will be something
else to worry about!
61. A. always B. rarely c. never D. ever
62. A. worried B. concerned c. curious D. hopeful
63. A. staring at B. glancing at c. looking D. watching
64. A. supposed B. occupied c. interested D. absorbed
65. A. do B. have c. make D. create
66. A. word B. promise c. vow D. claim
67. A. holding B. sticking c. following D. accepting
68. A. possibly B. necessarily c. probably D. consequently
69. A. absolutely B. more c. quite D. a lot
70. A. going B. passing c. travelling D. walking
PASSAGE 2:
The PBT is a pencil and (7 1)______ test that is offered for two purposes.
One purpose of the PBT is for (7 2)______ and process evaluation. Colleges or
other institutions use the PBT to test-their students. The scores are not valid
outside of the place where they are (73) , but the college or institution
accepts the PBT that they administered as an official score. This PBT is also
(7 4) ________ an Institution TOEFL. T he (7 5) _________purpose of the PBT is
(76)_____ the official Computer-Based TOEFL in areas where computer-based
testing is not possible. The scores are usually valid outside of the place where
they are administered. This PBT is also called a Supplement TOEFL.
The Paper-Based TOEFL has three parts: Listening Comprehension, Structure
and Written Expression, and Reading. (77)_____ , the TEST of Written English
(TWE) is an essay that is required to provide a (7 8)_____ score. The PBT is a
(7 9)_____ test, which means that everyone who takes the TOEFL during the
same administration will see and answer the same questions. The (80)
score is based on a scale o f 310-677.
442
71. A. note-paper B. letter c. draft D. paper
72. A. place B. placing c . placement D. placeability
73. A. administered B. managed c . controlled D. protected
74. A. called B. seemed c . considered D. appeared
75. A. some B. other c. another D. main
76. A. supplement B. to supplement c. supplementing D. supplemented
B. On the other hand
77. A. In other words D. In addition
c. Besides
78. A. reading B. speaking c. listening D. writing
79. A. style B. form c. standard D. linear
80. A. total B. final c. whole D. all
B. W R IT TEN TES T (70 PTS)
I. OPEN CLOZE TESTS (20 PTS)
Read the texts below and think of the word which best fits each space. Use
only ONE WORD for each space.
CLOZE TEST 1:
Are you looking forward to another busy week? You should be according to
some experts. They argue that the stress encountered in our daily lives is not
only good for US, but (1) ............. . to survival. They say that the response to
stress, which creates a chemical called adrenalin, helps the mind and body to act
quickly in emergencies. Animals and human beings use it to meet the hostile
(2 )...... .-............which exist on the planet.
Whilst nobody denies the pressures o f everyday life, what is surprising is that
we are yet to (3 ). ........... . successful ways of dealing with them. Even the
experts consider the current strategies to be inadequate and often dangerous..
They believe that instead of trying to manage oụr (4)............. . to stress with
drugs or relaxation techniques, we must exploit it. Apparently, research shows
that people who create conditions o f stress for (5)...............by doing exciting and.
risky sports or looking for challenges, cope much better with life’s problems.
Activities of this type have been shown to create a lot o f emotion; people may
actually cry or feel extremely (6 )............... But there is a point at which they
realise they have succeeded and know that it was a positive experience. This is
because we learn through (7)................ and difficulty, That’s how we get our
wisdom. Few of US, (8 )............. . understand this fact. For example, many
people believe they (9)............... from stress at work, and take time off as a
result. Yet it has been found in some companies that by far the healthiest people
are those with the most responsibility. So next time you’re in a (10)........... .
situation, ju st remember that it will be a positive learning experience and could
also benefit your health!
CLOZE TEST 2:
The issue o f fate or free will is a very controversial one and has been debated
(1 1) .............. the beginning of the fifth century. The question was first addressed
by Saint Augustine. Is man’s existence (12)........... ...byT ate or by free will?
443
Belief in free will is based on the conviction that humans have a hand in
shaping their own lives and, therefore, are ultimately responsible for their own
actions. This theory, called existentialism, was derived from the ideas of a
Danish philosopher and later made popular by a French writer. The theory
(13)....... man as a unique and isolated individual in a violent and (14)...............
world, able to choose his own destiny.
On the other side of the coin, the theory of predestination is based on the
belief that everything which happens has been (15)............... by God and that
man is not able to change it. So, the fork in the road of life seemingly offering
man a choice between two very different paths, may not exist in reality. If this is
true, can man be held responsible for his actions? If choices are beyond our
(16) ....... , are we wasting endless hours of our time trying to come to a decision
that has already been made for US? Can the man who murders his neighbour be
(17) ............... for doing so simply because he had no choice?
In trying to answer these kinds of questions, many philosophies have
developed compromises between the two (18)........One such theory developed
in the seventeenth century (19)............... that human actions are formed
according to past experiences but that through the use of imagination and reason
man can turn these past experiences into foresight, therefore allowing him to
shape his future.
Whatever theories are formed, the controversial (20)............... remains for
public debate even today. But, perhaps, simply by choosing to believe in the
philosophy of fate or that of free will, man is in fact creating his own life.
Existentialism or predestination - which do you choose to believe in?
II. WORD FORMS: (20 PTS)
PART 1: Give the correct forms of the words in brackets.
1. He left us with the ......................... impression that we had been speaking to a
future leader. (ERADICATE)
2. At college 1 became a social..... '......... ...... because I didn’t like going out in
the evenings. (FIT)
3. I'm looking for a travel insurance policy that will cover me for an y.................
' (EVENT)
4. He has abandoned all ambition to become president and is no w ... ............
wholly by altruism. (ACTUAL)
5. O f course there were some................... problems along the way but eventually
we got the project finished. (LOOK)
6. Every attempt is being made to ensure that these works of art are preserved
f o r .. ........... .......... (POST)
7. He’s been nominated for............thanks to his diligence and professionalism.
(PREFER)
444
8. C an’t believe her car can be so.......................... refuelling once every two days,
how costly! (PETROL)
9. The vigorous exercises are designed to be.................. T......Participants will
feel energetic and lifelike after taking them. (TENSE)
10. Many MBA courses are highly theoretical, esoteric and impractical, instead
of bei ng .., ........ '....... .. .. .as they should. (BUSINESS)
PART 2: Complete the passage with appropriate forms from the words given
in the box.
appetite believe create deny .
marvel mix origin resist
The English are famous for talking about the weather but listen to any Italian
conversation and it will normally be about cookery.
One of the most famous Italian sauces is pesto. Made from a (11)............... of
garlic, basil, pine nuts, cheese and olive oil, it is (12)............... delicious. We all
buy it from the supermarket, but you have to try the (13)................... from its
home, Genoa, to really experience it. The word pesto means ‘to pound ’ and it is
the action of the pestle and m ortar which is all important for this (14 )...............
sauce. Along with the best basil and local olive oil, the pesto makers of Genoa
use their wrist action to make an (15 )...........’...... pale, luscious green sauce.
Genoans really take their pesto seriously, and the (16)................... of the Knights
of the Confraternity of authentic pesto makers shows this. If you want to sample
their cuisine, go to Genoa as I did. I visited a (17)...................village called
Vernazza. Sitting in a.trattoria in the harbour, the (18 );................ smell of fish
frying wafted over US as we tried, amongst other things, a kind of pesto called
salsa di noce. It was a true culinary delight, made wjth the (19)................... of
past Italians who couldn’t afford meat. It may sound (2 0) ... ............... to those of
you who want meat in a meal, but you would be mad to forgo it.
III. ERROR IDENTIFICATION (10 PTS)
Identify 10 errors in the following passage and correct them.
With Europe facing the more serious flu epidemic in years, the World Health
Organisation is warning the public, specially the sick and elderly, to do an
innoculation now.
Health experts warn that this winter’s breakthrough could be as severe as the
1989 epidemic, which killed 25,000 people in Britain only.
WHO has stressed the importance of vaccination, and several countries are
already taking precautions to ensure that those at high-risk categories, such as
diabetics and those with heart or lung complaints, are vaccinated.
Nearly all previous strains o f ‘flu are known to originate in China and the Far
East, and whereas in the past infection spreading slow across the world, in these
days o f mass air travel rapid global infection is a great threat.
445
Sweden has reported cases of para-influenza, effecting mainly children, and
Danish doctors had recorded 1,500 cases o f'flu in the past week.
4.
7.
2.
5.
8.
3. _
6. _
9. _
10.
IV. SENTENCE TRANSFORMA TION (20pts)
Rewrite the sentences so that they mean almost the same as the ones before them.
1. No real sportsman wants to be suspected o f crowing over his opponents’
misfortunes.
—> E xultin g___ ______________________________ __________________ _
.2. O f all the myths of the Welfare State, stories of the work-shy have been the
least well-founded on evidence.
-> There is less _________________ _________________________________
3. After strong pressure from Labour councillors, the Council agreed to
reconsider the road plan.
—> The Council w as ____________ ________________ • ______________
4. Kip Keino wasn’t just the first African international mile winner, he was a
national symbol.
-» N ot_____________________________________________________ '
5. Sam was really anxious, waiting to see if he had got a place in the cricket
team. (TENTERHOOKS)
—> Sam w as _____________________________________________________
6. The heir to the family fortune was eventually found to be an impostor.
(EXPOSED)
—> The he -____
ir_____________ '_______________________
__ ___
7. He claimed my
point was not
relevant to the
proceedings.
(DISMISSED)
___________ _
—> H e ________________________________
____
8. So completely incompetent was that secretary that we lost heaps o f invoices.
(RANK)
-> Su ch_________________________________________________ _______
9. A lot of the c ountry’s two million unemployed will probably never work
again. (MANY)
—> There are two m illion___________________________'___________ ___
10. It was difficult to understand her colleagues overt hostility towards her
proposal. (OPENLY)
-» That her colleag ues ____________________________________________
446
TRƯỜNG ĨH P Ĩ PHAN CHÂU TRKIH - ĐÀ NẮNG
A. MULTIPLE CHOICE QUESTIONS (40pts)
ĩ. W OR D CH OI CE (5 pts)
Sele ct the option that best fits each blank. Write your a nswers (A, B, c, or D)
in the b ox for answ ers .
1. His strange b ehavior aroused the ____ of police.
A. do ubt B. suspicion c. disb elief ' D. notice
2. M edia reports on the outcome of military intervention of ten____ the true facts.
A. div ert B. d etract c. dep ose D. dis tort
3. The day th eir first ch ild w as born, the new parents w ere ____ with joy.
A. overthro wn B. over pow ered c. ove rtaken D. o ver com e
4. Co mp anies are jo ining forc es with gover nm ent s in Africa t o _____ regional
camp aig ns again st malaria.
A. fab ricate B. o riginate c. mo unt D. produce
5. The authorities have__ their original decision to allow development o f this rural area.
A. dis pensed B. d etache d c. refr ained D. revoke d
6. T he integr ity o f a m arine ecosystem cou ld be____________ due to the imp act
of coa sta l reclam atio n.
A. comp rom ise d B. conc ede d c. con cili ated D. con fronted
7. T he ren ewed inte rest in E lizabethan times is ev ident in t h e ____ of new
Hollyw ood film s set du ring that period.
A. spate B. h ypocr isy c. transienc e D. dem ise
8. The car w a s____ speed.
A. gathe rin g B. col lecting c. con sum ing D. f irin g
9. Lac k o f rain early in the season means th at the fi el d____ a poor crop.
A. sur ren dered B. yield ed c . gen erated D. suffere d
1 0 .1 hop e you won ’t ta k e____ if I tell the truth.
A. annoyance B. r ese ntm ent c. offenc e D. irritation
II. ST RU CT UR ES A ND GR AM MA R: (5pts)
Select the optio n that best fits each blank. Write y our answers (A, B, c, or D)
in the box for answ ers.
1. ____ know led ge a bout gen etic diseases has increased is w elc om e news.
A. Th at sci ent ific B. It w as scientific c. Tho ugh scientific D. Scien ce
2. I’ll be kind to h e r____ she decides to leav e me.
A. in case B. w her eas c. so as not D. lest
2 .____ often serve as places o f pub lic ent ertain me nt and f estivals, the y can a lso
be pla ces where peo ple ca n find p eace and solitude .
A. Even though city parks B. C ity parks
c. City parks th at D. T here are city park s w hich
447
4. “I’m totally broke. Have you got any money on you?” - “____ at all.”
A. Not B. None c. Nothing D. No
5. Professor Lockwood recommended that M ichael____ in chemistry.
A. not major B. not to major, c. wouldn’t major D. isn ’t majoring
6. ____, one tin will last for ạt least six weeks.
A. Used economical
c. Used economically
7. A new generation of performers,
B. Using economical
D. Using economically
_____those who by now had become
a household name, honed their skills before following the same path onto
television.
B. along with talented as
D. having been more talented than
A. no less talented than
c. together with talented as
8. We all wished to be treate d____ . ‘
A. as equal B. as equally c. as equals D. as equal as
9. _________ the US superiority at that time, it was probable that any threatened
US response would have deterred the Soviet Union.
A. If B. Given c. Although D. Since
10. S he ____fainted when she heard that her son had died.
A. rather than B. nothing but c. all but D. near
III. PHRASAL VERBS AND PREPOSITIONS (5pts)
Select the option that best fits each blank. Write your answers (A, B, c , or D)
in the box for answers.
1. Half of Martha 's class h as ____influenza.
A. got down to B. come up with c. come down with D. gone down with
2. The new record will appeal to older listeners, but I d on't think it will catch
____among younger pop fans.
A. up B. in c. for D. on
3. Please remember to keep a copy for yourself as we cannot return copies,____
of whether or not they are published.
A. in view B. irrespective c. because
4. Her success wen t____ her expectation.
A. beyond B. over c. beneath
5. Our holiday plans fe ll____ due to lack o f money.
A. for B. by c. over D. through
6. Do you mind if I ____with my work while you are getting tea ready?
A. get through B. turn to c. carry-out D. carry on
7. At first the children enjoyed the game but quite soon____the novelty.
A. wore off B. went off c. died out D. died down
8. When Helen and Andrew saw how ridiculous they look ed____ laughing.
A, came round B. broke in c. burst out D. flowed over
9. She is always _ __ her friends behind their backs. She won’t have any friends left.
A. falling out B. breaking off c. putting off D. running down
D. on account
D. above
448
10. Rosa ce rta inly____ her moth er in her devil-may-care approach to life.
A. complies with B. calms down c. takes after D. cares for
IV. COL LOC ATIONS AND IDIOMS (5pts)
Complete each of the following sentences by choosing the correct word (A-D).
1 . Not getting the promotion felt like a real kick in th e ................. as I'd put in so
much hard work for the company.
A. head B. teeth c. leg D. back
2. Could somebody please help me as I am at the end of m y ..... ......... over next
door's barking dog.
A. tethe r B. leash c. lead D. rope
3 .1 hope readers have enjoyed this article and that it has offered so m e......... for
thought.
A. bread B. dough c. food D. grains
4 .1 w en t........on a limb for you, and this is how you thank me!
A. o ff B. out c. away D. back
5. He ràn up three flights of stairs and wasn't the slightest bit out o f breath even
though he's n o ....... chicken.
A. spring B. young c. fresh D. baby
6. The two young fighters fought toe to toe and both s ho wed ...... . determination.
A. grim B. strong c. harsh D. mean
7.1 would like to welcome you all on this auspicious....... , the 25th anniversary
of the founding o f our company.
A. occasion B. moment c. meeting ‘ D. gathering
8. As a c on servati ve........I think we need to put aside £4,000 for the wedding.
A. forecast B. approximation c. guess D. estimate
9. Both parents were unemployed and the family had a .........lifestyle.
A. cautious B. frugal c. careful D. tight
10. The car screeched to a ....... at the junction and two men jumped out and ran
down the street.
A. halt B. stop c. end D. standstill
V. READING COMPREH ENSION
PA RT 1. Read the following passage and choose the correct answer (A, B, c , or
D) to each of the questions. Write your answers in the numbered box. (5pts)
Even with his diverse experience as an elected official at the state level,
Andrew Johnson was the first president of the United States ever to be
impeached, primarily because o f his violent temper and unyielding stubbornness.
His career started in 1828 with his collection to the city council of Greenville,
Tennessee, and after two years as an alderman, he took office as mayor. His
advancem ents followed in rapid succession when he was elected to the
Tennessee state senate, then as the state governor, and later to the US House of
Representatives for five consecutive terms.
449
In 1864, Johnson ran for the office of vice-president on the Lincoln-Johnson
ticket and was inaugurated in 1865. After Lincoln 's assassination six weeks into
his term, Johnson found himself president a t a time when southern leaders were
concerned about their forced alliance with the northern states and feared
retaliation for their support of the secession. Instead, however, with the
diplomatic skill he had learned from Lincoln, Johnson offered full pardon to
almost all Confederates on the condition that they take an oath of allegiance. He
further reorganized the former Confederate states and set up legislative elections.
Congressional opposition to his peace-m aking policies resulted in gridlock
between the House and Johnson, and the stalemate grew into an open conflict on
the issue of the emancipation of slaves. While Johnson held the view that newly freed
slaves lacked understanding and knowledge of civil liberties to vote intelligently,
Congress overrode Johnson’s veto of the Civil Rights Bill, which awarded them
citizenship and ratified the Fourteenth Amendment. In the years that followed,
Congress passed bills depriving the president of the power to pardon political
criminals, stripping away his status of commander-in-chief, and taking away
Johnson’s right to dismiss civil and executive officers from their duties. Johnson
vetoed each bill, and each veto was overridden. When Johnson dismissed the
secretary o f war, Edwin Stanton, Stanton refused to step down and was supported
by the House of Representatives, which voted to impeach Johnson. At the trial, the
Senate came one vote short of the two-thirds majority necessary to remove him
from office. After Johnson’s term expired, he returned to his home state, but in
1875 he was elected senator and went back to Washington to take his seat.
1. What does the passage mainly discuss?
A. Andrew Johnson’s personal characteristics
B. Andrew Joh nson's career as a politician
c. Congressional decisions in the late 1800s
D. Congressional decisions and procedures in the late 1800s
2. The phrasè “took o ffic e” is closest in meaning t o ____.
A. moved into an office B became an official
c. began a government job D. rearranged the office
3. What can be inferred from the first paragraph about Andrew Jo hnso n’s work
in Tennessee?
A. His personality precluded him from important positions.
B. His work became known to the governor,
c. He was elected to several important posts.
D. He was represented to the posts five times.
4. According to the passage, what led to Johnson’s downfall?
Ă. The state o f the nation’s economy B. His liberal position on slavery
c. His personal characteristics
D. His waffling and hesitation
450
5. The auth or o f the passage implies that when Johnson becam e president he .
A. was a dedicated supporter of civil rights
B. was a soft-spoken and careful diplomat
c. had an extensive background in politics
D. had already experienced political turmoil
6. According to the passage, at the beginning of J ohn son's term as president
southerners w er e____ .
A. expected to secede from the union B. apprehensive about their future
c. singled out as scapegoats D. afraid of his violent temper
7. According to the passage, Congress’s disapproval of Andrew Johnson’s
policies was • .
A. short-lived and groundless B. detrimental to his presidency
c. directed at his civic duties D. stopped as soon as it emerged
8. The word “pardo n” is closest in meaning to ____
A. parade B. patronize c. exonerate D. extricate
9. The autho r o f the passage implies that the Stanton affair proved the president’s
A. lack o f stamina B. lack of electoral vote
c. loss o f willpower D. loss o f authority
10. According to the passage, the attempt to impeach Andrew Jo hnson____ .
A. succeeded as expected by the House
B. failed by a minimal margin
c. put an end to his political career
D overwhelmed his supporters in Tennessee
PART 2. Read the following passage and choose th e c orrect answ er (A, B, c, or
D) to each o f the questions. W rite your a nswers in the numb ered box. (5pts)
Very few people, groups, or governments oppose globalization in its entirety.
Instead, critics of globalisation believe aspects of the way globalisation operates
should be changed. The debate over globalisation is about what the best rules are
for governing the global economy so that its advantages can grow while its
problems can be solved.
On one side of this debate are those who stress the benefits of removing
barriers to international trade and investment, allowing capital to be allo cated
more efficiently and giving consumers greater freedom of choice.
With free-market globalisation, investment funds can move unimpeded from
the rich countries to the developing countries. Consumers can benefit from
cheaper products because reduced taxes make goods produced at low cost from
faraway places cheaper to buy. Producers of goods gain by selling to a wider
market. More competition kee ns sellers on the ir toes and allows ideas and new
technology to spread and benefit others.
On the other side of the debate are critics who see neo-liberal policies as
producing greater poverty, inequality, social conflict, cultural destruction, and
environmental damage. They say that the most developed nations - the United
451
States, Germany, and Japan - succeeded not because o f free trade but because of
protectionism and subsidies. They argue that the more recently successful
econom ies of South Korea, Taiwan, and China all had strong state-led
development strategies that did not follow neo-liberalism. These critics think that
government encouragement of “infant industries'” - that is, industries that are just
beginning to develop - enables a country to become internationally competitive.
Furthermore, those who criticise the Washington Consensus suggest that the
inflow and outflow of money from speculative investors must be limited to
prevent bubbles. These bubbles are characterised by the. rapid inflow of foreign
funds that bid up domestic stock markets and property values. When the
econom y cannot sustain such expectations, the bubbles burst as investors panic
and pull their money out o f the country.
Protests by what is called the anti-globalisation movement are seldom directed
against globalisation itself but rather against abuses that harm the rights o f workers
and the environment. The question raised by nongovernmental organisations and
protesters at WTO and IMF gatherings is whether globalisation will result in a rise
of living standards or a race to the bottom as competition takes the form of
lowering living standards and under min ing environmental regulations.
One of the key problems of the 21st century will be determ ining to what
extent markets should be regulated to promote fair competition, honest dealing,
and fair distribution o f public goods on a global scale.
1. It is stated in the passage th at ____
A. critics of globalisation say that the successful economies are all in Asia scale.
B. the protests of globalisation are directed against globalisation itself
c. supporters of globalisation stress the benefits of removing trade barriers
D. the United States, Germany, and Japan succeeded in helping infant
2. Supporters o f free-market globalisation point out th at ____.
A. investment will be allocated only to rich countries
B. taxes that are paid on goods will be increased
c. there will be less competition among producers
D. consumers can benefit from cheaper products
3. The word “allocated ” in the passage mostly m eans____.
A. distributed B. solved c. removed D. offered
4. The phrase “keens se ller s on the ir toes” in the passage mostly means___
A. prevents sellers from selling new products
B. forces sellers to go bare-footed
c. makes sellers responsive to any changes
D. allows sellers to stand on their own feet
5. According to critics of globalisation, several developed countries have become
rich because o f____ .
A. their help to developing countries B. their neo-liberal policies
c. their protectionism and subsidies D. their prevention o f bubbles
452
6. The word “u nd ermining” in the passage mostly me ans____ .
A. obeying B. making less effective
c. observing D. making more effective
7. Infant industries mentioned in the passage a re _____ .
A. young companies B. development strategies
c. young industries D. successful economies
8. Which of the following is NOT mentioned in the passage?
A. Critics believe the way globalisation operates should be changed.
B. The anti-globalisation movement was set up to end globalisation,
c. Hardly anyone disapproves of globalisation in its entirety.
D. Some Asian countries had strong state-led economic strategies.
9. The debate over globalisation is about how ____.
A. to spread ideas and strategies for globalisation
B. to govern the global economy for the benefit of the community
c. to use neo-liberal policies for the benefit of the rich countries
D. to terminate globalisation in its entirety
10. The author seems to b e ____ lowering living standards and undermining
environmental regulations.
A. supportive of
B. pessimistic about c. indifferent to D. opposed to
VI. CLOZE TEST
PART 1. Select the option which best fits each space in the following passage.
Write your answers (A, B, c or D) in the box provided. (5pts)
Greenhouse gases arise from a wide range of sources and their increasing
concentration is largely related to the compound effects of increased population,
improved living standards and changes in lifestyle. From a current base of 5
billion, the United Nations predicts that the global population may stabilize in
the twenty-first century between 8 and 14 billion, with more than 90 percent of
the (1) _ ___ increase place in the world’s developing nations. The (2 )____
activities to support that growth, particularly to produce the required energy and
food; will cause further increases in greenhouse gas emissions. The challenge,
therefore, is to attain a (3 )___ỵ balance between population, economic growth
and the environment.
The major greenhouse gas emissions from human our activities are carbon
dioxide (CO2), methane and nitrous oxide. Chlorofluorocarbons (CFCs) are the
only major ( 4 ) __ _ to the greenhouse effect that does not occur naturally,
coming from such sources as refrigeration, plastics and manufacture. Coal’s total
contribution io greenhouse gas emissions is thought to be about. 18 per cent, with
about h alf of this coming from (5) ____generation.
The worldwide coal industry allocates extensive resources to researching and
developing new technologies and ways capturing greenhouse gases. (6 )______
are likely to be improved dramatically, and hence co2 emissions reduced,
453
through combustion and gasification techniques which are now at (7 )____ and
demonstration stages.
Clean coal is another avenue for improving fuel conversion efficiency.
Investigations are u nder (8 )____________ into super-clean coal (35 percent ash)
and ultraclean coal (less than 1 per cent ash). Super-clean coal has the potential
to enhance the combustion efficiency of conventional stabilized fuel power
plants. Ultraclean coal will enable coal to be used and in (9)____ power systems
such as coal-fired gas turbines which, when (1 0)____ in combined cycle, have
the potential to achieve much greater efficiencies.
1. A intended B. projected c. wanted D. calculated
2. A remarkable B. major c. associated D. mutual
3. A reachable B. sustainable c. eligible D. visual
4. A substance B. contributor c. chemical D. emission
5. A electronic B. electric c. electrical D. electricity
6. A Influences B. Results c. Efficiencies D. Conseque
7. A pilot B. launch c. experiment D. beginning
8. A experiment B. development c. way D. progress
9. A electrified B. developed c. advanced D. controlled
10. A involved B. operated c. managed D. made
PART 2. Select the option which best fits each space in the following passage.
Write your answers (A, B, c or D) in the box provided. (5pts)
ỉn 1830, there were under 100 miles of public railway in Britain. Yet within
20 years, this figure had grown to more than 5000 miles. By the end of the
century, almost enough rail track to ( 1)____ the world covered this small island,
(2 )____ the nature of travel forever and contributing to the industrial revolution
that changed the ( 3) __ _ of history in many parts of the world.
Wherever railways were introduced, economic and social progress quickly
(4 )____ . In a single day, rail passengers could travel hundreds of miles, (5)____
previous journey times by huge margins and bringing rapid travel within the
(6) ____ of ordinary people. Previously, many people had never ventured
beyond the outskirts of their town and villages. The railway brought them greater
freedom and enlightenment.
In the 19th century, the railway in Britain (7 )____ something more than just
the business o f carrying goods passengers. Trains were associated with romance,
adventure and, frequently, (8) ____ luxury. But the railways did more than
revolutionized travel; they also (9)_____. a distinctive and permanent mark on
the British landscape. Whole towns and industrial centers (1 0\______up around
major rail junctions, monumental bridges and viaducts crossed rivers and valleys
and the railway stations themselves became desirable places to spend time
between journeys.
454
1. A. revolve B. enclose c. encircle D. orbit
2. A. altering B. amending c. adapting D. adjusting
3. A. route B. way c. line D. course
4. A. pursued B. followed c.succeeded D. chased
5. A. cancelling B. subtracting c. cutting D. abolishing
6. A. reach B. capacity c. facility D. hold
7. A. served B. functioned c. represented D. performed
8. A. considerable B. generous c. plentiful D. sizeable
9. A. laid B. set c. settled D left
10. A. jumped B. sto od c. burst D. sprang
B. WRITTEN TASKS (70pts)
I. OPEN CLOZE TEST (20pts)
PA RT 1. Fill each gap with ONE suitable word. Write your answers in the
box provided.
CUSTOMER RIGHTS
I once called over the wine waiter in an expensive restaurant to tell him that 1
thought the wine I had ordered was off. (1 )_______ the mere suggestion that
something might be wrong, he became most unpleasant (2 )_______ reluctantly
tasted it, however, he immediately apologized and brought another bottle. It
helped that 1 knew I was legally ( 3 )_______ _ the right. (4) ________ in a
restaurant or a bar, the food or drink must be fit for human consumption and o f a
quality that you are entitled to expect in an establishment of that category.
(5 )_______ the customer, you have considerable rights. The menu, for example,
is a vital legal docum ent and a restaurant can be fined up to $5000 ( 6)_______ it
fail to display one outside or immediately inside the door. Potential customers
have the right to know in advance what they are committing (7 )_______ to and
it is an offence (8 )_______ the Trade Descriptions Act for any establishment to
give a false description of its food. Everything must be (9 )_______ it claims to
be and in cases where it is not, you should complain. Fresh fruit salad must only
consist of fresh, (10) ______ tinned, fruit, Pâté maison must be made on
premises. The same principle applies to wine. If you are brought a vintage
different form that stated on the wine list, send it back.
PART 2. Fill each gap with ONE suitable word. Write your answers in the
box provided.
A MODERN-DAY PROBLEM
In the hustle and (1 )______ of today’s fast -and-(2 )______ world all of US,
without exception, have to contend with some level of stress. It goes without
(3 )______ that the source and amount of stress are relative to the individual.
Just as causes and quantities of stress are subject to personal factors, (4 )_______
is the way in which a person deals with. It is ( 5 )______ knowledge that some
people flourish if faced with a potentially stress-causing task or situation. On the
other hand, the majority of people are (6 )______ affected when confronted with
455
a serious dilemma. Abnormal levels o f stress can be a serious health hazard and
may prove detrimental to one's physical health. Stress is said to be the
(7) ______ in a high percentage of heart problems and stomach disorders. Even
certain types of cancer are, day malady which we all, to a greater or lesser
(8) ______, suffer from, has (9) _______ many to begin looking seriously
at ways of controlling stress. Owing to the inevitable fact that stress will always
play a part in our lives, it is o f ( 10 )______ importance that strategies of stress
management be found.
II. W OR D FO RM (20pts)
PART 1. Use the rig ht form of the words given in cap itals to comp lete the
sen ten ces b elow. Wr ite your a nsw ers in the box pro vided.
1. If this theory is correct, then it is 2_____ that we will be able to cure all
diseases in the foreseeable future. (VISION)
2. They were divorced because there was nothing she could do to make her
husband change. He was such a(n )______ flirt. (RIGHT)
3. It is concluded from the tale The Magic Cross-bow: The Story o f My Chau and
Trong Thuy that the rights of individuals should b e______ to those of society
as a whole. (SERVE)
4. According to the government figures, t he_______of jobs in the next century
will be in service-related fields, such as health and business. (PONDER)
5. Though many parties regarded his behavior as a( n) ______ , the public still had
much faith in his ability to become a leader. (ERR)
6. The decision to close the local hospital caused such a p ub lic _______ that the
authorities decided to re- examine their options and keep it open. (CRY)
7. She gave her son quite a f ew ______for playing truant. (TELL)
8. You should avoid those journalists. They are ju st______ only distressing and
prying into your private life. (CHIEF)
9. Those who made this dress are going to be ____ _ designers. They are
nothing but awesome! (TREND)
10. At first we thought the plan would be a flop, but it turned out to be a(n)
success. (DREAM)
PART 2: Fill in each blank with the cor rect form of one a ppropriate w ord in
the box. Use each word only ONCE and write your answer in the numbered box.-
close follow look mock prin t topic
delete inflate minute present
FAKE IT ON FACEBOOK
New research from the University of Helsinki has shown that even the
smallest of tweaks to your Facebook profile can convince ( l)th at you are more
confident, open- minded and attractive than you really are. Here are six steps to
set you on the right course.
456
1. Make a new profile photo
Dozens of studies have shown how initial meetings create stubborn (2 )____
that are difficult to scrub out. Profile pictures which included social cues (i.e.
you holding a guitar or playing tennis: something that demonstrates who you
are) were seen by others as being more socially and physically attractive.
2. Pick your friends carefully
“A person is known by the company they keep". People online are very
responsive to the types of comments that others make on your photo and your
profile in general. These should validate your online (3)_________ or readers
may smell a rat. It's better to get rid o f any silly or sarcastic comments.
3. Don’t seem shallow
A research from Michigan State University found five (4)_______ of almost
identical Facebook profiles, and then showed to participants, asking them
how attractive the profiles were. The only difference was the number of
friends displayed (102-902). Profiles with 302 friends were considered the
most socially attractive to others. A(n) (5 )____ number o f friends, though, is
jus t one way in which we might exaggerate who we really know and what
we’re really like. Even though you could be anyone online, most people
display their real selves - and that’s what they’ll expect from others.
4. Keep (some of) your inhibitions
While it's probably no bad thing to display your true personality and tell
people about who you are and what you’re interested in, be aware that the
internet is not as anonymous as it feels. First impressions are near (6 )_____ ,
so it’s worth remembering that those pictures of you at 3am on St Patrick's
Day might not be showing your best side.
5. Fill your profile
Those who completed their user profile, including things like location and
web address, as well as providing a longer description, generally attracted
more followers. Other users tend to respond positively to these little (7)___,
just like in an offline conversation when you share something personal with
another person, it brings you slightly closer together.
6. Inform, don’t “meform”
With social media the temptation is to go on about yourself. The mechanism
of loads of little updates seems perfect for relating the (8 )____ of everyday
life. The problem with being a ‘meformer’ is there’s only a limited audience
who are interested in hearing about you and your activities, and you wo n't .
stand out. For.m ore success on social networks, you’ve got to fill at least
some of your tweets or updates with some juicy information. So if you want
to increase (9)_____then you’ve got to find your informational niche and
demonstrate that you’ll feed them with fascinating.(10)______ stuff.
457
HI. ERROR IDENTIFICATIO N (1 Opts)
There are 10 mistakes in spelling or use of English in the text. Underline the
mistakes and correct them in the box below.
Skiing is one of the most popular sports in the world. According to recent
estimation, about one hundred millions of people ski regularly or occasionally.
Sliding across the snow on skis is also one of the most ancient methods of
transport known to the man. It has demonstrated that men were already traveling
across the snow by means of primitive skis before the invention of the wheel. In
the Asiatic region of Altai and in Scandinavia, for example, the remains of skis
have been found which dated back to 4,000 BC. Further evidence is supplied by
ancient cave paintings which depict people skiing, and a Norw ay saga which
tells the story of an invasion of its territory 8,000 years ago by a tribe of skiers
who came from the north.
Nowadays, skiing, apart from a sport, has become a big industry and a
notable feature of leisure culture. Ski resorts and all the activity that they
generate is the main source of wealth in'm any mountain regions, which were
previously remote and accessible. And far from its once elitist image, skiing is
now enjoyed by an increasingly broader spectra of society.
1. _ _____________ 2 ._______________ 3 .______________ _
4 ._______________ 5 ._______________ 6 ._______________
7 .___________ 8. _______________ 9 . ____________
10.______________
IV. SENTENCE TRANSFORM ATION (20pts)
PART 1. Finish each of the following sentences in such a way that it is as
similar as possible in meaning to the sentence printed before it.
-1. He seems to find the way Mimi behaves more a source of amusement than
embarrassment.
- Far from _________________ '______ __________________ amused by it.
2. It was not until 5 years has elapsed that the whole truth about the murder came out.
- Not fo r_______________________________________________________
3. The only reason why she got promotion is that she is very competent.
- Were i t _______________________________________________ -
4. There is grave concern about confirmed cases of cholera that originated in the
makeshift shelters.
- O f_____________________________________
PART 2. Complete the second sentence so that it has a similar meaning to
the first sentence, using the word given. Do not change the word given.
I. What has this experience taught you?
- What conclusions _____________________________. __________?
458
2. She gets bored quickly at parties because there is so much trivial conversation.
(SMALL)
- Too m uc h___________________________________________________ ___
3.1 think that my parents cope calmly with everything I do. (STRID E)
- I'm un de r_____________________________________ __________t____ .
4. He indicated quite clearly that he would do anything to obtain the contract.
(LENGTHS) /•
- He ga ve ________________________ "_____________________________
5. Now adays I consider casual friendships to be far less important than 1 used to.
(STORE)
- Nowadays I d on’t _____________________ ____________________
6. I'm afraid that negotiations for a settlement are completely impossible.
(WHATSOEVER)
- I 'm afraid that t he re_________________________ ____________________
TRƯỜNG THP T CHUYÊN TRẦN HƯNG ĐẠO - BÌNH THUẬN
A. MULTIPL E CHOICE
I. WORD CHOICE
Choose the best options to complete the following sentences.
1. His efforts, though futile, are s ti ll .............
A. refined B. intimidated C. pious D. commendable
2. The vegetation on the island w as.............
A. exuberant B. chivalrous C. overcast D. ingenious
3. Ronald had t h e ..................to blame his teachers for his failure.
A. concern B. chivalry C. regard D. audacity
4. Many poets hav e........ . the beauties of the countryside.
A. extolled B. feared c. excited D. inhibited
5. The plague, otherwise known as the Black Death, was a .............. disease.
A. contingent B. contiguous c. contagious D. congenial
6. It was obvious that the child suffered from a serious sp ee ch ...... .......
A. handicap B. hindrance c. inhibition D. impediment
7. His heavy bo ot s..............him at work.
A. repelled B. stemmed C. hampered D. compelled
8. His driving license has b ee n............. on the grounds o f drink driving.
A. repealed B. revoked c. nullified D. recalled
9. Mary is a (n ).............. liar. She was even arrested for lying to a police officer
A. physical B. congenital c. naive D. abnormal
10. The Prime Minister will decide whether to release the prisoner or not; that’s
h is ..............
A. prerogative B. derogatory c. abdication D. humanity
459
II. GRA MMAR AND STRUCTURES
Choose the best options to complete the following sentences.
1 ................the town posted notices urging people to boil their water.
A. The pollution of the municipal well having been discovered,
B. The municipal well's pollution being discovered,
c. After discovering pollution in the municipal well,
- D. When having made the discovery' o f the pollution of the water in the
municipal well;
2 .................... to Rose is unclear, but the letter would definitely have given her
morale a boost.
A. That Charles in fact sent the e-mail
B. Whether in reality Charles sent the e-mail or did not
c. The actuality of the sending o f the e-mail by Charles
D. Whether Charles in fact sent the e-mail or did not
3. An event in Richard's life story that moved me greatly w as................
A. when he was separated from the family
B. his separation from the family
c. when he and the family were separated
D. the separating from the family
4. Not all athletes who ........................ in a sport can be assured of a place on an
Olympic team.
A. are highly motivated or significantly talented
B. have high motivation or are significantly talented
c. have either high motivation or else talent in significant amounts
D. are highly motivated or who have significant talent instead
5. Nuclear waste disposal is a growing p rob lem ............ ...
A. considering that no state permits radioactive material transported on its
roads or to bury it inside its borders
B. considering that no state permits neither radioactive material transported
on its roads or buried inside its borders
c. because no state permits radioactive material transported on its roads or
buried inside its borders
D. because no state will permit radioactive material not only to be carried on
its roads but in addition also buried inside its borders
6. To celebrate the 100th anniversary of the colleg e,................
A. honoring those alumni who had graduated fifty years ago
B. ceremonies for graduates of fifty years ago were held in honor of these
alumni
c. alumni graduating fifty years ago received honors
D. alumni who had graduated fifty years ago were honored
460
7. Carbon dioxide may be absorbed by trees or water bodies, or it may stay in the
atmosphere w hen................................ while it is only in the atmosphere that
clorofluorocarbons find the home.
A. cars that release emissions B. released from car emissions
c. by releasing emissions from cars D. emissions are released by cars
8. In the eastern part of New Jer se y...................................... a major shipping and
manufacturing center.
A. around the city o f Elizabeth lies B. the city of Elizabeth lies there
c. there lies the city o f Elizabeth around D. lies the city o f Elizabeth
9................ parrots are native to tropical region is untrue.
A. Since all B. That all c. Why all D. All
10. Good erasers are soft enough not to damage paper................ they crumble
gradually when used.
A. so hard are they B. and hard enough so that
c. and they are so hard that D. but hard enough so that
III. PH RASAL VERBS AND PREPOSITIONS
Choose the best options to complete the following sentences.
1. If a child knows that he can nev er............. a bad behavior, then he is less
likely to do it.
A. make out B. put through c. keep up D. get away with
2. My mother always told me that 1 should .............. the things I believe in,
regardless o f how others perceive them.
A. stand up for B. get on with c. put up with D. come up to
3. The doctor told him to ke ep ....... ......sweets and chocolate to lose weight.
A. up B. at c. off D. back
4. Some o f our volunteers wou ld ............. for teachers in the event of a strike.
A. act up B. fill in c. fit in D. work out
5. She got a bit h o t...... . the collar when a colleague started criticizing her work.
A. under B. on c. beyond D. from
6. She bo ug ht .... .........the deal even though nobody thought she was capable o f
doing it.
A. in B. down c. out D. off
7. Let’s find a place where we can ............. the storm.
A. wait out B. wear off c. shrug o ff . D. pull through
8. The boss was away, so his assistant had t o ..............him and make a speech.
A. make o ff with B. do away with c. stand in for D. take up on
9. Beaches w ere ..............as police searched for canisters for toxic waste from
the damaged ship.
A. sealed off B.- cut off c. washed up D. kept out
10 .1 thought she was being serious, but she was only having m e ................
A. up B. on c. over D. round
461
IV. COLLO CATIONS AND IDIOMS
Choose the best options to complete the following sentences.
1. I t ..............that the best way to learn to speak a foreign language is to practice
using it.
A. stands in relation B. is a wake-up call
c. stands to reason D. agrees to differ
2. Police are warning the public to be on t h e ....... ......for suspicious packages.
A. care B. alert c. guard D. alarm
.3. Vietnam 's Got Talent is the game show that has taken a ud ien ces..............
A. by heart B. by night C. by wind D. by storm
4. Clinics will be subject to a n ew ..............of conduct and stronger controls by
local authorities.
A. ground B. system c. code D. set
5. My m ot he r.......... . when she found out that I'd forgotten to do the washingup again.
A. made my blood boil B. felt o ff color
c. hit the ceiling D. stood her ground
6. You sh oul dn' t............. other people’s problems, even if you do n't consider
them to be very serious.
A. shed light upon B. come to light
c. make light o f D. see the light
7. A journalist is ..............on a politician in order to damage his image.
A. digging it out B. digging up out o f the earth
c. digging up dirt D. digging his own grave
8. We were having dinner in a restaurant last night when this guy at the next
ta ble ..............because the waiter brought the wrong thing.
A. flew into the face o f danger B. flew off the handle
c. flew by the seat o f his pants D. flushed out of some place
9. Losing my job w a s.............. I never would have found this one if it hadn ’t
happened.
A.a bone to pick with
c. a bleeding heart
10. The inquiry is by no means
A. chop and change
c. head and shoulders
B. a breath of fresh air
D. a blessing in disguise
B. cut and dried
D. tooth and nail
V. READING PASSAGE 1
Read the text below and choose the best answer to each question.
A folk culture is small, isolated, cohesive, conservative, nearly self-sufficient
group that is homogeneous in custom and race, with a strong family or clan
structure and highly developed rituals. Order is maintained through sanctions
based in the, religion or family, and interpersonal relationships are strong.
462
Tradition is paramount, and change comes infrequently and slowly. There is
relatively little division of labor into specialized duties. Rather, each person is
expected to perform a great variety of tasks, though duties may differ between
the sexes. Most goods are handmade, and a subsistence economy prevails.
Individualism is weakly developed in folk cultures, as are social classes.
Unaltered folk cultures no longer exist in industrialized countries such as the
United States and Canada. Perhaps the nearest modern equivalent in An gloAmerica is the
Amish, a German American farming sect that largely renounces
the products and labor saving devices o f the industrial age. In Amish areas, horsedrawn
buggies till serve as a local transportation device, and the faithful are not
permitted to own automobiles. The Amish's central religious concept of Demut,
"humility", clearly reflects the weakness of individualism and social class so
typical of folk cultures, and there is a corresponding strength of Amish group
identity. Rarely do the Amish many outside their sect. The religion, a variety of
the Mennonite faith, provides the principal mechanism for maintaining order.
By contrast, a popular culture is a large heterogeneous group, often highly
individualistic and constantly changing. Relationships tend to be impersonal, and
a pronounced division of labor exists, leading to the establishment of many
specialized professions. Secular institutions, of control such as the police and
army take the place of religion and family in maintaining order, and a moneybased economy
prevails. Because of these contrasts, "popular" may be viewedas
clearly different from "folk".
The popular is replacing the folk in industrialized countries and in many
developing nations, Folk-rnade objects give way to their popular equivalent,
usually because the popular item is more quickly or cheaply produced, is easier
òr time saving to use, or lends more prestige to the owner.
1. What does the passage mainly discuss?
A. Two decades in modem society
B. The influence of industrial technology
c. The characteristics of "folk" and "popular" societies
D. The specialization of labor in Canada and the United States
2. The word "homogeneous" is closest in meaning to ....
A. uniform B. general c. primitive D. traditional
3. Which of the following is typical o f folk cultures?
A. There is a money-based economy.
B. Social change occurs slowly.
c. Contact with other cultures is encouraged.
D. Each person develops one specialized skill.
4. What does the author imply about the United States and Canada?
A. They value folk cultures. B. They have no social classes,
c. They have popular cultures. D. They do not value individualism.
463
5. The phrase ''largely renounces" is closest in meaning to ....
A. generally rejects B. greatly modifies
c. loudly declares D. often criticizes
6. What is the main source of order in Amish society?
A. The government B. The economy
c. The clan structure D. The religion
7. Which of the following statements about Amish beliefs does the passage
support?
A. A variety of religious practices is tolerated.
B. individualism and competition are important,
c. Pre-modem technology is preferred.
D. People are defined according to their class.
8. Which o f the following would probably NOT be found in a folk culture?
A. A carpenter B. A farmer c. A weaver D. A banker
9. The word "prevails" is closest in meaning to ....
A. dominates B. provides c. develops D. invests
10. Which of following is NOT given as a reason why folk-made objects are
replaced by mass-produced objects?
A. cost B. prestige c. quality D. convenience
VI. READING PASSAGE 2
Read the text below and choose the best answer to each question.
FINDING THE CAREER THAT FITS YOUR PERSONALITY
'If you've finished your exams and have absolutely no idea what to do next,
you're not alone,' says Sheridan Hughes, an occupational psychologist at Career
Analysts, a career counseling service. 'At 18, it can be very difficult to know
what you want to do because you don’t really know what you're interested in.'
Careers guidance, adds Alexis Hallam, one of her colleagues, is generally poor
and 'people can end up in the wrong job and stay there for years because they're
good at something without actually enjoying it.'
To discover what people are good at, and more fundamentally, what they will
enjoy doing, Career Analysts give their clients a battery of personality profile
questionnaires and psychometric tests. An in-depth interview follows, in which
the test results are discussed and different career paths and options are explored
with the aid o f an occupational psychologist. Career Analysts offers guidance to
everyone, from teenagers to retirees looking for a new focus in life. The service
sounded just, what I needed. Dividing my time as I do between teaching and
freelance journalism, I definitely need advice about consolidating my career.
Being too ancient for Career Analysts' student career option guidance and not,
unfortunately, at the executive level yet, I opted for the career management
package. This is aimed at people who are established in their jobs and who either
want a change or some advice about planning the next step in their careers.
464
Having tilled in a multitude of personality indicator questionnaires at home, 1
then spent a rather grueling morning being aptitude-tested at Career Analysts*
offices. The tests consisted of logical reasoning followed by verbal, mechanical
and spatial aptitude papers. Logical reasoning required me to pick out the next
shape in a sequence of triangles, squares and oblongs. I tried my best but knew
that it was really a lost cause. 1 fared rather better when it came to verbal
aptitude - finding the odd one out in a series of words couldn't be simpler. My
complacency was short-lived, however, when I was confronted with images of
levers and pulleys for the mechanical aptitude papers. My mind went blank. 1
had no idea what would happen to wheel X when string Y was pulled.
At home, filling in questionnaires, 1 had been asked to give my instinctive
reaction (not an over-considered one) to statements like: 'It bothers me if people
think I'm being odd or unconventional’, or 'I like to do my planning alone
without interruptions from others.' I was asked to agree or disagree on a scale of
one to five with '1 often take on impossible odds', or 'It is impossible for me to
believe that chance or luck plays an important role in my life.' I was told to
indicate how important 1 consider status to be in a job, and how important money
and material benefits.
The questions attempt to construct a picture of the complete individual. Using
aptitude tests alongside personality profiling, occupational psychologists will,
the theory goes, be able to guide a client towards a rewarding, fulfilling career.
Some questions are as straightforward as indicating whether or not you would
enjoy a particular job. Designing aircraft runways?
Preparing legal documents? Playing a musical instrument? Every career going
makes an appearance and, as 1 was shown later, the responses tend to form a
coherent pattern.
Having completed my personality and aptitude tests, I sat down with Sheridan
Hughes, who asked me fairly searching personal and professional questions.
What do my parents and siblings do for a living? Why had 1 chosen to do an
English degree? 'I need to get a picture of you as a person and how you've come
to be who you are,’ she explained. 'What we do works because it’s a mixture of
science and counseling. We use objective psychometric measures to discover our
clients' natural strengths and abilities and then we talk to them about what they
want from life.'
There were no real surprises in my own test results, nor in the interview that
followed it. 'We're interested in patterns,' Mrs Hughes explained, 'and the pattern
for you is strongly verbal and communicative.' This was putting it rather kindly.
I had come out as average on the verbal skills test and below average in logic,
numerical, perceptual and mechanical reasoning. My spatial visualization was so
bad it was almost off the scale. 'A career in cartography, navigation, tiling or
architecture would not be playing to your strengths,' she said delicately.
Mrs Hughes encouraged me to expand the writing side o f my career and gave
me straightforward, practical suggestions as to how I could go about it. 'Widen
465
the scope of your articles,' she said. 'You could develop an interest in medical
and psychological fields.' These latter, she said, would sit comfortably with an
interest in human behavior indicated on my personality-profiling questionnaires.
She suggested that I consider writing e-learning content for on-line courses, an
avenue that would never have occurred to me.
1. Which of the following is mentioned in the first paragraph?
A. people underestimating their own abilities
B. people accepting inappropriate advice
c. people being unwilling to take risks
D people constantly changing their minds
2. What does the writer say about Career Analysts in the second paragraph?
A. It is about to offer a service for people at executive level.
B. The range of services it offers is unique.
c. She was initially doubtful that it could be useful to her.
D. Only one of its services was relevant to her.
3. What happened when the writer took the aptitude tests?
A. She found two of the papers extremely difficult.
B. She put in very little effort on any of them.
c. She didn't understand what she was required to do on one o f them.
D The papers were not what she had been expecting.
4. The phrase "a lost cause " in paragraph 3 is closest in meaning to ....
A. a dead-end B. a shortcut c. a vain attempt D. a misjudgement
5. What does the writer say about the statements on the questionnaires?
A. She thought about them for longer than she was supposed to.
B. She found some o f them rather strange.
c. One of them focused on her attitude to risk.
D. One o f them concerned her current situation only.
6. The writer says that the idea behind the questionnaires is that
A. people will find some o f the questions quite hard to answer.
B. the answers to thernx and the aptitude tests will provide all the necessary
information. .
c. they will encourage people to have new ideas about possible careers.
D. they will give a more accurate picture of people than the aptitude tests.
7. Some o f the questions Sheridan Hughes asked concerned the writer’s
A. opinions of the tests and questionnaires.
B. relationships with family members,
c. main regrets.
D. progress through life.
8. The writer felt that during the interview, Mrs Hughes
A. was keen not to upset her concerning her test results.
B. seemed surprised at how badly she had done in the tests,
c. was being honest about her strengths and weaknesses.
D. preferred to avoid talking about her test results.
466
9. The phrase "off the scale" in paragraph 7 is closest in meaning to ....
A. unsatisfactory B. not so good c. worrying D. immeasurably low
10. The advice Mrs Hughes gave to the writer included the suggestion that she
should
A. think about taking a course on writing.
B. concentrate only on writing and not on any other kind o f work,
c. increase the number of subjects she writes about.
D. do something she had previously considered unappealing.
VII. GUIDED CLOZE 1
Read the text below and decide which answer best fits each space.
There can be no (1)............. that online shopping is of huge benefit to the
consigner. Far from becoming (2)............. , online shoppers are very demanding.
Overpriced merchants with poor services should beware. Gone are the days
when stores could charge what they liked for goods and get away with it. The
same, too, for shady manufacturers: smarter consumers know which products
have a good (3)............ and which do not, because online they now read not only
the sales (4)........... but also reviews from previous purchasers. And i f customers
are disappointed, a few (5)............. of the mouse will take them to places where
they can let the world know. Nowadays there is nothing more damning than a
flood o f negative comments on the internet.
However, the big boys, as always, are ahead of the game. Some companies
are already adjusting their business models to take account of these trends. The
stores run by Sony and Apple, for instance, are more like brand showrooms than
shops. They are there for people to try out (6)............. and to ask questions to
knowledgeable staff. Whether the products are ultimately bought online or
offline is of secondary importance.
Online traders must also adjust. Amazon, for one, is (7)............. turning from
being primarily a bookseller to becoming a (8)............ retailer by letting other
companies sell products on its site, rather like a marketplace. During America's
Thanksgiving weekend last November, Amazon's sales of consumer electronics
in the United States (9)........... its book sales for the first time in its history.
Other transformations in the retail business are (10)............. to follow.
1. A. query B. examination c. question D. proposal
2. A. complacent B. dissatisfied. c. competent D. compassio
3. A. distinction B. resolution c. opinion D. reputation
4. A. bubble B. message c. blare D. blurb
5. A. taps B. clucks c. clicks D. prods
6. A. devices B. tools c. emblems D. schemes
7. A. mistakenly B. rapidly c. unreasonably D. secretly
8. A. mass B. block c. lump D.chunk
9. A.reced ed B. excluded c. repressed D. exceeded
10. A. tied B.secured c.b ou nd D. fastened
467
VIII. GUIDED CLOZE 2
Read the text below and decide which answer best fits each space.
Greenhouse gases are being released into the atmosphere 30 times'faster than
the time when the Earth experienced a (1)............. episode of global warming. A
study comparing the rate at which carbon dioxide and methane are being
(2)............. now, compared to 55 million years ago when global warming also
occurred, has found dramatic differences in the speed of release. James Zachos,
professor of earth sciences at the University of California, Santa Cruz, said the
speed of the present buildup of greenhouse gases is far greater than during the
global warming after the (3)........ ..... of the dinosaurs. "The emissions that
caused this past episode of global warming probably lasted 10,000 years,"
Professor Zachos told the American Association for the Advancement of Science
at a meeting in St. Louis. "By burning fossil fuels, we are likely to emit the same
amount over the next three centuries." He warned that studies of global warming
events in the geological past (4)..... ........the Earth's climate passes a (5)...............
beyond which climate change accelerates with the help of positive feedbacks -
vicious circles of warming. Professor Zachos is a leading (6).............. on the
episode of global warming known as the palaeocene-eocene thermal maximum,
when average global temperatures increased by up to 5°c due to a massive
release o f carbon dioxide and methane.
His research into the deep ocean (7)..... ........suggests at this time that about
4.5 billion tons of carbon entered the atmosphere over 10,000 years. "This will
be the same amount of carbon released into the atmosphere from cars and
industrial emissions over the next 300 years if present (8)............. continue", he
said. Although carbon can be released suddenly and naturally into the
atmosphere from volcanic activity, it takes many thousands of years for it to be
removed permanently by natural processes. The ocean is capable of removing
carbon, and quickly, but this natural (9)............. can be easily (10)..............,
which is probably what happened 55 million years ago. "It will take tens of
thousands of years before atmospheric carbon dioxide comes down to
preindustrial levels," the professor said. "Even after humans stop burning fossil
fuels, the effects will be long-lasting."
1. A. prearranged B. premier c. previous D. fundamen
2. A. emitted B. exhaled c. incorporated D. digested
3. A. dementia B. demolition c. detachment D. demise
4. A. comment B. mark c. compliment D. indicate
5. A. barricade B. verge c. threshold D. perimeter
6. A. autocrat B. authority c. administrator D. proprietor
7. A. dusts B. sediments c. dirt D. powder
8. A. trends B. gadgets c. fads D. crazes
9. A. capacity B. competence c. intelligence D. bulk
10. A. overcharged B. overstated c. overshadowed D. overwhelm
468
B. WRITTEN TEST
I. CLOZE TEST: Read the texts below and think of the word which best fits
each space. USe only ONE WORD for each space.
OPEN CLOZE 1
As petrol prices continue to (1)..............many people are looking for ways to
reduce the (2)..............of higher prices while still doing the driving necessary to
their work and other activities. (3)..............are some suggestions which will save
you a (4)..............amount of money on petrol.
1. Ask yourself every time you (5)..............to use your car. truck, SUV, or van,
"Is this trip really necessary?" Every mile you drive your vehicle will cost
you at least an (6)..............of 36 cents. If the trip is not necessary, think twice
before using your vehicle.
2. Drive at a (7)................. speed on the motorway. According to the Department
of Energy, most automobiles get about 20 percent more miles per gallon on
the motorway at 55 miles per hour than they do at 70 miles per hour.
3. Consider (8).............. an automobile which gets the best petrol mileage. For
example, generally, the following get better petrol mileage: lighter weight
vehicles, vehicles with smaller engines, vehicles with manual transmissions,
those with four cylinders, and those with fewer accessories. Check the "fuel
economy" labels (9)............. to the windows of new automobiles to find the
average estimated miles per gallon for given makes and models.
4. Decrease the number of short trips you make. Short trips (10)..............reduce
petrol mileage. If an automobile gets 20 miles per gallon in general, it may
get only 4 miles per gallon on a short trip of 5 miles or less.
OPEN CLOZE 2
A Nobel Prize-winning scientist has proposed a controversial method for
protecting Earth from global warming: (1)............. the atmosphere with sulfur to
reflect the sun’s rays. Paul Crutzen of Germany's Max Planck Institute for
Chemistry suggests (2)............. particles of sulfur into the (3).............. - the
upper layer of the atmosphere - to cool the planet and buy time for humans to
reduce greenhouse gas emissions. The sulfur (4)............. would be dropped from
high-altitude balloons or fired into the atmosphere with heavy artillery shells.
Once airborne the particles would act like tiny mirrors, (5)...... ....... the sun's
light and heat back into space. Crutzen’s plan would imitate the cooling effects
of volcanic eruptions, which send large sulfur-rich clouds into the atmosphere.
This is not the first time that scientists have suggested (6).............. with the
Earth's climate in order to reduce the impact of global warming. John Latham
and his colleagues had put forward a plan to (7)............. ......... up seawater to
encourage cloud formation in the lower atmosphere, (8)............ . reflecting
radiation back into space. Latham, who has commented on Crutzen's idea,
believes that his plan is (9)............. . but that further investigation is needed.
469
Crutzen admits that there is risk of the sulfur becoming a health hazard if it
rained back down on earth. In addition there could be an increase in damage to
the ozone layer and a whitening of the sky. On the (10)............... sunsets and
sunrises would become more spectacular.
n. WORD FORMATION
PA RT I:
Complete each sentence, using the correct form o f the word in parentheses.
1. Patrician and charming, he controls his empire with quiet .............. from his
office in St Rémy, his native town. (PATERNAL)
2. At the beginning of the 19th century, Britain 's.............was France. (ENEMY)
3. There is an ..............(CURRENT) of menace and barely suppressed violent
that gives the picture o f symbolic edge.
4. I ts ..............makes food distribution difficult. (ACCESS)
5. She stood there completely .............., so I had no idea at all what she was
thinking. (EXPRESS)
6. The majority of infected people a re ............. and unaware of their condition.
(SYMPTOM)
7................conditions can be diagnosed from the early stage. (CANCER)
8. The new law represents a/an ............. intrusion into the privacy of the
individual. (JUSTIFY)
9. The draft law was passed by 134 votes to 19, with 5 ................(ABSTAIN)
10. Behavior, such as.......and cooperation is not bred in the bone. (RECIPROCATE)
PART 2:
Complete the passage with appropriate forms of the words given in the box.
hold credit cogitative allegation rely
controvert celerity hospitality predict derecpiisition
With the rapid depletion of fossil fuel reserves, nuclear power is back on the
political agenda as the greenest and most realistic energy source for the future,
not without (1).............. , though. Until recently, disasters such as Three Mile
Island and Chernobyl underpinned the commonly-held view that the technology
was fundamentally (2)...............However, as traditional fossil fuel counterparts
choke the atmosphere and climate change reduces workable pasture land to
(3)............. deserts, a new tolerance may have to be (4)................ Those who
once (5)..............it face the unsettling realization that nuclear power may be the
least harmful for future energy needs and the most viable in comparison with
renewable energy sources like wind, wave, and solar power.
(6)...........highlight the fact that nuclear power has no regulated air emissions,
which means no greenhouse gases; in terms of radioactivity, they (7)..............
that fossil fuel equivalents, such as coal, release far more radiation than nuclear
power plants. They also list the number o f deaths per year that can be (8)............
470
to mining operations and question a (9)............. on fossil fuel combustion that
has brought the Earth to the brink of ecological catastrophe. According to these
arguments, nuclear power is the key to the (10)............. demands of our energyintense society.
III. ERROR CORRECTION
The following passage contain 10 errors. Identify and correct them.
15 An environmental history of
10 mankind would have to be a
15 history of the
20 exploitation of abundant
natural resources, the spiral
demand for these
resources, and their
inevitable depletion. As
humanity spread over the
globe, leaving colonies in
their wake, essential
resources such as coal, oil,
and even fresh water were
extracted through industrial
mining and
manufacturing operations
that had a massive impact on
the Earth itself.
That was once a plethora of
riches has inexorably
declined, and
entrepreneurs are now
looking into the limits of
land, sea, and sky; the
new target is the stars and,
more especially, asteroids.
Spinning around
the Sun are tens of
thousands of asteroids, and
scientists have convinced
that these mountain-like
formations contain a
treasure trove of minerals
and metals. The asteroid 16
Psyche has enough iron-
nickel ore to sustain
the Earth for several million
years. Even a comparatively
small asteroid
could contain more than
2,000 million metric tons o f
serviceable mineral
metal reserves. In addition,
some asteroids have a high
ice content which
means that they could be an
economically viable source
of fresh water.
Large-scale mining of
asteroids is possibly the key
to solve many of our
escalating environmental
problems. Any mining
venture contains an
element of risk, however.
Asteroids have traditionally
been considered
dead perils lurking out in the
dark depths of space, bided
their time before
smashing headlong into our
insignificant planet. In order
to transplant
mining operations from the
Earth to the stars, we need to
find solutions to
extensive safety and
logistical problems.
1._______________
4 ._______________
7 ._______________
2.
5._______________
8._______________
3. ___________
6. ■ ___________
9. _______________
10.
IV. SENTENCE TRANSFORMATION
Rewrite the following sentences using the words given.
1. Coming second didn’t make her feel any better because she only wanted to
win. (CONSOLIDATION)
Coming seco nd ................................................. ...................................................
2. His analysis of the situation was far too complex for me to grasp. (HEAD)
His analysis of the situ ation................................................................................
471
3. The committee members said that they would remain loyal to the chairman.
(PLEDGED)
The committee mem bers................................................................. ......................
4 .1 can spend more time with my grandchildren when I retire. (FREE)
Re tirem ent............................................................................................................. <
5. In the area, Thailand is much better than all other countries in football.
(ABOVE)
In the a re a, ..............................................................................................................
6. Th ey'v e been having discussions on the issue for over two weeks.
Discu ssions............................................................................................................
7. What put me off the idea was simply how expensive it was going to.
The sheer ...................................................................................... :........................
8. Every possible effort was made by orphanage to find the boy 's parents.
The orphanage left no ston e................................................................................
9. Whatever the methods used to obtain the results, drugs were definitely not
involved.
There was no qu estio n...........................................................................................
10. He threatened the officers with violence.
He m ad e.................................................................................................................
TRƯỜNG THPT CHUYÊN PHAN NGỌC HIỂN - CÀ MAU
A. MULTIPLE CHOICE (40 pts)
I. WORD CHOICE (10 pts)
1. When the ship docked at Hamburg, they found a ____________ in the hold.
A. gate-crasher B. stowaway c. interloper D. trespasser
2. Although Vicky looked pretty much the same after all those years, I noticed
___________ changes which made her look even more beautiful than I
remembered.
A. subtle B. sensitive c. fair D. joint
3. Several of the advertising hoardings had be en _______ by anti-exist slogans.
A. deleted B. mutilated c. erased D. defaced
4. The s mok e_____ from the burning tyres could be seen for miles.
A. bulging B. radiating c. billowing D. sweeping
5. The cat slept peacefully ______________ in the long grass.
A. nestled B. huddled c. snuggled D. cuddled
6. It was decided that the cost of the project would be_____ and so it was
abandoned.
A. repressive B. prohibitive c. restrictive D. exclusive
7. Pulling the contract out o f the envelope, she ripped it to ________
A. smithereens
B, tiny pieces c. shreds D. half
472

8. Employees of the company are forbidden to _____________ information


about the secret formula.
A. betray B. divulge c. portray D. unveil
9. The fire ____________ for days until the monsoon rains put it out.
A. ignited B. scorched c. flashed
10. My mother had to take private pupils in order to _
_____ her salary as
a teacher.
A. augment B. expand c. complete
D.raged
D. inflate
II. STRUCTURES AND GRAMMAR (10 pts)
Choose the best option A, B, c or D.
11. In her writing, Elimor Wylie often dealt with her own personality as it was,
rather th an _________ .
A. as was defines by others B as others defined it
c. other's definition D. its definitions by others
12. It would be difficult for a man o f his political affilia tion,_______ , to become
a senator from the south.
A. though charming and capable is he B. even with charm and so capable
c. charming and having capability D. however charming and capable
13. _______________ left before the deadline, it doesn't seem likely that John
will accomplish the job.
A. Although such a short time B. It is such a short
c. With so short time D. With such a short time
14. _______ around stones that are sunwarmed, even the smallest of stones
creates tiny currents of warm air.
A. The cool air B. If the air is cool
c. That the air cools D. The cooler the air
15. Everything looks very positive for the com pany,_____ the current investors
do not default on their agreements.
A. assuming that B. whether c. whereas D. as if
16. _____ ca me_____ as the injured cat was so weak.
A. Afterwards/dying B. Death/X c. Soon/death D. Then/to die
17. Probably no man had more effect on the daily lives of most people in the
United Sta tes_____.
A. as Henry Ford, a pioneer in automobile production.
B. rather than Henry Ford, a pioneer in automobile production,
c. than did Henry Ford, a pioneer in automobile production.
D. more than Henry Ford, a pioneer in automobile production,
18. For the first few months the babies looked so alike I couldn't t el l_____ .
A. who is who B. which is which
c. which from which D. whom with whom
473
19 ._____ , the meeting began.
A. After we have sat down B. All of US having taken the seats
C. Our having seated D. Once we had seated
2O.She knew her father___ when he threatened to ground her if she failed the test.
A. was begging the question B. hit the nail on the head
c. meant business D. was a glutton for punishment
III. PREPOSITION S AND PHRASAL VERBS (10 pts)
Choose the best option.
21. It’s like banging your h ea d............................a brick wall.
A. into B. against c. onto D. up
22. We lay in the sun until our bodies felt saturated.......... ....................the heat.
A. up B. with c. upon D. within
23. Some very important issues was taking..............................all his attention.
A. up B. on c. out D. within
24. H e......................... by pointing out the dangers involved in rock climbing.
A. started on B. started o ff c. started up D. started out
25. S he......................... till the early hours listening to pop music.
A. help me up B. caught me up c. kept me up D. took me up
26. Old Mr Brown's condition looks very serious and it is doubtful if he will.....
A. pull through B. pull upc. pull back D. pull out
27. I do wish you wouldn’t .............Charles. I'm trying to talk to your father.
A. put off B. call up c. butt in D. tell off
28. After the accident at the nuclear power station, the authorities tried to
..................the danger to the public from radioactive waste.
A. calm down B. get through c. get over D. play down
29. The witn ess's ev idence ................what Peter had said.
A. bore out B. gave of f c. told off D. wiped out
30. When David started speaking everyone f e ll ............ laughing.
A. up B. at c. in D. about
IV. COLLOCATIONS AND IDIOMS (10 pts)
Choose the best option.
31. Thanks for listening to me complain about my boss. 1 just needed to_______
A. cry my heart out B. get this off my chest
c. face the music D. stick my neck out
32. The news o f his death was like a bolt_____
A. from the red B. from the blue c. from the black D. from the white
33. This is an exciting book wh ich ____________ new ground in the educational
research
A. breaks B. reaches c. scratches D. turns
34. Were you on th e__________ when you said you had resigned from work?
A. wagon B. flat c. level D. town
474
35. This painting stands a go od _____ of winning the prize.
A. possibility B. chance . c.opportunity D. certainty
36. The optician says you have to wear glasses, like it o r___________ it.
A. jump B. loathe c. dislike D. lump
37. When her daughter didn't come home on the last bus, Mrs. Lee was a t_____ .
A. a loss B. death's door
c. her wits' end D. the crack of dawn
38. Her health has improved in _____.
A. leaps and bounds B. odds and ends
c. sick and tired D. trial and error
39. We've lost everything. Still, there's no point in complaining. We’ll jus t have
to try and mak e_____ .
A. the best o f a bad job B. no bones about it
c. a splash D. a clean sweep
40. He made a number o f_____ remarks about my cooking, which upset us.
A. slashing B. stabbing c. chopping D. cutting
V. READING COMPREHENSION (20 pts)
READING PASSAGE 1 (10 pts)
Read the following text and choose the best option to answer the question.
The first peoples to inhabit what today is the southeastern United States
sustained themselves as hunters and gathers. Sometimes early in the first
millennium A.D., however, they began to cultivate corn and other crops.
Gradually, as they became more skilled at gardening, they settled into permanent
villages and developed a rich culture, characterized by the great earthen mounds
they erected as monuments to their gods and as tombs for their distinguished
dead. Most of these early mound builders were part of the Adena-Hopewell
culture, which had its beginnings near the Ohio River and takes its name from
sites in Ohio. The culture spread southward into the present-day states of
Louisiana, Alabama, Georgia, and Florida. Its peoples became great traders,
bartering jewellery, pottery, animal pelts, tools, and other goods along extensive
trading networks that stretched up and down eastern North America and as far
west as the Rocky Mountains.
About A.D. 400, the Hopewell culture fell into decay. Over the next centuries,
it was supplanted by another culture, the Mississippian, named after the river
along which many of its earliest villages were located. This complex civilization
dominated the Southeast from about A.D. 700 until shortly before the Europeans
began arriving in the sixteenth century. At the peak of its strength, about the year
1200, it was the most advanced culture in North America. Like their Hopewell
predecessors, the Mississippians became highly skilled at growing food, although
on a grander scale. They developed an improved strain of com, which could
survive in wet soil and a relatively cool climate, and also learned to cultivate
475
beans. Indeed, agriculture became so important to the Mississippians that it
became closely associated with the Sun - the guarantor o f good crops. Many tribes
called themselves "children of the Sun" and believed their omnipotent priestchiefs were
descendants o f the great sun god.
Although most Mississippians lived in small villages, many others inhabited
large towns. Most of these towns boasted at least one major flat-topped mound
on which stood a temple that contained a sacred flame. Only priests and those
charged with guarding the flame could enter the temples. The mounds also served
as ceremonial and trading sites, and at times they were used as burial grounds.
41. What does the passage mainly discuss?
A. The development o f agriculture
B. The locations of towns and villages
c. The early people and cultures of the United States
D. The construction of burial mounds
42. Which of the following resulted from the rise o f agriculture in the southeastern
United States?
A. The development of trade in North America
B. The establishment of permanent settlements
c. Conflicts with other Native American groups over land
D. A migration of these peoples to the Rocky Mountains.
43. What does the term ’’Ad en a-Ho pe we ll” (line 7) designate?
A. The early locations of the Adena-Hopewell culture
B. The two most important nations of the Adena-Hopewell culture
c. Two former leaders who were honored with large burial mounds.
D. Two important trade routes in eastern North America
44. The word ’’bart ering” in line 11 is closest in meaning to
A. producing B. exchanging c. transporting D. loading
45. The word ’’supp lan ted” in line 15 is closest in meaning to
A. conquered B. preceded c. replaced D. imitated
46. According to the passage, when did the Mississippian culture reach its
highest point of development?
A. About A'D. 400 B. Between A.D. 400 and A.D. 700
c. About A.D. 1200 D. In the sixteenth century
47. According to the passage, how did the agriculture of the Mississippians differ
from that o f their Hopewell predecessors?
A. The Mississippians produced more durable and larger crops of food.
B. The Mississippians sold their food to other groups.
c. The Mississippians could only grow plants in warm, dry climates.
D. The Mississippians produced special foods for their religious leaders.
48. Why does the author mention that many Mississippians tribes called themselves
’’childr en of the Sun" (line 25)?
A. To explain why they were obedient to their priest-chiefs.
476
B. To argue about the importance of religion in their culture,
c. To illustrate the great importance they placed on agriculture.
D. 1 o provide an example of their religious rituals.
49. The phrase ’’charg ed w ith" in line 30 is closest in meaning to
A. passed on B. experienced at
c. interested in D. assigned to
50. According to the passage, the flat-topped mounds in Mississippian towns
were used for all of the following purposes EXCEPT
A. religious ceremonies B. meeting places for the entire community
c. sites for comm erce D. burial sites
READING PASSAGE 2 (10 pts)
Read the f ollow ing text and cho ose the best o ption to answer the q ues tion.
SPEAKING IN ONE TONGUE
English is spreading and it has been predicted that one in ten of the world's
6,000 languages will become extinct over the next century. Up to half of the
wor ld's languages are no longer being taught to children, threatening them with
eventual extinction. Even countries with millions o f native language speakers are
so worried by the growth of English that they have devised policies to fight back.
The French have brought in regulations to combat what they see as an
American cultural invasion. Corporations and government bodies are not
allowed to use English terms where there are French equivalents. And to ensure
there are as many of these as possible, a Terminology Commission has been set
up with the task of creating them. There is widespread concern that the American
influence could mean local films, TV, music and books get pushed into the
background.-In order to protect local language and culture, The European Union
introduced new legislation which states that half of the TV programmers shown
in member states must be European.
New technology does not make things easy for other languages. It is especially
difficult to bold bac k the tide of English words in high-tech industries because
many of the innovations are American. The German have their own words for
“computer”, 'smart-card", “DVD”, “modem” and “handheld PC”, but hardly
anyone uses them. Until recently all university subjects in Malaysia were taught
in Malay. Now, however, universities have had to make exceptions for
Information Technology, as the majority o f IT textbooks are in English and they
simply do not have the time or resources to translate them.
Today, there is another medium to worry about: The Internet. English accounts
for about 90 percent of traffic and World Wide Web will only accelerate its spread
around the world. Unlike broadcasting, however, most communication on the Net is
written, so it indent pose the same threat to regional accents and dialects. It is also
decentralized and more interactive than broadcasting, which may help to prevent the
disappearance of minority languages.
477
Even if English were universally adopted, this would not mean the end of
diversity. Languages are constantly adapting and English is not exactly the same
the world over: you only have to compare American and British English to realize
that. The Net, too, is an instrument of change. Because Net communication is
mainly written, many people use abbreviations to speed things up and this trend
means the language is evolving with Internet use. In 10 to 20 years from now the
English on the Net may well be unrecognizable compared with the English we
know now.
English will continue to spread and evolve with the Net but people will still
speak their own languages. In most cases, they'll use English for electronic
communication, but native languages at home. Languages are more than ju st a
means o f communication: they are also an important part of regional cultures and
identities and they do not disappear easily. For centuries, the trading world spoke
Latin but it did n't go away with French or German. Instead, Latin became the
dead language.
Languages have their own dynamics and there is little governments can do to
change their course. In 18th - century Germany. Frederick II set up a commission
to get rid of French words from German. It failed because the dynamics of a
language come from the bottom up rather than the top down.
English may dominate but it wo n't wipe every other language off the face o f
the Earth and it wo n't be the same English spoken everywhere. This may not be
enough to prevent the disappearance of some of the world’s languages, but it
does mean that there will not be a boring worldwide uniformity.
51. Faced with the dominance of English, what does the writer think will happen
to the world's other languages?
A. Many will exist alongside English.
B. They will only be spoken at home,
c. Most of them will die out.
D. They will consist mainly of English words.
52. According to the writer, one reason why some languages will become extinct
is th at........
A. English has become the first language in some countries.
B. there are not enough language teachers,
c. they are not spoken all the time.
D. young people are not learning them.
53. The French government introduced regulations in order to...........
A. limit the use of English words.
B. increase the number of native language speakers,
c. control the amount of American English spoken.
D. restrict the number of English films on television.
478
; 54. The phrase ’’hold back " in paragraph 3 is closest in meaning to
A. impede the progress of. B. have the influence of
c. maintain the grip o f D. maintain the position of
55. What does “they' (in the last sentence of paragraph 3) refer to?
A. university subjects B. Malaysian universities
c. exceptions D. IT textbooks
56. The w riter says that as a result of the Internet some languages mig ht.........
A. be saved from extinction B. become more widespread
c. only be used in written form D. no longer be used on radio and television
57. According to the writer, what effect will the Internet have on English?
A. English on the Internet could soon be difficult to understand.
B. English will be quicker to read.
c. British English words will not be used on the Internet.
D. The appearance of English could change considerably on the Internet.
58. What does the writer mean when he says “ the dynamics of a language come
from the bottom up rather than the top down” (in next-to -t he last paragraph)?
A. The least frequent words of a language are the first to change.
B. Changes in language are not determined by how powerful a country is.
c. People, not governments, determine what happens to a language.
D. Unwanted foreign words disappear naturally.
59. The phrase “wi pe o ff ’ in the last paragraph is closest in meaning to
A. cancel B. remove c. delete D. clean
60. The word “u nif ormity ” in the last paragraph is closest in meaning to
A. variety B. sameness c. uniqueness D. consistency
VI. GU IDED CL OZ E TE ST (20 pts)
Read the fo llow ing passages and choo se the op tions that best complete the blanks.
GUIDED CLOZE TEST I (10 pts)
A COMMUNITY CHOIR
Open Voices is a community choir based in the town o f Kingston, in Ontario,
Canada. It was founded last year by a man called Andy Rush, an accomplished
' musician with a 16-year (0)____B____ record as a choir director.
Andy began by searching the way other community choirs were (6 1) ______ ,
before deciding on the (6 2)_________ he wished to use for his own choir. He
then advertised for people to come and try out the choir. There were no auditions
and a (63) _________ in music was not necessary. The purpose of the trial
session was simply to give people the opportunity to sing in a choir and get a
(6 4)_________ for the experience before making any (6 5) __________ to it. In
order to overcome the barriers that can prevent people jo ining groups like this,
he provided transportation, childcare, and subsidized membership fees for those
who needed them. His goal was to make Open voices inclusive and welcoming,
and to (6 6) ____________ people from a variety of musical, cultural and social
environments.
479
Andy expected 50 or 60 responses to his advertisement. In the event, 279
people (6 7)________ an interest in joining the choir. In ord er to (6 8)_________
everyone, the choir had to be (6 9 )____________ into two groups, each one
rehearsing on a different day. Open Voices has now held several (7 0) _________
successful concerts and has many more planned for the future.
0. A. course B. track c. path D. line
61. A. set down B. set off c. set up D. set on
62. A. guide B. model c. sample D. pattern
6 3 .A. backup B. backdrop c. background D. backlog
64. A. touch B. mood c. se ns e D. feel
65. A. commitment B. allegiance c. dedication D. assurance
66. A. catch on B. pull up c. take out D. bring in
67. A. suggested B. expressed c. described D.conv eyed
68. A. accommodate B. contain c. arrange D. maintain
69. A. torn B. cut c. sliced D. split
70. A. fully B. deeply c. highly D. greatly
GU IDED CL OZ E TE ST 2 (10 pts)
Read the text below and decide which answer (A, B, c , or D) best fits each space.
Interpreting the feelings of other people is not always easy, as we all know,
and we rely as much on what they seem to be telling US, as on the (71 ).................
words they say. Facial (72)................ and tone of voice are obvious ways of
showing our ( 73)....... to something, and it may well be that we unconsciously
(74) .....................views that we are trying to hide. The art of being tactful lies in
(75) ...............these signals, realizing what the other person is trying to say, and
acting so that they are not embarrassed in any way. For example, we may
understand that they are in fact reluctant to answer our question, and so we stop
pressing them. Body movements in general may also (76)....... .................
feelings, and interviewers often pay particular attention to the way a candidate
for a job walks into the room and sits down, However, it is not difficult tó
present the right kind of appearance, while what many employers want to know
relates to the candida te's character (7 7)....................., and psychological stability.
This raises the (78 )......................... questions of whether job candidates should
be asked to complete psychological tests, and the further problem of whether
such tests actually produce reliable results. For many people, being asked to take
part in such a test would be objectionable (79)........ into their private lives. Quite
apart from this problem, can such tests predict whether a person is likely to be a
(80).............employee or a valued colleague?
71. A. other B. real c. identical D. actual
72. A. looks B. expression c. image D. manner
73. A. view Bi feeling c. notion D. reaction
74. A. express B. declare c. exhibit D. utter
75. A. taking down B. putting across c. picking up D. going ove
76. A. display B. indicate c. imply D. infer
48 0
77. A. quirks B. mannerisms c. traits D. points
78. A. awkward B. risky c. unpleasant D. touchy
79. A. invasion B. intrusion c. infringement D. interferen
80. A. pedantic B. particular c. laborious D. conscienti
B. WRITTEN TEST
I. OPEN CLOZE TEXT
OPEN CLOZE TEXT 1 (10 pts)
Fill each of the numbered blanks in the passage with one suitable word.
When you want to book a flight to a certain place, visit your (1)_________
travel agent. Tell the travel agent the date when you want to travel and the
(2) ____________ to which you want to go. The travel agent will then type the
(3) _________ into a small computer. After checking everything on the computer
screen, he/she will send the information to a central computer. The central
computer contains all information about (4) ______ and destinations, and
sends a reply, (5)_______ _ which are the most suitable flights. It also shows
whether the flights are (6)_________booked or not. The information which is
now shown on the small computer screen is continually changing (7)________
other bookings are made in other parts o f the world.
The travel agent now types in your booking, then the computer will ask for
your name and address as well as for (8)_________ other information. It will also
ask how you will pay for your ticket (by cheque or by credit card). Next the
computer (9)_________ the booking and makes a request for payment. When you
have paid for your ticket, the travel agent types the information into the
computer as well. Finally, some computers (10)______out a ticket before you
leave the travel agent's.
OPEN CLOZE TEXT 2: (10 pts)
Fill each gap with ONE suitable word.
Large animals that (11)____the desert have evolved a number of adaptations
for (12)___________ the effects of extreme heat. One adaptation is to be light in
color, and to (13)_______ rather than absorb the Sun's rays. Desert mammals
also depart from the normal mammalian practice of maintaining a (14)_______
body temperature. Instead of trying to keep down the body temperature deep
(15)_____ the body, which would involve the expenditure of water and energy,
desert mammals allow their temperatures to rise to (16)____________ would
normally be fever height, and temperatures as high as 46 degrees Celsius have
been measured in Grant’s gazelles. The (17)_________body then cools down
during the cold desert night, and indeed the temperature may fall unusually
(18)___________ by dawn, as low as 34 degrees Celsius in the camel. This is an
advantage since the heat of the first few hours of daylight is absorbed in
warming up the body, and an excessive (19)_______o f heat does not begin until
well (2 0)_ ____ the day.
481
II. WORD FORMS (20 pts)
1. Supply the correct word form (10 pts)
1. He was charged with causing a _______ after the game. (DISTURB)
2. Children of school age are very____________ and tend to believe what they
are told. (IMPRESSION)
3. She found the idea d eeply ___________ (REPEL)
4. The two pairs of twins make a pleasant____ during the gam e o f golf. (FOUR)
5. T he____ woman did not utter a single word when her husband abused her.
(SUBMIT)
6. Blinking to ____________ herself to the darkness, Lexa already had her hands
over her ears when the deafening blast of thunder reached her. (CUSTOM)
7. In my heart of hearts, I think______________ laws such as these are
abominations that bring the entire legal system into disrepute.
8. The conference touched on many things, but one more deeply than the fragile
environment that still exists around the very concept o f_________ . (TOUR)
9. Time and again, the club has b een__________ on the brink o f selling off their
antiquated ground. (SUPPOSE)
10. We must value that reputation and work together to nurture it and remove
an y_______________ that will put it at risk. (CONCEIVE)
2. Supply each gap with the correct form of the word given in the
box (10 pts)

problem endurance authorise perc eive


heart
imitate face illusion
-•
In 1997 I went back to Beijing for the first time since the (0) ... disastrous....
events of 1989. The Chinese (11)........ had been reluctant to re-admit foreign
journ alists who had witnessed the Tiananmen Square student protests. Every
eight years later ,it was still (12) .. .. .. .to get into the Square with a television
camera, but we managed it. I looked for the bullet holes on the steps of the
central monument, but they had all been expertly filled in; a faint discoloration
perhaps, but almost (13) ......... The most critical moment in Chinese history
after Mao Zedong's death seemed to have been entirely forgotten. My time in
China had given me an (14) ........ interest in Chinese art, so 1 decided to go to
Liu Li Chang, where for centuries there has been an antiquities market.
Unfortunately, many things for sale there nowadays are modern (15) .........
Empty-handed and somewhat ( 1 6)........ , I went into a tea house and sat through
the usual ceremony, but there were (17) .......diffe ren ces here too: it seemed
quicker and the tea lacked that extraordinary lingering scent. Thoroughly
(1 8)........ , I returned to my hotel: one o f the enormous, (19) ....... places which
have sprung up everywhere. Yet here, in a dark shop tucked away off the lobby,
my melancholy mood disappeared, for I met a (20) ........ from 1989, who
remembered me instantly. Not everything had been entirely forgotten
482
III. ERROR IDENTIFICATION (20 pts)
Read the following passage. There are 10 errors. Identify the errors and
then correct them. (10 pts)
Human memory, formally
believed to rather
inefficient, is now really
more sophisticated than that
of a computer. Researchers
approaches the
problem from a variety of
points of view have all
concluded that there
is a great deal more stores in
our minds than has been
generally supposed.
Dr Wilder, a Canadian
neurosurgeon, proved that
on stimulating their
brains electric, he could
elicit the total recall of
specific events in the
subjects’ lives. Even dreams
and other minor events
15 supposed forgotten
10 for many years suddenly
emerged in detail.
Although the physical basis
for memory is not still
understood, one
theory is that the fantastic
capacity for storage in the
brain is the result of
an almost limited
combination of
interconnections between
brain cells,
stimulated by patterns of
activity. Repeated references
to the same
information supports recall.
Or, to say that another way,
improved
performance is the result of
strength the chemical bonds
in the memory.
2 ._______________ 3._______________
5 ._______________ 6 ._______________
9 .______ ;_________
8. _______________
10.______________
IV. SENTENCE TRANSFORMATION (20 pts)
Rewrite the sentences with the given words or beginning in such a way that
their meanings remain unchanged.
1. It was the goalkeeper that saved the match for US.
-> H ad ._______________________________________________
2. Most people seem to think that I will be next to be promoted. LINE
Most people seem to think that I _____________;_________ a promotion.
3. In the area, Thailand is much better than all other countries in football.
(SHOULDERS)
—>1 n the area, Thailand___________________________________________
4. The children are in disgrace for being so badly behaved.
—> The children_________________________________________________
5. We regret to inform you that your application has not been successful.
—> Much_______________________________________________________
6 .1 left without saying goodbye as I didn’t want to disturb the meeting.
—> Rather _______________________________________ ________
1. Driving at that speed is dangerous whether you are an experienced driver or not.
—*H oweve r_____________________________________________________
8.My protests were ignored by everybody.
—> Nobody______________________________ _______________________
483
9. We’ve agreed to share the cost. AGR EEM ENT
10. What has this experience taught you. DRAWN
TRƯỜNG THPT NGÔ GIA Tự - ĐẮ K LẮK
A. MUL TIPL E CHOIC E. (4 0 PT S)
I. PHONO LOGY (5 PTS)
Choose the word w hose underlined part is pronounced differently from the
others. *
1. A. psychiatrist
2. A. confusedly
3. A. mezzanine
4. A. hazard
B. psychiatric
B. allegedly
B. Sw itzerland
B. bombard
B. pharaoh
c. psychics D.psyching
c. supposedly D.wickedly
c. pizza D.pretzel
c. custard D.mustard
5. A. exhilarate c. diarrhoea D. carbohydrate
Choose the word which is stressed differently from the other three.
6. A. justapose
7. A. stratosphere
8. A. ultraviolet
9. A. legitimate
10. A.advanturous
B.monoculture
B.dictaphone
B. paradoxical
B. extravagant
B. luminous
c. geothermal
c .cornerback
c. correlative
c. septicaemia
c. laborious
□.intermittent
D.hendiadys
□.influential
□.dem onstrative
D. autonomous
II. WORD CHOICE (5 PTS)
Choose the best options to complete the following sentences.
1. I think I understand the nuts and__________ of the operation.
A. crews B. hammer C. bolts D. nail
2. Tom w on't buy that old car because it has too much__________ on i t .
A. ups and downs B. odds and ends c. wear and tear D. white lie
3. Job losses are mainly in the__________ sectors of the industry .
A. blue-collar B. red-collar C. stiff-collar D. high-collar
4. Since he is too old to be a porter, they have decided to put him out to______ .
A. the door B. grounds c. grass D. the kerb
5. It was an extremely hostile article which cast__________ on the conduct of the
entire cabinet.
A. criticism B. aspersions C. disapproval D. abuse
6. Some countries alway s__________ resolutions at the United Nations which
don ’t suit them.
A. bail B. countenance C. veil D. veto
7. I didn’t need any medicine. I’m as right as__________ .
A. clouds B. rays c. rain D. a haze
8. My sunburnt nose made me feel rather__________ for the first few days of the
holiday.
A. self-confident B. self-centre c. self-conscious D. self-evident
484
9. My mother is a real__________ potato. She watches T.v all the time.
A. couch B. sofa c. armchair D. cushion
10. The red cross is __________ an international aid organization.
A. intriguingly B. intrusively c. intrinsically D. intrepidly
III. GRAMMAR AND STRUCTURES (5 PTS)
1 .1 hope everything ’s OK. They__________ several hours ago.
A. would have called B. must have called
c. were to have called D. supposed to call
2. _______ with her boyfriend yesterday, she doesn’t want to answer his
phone call.
A. Having quarreled B. Because having quarreled
c. Because of she quarreled D. Having quarreled
3. Superconductivity will revolutionize the way that energy is used for the next
millennium, and__________ the first truly superconductive substance will be
remembered as a technological hero.
A. what the discovery o f B. the discovery of
c. whoever discovers D. whose discovery
4. The important of the hand, and more generally of the body, in children’s
acquisition o f arithmetic__________
A. can hardly be exaggerated . B. hardly exaggerated can be.
c. can be exaggerated hardly. D. exaggerated can be hardly.
5. Why d idn't you ask for h elp,__________ to do it on your own?
A. in view of trying B. nowhere near trying
c. rather than trying D. far from trying
6. __________ , only people who agree with her are real jews.
A. We can infer how she is concerned
B. Inferred from what she is concerned
c. We can infer what she is concerned
D. We can infer that as far as she is concerned.
7. It is possible to stay overnight here although there is no guest house
A. at that B. as such c. in itself D. for that
8. __________ ,1 haven’t seen Catherine all day.
A. Coming to think o f it B. Coming tó think o f it
c. To come to think of it D. Come to think o f it
9. It turned out that we __________ rushed to the airport as the plane was
delayed by several hours.
A. hadn't B. should have c. musn’t D. needn’t have
10. When Nadine arrived, she soon__________ at all her jokes .
A. had everyone laughed B. had laughing everyone
c. had everyone laughing D. had laughed everyone
IV. PHRASAL VERBS AND PREPOSITIONS. (5 PTS)
1. They a two-month tour of the US with a pally in Washington.
A. set forth B. brought off c. came up with D. kicked off
485
2. I was promised a good job from January this year, but it’s April now and I’m
afraid that they are just me along.
A. cheating B. swindling c. stringing D. bringing
3. Noone can function properly if he or she is ________ -adequate sleep.
A. took away B. deprived of c. deprived from D. got rid o f
4. Susan says she feels less nervous since she_________ on tea and coffee.
A. cut down B. stopped off c. turned back D. cut out
5. To get his proposal accepted, the Finance Manager had to_________ heavy
pressure from colleag ues.
A. fend off B. laugh off c. send off D. push off
6. Time was________ and we were nowhere near finished.
A. cracking on B. bringing down
c. going down with D. letting down
7. The past decade has seen an increasing number of famous people crashing
________ on their celebrity status by getting their autobiographies published .
A. in B. with c. for D. to
8. Good instructors w ill_________ earlysigns o f failure in their students.
A. get through with B. come up with c. think back on D. look out for
9. Two of the students in our class are identical twins and most of the teachers
can’t________
A. see between them B. tell them apart
c see them through D.find them between
10. He really g ets_________ my nerves. He never stops complaining.
A. down B. up c. in D. on
V. READING PASSAGE 1 (5 PTS)
COMMUNICATING WITH THE FUTURE
In the 1980s the United States Department o f Energy was looking for suitable
sites to bury radioactive waste material generated by its nuclear energy
programs. The government was considering burying the dangerous wastes in
deep underground chambers in remote desert areas. The problem, however, was
that nuclear waste remains highly radioactive for thousands of years. The
commission entrusted with tackling the problem of waste disposal was aware
that the dangers posed by radioactive emissions must be communicated to our
descendants of at least 10,000 years hence. So the task became one of finding a
way to tell future societies about the risk posed by these deadly deposits.
Of course, human society in the distant future may be well aware of the
hazards of radiation. Technological advances may one day provide the solutions
to this dilemma. But the belief in constant technological advancement is based
on our perceptions of advances made throughout history and prehistory. We
cannot be sure that society won’t have slipped backward into an age of
barbarism due to any of several catastrophic events, whether the result of nature
such as the onset of a new ice age or perhaps mankind’s failure to solve the
scou rges of war and pollution. In the event of global catastrophe, it is quite
486
possible that humans of the distant future will be on the far side of a broken link
of comm unication and technological understanding.
The problem then becomes how to inform our descendants that they must
avoid areas of potential radioactive seepage given that they may not understand
any currently existing language and may have no historical or cultural memory.
So, any message indicated to future reception and decipherment must be as
universally understandable as possible,
It was soon realized by the specialists assigned the task of devising the
communication system that material in which the message was written might not
physically endure the great lengths of time demanded. The second law of
thermodynamics shows that all material disintegrates over time. Even computers
that might carry the message cannot be expected to endure long enough. Besides,
electricity supplies might not be available in 300 generations. Other media
storage methods were considered and rejected for similar reasons.
The task force under the linguist Thomas Sebeok finally agreed that no
foolproo f way would be found to send a message across so many generations
and have it survive physically and be decipherable by a people with few cultural
similarities to US. Given this restriction, Sebeok suggested the only possible
solution was the formation of a committee of guardians of knowledge. Its task
would be to dedicate itself to maintaining and passing the knowledge of the
whereabouts and dangers of the nuclear waste deposits. This socalled atomic
priesthood would be entrusted with keeping knowledge of this tradition alive
through millennia and developing the tradition into a kind of mythical taboo
forbidding people to tamper in a way with the nuclear waste sites. Only the
initiated atomic priesthood of experts would have the scientific knowledge to
fully understand the danger. Those outside the priesthood would be kept away by
a combination of rituals and legends designed to warn off intruders.
This proposal has been criticized because of the possibility of a break in
continuity of the original message. Furthermore, there is no guarantee that any
warning or sanction passed on for millennia would be obeyed, nor that it could
survive with its original meaning intact. To counterbalance this possibility,
Sebeok’s group proposed a “relay system” in which information is passed on
over relatively short periods of time, just three generations ahead. The message
then to be renewed and redesigned if necessary for the following three
generations and so on over the required time span. In this way information could
be relayed into the future and avoid the possibility of physical degradation.
A second defect is more difficult to dismiss, however. This is the problem of
social exclusiveness brought about through possession of vital knowledge.
Critics point out that the atomic priesthood could use its secret knowledge to
control those who are scientifically ignorant. The establishment of such an
association of insiders holding powerful knowledge not available except in
mythic form to nonmembers would be a dangerous precedent for future social
developments.
487
1. The word ’’ch am be rs ” in the passage is closest in meaning to
A. partitions B. openings c. cavities D. fissures
2. What problem faced the commission assigned to deal with the burial of
nuclear waste?
A. How to reduce the radioactive life of nuclear waste materials
B. How to form a committee that could adequately express various nuclear risks
c. How to notify future generations of the risks of nuclear contamination
D. How to choose burial sites so as to minimize dangers to people.
In paragraph 2, the author explains the possible circumstances of future societies
A. to warn about the possible natural catastrophe
B. to question the value of advances
c. to highlight humankind's inability to resolve problems
D. to demonstrate the reason nuclear hazards must be communicated
4. The word ’’scou rg es” in the passage is closest in meaning to
A. pressures B. afflictions c. worries D. annoyances
5. In paragraph 4, the author mentions the second law of thermody namics
A. to support the view that nuclear was.te will disperse with time
B. to show that knowledge can be sustained over millennia
c. to give the basic scientific reason behind the breakdown o f material objects
D. to contrast the potential life span of knowledge with that of material objects
6. The word "Its” in the passage refers to •
A. knowledge B. committee c. solution D. guardians
7. In paragraph 5, why is the proposed committee of guardians referred to as the
’’ato mic pries thood” ?
A. Because they would be an exclusive group with knowledge about nuclear
waste sites.
B. Because they would use rituals and legends to maintain their exclusiveness
c. Because they would be an exclusive religious order
D. Because they would develop mythical taboos surrounding their traditions
8. According to the author, why did the task force under Sebeok propose a relay
system for passing on information?
A. To show that S ebeok's ideas created more problems than they solved
B. To support the belief that breaks in communication are inevitable over time
c. To contrast Sebeok's ideas with those proposed by his main critics
D. To compensate for the fact that meaning will not stable over long periods
of time
9. According to paragraph 7, the second defect o f the atomic priesthood proposal
is that it could lead to
A. the nonmembers turning knowledge into dangerous mythical forms
B. the possible misuse of exclusive knowledge
c. the establishment o f a scientifically ignorant society
D. the priesthood's criticism of points concerning vital knowledge
488
10. All of the follow ing are mentio ned in the passage as difficulties in d evi sing a
com mu nic ation system with the future EXCEP T
A. the failure to ma intain com mu nicatio n link
B. the loss of know ledge abo ut today's c iviliza tion
c.the inability o f materials to end ure ov er time
D. the e xclus ive nes s of prie sthood
VI. READING PASSAGE 2 (5 PTS)
Read the text below and choose the best answer to each question.
ENERGY FROM THE WAVES
The quest for sustain able sources of energy has led hum ans to stud y the
ene rgy potential of the sun and the wind, as well as the imm ense power crea ted
by dam me d rivers. The oceans, too, represent an impress ive source of potentia l
ene rgy. For exa mp le, it has been estimated that the oceans could provide nearly
3,000 tim es the energy generated by hyd roelectric dam s such as the Hoover
Dam . Yet, this source rem ains quite d ifficult to exploit.
But this cha llen ge has not prevented scientists from trying. Within the last
few deca des , several techno logies that can transform the o cean’s immense forc es
into usa ble electricity have been invented and introduced. Some focus on
cap turing the power of the changing tides, while othe rs rely on therma l energy
created by oce ans in certain tropical regions. How ever, the most com mon and
easiest-to-dev elop techno logies are those designed to harness the powe r inhe rent
in th e o ce an ’s waves.
There are several methods by which ocean-wave energy can be collected. All
of them wo rk because the movem ent of the water that the waves induce crea tes
stor able energy by dire ctly or indirectly driving a power gen erator. In one such
techno logy, the cha nging water levels in the ocean tha t are produc ed by wav es
lift a long floating tube comprised of many sections connected by hinges. As the
sections move up and down with the water, they pump a special fluid through the
tube that can be used to drive a generator. Another technique works on a similar
principle, only the floating obje ct rocks back and forth with the motion of the
water in stead of up and down. A third method of collec ting wave energy relies on
the risin g water from the waves to compress air in a partially submerged cham ber.
As the waves rush into the cham ber, the y push the air out throu gh a narro w tunnel.
Located inside this tunnel is a turbine connected to a pow er generator. The
movem ent o f the a ir turns the turbine, which feeds energy into the generator.
The draw back to each of these concepts is that they make it necessary to have
many pieces of mach inery linked together. This presents a problem because the
larger the device, the more vulnerable it is to damage from hazardou s ocean
environments, and the more likely it is to interfere with otherwise unspoiled coastal
scenery. Also, these methods demand the construction of site- specific machines that
take into consideration average local wave heights and sea conditions. In other
words, the ability to get po wer from waves differs from region to region.
489
Japan, Norway, and the UK have all attempted to generate energy by capturing
the power of.o cean waves. In northern Scotland, the first power plant to use
wave power, OSPREY (Ocean Swell Powered Renewable Energy), began operating
in 1995. It followed the principle of the third method described above: waves
entering a partially submerged chamber pushed air into turbines ; to generate
electricity. The electricity was then transmitted to power collectors on the shore via
underwater cables. Unfortunately, the OSPREY plant was destroyed in a large
storm, highlighting an unavoidable difficulty associated with this kind of power
generation.
The potential benefits of wave-based energy are hard to ignore. Once the
proper machinery is produced and installed, the energy is free. Maintenance
costs are small, and the equipment does not pose any threats of environmental
pollution. And best o f all, the amounts of energy produced are enormous.
However, these theoretical advantages have yet to be fully realized. In many
cases, a lack of government funding has inhibited the technologies from advancing.
For example, despite the relative abundance of proposed wave -pow er devices,
many have not been adequately tested, and most have been evaluated only in
artificial pools where they are not subjected to the harsh marine conditions that
exist in actual oceans. Protecting the equipment from the sea’s destructive
forces, as well as the fundamental task of determining feasible locations for
collecting energy, also present formidable challenges. All in all, while ocean
power offers some intriguing possibilities, the difficulties involved in harnessing
this energy source are substantial and will require more time to overcom e.
1. The phrase this source in the passage refers to
A. sun B. win c. dammed rivers D. oceans
2. The word exploit in the passage is closest in meaning to
A. utilize B. declare c. contain D. determine
3. Why does the author mention the Hoover Dam in paragraph one?
A. To give a current example o f ocean-based energy technology
B. To explain that dams are effective producers of sustainable energy
c. To draw a comparison between two sources of renewable energy
D. To show that alternative energy sources have not been successful
4. In paragraph 2, the author states that
A. waves do not represent the only form of ocean power
B. tropical oceans produce the greatest amount o f energy
c. scientists first attempted to collect power from ocean tides
D. most of the electricity created by oceans is not usable
5. The word induce in the passage is closest in meaning to
A. cause B. define c. order D. monitor
6. According to paragraph three, which of the following is true about wave-power
- technologies?
A. Many o f them use submerged objects to obtain the waves' energy.
B. Compressed air must be present for them to work properly.
490
c. They undertake three steps in order to collect wave power.
D. They rely on the water s motion to create electricity.
7. According to paragraph five, what part did the cables play in OSPREY'S design?
A. They attached the partially submerged chamber to the sea floor.
B. They generated the electricity which was then collected in turbines,
c. They conducted the electricity from the generator to the shore.
D. They provided stability during powerful ocean storms.
8. The word inhibited in the passage is closest in meaning to
A. delivered B. prevented c. protected D. approved
9. What can be inferred from paragraph seven about governments?
A. They do not believe wave-energy devices can withstand ocean forces.
B. Their interests often conflict with those o f the energy industries.
c. They demand much scientific research before they provide funding.
D. Their support is often essential to the success of new endeavors.
10. All of these are problems associated with the collection of wave energy EXCEPT
A. the difficulty o f finding feasible locations.
B. the destructive power of the ocean,
c. the size o f the equipment involved.
D. the constant changing of the tides.
VII. GUIDED CLOZE Í (5 PTS)
Read the text below and decide which answer best fits each space.
ENVIRONMENTAL ISSUES
The issues for (1 )______ ______ economies are a little more straightforward.
The desire to build on undeveloped land is not (2) _________ __ out of
desperation or necessity, but is a result of the relentless march of progress.
Cheap labour and a relatively highly-skilled workforce make these countries
highly competitive and there is a flood of inward investment, particularly from
(3 )____________ looking to take advantage of the low wages before the cost
and standard of living begin to rise. It is factors such as these that are making
many Asian economies extremely attractive when viewed as investment
opportunities at the moment. Similarly, in Africa, the relative (4 )____________
of precious metals and natural resources tends to attract a lot o f (5) ___________
companies and a whole sub-industry develops around and is completely
dependent on this foreign-direct investment. It is understandable that countries
that are the focus of this sort of attention can lose sight of the environmental
implications of large-scale industrial development, and this can have devastating
consequences for the natural world. And it is a (6 )____________ cycle because
the more industrially active a nation becomes, the greater the demand for and
harvesting of natural resources. For some, the environmental issues, though they
can hardly be ignored, are viewed as a (7) • concern. Indeed,
having an environmental conscience or taking environmental matters into
consideration when it comes to decisions on whether or not to build rubber-tree
(8) ____________ or grow biofuel crops would be quite ( 9 ) ____________
491
indeed. For those involved in such schemes it is a pretty black-and-white issue.
And, for vast (1 0 )____________ of land in Latin America, for exam ple, it is
clear that the welfare of the rainforests matters little to local government when
vast sums of money can be made from cultivating the land.
1. A. emerging B. emergent c. convergen t D.res urg ent
2. A. grown B. born c . bred D. arisen
3. A. multinationals B. migrants c. continentals D.intercontin
4. A. premonition B. abundance c. amplitude D. accumula
5. A. exploitation B. exploration c.s urve ya nc e D. research
6. A.v acu ous B. viscous c. vexatious D. vicious
7. A. parallel B. extrinsic c. peripheral D. exponenti
8. A. plantations B. homesteads c. ranches D. holdings
9. A. proscriptive B. prohibitive c. prospective D. imperativ
10. A. regions B. plots c. tracts D. sectors
VIII. GUIDED CLOZE 2 (5 PTS)
Read the text below and decide which answer best fits each space.
MIDSUMMER N IGH T'S EVE
In Europe, Midsummer Night's Eve, also known as St John's Eve, occurs on
June 23rd. It originates from the pagan celebrations of the summ er solstice
which were held on June 21st. On that night throughout Europe bonfires were lit
along hillsides to ( 1)_______ _the shortest night o f the year. It must have looked
as if some kind of violent insurrection was taking place down the coast of
Scotland and England, but these signal fires in fact had a very important purpose.
Bones of farm animals (2 )________ the previous autumn were burned and, when
the fires had (3)________ the remaining ash was put to good use: it was spread
on the fields to enrich the land and ensure a good harvest. The word 'bonfire' is
(4)________ from'bonefire'.
In Brazil too St John's Eve means bonfires and fireworks. Another quaint
tradition involves the (5)________ of small paper hot-air balloons, although they
are prohibited by law in the cities because of the fire (6 )________ . Bonfires
mark the beginning o f spring rather than the summer in Sweden and are lit on the
last night of April. In the Swedish Midsummer's Eve (7)________ , held on June
24th, a large pole, decorated with flowers and leaves, is placed in the ground.
Thistles also have a significant role in the celebration of Midsummer's Night in
Europe. In the past they were thought to (8 )________ witches. The pretty,
prickly plant was nailed over barn doors and used in wreaths, the circular shape
being a symbol of the turning of the seasons. Wheels faced with straw and
soaked in pitch were lit from the bonfires and then rolled down hills.
There is less risk of fire in a (9)________ tradition to many Slavic countries.
Young women and girls float little baskets of flowers and lighted candles down
streams. Local boys swim out to (10)________ a basket, find the girl it belongs
to and claim a dance at the town's Midsummer's Eve Party.
492
c. commemorate D.
1. A. celebrate B.honour
commiserate
2. A. revised B. assassinated c. slaughtered D. sacrificed
3. A. doused B. extinguished C. smothered D. gone out
4. A. derived B. developed C. evolved D.d eca yed
5. A landing B. launching c. propelling D. ejecting
6. A. certainty B. peril C. jeopardy D. hazard
7. A. tradition B. custom C. ceremony D. practice
8. A. deflect B. ward off C. attract D. avert
9. A. unique B. common C. mutual D. prevalent
10. A. salvage B.rescue C. set free D. liberate
B. WRITTEN TEST (70 PTS)
I. CLOZE TEST: Read the text below and think of the word which bejst fits
each space. Use only ONE WORD for each space.
OPEN CLOZE 1. (10 PTS)
After living (1 )___________ the threat of extinction for more than 30 years,
the national bird of the United States has been granted an official reprieve, as the
bald eagle and twenty- eight other animal and plant species have been earmarked
(2 )___________ removal from America’s list of endangered species. The bald
eagle, also known as the white-headed sea eagle, took pride o f place at the top o f
a list of species likely to (3 )___________ taken o ff the endangered register in the
coming years. The proposed “delistings” are being promoted by the US interior,
secretary to counter a growing feeling among Republicans that endangered
species laws do not work. Charges of ineffectiveness have been (4 )__________
against these laws before, but more recently it has even been suggested that the
situation may actually have been (5 )_______, worse by them. The recovery
of the bald eagle follows thirty-one years on the critical list. Its numbers had
been (6 )___________ to fewer than five hundred through the use of pesticides
that reacted adversely (7 )___________ its reproductive system. The number of
nesting pairs is now estimated at five thousand. The interior secretary claims that the
new list was a vindication of the legislation under which the eagle, a national symbol
(8) ___________ originally from the Indians, and more than a thousand other
species were protected.(A spokesperson denied that it was simply a publicity stunt
timed to coincide with the swearing (9 )______ of the president for his second
term). Until now, few species have (1 0) __________been removed from the list.
When they were, it was usually because they had become extinct.
OPEN CLOZE 2. (10 PTS)
Advances in technology have done much to change the way we live and
communicate, but no breakthrough revolutionized life as much as the advent of
wireless technology, according to a panel of experts assembled by CNN 2010.
Back in 1980, when it (1)__________ to communication technology, telephones
493
were stuck to walls, facts were found in books, and people had to browse shelves
in a record store if they wanted to buy the latest music. However, developments
that occurred at the start of the twenty century (2 )__________ we could access
all that information just by reaching into our pockets for our mobile phone. Form
the (3 )__________ beginning of the Information Age, having immediate access
to data was fundamental to progress. In some cases, technological advances
(4 )__________ to overhaul industries at a lightning - fa st pace. For example, the
internet helped tech-savvy music lovers to force changes in the way the music
industry did ( 5 ) __________ , while text messaging over cell phones made
written (6 )__________ more immediate.
We embraced information technology to such an (7 )_______ that having
computer access in our homes was not (8 )__________ enough. We wanted to
use these tools everyw here. Wireless hot spots popped up in cafes, airports,
universities - even the whole city blocks’. And ju st as comp uter use worldwide
exploded, the brains behind them - the processors - became increasingly
smaller. A computer chip that you could balance on your fingertips could hold
massive (9 )__________ of data. This ability to store portable. The entertainment
industry harnessed satellite technology to give consumers more choice, and
video game technology developed to the (1 0)__________ where it became a $
10 billion a year industry, producing games that invite players into a virtual
world of fantasy, sport or even war.
IL WORD FORMATION (20 PTS)
PART 1:
Complete each sentence, using the correct form of the word in parentheses.
1. ______________________of East and West Germany began in 1900 when the
Berlin Wall came down. (UNITY)
2. The distribution of nonideal fish communities can be roughly canculated by
th e_____________________ of suitable variables. (METER)
3. The boy was very violent and his parents found him_____________________
(MANAGE)
4. These remote areas are still _________________ to the Internet. (ACCESS)
5.If the cabin 2_______ , oxygen masks will automatically drop down. (PRESS)
6. “ Help!” she cried, waving her ar ms_____________________ in an attempt to
attract som eone’s attention. (DISTRACT)
7. He was fined $ 500 a n d ________________ from driving for three years.
(QUALIFY)
8. I could not get much work done as I w as ___________• being interrupted
by people telephoning me. (CONTINUE)
9. My friends started going out late to nightclubs so I decided to _ _________
myself from the group. (SOCIAL) ,
10. His workmates were very__________ _______and selfish. (COOPERATE)
494
PART 2:
Complete the passage with appropriate forms of the words given in the box.
PUTTING SOME FUN 1. PREFER
INTO THE WORKPLACE 2 LOYAL
A study of 737 chief 3. LIKELY
executives of major 4. BENEFIT
corporations found
that 98 per cent would hire
someone with a good sense
of
humor in (1) to someone
who seemed to lack one.
Having fun at work also
inspires (2) in
employees. According to a
survey of 1,000 workers,
those
who rated their manager's
sense of humor “above
average”
said there was a 90 per cent
probability that they would
remain in their job for more
than a year. If they worked
for a
boss whose sense of humor
they described as 'average'
or 5 APPLY
'below', the emplovee's (3) 6. NECESSARY
of staying dropped 7. COMEDY
to 77 per cent. 8. FORTUNE
So laughter is (4) and good 9. CONTINUE
for business. This 10. SATISFY
knowledge could, however,
add to the stress of (5)
when they are interviewed if
making joke s doesn't come
naturally to them. But being
funny doesn't (6)
mean being a stand-up (7) .
The important
thing is to strive for a light-
hearted atmosphere in the
workplace.
There is, (8) , a downside to
all this. For instance,
some people working in
retail jobs are required to
smile
(9) . Such enforced
happiness can apparently
cause
(10) at work and result in
emotional stress.
III. ERR OR CORRECTION (10 PTS)
The following passage contains 10 errors. Identify and correct them.
15 The destruction of the
10 rainforests is a pressing
problem of our times but
not one that is regarded
equally serious by everyone.
The more affluent
nations regard the issue as
one of preservation;
deforestation must stop.
When it comes.to the poorer
countries, the issue is not so
cut and dried.
For these people, the
rainforests represent a
source of economic
prosperity, a
point that obviously takes
precedence on ecological
concerns. A solution
must be found before the
damage caused by the
deforestation that is
destroying the rainforests
becoming irrevocable.
Deforestation is carried
out by those involving in the
timber industry and also by
migrant
fanners. The later occupy an
area o f land, strip it, farm it
until its natural
mineral supply is used up
and then move on. The land
is left useless and
exposed and a process of
erosion comes into effect,
washing soil into rivers
thereby killing fish and
blocking the water's natural
course. The land is not
the only victim. Rainforests
are a rich populated habitat.
In the rainforests
495
15 of Madagascar there are at
20 most 150,000 individual
species of plants and
animals which are found
anywhere else in the world
and more are being
discovered all the time.
Furthermore, approximately
50% of all endangered
animal species live in the
world's rainforests. The
destruction of the forests
effectively represents
acomplete removal of all
these plants and animals.
Deprived of their natural
environments, they will
disappear altogether.
Again, this process is
reversible. Man, no matter
how powerful he considers
himself, does not have the
power to establish the
species he is so willfully
destroying.
1.________________
4 ._____________ __
7 .________________
2 .________________
5 ._______________
8 ._______________
3 .________________
6.________________
9. ________________
10.
IV. S ENTE NCE TR ANSFORMATION. (20PTS)
Rewrite the following sentences using the words given.
1 .1 only realized the full implications of what had happened until sometime later.
-» It was n’t ............................................................................................................
2. The judge said that it was only because of the wom an's age he had not sent
her to jail.
—» The judge said that h a d ........................... ........... ...........................................
3. The last time there was such an environmental catastrophe was a thousand
years ago.
-> N ot .............................................................................................. proportions.
4. We we ren 't surprised by his success.
-> It ca m e.................................................. ............. .............................................
5. He has an obsession about the dishonesty of lawyers. (BEE)
-> He.......................................................... ;............... .................................... .....
6. I must have offended her because she isn’t being friendly towards me.
(SHOULDER)
-» I ....... ...............................................................................................................
7. You need to make sure that he has enough money to pay for the car before you
sell it to him.
—> You need to see.......................................................................... .....................
8. He threatened the officers with violence. (THREATS)
-> H e ............................,............... ............................................. ..........................
9. The students’ rebellious behaviours should have been severely punishment.
(DESERVED)
-» The stud en ts.......................................................................... :.......................
10. There's no one available in this moment to take her class. (NOTICE)
—> There ’s ............... .......... ....................................................................................
496
P h ần I t
ĐÁP ÁN ĐỀ THI OLYMPIC TRUYỀN T Hố NQ 3 0 /4
LẦN THỨ XXIV - NĂM 2 0 1 8
lói* ìoỊ
ĐỀ CHÍ NH THỨC
A. MULTIPLE CHOICE.
l.c 2. B 3. B 4. c
11.D 12. A 13 .B 14. A
21. B 22. B 23. D 24. B
31. c 32. B 33. D 34. A
41 B 42. D 43. B 44. D
5 1 .c 52 D 53. A 54. c
61 B 62. c 63. c 64. D
71 B 72 A 73. B 74. D
B. WRITTEN TEST
I. 1. about 2. yourself
5. then 6. distances
9. keep / stay 10. not
3. provide I supply
7. submarine
11. ability
13. discharges 14. vulner ability/susceptibility
16. in 17. degradation 18. seasonal
4. impression
8. that
12. threats
15. overuses
19. more 20. m eet
II.
1. uninitiated 2. unequivocal
6. enm eshed ■ 7. bilabial
10. self-righteous
13. licensed 14. Unabridged
17. allegedly 18. subsequent
III.
3. misgivings 4. escapism
8. counter-intuitively
11. pseudonym
15. companion
19. ill-health
12. lifelong
16. incorporated
20. re-gaining
5. hungover
9. assimilated
LINE MISTAKE CORRECTION
most people / almost all
2 almost people
people
3 live alive
4 succumb for succumb to
8 infinitely countless countless
8 provide providing
10 arranging arranged
12 theirs its
13 which that / this
15 cells the cells
15 but for but rather
497
IV. 1. Such wa s her nervousness that s he came o ver as a bashful girl.
2. The instant the f unds pet ered out, the y had to abandon the s chem e.
3. I'd ra ther/so on er yo u ha dn ’t b rea the d a w ord about my plan to Ka thy .
4. Scarcely had his f athe r retired wh en David too k charge / wa s put in
ch ar ge o f the fam ily business.
5. It was y ou r encour ag em en t tha t h elped (to) c ush ion the blo w after suc h a
heavy loss.
6. Had it not b een for her craz e for stam ps, T ina w ou ld n’t h ave splas hed out
on the m every month .
7. Try as he might, he co uldn ’t m ake am ends for w ha t...
8. All dogs are tho ught to hav e been desce nded from w olve s by / a ccording
to experts.
9. Not until later did w e lear n he (still) eked out a (m ise rable ) living / an
existence as a w ait er.
10. It’s still touch and go wh ether the p roject will be given / w ill get the goahead or not.
TRƯỞNG THPT CHUYÊN HUỲNH MẦN DẠT - KIÊN GIANG
A. MUL TIPL E CH OI CE (4 0 P T S)
I. l. D 2. c 3. B 4. D 5. c 6. A 7. B 8. c 9. B 10. D
n . l . c 2. D 3 c 4 B 5.C 6. D 7. A 8. B 9. c 10. B
9. A ' 10.
III. 1. C 2. D 3 D 4. C 5 c 6. B 7. A 8. D
B
IV. 1.A 2. c 3. D 4. D 5 A 6. A 7. c 8. B 9. B 10. A
V. READING
PASSAGE 1 (5 pts) 6. A 7. A 8. c 9. D 10. A
l. D 2. c 3. c 4. B 5.B
READING PASSAGE 2
(5 pts) 6. c . 7. D 8. D 9. B 10. A
l . c 2. D 3. A 4.C 5. c
VI. GUIDED CLOZE
TEST 1 (5 pts) 6. D 7. c 8. c 9. A 10. B
l. D 2. D 3. c 4. A 5. c
GUIDED CLOZE TEST
2 (5 pts) 6. c 7. B 8. D 9. A 10. D
l. D 2. B 3. A 4. c 5. B
B. W RI TT EN T E ST
(70 pts)
I. OPEN CLOZE TEST
4. go 5. themselves
1 (10 pts)
1. néver/rarely 2. to 3.
once
6. singing 7. take 8. at 9. But 10. showing
OPEN CLOZE TEST 2
(10 pts) 3. business 4.
I. redundant 2. premises
rare/precious/unique/..
5. self-em ployed 6.
7. loan 8. specialize
aspects/matters/factors/...
9. payment 10. honest/kind/reliable/...
498
II. 1.
1. expressionless
5. self-evaluation
9. imperceptibly
2.
1. recourse 2. reality
6. multitude 7. recession
I II . 1. Line 3: the —> this
3. Line 5: kinds -» sorts
5. Line 7: into - » in
2. typecast 3. requisitioned
6. confidentiality 7. editorials
10. triumphant
4. outlaw
8. multi-facetted
3. alternative 4. miserable
8. like-minded 9. residence ,
2. Line 4: serious -> seriously
4. Line 5: treatment —> treats
6. Line 8: sweetened —> sweet
5. surrounds
-10. slouching
7. Line 10: away —> down
8. Line 11: a big businesses -> a big business I big businesses
9. Line 13: long -> far 10. Line 15: crafting -» crafts
IV . 1. “In the light o f this new evidence I have no alternativ e but to release you,”
the judg e told the accused.
2. It is important to know how to draw the line between a joke and a lie.
3. The government favours a carrot and stick approach/policy to young offenders.
4. W e’ll have this job done in the next to no time if we cooperate.
5. He was gre en with envy to see his broth er’s new car.
6. Sad as/ though it is, unemployment is unlikely to go down this year.
7. Paper is believed to have been invented by the Chinese in 105 A.D.
8. I’d rather you hadn’t written that letter.
9. Rather than take the thing back to the shop, he decided to repair it himself.
10. It can’t have been Mrs. Elton you saw because she’s in Bristol.
TRƯỜNG THPT CHUYÊN LÊ HỒNG PHONG - TP. Hổ CHÍ MINH
A. MULTIPLE CHOICE:
I. l.B 2. c 3. A 4. A 5. D
II. 1. A 2. A 3.C 4. D 5.C
I I I . 1 .D 2 . c 3. A 4. C 5.D
IV . l. D 2. A 3. D 4. B 5. D
V. READING COMPREHENSION 1:
LA 2. B 3. A 4. B 5.D
READING 6.C 7.B 8.A
COMPREHENSION 2
l. D 2. B 3. C 4. A
5.C 6. B 7. A
V I. GUIDED CLOZE 1:
L A 2. A 3. A 4.C
5. D 6. A 7. B
GUIDED CLOZE 2:
l. D 2. c 3. A 4. B 5. D 6. B 7.C
499
B. WRITTEN TEST
I. OPEN CLOZE 1:
4. from 5. mistaking
l.ta ke 2. it 3. look
6. com parison 7. but 8. after 9. long 10. lenses
OPEN CLOZE 2
4. designs 5. winning
l. o ff 2. Between 3. after
6. from 7. describing 8.
9 .layers 10. with
moment
II. WORD FORMATION 1:
1. veh icula r 2. upended
5. decolonization
9. cro wd-pleaser
6. ex-directory
10. interspecies
3. anti-depressants. 4. housebound
7. afield 8. misnomer
WORD FORMATION 2:
1. provision/backup 2
3. decidedly 4
7. metacognition
8
III.
backup/provision
know-how 5. exceptionally
evaluating 9. over-direct
6. self-regulatory
10. spoon-feeding
Line Correction Line Correction
4 viewpoints —> views 13 so did ... —»
5 , reasons —> reason 14 opposite —>
syllabus
7 Including —> Included 18
curriculum/c
9 the later —> the latter 18 bom into —»
11 contributed —» attributed 20 slightly -> lig
IV. SENTENCE TRANSFORMATION:
1. The school pre stig e___ is believed to have increased beyond measure in the
last few decades.
2. It is advised th at __he (should) return home by 11 o ’clock at the latest.
3. Had Jim my___ not reacted so aggressively on impulse, his wife would not
have felt heartbroken.
4. I f__ only Tony hadn’t poured scorn on his friend’s business plan.
5. N o t__ until the lady moved forward into the light did it dawn on Harry
who she was.
6. From start to finish, the whole affair has been a chapter of
accidents______________
7. You should__ not have viewed what happened in negative___ light.
—>You should have viewed what happened in positive light.
8. No matter what happens, you should keep your chin up.
9. It is still in ____the balance whether the scheme will get / be given the
green lig ht ___or not.
10. Gi ven___ his medical history, this patient’s condition is rather worrying.
500
TRƯỜNG THPT CHUYÊ N LÊ KHIẾ T - QUẢNG NGÃI
A. MULTIPLE CHOICE
I. L A 2. A 3. c 4.A 5 .C 6. B
II. 11. B 12. B 13. c 14. B 15. A 16. B
III. 21 .C 22 . B2 3. A 24. c 25. D 26. A
IV. 31. A32 .D 33 . A
34. B 35.A 36. B
V. READING 1:
41. B 42. G 43. E
44. F 45. H 46. c
READING 2.
51. D 52. B 53. D
54.A 55. c 56. c
VI. CLOSE TEST 1:
61. D 62. B 63. D
64. c 65. A 66. B
CLOZE TEST 2:
71. B 72. D 73. A 74. c 75. B 76.D
B. WRITTEN TEST
I. OPEN CLOZE TEST 1:
81. bumed/burnt 82. with 83. contain
86. lost 87. o f 88. left
OPEN CLOZE TEST 2
91. each 92. background 93. although
96. original 97. do 98. risk
II. SE CT ION 1:
102. extraordinary 101.influential
106.containerized 105. elusive

109. unbecoming
110. vicissitudes
SECTION 2:
84. estimated
89. frame
94. influence
99. whatever
85. currently
90. save
95. took
100. for/ with
103. relationships 104. apparently
107. sidestepped 108. telegenic
112. commentator
116. increasingly
120.organisational
113. differs 114. descriptive
117. opinionated. 118. impartiality
122. Line 4. principle —> principal
124. Line 6. militancy —> militant
126. Line 10. glamou r -> glamorous
128. Line 12. were -> are
111. invariably
115. observations
119. unpredictable
III . 121. Line 2. equally -> equal
123. Line 5. super superior
125. Line 7. sexy —> sexist
127. Line 1 1. exploit -> exploitation
129. Line 13. male-dominate —> male-dominated
130. Line 13. moreover -> however
IV. 131. Thomas was kept in the dark about the company’s new projects.
132. Thailand is/ stands head and shoulders above all other countries in football
133. These problems o f ours are ju st the tip of the iceberg.
501
134. The final version o f tile plan bore/ had no/ little resemblan ce to the final draft.
Or: There was no (little/ not) much resemblance between the final version of
the plan and the initial draft.
135. The film d idn 't live up to my expectations at all.
136. You must take step s to make sure/ ensure (that) this d oesn’t happen again.
137. Much as I admire his courage, I think he is a foolish.
138. He gives an impression that he has spent all his life abroad.
139. The way he has recently behaved are out o f the ordinary.
140. If I had taken/ accepted the job 1 was offered in January, 1 would be on
twice the salary la m now/ J would he earning twice as m uch as I am now.
TRƯỜNG THPT CHUYÊN LÊ QUÝ ĐÔN - ĐÀ NẮNG
A. MULTIPLE CHOICE
(40 points)
I. 1. B 2. A 3. B 4. c 5. c 6. c
I I .l . A 2. c 3. D 4. A 5. A 6. B
HL 1. A 2. D 3. c 4. A 5.C 6. c
IV. l.D 2. B 3 . A 4. A 5. c 6. A
V. PASSAGE 1 (5 points)
L B 2. A 3.C 4. A 5. B 6. B 7. D
PASSAGE 2 (5 points)
5. A 6.C 7. D
1.B 2. c 3.B 4. A
VI . CLOZE TEST 1 (5
points) 5. B 6. D 7. A
l. B 2. A 3. A 4. B
CLOZE TEST 2 (5 points)
5. A 6. B 7.C
1.D 2. c 3. B 4. D
B. WRITTEN TEST (70 points)
I. OPEN CLOZE TEST 1 (10 points)
1. as 2. every 3. whether 4. saying
6. only 7. Between 8. Neither/Nor 9. not
OPEN CLOZE TEST 2 (10 points)
1. for 2. keep 3. extinction 4. identified
6. divides 7. predict/foresee 8. given 9. necessarily
5. anything
10. Were
5. perform
10. rather
II. PART 1 (1 0p oin ts)
1. flawless 2. defamation 3. daybreak 4. rarities 5.escapism
6. outright 7. indestructible 8. electrification 9. commentator 10. impressionable
PART 2 (10 points)
1. unrelated 2. historians 3. philosophical 4. undertaken
5. revolutionary 6. replacement 7. innovator(s) 8. fruitful
9. strengthened 10. Reliance
502
II I. 1. Line 1: visiting -> visits
3. Line 4: for -> to
5. Line 7: with —» for
7. Line 11: excluding —> excluded
9. Line 13: has —> have
2. Line 2: better -» rather
4. Line 5: staying -» stay
6. Line 8: founded —» found
8. Line 12: frightening -» frightened
10. Line 14: positively -> positive
IV. 1. So strong was Sam's craze/desire/craving/... for stamps (1) I that he
splashed out on them every month.
2. Kathy comes over/across (1) I as an honest person.
3. There is no telling how long it will take the business (1) I to be/become
lucrative.
4. She was torn between (1) I staying up and turning in.
5. His argument had nothing to do with the case being discussed.
6. The father was as pleased as punch (1) I on/upon the birth of his son.
7. I'm not averse to (1) I putting you up at my house tonight.
8. In light of Tuan's determination to get the job (1) 11 knew he means business
with regard/respect to/ when it comes to his future.
9. You are at liberty (1) I to leave as you wish.
10. The young lady was beside herself with anger/rage (1) I that no one made a
clean breast of it and owned up to having broken her vase.
TRƯỞNG THPT CHUYÊN LÊ QUÝ ĐÔN - KHÁNH HÓA
A. MULTIPLE
CHOICE (40 PTS)
QUESTIONS
I. l. A 2. A 3. A 4.
6. A 7. D 8. A 9. B 10. D
A 5. D
II. l . c 2. c 3. B 4.
6. c 7.D 8. B 9. A 10. A
D 5. B
III. l. D 2. c 3. A
6. c 7. D 8. B 9.D 10. c
4. B 5. c
IV. 1. A 2. B 3. D
6.C 7. D 8. A 9. D 10. D
4. A 5. B
V. READING
PASSAGE 1 (5
PTS) 6.C 7. D 8. D 9. c 10. C
LA 2. B 3. D 4. D
5. c
READING
PASSAGE 2 (5
PTS) 6. A 7. B 8. c 9. B 10. A
l. B 2. c 3.B 4. B 5.
A
VI. GUIDED 6. D 7. B 8. A 9. D 10. B
CLOZE TEST 1
(5 PTS)
l. D 2.C 3.C 4.B 5.
A
GUIDED CLOZE
TEST 2 (5 PTS)
6. A 7. C 8. B 9. A 10. D
LB 2. D 3.C 4. B
5. D
B. WRITTEN
TEST (70 PTS)
I. OPEN CLOZE 3. of 4. on 5. to
TEST 1 (10 PTS)
1. along 2. being
6. another 7.
8. enough 9. this 10. and
because

OPEN CLOZE TEST 2 (10 PTS)


l.th e 2. it/this 3. with 4. has 5. the
6. is 7. to 8. one 9. that/which 10. for
II. PART 1.
1. self-deprecating 2. well-rounded 3. dissociate 4. assimilated
5. belittled 6. inflammation 7. infinitesimal 8. artefacts / artifacts
9. disinfectant(s) 10. proofreading
PART 2.
1. Intergovernmental 2. reappraise 3. projections 4. unequivocally
5. cattle-raising / cattle-
6. decomposing 7. livelihoods
rearing
8. unanimity 9. emissions 10. reaffirmed
III.
1. Line 1: strong —> 6. Line 13: build ->
strength buildings
2. Line 3: appeal —>
7. Line 17: age —» aged
appealing
8. Line 22: product ->
3. Line 5: with —> by
producers
4. Line 7: off —» of 9. Line 25: resist resistance
5. Line 11: especial —>
10. Line 27: another -> other
especially
IV. PART 1:
1. No sooner had Sandy won the race than she began training fo r the next one.
2. It's all the toing and froin g with all the cases that I can't stand about holidays.
3. Due to the bad weather some international flights a re/ will be subject to
possible delay.
4. Bill has turned over a new leaf since he came out o f prison.
5. Lengthy as /th ou gh their discussion wa s/m ight be, the committee couldn 7
reach any / a decision.
PART 2:
1. Watchfully standing behind the president were his bodyguards.
2. The more qualifications you are able to amass, the more success you will
have in the academic field.
3. What most irritates me about him is his clothes.
4. But for his father's early retirement Richard would not have taken over the
fam ily business.
5. Through nofault o f her own, Melanie ended up breaking the law.
TRƯỞNG THPT CHUYÊN LÊ QUỶ ĐÔN - NIN H THUẬN
A. M U L T IP L E C H O IC E (80 sentences / 40 marks)
I. l.D .2. B 3. A 4. c 5. A 6. A 7. D 8.D 9. B 10. A
II. 11. B 12. B 13. A 14. D 15. B 16. B 17. c 18. A 19. A 20. c
504
III. 2 1 .C 2 2 . C 23. A 24. A 25. c 26. D
IV .3 1.B 32. C 33. B 34. c 35. D 36. B
V. READING 1:
44. B 45. c 46. B
41. c 42 A 43. c
READING 2:
54. c 55. D 56. D
51. B 52. c 53. A
VI. CLOZE TEST 1
64. B 65.C 66. A
61. c 62. A 63. B
CLOZE TEST 2:
74. A 75.C . 76. B
71. B 72. A 73. D
B. WRITTEN TEST (60 sentences 170 marks)
I. CLOZE TEST 1
I. everything
6. by
CLOZE TEST 2
II . stress
16. themselves
2. Yet/But
7. Anoth er
12. in / during
17. at
3. without
8. and
13. Even
18. how
4. in
9. this/that
14. instead
19. suffer
5. What
10. could
15. who I that
20. the
II. A.
1. disenfranchisem ent
5. anti-inflationary
9. dismembered
B.
11. conference 12.
15. outside 16.
18. self-conscious 19.
loneliness
conditioning
thought
2. collaborative
6. precancerous
10. runaway
3. hard-luck 4. demystify
7. heavenwards 8. In-service
13. difficulty/difficulties 14. persistent
17. pre-disposition
20. partly
III.
line Mi stake cor rection line
1 Tell say 7
2 Pay paying 8
4 With without 9
5 Founded found 9
7 Children children’s 11
IV. 1. contribution(s) w as/were of great import to
2. as a terrible blow to
3. there could still be plenty o f room for improvement with regard/respect
4. desperate attempt/effort/bid to make amends for
5. heart/sights was/were set on prolonging our stay(,) so thrill were
6. The annual rainfall in/for the North West of Britain is greater/higher than that
in the southeast.
505
7. The excuse for the declaration o f the war is the defence of their territorial rights.
8. It's a foregone conclusion that Mansell will win this year 's championship.
9. This house is a far cry from the little flat we used to live in.
10. Such was the anger of the spectators that they had to cancel the football match.
TRƯỞNG THPT CHUYÊN LÊ QUÝ ĐÔN - BÀ R ỊA VŨNG TÀU
A. MULTIPLE CHOICE
I. l. D 2. A 3.C 4. C 5.C 6. A
II. l l . c 12. c 13. B 14. D 15. A 16. B
III. 21. D 22. A 23. A 24. c 25. B 26. D
IV. 31. A 32. C 33. D 34. A 35. B 36. B
V. GUIDED CLOZE 1
1. D original 2. B exhausted
6. c ravages 7. B suffer
GUIDED CLOZE 2
3. A sufficient 4. c equivalent 5. B basis
8. D thought 9. A tried 10. helps
l. c. 2. D. 3. A. 4. D. 5. A. 6. A. 7. c . 8. c 9. A. 10. c .
VI. PASSAGE 1
I. True 2. True
6. rain 7. (huge) sand (dunes)
II . 12. 1 3 .B /C /A
PASSAGE 2
3. Not Given 4. False
8. climate 9. AD 650
5. False
10. fortress
l. c 2. A 3. D 4. c 5. B 6. B 7. D
B. WRITTEN TEST
I. OPEN CLOZE 1
1. flavoured / mixed
5. they/ people
9. however I though
OPEN CLOZE 2
1. more 2. is
2. whic h/that/they 3. has/res ults/figures 4. since
6. in/ during 7. until/ till 8. even
10. in
3 .turn
6. of
II. PART 1:
7. although 8. readership
4. edition
9. have
5 .conveys
10. rest
1. understudy
5. self-defeating
9. dissociated
PART 2.
1. institutional
5. informed
9. interstate
2. write-off
6. non-essentials
10. disambiguate
2. lobbying
6. antigovernment 7. weakens
10. pickings
3. disaffected
7. imperfection
3. decimated
4. fact-finding
8. betterment
4. legislation
8. capitalism
506
III. 1. Line 3: much —> more
3. Line 5: has been —> which has been
5. Line 9: A way —> This/ The way
7. Line 12: so - » so that
9. Line 16: free-class —> class-free
2. Line 4: other people - » other people's
4. line 7: although -> because
6. Line 11: character -> characteristic
8. Line 13: voiced -> not voiced
10. Line 17: have told -> have been told
IV. 1. The accused showed no remorse for the crime he had committed.
2. Eating seafood brings my brother out in a terrible crash.
3. Journalists are expected to put in long.
4. Little did he realise / know what lay in store for / what was in store for
him when he walked into that room.
5.1 would NOT HAVE MISSED OUT ON such an experience for all the ’.V' dd.
6. The thieves MUST HAVE BEEN SEEN MAKING OFF withThe jewels.
7. Mary talked to the children ONE BY ONE / ONE AT A TIME IN HER/
AN attempt to discover how the disagreement had started.
8. Please DON’T TAKE MINNIE’S COMMENTS AMISS because she meant well.
9. These days, we ARE SPOILT FOR CHOICE when it comes to ethnic cuisine.
10. Discovering that his chicken was underdone PUT RAY IN A QUANDARY
- should he tell his host or not?
TRƯỜNG THP T CHUYÊN NGUYỄN DU - Đ ắK L ẮK
I. l . c 2. D 3.D 4. B 5. C 6. D 7. B 8. A 9.C 10. A
II. l.A 2. c 3. A 4. A 5. A 6. B 7. c 8. c 9. c 10. A
III. 1. A 2. D 3. B 4. A 5. D 6. c 7. c 8. B 9. B 10. D
rv i. A 2. D 3. B 4. D 5. D 6. c 7. B 8. B 9. B 10. A
V. PASSAGE
1
l . c 2. B 3. A 4. A 5. D 6. B •7. c 8. A 9. B 10. B
PASSAGE 2:
l. ix 7. physical chemistry
2. ii 8. thermodynamics
3. vii 9. adapt
4. i 10. immortality
(7 and 8 can be in either
5. viii
order)
6. iv
VI. PASSAGE 1
l . c 2. A 3.B 4. D 5. B 6. c 7. D 8. B 9. c 10. A
PASSAGE 2
L D 2. B 3. A 4.D 5. B 6. B 7. c 8. D 9. c 10. A
507
B. WRITT EN TE ST (70pts)
I. CLOZE TEST 1 (lOpts)
l.w lio 2. to 3. having
6. what 7. a 8. it
CLOZE TEST 2 (lOpts)
1. using/ analysing 2. came
4. laced/ tackled/ considered/ solved
4. o f
9. including
5. due
10. that
6. pay 7. nor
II. PART 1:
1. accreditation
5.cyber-cafe
9.counter-factuals
PA RT 2:
1. outbursts
5. respectful
8. make
3. extracts/ sounds/ utterances
5. those
9. switch
2. endearingly 3. inflammatory
6. self-evaluation 7. deformed
10. eventuality
10. being
4. insolvent
8. expressionless
2. offence 3. constructively
6. intolerant 7. irritable
9. quick-tempered/ hot-tempered/bad-tempered
III.
4. composure
8. background
10. disorganized
Lines Mi stakes Correctio n Lin es Mistakes Co rre ction
0. (1.0) Up 0
1.(1. 3) sport sports/sporting 6. (1. 8) for at
2. (1.3) Beside Besides 7. (1. 8) neither either
couch-
3. (1. 5) and to 8.(1. 11) couch-potato
potatoes
4. (1.5) film films 9. (1. 13) his their
5. (1.7) them those 10.(1. 13) of 0
IV. 1. 1 lacked the courage to tell him what 1 really thought.
2. George is widely assumed to have been wrongly accused.
3. He let it be known that he disapproved of the behaviour of some of his supporters.
4. Not a great deal of damage was done/caused to my car by the collision.
5. Despite a 8-year-old difference they were good friends.
6. Can 1take it as read that you are coming to the party Miriam?
7. After a long hard journey, my spirits ros e/w ere rai sed /lifted up whe n. I
caught sig ht of my home.
8. Yvonne we nt to g rea t leng ths to ensure the success.
9. Russ wa s at odds with his fellow wo rkers over/co ncernin g the new
management policies.
10. I did every thi ng w ithin my power to pre ven t this problem from arising.
TRƯỞNG THPT CHUYÊN NGUYỄN THỊ MINH KHAI - s ú c TRĂNG
A. MU LT IPL E CHOIC E (40 points)
I . l . c 2. A 3. D 4. c 5. A 6. D
II. ll.B 12. A 13. c 14. A 15. B 16. c
III. 21.D 22. A 23. D 24. D 25. B 26. c
508
IV. 31. c 32. B 33. A 34. D 35. B 36. A
V. 41 .C 42. A 43. B 44. D 45.C 46. c
51. A 52. D 53. c 54 D 55. B 56 B
VI. 61. c 62. A63. D 64. D 65. c 66 A
71 c 72 B 73 B 74 A 75 c 76 c
B. WRITTEN QUESTIONS (70 points)
3.
I. 1. would 2. prone
Ability/physique/agility
4. would 5. normally/ usually 6. capable
7. Rare/unusual 8. every 9. Then/again 10. rule
11. had 12. stolen 13 .had 1 4. been 15. disappeared
16. others 17. taken 18 .happens 19. most 20. by
II. 21. explanation 22. definition 23. assumption 24. Logically
25. reasonable 26. thought 27. confusion 28. concept
29. undoubtedly 30. sensible
31. novelist 32. supplements 33. subtitles 34. manifesto
3 5 .handbook 36. columnist 37. critic 38. reviewer
39. tabloid 40. ghostwriters
III.
line mistake correction line
41. 1 it's seeming it seems
42. 3 I'm thinking I think
43. 7 are being are
44. 9 sounds sound like
45. 12 angrily angry
IV. 51. vociferous opponent
52. gauge (public/people's) reaction
53. passionate entreaty
54. regular updates
55. Having introduced my guests to each other, I made a long speech on the
current changes in the computer technologies.
56. George won’t lend his tape recorder to you unless you promise to bring it
back by Saturday.
57. Jerry could hardly solve the riddle.
58. Our representatives have been critical of the new concept.
59. It doesn’t matter whether they paint the board white or yellow.
60. The new prime minister has been appointed at last.
TRƯỞNG THPT CHUYÊN PHAN NGỌC HIỂN - CÀ MAU
A. MULTIPLE CHOICE (40 PTS)
I. l.A 2. c 3. B 4. D 5. D
II. 1. A 2. c 3. c 4. D 5. D
III. 1. A 2. A 3. c 4B 5. c
509
IV. 1 .A 2 .D
3 .B 4. A 5. B 6. D 7. B 8. A 9. B 10. c
V. PASSAGE A
l.D 2. c 3. A 4. B
5. c 6. B 7. D 8. A 9. A 10. D
PASSAGE B:
l.D 2. c 3 .B 4. B
5. D
6. B 7. D 8. B 9. D 10. B
VI. PASSAGE
1
l . c 2. A 3. D 4.
A 5. A 6. c 7. D 8. A 9. A 10. D
PASSAGE 2
l.B 2. D 3. A 4.
A 5. A
B. WRITTEN 6.C 7. D 8. A 9. B 10. c
TEST (70 pts)
I. PASSAGE A
1. impacts /ef 3. as 4. aid / support 5. wh ere
fec t 2. includ
ing
6. least 7. a mo
ng/am ongst 8.
more 10. th rou gh
with 9.
PASSAGE B
1. survival 2.
reactio ns . 4. sense 5. although
required 3.
6. to 7. d eliver
kinship 9. experience 10. se ts
ing 8.
II. PART 1:
1. v ivisec tion
6. slap-u p
10. dem arc ate
PART 2:
2. antipersonne l
7. e xpe nditio us
3. ma lconte nts 4. parlay
8. a nticoa gulan ts 9. enc om ia
5. bespect acl ed
1. reprod uc tiv e2. lum ine sce nt 3. disorienting 4. tem porar ily 5. util ised
6. respective
7. infections 8. ind epe ndent 9. rep lac em ent s 10. non-re newa ble
III.
1. line 3: cut and dry —> cut and dried
3. line 8: un der on
5. line 13: cling to -» cling
7. line 18: o the r - » rather
9. line 22: few —» a few
2. line 5: e duc atio n —» e ducation al
4. line 10: fundam entally —> f undame nta l
6. line 17: such -> that
8. line 19: sand —> m ud
10. line 20: loosing d own —> backing down
IV. 1. It's all toing and froing with all the cases tha t I ca n’t stan d about h olid ays .
2. A nation al sup erm ark et cha in has ta ken cha rge over the ma nageme nt.
3. On no account m ust this door eve r be locked because it is an em erg ency ex it.
4. Yo lan da w as talk ed into ent ering th e comp etition by her family.
5. T here was much JUBILA TIO N when the governm ent lost th e ele ction.
6. The board met behind the closed door in order to discuss changes in com pany policy.
7. Peter was in hot water because he didn ’t finish an important project by the deadline.
8. She let it be kno wn tha t she had been give n the sack by the c om pan y.
9. In th e event of a pro blem, drop a line at all costs.
10. Slugg ish as the ma nag er is, he has the gift o f the g ab
TRƯỜNG THPT CHUYÊN THĂNG LONG - ĐÀ LẠT - LÃM ĐỒNG
A. MULTIPLE CHOICE QUESTIONS
I. 1. D 2. A 3. A 4. A 5. B 6. A 7. B 8. c 9. c 10. B
II. 1. B 2. D 3. B 4. B 5. D 6. B 7 B 8. B 9. D 10. c
51 0
III. 1 .C 2 .D 3 .A 4. c
IV. l.B 2 . C 3 . D 4. B
V. READING 1
5. A 6D 7. c
5. A 6. B 7 B
8. B 9. D
8 A 9. A
10. B
10. D
1. B 2. A 3 .C
4 .C 5.C 6. A
READING 2
l . c 2. B 3 .B
4. A 5. A 6. B
VI. CLOZE TEST 1
l.D 2. B 3. A
4. B 5. A 6. B
CLOZE TEST 2
1. B 2 .C 3.B 4. D 5.C 6. c
B. WRITTEN TEST
I. TEST 1
1. such 2. that 3. o urselves 4. the 5. trouble / problems
6. no 7. can 8. difference 9. what 10. th ere
TEST 2
3. expe ctin g 4. loyal
1. som eth ing 2. attentio n
/devoted
5. along 6. lyrics 7. d espite / notw ithstanding
8. e nde d 9. v ersions 10. worth
II. A.
1. outsh one 2. w ar-torn 3. racist 4. extram arital
7. cutting-edge 8. infuriatin
5. n on- rep aya ble 6. c ontinually
g
9. inte rde pen den ce
B. 10. overseers
1. evo lution
2. w ilde rness 3. colon ial 4. establishme nt
5. a dvantag eousl y 6. sett lem ents 7. plantati ons 8. isolation
9. self-sufficiency 10. ac cessible
III. 1. Line 1: com es —>
6. Line 7: with —> in
came
2. Line 2: sim ilarly —> 7. Line 9: w hich is - >
similar which
8. Line 9: enlig hten -»
3. Line 3: ring s —» calls
lighten
4. Line 4: in —> to 9. Line 13: at -> from
5. Line 6: han dho lds - >
10. Line 14: do ing - > done
handhelds
IV. 1. It’s the way th eir fans adulate them that corru pts pop stars.
2. Reading between the lines, it seem s neither Cole nor Ledley King will be
goin g to Japan.
3. There was nothing whatsoev er eme rgency services could do on reaching the
crash victim.
4. So intense was the hatred for Frank’s new policies that the st aff wen t on strike.
5. Only o n his fourth proposal did she agree to m an y him.
6. Hard wo rking as he w as, he was unable to make ends meet.
7. In the face /tee th of fierce com petition, Frozen looks set to win the award for
Best Animation of the year.
8. Her late st novel isn't on a p ar with her previous one.
9. Her son tam pering with her computer, she blew her stack.
10. On her own adm ission, she was r ath er selfish
I
511
TRƯỜNG THPT CHUYÊN TIỀN GIANG - TIÉN GIANG
A. MULTIPLE CHOICE
1. l. B . 2. c . 3. A. 4.C. 5. D. 6. c .
II. l. B . 2. B. 3.B. 4. D. 5.C. 6. A.
III. 1. A .2. c. 3. A. 4. A. 5.C . 6. A.
IV. l .c . 2. A. 3 D. 4. A. 5. B. 6. B.
V. READING 1
4. B. 5. A. 6. B.
l .c . 2. A. 3.C.
READING 2
4. D. 5. A. 6. A.
l .c . 2. A. 3.B.
VI. CLOZE TEST 1
4. A. 5. B 6. A.
l. B . 2. D. 3. A
CLOZE TEST 2
4. D. 5.C . 6. B.
l.B . 2. D. 3. A.

B. WRITTEN TEST
I. CLOZE TEST 1
1. aged 2. affected
6. amount 7. much
CLOZE TEST 2
I. got 2. perform
6. cleared 7. require
II. WORD FORM 1
1. overpriced 2. outward 3
6. incriminate 7. foretaste 8
WORD FORM 2
3. fair
8. loss
3. previously
8. come
unattainable
proofreading
4. majority
9. describes
4. sign
9. aware
4. impartiality
9. cowardice
5 .beyond
10. addition
5. greatly
10. price
5. precooked
10. misinterpreted
1. insignificant 2. evolutionary 3. sufficient 4. interminable 5. consciousness
6. submissively 7. empower 8. resourceful 9. footing 10. natural
III.
1. Line 2: indulgently —> indulgence 2. Line 4: guest —> guests
3. Line 5: lucky —» luckily 4. Line 8: Connected —> Connecting
5. Line 9: environmental -> environmentally 6. Line 11: done —> doing
7. Line 12: o n—> in 8. Line 15: T o-> For
9. Line 16: industrial —> industry 10. Line 17: combination —» combined
IV. 1. Without absolute secrecy this mission would not have succeeded I been
successful / been a success / would have failed
2. At no time was there any likelihood of the two sides reaching an agreement.
3. So great a hypocrite is he that in public he condemns smokers, while he
smokes a packet a day himself.
4. Ruth is too proud to ask anyone for a loan as she doesn't like to admit she has
financial problems.
5. He flies off the handle at all things, even the slightest one.
6. Losing one's job has proved a blessing in disguise for some people.
512
7. He's a pleasant mail socially, but drives a hard bargain in business.
8. As a champion swimmer, she is in her prime.
9. A summary cannot do justice to (the high quality of) this book.
10. The inspector drew our attention to four potential health hazards.
TRƯỞNG THPT GIA ĐỊNH ■ TP. Hồ CHÍ MINH
A. MULTIPLE CHOICE (40 PTS)
I. l.c. 2. B. 3.D. 4. A. 5. D. 6. D.
II. ll .D . 12. D. 13. A 14. B. 15. D.
III. 21. B. 22. B. 23. A. 24. c. 25 B
IV. 31. c. 32 c. 33 B. 34. A. 35. B.
V. 41. A. 42. c. 43. B. 44. A. 45. D.
VI. 51. B. 52. D. 53. A. 54. D. 55. B.
VII. 61. c 62. A 63. B 64. D 65. A
71 B. 72 A. 73 D. 74 c. 75 c.
B. WRITTEN TEST
I. PASSAGE 1
.1. credited 2. on 3. Its
6. attributed 7. around 8. awarded
PASSAGE 2:
II . has 12. which
16. ever 17. drowning/ dying/ perishing
19. amount 20. did
II. PART 1:
I. full-service
5. unquestioning
9. painstakingness
PART 2: 10. amphibious 14. Absence
II. invariably
12. Alternatively
15. Observations
19. Unpredictable 20. irregular
III. Line 3: therefore -> however Line 4: those -> that
Line 6: hide -> hides Line 7: to —> for
Line 10: circle —> cycle Line 11: in —» on
Line 11: 360-years -> 360 years Line 15: have -> having
Line 16: other -> another Line 18: Suns -> Sun’s
13. Not
16. c .
26. B.
36 c.
46. B.
56. B.
66. c
76 A.
7. D.
17. B.
27. D.
37. D.
47. c .
57. A.
67. D
77 D.
8. c.
18. A.
28. A.
38. c .
48. B.
58. B.
68. D
78 A.
9. B.
19. A.
29. B.
39. B.
49. D.
59. c .
69. A
79 c .
10. c .
20. A.
30. B.
40. A.
50. A.
60. A.
70. c
80 c.
4. when
9. looked
14. so
5. from
10. site
15. being
18. entire/ whole
2. misuse
6. perfectionist
16. opinionated
3. counter-productive 4. loss-making
7. disfiguration 8. uninhabited
13. Descriptive
17. Fairness
18. Enthusiasm
IV. 1. Not for another five years did the whole truth about the murder come out.
2. Many's the time that I have made stupid mistakes like that..
3. Don’t breathe a word of/about this to the boss.
4. The young man appeared to be having a chip on his shoulder about his family
background.
5. He has a bee in his bonnet about the dishonesty o f lawyers.
513
6. He was fed up to the back teeth with being criticized in public all the time.
7. It 's not beyond the bounds of possibility that the looming crisis will never/ not
actually materialize.
8. What little I remember about my mother is that she was very kind and loving
towards US.
9. You have hit the nail on the head
10. The stadium was designed in such a way (as) to / that it could/would stamp/
rule/iron out hooliganism.
TRƯỞNG THPĨ NGUYỄN THƯỢNG HẵỂN - TP. Hồ CHÍ MINH
A. MULTIPLE CHOICE
QUESTIONS
I. l . c 2. D 3. B 4. c 5. D
II. LA 2. c 3. D 4. A 5. B
III. 1. B 2. A 3. D 4. c 5. c
IV. 1. D 2. B 3.B 4. A 5. B
IV. PASSAGE 1
LB 2. B 3. A 4. D 5.C
PASSAGE 2
3. A 4. B 5. A
l. A 2. D
V. PASSAGE 1
3. D 4. D 5. B
L A 2. c
PASSAGE 2
3.B 4. A 5.C
L B 2. A
B. WRITTEN TEST
I. PASSAGE 1
1. making 2. once
6. off 7. steer
PASSAGE 2
1. originated 2
4. unlikely / improbable 5
7. known 8
II. PART 1.
1. unequivocally
3. disobedience
7. irresistibly
PART 2
3. through 4.
8. they 9.
Another 3. defend
about 6. earliest
before / earlier 9. whoever ■
2. maltreatment / mistreatment
4. enigmatic 5. dispersal
8. uninitiated 9. erroneous
1, assured 2. laboriously 3. affirmative 4. exceptionally
5. solidarity 6. alternative
7. insecurity / insecurities

10. confronted
8. unintentionally 9. tendency
2. Line 4: at the end -> in the end •
III. 1. Line 3: using —> used
3. Line 5: exclusive —» exclusively
5. Line 7: where - » whereas / while
of 5. what
down 10. just
10. subject
6. abstentions
10. formative
4. Line 7: distinct - » distinction
6. Line 10: what —» how
51 4
7. Line 14: himself —> itself 8. Line 20: crude -» raw
9. Line 20: should -> would 10. Line 21: to —» for
ỈV. 1. Not until the film (had) ended did 1 remember to switch off the oven.
2. The manuscript is I has been attributed to a fourteenth century scholar.
3. You do n't happen to have seen my glasses anywhere, do you?
4. On no account must you go into my office without permission.
5. It's a good idea to take out a small loan to (help) get. your business off the ground.
6. I've got missing the rush hour down to a fine art over the last few w eeks.
7. Despite (him/ his) being dead (set) against playing cricket on Sunday, Rudy
agreed in the end.
8. I cast my mind back to what has happened all those years before.
9. My parents, who are up in amis about my/ me getting a tattoo, have grounded me.
10. Since the company 's methods were exposed in a newspaper, it has fallen into
disrepute.
TRƯỜNG T HPĨ THỰC HÀNH CAO NGUYEN - ĐẮK LÃK
A. MULTIPLE CHOICE (40 PTS)
I. l. A 2. c 3. A 4. A 5. c
l. B 2. c 3. D 4. A 5. B
II.1 .D 2. B 3. D 4. B 5.C 6. B 7. B 8. D 9. c 10. B
III. 1. D 2.D 3.A 4.B 5.B 6.C 7.B 8.A 9.D 10.C
IV. 1. B 2. c 3. D 4. A 5.C 6. c 7. A 8. B 9. D 10 c
V.
PASSAGE 1
l . c 2. A 3. c 4. D 5.C 6. B 7. A 8. B 9. D 10. B
PASSAGE
2:
l . c 2. D 3. A 4. D 5.C 6. c 7. D ■ 8. D 9. A 10. c
VI.
PASSAGE
1:
l. D 2. B 3. A 4. c 5. A 6. D 7. B 8. c 9. B 10. A
PASSAGE
2:
1. Although 2. Worth 3. Only 4. At/on/in 5. Silence
6. Over 7. Should 8. possibly 9. Great 10 .happene
d
B. WRITTEN TEST (70PTS)
I. PASSAGE 1:
1. on 2. when/if 3. is 4. from 5. go
• 8.
6. real 7. however type/sort/kind 9 10. Unless
out
PASSAGE 2:
12. although 13.
11 .ago 14. In 15. where
for
16. salary 17. very 18. a/per 19. situation 20. about
II. PART 1
1. qualifications
stylishly 3. electrification 4. denial
2.
5. assistance 6. disconnected 7. theoretically 8. humanitarian
9. cyclists 10. irresponsible
515
PART 2:
I. conversation
6. interruption
. research
. meaningful
3. housework
8. argumentative
4. dom inance
9. listeners
5. unpleasant
10. impression
Il l. I. of (line I) -> for
3. firstly (line 3) -> first
5. which (line 7) -> where
7. too (line 13) —> to
9. another (line 13) -> a / one
2. that (line 2) -> one
4. reasonable (line 6) —» reasonably
6. still (line 9) -» and
8. gooder (line 13) -> better
10. hundred (line 14) -> hundreds
IV. 1. Much as I admire his courage, 1 think he’s foolish.
2. All dogs are thought (by experts) to have evolved from wolves.
3. Had he not lacked right qualifications, the university would have prepared to
consider his application.
4. Should you find it necessary, you can contact me on this number
5. Many people nowadays find it increasingly difficult to make ends meet / to
make both ends meet.
6. I've fed up with going to the same place all the time.
7. Mary prides herself on being a good cook.
8. The rain was coming down cats and dogs.
9. They arrived at their destination safe and sound.
10. The burglar was caught red-handed.
TRƯỜNG THPT CHUYÊN NGUYỄN THIỆN THÀNH - TRÀ VINH
A. MULTIPLE CHOICE (40 pts)
I. l . c 2. B 3. B 4. B 5. c
II. 1. D 2. B 3. c 4. A 5. D
III. 1. C 2. B 3. B 4. c 5. D
IV. 1. C2. A 3.C 4. B 5. B
V. READING 1:
l . c 2.B 3.C 4. A 5. D 6. B
READING 2:
4. A 5. D 6. B
l. A 2. A 3. A
VI. CLOZE TEST 1:
4. A 5. c 6. C
l. B 2. A 3. D
CLOZE TEST 2:
4. C 5. D 6. B
l. B 2. C 3. A
B. WRITTEN TEST (70pts)
1. OPEN CLOSE TEST 1:
1. to 2. produce 3. place 4. up
6. sell 7. covered 8. including 9. refer
516
OPEN CLOSE TEST 2:
1. two-way 2. communication 3 .sender 4. message 5. mention
6. voice 7. may 8. eye 9. like 10 .sensory
II. PART 1:
1. withdraw 2. impassable 3. interpretation 4. untrue 5. imaginative
6. memorized 7. ghostwriter 8. untrustworthy 9. playwright 10. invariable
PART 2:
11. tension 12. emotional 13. respiratory 14. digesting 15. anxious
16. overreact 17. Long-term 18. overuse 19. disorders 20. Untreated
III. 1. Line 1 : in —> on 2. Line 3: had has
3. Line 6: included —> including 4. Line 8: his head —> her head
5. Line 9: and Bartholdi -> but Bartholdi 6. Line 11: o f copper -> from copper
7. Line 13: so hard —> too hard 6. Line 14: its weight —> their weight
9. Line 16: exciting —> excited 10. Line 19: plan—» planned
IV. 1. Vegetables should be washed before bein g eaten.
2. These ancient woodlands are und er ma jor / gre at thr eat from new road
developments.
3. 1 hav e no op tion/ choic e/ altern ative but to give up the whole idea.
4. He cou ldn 't resist the tem ptation to open the letter.
5. The research sho ws a cor relatio n betw een heart diseases and smoking.
6. The diplomat has been arrested on suspicion of spying for his government.
7 He got up at the cra ck of dawn this morning.
8. Though they had had many differences, they hadn’t fallen out.
9. I am resentful of/ at/ about the way that she look s dow n her nose at me.
10. As luck w ould have it, his route took him very near where they lived.
TRƯỞNG TH PT CHUYÊN NGUYỄN QUANG DIÊU - ĐỒNG THÁP
A. MULTIPLE CHOICE (40 pts)
I. l. B 2. B 3.
4. A 5. D 6. A 7. D 8. A 9. c 10. B
B
II.1. B2. c 3.
4. c 5.D 6. D 7. c 8. D 9. B 10. D
B
III. LA 2. B 3.
4. D 5.C 6. B 7. A 8. A 9. D 10. B
D
IV.
3. Reading
1. word for 2. in a
between the
word word
lines
4. word 5. lost for 6. take it as 7. a dark
perfect words read horse
9. the light
8. a shot in the in the end 10. take it
dark o f the amiss
tunnel
V. READING
1:
l. D 2. C 3 .A 4. c 5. A 6. B 7. c 8. B 9. D 10. B
READING 2:
l . c 2. A 3. B 4. c 5. D 6. B 7. c 8. A 9. c 10. B
VI. CLOZE
TEST 1:
1. c 2. D 3.C 4. A 5. D 6. B 7. A 8. B 9. B 10. B
517
CLOZE TEST 2:
I. A 2. B 3.C 4. B 5. A
B. WRITTEN TEST (70pts)
I. OPEN CLOZE TEST a
1. use 2. today/alone
6. were 7. up 8. rather
OPEN CLOZE TEST b
l. as 2. look 3. to
6. reasons 7. let 8. out
3. deformity 4. multiracial
II. 1. Inequality 2. supernatural
6, personified
10. undernourished
14. misguided
18. invaluable
5. nonappearance
9. disheartened
13. impressive
17. inconsiderable
HI. 1. Line 1 : normal —> ordinary
3. Line 3: are -> move
5. Line 5: they —> it
7. Line 7: in —> on
9. Line 9: painting —> paint
6. D 7. A 8. B 9. C 10. c
3. which 4. light 5. their
9. to/with 10. this
4. fact 5. and
9. however 10. in
7. non-resident
11. adventurers
15. irrelevance
19. repeatedly
8. disrepair
12. understood
16. consultative
20. worthless
2. Line 2: If —> when
4. Line 3: inside —» inside them
6. Line 6: other —> another
8. Line 7: and —> or
10. Line 9: have -> have a
IV. 1. His conviction for fraud cost the trainer his license
2. I'm so annoyed by my little brother sometimes
3. The excuse for the war declaration (for the decla rat ion of war) was the
defense of their territorial rights.
Or The excuse for the war declaration was to defend their territorial rights
4. You ca n't go on a bus without hearing someone talking about snobbery
5. It's the way their fans adulate them that corrupts pop stars
6. There is little like hood that the television will blow up "at any time.
7. These problems of ours are just the tip of the iceberg
8. In the area, Thailand is head and shoulders above all other countries in football
9. Thomas was kept in the dark about the com pany's new projects.
10. I lacked (the) courage to tell him what I really thought
TRƯỜNG THPT CHUYÊN HÙNG VƯƠNG - GIA LAI
A. MULTIPLE CHOICE
QU ESTION S (4 0 pts )
I. LA 2. A 3.D 4. A 5. c 6. D
II. LA 2. A 3. A 4. c 5. B 6. A
III. l. A 2. c 3. B 4. A 5.C 6. c
IV. l. D 2. c 3. A 4. A 5. D 6. A
V. PASSAGE 1:
4. c 5. c 6. c
LB 2. D 3. B

518

PAS SAGE 2:
2. B 3.
1. A 4. C 5. D 6. B 7. c 8. A 9. D 10
D
IV.

PASSAGE I:
22. c
21. D 24. A 25. B 26. c 27. A 28. B 29. B 30
23. c
PASSAGE 2:
2. D 3.
l.c 4. c 5. B 6. A 7. c 8. B 9. D 10,
c
B. WRITTEN TEST
Ỉ. CLOSE TEST 1:
1-until 5. interfere
2. areas 3. ring
6. solve 10. turn
7. passengers 8. silent
CLOSE TEST 2:
I. in 2. own 3. by
6. in 7. where 8. all
II. PART 1:
1. disastrous 2. preoccupied
4. pacemaker
9 .lose
4. our
9. instead
5. few
10. among
5. abnormalities
10. disclosures
3. unexplored 4. deaden
6. specialties 7. provisionally 8. Uncompromisin 9. outspoken
PART 2:
1. traditional 2. generations 3. anger 4. foreigners 5. secretly
6. unfashionable 7. service 8. patience 9. politeness 10. frustrate
2. Line 3: and intricate —> an III. 1. Line 2: the second —> a
intricate second
4. Line 5: Reportoire Repertoire
6. Line 8: signature -> signing
8. Line 9: trands —> trends
10. Line 12: personnel —> personal
3. Line 4: is functioned —» functions
5. Line 7: promised —> promising
7. Line 9: to investigate —> investigating
9. Line 10: reciprocates reciprocate
V. 1. Rather than disturb the meeting, Ỉ left without saying goodbye.
2. Further information can be obtained by sending a self-addressed envelope to
the above address.
3. I might have known my children woidd not like the new, cheaper ice-cream.
4. Whatever the noise the children made, the baby slept soundly.
5. Don 'Í come to/ draw the conclusion that jo b is easy.
6. Russ wav at odds with his f 'ellow workers over the new management policies.
7. The you ng actress had butterflies in her stomach before the audition.
8. Margaret has a reputation fo r being a very good cook.
9. S he’s very fussy about the kind o f hotels she stays in.
10. Most salaried people h aven't been able to cope with the rocketing prices.
TRUỬNG THPT CHUYÊN LÝ Tự TRỤNG - CẦN THU
A. MULTIPLE CHOICE
I. 1. c 2. D 3. D 4. B
IL 1. B 2. B 3. A 4. D
III. 1. B 2. c 3.D 4. A
IV. J.c 2. B 3. D 4. A
519
V. PASSAGE 1: }
l. D 2.A 3.C
4. D 5. A 6. c
PASSAGE 2;
l. D 2.C 3. B 4. A 5.D 6. c
VI. PASSAGE 1:
l. A 2. c 3.D 4. c 5.B 6. A 7. D 8. c 9. D 10. B
PASSAGE 2:
l . c 2. B 3.B 4. D 5.D 6. B 7. A 8. c 9. A 10. D
B. WRITTEN TEST
5. teaching
10. opportunity
5. into
10. natural 6. endure
7. with 8. losing
9. navigate II. PART 1:
4. Unobjectionable
8. Requisitioned
I. PASSAGE 1:
1. regarded 2. habit 3. mind 4. prove
6. run 7. When 8. latter 9. time
PASSAGE 2:
I. habit 2. harsh 3. equipped 4. nests
1. incorrigible 2. interdepartmental 3. multi-tasking
5. misty-eyed 6. dumbfounded 7. assimilated
9. self-evaluation 10. gatecrashed
PART 2:
1. (re)assured 2. laboriously 3. affirmative 4. exceptionally 5. solidarity
6. checkout 7. alternative 8. insecurity 9. tendency 10. confronted
III. 1. line 1: composer —> composers
3. line 4: and —» although
5. line 7: controlled —> lost
7. line 11: uniquely —> only
9. line 13: never ever
2. line 3: publishing -» published
4. line 6: Like —> As
6. line 9: when —> where
8. line 12: played —> composed
10. line 16: that —> who
IV. 1. My new job is far and away the most satisfying one I've ever had.
2. But fo r Tom's prompt action in putting the fire out, there might have been
more damage to the house.
3. So you ng is a child of his age to he deceitful.
4. I've got missing the rush hour down to a fine art over the last few weeks.
5 .1 d on't think we should write of f her ideas as unworkable at this stage.
6. It’s crucial for USto keep a tight rein on the juven ile inmates.
7. You’ve done nothing hut moon around all day.
8. We d on’t hold out much hope fo r the survival o f the tourists in that accident.
9. Your story does not tie in with the facts.
10. His excuse fo r such had behavior cuts no ice with her.
52 0
LÓ P 11 ]
ĐỀ CHÍNH THỨC
A. MULTIPLE CHOICE (40 PTS)
l. B 2. A 3.C 4. D
l l .D 12. A 13. D 14. A
21. A 22. B 23. c 24. D
3 Ỉ.C 32. A 33. c 34. A
41. D 42. A 43. B 44. B
51. A 52. c 53. B 54. D
6l.c 62. A 63. B 64. A
71 c 72. B 73. A 74. c
B. WRITTEN TEST
2. stem
7. as 6. little/no
12. research II . don atio n
17. promised
3. in
8. Desp ite
13. from
18. helpin g
4. account
9. lab oratory/lab
14. second
19. fo refront
5. up
10. aw ay
15. quality
20. on
2. anticlimactic
6. anti-carcinogen ic
10. Ab dom inal
14. inward
18. industrially
3. w atchword
7. unim ped ed
11. em erging
4. well-d irecte d
8. to uchdown
12. unde velope d
I. 1. form
16. headquartere d
II.
1. incorrigible
5. u nheard- of
9. inaliena ble
13. re lentless
17. implica tion s
III.
1. line 5: the photo gra pher —» p hotographer
3. line 9: put -» leave
5. line 12: spoil —» sports / sporting
7. line 15: so —> as
8. line 18: tha n ju st simp ly —» than simply
10 line 20 : em otions w hich -> emotion s with which
15. multinationals 16. ab und anc e
19. periph eral 20. con scie nce
2. line 8: th at —> w hich
4. line 11: having —> o f
6. line 12: the ir -» its
9. line 19: take —> mak e
IV. 1. To put it bluntly, I do not sub scribe to the opinion that nature an d nurture
are o f equal significa nce.
2. In the area , Thaila nd do esn’t sta nd he ad an d sho uld ers above all oth er
countries in fo ot ba ll an y longer
3. For fear that he (should) lose his high position, he swept the matter under the carpet.
4. He gave e very indication o f go ing to any lengths to obta in the contract, / that
he wo uld go to ...
5. Had it not bee n fo r my fa th er 's presenc e, I' d have ha d the guts to tell the truth.
6. N ot for anoth er fi ve yea rs did the whole truth ab out the mu rde r come out.
7. His sud den m arr iag e proposal took her by s urp rise / aback.
521
8. Far from be ing e mba rras sed by M im i's be hav ior, he se em s io he am used by it.
9. It dawned on me aft er the e xp edition th at she w as no gre at sh ak es as a
researche r.
10. Th e acc use d may conceiv e o f do ing a wa y with the witn ess.
ĨRưỜNG ĨH P Ĩ CHUYÊN NGUYỄN BỈNH KHIÊM - VĨNH LUNG
A. MULTIPLE CHOICE
I. 1. D 2. B 3. C 4. C 5. D
II. l.D 2 A 3.C 4. B 5. D
III. 1 .C 2 .C 3.A 4. B 5. B
IV. 1. C 2 C 3. C 4. C 5. A
V. PASSAGE 1
1. B 2. C 3.D 4. B 5.A 6.D 7.A 8.D 9.A 10.D
PASSAGE
2:
l. A 2. C 3. B 4. A 5. A 6. B 7. A 8. D 9. A 10. E

IV. CLOZE TEST 1:

l.D 2.B 3.B 4.A 5.B 6.C 7.A 8.C 9.A 10.B
CLOZE
TEST 2:
1. C 2. c 3.D 4. A 5. B 6. D 7. A 8. c 9. B 10. D
B. WRITTEN TEST
I. OPEN CLOZE TEST 1
1. as 2. bec ause 3. e xha ust 4. provides
6. o r 7. ben efit 8. g radually 9. require
OPEN CLOZE TEST 2:
1. spe cifica lly 2. sub scribe rs 3. w hich 4. out 5. righ
6. basic 7. raise 8. milesto ne 9. primary 10. on
IL WORD FORM 1:
1. under estimated
5. activat ed
9. vis ual ize
10. deforestati on / disaffore station
WORD FORM 2:
1. Ty pic ally 2. dramatise d
6. encouraging 7. professionalised
2
III. 1. L ine 3: ap plica tio n —> a pp lie d
Li ne 5: principle —> p rin cip al
3.' Line 6: from —> by
4. Line 8: a rises —* rises
5. Line 8: suc ks —> sucking
6. Line 9: ju st as —> such as
7. Line 9: w ith —> in 8. L ine
9. Line 14: and last —> or last
2. indefensible 3. lux iria ting 4
6. a dditive s 7. dis rep air 8
3. g race ful
8. departure
unilate ral
inc essant ly
4. prec eden ce 5. unhindered
9. com mercial 10. spo nta nei ty
8. Line 12: d epe nd —> depending
10. Line 14: m otions —> motion
52 2
IV. 1. Wo uld it be possible for me to talk to som eon e abo ut my problem ?
2. The exc use for the declaration o f the w ar is the defence o f their territorial rights.
3. S uch is the p opularity of the play that the thea ter is likely to be full every night.
4. file boy w as on the point of crying when he ...
5. N o form al an no un cemen t o f the d ecision was m ade/ given.
6. He held/kept/s tood his ground aga ins t his adv ersary.
7. The meanin g of a “f reebie” sud den ly daw ned on me.
8. Mo st sto res will accept a c red it card as an altern ativ e to cash.
9. Th e bank rob bers made a get away in a stolen car.
10. Don ’t m ake a m ountain out o f the molehill.
THPT CHUYÊN BẢO LỘC - TP. BẢO LỘC - LÂM ĐỐNG
A. MULTIPLE CHOICE (40 PTS)
I. 1. D 2. B 3. A 4. B 5.C
II. 1. D 2. D 3.C 4. D 5. B
1 1 .A 12 .B 13. D 14 B 15.c
II I. 1 .C 2 . c 3. D 4. B 5. A
IV. T EXT 1
l . c 2. B 3. A 4. D 5.C
TEX T 2 '
l.D 2. A 3 .C 4. B 5. A
V. READING 1
1. c 2. B 3. A 4. A 5.B
READING 2
l.D 2. A 3. A 4. D 5. B

6. D 7. A 8. c 9. D
6. c 7. A 8. A 9. B
16. c 17. A 18. D 19. c
6. A 7. A 8. D 9. B
6. c 7. c 8. A 9. B
6. B 7. A 8. c 9. D
6. A 7. c 8. B 9. D
6. B 7. A 8. c 9. A
B. WRITTEN TEST (70 pts)
I. TEXT 1
1. exten t/degree
6. on
TEX T 2
2. as
7. ta ken
3. likelihood 4. lack/absence
8. reduc e 9. v iew /regard
1. Th ere fore
6. reached
II. PART 1:
1. ine xplicabl e
5. n on- a ggres siv ely
9. apportio n
2. con ducted/d one
7 On
3. whereas 4. rath er
8. com bin es 9. but
2. irreplaceable 3. obs ervatory
6. classifieds 7. good will
10. un substan tiated
PA RT 2:
1. idealis ts
6. rigoro us
2. marrie d
7. indirec t
3. being(s)
8. redu ces
4. reachable
9. argument
5. yet
10. though
5. affo rds
10. record
4. imper son ating
8. u nen viable
5. w holeh eartedly
10. unin telligible
52
III .
Line Error Correction L//ÍỨ
2 have has 17
refers to /is
3 ref ers as 18
referred to as
a t ... from ...
9 20
persp ectiv e per spe ctiv e
11 distin ct distinction 22
14 determi ned determinin g 23
IV. 1. H ad n't it b een for my fath er 's pre sen ce, I might/w ould hav e had the guts
to tell the truth.
2. With hin dsi ght, we could hav e run a tight ship/w e did n't run a tight ship.
3. I'd like to hav e paid my/the last respect to his fath er.
4. Hard as we tried/migh t try. our plan was just pie in th e sky.
5. As reg ard s politic al view , his com ments are some tim es/ at tim es bey ond the
pale .
6.1 objec t to alw ays h avi ng to draw my bo ss 's fire when things go wrong .
7. Th e adults sh ou ld n't have buried their head in the sand about the kids '
. problem s.
8. It h as n’t bee n made cle ar to me wh at is required o f me in t he/my new jo b.
9. It is crucial tha t A ndrew 's fath er (should) sup port his determination to bec om e
a d octor de spite his poor health.
10. Stephe n Hawking holds the belie f that that the universe is only governe d by
the laws o f scien ce due to his com mitm ent to atheism.
TRƯỞNG THPT CHUYÊN BÊN TRE - BÊN TRE
A. MULTIPLE CHOICE (40 PTS)
I. 1. B
3 .C 4. c 5. D 6. D 7. B 8. c 9. B 10. A
2. A
II. l l .B
13. A 14. B 15. B 16. A 17. A 18. D 19. B 20. D
12. c
III.
31 .D 33.A 34.D 35.B 36.A 37.B 38.A 39.C 40.B
32 .C
IV.
31 .D 33.B 34.D 35.A 36.D 37.C 38.A 39.B 40.B
32 .A
V .4 1.B
43.A 44.C 45.D 46.B 47.B 48.A 49.D 50.D
42.C
51.A
53.D 54.A 55 .B 56 D 57.B 58.C 59.A 60.B
52.D
VI.
61 .D
64.C • 65.A 66 .B 67.D 68.B 69.A 70.A
62 .
A63 .A
71.c
73.A 74.D 75 .B 76.A 77.C 78.A 79.B 80 .D
72.A
B. WRITTEN TEST
I. OPEN CLOZE 1 (10PTS)
3. exc ava
1. rec ogniz ed 2. in 4. trac ed 5. pap er
tions
6. vary 7.
Although
8. multiplied 9 .such 1 0 .sacr ific es
OPE N CLOZ E
2 (10PTS)
1. Some wh ere 2. over/for 3. feed ing 4. title 5. resist
6. entire 7. bottom 8. once 9. highly lO.lengths/pains
524
II. PART 1:
1. thera peutic
6. contrariwise
PART 2
. celeb rato ry
. side splitting
3. virulent
8. happenstance
4. w eather-beaten
9. insatiable
1. grassro ots
6. bypass
III.
2. mi schie vously 3
7. imperce ptive 8
ing eni ous 4
intimidated 9
docume nta ries
car pente red
5. ke ystone
10. posthumously
5. practition er
10. g rip pin g
1. Line 1: s om e time —> sometim e 6. Line 13: co ne-sh ape —> cone-shaped
2. Line 4: likely —> often 7. Line 17: al ike -> like
3. Line 6: memo rizing —> com mem orating 8. Line 22: products —» produce
4. L ine 8: spring —> spruce 9. Line 24: various -> varied
5. Line 9: stan ding —> outstanding 10. Line 27: decease —> deceased
IV. 1. Yo ur story do es n't tie /7/ with the fa ct s.
2. Martine ha s reac he d th e pi nn ac le (1) Io f his ca reer.
3. I would sooner yo u h adn 7( 1) 1 paid lip service to my view ye ste rd ay .
4. Sor ry, you are n ot in a po sit ion (1) Ito pick and choose
5. There is no lim it/r est ric tion on (1)1 how mu ch/wha t y ou ea t fo r $5 at the
new lunch-bar.
6. My fat her hit the ro of (1)1 when he dis co ve red som eone ha d tampe red with
his car.
7. Th ose obses sed with compu ter games (1)1 str ug gle to draw the li ne b etw een
the vir tual wo rld an d th e r ea l world.
8. Hav e a loo k at this pic tur e. It ma y help yo u jo g yo ur memory.
9. Ap paren tly, many an em plo yee wil l be thrown on the s crap heap (1)1 upon
the take ov er o f 21st Centu ry Fox.
10. Plain sa iling th ough /as the begin nin g o f a bu sin ess m ay be (1) |, one ha s to
be preo cc up ied with the w ors t-cast scenar io.
TRƯỜNG THPT CHUYÊN BÌNH LONG - BÌNH PHƯỚC
A. MULTIPLE CHOICE (40P TS )
I. 1.C 2. c 3. D 4. B 5
.A 6 . D 7. c
II. L A 2. A 3. B 4. B 5. c
II I. L B 2. B 3 .C 4. D 5. c
IV. 1.B 2.B 3.C 4.D 5.A
V. 1.
l.B 2. c 3. B 4. D 5. c
2. 1.B 2. c 3. D 4. B 5. A
VI. 1.
1.B 2.C 3.A 4.D 5.C
2.
3. D 4. B 5.C
L B 2. B
8. B 9. A 10. D
B 7. A 8. D 9. c 10. D
6. B 7. B 8. D 9.A 10. B 10.D
6.C 7.C 8.C 9.C
6. B 7. D 8. A 9. B 10. B
6. D 7. c 8. c 9. B 10. A
6.A 7.B 8.D 9.C 10.C
6. A 7. B 8. A 9. D 10. c
525
B. WRITTEN TEST ( 70PTS)
1. 1.
1 .bear s 2.com ponen ts
6, cle are d/d isa ppeared 7.verge
2.
2. atte ntion
7 des pite
3.for
8.blam e
4. Hardly
9. witho ut
2. far-fetched
7. unskilled
3. exp ecting
8. ended
4. loyal
9. v ersion s
4. sel f-m ade
9. o ver -rated
5. chu rn
10.in
5. alo ng
10. w orth
5. ina dvertently
10.ov erd ose
I. som eth ing
6. lyrics
II. 1.
1. sit-in
6. disagree able
2.
1 .sing le-handedly 2.topped
6. rev ere nce 7.ta ntrums
Iĩĩ . 1. Line 2: wh ere -> when
3. Line 5: salution -» salute
5. Line 8: humo r —> hum orist
7. L ine 11: m eas ure d —» me asu ring
9. Line 14: m ark - » m arks
3. cour iterfactual
8. footwork
3.demographic 4. unimposin g 5. rebelli on
8.be ing reco gnized 9. tim idity 10. loveab le
2. Line 3: flan g -> flung
4. Line 7 : it - > mys elf
6. Line 9: exp ected —» h ad expecte d
8. Line 12: w ig ling-» w igg ling
10. Line 14: on —> o nward
IV. 1. Ma nag ing th e com pan y shou ld not be any thing like as easy as the y say.
2. Had my father not gone awa y for the weekend , his sec retary would not have
min ded the s tore in te rms o f the business.
3. There is resista nce among the fam ily members to the fac t/id ea tha t the ir
fa ther's fortun e will go to charity.
4. Should she fail to accom plish her mission, her job will be on the line.
5. 1 had/f elt no inclina tion to get out of bed the day after losi ng my job .
6. It is n ecess ary to free our busines s from the dead hand of bu reaucr acy .
7. The commuters on the train were packed like sardines during the morning rush hour.
8. Since the beg inn ing o f the season, the club h as been on the up and up.
9. The boys zeroed in on wh at the tea cher was saying in order not to mis s any
small details .
10. As he cause s so muc h trouble, the re’s nothing we can do now besid es from
letting him stew in his own juic e.
TRƯỜNG THPT CHUYÊN HÙNG VƯƠNG - BÌNH DƯƠNG
A. MULTIPLE CHOICE QUESTIONS (40pts)
I. L A . 2. B. 3.C. 4. D. 5. c 6. A
II. l l .D 12 D 13. B 14. c 15. B 16. B
IV. 21. B 22 . D 23 D 24. A 25. c 26. A
III. 31. A 32. c 33. D 34. B 35. B 36. A
VI. 41.C 42. C 4 3 .D 44. c 45. D 46. D
51 A 52. B 53. D 54. D 55. A 56. B
V. 61. A 62. c 63. B 64. D 65. A 66. B
71. c 72. A 73. c 74. c 75. D 76. D
526
B. WRITTEN TEST
1. PASSAGE A:
I. therapy 2
6. habitually 7
PASSAGE B:
II . reconnecting
16. emotional
n . PART 1
variety 3. stress 4. household 5. cut
kills 8. inco nven iences 9. alternative 10. knots
12. implicit 13. surface- 14. automatically 15. retrieve
17, activations 18. concrete 19. pleasant 20. identities
1. ill-advised/ inadvisable
5. Desensitized
9. apolitical
PART 2:
2. armament
6. insurmountable
10. burdensome
3. Breakneck
7. malpractices
4. absurdity
8. equidistant
II . preponderance
15. foreseeable
18. substandard
III.
12. leisurely
16. resignedly
19. shipwrecked
13. transatlantic 14. hold-ups
17. discontent(ment)
20. overtaxed
1. Line 1: phenomenon —> 6. Line 11: create —»
phenomena creating
7. Line 14: contact —»
2. L ine 6: couple -> coupled
contact with
3. L ine 6: from —> in 8. Line 16: on -> across
4. Lin e 9: rotated —> 9. Lin e 18: not common T-»
rotating common
5. L ine 11: acelerate —»
10. Line 20: if —> as / when
accelerate

IV.
1. Bui for Jack 's being so affluent, she would not be dating with him.
2. They prob ably for got about the extra class.
3. I’m looking for a woolen, green and fairly long sc arf
4. Under no circumstance should you lock this door when the building is open
to the public.
5. Death is more likely to be caused by a bee sting than by a snake bite these days.
6. If you have a problem, why don 't you get it ợ//’your chest?
7. Why don 'tyou have it out with your boss right now ?
8. Anna was sure Tom was lying through his teeth about what happened
9.1 put my sister's bad-temper down to the fact that she was tired.
10. The accused showed no remorse for the crime he had committed.
TRƯỜNG THPT CHUYÊN HÙNG VƯƠNG - GIA LAI
A. MULTIPLE CHOICE
(40 points)
I. 1. B 2. c 3. B 4. D 5.D
II. 1. D2. D 3. B 4. B 5.A
III. ỉ. A2. B 3.C 4. B 5. D
IV. 1. A 2. B 3.C 4. D 5. A
527
V. READING
PASSAGE 1 (5
points)
l . c 2. A 3. C 4.
D 5. c
READING 6. A 7. B 8. A 9. D 10.
PASSAGE 2 (5
points)
l. D 2. c 3. D 4.
A 5.B
6. A 7. D 8. B 9. B 10.
VI. CLOZE
TEST 1
l. A 2. c 3. A 4.
A 5.B
6. C 7. A 8. D 9. A 10.
CLOZE TEST
2
l. B 2. c 3. B 4.
D 5.C 6. A
B. WRITTEN
TEST (70
7. D 8. C 9. A 10.
points)
I. CLOZE
TEST 1: (10
points)
1. every 2.
whether 3.
saying 4. transmitting 5. complete
6. involves 7. 9. apart 10. Were
handful 8.
between
CLOZE TEST 2: (10 points)
l.but 2. enough 3. incapable 4. on
6. with 7. extent/ degree 8. ahead/forward 9. giving
II. PART 1:
1. bemoaning 2. conscientious 3. nonsensical 4. diettician 5. absenteeism
6. awestruck 7. adultery 8. equidistant 9. irrevocable 10. incorrigible
PART 2:
1. infancy 2. institutions 3. exclusively 4. insight 5. disappearing
6. commitment 7. inaccessible 8. loneliness 9. immersion 10. undeniable
III. 1. Line 1: for —> to
3. Line 7: giving giving o ff
5. Line 8: in —> into
7. Line 16: resembles to —> resembles
9. Line 19: to —> from
2. Line 5: it —> they
4. Line 7: change —> changes (verb)
6. Line 10: limits —> limitations
8. Line-17: identified —» have identified
10. Line 20: last -> least
IV. 1. Along came the police and away went the mob/ the mob went away.
2. The admission that unemployment was inevitable was something nobody
cared for / cared to make.
3. Without absolute secrecy this mission would not have succeeded/ been
successful/ been a success/ would have failed.
4. Urgent action is/ measures are necessary/ essential/ steps/action must be taken/
measures must be adopted to solve the problem/ if the problem is to be solved.
5. An turns a blind eye to M inlf s absence
6 ..........do n't take Minnie's comments amiss.........
7 .............get to grips with........................
8. 1 was given no indication by my supervisor that he was about to resign.
9. When I saw the exam questions, my mind just went blank.
10..........come to our attention.................
528
TRƯỞNG THPT CHUYÊN LONG AN - LONG AN
A. MULTIPLE CHOICE (40 pts)
I. 1. c 2. B 3. A 4. D 5. B
II. l. A 2. D 3. B 4. A 5.D
III. 1 .D 2.B 3. B 4. B 5. A
IV. 1. c 2. D 3.D 4. A 5. D
V. Passage 1:
3. A 4. c 5. D
l . c 2.B
Passage 2:
3. D 4. A 5.D
1. c 2. B
VI. P assage 1:
3. A 4. A 5. D
l. B 2.B
Passage 2: 3.B 4. D 5. B
1. D 2 A
B. WRITTEN TEST:
I. Part 1: 3. Although/
2. if 4. how 5. like •
1. on While
6. who 7. cope/ deal 8. few 9. means 10. however
Part 2:
1. would 2. them 3. when 4. without 5. it/ this/ that
6. What 7. over 8. their 9. out 10. such
4. asymmetrical/ asymmetric
8. onlookers
4. beneficial 5. gravitational
9. scepticism 10. dismissive
II. PART 1:
1. antipathy 2. misjudged 3. multitasking
5. uprising 6. counteracting 7. bystanders
9. outplayed 10. disorganizing
PART 2:
1. apparently 2. considerably 3. longevity
6. upredictable 7. Confusingly 8. horticulturists
III. 1. Line 2: active -> actively
2. Line 4: highly spending highly-spending
3. Line 5: by defining -> by definition 4. Line 6: Like —> As
5. Line 8: vulnerable o f —> vulnerable to 6. Line 9: inhabitation -» inhabitants
7. Line 11: character characteristic
8. Line 12: human acts - » human actions
9. Line 15: its - > their 10. Line 18: economics -» economies
IV. 1. It was at you r insistence that we came to this restaurant.
2. Busy as /th ou gh he was, M elissa’s father still played with her.
3. The more qualifications y ou are able to amass, the more success yo u will
have in the academic field.
4. Only by chance was a policeman passing as I was attacked.
529
5 .1 couldn't pin Julie down to a definite answer.
6. Hopefully when he tells US his story, it will shed light on what happened.
7. He finally got into the swing of the new working condition.
8. She played second fiddle in the plot.
9. The offer to go and work in Brazil came (completely) out o f the blue.
10. This liver condition is common in heavy drinkers.
TRÍÍÔNB THPT CKUYẼN LIỈƠNG THỄ VINK - ĐỒNG NAI
A. MULTIPLE-CHOICE QUESTIONS (40 PTS)
H l A /“ » A ■r-'- r— » Í-V t

1. 1. A 2.
4. c 5. B 6. D 7. B 8. A 9. D 10.
B 3. D
II. 11. A
12.B 13. 14. c 15. D 16. A 17. c 18. B 19. D 20.
A
III. 21. A
24. A 25. A 26. B 27. c 28. c 29. D 30.
22. c 23. c
IV. 31. B
34. D 35. B 36. B 37, c 38. D 39. D 40.
32. A 33 c
V .4 1. B 45. B 46. D 47. c 48. A 49. B 50.
42. c 43.
D 44. A
51. c 52. B
54. D 55. B 56. D 57. D 58. c 59 A 60.
53. B
VI. 61. A
62. D 63. 65. c 66. c 67. B 68. c 69. B 70.
B 64. D
71. A 72.
74. A 75. D 76. D 77. D 78. D 79. B 80.
B 73. D
B. WRITTEN QUESTIONS (60 PTS)
I. 81. set 82. as 83. low 84. key 85. provide
86. including 87. signs 88. experienced 89. which 90. on
91. effort 92. tourism 93. put 94. on 95. recover
96. reefs
97. has 98. crashing 99. dumped 100. closure
TT
JLA*
101. 102. innumerate 103. connote
barbarism/barbarity
104. lifespan 105.undertake
06. waterproof/water-resi slant 107. inflammation
08. refrigerants 109. pseudo-intellectually 110. foreground
112. Rhythmically 113. perception(s)
115. Controversial 116. researchers 117. conviction
119. Decisive 120. coherent
122. Line 2: criticism —>critic
111. electrical
114. (un)arguably
118. Underestimated
III. 121. Line 2: targeted —> target
123. Line 5: whom -> who
124. Line 6: ever-increasing —» ever-increasingly
125. Line 9: t o —» in
127. Line : in —>
129. Line : -> Opponents
IV. 131. gave the false impression
132. strike a balance between
133. lack of experience/inexperience counted against her
134. the man that/wno flew of f the handle
135. (our) keeping a straight face was
126. Line 11: neither -> either
128. Line 14: equality —» inequality
130. Line 16: Opponent -» to
530
136. up in a rm s about such
137. (that) s he has set her hear t on going /he r hea rt is set on going
138. have ( got) no option but to get
139. of effic ien cy r eflects bad ly on
140. with the utm ost care
TR ƯƠ NG ĨHPĨ CH UY ÊN LƯƠNG VÃ N CH ÁN H - PH Ú YÊN
A. MULTIPLE CHOICE
I. L B 2. A 3.D 4. B 5. B
II. 1. C 2. A 3. D 4 .C 5.C
III. l . c 2. A 3.B 4 .C 5. D
IV. 1. B 2. c 3 .C 4. B 5.C
V. READING 1
l . c 2. B 3.B 4. C 5. B 6. B
READING 2
4. C 5.B 6. A
l.D 2. B 3.A
VI. CLOZE TEST 1
4. B 5.C 6. C
1. D 2. B 3. A
CLOZE TEST 2
4.A 5.C 6. B
l. A 2 .C 3.D
B. WRITTEN TEST
I. OPEN CLOZE TEST 1
1. heavily/stron giy /highly/largel y/p rọm ine ntly 2. fath er
3. lease . 4. mythology 5. does 6. parts
7. p rey /victim 8. No toriou sly 9. put
OPEN CLOZE TEST 2
1. gue ss 2. acc ura te 3. infancy
6. odd s 7. o ccu rred 8. ch ances
10. equally
4. pose d 5. disposal
9. trem or 10. ignored
II. WORD FORMS 1
1. all-perva siv e/a ll-perva din g
4. ass onance 5. foo lproof
8. Go -ge ttin g 9. about-turn
WORD FORMS 2
2. satu rnin e 3
6. s upe rannuated 7
10. verisim ilitu de
unava ilin g
half-heartedly
2. astrono mic al
6. alignment
10. heavenly
3. q uintess ential 4. heliocentric
7. polarity 8. analogy
2. Line 5: from —» until
4. Line 9: Mostly —> alm ost
1. v ang uard
5. equid istant
9. sph erical
HL 1. Line 3: w h a t- > that
3. Line 8: a gin g -» aged
5. L ine 11: the - > a
6. Line 11: hear tbroken -» hear tbreak ing / heart-re nding
7. Line 13: no t - > no 8. Line 14; few -> little
9. Line 16: bu t -> and 10. Line 17: f latt ing -> flat
531
IV. 1. Far from being e mba rrassed by Tom ’s be ha vio(u) r, she see ms to be
am used by it.
2. The excuse for the d ecla ration o f war was the defend of the ir terr itorial rights.
3. She let it be known tha t she w as lookin g for a flat.
4. Abrams will stand trial murder at the Hig h Co urt next we ek.
5. Failure to obey the regulat ion s may lead to I r esu lt in disqualifica tio n.
6. Obs ervation of th e comp any rules is o f utm ost imp ortan ce.
7. Through no fault of his ow n, the personn el man ag er h anded in his notice.
8. To put it blu ntly, I do not subs cribe to the opinion tha t nature and
nu rtu re a re of equ al significanc e.
9. The business is ha rdly break ing even at the m om ent .
1 0 .1 ca n't put m y fin ger on wh y/t he reason but I d on 't trust him.
11 .1 felt at the ba ck of m y mind that so mething was w rong , but what w as it?
TR ƯỜ NG THPT CH UYÊN LÝ Tự TR ỌN G - CẨN THƠ
A. MULTIPLE CHOICE
I. 1. c 2.D 3. D 4. B 5. A
II. l. B 2.B 3. A 4. D 5. D
III. 1. B 2.C 3. D 4. A 5.D
IV. l . c 2.B 3.D 4. A 5. B
V. PASSAGE 1:
l. D 2. A
3.C 4. D 5. A 6.C 7. D 8. A 9. B 10. D
PASSAGE 2
l. D 2.C 3. B
VI. 4. A 5. D 6. c 7. c 8. A 9. D 10. D
PASSAGE 1:
l. A 2. c
3. D 4. c 5. B 6. A 7. D 8. c 9. D 10. B
PASSAGE 2:
l . c 2.B 3. B 4. D 5. D 6. B 7. A 8. c 9. A 10. D
B. WR ITTEN TEST
I. PASSAGE 1:
1. regarded 2. habit 3. mind 4. prove 5. teaching
6. run 7. When 8. latter 9. time 10. opportunity
PASSAGE 2:
1. habit 2. harsh 3. equipped 4. nests 5. into
6. endure 7. with 8. losing 9. navigate 10. natural
II. PART 1:
2. 3. multi-
1. incorrigible 4. Unobjectionable
interdepartmental tasking
5. misty-eyed 6. dumbfounded 7. assimilated 8. Requisitioned
9. self-
evaluation 10.
gatecrashed
532
PART 2:
1. (re)assured 2. laboriously 3. affirmative 4. exceptionally 5. solidarity
6. checkout 7. alternative 8. insecurity 9. tendency 10. confronted
III. 1. Line 1: composer —> composers 2. Line 3: publishing —» published
3. Line 4: and —> although 4. Line 6: Like —> As
5. Line 7: controlled —> lost 6. Line 8: when —> where
7. Line 9: uniquely —> only . 8. Line 10: played —» composed
9. Line 11: never —* ever 10. Line 14: that —> who
IV. 1. My new job is far and away the most satisfying one I've ever had.
2. But /o r Tom's prompt action ill putting the fire out, there might have been
more damage to the house.
3. So you ng is a child o f his age to he deceitful.
4. I've got missing the rush hour down to a fine art over the last few weeks.
5. 1 do n't think w should write o ff her ideas as unworkable at this stage.
6. It's crucial for USto keep a tight rein on the juvenile inmates.
7. Yo u've done nothing hut moon around all day.
8. We don't hold out much hope fo r the survival o f the tourists in that accident.
9. Your Story does not tie in with the facts.
10. His excuse fo r such had behavior cuts no ice with her.
TRƯỜNG THPT CHUYÊN NGUYỄN CHÍ THANH - ĐẮ K NÔ NG
A. MULTIPLE CHOICE
I. 1. D 2. B 3. c 4. c 5. A 6. B
II. ll.D 12. D 13. A 14. B 15. B 16. c
III. 21. B 22. A 23. B 24. D 25. B 26. B
IV. 31. C 32 . B33. D 34. B 35. B 36. B
V. 41 . c 42. B43. D 44. A. 45. D 46. A
51. B 52. c 53 A 54. D 55. D 56 B
VI. 61. c 62. B 63. A 64. D 65. B 66. A
71. D 72. B 73. c 74. A 75. A 76. c
B. WRITTEN TEST
I. 1. but 2. exist 3. part 4. As
6. various 7. another 8. applied 9. reason/ ca
11. out 12. heart 13. skill 14. overcome
16. turn 17. warming 18. date 19. devoted /dedicate d 20. d
II. Supply each gap with correct form o f the word given in brackets.
elivered
1. signals/ signalled
5. futuristic
9. imprinted
2. notoriety
6. accentuate
10. outspoken
3. undoubtedly
7. undertake
4. troublesom e
8. expressionless
533
Supply each gap with correct form o f the word given in the box. (ỈOpts)
1. comfortably 2. impressionable 3. caressingly 4. ceremonial 5. newly
6. acquaintance 7. courteously 8. instinctive 9. relationships 10. disdainfully
HI.

Lille Mistake Correction Lin e


2 divide share 8
3 believed have believed 8
5 University the University 10
6 just as just like 11
7 in which among which 14
IV. 1. After a long hard journ ey, my spirits... ro se/w er e lifted when I caught
sight of my home.
2. He didn 't want to expose hims elf to the possibility of losing all his money
3. The company was/ were deluged with calls response to the advert.
4. Children get a kick out of/froni unwrapping their Christmas presents.
5. He sw ept the m atter u nd er the carpetbec ause he didn't want to lose his high
position.
6. You are under no obligation to come and see the new house if you don't wish.
7. He made threats of violence against the officers
8. The headmaster holds the physics teacher in high esteem
9. On his own admission, he is afraid of spiders
10. When it comes to technical knowledge, Suzanne is far superior to me
TRưỜNG THPĨ CHUYÊN NGUYỄN ĐÌNH CHIỂU - ĐỔNG THÁP
A. MULTIPLE CHOICE (40 PTS)
I. l. D 2. B 3. B . 4. c. 5. D 6. D 7. c
II. l l . c 12. A 13. A 14. B 15.C 16. D 17. B
III. 21. A 22. C23. D 24. A 25. c 26. B 27. A
IV7. 31. A 32. B 33. c 34. D 35. A 36. c 37. D
V. 41. D 42. A 43. A 44 D 45. c 46. B 47. c
51. c 52. A 53. 3 54. B 55. D 56. c 57. A
VI. 61. B 62.A 63 A 64 D 65. c 66. c 67. A
71. c 72. A 73. A 74 A 75. D 76. c 77. D
8. WRITTEN TEST (70 PTS)
I. 1. regard 3. of 5. by 7. another
2. own 4. great 6. much 8. than
1 1. from 13. other 15. however 17. same
12. see 14. are 16. those 18. what
lĩ . PART 1:
1. OMNIPRESENT
4. SNOW-CAPPED
2. AUTOIMMUNE
5. SMOKESCREEN
3. SLAP-UP
6. HUMANOID
534
7. LIBRETTISTS 8. UNADULTERATED
9. EXPROPRIATING 10. BIENNIAL
PART 2:
1. barded-wire
6. disillusioned
2. anew
7. second-class
3. mount 4. unbefitting
8. heatedly 9. homeland
5. ex-wife
10. resurefaced
2. line 3: for -» against ỈỈĨ. 1. line 1: inheriting -> inherited
4. line 5: off —> up 3. line 4: zoologic —> zoological
6. line 9: for -> to
8. line 16: un-existent —> non-existent
10. line 24: diverse -> diversity
5. line 6: captive —> captivity
7. line 10: minimally —> minimal
9. line 23: remain —> remains
IV. 1. The operation left David fe el (even) weaker than he had expected.
2. There is no (other) statesman that I admire more than/as much as the
president.
3. Nowhere else am Ĩ as practical as you are.
4. Could you keep an eye on my hag when Pm away?
5. So intense was the hatredfor Frank's new policies that the staff went on strike.
6. There is resistance among the fam ily members to the fac t/idea that their
father's fortun e will go to the charity.
7. His analysis of the situation was righted over my head.
8. Pull you r finger out now i f you want to become a man fl/substance.
9. Only through the agency o f John could she get the job.
10. Peter never lifts a figure to help his team when ItZstime for action.
TWỜNG ĨH P Ĩ CHUYÊN THOẠI NGỌC HẦU - AN GIANG
A. MULTIPLE CHOICE QUESTIONS
L l . c 2. D 3. B
4. D 5. c 6. D
ỈI. 1.D 2. D 3. A
4. c 5. A 6. B
III. l. B 2. A 3. A
4. B 5. D 6. B
IV. 1. B2. c 3. A
4. D 5. c 6. B
V. READING Ỉ:
1.B 2. A 3. D 4. B 5. A 6. c
READING 2:
l. D 2. D 3. A
4. B 5. A 6. B
VI. CLOZE-TEST 1.
6.
LB 2. A 3.B 4. D 5. A
B
CLOZE-TEST 2.
EC 2. D 3 .A
4. B 5. D 6. B
7. A 8. D 9. B
7 A 8. D 9. D
7. c 8. A 9. c
7. A 8. A 9. B
7. A 8. D 9. A
7. c 8. D 9. D
7. B 8. c 9. D
7. A 8. B 9. A
10. B
10. c
10. A
10. D
10. A
10. c
10. A
10. c
B. WRITTEN TEST:
I.CLOZE TEST 1
1. apart 2. them 3. behind 4. different
6. as 7. kind 8. over / by 9. at
535
CLOZE TEST 2
I. use
6. to/with
II. P ARTI:
1. unstintingly
5. overexposed
8. Casualisation
PART 2:
1. introduction
5. effectiv ely
9. assistance
2. which 3. light - 4. span 5. up
7. much 8. after/following 9. what 10. from
2. half-hearted 3. action-packed4. absenteeism
6. poverty-stricken 7. non-biodegradable
9. mouth-watering 0.justifiably
4. displac e / replace
8. precautions
2. successful 3. immunity
6. depopulation 7. formulation
10. invasion
III. 1. Line 2: from their wake —» in their wake
2. Line 3: estimated —> is estimated
3. Line 5: Wind —> The wind
4. Line 7: increased violen ce —> increased in violence
5. Line 11: as —> like
6. Line 16: came to crash —> came crashing
7. Line 17: has been risen —> has been raised
8. Line 19: that the hurrican e —> but that the hurricane
9. Line 22: Therefore —> M eanwhile
10. Line 23: bracing themselv es to —> bracing themselves for
IV. 1. Not on any account should the house be left unlocked.
2. The first priority for the council is the restoration of communications and
essential services.
3. The gov ernment’s immigration policy has been under review for some time now.
4. He let it be known that he disapproved of the behavior of some, of his supporters.
5. They hav e been testing a new flu vaccine since the beginning of the year.
6. T hey may have made their g etaway through the backdoor.
7. Y ou’ve done nothing but moon about all day.
8. Jim was poised to go on the business trip at a moment's notice.
9. We received shoddy service from the waiters in the n ew restaurant.
10. Sean wasn’t given any credit for doing all the illustrations for the book.
TRƯỜNG THPT CHUYÊN TRẦN OẠI NGHĨA - TP. Hồ CHI MIN H
I. 1. A 2. A 3.C 4. D 5. B 6. B
II. 11. A 12. c 13. c 14. D 15. A 16. B
III. 21. B 22. C2 3.C 24. A 25. A 26. A
IV. 31. A 32. B33. A 34. c 35. D 36 c
V. 41. C42 . c 43. B 44. c 45. A 46. A
51. A 52. c 53. D 54. A 55. B 56. B
IV. 61.B 62. B63. B 64. A 65. A 66. c
71. c 72. B 73. D 74. A 75. c 76. B
536
3. all
8. being
4 .shrouded
9. among
3. holds
8. of
2. given
7. passing
2. malcontents
6. incommensurable
9. anti-inflammatory
3. extempore
7. aftermath
10. heterogeneroussness
5. said
10. open
4. ready 5. seethes
9. little 10. o f
4. utilitarian
B. W R IT T E N TE S T
I. PASSAGE A
1. until 2. telling
6. sign 7. when
PASSAGE B
I. measure
6. rough
II. PART 1:
I. unimpeachable
5. taxidermically
.8. memorabilia
PART 2:
13. undeniably 14.
II . ascent 12. disproved
multitudinous 15. copyright
16. dispossessed 17.
fulfillment 18. invariably 19. longings 20. oddity
III. 1. Line 1: dead - deadliest
2. Line 2 : thousand -
thousands
3. Line 2: life time - lifetime
5. Line 6: not - any
7. Line 9: much - many
4. Line 3: few - fewer
6. Line 7: number - deal
8. Line 11: for - from
9. Line 14: alarmed - alarming
10. Line 17: avoiding - to avoid
IV. 1. Rarely do I sleep in the after noo n.
2. It was the coac h’s sm art and flexib le tactics that led Vietnam 's national
football team to the victory in the AFC Championship.
3 .1 know he has tried hard; be that as it may, I wish his wo rk were good enough.
4. Junk food can nowhere near as nutritious as a cooked meal, acc ord ing to
popular belief.
5. Had the quirks of the ant iqu es bus iness not been beyond my ken, I could
hav e given you som e adv ice.
6. Having mulled it over for three weeks, his par ents gav e him the go-ah ead
to buy an iPh one X.
7. Notwithstanding their wo rking -class bac kgr oun d, the peo ple here are the
salt of the earth.
8. On account of having / her spo one rism, Tom is foreve r taking the mickey
out o f Daisy/h er.
9. It was about time (tha t) I we nt over th ere and held out I offer ed / e xtend ed
the olive b ran ch to her.
10. Remem ber to check out our baker’s dozen lan guage lea rning tips!
TRƯỜNG THPT MẠC ĐĨNH C HI - T P. HỒ CHI MINH
A. M U LTIP LE CHOIC E QU ESTIO NS
10.
D
L l . c 2. c 3.D 4. B 5. B 6. D 7. c 8. A 9. A
20.
c
II. 11. B 12. A 14. D 15. A 16. B 17. D 18. B
13. D 19. A
537
III. 21. B 22.B 23. A 24.D 25. D 26. A
IV. 31. D 32. A 33. D 34. B 35. A 36.’c
V. 41.D 42. D 43. A 44. B 45. A 46. B
51. A 52. B 53. A 54. c 55. A 56. B
VI. 61 .D 62. c 63. A 64.D 65. c 66. A
71.D 72. c 73. A 74. A 75. D 76. B
B, WRITTEN TEST (70 PTS)
Í. 1. essential 2. conditions
5. themselves 6. uncomfortable
9. suffer
13. describes
10. stressful
14. meaningless
17. forgiven/pardoned
19. states/suggests/says/claims
II.
1. ineradicable 2. misfit
5. unlooked-for 6. posterity
9. tension-relieving 10. business-focused
13. original 14. marvellous
17. fishing 18. irresistible
III. Line 1: more —> most
Line 2: specially —> especially
Line 2: do —> have
Line 4: breakthrough —> outbreak
Line 5: only —> alone
3. develop
7. challenge
11. since
15. predetermined
18. extremes
20. issue
4. response
8. unfortunately
12. determined/governed
16. control
3. eventuality
7. preferment
11. mixture
15. unbelievably
4. actuated
8. petrol-guzzling
12. undeniably
16. creation
19. resourcefulness 20. unappetising
Line 9: to originate -» to have originated
Line 10: spreading - » spread
Line 10: slow —> slowly
Line 12: effecting —> affecting
Line 13: had —» have
IV. 1. Exulting in/ove r his op po ne nt s’ m isfo rtun e is s om ethi ng t h at no re al
sp or ts m an w an ts to be su sp ec ted of. 2. There is less ev iden ce to su pport
stor ie s of the w or k- sh y th an (the re is) to su pport a ny o f the myths of th e
W el fa re S tat e.
3. The Council was pe rs ua de d to re co ns id er the ro ad pl an aft er st ro ng
pr es su re from L ab our co un cil lors..
4. Not only was Ki p Ke ino th e firs t African in te rn at io nal mile w in ner , he
wa s a na tion al s ym bol as well.
5. Sam was on ten terhooks, waiting to see if he ha d got a place in the cr icke t te am.
6. The heir to the fam ily f or tu ne was e ve ntua lly exposed as an im po stor .
7. He dism iss ed my po int as irre le van t to the proc ee ding s.
8. Such was t ha t sec retary’s ra nk incompetenc e th at we lost hea ps of invoices.
9. There are two million un em ploy ed in th e co un try, m an y of who m will
pr ob ab ly ne ve r w or k ag ain.
10. That her colleagues wa s (so) op enly ho sti le to w ar ds her pr op os al wa s
difficul t to un de rs ta nd.
538
_ _ _ _ _ _ _ _ _ TRựỜNG ĨHP T PHflH CBflli TRINH ■ BÀ MANS_ _ _ _ _ _ _ _ _ _
A. MULTIPLE CHOICE QUESTIONS (40pts)
I. 1. B 2. D 3. A 4 .C 5.D
IL L A 2. D 3. A 4. B 5. A
HI. 1. D 2. D 3 .B 4. A 5. D
I V .l .B 2. A 3 .C 4. B 5. A
V. PART 1.
3 .C 4. c 5.C
L B 2. c
PART 2.
3. A 4. c 5.C
L c 2. D
VI. PART 1.
3. B 4. B 5. D
L B 2 .C
PART 2.
3. D 4. B 5.C
l . c 2. A
B. WRITTEN TASKS (70pts)
I. OP EN CL OZ E TE ST (20pts)
PART 1.
1 .at / on 2. Having 3. in 4. Wh ether
6. sho uld 7. th em selves 8. agai nst / under 9. w hat
PART 2.
1. bus tle 2. furiou s 3. saying 4. so / as 5. c om mo n / public
6. adverse ly I negatively I severely 7. c ulp rit / o ffe nder
8. exten t/d egree 9. pro mpted 10. utmo st I para mo unt I o ver riding
IL PART 1.
1. en visio ne d / envisa ge d
4. prep on de ranc e
8. mischie f-m akers
PART 2:
2. incorrigib le
5. abe rra tion
9. trendsetting
3. subserv ien t
6. o utc ry 7. tell ings -off
10. un dr ea m t-of / un dre amed.-of
I. onloo kers
5. over- inflated
9. fol low er- count(s)
ĨỈ Ĩ.
2. Imprints 3.-seỉf-(re)pỉcsen taỉion 4. mo ck- ups
6. inde lible 7. ( self-)disclosures 8. min utiae
10. on -top ic
Line Error Correction Line
Í. 2 estimation esti mates
2. 2 millio ns of million
3. 4 the man man
4. 4 has has been
5. 7 dated date
IV. PART 1.
1. Far from bei ng emb arrass ed by Mim i's behavior, he seems to be amuse d by it.
2. No t for anoth er 5 years did the whole truth abo ut the mu rde r c ome out
3. Were it n ot for h er c om pet ence, she wou ldn't get prom otio n.
53 9
4. Of grave concern arc confirmed cases of cholera that originated in the
makeshift shelters.
PART 2
1. What conclusions have you drawn from this experience?
2. Too much small talk means that I is the reason why she gets bored quickly at
parties.
3. I'm under the impression that my parents take everything I do in their stride.
4. f ie gave every indication of going to any lengths to obtain the contract.
5. Nowadays 1 don’t set I put / lay (nearly) as much store by casual friendships as
1 used to.
6. I 'm afraid th at there is no chance I possibility whatsoever o f negotiations for a
settlement.
TRƯỞNG THPT CHUYÊN TRẦN HUNG ĐẠU - BÌNH THUẬN
A. MULTIPLE CHOICE
I. 1.D 2. A
II. 1. c 2 A
III. 1.D 2.A
IV. L C 2 .B
3. D
3. B
3.C
3. D
4. A
4. A
4. B
4. c
V. READING PASSAGE 1
l.c 2. A 3. B 4. c
READING PASSAGE 2
l. B 2. D 3. A 4. c
VII. GUIDED CLOZE 1
4.
l.c 2. A 3.D
D
GUIDED CLOZE 2
4.
l.c 2. A 3.D
D
5.C
5.C
5. A
5.C
5. A
5.C
5.C
5. c
6. D 7.c
6. D 7. B
6. D 7.A
6. B 7.c
6. D 7. c
6. B 7. D
6. A 7. B
6. B 7. B
8. B 9. B 10. A
8. D 9. B 10. D
8. c 9. D 10. B
8. B 9. D 10. B
8. D 9. A 10. c
8. A 9. D 10. c
8. A 9. D 10. c
8. A 9. A 10. D
B. WRITTEN TEST
I. OPEN CLOZE 1
1. escalate 2. burden 3. Below
5. plan 6. average 7. conservative
9. attached 10. drastically
OPEN CLOZE 2
I. seeding 2. injecting 3. stratosphere 4. particles
6. meddling
7. whisk 8. thereby 9. feasible
II. PART 1:
4. considerable
8. purchasing
1. paternalism
5. expressionless
9. abstentions
PART 2:
2. archenemy 3. undercurrent
6. asymptomatic 7. Precancerous
10. reciprocity
5. bouncing
10. upside
4. inaccessibility
8. unjustifiable
1. controversy
5. relinquish
9. reliance
2. unpredictable
6. Upholders
10. accelerating
3. inhospitable
7. allege
4. excogitated
8. accredited
540
ni.
1. Line 2: spiral (demand) spiraling
2. Line 3: over (the globe) -> across
3. Line 4: their (wake) —> its
4. Line 6: That (was once) —> What
5. Line 8: into (the limits) -> beyond
6. Line 9: (more) especially specifically
7. Line 10 : have (convinced) —> arc
8. Line 17: (to) solve -> solving
9. Line 20: dead (perils) -» deadly
10. Line 20: bided (their time) —> biding
IV. 1. Coming seco nd wa sfo i) no consolidation (to her) because winning was all
that m attered to her.
2. His analysis of the situation went right over my head.
3. The committee members pledged their loyalty to the chairman.
4. Retirement will free me up to spend more time with my grandchildren.
5. In the area. Thailand is head and shoulders above all other countries in football.
6. Discussions on the issue have been in progress for over two weeks.
7. The sheer expense/ cost was what put me off the idea.
8. The orphanage left no stone unturned in their attempt to find the b oy's parents.
9. There was no question of drugs being involved, whatever the methods used to
obtain the results.
10. He made threats of violence against the officers.
TRƯỞNG TH PĨ CHUYÊN PHAN NGỌC HIỂN - CÀ MAU
A. MULTIPLE CHOICE (40 pts)
I. LB 2.A 3. D 4.C 5. A
II. 11. B 12. D 13. D 14. B 15.C
III. 21 . B2 2. B23 .A 24. B 25. c
IV. 3L . B3 2. B 33. A 34. c 35. B
V .4 L c 42. B 43. A 44. B 45.C
51. A 52. D 53. A 54. A 55. B
VI. 61 . c 62. B 63. c 64. D 65. A
7L D 72. B 73. D 74. A 75. c
B. WRITTEN TEST
I OPEN CLOZE TEXT 1 (10 pts)
1. nearest/local 2. destination 3. information 4. bookings
5. showing 6. fully 7. as/when 8. certain
9. confirms 10. print
OPEN CLOZE
TEXT 2: (10 pts)
2. reducing 13. reflect
11. inhabit 1 15. inside
14. constant
16. what 1 7. overheated 18. low 19. buildup
II. 1. z
1. disturbance 2. impressionable 3. repulsive 4. foursome
5. submissive 6. reaccustom 7. unenforceable 8. ecotourism
9. supposedly 10. misconceptions
541
2
2. problematic
6. disillusioned
9. faceless
I. authorities
5. imitations
8. dishearted/downhearted
III
1. Line 1: formally -* formerly
3. Line 4: stores-* stored
5. Line 6: electric -* electrically
7. Line 9: still -* yet
9. Line 12: pattern -> patterns
3. imperceptible 4. enduring
7. identifiable
10. survivor
2. Line 2: approa ches -* approaching
4. Line 5: on-* by
6. Line 7: supposed ~* supposedly
8. Line 11: limited -* unlimitted
10. Line 14: strength -* strengthening
IV. 1. it not been for the goalkeeper, we would have lost the match.
2. am next in the line for
3. is head an d shoulders above all other countries in football.
4. are under a cloud because o f their b ad behaviour\
5. to our regret we must inform ...
6. than disturbing the meeting, I left without saying goodbye.
7. experienced a driver you are, driving at that speed is dangerous.
8. took any notice o f my protests.
9. We ’vé come to an agreement to share the cost.
10. What conclusions have you drawn from this experience?
ĨRIÍỜNG ĨHPT r ô ffl A ĩự - ĐẮK LẮK
A. MULTIPLE
CHOICE. (40 PTS)
I. L A 2. A 3. A 4. B
6. B 7. D 8. c 9.C 10.
5. D
II. l . c 2. c 3. A 4. c 6. Đ 7. c 8 c 9. A 10,
5B
III. l . c 2. A 3.C 4.
6. D 7. B 8. D 9. D 10.
A5c
IV. l. D 2. c 3.B 4. A
6. A 7. A 8 D 9. B 10.
5A
V. READING
PASSAGE 1 (5
PTS)
l . c 2. c 3.D 4. B 5.
6. B 7. A 8. D 9. B 10
C
READING
PASSAGE 2 (5
PTS)
Í. D 2. A 3. c 4. A 5.
6. D 7. c 8. B 9. D 10,
A
VII. GUIDED
CLOZE 1 (5 PTS)
LA 2. B 3. A 4. B 5.
6. D 7. c 8. A 9. B 10
B
GUIDED CLOZE 2
(5 PTS)
LA 2. c 3.D 4. A 5.
6. D 7.C 8. B 9. B 10
B
B. WRITTEN
TEST (70 PTS)
I. OPEN CLOZE 1.
(10 PTS)
4. leveled 5.
1. under 2. for 3. be
made
6. reduced 7. with 8.
10. ever
taken/ borrowed 9. in
OPEN CLOZE 2.
(10 PTS)
1, came 2. meant 3.
served 5. business
very 4.
6. communication 7.
amounts 10. point
extent 8. quite 9.
BccD D D c B
542
II. PART 1:
1. Reunification
4, inaccessible
8. continually
PA RT 2:
1. preference
5. Applicants
9, continuously
III. 1. Line 2: serious —> seriously 2. Line 6: on -> over
3. Line 8: becoming -» becomes 4. Line 9: involving —> involved
5. Line 10: later —> latter
7. Line 15: m ost —> least
9. Line 19: reversible —> irreversible
2. parameterization
5. depressurizes 6. distractedly
9. dissociate 10. uncooperative
3. unmanageable
7 .disqualified
2. loyalty
6. necessarily
10. dissatisfaction
3. likelihood
7. comedian
4. beneficial
8. unfortunately
6. Line 14: rich —> richly
8. Line 16: anywhere —> nowhere
10. Line 22: establish -> re-establish
IV. 1. It wasn't until som etime later that I rea lize d the full im plica ton s of
wha t had happene d.
2. The judge said that had it not been for her age he wo uld have sent the
wo ma n to jail.
3. No t since a thousand year ago has there been an environment catastrophe of
such proport ion s.
4. It came as no sur prised to us that he wa s/ had been succe ssful.
5. He has a bee in his bonnet about the dishones ty of lawyer s.
6.1 must hav e offended her b eca use sh e is g ivin g me a cold sho uld er.
7. You need to see the col our of his money before you sell him the car.
8. He made thr eats of violence agains t the officers.
9. The stu dents deserved severe punishment for their rebellious behaviours.
10. T he re ’s no one available at such short notice to take her class.

You might also like